Sei sulla pagina 1di 327

FINALS CONSTITUTION I ACJUCO 1

G.R. No. 166715 August 14, 2008 rewards and incentives, the law "transform[s] the officials and
employees of the BIR and the BOC into mercenaries and
ABAKADA GURO PARTY LIST (formerly bounty hunters" as they will do their best only in consideration
of such rewards. Thus, the system of rewards and incentives
AASJS)1 OFFICERS/MEMBERS SAMSON S. ALCANTARA,
ED VINCENT S. ALBANO, ROMEO R. ROBISO, RENE B. invites corruption and undermines the constitutionally
GOROSPE and EDWIN R. SANDOVAL, petitioners, mandated duty of these officials and employees to serve the
vs. people with utmost responsibility, integrity, loyalty and
HON. CESAR V. PURISIMA, in his capacity as Secretary of efficiency.
Finance, HON. GUILLERMO L. PARAYNO, JR., in his
capacity as Commissioner of the Bureau of Internal Petitioners also claim that limiting the scope of the system of
Revenue, and HON. ALBERTO D. LINA, in his Capacity as rewards and incentives only to officials and employees of the
Commissioner of Bureau of Customs, respondents. BIR and the BOC violates the constitutional guarantee of equal
protection. There is no valid basis for classification or
DECISION distinction as to why such a system should not apply to officials
and employees of all other government agencies.

CORONA, J.:
In addition, petitioners assert that the law unduly delegates the
power to fix revenue targets to the President as it lacks a
This petition for prohibition1 seeks to prevent respondents from sufficient standard on that matter. While Section 7(b) and (c) of
implementing and enforcing Republic Act (RA) 93352 (Attrition RA 9335 provides that BIR and BOC officials may be
Act of 2005). dismissed from the service if their revenue collections fall short
of the target by at least 7.5%, the law does not, however, fix
RA 9335 was enacted to optimize the revenue-generation the revenue targets to be achieved. Instead, the fixing of
capability and collection of the Bureau of Internal Revenue revenue targets has been delegated to the President without
(BIR) and the Bureau of Customs (BOC). The law intends to sufficient standards. It will therefore be easy for the President
encourage BIR and BOC officials and employees to exceed to fix an unrealistic and unattainable target in order to dismiss
their revenue targets by providing a system of rewards and BIR or BOC personnel.
sanctions through the creation of a Rewards and Incentives
Fund (Fund) and a Revenue Performance Evaluation Board Finally, petitioners assail the creation of a congressional
(Board).3 It covers all officials and employees of the BIR and oversight committee on the ground that it violates the doctrine
the BOC with at least six months of service, regardless of of separation of powers. While the legislative function is
employment status.4 deemed accomplished and completed upon the enactment
and approval of the law, the creation of the congressional
The Fund is sourced from the collection of the BIR and the oversight committee permits legislative participation in the
BOC in excess of their revenue targets for the year, as implementation and enforcement of the law.
determined by the Development Budget and Coordinating
Committee (DBCC). Any incentive or reward is taken from the In their comment, respondents, through the Office of the
fund and allocated to the BIR and the BOC in proportion to their Solicitor General, question the petition for being premature as
contribution in the excess collection of the targeted amount of there is no actual case or controversy yet. Petitioners have not
tax revenue.5 asserted any right or claim that will necessitate the exercise of
this Court’s jurisdiction. Nevertheless, respondents
The Boards in the BIR and the BOC are composed of the acknowledge that public policy requires the resolution of the
Secretary of the Department of Finance (DOF) or his/her constitutional issues involved in this case. They assert that the
Undersecretary, the Secretary of the Department of Budget allegation that the reward system will breed mercenaries is
and Management (DBM) or his/her Undersecretary, the mere speculation and does not suffice to invalidate the law.
Director General of the National Economic Development Seen in conjunction with the declared objective of RA 9335,
Authority (NEDA) or his/her Deputy Director General, the the law validly classifies the BIR and the BOC because the
Commissioners of the BIR and the BOC or their Deputy functions they perform are distinct from those of the other
Commissioners, two representatives from the rank-and-file government agencies and instrumentalities. Moreover, the law
employees and a representative from the officials nominated provides a sufficient standard that will guide the executive in
by their recognized organization.6 the implementation of its provisions. Lastly, the creation of the
congressional oversight committee under the law enhances,
rather than violates, separation of powers. It ensures the
Each Board has the duty to (1) prescribe the rules and fulfillment of the legislative policy and serves as a check to any
guidelines for the allocation, distribution and release of the over-accumulation of power on the part of the executive and
Fund; (2) set criteria and procedures for removing from the the implementing agencies.
service officials and employees whose revenue collection falls
short of the target; (3) terminate personnel in accordance with
the criteria adopted by the Board; (4) prescribe a system for After a careful consideration of the conflicting contentions of
performance evaluation; (5) perform other functions, including the parties, the Court finds that petitioners have failed to
the issuance of rules and regulations and (6) submit an annual overcome the presumption of constitutionality in favor of RA
report to Congress.7 9335, except as shall hereafter be discussed.

The DOF, DBM, NEDA, BIR, BOC and the Civil Service Actual Case And Ripeness
Commission (CSC) were tasked to promulgate and issue the
implementing rules and regulations of RA 9335,8 to be An actual case or controversy involves a conflict of legal rights,
approved by a Joint Congressional Oversight Committee an assertion of opposite legal claims susceptible of judicial
created for such purpose.9 adjudication.10 A closely related requirement is ripeness, that
is, the question must be ripe for adjudication. And a
Petitioners, invoking their right as taxpayers filed this petition constitutional question is ripe for adjudication when the
challenging the constitutionality of RA 9335, a tax reform governmental act being challenged has a direct adverse effect
legislation. They contend that, by establishing a system of on the individual challenging it. 11Thus, to be ripe for judicial
FINALS CONSTITUTION I ACJUCO 2

adjudication, the petitioner must show a personal stake in the Public service is its own reward. Nevertheless, public officers
outcome of the case or an injury to himself that can be may by law be rewarded for exemplary and exceptional
redressed by a favorable decision of the Court. 12 performance. A system of incentives for exceeding the set
expectations of a public office is not anathema to the concept
In this case, aside from the general claim that the dispute has of public accountability. In fact, it recognizes and reinforces
ripened into a judicial controversy by the mere enactment of dedication to duty, industry, efficiency and loyalty to public
the law even without any further overt act, 13 petitioners fail service of deserving government personnel.
either to assert any specific and concrete legal claim or to
demonstrate any direct adverse effect of the law on them. They In United States v. Matthews,17 the U.S. Supreme Court
are unable to show a personal stake in the outcome of this validated a law which awards to officers of the customs as well
case or an injury to themselves. On this account, their petition as other parties an amount not exceeding one-half of the net
is procedurally infirm. proceeds of forfeitures in violation of the laws against
smuggling. Citing Dorsheimer v. United States,18 the U.S.
This notwithstanding, public interest requires the resolution of Supreme Court said:
the constitutional issues raised by petitioners. The grave
nature of their allegations tends to cast a cloud on the The offer of a portion of such penalties to the
presumption of constitutionality in favor of the law. And where collectors is to stimulate and reward their zeal and
an action of the legislative branch is alleged to have infringed industry in detecting fraudulent attempts to evade
the Constitution, it becomes not only the right but in fact the payment of duties and taxes.
duty of the judiciary to settle the dispute.14
In the same vein, employees of the BIR and the BOC may by
Accountability of law be entitled to a reward when, as a consequence of their
Public Officers zeal in the enforcement of tax and customs laws, they exceed
their revenue targets. In addition, RA 9335 establishes
safeguards to ensure that the reward will not be claimed if it
Section 1, Article 11 of the Constitution states:
will be either the fruit of "bounty hunting or mercenary activity"
or the product of the irregular performance of official duties.
Sec. 1. Public office is a public trust. Public officers One of these precautionary measures is embodied in Section
and employees must at all times be accountable to 8 of the law:
the people, serve them with utmost responsibility,
integrity, loyalty, and efficiency, act with patriotism,
SEC. 8. Liability of Officials, Examiners and
and justice, and lead modest lives.
Employees of the BIR and the BOC. – The officials,
examiners, and employees of the [BIR] and the
Public office is a public trust. It must be discharged by its holder [BOC] who violate this Act or who are guilty of
not for his own personal gain but for the benefit of the public negligence, abuses or acts of malfeasance or
for whom he holds it in trust. By demanding accountability and misfeasance or fail to exercise extraordinary
service with responsibility, integrity, loyalty, efficiency, diligence in the performance of their duties shall be
patriotism and justice, all government officials and employees held liable for any loss or injury suffered by any
have the duty to be responsive to the needs of the people they business establishment or taxpayer as a result of
are called upon to serve. such violation, negligence, abuse, malfeasance,
misfeasance or failure to exercise extraordinary
Public officers enjoy the presumption of regularity in the diligence.
performance of their duties. This presumption necessarily
obtains in favor of BIR and BOC officials and employees. RA Equal Protection
9335 operates on the basis thereof and reinforces it by
providing a system of rewards and sanctions for the purpose
of encouraging the officials and employees of the BIR and the Equality guaranteed under the equal protection clause is
BOC to exceed their revenue targets and optimize their equality under the same conditions and among persons
revenue-generation capability and collection.15 similarly situated; it is equality among equals, not similarity of
treatment of persons who are classified based on substantial
differences in relation to the object to be accomplished. 19When
The presumption is disputable but proof to the contrary is things or persons are different in fact or circumstance, they
required to rebut it. It cannot be overturned by mere conjecture may be treated in law differently. In Victoriano v. Elizalde Rope
or denied in advance (as petitioners would have the Court do) Workers’ Union,20 this Court declared:
specially in this case where it is an underlying principle to
advance a declared public policy.
The guaranty of equal protection of the laws is not a
guaranty of equality in the application of the laws
Petitioners’ claim that the implementation of RA 9335 will turn upon all citizens of the [S]tate. It is not, therefore, a
BIR and BOC officials and employees into "bounty hunters and requirement, in order to avoid the constitutional
mercenaries" is not only without any factual and legal basis; it prohibition against inequality, that every man, woman
is also purely speculative. and child should be affected alike by a statute.
Equality of operation of statutes does not mean
A law enacted by Congress enjoys the strong presumption of indiscriminate operation on persons merely as such,
constitutionality. To justify its nullification, there must be a clear but on persons according to the circumstances
and unequivocal breach of the Constitution, not a doubtful and surrounding them. It guarantees equality, not identity
equivocal one.16 To invalidate RA 9335 based on petitioners’ of rights. The Constitution does not require that
baseless supposition is an affront to the wisdom not only of the things which are different in fact be treated in law
legislature that passed it but also of the executive which as though they were the same. The equal
approved it. protection clause does not forbid discrimination
as to things that are different. It does not prohibit
legislation which is limited either in the object to
FINALS CONSTITUTION I ACJUCO 3

which it is directed or by the territory within which it (3) Prevent and prosecute tax evasions and all other
is to operate. illegal economic activities;

The equal protection of the laws clause of the (4) Exercise supervision and control over its
Constitution allows classification. Classification in constituent and subordinate units; and
law, as in the other departments of knowledge or
practice, is the grouping of things in speculation or (5) Perform such other functions as may be provided
practice because they agree with one another in
by law.24
certain particulars. A law is not invalid because of
simple inequality. The very idea of classification is
that of inequality, so that it goes without saying that xxx xxx xxx (emphasis supplied)
the mere fact of inequality in no manner determines
the matter of constitutionality. All that is required of On the other hand, the BOC has the following functions:
a valid classification is that it be reasonable,
which means that the classification should be
based on substantial distinctions which make for Sec. 23. The Bureau of Customs. – The Bureau of
real differences, that it must be germane to the Customs which shall be headed and subject to the
purpose of the law; that it must not be limited to management and control of the Commissioner of
existing conditions only; and that it must apply Customs, who shall be appointed by the President
equally to each member of the class. This Court upon the recommendation of the Secretary[of the
has held that the standard is satisfied if the DOF] and hereinafter referred to as Commissioner,
classification or distinction is based on a shall have the following functions:
reasonable foundation or rational basis and is
not palpably arbitrary. (1) Collect custom duties, taxes and the
corresponding fees, charges and penalties;
In the exercise of its power to make classifications for
the purpose of enacting laws over matters within its (2) Account for all customs revenues collected;
jurisdiction, the state is recognized as enjoying a
wide range of discretion. It is not necessary that the
classification be based on scientific or marked (3) Exercise police authority for the enforcement of
differences of things or in their relation. Neither is it tariff and customs laws;
necessary that the classification be made with
mathematical nicety. Hence, legislative classification (4) Prevent and suppress smuggling, pilferage and
may in many cases properly rest on narrow all other economic frauds within all ports of entry;
distinctions, for the equal protection guaranty does
not preclude the legislature from recognizing degrees
(5) Supervise and control exports, imports, foreign
of evil or harm, and legislation is addressed to evils
mails and the clearance of vessels and aircrafts in all
as they may appear.21 (emphasis supplied)
ports of entry;

The equal protection clause recognizes a valid classification,


(6) Administer all legal requirements that are
that is, a classification that has a reasonable foundation or
appropriate;
rational basis and not arbitrary.22 With respect to RA 9335, its
expressed public policy is the optimization of the revenue-
generation capability and collection of the BIR and the (7) Prevent and prosecute smuggling and other
BOC.23 Since the subject of the law is the revenue- generation illegal activities in all ports under its jurisdiction;
capability and collection of the BIR and the BOC, the incentives
and/or sanctions provided in the law should logically pertain to (8) Exercise supervision and control over its
the said agencies. Moreover, the law concerns only the BIR constituent units;
and the BOC because they have the common distinct primary
function of generating revenues for the national government
through the collection of taxes, customs duties, fees and (9) Perform such other functions as may be provided
charges. by law.25

The BIR performs the following functions: xxx xxx xxx (emphasis supplied)

Sec. 18. The Bureau of Internal Revenue. – The Both the BIR and the BOC are bureaus under the DOF. They
Bureau of Internal Revenue, which shall be headed principally perform the special function of being the
by and subject to the supervision and control of the instrumentalities through which the State exercises one of its
Commissioner of Internal Revenue, who shall be great inherent functions – taxation. Indubitably, such
appointed by the President upon the substantial distinction is germane and intimately related to the
recommendation of the Secretary [of the DOF], shall purpose of the law. Hence, the classification and treatment
have the following functions: accorded to the BIR and the BOC under RA 9335 fully satisfy
the demands of equal protection.
(1) Assess and collect all taxes, fees and charges
and account for all revenues collected; Undue Delegation

(2) Exercise duly delegated police powers for the Two tests determine the validity of delegation of legislative
proper performance of its functions and duties; power: (1) the completeness test and (2) the sufficient standard
test. A law is complete when it sets forth therein the policy to
be executed, carried out or implemented by the delegate. 26 It
lays down a sufficient standard when it provides adequate
FINALS CONSTITUTION I ACJUCO 4

guidelines or limitations in the law to map out the boundaries employees whose revenue collection falls short of the target by
of the delegate’s authority and prevent the delegation from at least 7.5% may be removed from the service:
running riot.27 To be sufficient, the standard must specify the
limits of the delegate’s authority, announce the legislative
SEC. 7. Powers and Functions of the Board. – The
policy and identify the conditions under which it is to be Board in the agency shall have the following powers
implemented.28 and functions:

RA 9335 adequately states the policy and standards to guide


xxx xxx xxx
the President in fixing revenue targets and the implementing
agencies in carrying out the provisions of the law. Section 2
spells out the policy of the law: (b) To set the criteria and procedures for removing
from service officials and employees whose
revenue collection falls short of the target by at
SEC. 2. Declaration of Policy. – It is the policy of the least seven and a half percent (7.5%), with due
State to optimize the revenue-generation capability
consideration of all relevant factors affecting the
and collection of the Bureau of Internal Revenue level of collection as provided in the rules and
(BIR) and the Bureau of Customs (BOC) by providing regulations promulgated under this Act, subject to
for a system of rewards and sanctions through the civil service laws, rules and regulations and
creation of a Rewards and Incentives Fund and a compliance with substantive and procedural due
Revenue Performance Evaluation Board in the process: Provided, That the following exemptions
above agencies for the purpose of encouraging their shall apply:
officials and employees to exceed their revenue
targets.
1. Where the district or area of
responsibility is newly-created, not
Section 4 "canalized within banks that keep it from exceeding two years in operation, as has
overflowing"29 the delegated power to the President to fix
no historical record of collection
revenue targets: performance that can be used as basis for
evaluation; and
SEC. 4. Rewards and Incentives Fund. – A Rewards
and Incentives Fund, hereinafter referred to as the 2. Where the revenue or customs official or
Fund, is hereby created, to be sourced from the employee is a recent transferee in the
collection of the BIR and the BOC in excess of their
middle of the period under consideration
respective revenue targets of the year, as unless the transfer was due to
determined by the Development Budget and
nonperformance of revenue targets or
Coordinating Committee (DBCC), in the following potential nonperformance of revenue
percentages: targets: Provided, however, That when the
district or area of responsibility covered by
Excess of Collection of the Excess the Percent (%) of the Excess CollectionrevenuetoorAccrue
customs
to officials
the or employees
Revenue Targets Fund has suffered from economic difficulties
30% or below – 15% brought about by natural calamities or force
More than 30% – 15% of the first 30% plus 20%majeure or economic
of the remaining excesscauses as may be
determined by the Board, termination shall
be considered only after careful and proper
The Fund shall be deemed automatically review by the Board.
appropriated the year immediately following the year
when the revenue collection target was exceeded
(c) To terminate personnel in accordance with the
and shall be released on the same fiscal year.
criteria adopted in the preceding paragraph:
Provided, That such decision shall be immediately
Revenue targets shall refer to the original executory: Provided, further, That the application of
estimated revenue collection expected of the BIR the criteria for the separation of an official or
and the BOC for a given fiscal year as stated in employee from service under this Act shall be
the Budget of Expenditures and Sources of without prejudice to the application of other
Financing (BESF) submitted by the President to relevant laws on accountability of public officers
Congress. The BIR and the BOC shall submit to the and employees, such as the Code of Conduct and
DBCC the distribution of the agencies’ revenue Ethical Standards of Public Officers and
targets as allocated among its revenue districts in the Employees and the Anti-Graft and Corrupt
case of the BIR, and the collection districts in the Practices Act;
case of the BOC.
xxx xxx xxx (emphasis supplied)
xxx xxx xxx (emphasis supplied)
Clearly, RA 9335 in no way violates the security of tenure of
Revenue targets are based on the original estimated revenue officials and employees of the BIR and the BOC. The
collection expected respectively of the BIR and the BOC for a guarantee of security of tenure only means that an employee
given fiscal year as approved by the DBCC and stated in the cannot be dismissed from the service for causes other than
BESF submitted by the President to Congress. 30 Thus, the those provided by law and only after due process is accorded
determination of revenue targets does not rest solely on the the employee.31 In the case of RA 9335, it lays down a
President as it also undergoes the scrutiny of the DBCC. reasonable yardstick for removal (when the revenue collection
falls short of the target by at least 7.5%) with due consideration
On the other hand, Section 7 specifies the limits of the Board’s of all relevant factors affecting the level of collection. This
authority and identifies the conditions under which officials and standard is analogous to inefficiency and incompetence in the
performance of official duties, a ground for disciplinary action
FINALS CONSTITUTION I ACJUCO 5

under civil service laws.32 The action for removal is also subject Over the years, Congress has invoked its oversight
to civil service laws, rules and regulations and compliance with power with increased frequency to check the
substantive and procedural due process. perceived "exponential accumulation of power" by
the executive branch. By the beginning of the
At any rate, this Court has recognized the following as 20th century, Congress has delegated an enormous
sufficient standards: "public interest," "justice and equity," amount of legislative authority to the executive
"public convenience and welfare" and "simplicity, economy and branch and the administrative agencies. Congress,
welfare."33 In this case, the declared policy of optimization of thus, uses its oversight power to make sure that the
the revenue-generation capability and collection of the BIR and administrative agencies perform their functions
the BOC is infused with public interest. within the authority delegated to them. x x x x x x x x
x

Separation Of Powers
Categories of congressional oversight functions

Section 12 of RA 9335 provides:


The acts done by Congress purportedly in the
exercise of its oversight powers may be divided
SEC. 12. Joint Congressional Oversight Committee. into three categories,
– There is hereby created a Joint Congressional namely: scrutiny, investigation and supervision.
Oversight Committee composed of seven Members
from the Senate and seven Members from the House
of Representatives. The Members from the Senate a. Scrutiny
shall be appointed by the Senate President, with at
least two senators representing the minority. The Congressional scrutiny implies a lesser
Members from the House of Representatives shall be intensity and continuity of attention to
appointed by the Speaker with at least two members administrative operations. Its primary
representing the minority. After the Oversight purpose is to determine economy and
Committee will have approved the implementing efficiency of the operation of government
rules and regulations (IRR) it shall thereafter activities. In the exercise of legislative
become functus officio and therefore cease to exist. scrutiny, Congress may request
information and report from the other
The Joint Congressional Oversight Committee in RA 9335 was branches of government. It can give
recommendations or pass resolutions for
created for the purpose of approving the implementing rules
and regulations (IRR) formulated by the DOF, DBM, NEDA, consideration of the agency involved.
BIR, BOC and CSC. On May 22, 2006, it approved the said
IRR. From then on, it became functus officio and ceased to xxx xxx xxx
exist. Hence, the issue of its alleged encroachment on the
executive function of implementing and enforcing the law may b. Congressional investigation
be considered moot and academic.

While congressional scrutiny is regarded


This notwithstanding, this might be as good a time as any for
as a passive process of looking at the facts
the Court to confront the issue of the constitutionality of the that are readily available, congressional
Joint Congressional Oversight Committee created under RA
investigation involves a more intense
9335 (or other similar laws for that matter). digging of facts. The power of Congress to
conduct investigation is recognized by the
The scholarly discourse of Mr. Justice (now Chief Justice) 1987 Constitution under section 21, Article
Puno on the concept of congressional oversight in Macalintal VI, xxx xxx xxx
v. Commission on Elections34 is illuminating:
c. Legislative supervision
Concept and bases of congressional oversight
The third and most encompassing form by which
Broadly defined, the power of oversight embraces Congress exercises its oversight power is thru
all activities undertaken by Congress to enhance legislative supervision. "Supervision" connotes a
its understanding of and influence over continuing and informed awareness on the part of a
the implementation of legislation it has enacted. congressional committee regarding executive
Clearly, oversight concerns post- operations in a given administrative area. While both
enactment measures undertaken by Congress: congressional scrutiny and investigation involve
(a) to monitor bureaucratic compliance with inquiry into past executive branch actions in order to
program objectives, (b) to determine whether influence future executive branch
agencies are properly administered, (c) to performance, congressional supervision allows
eliminate executive waste and dishonesty, (d) to Congress to scrutinize the exercise of delegated law-
prevent executive usurpation of legislative making authority, and permits Congress to retain part
authority, and (d) to assess executive conformity of that delegated authority.
with the congressional perception of public
interest. Congress exercises supervision over the executive
agencies through its veto power. It typically utilizes
The power of oversight has been held to be intrinsic veto provisions when granting the President or an
in the grant of legislative power itself and integral to executive agency the power to promulgate
the checks and balances inherent in a democratic regulations with the force of law. These provisions
system of government. x x x x x x x x x require the President or an agency to present the
proposed regulations to Congress, which retains a
"right" to approve or disapprove any regulation
FINALS CONSTITUTION I ACJUCO 6

before it takes effect. Such legislative veto provisions permits Congress to participate prospectively in the
usually provide that a proposed regulation will approval or disapproval of "subordinate law" or those
become a law after the expiration of a certain period enacted by the executive branch pursuant to a
of time, only if Congress does not affirmatively delegation of authority by Congress. They further
disapprove of the regulation in the meantime. Less argue that legislative veto "is a necessary response
frequently, the statute provides that a proposed by Congress to the accretion of policy control by
regulation will become law if Congress affirmatively forces outside its chambers." In an era of delegated
approves it. authority, they point out that legislative veto "is the
most efficient means Congress has yet devised to
Supporters of legislative veto stress that it is retain control over the evolution and implementation
necessary to maintain the balance of power between of its policy as declared by statute."
the legislative and the executive branches of
government as it offers lawmakers a way to delegate In Immigration and Naturalization Service v.
vast power to the executive branch or to independent Chadha, the U.S. Supreme Court resolved the
agencies while retaining the option to cancel validity of legislative veto provisions. The case
particular exercise of such power without having to arose from the order of the immigration judge
pass new legislation or to repeal existing law. They suspending the deportation of Chadha pursuant to §
contend that this arrangement promotes democratic 244(c)(1) of the Immigration and Nationality Act. The
accountability as it provides legislative check on the United States House of Representatives passed a
activities of unelected administrative agencies. One resolution vetoing the suspension pursuant to §
proponent thus explains: 244(c)(2) authorizing either House of Congress, by
resolution, to invalidate the decision of the executive
It is too late to debate the merits of this branch to allow a particular deportable alien to
delegation policy: the policy is too deeply remain in the United States. The immigration judge
embedded in our law and practice. It reopened the deportation proceedings to implement
suffices to say that the complexities of the House order and the alien was ordered deported.
modern government have often led The Board of Immigration Appeals dismissed the
alien’s appeal, holding that it had no power to declare
Congress-whether by actual or perceived
necessity- to legislate by declaring broad unconstitutional an act of Congress. The United
States Court of Appeals for Ninth Circuit held that the
policy goals and general statutory
standards, leaving the choice of policy House was without constitutional authority to order
options to the discretion of an executive the alien’s deportation and that § 244(c)(2) violated
officer. Congress articulates legislative the constitutional doctrine on separation of powers.
aims, but leaves their implementation to the
judgment of parties who may or may not On appeal, the U.S. Supreme Court declared §
have participated in or agreed with the 244(c)(2) unconstitutional. But the Court shied
development of those aims. Consequently, away from the issue of separation of powers and
absent safeguards, in many instances the instead held that the provision violates the
reverse of our constitutional scheme could presentment clause and bicameralism. It held that
be effected: Congress proposes, the the one-house veto was essentially legislative in
Executive disposes. One safeguard, of purpose and effect. As such, it is subject to the
course, is the legislative power to enact procedures set out in Article I of the Constitution
new legislation or to change existing law. requiring the passage by a majority of both Houses
But without some means of overseeing and presentment to the President. x x x x x x x x x
post enactment activities of the executive
branch, Congress would be unable to Two weeks after the Chadha decision, the Court
determine whether its policies have been upheld, in memorandum decision, two lower court
implemented in accordance with legislative decisions invalidating the legislative veto provisions
intent and thus whether legislative in the Natural Gas Policy Act of 1978 and the Federal
intervention is appropriate. Trade Commission Improvement Act of 1980.
Following this precedence, lower courts invalidated
Its opponents, however, criticize the legislative statutes containing legislative veto provisions
veto as undue encroachment upon the executive although some of these provisions required the
prerogatives. They urge that any post-enactment approval of both Houses of Congress and thus met
measures undertaken by the legislative branch the bicameralism requirement of Article I. Indeed,
should be limited to scrutiny and investigation; some of these veto provisions were not even
any measure beyond that would undermine the exercised.35(emphasis supplied)
separation of powers guaranteed by the
Constitution. They contend that legislative veto In Macalintal, given the concept and configuration of the power
constitutes an impermissible evasion of the of congressional oversight and considering the nature and
President’s veto authority and intrusion into the powers of a constitutional body like the Commission on
powers vested in the executive or judicial branches Elections, the Court struck down the provision in RA 9189 (The
of government. Proponents counter that legislative Overseas Absentee Voting Act of 2003) creating a Joint
veto enhances separation of powers as it prevents Congressional Committee. The committee was tasked not only
the executive branch and independent agencies from to monitor and evaluate the implementation of the said law but
accumulating too much power. They submit that also to review, revise, amend and approve the IRR
reporting requirements and congressional committee
promulgated by the Commission on Elections. The Court held
investigations allow Congress to scrutinize only the that these functions infringed on the constitutional
exercise of delegated law-making authority. They do
independence of the Commission on Elections. 36
not allow Congress to review executive proposals
before they take effect and they do not afford the
opportunity for ongoing and binding expressions of With this backdrop, it is clear that congressional oversight is
congressional intent. In contrast, legislative veto not unconstitutional per se, meaning, it neither necessarily
FINALS CONSTITUTION I ACJUCO 7

constitutes an encroachment on the executive power to law and enjoy the presumption of constitutionality and legality
implement laws nor undermines the constitutional separation until they are set aside with finality in an appropriate case by a
of powers. Rather, it is integral to the checks and balances competent court.51 Congress, in the guise of assuming the role
inherent in a democratic system of government. It may in fact of an overseer, may not pass upon their legality by subjecting
even enhance the separation of powers as it prevents the over- them to its stamp of approval without disturbing the calculated
accumulation of power in the executive branch. balance of powers established by the Constitution. In
exercising discretion to approve or disapprove the IRR based
However, to forestall the danger of congressional on a determination of whether or not they conformed with the
encroachment "beyond the legislative sphere," the Constitution provisions of RA 9335, Congress arrogated judicial power unto
imposes two basic and related constraints on Congress. 37 It itself, a power exclusively vested in this Court by the
may not vest itself, any of its committees or its members with Constitution.
either executive or judicial power.38 And, when it exercises its
legislative power, it must follow the "single, finely wrought and Considered Opinion of
exhaustively considered, procedures" specified under the Mr. Justice Dante O. Tinga
Constitution,39 including the procedure for enactment of laws
and presentment. Moreover, the requirement that the implementing rules of a law
be subjected to approval by Congress as a condition for their
Thus, any post-enactment congressional measure such as this effectivity violates the cardinal constitutional principles of
should be limited to scrutiny and investigation. In particular, bicameralism and the rule on presentment. 52
congressional oversight must be confined to the following:
Section 1, Article VI of the Constitution states:
(1) scrutiny based primarily on Congress’ power of
appropriation and the budget hearings conducted in Section 1. The legislative power shall be vested in
connection with it, its power to ask heads of the Congress of the Philippines which shall
departments to appear before and be heard by either
consist of a Senate and a House of
of its Houses on any matter pertaining to their Representatives, except to the extent reserved to
departments and its power of confirmation40 and
the people by the provision on initiative and
referendum. (emphasis supplied)
(2) investigation and monitoring41 of the
implementation of laws pursuant to the power of Legislative power (or the power to propose, enact, amend and
Congress to conduct inquiries in aid of legislation. 42
repeal laws)53 is vested in Congress which consists of two
chambers, the Senate and the House of Representatives. A
Any action or step beyond that will undermine the separation valid exercise of legislative power requires the act of both
of powers guaranteed by the Constitution. Legislative vetoes chambers. Corrollarily, it can be exercised neither solely by
fall in this class. one of the two chambers nor by a committee of either or both
chambers. Thus, assuming the validity of a legislative veto,
both a single-chamber legislative veto and a congressional
Legislative veto is a statutory provision requiring the President
or an administrative agency to present the proposed committee legislative veto are invalid.
implementing rules and regulations of a law to Congress which,
by itself or through a committee formed by it, retains a "right" Additionally, Section 27(1), Article VI of the Constitution
or "power" to approve or disapprove such regulations before provides:
they take effect. As such, a legislative veto in the form of a
congressional oversight committee is in the form of an inward- Section 27. (1) Every bill passed by the Congress
turning delegation designed to attach a congressional leash shall, before it becomes a law, be presented to
(other than through scrutiny and investigation) to an agency to the President. If he approves the same, he shall sign
which Congress has by law initially delegated broad it, otherwise, he shall veto it and return the same with
powers.43 It radically changes the design or structure of the his objections to the House where it originated, which
Constitution’s diagram of power as it entrusts to Congress a shall enter the objections at large in its Journal and
direct role in enforcing, applying or implementing its own proceed to reconsider it. If, after such
laws.44
reconsideration, two-thirds of all the Members of
such House shall agree to pass the bill, it shall be
Congress has two options when enacting legislation to define sent, together with the objections, to the other House
national policy within the broad horizons of its legislative by which it shall likewise be reconsidered, and if
competence.45 It can itself formulate the details or it can assign approved by two-thirds of all the Members of that
to the executive branch the responsibility for making necessary House, it shall become a law. In all such cases, the
managerial decisions in conformity with those standards. 46 In votes of each House shall be determined
the latter case, the law must be complete in all its essential by yeas or nays, and the names of the members
terms and conditions when it leaves the hands of the voting for or against shall be entered in its Journal.
legislature.47 Thus, what is left for the executive branch or the The President shall communicate his veto of any bill
concerned administrative agency when it formulates rules and to the House where it originated within thirty days
regulations implementing the law is to fill up details after the date of receipt thereof; otherwise, it shall
(supplementary rule-making) or ascertain facts necessary to become a law as if he had signed it. (emphasis
bring the law into actual operation (contingent rule-making).48 supplied)

Administrative regulations enacted by administrative agencies Every bill passed by Congress must be presented to the
to implement and interpret the law which they are entrusted to President for approval or veto. In the absence of presentment
enforce have the force of law and are entitled to to the President, no bill passed by Congress can become a
respect.49 Such rules and regulations partake of the nature of law. In this sense, law-making under the Constitution is a joint
a statute50 and are just as binding as if they have been written act of the Legislature and of the Executive. Assuming that
in the statute itself. As such, they have the force and effect of legislative veto is a valid legislative act with the force of law, it
FINALS CONSTITUTION I ACJUCO 8

cannot take effect without such presentment even if approved of the statute. Subject to the indispensable requisite of
by both chambers of Congress. publication under the due process clause,61 the determination
as to when a law takes effect is wholly the prerogative of
Congress.62 As such, it is only upon its effectivity that a law
In sum, two steps are required before a bill becomes a law.
First, it must be approved by both Houses of may be executed and the executive branch acquires the duties
Congress.54 Second, it must be presented to and approved by and powers to execute the said law. Before that point, the role
the President.55 As summarized by Justice Isagani Cruz56 and of the executive branch, particularly of the President, is limited
Fr. Joaquin G. Bernas, S.J.57, the following is the procedure for to approving or vetoing the law.63
the approval of bills:
From the moment the law becomes effective, any provision of
A bill is introduced by any member of the House of law that empowers Congress or any of its members to play any
Representatives or the Senate except for some role in the implementation or enforcement of the law violates
measures that must originate only in the former the principle of separation of powers and is thus
chamber. unconstitutional. Under this principle, a provision that requires
Congress or its members to approve the implementing rules of
a law after it has already taken effect shall be unconstitutional,
The first reading involves only a reading of the as is a provision that allows Congress or its members to
number and title of the measure and its referral by overturn any directive or ruling made by the members of the
the Senate President or the Speaker to the proper executive branch charged with the implementation of the law.
committee for study.
Following this rationale, Section 12 of RA 9335 should be
The bill may be "killed" in the committee or it may be struck down as unconstitutional. While there may be similar
recommended for approval, with or without provisions of other laws that may be invalidated for failure to
amendments, sometimes after public hearings are pass this standard, the Court refrains from invalidating them
first held thereon. If there are other bills of the same wholesale but will do so at the proper time when an appropriate
nature or purpose, they may all be consolidated into case assailing those provisions is brought before us. 64
one bill under common authorship or as a committee
bill.
The next question to be resolved is: what is the effect of the
unconstitutionality of Section 12 of RA 9335 on the other
Once reported out, the bill shall be calendared for provisions of the law? Will it render the entire law
second reading. It is at this stage that the bill is read unconstitutional? No.
in its entirety, scrutinized, debated upon and
amended when desired. The second reading is the
Section 13 of RA 9335 provides:
most important stage in the passage of a bill.

SEC. 13. Separability Clause. – If any provision of


The bill as approved on second reading is printed in
its final form and copies thereof are distributed at this Act is declared invalid by a competent court, the
remainder of this Act or any provision not affected by
least three days before the third reading. On the third
reading, the members merely register their votes and such declaration of invalidity shall remain in force and
explain them if they are allowed by the rules. No effect.
further debate is allowed.
In Tatad v. Secretary of the Department of Energy,65 the Court
Once the bill passes third reading, it is sent to the laid down the following rules:
other chamber, where it will also undergo the three
readings. If there are differences between the The general rule is that where part of a statute is void
versions approved by the two chambers, a as repugnant to the Constitution, while another part
conference committee58 representing both Houses is valid, the valid portion, if separable from the invalid,
will draft a compromise measure that if ratified by the may stand and be enforced. The presence of a
Senate and the House of Representatives will then separability clause in a statute creates the
be submitted to the President for his consideration. presumption that the legislature intended
separability, rather than complete nullity of the
The bill is enrolled when printed as finally approved statute. To justify this result, the valid portion must be
by the Congress, thereafter authenticated with the so far independent of the invalid portion that it is fair
signatures of the Senate President, the Speaker, and to presume that the legislature would have enacted it
by itself if it had supposed that it could not
the Secretaries of their respective chambers…59
constitutionally enact the other. Enough must remain
to make a complete, intelligible and valid statute,
The President’s role in law-making. which carries out the legislative intent. x x x

The final step is submission to the President for The exception to the general rule is that when the
approval. Once approved, it takes effect as law after parts of a statute are so mutually dependent and
the required publication.60 connected, as conditions, considerations,
inducements, or compensations for each other, as to
Where Congress delegates the formulation of rules to warrant a belief that the legislature intended them as
implement the law it has enacted pursuant to sufficient a whole, the nullity of one part will vitiate the rest. In
standards established in the said law, the law must be making the parts of the statute dependent,
complete in all its essential terms and conditions when it leaves conditional, or connected with one another, the
the hands of the legislature. And it may be deemed to have left legislature intended the statute to be carried out as a
the hands of the legislature when it becomes effective because whole and would not have enacted it if one part is
it is only upon effectivity of the statute that legal rights and void, in which case if some parts are unconstitutional,
obligations become available to those entitled by the language
FINALS CONSTITUTION I ACJUCO 9

all the other provisions thus dependent, conditional,


or connected must fall with them.

The separability clause of RA 9335 reveals the intention of the


legislature to isolate and detach any invalid provision from the
other provisions so that the latter may continue in force and
effect. The valid portions can stand independently of the invalid
section. Without Section 12, the remaining provisions still
constitute a complete, intelligible and valid law which carries
out the legislative intent to optimize the revenue-generation
capability and collection of the BIR and the BOC by providing
for a system of rewards and sanctions through the Rewards
and Incentives Fund and a Revenue Performance Evaluation
Board.

To be effective, administrative rules and regulations must be


published in full if their purpose is to enforce or implement
existing law pursuant to a valid delegation. The IRR of RA 9335
were published on May 30, 2006 in two newspapers of general
circulation66 and became effective 15 days thereafter.67 Until
and unless the contrary is shown, the IRR are presumed valid
and effective even without the approval of the Joint
Congressional Oversight Committee.

WHEREFORE, the petition is hereby PARTIALLY


GRANTED. Section 12 of RA 9335 creating a Joint
Congressional Oversight Committee to approve the
implementing rules and regulations of the law is
declared UNCONSTITUTIONAL and
therefore NULL and VOID. The constitutionality of the
remaining provisions of RA 9335 is UPHELD. Pursuant to
Section 13 of RA 9335, the rest of the provisions remain in
force and effect.

SO ORDERED.
FINALS CONSTITUTION I ACJUCO 10

[G.R. No. 163783. June 22, 2004] Moreover, as pointed out in the Comment filed by the Senate
Panel for respondent Joint Committee and that of the Office of
PIMENTEL vs. CONGRESS the Solicitor General, the precedents set by the 1992 and 1998
Presidential Elections do not support the move to stop the
ongoing canvassing by the Joint Committee, they citing the
Gentlemen: observations of former Senate President Jovito Salonga.

Quoted hereunder, for your information, is a resolution of this Thus, during the 1992 Presidential elections, both Houses of
Court dated JUN 22 2004. Congress adjourned sine die on May 25, 1992. On June 16,
1992, the Joint Committee finished tallying the votes for
G. R. No. 163783 (Aquilino Q. Pimentel, Jr. vs. Joint President and Vice-President.[1] Thereafter, on June 22, 1992,
Committee of Congress to Canvass the Votes Cast for the Eighth Congress convened in joint public session as the
President and Vice-President in the May 10, 2004 Elections.) National Board of Canvassers, and on even date proclaimed
Fidel V. Ramos and Joseph Ejercito Estrada as President and
Vice-President, respectively.[2]
RESOLUTION
Upon the other hand, during the 1998 Presidential elections,
By the present Petition for Prohibition, petitioner Senator both Houses of Congress adjourned sine dieon May 25, 1998.
Aquilino Q. Pimentel, Jr. seeks a judgment declaring null and The Joint Committee completed the counting of the votes for
void the continued existence of the Joint Committee of President and Vice-President on May 27, 1998.[3] The Tenth
Congress (Joint Committee) to determine the authenticity and Congress then convened in joint public session on May 29,
due execution of the certificates of canvass and preliminarily 1998 as the National Board of Canvassers and proclaimed
canvass the votes cast for Presidential and Vice-Presidential Joseph Ejercito Estrada as President and Gloria Macapagal-
candidates in the May 10, 2004 elections following the Arroyo as President and Vice-President, respectively.[4]
adjournment of Congress sine die on June 11, 2004. The
petition corollarily prays for the issuance of a writ of prohibition
directing the Joint Committee to cease and desist from As for petitioner's argument that "the [e]xistence and
conducting any further proceedings pursuant to the Rules of [p]roceedings [o]f the Joint Committee of Congress [a]re
the Joint Public Session of Congress on Canvassing. [i]nvalid, [i]llegal and [u]nconstitutional [f]ollowing the
[a]djournment [s]ine [d]ie [o]f [b]oth Houses of Congress [o]f
[t]heir [r]egular [s]essions on June 11, 2004," he cites in
Petitioner posits that with "the adjournment sine die on June support thereof Section 15, Article VI of the Constitution which
11, 2004 by the Twelfth Congress of its last regular session, reads:
[its] term ... terminated and expired on the said day and the
said Twelfth Congress serving the term 2001 to 2004 passed
out of legal existence." Henceforth, petitioner goes on, "all Sec. 15. The Congress shall convene once every year on
pending matters and proceedings terminate upon the the fourth Monday of July for its regular session, unless a
expiration of ... Congress." To advance this view, he relies on different date is fixed by law, and shall continue to be in
"legislative procedure, precedent or practice [as] borne [out] by session for such number of days as it may determine until
the rules of both Houses of Congress." thirty days before the opening of its next regular session,
exclusive of Saturdays, Sundays, and legal holidays. The
President may call a special session at any time.
Given the importance of the constitutional issue raised and to
put to rest all questions regarding the regularity, validity or
constitutionality of the canvassing of votes fro President and Contrary to petitioner's argument, however, the term of
Vice-President in the recently concluded national elections, the present Twelfth Congress did not terminate and expire
this Court assumes jurisdiction over the instant petition upon the adjournment sine die of the regular session of
pursuant to its power and duty "to determine whether or not both Houses on June 11, 2004.
there has been a grave abuse of discretion amounting to lack
or excess of jurisdiction on the part of any branch or Section 15, Article VI of the Constitution cited by petitioner
instrumentality of the Government" under Section 1 of Article does not pertain to the term of Congress, but to its regular
VIII of the Constitution and its original jurisdiction over petitions annual legislative sessions and the mandatory 30-day
for prohibition under Section 5 of the same Article. recess before the opening of its next regular session (subject
to the power of the President to call a special session at any
After a considered and judicious examination of the arguments time).
raised by petitioner as well as those presented in the
Comments filed by the Solicitor General and respondent Joint Section 4 of Article VIII also of the Constitution clearly provides
Committee, this Court finds that the petition has absolutely no that "[t]he term of office of the Senators shall be six years and
basis under the Constitution and must, therefore, be shall commence, unless otherwise provided by law, at noon on
dismissed. the thirtieth day of June next following their election." Similarly,
Section 7 of the same Article provides that "[t]he Members of
Petitioner's claim that his arguments are buttressed by the House of Representatives shall be elected for
"legislative procedure, precedent or practice [as] borne [out] by a term of three years which shall begin, unless otherwise
the rules of both Houses of Congress" is directly contradicted provided by law, at noon on the thirtieth day of June next
by Section 42 of Rule XIV of the Rules adopted by the Senate, following their election." Consequently, there being no law to
of which he is an incumbent member. This section clearly the contrary, until June 30, 2004, the present Twelfth Congress
provides that the Senate shall convene in joint to which the present legislators belong cannot be said to have
session during any voluntary or compulsory recess to "passed out of legal existence."
canvass the votes for President and Vice-President not
later than thirty days after the day of the elections in The legislative functions of the Twelfth Congress may have
accordance with Section 4, Article VII of the Constitution. come to a close upon the final adjournment of its regular
sessions on June 11, 2004, but this does not affect its non-
legislative functions, such as that of being the National
FINALS CONSTITUTION I ACJUCO 11

Board of Canvassers. In fact, the joint public session of


both Houses of Congress convened by express directive
of Section 4, Article VII of the Constitution to canvass the
votes for and to proclaim the newly elected President and
Vice-President has not, and cannot, adjourn sine dieuntil
it has accomplished its constitutionally mandated tasks.
For only when a board of canvassers has completed its
functions is it rendered functus officio. Its membership
may change, but it retains its authority as a board until it
has accomplished its purposes. (Pelayo v. Commission on
Elections, 23 SCRA 1374, 1385 [1968], citing Bautista v.
Fugoso, 60 Phil. 383, 389 [1934] and Aquino v. Commission
on Elections, L-28392, January 29 1968)

Since the Twelfth Congress has not yet completed its non-
legislative duty to canvass the votes and proclaim the duly
elected President and Vice-President, its existence as the
National Board of Canvassers, as well as that of the Joint
Committee to which it referred the preliminary tasks of
authenticating and canvassing the certificates of canvass, has
not become functus officio.

In sum, despite the adjournment sine die of Congress, there is


no legal impediment to the Joint Committee completing the
tasks assigned to it and transmitting its report for the approval
of the joint public session of both Houses of Congress, which
may reconvene without need of call by the President to a
special session.

WHEREFORE, the instant Petition is hereby DISMISSED.


FINALS CONSTITUTION I ACJUCO 12

G.R. No. 146710-15 March 2, 2001 Calls for the resignation of the petitioner filled the air. On
October 11, Archbishop Jaime Cardinal Sin issued a pastoral
JOSEPH E. ESTRADA, petitioner, statement in behalf of the Presbyteral Council of the
Archdiocese of Manila, asking petitioner to step down from the
vs.
ANIANO DESIERTO, in his capacity as Ombudsman, presidency as he had lost the moral authority to govern. 3 Two
RAMON GONZALES, VOLUNTEERS AGAINST CRIME AND days later or on October 13, the Catholic Bishops Conference
CORRUPTION, GRAFT FREE PHILIPPINES FOUNDATION, of the Philippines joined the cry for the resignation of the
INC., LEONARD DE VERA, DENNIS FUNA, ROMEO petitioner.4 Four days later, or on October 17, former President
CAPULONG and ERNESTO B. FRANCISCO, Corazon C. Aquino also demanded that the petitioner take the
JR., respondent. "supreme self-sacrifice" of resignation.5 Former President
Fidel Ramos also joined the chorus. Early on, or on October
12, respondent Arroyo resigned as Secretary of the
---------------------------------------- Department of Social Welfare and Services 6 and later asked
for petitioner's resignation.7 However, petitioner strenuously
G.R. No. 146738 March 2, 2001 held on to his office and refused to resign.

JOSEPH E. ESTRADA, petitioner, The heat was on. On November 1, four (4) senior economic
vs. advisers, members of the Council of Senior Economic
GLORIA MACAPAGAL-ARROYO, respondent. Advisers, resigned. They were Jaime Augusto Zobel de Ayala,
former Prime Minister Cesar Virata, former Senator Vicente
Paterno and Washington Sycip.8 On November 2, Secretary
PUNO, J.: Mar Roxas II also resigned from the Department of Trade and
Industry.9 On November 3, Senate President Franklin Drilon,
On the line in the cases at bar is the office of the President. and House Speaker Manuel Villar, together with some 47
Petitioner Joseph Ejercito Estrada alleges that he is the representatives defected from the ruling coalition, Lapian ng
President on leave while respondent Gloria Macapagal-Arroyo Masang Pilipino.10
claims she is the President. The warring personalities are
important enough but more transcendental are the The month of November ended with a big bang. In a
constitutional issues embedded on the parties' dispute. While tumultuous session on November 13, House Speaker Villar
the significant issues are many, the jugular issue involves the transmitted the Articles of Impeachment 11 signed by 115
relationship between the ruler and the ruled in a democracy, representatives, or more than 1/3 of all the members of the
Philippine style. House of Representatives to the Senate. This caused political
convulsions in both houses of Congress. Senator Drilon was
First, we take a view of the panorama of events that replaced by Senator Pimentel as Senate President. Speaker
precipitated the crisis in the office of the President. Villar was unseated by Representative Fuentebella. 12 On
November 20, the Senate formally opened the impeachment
trial of the petitioner. Twenty-one (21) senators took their oath
In the May 11, 1998 elections, petitioner Joseph Ejercito as judges with Supreme Court Chief Justice Hilario G. Davide,
Estrada was elected President while respondent Gloria Jr., presiding.13
Macapagal-Arroyo was elected Vice-President. Some ten (10)
million Filipinos voted for the petitioner believing he would
rescue them from life's adversity. Both petitioner and the The political temperature rose despite the cold December. On
respondent were to serve a six-year term commencing on June December 7, the impeachment trial started.14 The battle royale
30, 1998. was fought by some of the marquee names in the legal
profession. Standing as prosecutors were then House Minority
Floor Leader Feliciano Belmonte and Representatives Joker
From the beginning of his term, however, petitioner was Arroyo, Wigberto Tañada, Sergio Apostol, Raul Gonzales,
plagued by a plethora of problems that slowly but surely eroded Oscar Moreno, Salacnib Baterina, Roan Libarios, Oscar
his popularity. His sharp descent from power started on Rodriguez, Clavel Martinez and Antonio Nachura. They were
October 4, 2000. Ilocos Sur Governor, Luis "Chavit" Singson, assisted by a battery of private prosecutors led by now
a longtime friend of the petitioner, went on air and accused the Secretary of Justice Hernando Perez and now Solicitor
petitioner, his family and friends of receiving millions of pesos General Simeon Marcelo. Serving as defense counsel were
from jueteng lords.1 former Chief Justice Andres Narvasa, former Solicitor General
and Secretary of Justice Estelito P. Mendoza, former City
The exposẻ immediately ignited reactions of rage. The next Fiscal of Manila Jose Flaminiano, former Deputy Speaker of
day, October 5, 2000, Senator Teofisto Guingona, Jr., then the the House Raul Daza, Atty. Siegfried Fortun and his brother,
Senate Minority Leader, took the floor and delivered a fiery Atty. Raymund Fortun. The day to day trial was covered by live
privilege speech entitled "I Accuse." He accused the petitioner TV and during its course enjoyed the highest viewing rating. Its
of receiving some P220 million in jueteng money from high and low points were the constant conversational piece of
Governor Singson from November 1998 to August 2000. He the chattering classes. The dramatic point of the December
also charged that the petitioner took from Governor Singson hearings was the testimony of Clarissa Ocampo, senior vice
P70 million on excise tax on cigarettes intended for Ilocos Sur. president of Equitable-PCI Bank. She testified that she was
The privilege speech was referred by then Senate President one foot away from petitioner Estrada when he affixed the
Franklin Drilon, to the Blue Ribbon Committee (then headed by signature "Jose Velarde" on documents involving a P500
Senator Aquilino Pimentel) and the Committee on Justice (then million investment agreement with their bank on February 4,
headed by Senator Renato Cayetano) for joint investigation. 2 2000.15

The House of Representatives did no less. The House After the testimony of Ocampo, the impeachment trial was
Committee on Public Order and Security, then headed by adjourned in the spirit of Christmas. When it resumed on
Representative Roilo Golez, decided to investigate the exposẻ January 2, 2001, more bombshells were exploded by the
of Governor Singson. On the other hand, Representatives prosecution. On January 11, Atty. Edgardo Espiritu who served
Heherson Alvarez, Ernesto Herrera and Michael Defensor as petitioner's Secretary of Finance took the witness stand. He
spearheaded the move to impeach the petitioner. alleged that the petitioner jointly owned BW Resources
FINALS CONSTITUTION I ACJUCO 13

Corporation with Mr. Dante Tan who was facing charges of broke out that Chief Justice Davide would administer the oath
insider trading.16 Then came the fateful day of January 16, to respondent Arroyo at high noon at the EDSA Shrine.
when by a vote of 11-1017 the senator-judges ruled against the
opening of the second envelope which allegedly contained
At about 12:00 noon, Chief Justice Davide administered the
evidence showing that petitioner held P3.3 billion in a secret oath to respondent Arroyo as President of the Philippines. 28 At
bank account under the name "Jose Velarde." The public and 2:30 p.m., petitioner and his family hurriedly left Malacañang
private prosecutors walked out in protest of the ruling. In Palace.29 He issued the following press statement:30
disgust, Senator Pimentel resigned as Senate President. 18 The
ruling made at 10:00 p.m. was met by a spontaneous outburst
of anger that hit the streets of the metropolis. By midnight, "20 January 2001
thousands had assembled at the EDSA Shrine and speeches
full of sulphur were delivered against the petitioner and the STATEMENT FROM
eleven (11) senators.
PRESIDENT JOSEPH EJERCITO ESTRADA
On January 17, the public prosecutors submitted a letter to
Speaker Fuentebella tendering their collective resignation.
They also filed their Manifestation of Withdrawal of At twelve o'clock noon today, Vice President Gloria
Appearance with the impeachment tribunal. 19Senator Raul Macapagal-Arroyo took her oath as President of the
Roco quickly moved for the indefinite postponement of the Republic of the Philippines. While along with many
impeachment proceedings until the House of Representatives other legal minds of our country, I have strong and
shall have resolved the issue of resignation of the public serious doubts about the legality and constitutionality
prosecutors. Chief Justice Davide granted the motion. 20 of her proclamation as President, I do not wish to be
a factor that will prevent the restoration of unity and
order in our civil society.
January 18 saw the high velocity intensification of the call for
petitioner's resignation. A 10-kilometer line of people holding
lighted candles formed a human chain from the Ninoy Aquino It is for this reason that I now leave Malacañang
Monument on Ayala Avenue in Makati City to the EDSA Shrine Palace, the seat of the presidency of this country, for
to symbolize the people's solidarity in demanding petitioner's the sake of peace and in order to begin the healing
resignation. Students and teachers walked out of their classes process of our nation. I leave the Palace of our
in Metro Manila to show their concordance. Speakers in the people with gratitude for the opportunities given to
continuing rallies at the EDSA Shrine, all masters of the me for service to our people. I will not shirk from any
physics of persuasion, attracted more and more people. 21 future challenges that may come ahead in the same
service of our country.
On January 19, the fall from power of the petitioner appeared
inevitable. At 1:20 p.m., the petitioner informed Executive I call on all my supporters and followers to join me in
Secretary Edgardo Angara that General Angelo Reyes, Chief to promotion of a constructive national spirit of
of Staff of the Armed Forces of the Philippines, had defected. reconciliation and solidarity.
At 2:30 p.m., petitioner agreed to the holding of a snap election
for President where he would not be a candidate. It did not May the Almighty bless our country and beloved
diffuse the growing crisis. At 3:00 p.m., Secretary of National people.
Defense Orlando Mercado and General Reyes, together with
the chiefs of all the armed services went to the EDSA
Shrine.22 In the presence of former Presidents Aquino and MABUHAY!
Ramos and hundreds of thousands of cheering demonstrators,
General Reyes declared that "on behalf of Your Armed Forces, (Sgd.) JOSEPH EJERCITO ESTRADA"
the 130,000 strong members of the Armed Forces, we wish to
announce that we are withdrawing our support to this
It also appears that on the same day, January 20, 2001, he
government."23 A little later, PNP Chief, Director General
signed the following letter:31
Panfilo Lacson and the major service commanders gave a
similar stunning announcement. 24 Some Cabinet secretaries,
undersecretaries, assistant secretaries, and bureau chiefs "Sir:
quickly resigned from their posts.25 Rallies for the resignation
of the petitioner exploded in various parts of the country. To By virtue of the provisions of Section 11, Article VII of
stem the tide of rage, petitioner announced he was ordering the Constitution, I am hereby transmitting this
his lawyers to agree to the opening of the highly controversial declaration that I am unable to exercise the powers
second envelope.26There was no turning back the tide. The and duties of my office. By operation of law and the
tide had become a tsunami. Constitution, the Vice-President shall be the Acting
President.
January 20 turned to be the day of surrender. At 12:20 a.m.,
the first round of negotiations for the peaceful and orderly (Sgd.) JOSEPH EJERCITO ESTRADA"
transfer of power started at Malacañang'' Mabini Hall, Office of
the Executive Secretary. Secretary Edgardo Angara, Senior
Deputy Executive Secretary Ramon Bagatsing, Political A copy of the letter was sent to former Speaker Fuentebella at
Adviser Angelito Banayo, Asst. Secretary Boying Remulla, and 8:30 a.m. on January 20.23 Another copy was transmitted to
Atty. Macel Fernandez, head of the Presidential Management Senate President Pimentel on the same day although it was
Staff, negotiated for the petitioner. Respondent Arroyo was received only at 9:00 p.m.33
represented by now Executive Secretary Renato de Villa, now
Secretary of Finance Alberto Romulo and now Secretary of On January 22, the Monday after taking her oath, respondent
Justice Hernando Perez.27 Outside the palace, there was a Arroyo immediately discharged the powers the duties of the
brief encounter at Mendiola between pro and anti-Estrada Presidency. On the same day, this Court issued the following
protesters which resulted in stone-throwing and caused minor Resolution in Administrative Matter No. 01-1-05-SC, to wit:
injuries. The negotiations consumed all morning until the news
FINALS CONSTITUTION I ACJUCO 14

"A.M. No. 01-1-05-SC — In re: Request of Vice Meanwhile, in a survey conducted by Pulse Asia, President
President Gloria Macapagal-Arroyo to Take her Oath Arroyo's public acceptance rating jacked up from 16% on
of Office as President of the Republic of the January 20, 2001 to 38% on January 26, 2001. 49 In another
Philippines before the Chief Justice — Acting on the survey conducted by the ABS-CBN/SWS from February 2-7,
urgent request of Vice President Gloria Macapagal- 2001, results showed that 61% of the Filipinos nationwide
Arroyo to be sworn in as President of the Republic of accepted President Arroyo as replacement of petitioner
the Philippines, addressed to the Chief Justice and Estrada. The survey also revealed that President Arroyo is
confirmed by a letter to the Court, dated January 20, accepted by 60% in Metro Manila, by also 60% in the balance
2001, which request was treated as an administrative of Luzon, by 71% in the Visayas, and 55% in Mindanao. Her
matter, the court Resolve unanimously to confirm the trust rating increased to 52%. Her presidency is accepted by
authority given by the twelve (12) members of the majorities in all social classes: 58% in the ABC or middle-to-
Court then present to the Chief Justice on January upper classes, 64% in the D or mass class, and 54% among
20, 2001 to administer the oath of office of Vice the E's or very poor class.50
President Gloria Macapagal-Arroyo as President of
the Philippines, at noon of January 20, 2001.
After his fall from the pedestal of power, the petitioner's legal
problems appeared in clusters. Several cases previously filed
This resolution is without prejudice to the disposition against him in the Office of the Ombudsman were set in
of any justiciable case that may be filed by a proper motion. These are: (1) OMB Case No. 0-00-1629, filed by
party." Ramon A. Gonzales on October 23, 2000 for bribery and graft
and corruption; (2) OMB Case No. 0-00-1754 filed by the
Respondent Arroyo appointed members of her Cabinet as well Volunteers Against Crime and Corruption on November 17,
as ambassadors and special envoys. 34 Recognition of 2000 for plunder, forfeiture, graft and corruption, bribery,
respondent Arroyo's government by foreign governments perjury, serious misconduct, violation of the Code of Conduct
swiftly followed. On January 23, in a reception or vin d' for Government Employees, etc; (3) OMB Case No. 0-00-1755
honneur at Malacañang, led by the Dean of the Diplomatic filed by the Graft Free Philippines Foundation, Inc. on
Corps, Papal Nuncio Antonio Franco, more than a hundred November 24, 2000 for plunder, forfeiture, graft and corruption,
foreign diplomats recognized the government of respondent bribery, perjury, serious misconduct; (4) OMB Case No. 0-00-
1756 filed by Romeo Capulong, et al., on November 28, 2000
Arroyo.35 US President George W. Bush gave the respondent
a telephone call from the White House conveying US for malversation of public funds, illegal use of public funds and
property, plunder, etc.; (5) OMB Case No. 0-00-1757 filed by
recognition of her government.36
Leonard de Vera, et al., on November 28, 2000 for bribery,
plunder, indirect bribery, violation of PD 1602, PD 1829, PD
On January 24, Representative Feliciano Belmonte was 46, and RA 7080; and (6) OMB Case No. 0-00-1758 filed by
elected new Speaker of the House of Representatives. 37The Ernesto B. Francisco, Jr. on December 4, 2000 for plunder,
House then passed Resolution No. 175 "expressing the full graft and corruption.
support of the House of Representatives to the administration
of Her Excellency, Gloria Macapagal-Arroyo, President of the
Philippines."38 It also approved Resolution No. 176 A special panel of investigators was forthwith created by the
"expressing the support of the House of Representatives to the respondent Ombudsman to investigate the charges against the
assumption into office by Vice President Gloria Macapagal- petitioner. It is chaired by Overall Deputy Ombudsman
Arroyo as President of the Republic of the Philippines, Margarito P. Gervasio with the following as members, viz:
Director Andrew Amuyutan, Prosecutor Pelayo Apostol, Atty.
extending its congratulations and expressing its support for her
administration as a partner in the attainment of the nation's Jose de Jesus and Atty. Emmanuel Laureso. On January 22,
the panel issued an Order directing the petitioner to file his
goals under the Constitution."39
counter-affidavit and the affidavits of his witnesses as well as
other supporting documents in answer to the aforementioned
On January 26, the respondent signed into law the Solid Waste complaints against him.
Management Act.40 A few days later, she also signed into law
the Political Advertising ban and Fair Election Practices Act. 41
Thus, the stage for the cases at bar was set. On February 5,
petitioner filed with this Court GR No. 146710-15, a petition for
On February 6, respondent Arroyo nominated Senator Teofisto prohibition with a prayer for a writ of preliminary injunction. It
Guingona, Jr., as her Vice President.42 The next day, February sought to enjoin the respondent Ombudsman from "conducting
7, the Senate adopted Resolution No. 82 confirming the any further proceedings in Case Nos. OMB 0-00-1629, 1754,
nomination of Senator Guingona, Jr.43Senators Miriam 1755, 1756, 1757 and 1758 or in any other criminal complaint
Defensor-Santiago, Juan Ponce Enrile, and John Osmena that may be filed in his office, until after the term of petitioner
voted "yes" with reservations, citing as reason therefor the as President is over and only if legally warranted." Thru
pending challenge on the legitimacy of respondent Arroyo's another counsel, petitioner, on February 6, filed GR No.
presidency before the Supreme Court. Senators Teresa 146738 for Quo Warranto. He prayed for judgment "confirming
Aquino-Oreta and Robert Barbers were absent. 44 The House petitioner to be the lawful and incumbent President of the
of Representatives also approved Senator Guingona's Republic of the Philippines temporarily unable to discharge the
nomination in Resolution No. 178.45 Senator Guingona, Jr. duties of his office, and declaring respondent to have taken her
took his oath as Vice President two (2) days later. 46 oath as and to be holding the Office of the President, only in
an acting capacity pursuant to the provisions of the
On February 7, the Senate passed Resolution No. 83 declaring Constitution." Acting on GR Nos. 146710-15, the Court, on the
that the impeachment court is functus officio and has been same day, February 6, required the respondents "to comment
terminated.47 Senator Miriam Defensor-Santiago stated "for thereon within a non-extendible period expiring on 12 February
the record" that she voted against the closure of the 2001." On February 13, the Court ordered the consolidation of
impeachment court on the grounds that the Senate had failed GR Nos. 146710-15 and GR No. 146738 and the filing of the
to decide on the impeachment case and that the resolution left respondents' comments "on or before 8:00 a.m. of February
open the question of whether Estrada was still qualified to run 15."
for another elective post.48
On February 15, the consolidated cases were orally argued in
a four-hour hearing. Before the hearing, Chief Justice Davide,
FINALS CONSTITUTION I ACJUCO 15

Jr.51 and Associate Justice Artemio Panganiban52 recused We shall discuss the issues in seriatim.
themselves on motion of petitioner's counsel, former Senator
Rene A. Saguisag. They debunked the charge of counsel I
Saguisag that they have "compromised themselves by
indicating that they have thrown their weight on one side" but
nonetheless inhibited themselves. Thereafter, the parties were Whether or not the cases at bar involve a political question
given the short period of five (5) days to file their memoranda
and two (2) days to submit their simultaneous replies. Private respondents54 raise the threshold issue that the cases
at bar pose a political question, and hence, are beyond the
In a resolution dated February 20, acting on the urgent motion jurisdiction of this Court to decide. They contend that shorn of
for copies of resolution and press statement for "Gag Order" its embroideries, the cases at bar assail the "legitimacy of the
on respondent Ombudsman filed by counsel for petitioner in Arroyo administration." They stress that respondent Arroyo
G.R. No. 146738, the Court resolved: ascended the presidency through people power; that she has
already taken her oath as the 14th President of the Republic;
that she has exercised the powers of the presidency and that
"(1) to inform the parties that the Court did not issue she has been recognized by foreign governments. They submit
a resolution on January 20, 2001 declaring the office that these realities on ground constitute the political thicket,
of the President vacant and that neither did the Chief which the Court cannot enter.
Justice issue a press statement justifying the alleged
resolution;
We reject private respondents' submission. To be sure, courts
here and abroad, have tried to lift the shroud on political
(2) to order the parties and especially their counsel question but its exact latitude still splits the best of legal minds.
who are officers of the Court under pain of being cited Developed by the courts in the 20th century, the political
for contempt to refrain from making any comment or question doctrine which rests on the principle of separation of
discussing in public the merits of the cases at bar powers and on prudential considerations, continue to be
while they are still pending decision by the Court, and
refined in the mills of constitutional law. 55 In the United States,
the most authoritative guidelines to determine whether a
(3) to issue a 30-day status quo order effective question is political were spelled out by Mr. Justice Brennan in
immediately enjoining the respondent Ombudsman the 1962 case or Baker v. Carr,56 viz:
from resolving or deciding the criminal cases pending
investigation in his office against petitioner, Joseph "x x x Prominent on the surface of any case held to
E. Estrada and subject of the cases at bar, it
involve a political question is found a textually
appearing from news reports that the respondent demonstrable constitutional commitment of the issue
Ombudsman may immediately resolve the cases
to a coordinate political department or a lack of
against petitioner Joseph E. Estrada seven (7) days judicially discoverable and manageable standards
after the hearing held on February 15, 2001, which for resolving it, or the impossibility of deciding without
action will make the cases at bar moot and an initial policy determination of a kind clearly for
academic."53 non-judicial discretion; or the impossibility of a court's
undertaking independent resolution without
The parties filed their replies on February 24. On this date, the expressing lack of the respect due coordinate
cases at bar were deemed submitted for decision. branches of government; or an unusual need for
unquestioning adherence to a political decision
The bedrock issues for resolution of this Court are: already made; or the potentiality of embarrassment
from multifarious pronouncements by various
departments on question. Unless one of these
I formulations is inextricable from the case at bar,
there should be no dismissal for non justiciability on
Whether the petitions present a justiciable the ground of a political question's presence. The
controversy. doctrine of which we treat is one of 'political
questions', not of 'political cases'."
II
In the Philippine setting, this Court has been continuously
confronted with cases calling for a firmer delineation of the
Assuming that the petitions present a justiciable inner and outer perimeters of a political question. 57 Our leading
controversy, whether petitioner Estrada is a case is Tanada v. Cuenco,58 where this Court, through former
President on leave while respondent Arroyo is an Chief Justice Roberto Concepcion, held that political questions
Acting President. refer "to those questions which, under the Constitution, are to
be decided by the people in their sovereign capacity, or in
III regard to which full discretionary authority has been
delegated to the legislative or executive branch of the
government. It is concerned with issues dependent upon
Whether conviction in the impeachment proceedings the wisdom, not legality of a particular measure." To a great
is a condition precedent for the criminal prosecution degree, the 1987 Constitution has narrowed the reach of the
of petitioner Estrada. In the negative and on the political question doctrine when it expanded the power of
assumption that petitioner is still President, whether judicial review of this court not only to settle actual
he is immune from criminal prosecution. controversies involving rights which are legally demandable
and enforceable but also to determine whether or not there
IV has been a grave abuse of discretion amounting to lack or
excess of jurisdiction on the part of any branch or
instrumentality of government.59 Heretofore, the judiciary
Whether the prosecution of petitioner Estrada should
has focused on the "thou shalt not's" of the Constitution
be enjoined on the ground of prejudicial publicity.
directed against the exercise of its jurisdiction. 60With the new
FINALS CONSTITUTION I ACJUCO 16

provision, however, courts are given a greater prerogative to 1900 issued by President McKinley, it is specifically provided
determine what it can do to prevent grave abuse of discretion "that no law shall be passed abridging the freedom of speech
amounting to lack or excess of jurisdiction on the part of any or of the press or of the rights of the people to peaceably
branch or instrumentality of government. Clearly, the new assemble and petition the Government for redress of
provision did not just grant the Court power of doing grievances." The guaranty was carried over in the Philippine
nothing. In sync and symmetry with this intent are other Bill, the Act of Congress of July 1, 1902 and the Jones Law,
provisions of the 1987 Constitution trimming the so called the Act of Congress of August 29, 1966.66
political thicket. Prominent of these provisions is section 18 of
Article VII which empowers this Court in limpid language to "x Thence on, the guaranty was set in stone in our 1935
x x review, in an appropriate proceeding filed by any citizen, Constitution,67 and the 197368 Constitution. These rights are
the sufficiency of the factual basis of the proclamation of now safely ensconced in section 4, Article III of the 1987
martial law or the suspension of the privilege of the writ (of Constitution, viz:
habeas corpus) or the extension thereof x x x."

"Sec. 4. No law shall be passed abridging the


Respondents rely on the case of Lawyers League for a Better freedom of speech, of expression, or of the press, or
Philippines and/or Oliver A. Lozano v. President Corazon the right of the people peaceably to assemble and
C. Aquino, et al.61 and related cases62 to support their thesis
petition the government for redress of grievances."
that since the cases at bar involve the legitimacy of the
government of respondent Arroyo, ergo, they present a
political question. A more cerebral reading of the cited cases The indispensability of the people's freedom of speech and of
will show that they are inapplicable. In the cited cases, we held assembly to democracy is now self-evident. The reasons are
that the government of former President Aquino was the well put by Emerson: first, freedom of expression is essential
result of a successful revolution by the sovereign people, as a means of assuring individual fulfillment; second, it is an
albeit a peaceful one. No less than the Freedom essential process for advancing knowledge and discovering
Constitution63 declared that the Aquino government was truth; third, it is essential to provide for participation in decision-
installed through a direct exercise of the power of the Filipino making by all members of society; and fourth, it is a method of
people "in defiance of the provisions of the 1973 achieving a more adaptable and hence, a more stable
Constitution, as amended." In is familiar learning that the community of maintaining the precarious balance between
legitimacy of a government sired by a successful revolution by healthy cleavage and necessary consensus."69 In this sense,
people power is beyond judicial scrutiny for that government freedom of speech and of assembly provides a framework
automatically orbits out of the constitutional loop. In checkered in which the "conflict necessary to the progress of a
contrast, the government of respondent Arroyo is not society can take place without destroying the
revolutionary in character. The oath that she took at the society."70In Hague v. Committee for Industrial
EDSA Shrine is the oath under the 1987 Constitution. 64 In her Organization,71 this function of free speech and assembly was
oath, she categorically swore to preserve and defend the echoed in the amicus curiae filed by the Bill of Rights
1987 Constitution. Indeed, she has stressed that she is Committee of the American Bar Association which emphasized
discharging the powers of the presidency under the authority that "the basis of the right of assembly is the substitution of the
of the 1987 Constitution.1âwphi1.nêt expression of opinion and belief by talk rather than force; and
this means talk for all and by all."72 In the relatively recent
case of Subayco v. Sandiganbayan,73 this Court similar
In fine, the legal distinction between EDSA People Power I stressed that "… it should be clear even to those with
EDSA People Power II is clear. EDSA I involves the exercise
intellectual deficits that when the sovereign people assemble
of the people power of revolution which overthrew the to petition for redress of grievances, all should listen. For in a
whole government. EDSA II is an exercise of people power
democracy, it is the people who count; those who are deaf
of freedom of speech and freedom of assembly to petition to their grievances are ciphers."
the government for redress of grievances which only
affected the office of the President. EDSA I is extra
constitutional and the legitimacy of the new government that Needless to state, the cases at bar pose legal and not political
resulted from it cannot be the subject of judicial review, questions. The principal issues for resolution require the
but EDSA II is intra constitutional and the resignation of the proper interpretation of certain provisions in the 1987
sitting President that it caused and the succession of the Vice Constitution, notably section 1 of Article II,74 and section 875 of
President as President are subject to judicial review. EDSA I Article VII, and the allocation of governmental powers under
presented a political question; EDSA II involves legal section 1176 of Article VII. The issues likewise call for a ruling
questions. A brief discourse on freedom of speech and of the on the scope of presidential immunity from suit. They also
freedom of assembly to petition the government for redress of involve the correct calibration of the right of petitioner against
grievance which are the cutting edge of EDSA People Power prejudicial publicity. As early as the 1803 case of Marbury v.
II is not inappropriate. Madison,77 the doctrine has been laid down that "it is
emphatically the province and duty of the judicial
department to say what the law is . . ." Thus, respondent's
Freedom of speech and the right of assembly are treasured by
in vocation of the doctrine of political question is but a foray in
Filipinos. Denial of these rights was one of the reasons of our the dark.
1898 revolution against Spain. Our national hero, Jose P.
Rizal, raised the clarion call for the recognition of freedom of
the press of the Filipinos and included it as among "the II
reforms sine quibus non."65 The Malolos Constitution, which
is the work of the revolutionary Congress in 1898, provided in Whether or not the petitioner
its Bill of Rights that Filipinos shall not be deprived (1) of the resigned as President
right to freely express his ideas or opinions, orally or in writing,
through the use of the press or other similar means; (2) of the
right of association for purposes of human life and which are We now slide to the second issue. None of the parties
not contrary to public means; and (3) of the right to send considered this issue as posing a political question. Indeed, it
petitions to the authorities, individually or collectively." These involves a legal question whose factual ingredient is
fundamental rights were preserved when the United determinable from the records of the case and by resort to
States acquired jurisdiction over the Philippines. In the judicial notice. Petitioner denies he resigned as President or
Instruction to the Second Philippine Commission of April 7, that he suffers from a permanent disability. Hence, he submits
FINALS CONSTITUTION I ACJUCO 17

that the office of the President was not vacant when As events approached January 20, we can have an
respondent Arroyo took her oath as President. authoritative window on the state of mind of the petitioner.
The window is provided in the "Final Days of Joseph Ejercito
Estrada," the diary of Executive Secretary Angara serialized in
The issue brings under the microscope the meaning of section
8, Article VII of the Constitution which provides: the Philippine Daily Inquirer.79 The Angara Diary reveals that
in the morning of January 19, petitioner's loyal advisers were
worried about the swelling of the crowd at EDSA, hence, they
"Sec. 8. In case of death, permanent disability, decided to create an ad hoc committee to handle it. Their worry
removal from office or resignation of the President, would worsen. At 1:20 p.m., petitioner pulled Secretary Angara
the Vice President shall become the President to into his small office at the presidential residence and
serve the unexpired term. In case of death, exclaimed: "Ed, seryoso na ito. Kumalas na si Angelo (Reyes)
permanent disability, removal from office, or (Ed, this is serious. Angelo has defected.)" 80 An hour later or
resignation of both the President and Vice President, at 2:30 p.m., the petitioner decided to call for a snap
the President of the Senate or, in case of his inability, presidential election and stressed he would not be a
the Speaker of the House of Representatives, shall candidate. The proposal for a snap election for president
then act as President until the President or Vice in May where he would not be a candidate is an indicium
President shall have been elected and qualified. that petitioner had intended to give up the presidency
even at that time. At 3:00 p.m., General Reyes joined the sea
x x x." of EDSA demonstrators demanding the resignation of the
petitioner and dramatically announced the AFP's withdrawal of
support from the petitioner and their pledge of support to
The issue then is whether the petitioner resigned as President respondent Arroyo. The seismic shift of support left petitioner
or should be considered resigned as of January 20, 2001 when weak as a president. According to Secretary Angara, he asked
respondent took her oath as the 14th President of the Public. Senator Pimentel to advise petitioner to consider the option
Resignation is not a high level legal abstraction. It is a factual of "dignified exit or resignation."81 Petitioner did not
question and its elements are beyond quibble: there must be disagree but listened intently. 82 The sky was falling fast on
an intent to resign and the intent must be coupled by acts the petitioner. At 9:30 p.m., Senator Pimentel repeated to the
of relinquishment.78 The validity of a resignation is not petitioner the urgency of making a graceful and dignified exit.
government by any formal requirement as to form. It can be He gave the proposal a sweetener by saying that petitioner
oral. It can be written. It can be express. It can be implied. As would be allowed to go abroad with enough funds to support
long as the resignation is clear, it must be given legal effect. him and his family.83 Significantly, the petitioner expressed
no objection to the suggestion for a graceful and dignified
In the cases at bar, the facts show that petitioner did not write exit but said he would never leave the country.84 At 10:00
any formal letter of resignation before he evacuated p.m., petitioner revealed to Secretary Angara, "Ed, Angie
Malacañang Palace in the afternoon of January 20, 2001 after (Reyes) guaranteed that I would have five days to a week in
the oath-taking of respondent Arroyo. Consequently, whether the palace."85 This is proof that petitioner had reconciled
or not petitioner resigned has to be determined from his act himself to the reality that he had to resign. His mind was
and omissions before, during and after January 20, 2001 or by already concerned with the five-day grace period he could
the totality of prior, contemporaneous and posterior facts stay in the palace. It was a matter of time.
and circumstantial evidence bearing a material relevance
on the issue. The pressure continued piling up. By 11:00 p.m., former
President Ramos called up Secretary Angara and requested,
Using this totality test, we hold that petitioner resigned as "Ed, magtulungan tayo para magkaroon tayo ng (let's
President. cooperate to ensure a) peaceful and orderly transfer of
power."86 There was no defiance to the request. Secretary
Angara readily agreed. Again, we note that at this stage, the
To appreciate the public pressure that led to the resignation of problem was already about a peaceful and orderly transfer
the petitioner, it is important to follow the succession of events of power. The resignation of the petitioner was implied.
after the exposẻ of Governor Singson. The Senate Blue
Ribbon Committee investigated. The more detailed revelations
of petitioner's alleged misgovernance in the Blue Ribbon The first negotiation for a peaceful and orderly transfer of
investigation spiked the hate against him. The Articles of power immediately started at 12:20 a.m. of January 20, that
Impeachment filed in the House of Representatives which fateful Saturday. The negotiation was limited to three (3)
initially was given a near cipher chance of succeeding points: (1) the transition period of five days after the petitioner's
snowballed. In express speed, it gained the signatures of 115 resignation; (2) the guarantee of the safety of the petitioner and
representatives or more than 1/3 of the House of his family, and (3) the agreement to open the second envelope
Representatives. Soon, petitioner's powerful political allies to vindicate the name of the petitioner. 87 Again, we note that
began deserting him. Respondent Arroyo quit as Secretary of the resignation of petitioner was not a disputed point. The
Social Welfare. Senate President Drilon and former Speaker petitioner cannot feign ignorance of this fact. According to
Villar defected with 47 representatives in tow. Then, his Secretary Angara, at 2:30 a.m., he briefed the petitioner on the
respected senior economic advisers resigned together with his three points and the following entry in the Angara Diary
Secretary of Trade and Industry. shows the reaction of the petitioner, viz:

As the political isolation of the petitioner worsened, the "x x x


people's call for his resignation intensified. The call reached a
new crescendo when the eleven (11) members of the I explain what happened during the first round of
impeachment tribunal refused to open the second envelope. It negotiations. The President immediately stresses
sent the people to paroxysms of outrage. Before the night of that he just wants the five-day period promised by
January 16 was over, the EDSA Shrine was swarming with Reyes, as well as to open the second envelope to
people crying for redress of their grievance. Their number grew clear his name.
exponentially. Rallies and demonstration quickly spread to the
countryside like a brush fire.
If the envelope is opened, on Monday, he says,
he will leave by Monday.
FINALS CONSTITUTION I ACJUCO 18

The President says. "Pagod na pagod na ako. presidency to Vice President Gloria Macapagal-
Ayoko na masyado nang masakit. Pagod na ako Arroyo.
sa red tape, bureaucracy, intriga. (I am very tired.
I don't want any more of this – it's too painful. I'm
'2. In return, President Estrada and his families are
tired of the red tape, the bureaucracy, the guaranteed security and safety of their person and
intrigue.) property throughout their natural lifetimes. Likewise,
President Estrada and his families are guarantee
I just want to clear my name, then I will go."88 freedom from persecution or retaliation from
government and the private sector throughout their
Again, this is high grade evidence that the petitioner has natural lifetimes.
resigned. The intent to resign is clear when he said "x x
x Ayoko na masyado nang masakit." "Ayoko na" are words This commitment shall be guaranteed by the Armed
of resignation. Forces of the Philippines (AFP) through the Chief of
Staff, as approved by the national military and police
The second round of negotiation resumed at 7:30 a.m. authorities – Vice President (Macapagal).
According to the Angara Diary, the following happened:
'3. Both parties shall endeavor to ensure that the
Senate sitting as an impeachment court will authorize
"Opposition's deal
the opening of the second envelope in the
impeachment trial as proof that the subject savings
7:30 a.m. – Rene arrives with Bert Romulo and (Ms. account does not belong to President Estrada.
Macapagal's spokesperson) Rene Corona. For this
round, I am accompanied by Dondon Bagatsing and
Macel. '4. During the five-day transition period between 20
January 2001 and 24 January 2001 (the 'Transition
Period"), the incoming Cabinet members shall
Rene pulls out a document titled "Negotiating receive an appropriate briefing from the outgoing
Points." It reads: Cabinet officials as part of the orientation program.

'1. The President shall sign a resignation document During the Transition Period, the AFP and the
within the day, 20 January 2001, that will be effective Philippine National Police (PNP) shall function Vice
on Wednesday, 24 January 2001, on which day the President (Macapagal) as national military and police
Vice President will assume the Presidency of the authorities.
Republic of the Philippines.
Both parties hereto agree that the AFP chief of staff
2. Beginning to day, 20 January 2001, the transition and PNP director general shall obtain all the
process for the assumption of the new administration necessary signatures as affixed to this agreement
shall commence, and persons designated by the and insure faithful implementation and observance
Vice President to various positions and offices of the thereof.
government shall start their orientation activities in
coordination with the incumbent officials concerned.
Vice President Gloria Macapagal-Arroyo shall issue
a public statement in the form and tenor provided for
3. The Armed Forces of the Philippines and the in "Annex A" heretofore attached to this
Philippine National Police shall function under the agreement."89
Vice President as national military and police
authority effective immediately.
The second round of negotiation cements the reading that the
petitioner has resigned. It will be noted that during this second
4. The Armed Forced of the Philippines, through its round of negotiation, the resignation of the petitioner was again
Chief of Staff, shall guarantee the security of the treated as a given fact. The only unsettled points at that time
President and his family as approved by the national were the measures to be undertaken by the parties during and
military and police authority (Vice President). after the transition period.

5. It is to be noted that the Senate will open the According to Secretary Angara, the draft agreement, which
second envelope in connection with the alleged was premised on the resignation of the petitioner was further
savings account of the President in the Equitable PCI refined. It was then, signed by their side and he was ready to
Bank in accordance with the rules of the Senate, fax it to General Reyes and Senator Pimentel to await the
pursuant to the request to the Senate President. signature of the United Opposition. However, the signing by
the party of the respondent Arroyo was aborted by her oath-
Our deal taking. The Angara diary narrates the fateful events, viz;90

We bring out, too, our discussion draft which reads: "xxx

The undersigned parties, for and in behalf of their 11:00 a.m. – Between General Reyes and myself,
respective principals, agree and undertake as there is a firm agreement on the five points to effect
follows: a peaceful transition. I can hear the general clearing
all these points with a group he is with. I hear voices
in the background.
'1. A transition will occur and take place on
Wednesday, 24 January 2001, at which time
President Joseph Ejercito Estrada will turn over the Agreement.
FINALS CONSTITUTION I ACJUCO 19

The agreement starts: 1. The President shall resign I direct Demaree Ravel to rush the original document
today, 20 January 2001, which resignation shall be to General Reyes for the signatures of the other side,
effective on 24 January 2001, on which day the Vice as it is important that the provisions on security, at
President will assume the presidency of the Republic least, should be respected.
of the Philippines.
I then advise the President that the Supreme Court
xxx has ruled that Chief Justice Davide will administer the
oath to Gloria at 12 noon.
The rest of the agreement follows:
The President is too stunned for words:
2. The transition process for the assumption of the
new administration shall commence on 20 January Final meal
2001, wherein persons designated by the Vice
President to various government positions shall start 12 noon – Gloria takes her oath as president of the
orientation activities with incumbent officials. Republic of the Philippines.

'3. The Armed Forces of the Philippines through its 12:20 p.m. – The PSG distributes firearms to some
Chief of Staff, shall guarantee the safety and security people inside the compound.
of the President and his families throughout their
natural lifetimes as approved by the national military
and police authority – Vice President. The president is having his final meal at the
presidential Residence with the few friends and
Cabinet members who have gathered.
'4. The AFP and the Philippine National Police (PNP)
shall function under the Vice President as national
military and police authorities. By this time, demonstrators have already broken
down the first line of defense at Mendiola. Only the
PSG is there to protect the Palace, since the police
'5. Both parties request the impeachment court to and military have already withdrawn their support for
open the second envelope in the impeachment trial,
the President.
the contents of which shall be offered as proof that
the subject savings account does not belong to the
President. 1 p.m. – The President's personal staff is rushing to
pack as many of the Estrada family's personal
possessions as they can.
The Vice President shall issue a public statement in
the form and tenor provided for in Annex "B"
heretofore attached to this agreement. During lunch, Ronnie Puno mentions that the
president needs to release a final statement before
leaving Malacañang.
11:20 a.m. – I am all set to fax General Reyes and
Nene Pimentel our agreement, signed by our side
and awaiting the signature of the United opposition. The statement reads: At twelve o'clock noon today,
Vice President Gloria Macapagal-Arroyo took her
oath as President of the Republic of the Philippines.
And then it happens. General Reyes calls me to say While along with many other legal minds of our
that the Supreme Court has decided that Gloria
country, I have strong and serious doubts about the
Macapagal-Arroyo is President and will be sworn in legality and constitutionality of her proclamation as
at 12 noon.
President, I do not wish to be a factor that will prevent
the restoration of unity and order in our civil society.
'Bakit hindi naman kayo nakahintay? Paano na ang
agreement (why couldn't you wait? What about the It is for this reason that I now leave Malacañang
agreement)?' I asked.
Palace, the seat of the presidency of this country, for
the sake of peace and in order to begin the healing
Reyes answered: 'Wala na, sir (it's over, sir).' process of our nation. I leave the Palace of our
people with gratitude for the opportunities given to
me for service to our people. I will not shirk from any
I ask him: Di yung transition period, moot and
academic na?' future challenges that may come ahead in the same
service of our country.

And General Reyes answers: ' Oo nga, I delete na


I call on all my supporters and followers to join me in
natin, sir (yes, we're deleting the part).'
the promotion of a constructive national spirit of
reconciliation and solidarity.
Contrary to subsequent reports, I do not react and
say that there was a double cross.
May the Almighty bless our country and our beloved
people.
But I immediately instruct Macel to delete the first
provision on resignation since this matter is already
moot and academic. Within moments, Macel erases MABUHAY!"'
the first provision and faxes the documents, which
have been signed by myself, Dondon and Macel, to It was curtain time for the petitioner.
Nene Pimentel and General Reyes.
FINALS CONSTITUTION I ACJUCO 20

In sum, we hold that the resignation of the petitioner cannot be "Sec. 12. No public officer shall be allowed to resign
doubted. It was confirmed by his leaving Malacañang. In the or retire pending an investigation, criminals or
press release containing his final statement, (1) he administrative, or pending a prosecution against him,
acknowledged the oath-taking of the respondent as President for any offense under this Act or under the provisions
of the Republic albeit with reservation about its legality; (2) he of the Revised Penal Code on bribery."
emphasized he was leaving the Palace, the seat of the
presidency, for the sake of peace and in order to begin the A reading of the legislative history of RA No. 3019 will hardly
healing process of our nation. He did not say he was leaving provide any comfort to the petitioner. RA No. 3019 originated
the Palace due to any kind inability and that he was going to form Senate Bill No. 293. The original draft of the bill, when it
re-assume the presidency as soon as the disability disappears: was submitted to the Senate, did not contain a provision similar
(3) he expressed his gratitude to the people for the opportunity to section 12 of the law as it now stands. However, in his
to serve them. Without doubt, he was referring to the past sponsorship speech, Senator Arturo Tolentino, the author of
opportunity given him to serve the people as President (4) he
the bill, "reserved to propose during the period of amendments
assured that he will not shirk from any future challenge that the inclusion of a provision to the effect that no public official
may come ahead in the same service of our country.
who is under prosecution for any act of graft or corruption, or
Petitioner's reference is to a future challenge after occupying is under administrative investigation, shall be allowed to
the office of the president which he has given up; and (5) he voluntarily resign or retire."92 During the period of
called on his supporters to join him in the promotion of a amendments, the following provision was inserted as section
constructive national spirit of reconciliation and solidarity. 15:
Certainly, the national spirit of reconciliation and solidarity
could not be attained if he did not give up the presidency. The
press release was petitioner's valedictory, his final act of "Sec. 15. Termination of office – No public official
farewell. His presidency is now in the part tense. shall be allowed to resign or retire pending an
investigation, criminal or administrative, or pending a
prosecution against him, for any offense under the
It is, however, urged that the petitioner did not resign but only Act or under the provisions of the Revised Penal
took a temporary leave dated January 20, 2001 of the Code on bribery.
petitioner sent to Senate President Pimentel and Speaker
Fuentebella is cited. Again, we refer to the said letter, viz:
The separation or cessation of a public official form
office shall not be a bar to his prosecution under this
"Sir. Act for an offense committed during his
incumbency."93
By virtue of the provisions of Section II, Article VII of
the Constitution, I am hereby transmitting this The bill was vetoed by then President Carlos P. Garcia who
declaration that I am unable to exercise the powers questioned the legality of the second paragraph of the
and duties of my office. By operation of law and the provision and insisted that the President's immunity should
Constitution, the Vice President shall be the Acting
extend after his tenure.
president.

Senate Bill No. 571, which was substantially similar Senate Bill
(Sgd.) Joseph Ejercito Estrada" No. 293, was thereafter passed. Section 15 above became
section 13 under the new bill, but the deliberations on this
To say the least, the above letter is wrapped in mystery.91 The particular provision mainly focused on the immunity of the
pleadings filed by the petitioner in the cases at bar did not President, which was one of the reasons for the veto of the
discuss, may even intimate, the circumstances that led to its original bill. There was hardly any debate on the prohibition
preparation. Neither did the counsel of the petitioner reveal to against the resignation or retirement of a public official with
the Court these circumstances during the oral argument. It pending criminal and administrative cases against him. Be that
strikes the Court as strange that the letter, despite its legal as it may, the intent of the law ought to be obvious. It is to
value, was never referred to by the petitioner during the week- prevent the act of resignation or retirement from being used by
long crisis. To be sure, there was not the slightest hint of its a public official as a protective shield to stop the investigation
existence when he issued his final press release. It was all too of a pending criminal or administrative case against him and to
easy for him to tell the Filipino people in his press release that prevent his prosecution under the Anti-Graft Law or
he was temporarily unable to govern and that he was leaving prosecution for bribery under the Revised Penal Code. To be
the reins of government to respondent Arroyo for the time sure, no person can be compelled to render service for that
bearing. Under any circumstance, however, the mysterious would be a violation of his constitutional right. 94 A public official
letter cannot negate the resignation of the petitioner. If it was has the right not to serve if he really wants to retire or resign.
prepared before the press release of the petitioner clearly as a Nevertheless, if at the time he resigns or retires, a public official
later act. If, however, it was prepared after the press released, is facing administrative or criminal investigation or prosecution,
still, it commands scant legal significance. Petitioner's such resignation or retirement will not cause the dismissal of
resignation from the presidency cannot be the subject of a the criminal or administrative proceedings against him. He
changing caprice nor of a whimsical will especially if the cannot use his resignation or retirement to avoid prosecution.
resignation is the result of his reputation by the people. There
is another reason why this Court cannot given any legal There is another reason why petitioner's contention should be
significance to petitioner's letter and this shall be discussed in rejected. In the cases at bar, the records show that when
issue number III of this Decision. petitioner resigned on January 20, 2001, the cases filed
against him before the Ombudsman were OMB Case Nos. 0-
After petitioner contended that as a matter of fact he did not 00-1629, 0-00-1755, 0-00-1756, 0-00-1757 and 0-00-1758.
resign, he also argues that he could not resign as a matter of While these cases have been filed, the respondent
law. He relies on section 12 of RA No. 3019, otherwise known Ombudsman refrained from conducting the preliminary
as the Anti-graft and Corrupt Practices Act, which allegedly investigation of the petitioner for the reason that as the sitting
prohibits his resignation, viz: President then, petitioner was immune from suit. Technically,
the said cases cannot be considered as pending for the
Ombudsman lacked jurisdiction to act on them. Section 12 of
FINALS CONSTITUTION I ACJUCO 21

RA No. 3019 cannot therefore be invoked by the petitioner for discharge the powers and duties of his office, the
it contemplates of cases whose investigation or prosecution do Congress shall decide the issue. For that purpose,
not suffer from any insuperable legal obstacle like the immunity the Congress shall convene, if it is not in session,
from suit of a sitting President. within forty-eight hours, in accordance with its rules
and without need of call.
Petitioner contends that the impeachment proceeding is an
administrative investigation that, under section 12 of RA 3019, If the Congress, within ten days after receipt of the
bars him from resigning. We hold otherwise. The exact nature last written declaration, or, if not in session, within
of an impeachment proceeding is debatable. But even twelve days after it is required to assemble,
assuming arguendo that it is an administrative proceeding, it determines by a two-thirds vote of both Houses,
can not be considered pending at the time petitioner resigned voting separately, that the President is unable to
because the process already broke down when a majority of discharge the powers and duties of his office, the
the senator-judges voted against the opening of the second Vice-President shall act as President; otherwise, the
envelope, the public and private prosecutors walked out, the President shall continue exercising the powers and
public prosecutors filed their Manifestation of Withdrawal of duties of his office."
Appearance, and the proceedings were postponed indefinitely.
There was, in effect, no impeachment case pending against
That is the law. Now, the operative facts:
petitioner when he resigned.

1. Petitioner, on January 20, 2001, sent the


III above letter claiming inability to the Senate
President and Speaker of the House;
Whether or not the petitioner Is only temporarily unable to 2. Unaware of the letter, respondent Arroyo
Act as President. took her oath of office as President on
January 20, 2001 at about 12:30 p.m.;
3. Despite receipt of the letter, the House of
We shall now tackle the contention of the petitioner that he is
merely temporarily unable to perform the powers and duties of Representatives passed on January 24,
2001 House Resolution No. 175;96
the presidency, and hence is a President on leave. As
aforestated, the inability claim is contained in the January 20,
2001 letter of petitioner sent on the same day to Senate On the same date, the House of the Representatives
President Pimentel and Speaker Fuentebella. passed House Resolution No. 17697 which states:

Petitioner postulates that respondent Arroyo as Vice President "RESOLUTION EXPRESSING THE SUPPORT OF
has no power to adjudge the inability of the petitioner to THE HOUSE OF REPRESENTATIVES TO THE
discharge the powers and duties of the presidency. His ASSUMPTION INTO OFFICE BY VICE PRESIDENT
significant submittal is that "Congress has the ultimate GLORIA MACAPAGAL-ARROYO AS PRESIDENT
authority under the Constitution to determine whether the OF THE REPUBLIC OF THE PHILIPPINES,
President is incapable of performing his functions in the EXTENDING ITS CONGRATULATIONS AND
manner provided for in section 11 of article VII."95 This EXPRESSING ITS SUPPORT FOR HER
contention is the centerpiece of petitioner's stance that he is ADMINISTRATION AS A PARTNER IN THE
a President on leave and respondent Arroyo is only an Acting ATTAINMENT OF THE NATION'S GOALS UNDER
President. THE CONSTITUTION

An examination of section 11, Article VII is in order. It provides: WHEREAS, as a consequence of the people's loss
of confidence on the ability of former President
"SEC. 11. Whenever the President transmits to the Joseph Ejercito Estrada to effectively govern, the
President of the Senate and the Speaker of the Armed Forces of the Philippines, the Philippine
House of Representatives his written declaration that National Police and majority of his cabinet had
he is unable to discharge the powers and duties of withdrawn support from him;
his office, and until he transmits to them a written
declaration to the contrary, such powers and duties WHEREAS, upon authority of an en banc resolution
shall be discharged by the Vice-President as Acting of the Supreme Court, Vice President Gloria
President. Macapagal-Arroyo was sworn in as President of the
Philippines on 20 January 2001 before Chief Justice
Hilario G. Davide, Jr.;
Whenever a majority of all the Members of the
Cabinet transmit to the President of the Senate and
to the Speaker of the House of Representatives their WHEREAS, immediately thereafter, members of the
written declaration that the President is unable to international community had extended their
discharge the powers and duties of his office, the recognition to Her Excellency, Gloria Macapagal-
Vice-President shall immediately assume the powers Arroyo as President of the Republic of the
and duties of the office as Acting President. Philippines;

Thereafter, when the President transmits to the WHEREAS, Her Excellency, President Gloria
President of the Senate and to the Speaker of the Macapagal-Arroyo has espoused a policy of national
House of Representatives his written declaration that healing and reconciliation with justice for the purpose
no inability exists, he shall reassume the powers and of national unity and development;
duties of his office. Meanwhile, should a majority of
all the Members of the Cabinet transmit within five WHEREAS, it is axiomatic that the obligations of the
days to the President of the Senate and to the government cannot be achieved if it is divided, thus
Speaker of the House of Representatives their by reason of the constitutional duty of the House of
written declaration that the President is unable to Representatives as an institution and that of the
FINALS CONSTITUTION I ACJUCO 22

individual members thereof of fealty to the supreme WHEREAS, Senator Teofisto T. Guingona Jr., is a
will of the people, the House of Representatives must public servant endowed with integrity, competence
ensure to the people a stable, continuing government and courage; who has served the Filipino people with
and therefore must remove all obstacles to the dedicated responsibility and patriotism;
attainment thereof;
WHEREAS, Senator Teofisto T. Guingona, Jr.
WHEREAS, it is a concomitant duty of the House of possesses sterling qualities of true statesmanship,
Representatives to exert all efforts to unify the nation, having served the government in various capacities,
to eliminate fractious tension, to heal social and among others, as Delegate to the Constitutional
political wounds, and to be an instrument of national Convention, Chairman of the Commission on Audit,
reconciliation and solidarity as it is a direct Executive Secretary, Secretary of Justice, Senator of
representative of the various segments of the whole the Philippines – qualities which merit his nomination
nation; to the position of Vice President of the Republic:
Now, therefore, be it
WHEREAS, without surrending its independence, it
is vital for the attainment of all the foregoing, for the Resolved as it is hereby resolved by the House of
House of Representatives to extend its support and Representatives, That the House of Representatives
collaboration to the administration of Her Excellency, confirms the nomination of Senator Teofisto T.
President Gloria Macapagal-Arroyo, and to be a Guingona, Jr. as the Vice President of the Republic
constructive partner in nation-building, the national of the Philippines.
interest demanding no less: Now, therefore, be it
Adopted,
Resolved by the House of Representatives, To
express its support to the assumption into office by (Sgd.) FELICIANO BELMONTE JR.
Vice President Gloria Macapagal-Arroyo as
Speaker
President of the Republic of the Philippines, to
extend its congratulations and to express its support
for her administration as a partner in the attainment This Resolution was adopted by the House of
of the Nation's goals under the Constitution. Representatives on February 7, 2001.

Adopted, (Sgd.) ROBERTO P. NAZARENO


Secretary General"
(Sgd.) FELICIANO BELMONTE JR.
Speaker (4) Also, despite receipt of petitioner's letter claiming
inability, some twelve (12) members of the Senate
signed the following:
This Resolution was adopted by the House of
Representatives on January 24, 2001.
"RESOLUTION
(Sgd.) ROBERTO P. NAZARENO
Secretary General" WHEREAS, the recent transition in government
offers the nation an opportunity for meaningful
change and challenge;
On February 7, 2001, the House of the Representatives
passed House Resolution No. 17898 which states:
WHEREAS, to attain desired changes and overcome
awesome challenges the nation needs unity of
"RESOLUTION CONFIRMING PRESIDENT purpose and resolve cohesive resolute (sic) will;
GLORIA MACAPAGAL-ARROYO'S NOMINATION
OF SENATOR TEOFISTO T. GUINGONA, JR. AS
VICE PRESIDENT OF THE REPUBLIC OF THE WHEREAS, the Senate of the Philippines has been
PHILIPPINES the forum for vital legislative measures in unity
despite diversities in perspectives;
WHEREAS, there is a vacancy in the Office of the
Vice President due to the assumption to the WHEREFORE, we recognize and express support to
Presidency of Vice President Gloria Macapagal- the new government of President Gloria Macapagal-
Arroyo; Arroyo and resolve to discharge and overcome the
nation's challenges." 99
WHEREAS, pursuant to Section 9, Article VII of the
Constitution, the President in the event of such On February 7, the Senate also passed Senate
vacancy shall nominate a Vice President from among Resolution No. 82100 which states:
the members of the Senate and the House of
Representatives who shall assume office upon "RESOLUTION CONFIRMING PRESIDENT
confirmation by a majority vote of all members of both GLORIA MACAPAGAL ARROYO'S NOMINATION
Houses voting separately; OF SEM. TEOFISTO T. GUINGONA, JR. AS VICE
PRESIDENT OF THE REPUBLIC OF THE
WHEREAS, Her Excellency, President Gloria PHILIPPINES
Macapagal-Arroyo has nominated Senate Minority
Leader Teofisto T. Guingona Jr., to the position of WHEREAS, there is vacancy in the Office of the Vice
Vice President of the Republic of the Philippines; President due to the assumption to the Presidency of
Vice President Gloria Macapagal-Arroyo;
FINALS CONSTITUTION I ACJUCO 23

WHEREAS, pursuant to Section 9 Article VII of the Resolved, finally. That all parties concerned be
Constitution, the President in the event of such furnished copies of this Resolution.
vacancy shall nominate a Vice President from among
the members of the Senate and the House of
Adopted,
Representatives who shall assume office upon
confirmation by a majority vote of all members of both
Houses voting separately; (Sgd.) AQUILINO Q. PIMENTEL, JR.
President of the Senate
WHEREAS, Her Excellency, President Gloria
Macapagal-Arroyo has nominated Senate Minority This Resolution was adopted by the Senate on
Leader Teofisto T. Guingona, Jr. to the position of February 7, 2001.
Vice President of the Republic of the Philippines;
(Sgd.) LUTGARDO B. BARBO
WHEREAS, Sen. Teofisto T. Guingona, Jr. is a public Secretary of the Senate"
servant endowed with integrity, competence and
courage; who has served the Filipino people with (5) On February 8, the Senate also passed Resolution No.
dedicated responsibility and patriotism; 84 "certifying to the existence of vacancy in the Senate and
calling on the COMELEC to fill up such vacancy through
WHEREAS, Sen. Teofisto T. Guingona, Jr. election to be held simultaneously with the regular election on
possesses sterling qualities of true statemanship, May 14, 2001 and the Senatorial candidate garnering the
having served the government in various capacities, thirteenth (13th) highest number of votes shall serve only for
among others, as Delegate to the Constitutional the unexpired term of Senator Teofisto T. Guingona, Jr.'
Convention, Chairman of the Commission on Audit,
Executive Secretary, Secretary of Justice, Senator of (6) Both houses of Congress started sending bills to be
the land - which qualities merit his nomination to the signed into law by respondent Arroyo as President.
position of Vice President of the Republic: Now,
therefore, be it
(7) Despite the lapse of time and still without any functioning
Cabinet, without any recognition from any sector of
Resolved, as it is hereby resolved, That the Senate government, and without any support from the Armed Forces
confirm the nomination of Sen. Teofisto T. Guingona, of the Philippines and the Philippine National Police, the
Jr. as Vice President of the Republic of the petitioner continues to claim that his inability to govern is only
Philippines. momentary.

Adopted, What leaps to the eye from these irrefutable facts is that
both houses of Congress have recognized respondent
(Sgd.) AQUILINO Q. PIMENTEL JR. Arroyo as the President. Implicitly clear in that recognition
President of the Senate is the premise that the inability of petitioner Estrada. Is no
longer temporary. Congress has clearly rejected
petitioner's claim of inability.
This Resolution was adopted by the Senate on
February 7, 2001.
The question is whether this Court has jurisdiction to
review the claim of temporary inability of petitioner
(Sgd.) LUTGARDO B. BARBO Estrada and thereafter revise the decision of both Houses
Secretary of the Senate" of Congress recognizing respondent Arroyo as president of
the Philippines. Following Tañada v. Cuenco,102 we hold that
On the same date, February 7, the Senate likewise this Court cannot exercise its judicial power or this is an issue
passed Senate Resolution No. 83101 which states: "in regard to which full discretionary authority has been
delegated to the Legislative xxx branch of the government." Or
"RESOLUTION RECOGNIZING THAT THE to use the language in Baker vs. Carr,103 there is a "textually
IMPEACHMENT COURT IS FUNCTUS OFFICIO demonstrable or a lack of judicially discoverable and
manageable standards for resolving it." Clearly, the Court
cannot pass upon petitioner's claim of inability to discharge the
Resolved, as it is hereby resolved. That the Senate power and duties of the presidency. The question is political
recognize that the Impeachment Court is functus in nature and addressed solely to Congress by
officioand has been terminated. constitutional fiat. It is a political issue, which cannot be
decided by this Court without transgressing the principle of
Resolved, further, That the Journals of the separation of powers.
Impeachment Court on Monday, January 15,
Tuesday, January 16 and Wednesday, January 17, In fine, even if the petitioner can prove that he did not
2001 be considered approved. resign, still, he cannot successfully claim that he is a
President on leave on the ground that he is merely unable
Resolved, further, That the records of the to govern temporarily. That claim has been laid to rest by
Impeachment Court including the "second envelope" Congress and the decision that respondent Arroyo is the
be transferred to the Archives of the Senate for de jure, president made by a co-equal branch of
proper safekeeping and preservation in accordance government cannot be reviewed by this Court.
with the Rules of the Senate. Disposition and
retrieval thereof shall be made only upon written IV
approval of the Senate president.
Whether or not the petitioner enjoys immunity from suit.
FINALS CONSTITUTION I ACJUCO 24

Assuming he enjoys immunity, the extent of the immunity provided the question of his authority was one over
which two men, reasonably qualified for that position,
Petitioner Estrada makes two submissions: first, the cases might honestly differ; but he s not protected if the lack
of authority to act is so plain that two such men could
filed against him before the respondent Ombudsman should
be prohibited because he has not been convicted in the not honestly differ over its determination. In such
impeachment proceedings against him; and second, he case, be acts, not as Governor-General but as a
enjoys immunity from all kinds of suit, whether criminal or private individual, and as such must answer for the
civil. consequences of his act."

Before resolving petitioner's contentions, a revisit of our legal Mr. Justice Johnson underscored the consequences if the
history executive immunity will be most enlightening. The Chief Executive was not granted immunity from suit, viz"xxx.
doctrine of executive immunity in this jurisdiction emerged as Action upon important matters of state delayed; the time and
a case law. In the 1910 case of Forbes, etc. vs. Chuoco substance of the chief executive spent in wrangling litigation;
Tiaco and Crosfield,104 the respondent Tiaco, a Chinese disrespect engendered for the person of one of the highest
citizen, sued petitioner W. Cameron Forbes, Governor- officials of the state and for the office he occupies; a tendency
General of the Philippine Islands. J.E. Harding and C.R. to unrest and disorder resulting in a way, in distrust as to the
integrity of government itself."105
Trowbridge, Chief of Police and Chief of the Secret Service of
the City of Manila, respectively, for damages for allegedly
conspiring to deport him to China. In granting a writ of Our 1935 Constitution took effect but it did not contain any
prohibition, this Court, speaking thru Mr. Justice Johnson, held: specific provision on executive immunity. Then came the
tumult of the martial law years under the late President
" The principle of nonliability, as herein enunciated, Ferdinand E. Marcos and the 1973 Constitution was born. In
1981, it was amended and one of the amendments involved
does not mean that the judiciary has no authority to
touch the acts of the Governor-General; that he may, executive immunity. Section 17, Article VII stated:
under cover of his office, do what he will, unimpeded
and unrestrained. Such a construction would mean "The President shall be immune from suit during his
that tyranny, under the guise of the execution of the tenure. Thereafter, no suit whatsoever shall lie for
law, could walk defiantly abroad, destroying rights of official acts done by him or by others pursuant to his
person and of property, wholly free from interference specific orders during his tenure.
of courts or legislatures. This does not mean, either
that a person injured by the executive authority by an The immunities herein provided shall apply to the
act unjustifiable under the law has n remedy, but incumbent President referred to in Article XVII of this
must submit in silence. On the contrary, it means, Constitution.
simply, that the governors-general, like the judges if
the courts and the members of the Legislature, may
not be personally mulcted in civil damages for the In his second Vicente G. Sinco professional Chair lecture
consequences of an act executed in the performance entitled, "Presidential Immunity and All The King's Men: The
of his official duties. The judiciary has full power to, Law of Privilege As a Defense To Actions For
and will, when the mater is properly presented to it Damages,"106 petitioner's learned counsel, former Dean of the
and the occasion justly warrants it, declare an act of UP College of Law, Atty. Pacificao Agabin, brightened the
the Governor-General illegal and void and place as modifications effected by this constitutional amendment on the
nearly as possible in status quo any person who has existing law on executive privilege. To quote his disquisition:
been deprived his liberty or his property by such act.
This remedy is assured to every person, however "In the Philippines, though, we sought to do the
humble or of whatever country, when his personal or Americans one better by enlarging and fortifying the
property rights have been invaded, even by the absolute immunity concept. First, we extended it to
highest authority of the state. The thing which the shield the President not only form civil claims but also
judiciary can not do is mulct the Governor-General from criminal cases and other claims. Second, we
personally in damages which result from the enlarged its scope so that it would cover even acts of
performance of his official duty, any more than it can the President outside the scope of official duties. And
a member of the Philippine Commission of the third, we broadened its coverage so as to include not
Philippine Assembly. Public policy forbids it. only the President but also other persons, be they
government officials or private individuals, who acted
Neither does this principle of nonliability mean that upon orders of the President. It can be said that at
the chief executive may not be personally sued at all that point most of us were suffering from AIDS (or
in relation to acts which he claims to perform as such absolute immunity defense syndrome)."
official. On the contrary, it clearly appears from the
discussion heretofore had, particularly that portion The Opposition in the then Batasan Pambansa sought the
which touched the liability of judges and drew an repeal of this Marcosian concept of executive immunity in the
analogy between such liability and that of the 1973 Constitution. The move was led by them Member of
Governor-General, that the latter is liable when he Parliament, now Secretary of Finance, Alberto Romulo, who
acts in a case so plainly outside of his power and argued that the after incumbency immunity granted to
authority that he can not be said to have exercised President Marcos violated the principle that a public office is a
discretion in determining whether or not he had the public trust. He denounced the immunity as a return to the
right to act. What is held here is that he will be anachronism "the king can do no wrong."107 The effort failed.
protected from personal liability for damages not only
when he acts within his authority, but also when he is
without authority, provided he actually used The 1973 Constitution ceased to exist when President Marcos
discretion and judgement, that is, the judicial faculty, was ousted from office by the People Power revolution in 1986.
in determining whether he had authority to act or not. When the 1987 Constitution was crafted, its framers did not
In other words, in determining the question of his reenact the executive immunity provision of the 1973
authority. If he decide wrongly, he is still protected
FINALS CONSTITUTION I ACJUCO 25

Constitution. The following explanation was given by delegate resignation would render the case moot and
J. Bernas vis:108 academic. However, as the provision says, the
criminal and civil aspects of it may continue in the
ordinary courts."
"Mr. Suarez. Thank you.

This is in accord with our ruling In Re: Saturnino


The last question is with reference to the
Committee's omitting in the draft proposal the Bermudez111 that 'incumbent Presidents are immune from suit
or from being brought to court during the period of their
immunity provision for the President. I agree with
Commissioner Nolledo that the Committee did very incumbency and tenure" but not beyond. Considering the
well in striking out second sentence, at the very least, peculiar circumstance that the impeachment process against
of the original provision on immunity from suit under the petitioner has been aborted and thereafter he lost the
the 1973 Constitution. But would the Committee presidency, petitioner Estrada cannot demand as a condition
members not agree to a restoration of at least the first sine qua non to his criminal prosecution before the
sentence that the President shall be immune from Ombudsman that he be convicted in the impeachment
suit during his tenure, considering that if we do not proceedings. His reliance on the case of Lecaroz vs.
provide him that kind of an immunity, he might be Sandiganbayan112 and related cases113 are inapropos for they
have a different factual milieu.
spending all his time facing litigation's, as the
President-in-exile in Hawaii is now facing litigation's
almost daily? We now come to the scope of immunity that can be claimed by
petitioner as a non-sitting President. The cases filed against
petitioner Estrada are criminal in character. They involve
Fr. Bernas. The reason for the omission is that we
consider it understood in present jurisprudence that plunder, bribery and graft and corruption. By no stretch of the
imagination can these crimes, especially plunder which carries
during his tenure he is immune from suit.
the death penalty, be covered by the alleged mantle of
immunity of a non-sitting president. Petitioner cannot cite any
Mr. Suarez. So there is no need to express it here. decision of this Court licensing the President to commit criminal
acts and wrapping him with post-tenure immunity from liability.
Fr. Bernas. There is no need. It was that way before. It will be anomalous to hold that immunity is an inoculation from
The only innovation made by the 1973 Constitution liability for unlawful acts and conditions. The rule is that
was to make that explicit and to add other things. unlawful acts of public officials are not acts of the State and the
officer who acts illegally is not acting as such but stands in the
same footing as any trespasser.114
Mr. Suarez. On that understanding, I will not press for
any more query, Madam President.
Indeed, critical reading of current literature on executive
immunity will reveal a judicial disinclination to expand the
I think the Commissioner for the clarifications." privilege especially when it impedes the search for truth or
impairs the vindication of a right. In the 1974 case of US v.
We shall now rule on the contentions of petitioner in the light Nixon,115 US President Richard Nixon, a sitting President, was
of this history. We reject his argument that he cannot be subpoenaed to produce certain recordings and documents
prosecuted for the reason that he must first be convicted in the relating to his conversations with aids and advisers. Seven
impeachment proceedings. The impeachment trial of petitioner advisers of President Nixon's associates were facing charges
Estrada was aborted by the walkout of the prosecutors and by of conspiracy to obstruct Justice and other offenses, which
the events that led to his loss of the presidency. Indeed, on were committed in a burglary of the Democratic National
February 7, 2001, the Senate passed Senate Resolution No. Headquarters in Washington's Watergate Hotel during the 972
83 "Recognizing that the Impeachment Court is Functus presidential campaign. President Nixon himself was named an
Officio."109 Since, the Impeachment Court is now functus unindicted co-conspirator. President Nixon moved to quash the
officio, it is untenable for petitioner to demand that he should subpoena on the ground, among others, that the President was
first be impeached and then convicted before he can be not subject to judicial process and that he should first be
prosecuted. The plea if granted, would put a perpetual bar impeached and removed from office before he could be made
against his prosecution. Such a submission has nothing to amenable to judicial proceedings. The claim was rejected by
commend itself for it will place him in a better situation than a the US Supreme Court. It concluded that "when the ground for
non-sitting President who has not been subjected to asserting privilege as to subpoenaed materials sought for use
impeachment proceedings and yet can be the object of a in a criminal trial is based only on the generalized interest in
criminal prosecution. To be sure, the debates in the confidentiality, it cannot prevail over the fundamental demands
Constitutional Commission make it clear that when of due process of law in the fair administration of criminal
impeachment proceedings have become moot due to the justice." In the 1982 case of Nixon v. Fitzgerald, 116 the US
resignation of the President, the proper criminal and civil cases Supreme Court further held that the immunity of the president
may already be filed against him, viz:110 from civil damages covers only "official acts." Recently, the US
Supreme Court had the occasion to reiterate this doctrine in
the case of Clinton v. Jones117 where it held that the US
"xxx President's immunity from suits for money damages arising out
of their official acts is inapplicable to unofficial conduct.
Mr. Aquino. On another point, if an impeachment
proceeding has been filed against the President, for There are more reasons not to be sympathetic to appeals to
example, and the President resigns before stretch the scope of executive immunity in our jurisdiction. One
judgement of conviction has been rendered by the of the great themes of the 1987 Constitution is that a public
impeachment court or by the body, how does it affect office is a public trust.118 It declared as a state policy that "the
the impeachment proceeding? Will it be necessarily State shall maintain honesty and integrity in the public service
dropped? and take positive and effective measures against graft and
corruptio."119 it ordained that "public officers and employees
Mr. Romulo. If we decide the purpose of must at all times be accountable to the people, serve them with
impeachment to remove one from office, then his utmost responsibility, integrity, loyalty, and efficiency act with
FINALS CONSTITUTION I ACJUCO 26

patriotism and justice, and lead modest lives." 120 It set the rule seal the minds of members of the bench from pre-
that 'the right of the State to recover properties unlawfully trial and other off-court publicity of sensational
acquired by public officials or employees, from them or from criminal cases. The state of the art of our
their nominees or transferees, shall not be barred by communication system brings news as they happen
prescription, latches or estoppel."121 It maintained the straight to our breakfast tables and right to our
Sandiganbayan as an anti-graft court.122 It created the office of bedrooms. These news form part of our everyday
the Ombudsman and endowed it with enormous powers, menu of the facts and fictions of life. For another, our
among which is to "investigate on its own, or on complaint by idea of a fair and impartial judge is not that of a hermit
any person, any act or omission of any public official, who is out of touch with the world. We have not
employee, office or agency, when such act or omission installed the jury system whose members are overly
appears to be illegal, unjust improper or inefficient." 123 The protected from publicity lest they lose there
Office of the Ombudsman was also given fiscal impartially. xxx xxx xxx. Our judges are learned in the
autonomy.124 These constitutional policies will be devalued if law and trained to disregard off-court evidence and
we sustain petitioner's claim that a non-sitting president enjoys on-camera performances of parties to litigation. Their
immunity from suit for criminal acts committed during his mere exposure to publications and publicity stunts
incumbency. does not per se fatally infect their impartiality.

V At best, appellant can only conjure possibility of


prejudice on the part of the trial judge due to the
Whether or not the prosecution of petitioner barrage of publicity that characterized the
investigation and trial of the case. In Martelino, et al.
v. Alejandro, et al., we rejected this standard of
Estrada should be enjoined due to prejudicial publicity possibility of prejudice and adopted the test of actual
prejudice as we ruled that to warrant a finding of
Petitioner also contends that the respondent Ombudsman prejudicial publicity, there must be allegation and
should be stopped from conducting the investigation of the proof that the judges have been unduly influenced,
cases filed against him due to the barrage of prejudicial not simply that they might be, by the barrage of
publicity on his guilt. He submits that the respondent publicity. In the case at a bar, the records do not
Ombudsman has developed bias and is all set file the criminal show that the trial judge developed actual bias
cases violation of his right to due process. against appellants as a consequence of the
extensive media coverage of the pre-trial and trial of
his case. The totality of circumstances of the case
There are two (2) principal legal and philosophical schools of does not prove that the trial judge acquired a fixed
thought on how to deal with the rain of unrestrained publicity opinion as a result of prejudicial publicity, which is
during the investigation and trial of high profile cases. 125 The incapable of change even by evidence presented
British approach the problem with the presumption that during the trial. Appellant has the burden to prove this
publicity will prejudice a jury. Thus, English courts readily stay actual bias and he has not discharged the burden.'
and stop criminal trials when the right of an accused to fair trial
suffers a threat.126 The American approach is different. US
courts assume a skeptical approach about the potential effect We expounded further on this doctrine in the subsequent case
of pervasive publicity on the right of an accused to a fair trial. of Webb vs. Hon. Raul de Leon, etc.130 and its companion
They have developed different strains of tests to resolve this cases, viz:
issue, i.e., substantial; probability of irreparable harm, strong
likelihood, clear and present danger, etc. "Again petitioners raise the effect of prejudicial
publicity on their right to due process while
This is not the first time the issue of trial by publicity has been undergoing preliminary investigation. We find no
raised in this Court to stop the trials or annul convictions in high procedural impediment to its early invocation
profile criminal cases.127 In People vs. Teehankee, Jr.,128 later considering the substantial risk to their liberty while
reiterated in the case of Larranaga vs. court of Appeals, et undergoing a preliminary investigation.
al.,129 we laid down the doctrine that:
xxx
"We cannot sustain appellant's claim that he was
denied the right to impartial trial due to prejudicial The democratic settings, media coverage of trials of
publicity. It is true that the print and broadcast media sensational cases cannot be avoided and oftentimes,
gave the case at bar pervasive publicity, just like all its excessiveness has been aggravated by kinetic
high profile and high stake criminal trials. Then and developments in the telecommunications industry.
now, we rule that the right of an accused to a fair trial For sure, few cases can match the high volume and
is not incompatible to a free press. To be sure, high velocity of publicity that attended the preliminary
responsible reporting enhances accused's right to a investigation of the case at bar. Our daily diet of facts
fair trial for, as well pointed out, a responsible press and fiction about the case continues unabated even
has always been regarded as the criminal field xxx. today. Commentators still bombard the public with
The press does not simply publish information about views not too many of which are sober and sublime.
trials but guards against the miscarriage of justice by Indeed, even the principal actors in the case – the
subjecting the police, prosecutors, and judicial NBI, the respondents, their lawyers and their
processes to extensive public scrutiny and criticism. sympathizers have participated in this media blitz.
The possibility of media abuses and their threat to a
Pervasive publicity is not per se prejudicial to the fair trial notwithstanding, criminal trials cannot be
right of an accused to fair trial. The mere fact that the completely closed to the press and public. In the
trial of appellant was given a day-to-day, gavel-to- seminal case of Richmond Newspapers, Inc. v.
gavel coverage does not by itself prove that the Virginia, it was
publicity so permeated the mind of the trial judge and
impaired his impartiality. For one, it is impossible to xxx
FINALS CONSTITUTION I ACJUCO 27

a. The historical evidence of the evolution of Be that as it may, we recognize that pervasive and
the criminal trial in Anglo-American justice prejudicial publicity under certain circumstances can
demonstrates conclusively that at the time deprive an accused of his due process right to fair
this Nation's organic laws were adopted, trial. Thus, in Martelino, et al. vs. Alejandro, et al., we
criminal trials both here and in England had held that to warrant a finding of prejudicial publicity
long been presumptively open, thus giving there must be allegation and proof that the judges
assurance that the proceedings were have been unduly influenced, not simply that they
conducted fairly to all concerned and might be, by the barrage of publicity. In the case at
discouraging perjury, the misconduct of bar, we find nothing in the records that will prove that
participants, or decisions based on secret the tone and content of the publicity that attended the
bias or partiality. In addition, the significant investigation of petitioners fatally infected the
community therapeutic value of public trials fairness and impartiality of the DOJ Panel.
was recognized when a shocking crime Petitioners cannot just rely on the subliminal effects
occurs a community reaction of outrage of publicity on the sense of fairness of the DOJ Panel,
and public protest often follows, and for these are basically unbeknown and beyond
thereafter the open processes of justice knowing. To be sure, the DOJ Panel is composed of
serve an important prophylactic purpose, an Assistant Chief State Prosecutor and Senior State
providing an outlet for community concern, Prosecutors. Their long experience in criminal
hostility and emotion. To work effectively, it investigation is a factor to consider in determining
is important that society's criminal process whether they can easily be blinded by the klieg lights
satisfy the appearance of justice,' Offutt v. of publicity. Indeed, their 26-page Resolution carries
United States, 348 US 11, 14, 99 L ED 11, no indubitable indicia of bias for it does not appear
75 S Ct 11, which can best be provided by that they considered any extra-record evidence
allowing people to observe such process. except evidence properly adduced by the parties.
From this unbroken, uncontradicted The length of time the investigation was conducted
history, supported by reasons as valid despite its summary nature and the generosity with
today as in centuries past, it must be which they accommodated the discovery motions of
concluded that a presumption of openness petitioners speak well of their fairness. At no
inheres in the very nature of a criminal trial instance, we note, did petitioners seek the
under this Nation's system of justice, Cf., disqualification of any member of the DOJ Panel on
e,g., Levine v. United States, 362 US 610, the ground of bias resulting from their bombardment
4 L Ed 2d 989, 80 S Ct 1038. of prejudicial publicity." (emphasis supplied)
b. The freedoms of speech. Press and
assembly, expressly guaranteed by the Applying the above ruling, we hold that there is not enough
First Amendment, share a common core
evidence to warrant this Court to enjoin the preliminary
purpose of assuring freedom of investigation of the petitioner by the respondent
communication on matters relating to the Ombudsman. Petitioner needs to offer more than hostile
functioning of government. In guaranteeing headlines to discharge his burden of proof. 131 He needs to
freedom such as those of speech and show more weighty social science evidence to successfully
press, the First Amendment can be read as prove the impaired capacity of a judge to render a bias-free
protecting the right of everyone to attend decision. Well to note, the cases against the petitioner are still
trials so as give meaning to those explicit undergoing preliminary investigation by a special panel of
guarantees; the First Amendment right to prosecutors in the office of the respondent Ombudsman. No
receive information and ideas means, in
allegation whatsoever has been made by the petitioner that the
the context of trials, that the guarantees of minds of the members of this special panel have already been
speech and press, standing alone, prohibit
infected by bias because of the pervasive prejudicial publicity
government from summarily closing against him. Indeed, the special panel has yet to come out with
courtroom doors which had long been open its findings and the Court cannot second guess whether its
to the public at the time the First recommendation will be unfavorable to the petitioner.
Amendment was adopted. Moreover, the
right of assembly is also relevant, having
been regarded not only as an independent The records show that petitioner has instead charged
right but also as a catalyst to augment the respondent Ombudsman himself with bias. To quote
free exercise of the other First Amendment petitioner's submission, the respondent Ombudsman "has
rights with which the draftsmen deliberately been influenced by the barrage of slanted news reports, and
linked it. A trial courtroom is a public place he has buckled to the threats and pressures directed at him by
where the people generally and the mobs."132 News reports have also been quoted to establish
representatives of the media have a right to that the respondent Ombudsman has already prejudged the
be present, and where their presence cases of the petitioner133 and it is postulated that the
historically has been thought to enhance prosecutors investigating the petitioner will be influenced by
the integrity and quality of what takes place. this bias of their superior.
c. Even though the Constitution contains no
provision which be its terms guarantees to Again, we hold that the evidence proffered by the petitioner
the public the right to attend criminal trials, is insubstantial. The accuracy of the news reports referred to
various fundamental rights, not expressly by the petitioner cannot be the subject of judicial notice by this
guaranteed, have been recognized as Court especially in light of the denials of the respondent
indispensable to the enjoyment of Ombudsman as to his alleged prejudice and the presumption
enumerated rights. The right to attend of good faith and regularity in the performance of official duty
criminal trial is implicit in the guarantees of to which he is entitled. Nor can we adopt the theory of
the First Amendment: without the freedom derivative prejudice of petitioner, i.e., that the prejudice of
to attend such trials, which people have respondent Ombudsman flows to his subordinates. In
exercised for centuries, important aspects truth, our Revised Rules of Criminal Procedure, give
of freedom of speech and of the press be investigation prosecutors the independence to make their own
eviscerated. findings and recommendations albeit they are reviewable by
FINALS CONSTITUTION I ACJUCO 28

their superiors.134 They can be reversed but they can not be


compelled cases which they believe deserve dismissal. In
other words, investigating prosecutors should not be treated
like unthinking slot machines. Moreover, if the respondent
Ombudsman resolves to file the cases against the petitioner
and the latter believes that the findings of probable cause
against him is the result of bias, he still has the remedy of
assailing it before the proper court.

VI.

Epilogue

A word of caution to the "hooting throng." The cases against


the petitioner will now acquire a different dimension and then
move to a new stage - - - the Office of the Ombudsman.
Predictably, the call from the majority for instant justice will hit
a higher decibel while the gnashing of teeth of the minority will
be more threatening. It is the sacred duty of the respondent
Ombudsman to balance the right of the State to prosecute the
guilty and the right of an accused to a fair investigation and trial
which has been categorized as the "most fundamental of all
freedoms."135To be sure, the duty of a prosecutor is more to do
justice and less to prosecute. His is the obligation to insure that
the preliminary investigation of the petitioner shall have a
circus-free atmosphere. He has to provide the restraint against
what Lord Bryce calls "the impatient vehemence of the
majority." Rights in a democracy are not decided by the mob
whose judgment is dictated by rage and not by reason. Nor are
rights necessarily resolved by the power of number for in a
democracy, the dogmatism of the majority is not and should
never be the definition of the rule of law. If democracy has
proved to be the best form of government, it is because it has
respected the right of the minority to convince the majority that
it is wrong. Tolerance of multiformity of thoughts, however
offensive they may be, is the key to man's progress from the
cave to civilization. Let us not throw away that key just to
pander to some people's prejudice.

IN VIEW WHEREOF, the petitions of Joseph Ejercito Estrada


challenging the respondent Gloria Macapagal-Arroyo as the de
jure 14th President of the Republic are DISMISSED.

SO ORDERED.
FINALS CONSTITUTION I ACJUCO 29

G.R. No. 83896 February 22, 1991 Sec. 3. In order to fully protect the interest of the
government in government-owned or controlled
CIVIL LIBERTIES UNION, petitioner, corporations, at least one-third (1/3) of the members
of the boards of such corporation should either be a
vs.
THE EXECUTIVE SECRETARY, respondent. secretary, or undersecretary, or assistant secretary.

G.R. No. 83815 February 22, 1991 Petitioners maintain that this Executive Order which, in effect,
allows members of the Cabinet, their undersecretaries and
assistant secretaries to hold other government offices or
ANTI-GRAFT LEAGUE OF THE PHILIPPINES, INC. and positions in addition to their primary positions, albeit subject to
CRISPIN T. REYES, petitioners, the limitation therein imposed, runs counter to Section 13,
vs. Article VII of the 1987 Constitution,2 which provides as follows:
PHILIP ELLA C. JUICO, as Secretary of Agrarian Reform;
CARLOS DOMINGUEZ, as Secretary of Agriculture;
Sec. 13. The President, Vice-President, the
LOURDES QUISUMBING, as Secretary of Education,
Culture and Sports; FULGENCIO FACTORAN, JR., as Members of the Cabinet, and their deputies or
Secretary of Environment and Natural Resources; assistants shall not, unless otherwise provided in this
VICENTE V. JAYME, as Secretary of Finance; SEDFREY Constitution, hold any other office or employment
ORDOÑEZ, as Secretary of Justice; FRANKLIN N. DRILON, during their tenure. They shall not, during said
as Secretary of Labor and Employment; LUIS SANTOS, as tenure, directly or indirectly practice any other
Secretary of Local Government; FIDEL V. RAMOS, as profession, participate in any business, or be
financially interested in any contract with, or in any
Secretary of National Defense; TEODORO F. BENIGNO, as
Press Secretary; JUANITO FERRER, as Secretary of franchise, or special privilege granted by the
Government or any subdivision, agency, or
Public Works and Highways; ANTONIO ARRIZABAL, as
Secretary of Science and Technology; JOSE instrumentality thereof, including government-owned
CONCEPCION, as Secretary of Trade and Industry; JOSE or controlled corporations or their subsidiaries. They
ANTONIO GONZALEZ, as Secretary of Tourism; ALFREDO shall strictly avoid conflict of interest in the conduct of
R.A. BENGZON, as Secretary of Health; REINERIO D. their office.
REYES, as Secretary of Transportation and
Communication; GUILLERMO CARAGUE, as It is alleged that the above-quoted Section 13, Article VII
Commissioner of the Budget; and SOLITA MONSOD, as prohibits public respondents, as members of the Cabinet,
Head of the National Economic Development along with the other public officials enumerated in the list
Authority, respondents. attached to the petitions as Annex "C" in G.R. No.
838153 and as Annex "B" in G.R. No. 838964 from holding any
Ignacio P. Lacsina, Luis R. Mauricio, Antonio R. Quintos and other office or employment during their tenure. In addition to
Juan T. David for petitioners in 83896. seeking a declaration of the unconstitutionality of Executive
Order No. 284, petitioner Anti-Graft League of the Philippines
Antonio P. Coronel for petitioners in 83815.
further seeks in G.R. No. 83815 the issuance of the
extraordinary writs of prohibition and mandamus, as well as a
temporary restraining order directing public respondents
therein to cease and desist from holding, in addition to their
primary positions, dual or multiple positions other than those
FERNAN, C.J.:p authorized by the 1987 Constitution and from receiving any
salaries, allowances, per diems and other forms of privileges
and the like appurtenant to their questioned positions, and
These two (2) petitions were consolidated per resolution dated compelling public respondents to return, reimburse or refund
August 9, 19881 and are being resolved jointly as both seek a any and all amounts or benefits that they may have received
declaration of the unconstitutionality of Executive Order No. from such positions.
284 issued by President Corazon C. Aquino on July 25, 1987.
The pertinent provisions of the assailed Executive Order are:
Specifically, petitioner Anti-Graft League of the Philippines
charges that notwithstanding the aforequoted "absolute and
Sec. 1. Even if allowed by law or by the ordinary self-executing" provision of the 1987 Constitution, then
functions of his position, a member of the Cabinet, Secretary of Justice Sedfrey Ordoñez, construing Section 13,
undersecretary or assistant secretary or other Article VII in relation to Section 7, par. (2), Article IX-B,
appointive officials of the Executive Department may, rendered on July 23, 1987 Opinion No. 73, series of
in addition to his primary position, hold not more than 1987,5 declaring that Cabinet members, their deputies
two positions in the government and government (undersecretaries) and assistant secretaries may hold other
corporations and receive the corresponding public office, including membership in the boards of
compensation therefor; Provided, that this limitation government corporations: (a) when directly provided for in the
shall not apply to ad hoc bodies or committees, or to Constitution as in the case of the Secretary of Justice who is
boards, councils or bodies of which the President is made an ex-officio member of the Judicial and Bar Council
the Chairman. under Section 8, paragraph 1, Article VIII; or (b) if allowed by
law; or (c) if allowed by the primary functions of their respective
Sec. 2. If a member of the cabinet, undersecretary or positions; and that on the basis of this Opinion, the President
assistant secretary or other appointive official of the of the Philippines, on July 25, 1987 or two (2) days before
Executive Department holds more positions than Congress convened on July 27, 1987: promulgated Executive
what is allowed in Section 1 hereof, they (sic) must Order No. 284.6
relinquish the excess position in favor of the
subordinate official who is next in rank, but in no case Petitioner Anti-Graft League of the Philippines objects to both
shall any official hold more than two positions other DOJ Opinion No. 73 and Executive Order No. 284 as they
than his primary position. allegedly "lumped together" Section 13, Article VII and the
general provision in another article, Section 7, par. (2), Article
I-XB. This "strained linkage" between the two provisions, each
FINALS CONSTITUTION I ACJUCO 30

addressed to a distinct and separate group of public officers – the Cabinet under the second paragraph of Section 3, Article
– one, the President and her official family, and the other, VII or the Secretary of Justice being designated an ex-
public servants in general –– allegedly "abolished the clearly officio member of the Judicial and Bar Council under Article
separate, higher, exclusive, and mandatory constitutional rank VIII, Sec. 8 (1). Public respondents, on the other hand,
assigned to the prohibition against multiple jobs for the maintain that the phrase "unless otherwise provided in the
President, the Vice-President, the members of the Cabinet, Constitution" in Section 13, Article VII makes reference to
and their deputies and subalterns, who are the leaders of Section 7, par. (2), Article I-XB insofar as the appointive
government expected to lead by example."7 Article IX-B, officials mentioned therein are concerned.
Section 7, par. (2)8 provides:
The threshold question therefore is: does the prohibition in
Sec. 7. . . . . . Section 13, Article VII of the 1987 Constitution insofar as
Cabinet members, their deputies or assistants are concerned
Unless otherwise allowed by law or by the primary admit of the broad exceptions made for appointive officials in
functions of his position, no appointive official shall general under Section 7, par. (2), Article I-XB which, for easy
hold any other office or employment in the reference is quoted anew, thus: "Unless otherwise allowed by
government or any subdivision, agency or law or by the primary functions of his position, no appointive
official shall hold any other office or employment in the
instrumentality thereof, including government-owned
or controlled corporations or their subsidiaries. Government or any subdivision, agency or instrumentality
thereof, including government-owned or controlled corporation
or their subsidiaries."
The Solicitor General counters that Department of Justice DOJ
Opinion No. 73, series of 1987, as further elucidated and
clarified by DOJ Opinion No. 129, series of 19879 and DOJ We rule in the negative.
Opinion No. 155, series of 1988,10 being the first official
construction and interpretation by the Secretary of Justice of A foolproof yardstick in constitutional construction is the
Section 13, Article VII and par. (2) of Section 7, Article I-XB of intention underlying the provision under consideration. Thus, it
the Constitution, involving the same subject of appointments or has been held that the Court in construing a Constitution
designations of an appointive executive official to positions should bear in mind the object sought to be accomplished by
other than his primary position, is "reasonably valid and its adoption, and the evils, if any, sought to be prevented or
constitutionally firm," and that Executive Order No. 284, remedied. A doubtful provision will be examined in the light of
promulgated pursuant to DOJ Opinion No. 73, series of 1987 the history of the times, and the condition and circumstances
is consequently constitutional. It is worth noting that DOJ under which the Constitution was framed. The object is to
Opinion No. 129, series of 1987 and DOJ Opinion No. 155, ascertain the reason which induced the framers of the
series of 1988 construed the limitation imposed by E.O. No. Constitution to enact the particular provision and the purpose
284 as not applying to ex-officio positions or to positions which, sought to be accomplished thereby, in order to construe the
although not so designated as ex-officio are allowed by the whole as to make the words consonant to that reason and
primary functions of the public official, but only to the holding calculated to effect that purpose.11
of multiple positions which are not related to or necessarily
included in the position of the public official concerned
The practice of designating members of the Cabinet, their
(disparate positions). deputies and assistants as members of the governing bodies
or boards of various government agencies and
In sum, the constitutionality of Executive Order No. 284 is instrumentalities, including government-owned and controlled
being challenged by petitioners on the principal submission corporations, became prevalent during the time legislative
that it adds exceptions to Section 13, Article VII other than powers in this country were exercised by former President
those provided in the Constitution. According to petitioners, by Ferdinand E. Marcos pursuant to his martial law authority.
virtue of the phrase "unless otherwise provided in this There was a proliferation of newly-created agencies,
Constitution," the only exceptions against holding any other instrumentalities and government-owned and controlled
office or employment in Government are those provided in the corporations created by presidential decrees and other modes
Constitution, namely: (1) The Vice-President may be appointed of presidential issuances where Cabinet members, their
as a Member of the Cabinet under Section 3, par. (2), Article deputies or assistants were designated to head or sit as
VII thereof; and (2) the Secretary of Justice is an ex- members of the board with the corresponding salaries,
officio member of the Judicial and Bar Council by virtue of emoluments, per diems, allowances and other perquisites of
Section 8 (1), Article VIII. office. Most of these instrumentalities have remained up to the
present time.
Petitioners further argue that the exception to the prohibition in
Section 7, par. (2), Article I-XB on the Civil Service This practice of holding multiple offices or positions in the
Commission applies to officers and employees of the Civil government soon led to abuses by unscrupulous public
Service in general and that said exceptions do not apply and officials who took advantage of this scheme for purposes of
cannot be extended to Section 13, Article VII which applies self-enrichment. In fact, the holding of multiple offices in
specifically to the President, Vice-President, Members of the government was strongly denounced on the floor of the
Cabinet and their deputies or assistants. Batasang Pambansa.12 This condemnation came in reaction to
the published report of the Commission on Audit, entitled "1983
There is no dispute that the prohibition against the President, Summary Annual Audit Report on: Government-Owned and
Vice-President, the members of the Cabinet and their deputies Controlled Corporations, Self-Governing Boards and
or assistants from holding dual or multiple positions in the Commissions" which carried as its Figure No. 4 a "Roaster of
Government admits of certain exceptions. The disagreement Membership in Governing Boards of Government-Owned and
Controlled Corporations as of December 31, 1983."
between petitioners and public respondents lies on the
constitutional basis of the exception. Petitioners insist that
because of the phrase "unless otherwise provided in this Particularly odious and revolting to the people's sense of
Constitution" used in Section 13 of Article VII, the exception propriety and morality in government service were the data
must be expressly provided in the Constitution, as in the case contained therein that Roberto V. Ongpin was a member of the
of the Vice-President being allowed to become a Member of governing boards of twenty-nine (29) governmental agencies,
FINALS CONSTITUTION I ACJUCO 31

instrumentalities and corporations; Imelda R. Marcos of and his official family is therefore all-embracing and covers
twenty-three (23); Cesar E.A. Virata of twenty-two (22); Arturo both public and private office or employment.
R. Tanco, Jr. of fifteen (15); Jesus S. Hipolito and Geronimo Z.
Velasco, of fourteen each (14); Cesar C. Zalamea of thirteen
Going further into Section 13, Article VII, the second sentence
(13); Ruben B. Ancheta and Jose A. Roño of twelve (12) each; provides: "They shall not, during said tenure, directly or
Manuel P. Alba, Gilberto O. Teodoro, and Edgardo Tordesillas indirectly, practice any other profession, participate in any
of eleven (11) each; and Lilia Bautista and Teodoro Q. Peña of business, or be financially interested in any contract with, or in
ten (10) each.13 any franchise, or special privilege granted by the Government
or any subdivision, agency or instrumentality thereof, including
The blatant betrayal of public trust evolved into one of the government-owned or controlled corporations or their
serious causes of discontent with the Marcos regime. It was subsidiaries." These sweeping, all-embracing prohibitions
therefore quite inevitable and in consonance with the imposed on the President and his official family, which
overwhelming sentiment of the people that the 1986 prohibitions are not similarly imposed on other public officials
Constitutional Commission, convened as it was after the or employees such as the Members of Congress, members of
people successfully unseated former President Marcos, should the civil service in general and members of the armed forces,
draft into its proposed Constitution the provisions under are proof of the intent of the 1987 Constitution to treat the
consideration which are envisioned to remedy, if not correct, President and his official family as a class by itself and to
the evils that flow from the holding of multiple governmental impose upon said class stricter prohibitions.
offices and employment. In fact, as keenly observed by Mr.
Justice Isagani A. Cruz during the deliberations in these cases, Such intent of the 1986 Constitutional Commission to be
one of the strongest selling points of the 1987 Constitution stricter with the President and his official family was also
during the campaign for its ratification was the assurance given succinctly articulated by Commissioner Vicente Foz after
by its proponents that the scandalous practice of Cabinet Commissioner Regalado Maambong noted during the floor
members holding multiple positions in the government and deliberations and debate that there was no symmetry between
collecting unconscionably excessive compensation therefrom the Civil Service prohibitions, originally found in the General
would be discontinued. Provisions and the anticipated report on the Executive
Department. Commissioner Foz Commented, "We actually
But what is indeed significant is the fact that although Section have to be stricter with the President and the members of the
7, Article I-XB already contains a blanket prohibition against Cabinet because they exercise more powers and, therefore,
the holding of multiple offices or employment in the more cheeks and restraints on them are called for because
government subsuming both elective and appointive public there is more possibility of abuse in their case."14
officials, the Constitutional Commission should see it fit to
formulate another provision, Sec. 13, Article VII, specifically Thus, while all other appointive officials in the civil service are
prohibiting the President, Vice-President, members of the
allowed to hold other office or employment in the government
Cabinet, their deputies and assistants from holding any other during their tenure when such is allowed by law or by the
office or employment during their tenure, unless otherwise primary functions of their positions, members of the Cabinet,
provided in the Constitution itself. their deputies and assistants may do so only when expressly
authorized by the Constitution itself. In other words, Section 7,
Evidently, from this move as well as in the different Article I-XB is meant to lay down the general rule applicable to
phraseologies of the constitutional provisions in question, the all elective and appointive public officials and employees, while
intent of the framers of the Constitution was to impose a stricter Section 13, Article VII is meant to be the exception applicable
prohibition on the President and his official family in so far as only to the President, the Vice- President, Members of the
holding other offices or employment in the government or Cabinet, their deputies and assistants.
elsewhere is concerned.
This being the case, the qualifying phrase "unless otherwise
Moreover, such intent is underscored by a comparison of provided in this Constitution" in Section 13, Article VII cannot
Section 13, Article VII with other provisions of the Constitution possibly refer to the broad exceptions provided under Section
on the disqualifications of certain public officials or employees 7, Article I-XB of the 1987 Constitution. To construe said
from holding other offices or employment. Under Section 13, qualifying phrase as respondents would have us do, would
Article VI, "(N)o Senator or Member of the House of render nugatory and meaningless the manifest intent and
Representatives may hold any other office or employment in purpose of the framers of the Constitution to impose a stricter
the Government . . .". Under Section 5(4), Article XVI, "(N)o prohibition on the President, Vice-President, Members of the
member of the armed forces in the active service shall, at any Cabinet, their deputies and assistants with respect to holding
time, be appointed in any capacity to a civilian position in the other offices or employment in the government during their
Government,including government-owned or controlled tenure. Respondents' interpretation that Section 13 of Article
corporations or any of their subsidiaries." Even Section 7 (2), VII admits of the exceptions found in Section 7, par. (2) of
Article IX-B, relied upon by respondents provides "(U)nless Article IX-B would obliterate the distinction so carefully set by
otherwise allowed by law or by the primary functions of his the framers of the Constitution as to when the high-ranking
position, no appointive official shall hold any other office or officials of the Executive Branch from the President to
employment in the Government." Assistant Secretary, on the one hand, and the generality of civil
servants from the rank immediately below Assistant Secretary
downwards, on the other, may hold any other office or position
It is quite notable that in all these provisions on disqualifications
to hold other office or employment, the prohibition pertains to in the government during their tenure.
an office or employment in the government and government-
owned or controlled corporations or their subsidiaries. In Moreover, respondents' reading of the provisions in question
striking contrast is the wording of Section 13, Article VII which would render certain parts of the Constitution inoperative. This
states that "(T)he President, Vice-President, the Members of observation applies particularly to the Vice-President who,
the Cabinet, and their deputies or assistants shall not, unless under Section 13 of Article VII is allowed to hold other office or
otherwise provided in this Constitution, hold any other office or employment when so authorized by the Constitution, but who
employment during their tenure." In the latter provision, the as an elective public official under Sec. 7, par. (1) of Article I-
disqualification is absolute, not being qualified by the phrase XB is absolutely ineligible "for appointment or designation in
"in the Government." The prohibition imposed on the President any capacity to any public office or position during his tenure."
FINALS CONSTITUTION I ACJUCO 32

Surely, to say that the phrase "unless otherwise provided in The Secretary of Labor and Employment cannot chair the
this Constitution" found in Section 13, Article VII has reference Board of Trustees of the National Manpower and Youth
to Section 7, par. (1) of Article I-XB would render meaningless Council (NMYC) or the Philippine Overseas Employment
the specific provisions of the Constitution authorizing the Vice- Administration (POEA), both of which are attached to his
President to become a member of the Cabinet,15 and to act as department for policy coordination and guidance. Neither can
President without relinquishing the Vice-Presidency where the his Undersecretaries and Assistant Secretaries chair these
President shall not nave been chosen or fails to qualify. 16 Such agencies.
absurd consequence can be avoided only by interpreting the
two provisions under consideration as one, i.e., Section 7, par. The Secretaries of Finance and Budget cannot sit in the
(1) of Article I-XB providing the general rule and the other, i.e., Monetary Board.24 Neither can their respective
Section 13, Article VII as constituting the exception thereto. In undersecretaries and assistant secretaries. The Central Bank
the same manner must Section 7, par. (2) of Article I-XB be Governor would then be assisted by lower ranking employees
construed vis-a-vis Section 13, Article VII.
in providing policy direction in the areas of money, banking and
credit.25
It is a well-established rule in Constitutional construction that
no one provision of the Constitution is to be separated from all Indeed, the framers of our Constitution could not have intended
the others, to be considered alone, but that all the provisions
such absurd consequences. A Constitution, viewed as a
bearing upon a particular subject are to be brought into view continuously operative charter of government, is not to be
and to be so interpreted as to effectuate the great purposes of
interpreted as demanding the impossible or the impracticable;
the instrument.17 Sections bearing on a particular subject and unreasonable or absurd consequences, if possible, should
should be considered and interpreted together as to effectuate be avoided.26
the whole purpose of the Constitution18 and one section is not
to be allowed to defeat another, if by any reasonable
construction, the two can be made to stand together. 19 To reiterate, the prohibition under Section 13, Article VII is not
to be interpreted as covering positions held without additional
compensation in ex-officio capacities as provided by law and
In other words, the court must harmonize them, if practicable, as required by the primary functions of the concerned official's
and must lean in favor of a construction which will render every office. The term ex-officio means "from office; by virtue of
word operative, rather than one which may make the words office." It refers to an "authority derived from official character
idle and nugatory.20 merely, not expressly conferred upon the individual character,
but rather annexed to the official position." Ex-officio likewise
Since the evident purpose of the framers of the 1987 denotes an "act done in an official character, or as a
Constitution is to impose a stricter prohibition on the President, consequence of office, and without any other appointment or
Vice-President, members of the Cabinet, their deputies and authority than that conferred by the office." 27 An ex-
assistants with respect to holding multiple offices or officio member of a board is one who is a member by virtue of
employment in the government during their tenure, the his title to a certain office, and without further warrant or
exception to this prohibition must be read with equal severity. appointment.28 To illustrate, by express provision of law, the
On its face, the language of Section 13, Article VII is prohibitory Secretary of Transportation and Communications is the ex-
so that it must be understood as intended to be a positive and officioChairman of the Board of the Philippine Ports
unequivocal negation of the privilege of holding multiple Authority,29 and the Light Rail Transit Authority.30
government offices or employment. Verily, wherever the
language used in the constitution is prohibitory, it is to be The Court had occasion to explain the meaning of an ex-
understood as intended to be a positive and unequivocal officio position in Rafael vs. Embroidery and Apparel Control
negation.21 The phrase "unless otherwise provided in this and Inspection Board,31 thus: "An examination of section 2 of
Constitution" must be given a literal interpretation to refer only the questioned statute (R.A. 3137) reveals that for the
to those particular instances cited in the Constitution itself, to chairman and members of the Board to qualify they need only
wit: the Vice-President being appointed as a member of the be designated by the respective department heads. With the
Cabinet under Section 3, par. (2), Article VII; or acting as
exception of the representative from the private sector, they
President in those instances provided under Section 7, pars. sit ex-officio. In order to be designated they must already be
(2) and (3), Article VII; and, the Secretary of Justice being ex-
holding positions in the offices mentioned in the law. Thus, for
officiomember of the Judicial and Bar Council by virtue of instance, one who does not hold a previous appointment in the
Section 8 (1), Article VIII. Bureau of Customs, cannot, under the act, be designated a
representative from that office. The same is true with respect
The prohibition against holding dual or multiple offices or to the representatives from the other offices. No new
employment under Section 13, Article VII of the Constitution appointments are necessary. This is as it should be, because
must not, however, be construed as applying to posts occupied the representatives so designated merely perform duties in the
by the Executive officials specified therein without additional Board in addition to those already performed under their
compensation in an ex-officio capacity as provided by law and original appointments."32
as required22 by the primary functions of said officials' office.
The reason is that these posts do no comprise "any other The term "primary" used to describe "functions" refers to the
office" within the contemplation of the constitutional prohibition
order of importance and thus means chief or principal function.
but are properly an imposition of additional duties and functions The term is not restricted to the singular but may refer to the
on said officials.23 To characterize these posts otherwise would
plural.33 The additional duties must not only be closely related
lead to absurd consequences, among which are: The to, but must be required by the official's primary functions.
President of the Philippines cannot chair the National Security
Examples of designations to positions by virtue of one's
Council reorganized under Executive Order No. 115 primary functions are the Secretaries of Finance and Budget
(December 24, 1986). Neither can the Vice-President, the sitting as members of the Monetary Board, and the Secretary
Executive Secretary, and the Secretaries of National Defense, of Transportation and Communications acting as Chairman of
Justice, Labor and Employment and Local Government sit in the Maritime Industry Authority34 and the Civil Aeronautics
this Council, which would then have no reason to exist for lack Board.
of a chairperson and members. The respective
undersecretaries and assistant secretaries, would also be
prohibited.
FINALS CONSTITUTION I ACJUCO 33

If the functions required to be performed are merely incidental, officials' primary functions. The example given by
remotely related, inconsistent, incompatible, or otherwise alien Commissioner Monsod was the Minister of Trade and
to the primary function of a cabinet official, such additional Industry.38
functions would fall under the purview of "any other office"
prohibited by the Constitution. An example would be the Press While this exchange between Commissioners Monsod and
Undersecretary sitting as a member of the Board of the Ople may be used as authority for saying that additional
Philippine Amusement and Gaming Corporation. The same functions and duties flowing from the primary functions of the
rule applies to such positions which confer on the cabinet official may be imposed upon him without offending the
official management functions and/or monetary compensation, constitutional prohibition under consideration, it cannot,
such as but not limited to chairmanships or directorships in however, be taken as authority for saying that this exception is
government-owned or controlled corporations and their by virtue of Section 7, par. (2) of Article I-XB. This colloquy
subsidiaries. between the two Commissioners took place in the plenary
session of September 27, 1986. Under consideration then was
Mandating additional duties and functions to the President, Section 3 of Committee Resolution No. 531 which was the
Vice-President, Cabinet Members, their deputies or assistants proposed article on General Provisions. 39 At that time, the
which are not inconsistent with those already prescribed by article on the Civil Service Commission had been approved on
their offices or appointments by virtue of their special third reading on July 22, 1986,40 while the article on the
knowledge, expertise and skill in their respective executive Executive Department, containing the more specific prohibition
offices is a practice long-recognized in many jurisdictions. It is in Section 13, had also been earlier approved on third reading
a practice justified by the demands of efficiency, policy on August 26, 1986.41 It was only after the draft Constitution
direction, continuity and coordination among the different had undergone reformatting and "styling" by the Committee on
offices in the Executive Branch in the discharge of its Style that said Section 3 of the General Provisions became
multifarious tasks of executing and implementing laws Section 7, par. (2) of Article IX-B and reworded "Unless
affecting national interest and general welfare and delivering otherwise allowed by law or by the primary functions of his
basic services to the people. It is consistent with the power position. . . ."
vested on the President and his alter egos, the Cabinet
members, to have control of all the executive departments, What was clearly being discussed then were general principles
bureaus and offices and to ensure that the laws are faithfully
which would serve as constitutional guidelines in the absence
executed.35 Without these additional duties and functions of specific constitutional provisions on the matter. What was
being assigned to the President and his official family to sit in
primarily at issue and approved on that occasion was the
the governing bodies or boards of governmental agencies or adoption of the qualified and delimited phrase "primary
instrumentalities in an ex-officio capacity as provided by law functions" as the basis of an exception to the general rule
and as required by their primary functions, they would be covering all appointive public officials. Had the Constitutional
supervision, thereby deprived of the means for control and Commission intended to dilute the specific prohibition in said
resulting in an unwieldy and confused bureaucracy. Section 13 of Article VII, it could have re-worded said Section
13 to conform to the wider exceptions provided in then Section
It bears repeating though that in order that such additional 3 of the proposed general Provisions, later placed as Section
duties or functions may not transgress the prohibition 7, par. (2) of Article IX-B on the Civil Service Commission.
embodied in Section 13, Article VII of the 1987 Constitution,
such additional duties or functions must be required by the That this exception would in the final analysis apply also to the
primary functions of the official concerned, who is to perform
President and his official family is by reason of the legal
the same in an ex-officio capacity as provided by law, without principles governing additional functions and duties of public
receiving any additional compensation therefor.
officials rather than by virtue of Section 7, par. 2, Article IX-B
At any rate, we have made it clear that only the additional
The ex-officio position being actually and in legal functions and duties "required," as opposed to "allowed," by
contemplation part of the principal office, it follows that the the primary functions may be considered as not constituting
official concerned has no right to receive additional "any other office."
compensation for his services in the said position. The reason
is that these services are already paid for and covered by the
While it is permissible in this jurisdiction to consult the debates
compensation attached to his principal office. It should be and proceedings of the constitutional convention in order to
obvious that if, say, the Secretary of Finance attends a meeting arrive at the reason and purpose of the resulting Constitution,
of the Monetary Board as an ex-officio member thereof, he is resort thereto may be had only when other guides fail42 as said
actually and in legal contemplation performing the primary proceedings are powerless to vary the terms of the Constitution
function of his principal office in defining policy in monetary and when the meaning is clear.1âwphi1Debates in the
banking matters, which come under the jurisdiction of his constitutional convention "are of value as showing the views of
department. For such attendance, therefore, he is not entitled
the individual members, and as indicating the reasons for their
to collect any extra compensation, whether it be in the form of votes, but they give us no light as to the views of the large
a per them or an honorarium or an allowance, or some other
majority who did not talk, much less of the mass of our fellow
such euphemism. By whatever name it is designated, such citizens whose votes at the polls gave that instrument the force
additional compensation is prohibited by the Constitution. of fundamental law. We think it safer to construe the
constitution from what appears upon its face." 43 The proper
It is interesting to note that during the floor deliberations on the interpretation therefore depends more on how it was
proposal of Commissioner Christian Monsod to add to Section understood by the people adopting it than in the framers's
7, par. (2), Article IX-B, originally found as Section 3 of the understanding thereof.44
General Provisions, the exception "unless required by the
functions of his position,"36 express reference to certain high- It being clear, as it was in fact one of its best selling points, that
ranking appointive public officials like members of the Cabinet the 1987 Constitution seeks to prohibit the President, Vice-
were made.37 Responding to a query of Commissioner Blas President, members of the Cabinet, their deputies or assistants
Ople, Commissioner Monsod pointed out that there are from holding during their tenure multiple offices or employment
instances when although not required by current law, in the government, except in those cases specified in the
membership of certain high-ranking executive officials in other
Constitution itself and as above clarified with respect to posts
offices and corporations is necessary by reason of said held without additional compensation in an ex-officio capacity
FINALS CONSTITUTION I ACJUCO 34

as provided by law and as required by the primary functions of legally entitled to the emoluments of the office, and may in an
their office, the citation of Cabinet members (then called appropriate action recover the salary, fees and other
Ministers) as examples during the debate and deliberation on compensations attached to the office. This doctrine is,
the general rule laid down for all appointive officials should be undoubtedly, supported on equitable grounds since it seems
considered as mere personal opinions which cannot override unjust that the public should benefit by the services of an
the constitution's manifest intent and the people' officer de facto and then be freed from all liability to pay any
understanding thereof. one for such services.47 Any per diem, allowances or other
emoluments received by the respondents by virtue of actual
In the light of the construction given to Section 13, Article VII in services rendered in the questioned positions may therefore
relation to Section 7, par. (2), Article IX-B of the 1987 be retained by them.
Constitution, Executive Order No. 284 dated July 23, 1987 is
unconstitutional. Ostensibly restricting the number of positions WHEREFORE, subject to the qualification above-stated, the
that Cabinet members, undersecretaries or assistant petitions are GRANTED. Executive Order No. 284 is hereby
secretaries may hold in addition to their primary position to not declared null and void and is accordingly set aside.
more than two (2) positions in the government and government
corporations, Executive Order No. 284 actually allows them to SO ORDERED.
hold multiple offices or employment in direct contravention of
the express mandate of Section 13, Article VII of the 1987
Constitution prohibiting them from doing so, unless otherwise
provided in the 1987 Constitution itself.

The Court is alerted by respondents to the impractical


consequences that will result from a strict application of the
prohibition mandated under Section 13, Article VII on the
operations of the Government, considering that Cabinet
members would be stripped of their offices held in an ex-
officio capacity, by reason of their primary positions or by virtue
of legislation. As earlier clarified in this decision, ex-
officio posts held by the executive official concerned without
additional compensation as provided by law and as required
by the primary functions of his office do not fall under the
definition of "any other office" within the contemplation of the
constitutional prohibition. With respect to other offices or
employment held by virtue of legislation, including
chairmanships or directorships in government-owned or
controlled corporations and their subsidiaries, suffice it to say
that the feared impractical consequences are more apparent
than real. Being head of an executive department is no mean
job. It is more than a full-time job, requiring full attention,
specialized knowledge, skills and expertise. If maximum
benefits are to be derived from a department head's ability and
expertise, he should be allowed to attend to his duties and
responsibilities without the distraction of other governmental
offices or employment. He should be precluded from
dissipating his efforts, attention and energy among too many
positions of responsibility, which may result in haphazardness
and inefficiency. Surely the advantages to be derived from this
concentration of attention, knowledge and expertise,
particularly at this stage of our national and economic
development, far outweigh the benefits, if any, that may be
gained from a department head spreading himself too thin and
taking in more than what he can handle.

Finding Executive Order No. 284 to be constitutionally infirm,


the court hereby orders respondents Secretary of Environment
and Natural Resources Fulgencio Factoran, Jr., Secretary of
Local Government45 Luis Santos, Secretary of National
Defense Fidel V. Ramos, Secretary of Health Alfredo R.A.
Bengzon and Secretary of the Budget Guillermo Carague to
immediately relinquish their other offices or employment, as
herein defined, in the government, including government-
owned or controlled corporations and their subsidiaries. With
respect to the other named respondents, the petitions have
become moot and academic as they are no longer occupying
the positions complained of.

During their tenure in the questioned positions, respondents


may be considered de facto officers and as such entitled to
emoluments for actual services rendered. 46 It has been held
that "in cases where there is no de jure,officer, a de
facto officer, who, in good faith has had possession of the
office and has discharged the duties pertaining thereto, is
FINALS CONSTITUTION I ACJUCO 35

ELEANOR DELA CRUZ, FEDERICO LUCHICO, JR., DOF Eleanor dela Cruz P25,200.00
(1991-1993)
SOLEDAD EMILIA CRUZ, JOEL LUSTRIA, HENRY PAREL,
HELENA HABULAN, PORFIRIO VILLENA, JOSEPH
FRANCIA, CARMELLA TORRES, JOB DAVID, CESAR DTI Federico Luchico, Jr. 36,450.00
MEJIA, MA. LOURDES V. DEDAL, ALICE TIONGSON, (1991-1992)
REYDELUZ CONFERIDO, PHILIPPE LIM, NERISSA
SANCHEZ, MARY LUZ ELAINE PURACAN, RODOLFO DOF Soledad Emilia Cruz 57,300.00
QUIMBO, TITO GENILO and OSCAR ABUNDO, as (1992-1995)
members of the Board of the National Housing Authority
from the period covering 1991-1996, petitioners, vs.
DOLE Joel Lustria 4,500.00
COMMISSION ON AUDIT, represented by its
(1992)
Commissioners, respondents.

DOLE Henry Parel 2,250.00


DECISION
(1992)
SANDOVAL-GUTIERREZ, J.:
DOF Helena Habulan 4,050.00
This petition for certiorari[1] assails the Decision No. 98- (1993-1994)
381 dated September 22, 1998, rendered by the Commission
on Audit (COA), denying petitioners appeal from the Notice of DOF Porfirio Villena 6,750.00
Disallowance No. 97-011-061 issued by the NHA Resident (1993)
Auditor on October 23, 1997. Such Notice disallowed
payment to petitioners of their representation allowances
and per diems for the period from August 19, 1991 to DTI Joseph Francia 73,500.00
August 31, 1996 in the total amount of P276,600.00. (1993-1995)

Petitioners, numbering 20, were members of the Board DOLE Carmela Torres 4,500.00
of Directors of the National Housing Authority (NHA) from 1991 (1993)
to 1996.
On September 19, 1997, the COA issued Memorandum DPWH Job David 6,750.00
No. 97-038[2] directing all unit heads/auditors/team leaders of (1993-1994)
the national government agencies and government-owned and
controlled corporations which have effected payment of any DPWH Cesar Mejia 3,150.00
form of additional compensation or remuneration to cabinet (1993)
secretaries, their deputies and assistants, or their
representatives, in violation of the rule on multiple positions, to
(a) immediately cause the disallowance of such additional DOF Ma. Lourdes V. Dedal 2,250.00
compensation or remuneration given to and received by the (1993)
concerned officials, and (b) effect the refund of the same from
the time of the finality of the Supreme Court En Banc Decision DTI Alice Tiongson 900.00
in the consolidated cases of Civil Liberties Union vs. (1994)
Exexcutive Secretary and Anti-Graft League of the Philippines,
Inc. et al. vs. Secretary of Agrarian Reform, et al., promulgated
on February 22, 1991.[3] The COA Memorandum further stated DOLE Reynaluz Conferido 11,250.00
that the said Supreme Court Decision, which became final and (1994-1995)
executory on August 19, 1991,[4] declared Executive Order No.
284 unconstitutional insofar as it allows Cabinet members, DOLE Philippe Lim 4,500.00
their deputies and assistants to hold other offices, in addition (1994-1995)
to their primary offices, and to receive compensation therefor.
Accordingly, on October 23, 1997, NHA Resident Auditor DOF Nerissa Sanchez 2,700.00
Salvador J. Vasquez issued Notice of Disallowance No. 97- (1995)
011-061[5] disallowing in audit the payment of
representation allowances and per diems of "Cabinet DOF Mary Luz Elaine Puracan 1,800.00
members who were the ex- officio members of the NHA (1995)
Board of Directors and/or their respective alternates who
actually received the payments." The total disallowed
amount of P276,600 paid as representation allowances and DOLE Rodolfo Quimbo 7,200.00
per diems to each of the petitioners named below, covering the (1995)
period from August 19, 1991 to August 31, 1996, is broken
down as follows: [6] DOLE Tito Genilo 14,400.00
(1995)
NATIONAL HOUSING AUTHORITY
SCHEDULE OF PAID REPRESENTATION/PER DPWH Oscar Abundo 7,200.00
DIEM OF THE BOARD OF DIRECTORS (1995-1996) _____________
For the period August 19, 1991 to August 31, 1996 P276,600.00
============
AGENCY MEMBERS OF BOARD OF AMOUNT
DISALLOWED Petitioners, through then Chairman Dionisio C. Dela
DIRECTORS Serna of the NHA Board of Directors, appealed from the Notice
FINALS CONSTITUTION I ACJUCO 36

of Disallowance to the Commission on Audit [7] based on the Chairman, the Board shall elect a temporary presiding officer.
following grounds: x x x (Emphasis ours)

1. The Decision of the Supreme Court in Civil


Liberties Union and Anti-Graft League of the It bears stressing that under the above provisions, the
Philippines, Inc. was clarified in the Resolution persons mandated by law to sit as members of the NHA Board
of the Court En Banc on August 1, 1991, in that are the following: (1) the Secretary of Public Works,
the constitutional ban against dual or multiple Transportation and Communications, (2) the Director-
positions applies only to the members of the General of the National Economic and Development
Cabinet, their deputies or assistants. It does Authority, (3) the Secretary of Finance, (4) the Secretary of
not cover other appointive officials with Labor, (5) the Secretary of Industry, (6) the Executive
equivalent rank or those lower than the position Secretary, and (7) the General Manager of the NHA. While
of Assistant Secretary; and petitioners are not among those officers, however, they are
alternates of the said officers, whose acts shall be considered
2. The NHA Directors are not Secretaries, the acts of their principals.
Undersecretaries or Assistant Secretaries and
that they occupy positions lower than the On this point, Section 13, Art. VII of the 1987
position of Assistant Secretary. Constitution, provides:

On September 22, 1998, the COA issued Decision No. SEC. 13. The President, Vice-President, the Members of
98-381[8] denying petitioners' appeal, thus: the Cabinet, and their deputies or assistants shall not,
unless otherwise provided in this Constitution, hold any
After circumspect evaluation of the facts and issues raised other office or employment during their tenure. They shall
herein, this Commission finds the instant appeal devoid of not, during their tenure, directly or indirectly practice any
merit. It must be stressed at the outset that the Directors other profession, participate in any business, or be
concerned were not sitting in the NHA Board in their own right financially interested in any contract with, or in any
but as representatives of cabinet members and who are franchise, or special privilege granted by the Government
constitutionally prohibited from holding any other office or or any subdivision, agency or instrumentality thereof,
employment and receive compensation therefor, during their including any government-owned or controlled
tenure (Section 13, Article VII, Constitution; Civil Liberties corporations or their subsidiaries. They shall strictly
Union vs. Executive Secretary, 194 SCRA 317). avoid conflict of interest in the conduct of their office.

It may be conceded that the directors concerned occupy The spouse and relatives by consanguinity or affinity
positions lower than Assistant Secretary which may exempt within the fourth civil degree of the President shall not
them from the prohibition (under) the doctrine enunciated during his tenure be appointed as Members of the
in Civil Liberties Union vs. Executive Secretary, Constitutional Commissions, or the Office of
supra. However, their positions are merely derivative; they Ombudsman, or as Secretaries, Undersecretaries,
derive their authority as agents of the authority they are Chairmen, or heads of bureaus of offices, including
representing; their power and authority is sourced from the government-owned or controlled corporations and their
power and authority of the cabinet members they are sitting subsidiaries.
for. Sans the cabinet members, they are non-entities, without
power and without personality to act in any manner with Interpreting the foregoing Constitutional provisions, this
respect to the official transactions of the NHA. The agent or Court, in Civil Liberties Union and Anti-Graft League of the
representative can only validly act and receive benefits for Philippines, Inc.,[10] held:
such action if the principal authority he is representing can
legally do so for the agent can only do so much as his principal
can do. The agent can never be larger than the principal. If the The prohibition against holding dual or multiple offices or
principal is absolutely barred from holding any position in and employment under Section 13, Article VII of the Constitution
absolutely prohibited from receiving any remuneration from the must not, however, be construed as applying to posts occupied
NHA or any government agency, for that matter, so must the by the Executive officials specified therein without additional
agent be. Indeed, the water cannot rise above its source.[9] compensation in an ex-officio capacity as provided by law and
as required by the primary functions of said officials'
office. The reason is that these posts do not comprise any
Hence, this petition. other office within the contemplation of the constitutional
Presidential Decree No. 757 is the law "Creating the prohibition but are properly an imposition of additional
National Housing Authority and dissolving the existing housing duties and functions on said officials. x x x
agencies, defining its powers and functions, providing funds
therefor, and for other purposes." Section 7 thereof provides: xxxxxxxxx

SEC. 7. Board of Directors. - The Authority shall be governed To reiterate, the prohibition under Section 13, Article VII is not
by a Board of Directors, hereinafter referred to as the Board, to be interpreted as covering positions held without additional
which shall be composed of the Secretary of Public Works, compensation in ex-officio capacities as provided by law and
Transportation and Communication, the Director-General as required by the primary functions of the concerned officials
of the National Economic and Development Authority, the office. The term ex-officio means from office; by virtue of
Secretary of Finance, the Secretary of Labor, the Secretary office. It refers to an authority derived from official
of Industry, the Executive Secretary and the General character merely, not expressly conferred upon the
Manager of the Authority. From among the members, the individual character, but rather annexed to the official
President will appoint a chairman. The members of the Board position. Ex-officio likewise denotes an act done in an official
may have their respective alternates who shall be the character, or as a consequence of office, and without any other
officials next in rank to them and whose acts shall be appointment or authority than that conferred by the
considered the acts of their principals with the right to office. An ex-officio member of a board is one who is a member
receive their benefit: Provided, that in the absence of the by virtue of his title to a certain office, and without further
warrant or appointment. To illustrate, by express provision of
FINALS CONSTITUTION I ACJUCO 37

law, the Secretary of Transportation and Communications is


the ex-officio Chairman of the Board of the Philippine Ports
Authority, and the Light Rail Transit Authority.

xxxxxxxxx

The ex-officio position being actually and in legal


contemplation part of the principal office, it follows that
the official concerned has no right to receive additional
compensation for his services in the said position. The
reason is that these services are already paid for and
covered by the compensation attached to his principal
office. It should be obvious that if, say, the Secretary of
Finance attends a meeting of the Monetary Board as an ex-
officio member thereof, he is actually and in legal
contemplation performing the primary function of
his principal office in defining policy in monetary banking
matters, which come under the jurisdiction of his
department. For such attendance, therefore, he is not
entitled to collect any extra compensation, whether it be
in the form of a per diem or an honorarium or an
allowance, or some other such euphemism. By whatever
name it is designated, such additional compensation is
prohibited by the Constitution.

xxxxxxxxx

(Emphasis ours)

Since the Executive Department Secretaries, as ex-


oficio members of the NHA Board, are prohibited from
receiving extra (additional) compensation, whether it be in the
form of a per diem or an honorarium or an allowance, or some
other such euphemism," it follows that petitioners who sit
as their alternates cannot likewise be entitled to receive
such compensation. A contrary rule would give petitioners a
better right than their principals.

We thus rule that in rendering its challenged Decision,


the COA did not gravely abuse its discretion.
WHEREFORE, the petition is DISMISSED.
SO ORDERED.
FINALS CONSTITUTION I ACJUCO 38

[G. R. No. 156982. September 8, 2004] Regulations of Proclamation No. 347), which was approved by
then President Joseph Estrada on October 19, 1999. Section
NATIONAL AMNESTY COMMISSION, petitioner, 1, Rule II thereof provides:
vs. COMMISSION ON AUDIT, JUANITO G. ESPINO,
Director IV, NCR, Commission on Audit, and ERNESTO C. Section 1, Composition The NAC shall be composed of
EULALIA, Resident Auditor, National Amnesty seven (7) members:
Commission. respondents.

a) A Chairperson who shall be appointed by the President;


DECISION

CORONA, J.: b) Three (3) Commissioners who shall be appointed by the


President;
This petition for review[1] seeks to annul the two decisions
of respondent Commission on Audit (COA) [2] dated July 26, c) Three (3) Ex-officio Members
2001[3] and January 30, 2003,[4] affirming the September 21,
1998 ruling[5] of the National Government Audit Office (NGAO).
1. Secretary of Justice
The latter in turn upheld Auditor Ernesto C. Eulalias order
2. Secretary of National Defense
disallowing the payment of honoraria to the representatives of
3. Secretary of the Interior and Local
petitioners’ ex officio members, per COA Memorandum No.
Government
97-038.

Petitioner National Amnesty Commission (NAC) is a The ex officio members


government agency created on March 25, 1994 by then may designate their representatives to the Commission.
President Fidel V. Ramos through Proclamation No. 347. The Said Representatives shall be entitled to per
NAC is tasked to receive, process and review amnesty diems, allowances, bonuses and other benefits as may be
applications. It is composed of seven members: a authorized by law. (Emphasis supplied)
Chairperson, three regular members appointed by the
President, and the Secretaries of Justice, National
Defense and Interior and Local Government as ex Petitioner invoked Administrative Order No. 2 in assailing
officio members.[6] before the COA the rulings of the resident auditor and the
NGAO disallowing payment of honoraria to the ex
It appears that after personally attending the initial NAC officio members representatives, to no avail.
meetings, the three ex officio members turned over said
responsibility to their representatives who were paid honoraria Hence, on March 14, 2003, the NAC filed the present
beginning December 12, 1994. However, on October 15, 1997, petition, contending that the COA committed grave abuse of
NAC resident auditor Eulalia disallowed on audit the discretion in: (1) implementing COA Memorandum No. 97-038
payment of honoraria to these representatives amounting without the required notice and publication under Article 2 of
to P255, 750 for the period December 12, 1994 to June 27, the Civil Code; (2) invoking paragraph 2, Section 7, Article IX-
1997, pursuant to COA Memorandum No. 97-038. On B of the 1987 Constitution to sustain the disallowance
September 1, 1998, the NGAO upheld the auditors order and of honoraria under said Memorandum; (3) applying the
notices of disallowance were subsequently issued to the Memorandum to the NAC ex officio members representatives
following:[7] who were all appointive officials with ranks below that of an
Assistant Secretary; (4) interpreting laws and rules outside of
REPRESENTATIVES AMOUNT its mandate and declaring Section 1, Rule II of Administrative
Order No. 2 null and void, and (5) disallowing the payment
of honoraria on the ground of lack of authority of
1. Cesar Averilla
representatives to attend the NAC meetings in behalf of the ex
officio members.[8]
Department of National Defense P 2,500.00
We hold that the position of petitioner NAC is against
the law and jurisprudence. The COA is correct that there is
2. Ramon Martinez
no legal basis to grant per diem, honoraria or any allowance
Department of National Defense 73,750.00
whatsoever to the NAC ex officio members official
representatives.
3. Cielito Mindaro,
Department of Justice 18,750.00 The Constitution mandates the Commission on Audit to
ensure that the funds and properties of the government are
4. Purita Deynata validly, efficiently and conscientiously used. Thus, Article IX-D
Department of Justice 62,000.00 of the Constitution ordains the COA to exercise exclusive and
broad auditing powers over all government entities or trustees,
5. Alberto Bernardo without any exception:
Department of the Interior
And Local Government 71,250.00 Section 2. (1) The Commission on Audit shall have the power,
authority and duty to examine, audit, and settle all accounts
6. Stephen Villaflor pertaining to the revenue and receipts of, and
Department of the Interior and expenditures or uses of funds and property, owned or held
Local Government 26,250.00 in trust by, or pertaining to, the Government, or any of its
subdivisions, agencies, or instrumentalities, including
government-owned and controlled corporations with original
7. Artemio Aspiras
charters, and on a post-audit basis: (a) constitutional bodies,
Department of Justice 1,250.00
commissions and offices that have been granted fiscal
P255, 750.00
autonomy under this Constitution; (b) autonomous state
Meanwhile, on April 28, 1999, the NAC passed colleges and universities; (c) other government-owned or
Administrative Order No. 2 (the new Implementing Rules and controlled corporations and their subsidiaries; and (d) such
FINALS CONSTITUTION I ACJUCO 39

non-governmental entities receiving subsidy or equity, directly disallowance and corresponding certificate
or indirectly, from or through the government, which are of settlements and balances.
required by law of the granting institution to submit to such
audit as a condition of subsidy or equity. However, where the 2. On accounts that have been audited and settled
internal control system of the audited agencies is inadequate, under certificate of settlements and balances
the Commission may adopt such measures, including on record to review and re-open said accounts,
temporary or special pre-audit, as are necessary and issue the corresponding notices of
appropriate to correct the deficiencies. It shall keep the general disallowance, and certify a new balance
accounts of the Government and, for such period as may be thereon. It is understood that the re-opening
provided by law, preserve the vouchers and other supporting of accounts shall be limited to those that
papers pertaining thereto. were settled within the prescriptive period
of three (3) years prescribed in Section 52
of P.D. 1445.
(2) The Commission shall have exclusive authority, subject to
the limitations in this Article, to define the scope of its audit 3. On disallowances previously made on these
and examination, establish the techniques and methods accounts to submit a report on the status of the
required therefor, and promulgate accounting and disallowances indicating whether those have
auditing rules and regulations, including those for the been refunded/settled or have become final
prevention and disallowance of irregular, unnecessary, and executory and the latest action taken by
inexpensive, extravagant, or unconscionable the Auditor thereon.
expenditures, or uses of government funds and
properties.
All auditors concerned shall ensure that all documents
evidencing the disallowed payments are kept intact on file in
Section 3. No law shall be passed exempting any entity of their respective offices.
the Government or its subsidiary in any guise whatever, or any
investment of public funds, from the jurisdiction of the
Any problem/issue arising from the implementation of this
Commission on Audit. (Emphasis supplied).
Memorandum shall be brought promptly to the attention of the
Committee created under COA Officer Order No. 97-698 thru
It is in accordance with this constitutional mandate that the Director concerned, for immediate resolution.
the COA issued Memorandum No. 97-038 on September 19,
1997:
An initial report on the implementation of this Memorandum
shall be submitted to the Directors concerned not later than
COMMISSION ON AUDIT MEMORANDUM NO. 97-038 October 31, 1997. Thereafter, a quarterly progress report on
the status of disallowances made shall be submitted, until all
SUBJECT: Implementation of Senate Committee Report No. the disallowances shall have been enforced.
509, Committee on Accountability of Public Officers and
Investigations and Committee on Civil Service and The Committee created under COA Office Order No. 97-698,
Government Reorganization. dated September 10, 1997, shall supervise the implementation
of this Memorandum which shall take effect immediately and
The Commission received a copy of Senate Committee Report shall submit a consolidated report thereon in response to the
No. 509 urging the Commission on Audit to immediately recommendation of the Senate Committee on Accountability of
cause the disallowance of any payment of any form of Public Officers and Investigation and Committee on Civil
additional compensation or remuneration to cabinet Service and Government Reorganization. [9] (Emphasis
secretaries, their deputies and assistants, or their supplied)
representatives, in violation of the rule on multiple
positions, and to effect the refund of any and all such Contrary to petitioners claim, COA Memorandum No. 97-
additional compensation given to and received by the 038 does not need, for validity and effectivity, the publication
officials concerned, or their representatives, from the time required by Article 2 of the Civil Code:
of the finality of the Supreme Court ruling in Civil Liberties
Union v. Executive Secretary to the present. In the Civil
Art. 2. Laws shall take effect after fifteen days following the
Liberties Union case, the Supreme Court ruled that Cabinet
completion of their publication in the Official Gazette, unless it
Secretaries, their deputies and assistants may not hold
is otherwise provided. This Code shall take effect one year
any other office or employment. It declared Executive
after such publication.
Order 284 unconstitutional insofar as it allows Cabinet
members, their deputies and assistants to hold other
offices in addition to their primary office and to receive We clarified this publication requirement
compensation therefor. The said decision became final in Taada vs. Tuvera:[10]
and executory on August 19, 1991.
[A]ll statutes, including those of local application and private
In view thereof, all unit heads/auditors/team leaders of the laws, shall be published as a condition for their effectivity,
national government agencies and government owned or which shall begin fifteen days after publication unless a
controlled corporations which have effected payment of different effectivity date is fixed by the legislature.
subject allowances, are directed to implement the
recommendation contained in the subject Senate Committee Covered by this rule are presidential decrees and
Report by undertaking the following audit action: executive orders promulgated by the President in the
exercise of legislative powers whenever the same are
1. On accounts that have not been audited and validly delegated by the legislature or, at present, directly
settled under certificate of settlements and conferred by the Constitution. Administrative rules and
balances on record from August 19, 1991 to regulations must also be published if their purpose is to
present to immediately issue the Notices of enforce or implement existing law pursuant to a valid
delegation.
FINALS CONSTITUTION I ACJUCO 40

Interpretative regulations and those merely internal in tenure, unless otherwise provided in the Constitution
nature, that is, regulating only the personnel of the itself.
administrative agency and not the public, need not be
published.Neither is publication required of the so-called
xxxxxxxxx
letters of instructions issued by administrative superiors
concerning the rules or guidelines to be followed by their
subordinates in the performance of their duties. (Emphasis Thus, while all other appointive officials in the civil service
supplied.) are allowed to hold other office or employment in the
government during their tenure when such is allowed by
law or by the primary functions of their positions,
COA Memorandum No. 97-038 is merely an internal members of the Cabinet, their deputies and assistants
and interpretative regulation or letter of instruction which may do so only when expressly authorized by the
does not need publication to be effective and valid. It is not Constitution itself. In other words, Section 7, Article IX-B
an implementing rule or regulation of a statute but a directive is meant to lay down the general rule applicable to all
issued by the COA to its auditors to enforce the self-executing elective and appointive public officials and employees,
prohibition imposed by Section 13, Article VII of the while Section 13, Article VII is meant to be the exception
Constitution on the President and his official family, their applicable only to the President, the Vice-President,
deputies and assistants, or their representatives from holding
Members of the Cabinet, their deputies and assistants.
multiple offices and receiving double compensation.
Six years prior to the issuance of COA Memorandum No. This being the case, the qualifying phrase "unless
97-038, the Court had the occasion to categorically explain this otherwise provided in this Constitution" in Section 13,
constitutional prohibition in Civil Liberties Union Article VII cannot possibly refer to the broad exceptions
vs. TheExecutive Secretary:[11] provided under Section 7, Article IX-B of the 1987
Constitution. . . .
Petitioners maintain that this Executive Order which, in effect,
allows members of the Cabinet, their undersecretaries and xxxxxxxxx
assistant secretaries to hold other government offices or
positions in addition to their primary positions, albeit subject to
The prohibition against holding dual or multiple offices or
the limitation therein imposed, runs counter to Section 13,
employment under Section 13, Article VII of the
Article VII of the 1987 Constitution, which provides as follows:
Constitution must not, however, be construed as applying
to posts occupied by the Executive officials specified
Sec. 13. The President, Vice-President, the Members of the therein without additional compensation in an ex-
Cabinet, and their deputies or assistants shall not, unless officio capacity as provided by law and as required by the
otherwise provided in this Constitution, hold any other office or primary functions of said officials' office. The reason is
employment during their tenure. They shall not, during said that these posts do no comprise "any other office" within
tenure, directly or indirectly practice any other profession, the contemplation of the constitutional prohibition but are
participate in any business, or be financially interested in any properly an imposition of additional duties and functions
contract with, or in any franchise, or special privilege granted on said officials.
by the Government or any subdivision, agency, or
instrumentality thereof, including government-owned or
xxxxxxxxx
controlled corporations or their subsidiaries. They shall strictly
avoid conflict of interest in the conduct of their office.
[T]he prohibition under Section 13, Article VII is not to be
interpreted as covering positions held without additional
xxxxxxxxx
compensation in ex-officio capacities as provided by law
and as required by the primary functions of the concerned
[D]oes the prohibition in Section 13, Article VII of the 1987 official's office. The term ex-officio means "from office; by
Constitution insofar as Cabinet members, their deputies virtue of office." It refers to an "authority derived from official
or assistants are concerned admit of the broad exceptions character merely, not expressly conferred upon the individual
made for appointive officials in general under Section 7, character, but rather annexed to the official position." Ex-officio
par. (2), Article IX-B which, for easy reference is quoted likewise denotes an "act done in an official character, or as a
anew, thus: "Unless otherwise allowed by law or by the consequence of office, and without any other appointment or
primary functions of his position, no appointive official shall authority than that conferred by the office." An ex-officio
hold any other office or employment in the Government or any member of a board is one who is a member by virtue of his title
subdivision, agency or instrumentality thereof, including to a certain office, and without further warrant or appointment.
government-owned or controlled corporation or their To illustrate, by express provision of law, the Secretary of
subsidiaries." Transportation and Communications is the ex-officio Chairman
of the Board of the Philippine Ports Authority, and the Light Rail
We rule in the negative. Transit Authority.

xxxxxxxxx xxxxxxxxx

But what is indeed significant is the fact that although Section The ex-officio position being actually and in legal
7, Article IX-B already contains a blanket prohibition contemplation part of the principal office, it follows that the
against the holding of multiple offices or employment in official concerned has no right to receive additional
the government subsuming both elective and appointive compensation for his services in the said position. The
public officials, the Constitutional Commission should reason is that these services are already paid for and
see it fit to formulate another provision, Sec. 13, Article VII, covered by the compensation attached to his principal
specifically prohibiting the President, Vice-President, office. x x x
members of the Cabinet, their deputies and assistants
from holding any other office or employment during their xxxxxxxxx
FINALS CONSTITUTION I ACJUCO 41

[E]x-officio posts held by the executive official concerned RA 6758, the Salary Standardization Law, also bars the
without additional compensation as provided by law and receipt of such additional emolument.
as required by the primary functions of his office do not
fall under the definition of "any other office" within the The representatives in fact assumed their responsibilities
contemplation of the constitutional not by virtue of a new appointment but by mere designation
prohibition... (Emphasis supplied). from the ex officio members who were themselves also
designated as such.
Judicial decisions applying or interpreting the laws or the There is a considerable difference between an
Constitution, such as the Civil Liberties Union doctrine, form appointment and designation. An appointment is the selection
part of our legal system.[12] Supreme Court decisions assume by the proper authority of an individual who is to exercise the
the same authority as valid statutes. [13] The Courts powers and functions of a given office; a designation merely
interpretation of the law is part of that law as of the date of connotes an imposition of additional duties, usually by law,
enactment because its interpretation merely establishes the upon a person already in the public service by virtue of an
contemporary legislative intent that the construed law purports earlier appointment.[15]
to carry into effect.[14]
Designation does not entail payment of additional
COA Memorandum No. 97-038 does not, in any manner benefits or grant upon the person so designated the right to
or on its own, rule against or affect the right of any individual, claim the salary attached to the position. Without an
except those provided for under the Constitution. Hence, appointment, a designation does not entitle the officer to
publication of said Memorandum is not required for it to be receive the salary of the position. The legal basis of an
valid, effective and enforceable. employees right to claim the salary attached thereto is a duly
issued and approved appointment to the position, [16] and not a
In Civil Liberties Union, we elucidated on the two mere designation.
constitutional prohibitions against holding multiple positions in
the government and receiving double compensation: (1) the Second, the ex officio members representatives are also
blanket prohibition of paragraph 2, Section 7, Article IX-B on all covered by the strict constitutional prohibition imposed on the
government employees against holding multiple government President and his official family.
offices, unless otherwise allowed by law or the primary
functions of their positions, and (2) the stricter prohibition under Again, in Civil Liberties Union, we held that cabinet
Section 13, Article VII on the President and his official family secretaries, including their deputies and assistants, who hold
from holding any other office, profession, business or financial positions in ex officio capacities, are proscribed from receiving
interest, whether government or private, unless allowed by the additional compensation because their services are already
Constitution. paid for and covered by the compensation attached to their
principal offices. Thus, in the attendance of the NAC meetings,
The NAC ex officio members representatives who the ex officio members were not entitled to, and were in fact
were all appointive officials with ranks below Assistant prohibited from, collecting extra compensation, whether it was
Secretary are covered by the two constitutional called per diem, honorarium, allowance or some other
prohibitions. euphemism. Such additional compensation is prohibited by the
Constitution.
First, the NAC ex officio members representatives are
not exempt from the general prohibition because there is no Furthermore, in de la Cruz vs. COA[17] and Bitonio vs.
law or administrative order creating a new office or position and [18]
COA, we upheld COAs disallowance of the payment
authorizing additional compensation therefor. of honoraria and per diems to the officers concerned who sat
as ex officio members or alternates. The agent, alternate or
Sections 54 and 56 of the Administrative Code of 1987 representative cannot have a better right than his principal,
reiterate the constitutional prohibition against multiple positions the ex officio member. The laws, rules, prohibitions or
in the government and receiving additional or double restrictions that cover the ex officio member apply with equal
compensation: force to his representative. In short, since the ex
officio member is prohibited from receiving additional
SEC. 54. Limitation on Appointment. (1) No elective official compensation for a position held in an ex officio capacity, so is
shall be eligible for appointment or designation in any capacity his representative likewise restricted.
to any public office or position during his tenure.
The Court also finds that the re-opening of the NAC
accounts within three years after its settlement is
xxxxxxxxx within COAs jurisdiction under Section 52 of Presidential
Decree No. 1445, promulgated on June 11, 1978:
(3) Unless otherwise allowed by law or by the primary functions
of his position, no appointive official shall hold any other office SECTION 52. Opening and revision of settled accounts. (1) At
or employment in the Government or any subdivision, agency any time before the expiration of three years after the
or instrumentality thereof, including government-owned or settlement of any account by an auditor, the Commission
controlled corporations or their subsidiaries. may motupropio review and revise the account or settlement
and certify a new balance.
xxxxxxxxx
More importantly, the Government is never estopped by
SEC. 56. Additional or Double Compensation. -- No elective or the mistake or error on the part of its agents. [19] Erroneous
appointive public officer or employee shall receive additional or application and enforcement of the law by public officers do not
double compensation unless specifically authorized by law nor preclude subsequent corrective application of the statute.
accept without the consent of the President, any present,
In declaring Section 1, Rule II of Administrative Order No.
emolument, office, or title of any kind form any foreign state.
2 s. 1999 null and void, the COA ruled that:

Pensions and gratuities shall not be considered as additional,


Petitioner further contends that with the new IRR issued by the
double or indirect compensation.
NAC authorizing the ex-officio members to designate
representatives to attend commission meetings and entitling
FINALS CONSTITUTION I ACJUCO 42

them to receive per diems, honoraria and other allowances, prohibition of the Constitution and the finality of our decision
there is now no legal impediment since it was approved by the in Civil Liberties Union prior to their receipt of such allowances.
President. This Commission begs to disagree. Said provision
in the new IRR is null and void for having been promulgated in WHEREFORE the petition is hereby DISMISSED for
excess of its rule-making authority. Proclamation No. 347, the lack of merit.
presidential issuance creating the NAC, makes no mention that
SO ORDERED.
representatives of ex-officio members can take the place of
said ex-officio members during its meetings and can receive
per diems and allowances. This being the case, the NAC, in
the exercise of its quasi-legislative powers, cannot add,
expand or enlarge the provisions of the issuance it seeks to
implement without committing an ultra vires act.[20]

We find that, on its face, Section 1, Rule II of


Administrative Order No. 2 is valid, as it merely provides that:
The ex officio members may designate their
representatives to the Commission. Said Representatives
shall be entitled to per diems, allowances, bonuses and
other benefits as may be authorized by law. (Emphasis
supplied).
The problem lies not in the administrative order but how
the NAC and the COA interpreted it.
First, the administrative order itself acknowledges
that payment of allowances to the representatives must be
authorized by the law, that is, the Constitution, statutes
and judicial decisions. However, as already discussed, the
payment of such allowances is not allowed, prohibited even.
Second, the administrative order merely allows the ex
officio members to designate their representatives to NAC
meetings but not to decide for them while attending such
meetings. Section 4 of the administrative order categorically
states:

Decisions of the NAC shall be arrived at by a majority vote in a


meeting where there is a quorum consisting of at least four
members.

Thus, although the administrative order does not


preclude the representatives from attending the NAC
meetings, they may do so only as guests or witnesses to the
proceedings. They cannot substitute for the ex
officio members for purposes of determining quorum,
participating in deliberations and making decisions.
Lastly, we disagree with NACs position that the
representatives are de facto officers and as such are entitled
to allowances, pursuant to our pronouncement in Civil Liberties
Union:

where there is no de jure officer, a de facto officer, who in good


faith has had possession of the office and has discharged the
duties pertaining thereto, is legally entitled to the emoluments
of the office, and may in appropriate action recover the salary,
fees and other compensation attached to the office.

A de facto officer derives his appointment from one


having colorable authority to appoint, if the office is an
appointive office, and whose appointment is valid on its face.
(He is) one who is in possession of an office and is discharging
its duties under color of authority, by which is meant authority
derived from an appointment, however irregular or informal, so
that the incumbent be not a mere volunteer. [21]
The representatives cannot be considered de
facto officers because they were not appointed but were
merely designated to act as such. Furthermore, they are not
entitled to something their own principals are prohibited from
receiving. Neither can they claim good faith, given the express
FINALS CONSTITUTION I ACJUCO 43

G.R. No. 88211 October 27, 1989 29.] Thus, he prays that the Motion for Reconsideration be
denied for lack of merit.
FERDINAND E. MARCOS, IMELDA R. MARCOS,
FERDINAND R. MARCOS. JR., IRENE M. ARANETA, IMEE We deny the motion for reconsideration.
M. MANOTOC, TOMAS MANOTOC, GREGORIO ARANETA,
PACIFICO E. MARCOS, NICANOR YÑIGUEZ and
1. It must be emphasized that as in all motions for
PHILIPPINE CONSTITUTION ASSOCIATION reconsideration, the burden is upon the movants, petitioner
(PHILCONSA), represented by its President, CONRADO F.
herein, to show that there are compelling reasons to reconsider
ESTRELLA, petitioners, the decision of the Court.
vs.
HONORABLE RAUL MANGLAPUS, CATALINO
MACARAIG, SEDFREY ORDOÑEZ, MIRIAM DEFENSOR 2. After a thorough consideration of the matters raised in the
SANTIAGO, FIDEL RAMOS, RENATO DE VILLA, in their motion for reconsideration, the Court is of the view that no
capacity as Secretary of Foreign Affairs, Executive compelling reasons have been established by petitioners
Secretary, Secretary of Justice, Immigration to warrant a reconsideration of the Court's decision.
Commissioner, Secretary of National Defense and Chief of
Staff, respectively, respondents. The death of Mr. Marcos, although it may be viewed as a
supervening event, has not changed the factual scenario under
RESOLUTION which the Court's decision was rendered. The threats to the
government, to which the return of the Marcoses has been
viewed to provide a catalytic effect, have not been shown
EN BANC: to have ceased. On the contrary, instead of erasing fears as
to the destabilization that will be caused by the return of the
In its decision dated September 15,1989, the Court, by a vote Marcoses, Mrs. Marcos reinforced the basis for the decision to
of eight (8) to seven (7), dismissed the petition, after finding bar their return when she called President Aquino "illegal,"
that the President did not act arbitrarily or with grave abuse of claiming that it is Mr. Marcos, not Mrs. Aquino, who is the
discretion in determining that the return of former President "legal" President of the Philippines, and declared that the
Marcos and his family at the present time and under present matter "should be brought to all the courts of the world."
circumstances pose a threat to national interest and welfare [Comment, p. 1; Philippine Star, October 4, 1989.]
and in prohibiting their return to the Philippines. On September
28, 1989, former President Marcos died in Honolulu, Hawaii. In 3. Contrary to petitioners' view, it cannot be denied that the
a statement, President Aquino said:
President, upon whom executive power is vested, has
unstated residual powers which are implied from the grant
In the interest of the safety of those who will take the death of of executive power and which are necessary for her to
Mr. Marcos in widely and passionately conflicting ways, and for comply with her duties under the Constitution. The powers
the tranquility of the state and order of society, the remains of of the President are not limited to what are expressly
Ferdinand E. Marcos will not be allowed to be brought to our enumerated in the article on the Executive Department
country until such time as the government, be it under this and in scattered provisions of the Constitution. This is so,
administration or the succeeding one, shall otherwise decide. notwithstanding the avowed intent of the members of the
[Motion for Reconsideration, p. 1; Rollo, p, 443.] Constitutional Commission of 1986 to limit the powers of
the President as a reaction to the abuses under the regime
On October 2, 1989, a Motion for Reconsideration was filed by of Mr. Marcos, for the result was a limitation of specific
petitioners, raising the following major arguments: power of the President, particularly those relating to the
commander-in-chief clause, but not a diminution of the
general grant of executive power.
1. to bar former President Marcos and his family from returning
to the Philippines is to deny them not only the inherent right of
citizens to return to their country of birth but also the protection That the President has powers other than those expressly
of the Constitution and all of the rights guaranteed to Filipinos stated in the Constitution is nothing new. This is recognized
under the Constitution; under the U.S. Constitution from which we have patterned the
distribution of governmental powers among three (3) separate
branches.
2. The President has no power to bar a Filipino from his own
country; if she has, she had exercised it arbitrarily; and
Article II, [section] 1, provides that "The Executive Power shall
be vested in a President of the United States of America." In
3. There is no basis for barring the return of the family of former Alexander Hamilton's widely accepted view, this statement
President Marcos. Thus, petitioners prayed that the Court cannot be read as mere shorthand for the specific executive
reconsider its decision, order respondents to issue the authorizations that follow it in [sections] 2 and 3. Hamilton
necessary travel documents to enable Mrs. Imelda R. Marcos, stressed the difference between the sweeping language of
Ferdinand R. Marcos, Jr., Irene M. Araneta, Imee M. Manotoc, article II, section 1, and the conditional language of article I,
Tommy Manotoc and Gregorio Araneta to return to the [section] 1: "All legislative Powers herein granted shall be
Philippines, and enjoin respondents from implementing vested in a Congress of the United States . . ." Hamilton
President Aquino's decision to bar the return of the remains of submitted that "[t]he [article III enumeration [in sections 2 and
Mr. Marcos, and the other petitioners, to the Philippines. 31 ought therefore to be considered, as intended merely to
specify the principal articles implied in the definition of
Commenting on the motion for reconsideration, the Solicitor execution power; leaving the rest to flow from the general grant
General argued that the motion for reconsideration is moot and of that power, interpreted in confomity with other parts of the
academic as to the deceased Mr. Marcos. Moreover, he Constitution...
asserts that "the 'formal' rights being invoked by the Marcoses
under the label 'right to return', including the label 'return of In Myers v. United States, the Supreme Court — accepted
Marcos' remains, is in reality or substance a 'right' to Hamilton's proposition, concluding that the federal executive,
destabilize the country, a 'right' to hide the Marcoses' incessant unlike the Congress, could exercise power from sources
shadowy orchestrated efforts at destabilization." [Comment, p.
FINALS CONSTITUTION I ACJUCO 44

not enumerated, so long as not forbidden by the


constitutional text: the executive power was given in
general terms, strengthened by specific terms where
emphasis was regarded as appropriate, and was limited
by direct expressions where limitation was needed. . ." The
language of Chief Justice Taft in Myers makes clear that the
constitutional concept of inherent power is not a synonym
for power without limit; rather, the concept suggests only that
not all powers granted in the Constitution are themselves
exhausted by internal enumeration, so that, within a sphere
properly regarded as one of "executive' power, authority is
implied unless there or elsewhere expressly limited. [TRIBE,
AMERICAN CONSTITUTIONAL LAW 158-159 (1978).]

And neither can we subscribe to the view that a recognition of


the President's implied or residual powers is tantamount to
setting the stage for another dictatorship. Despite petitioners'
strained analogy, the residual powers of the President under
the Constitution should not be confused with the power of the
President under the 1973 Constitution to legislate pursuant to
Amendment No. 6 which provides:

Whenever in the judgment of the President (Prime Minister),


there exists a grave emergency or a threat or imminence
thereof, or whenever the interim Batasang Pambansa or the
regular National Assembly fails or is unable to act adequately
on any matter for any reason that in his judgment requires
immediate action, he may, in order to meet the exigency, issue
the necessary decrees, orders, or letters of instruction, which
shall form part of the law of the land,

There is no similarity between the residual powers of the


President under the 1987 Constitution and the power of the
President under the 1973 Constitution pursuant to Amendment
No. 6. First of all, Amendment No. 6 refers to an express grant
of power. It is not implied. Then, Amendment No. 6 refers to a
grant to the President of the specific power of legislation.

4. Among the duties of the President under the Constitution, in


compliance with his (or her) oath of office, is to protect and
promote the interest and welfare of the people. Her
decision to bar the return of the Marcoses and
subsequently, the remains of Mr. Marcos at the present
time and under present circumstances is in compliance
with this bounden duty. In the absence of a clear showing
that she had acted with arbitrariness or with grave abuse
of discretion in arriving at this decision, the Court will not
enjoin the implementation of this decision.

ACCORDINGLY, the Court resolved to DENY the Motion for


Reconsideration for lack of merit."
FINALS CONSTITUTION I ACJUCO 45

G.R. No. 189028 July 16, 2013 perceived to evoke and produce a spirit of harmony. Art is also
considered as a civilizing force, a catalyst of nation-building.
NATIONAL ARTIST FOR LITERATURE VIRGILIO The notion of art and artists as privileged expressions of
national culture helped shape the grand narratives of the
ALMARIO, NATIONAL ARTIST FOR LITERATURE
BIENVENIDO LUMBERA, NATIONAL ARTIST FOR VISUAL nation and shared symbols of the people. The artist does not
ARTS (PAINTING) BENEDICTO CABRERA, NATIONAL simply express his/her own individual inspiration but articulates
ARTIST FOR VISUAL ARTS (SCULPTURE) NAPOLEON the deeper aspirations of history and the soul of the
ABUEVA, NATIONAL ARTIST FOR VISUAL ARTS people.2 The law recognizes this role and views art as
(PAINTING AND SCULPTURE) ARTURO LUZ, NATIONAL something that "reflects and shapes values, beliefs,
ARTIST FOR PRODUCTION DESIGN SALVADOR BERNAL, aspirations, thereby defining a people’s national
UNIVERSITY PROFESSOR EMERITUS GEMINO ABAD, identity."3If unduly politicized, however, art and artists could
DEAN MARVIC M.V.F. LEONEN (UP COLLEGE OF LAW), stir controversy and may even cause discord, as what
happened in this case.
DEAN DANILO SILVESTRE (UP COLLEGE OF
ARCHITECTURE), DEAN ROLAND TOLENTINO (UP
COLLEGE OF MASS COMMUNICATION), PROF. JOSE The Antecedents
DALISAY, DR. ANTON JUAN, DR. ALEXANDER CORTEZ,
DR. JOSE NEIL GARCIA, DR. PEDRO JUN CRUZ REYES,
History of the Order of National Artists
PROF. JOSE CLAUDIO GUERRERO, PROF. MICHAEL M.
COROZA, PROF. GERARD LICO, PROF. VERNE DE LA
PENA, PROF. MARIAN ABUAN, PROF. THEODORE O. TE, On April 27, 1972, former President Ferdinand E. Marcos
DR. CRISTINA PANTOJA-HIDALGO, PROF. JOSE issued Proclamation No. 10014 and, upon recommendation
WENDELL CAPILI, PROF. SIR ANRIAL TIATCO, PROF. of the Board of Trustees of the Cultural Center of the
NICOLO DEL CASTILLO, PROF. HORACIO DUMANLIG, Philippines (CCP), created the category of Award and
PROF. DANTON REMOTO, PROF. PRISCELINA Decoration of National Artist to be awarded to Filipinos who
PATAJOLEGASTO, PROF. BELEN CALINGACION, PROF. have made distinct contributions to arts and letters. In the same
AMIEL Y. LEONARDIA, PROF. VIM NADERA, PROF. issuance, Fernando Amorsolo was declared as the first
MARILYN CANTA, PROF. CECILIA DELA PAZ, ROF. National Artist.
CHARLSON ONG, PROF. CLOD MARLON YAMBAO,
PROF. KENNETH JAMANDRE, PROF. JETHRO JOAQUIN, On May 15, 1973, Proclamation No. 11445 was issued. It
ATTY. F.D. NICOLAS B. PICHAY, ATTY. ROSE BEATRIX amended Proclamation No. 1001 "by creating a National
ANGELES, MR. FERNANDO JOSEF, MS. SUSAN S. LARA, Artists Awards Committee" that would "administer the
MR. ALFRED YUSON, MS. JING PANGANIBANMENDOZA, conferment of the category of National Artist" upon deserving
MR. ROMULO BAQUIRAN, JR., MR. CARLJOE JAVIER, Filipino artists. The Committee, composed of members of the
MS. REBECCA T. ANONUEVO, MR. JP ANTHONY D. Board of Trustees of the CCP, was tasked to "draft the rules to
CUNADA, MS. LEAH NAVARRO, MR. MARK MEILLY, MR. guide its deliberations in the choice of National Artists, to the
VERGEL O. SANTOS, MR. GIL OLEA MENDOZA, MR. end that those who have created a body of work in the arts and
EDGAR C. SAMAR, MS. CHRISTINE BELLEN, MR. letters capable of withstanding the test of time will be so
ANGELO R. LACUESTA, MS. ANNA MARIA recognized."
KATIGBAKLACUESTA, MR. LEX LEDESMA, MS. KELLY
PERIQUET, MS. CARLA PACIS, MR. J. ALBERT GAMBOA,
MR. CESAR EVANGELISTA BUENDIA, MR. PAOLO The authority of the National Artists Awards Committee to
ALCAZAREN, MR. ALWYN C. JAVIER, MR. RAYMOND administer the conferment of the National Artist Award was
MAGNO GARLITOS, MS. GANG BADOY, MR. LESLIE again reiterated in Presidential Decree No. 2086 issued on
BOCOBO, MS. FRANCES BRETANA, MS. JUDITH June 7, 1973.
TORRES, MS. JANNETTE PINZON, MS. JUNE POTICAR-
DALISAY, MS. CAMILLE DE LA ROSA, MR. JAMES On April 3, 1992, Republic Act No. 7356, otherwise known as
LADIORAY, MR. RENATO CONSTANTINO, JR., and the Law Creating the National Commission for Culture and
CONCERNED ARTISTS OF THE PHILIPPINES the Arts, was signed into law. It established the National
(CAP), Petitioners, Commission for Culture and the Arts (NCCA) and gave it an
vs. extensive mandate over the development, promotion and
THE EXECUTIVE SECRETARY, THE SECRETARY OF THE preservation of the Filipino national culture and arts and
DEPARTMENT OF BUDGET AND MANAGEMENT, THE the Filipino cultural heritage. The NCCA was tasked with the
CULTURAL CENTER OF THE PHILIPPINES, THE following:
NATIONAL COMMISSION ON CULTURE AND THE ARTS,
MS. CECILE GUIDOTE-ALVAREZ, MR. CARLO MAGNO
JOSE CAPARAS,1 MR. JOSE MORENO, MR. FRANCISCO Sec. 8. The Commission. – A National Commission for Culture
MANOSA, AND ALL PERSONS, PUBLIC AND PRIVATE, and Arts is hereby created to formulate policies for the
ACTING UNDER THEIR INSTRUCTIONS, DIRECTION, development of culture and arts; implement these policies in
CONTROL AND SUPERVISION IN RELATION TO THE coordination with affiliated cultural agencies; coordinate the
CONFERMENT OF THE ORDER OF THE NATIONAL implementation of programs of these affiliated agencies;
ARTIST AND THE RELEASE OF FUNDS IN RELATION TO administer the National Endowment Fund for Culture and Arts
THE CONFERMENT OF THE HONORS AND PRIVILEGES (NEFCA); encourage artistic creation within a climate of artistic
OF THE ORDER OF NATIONAL ARTISTS ON freedom; develop and promote the Filipino national culture and
RESPONDENTS GUIDOTE-ALVAREZ, CAPARAS, arts; and preserve Filipino cultural heritage. The Commission
MORENO AND MANOSA, Respondents. shall be an independent agency. It shall render an annual
report of its activities and achievements to the President
and to Congress.
DECISION
Among the specific mandates of the NCCA under Republic Act
LEONARDO-DE CASTRO, J.: No. 7356 is to "extend recognition of artistic achievement
through awards, grants and services to artists and cultural
Art has traditionally been viewed as the expression of groups which contribute significantly to the Filipino’s
everything that is true, good and beautiful. As such, it is cultural legacy."7 In connection with this mandate, the NCCA
FINALS CONSTITUTION I ACJUCO 46

is vested with the power to "advise the President on matters (a) should have achieved
pertaining to culture and the arts, including the creation of a authority, credibility and track
special decoration or award, for persons who have significantly record in his field(s) of expertise;
contributed to the development and promotion of Philippine
culture and arts."8 (b) should have extensive
knowledge in his field(s) and his
As both the CCP Board of Trustees and the NCCA have been views on Philippine art and
mandated by law to promote, develop and protect the culture must be national in
Philippine national culture and the arts, and authorized to give perspective;
awards to deserving Filipino artists, the two bodies decided to
team up and jointly administer the National Artists (c) should be a recognized
Award.9 Thereafter, they reviewed the guidelines for the authority in the study or research
nomination, selection and administration of the National Artists of Philippine art and culture;
Award. Pursuant to their respective powers to draft and
promulgate rules, regulations and measures to guide them in
their deliberations in the choice of National Artists, the CCP (d) must be willing to devote
and NCCA adopted the following revised guidelines in sufficient time and effort to the
September 200710: work of the Council;

4. ADMINISTRATION OF THE AWARD (e) must be willing to sign a non-


disclosure statement in order to
safeguard the confidentiality of
4.1. The National Commission for Culture the deliberations;
and the Arts (NCCA) shall plan, organize
and implement the Order of National Artists
in coordination with the Cultural Center of (f) must not have been convicted
the Philippines (CCP). with finality of any crime by a
court of justice or dismissed for
cause by any organization,
4.2. It shall enlist the support and whether public or private.
cooperation of private sector experts from
the various fields of art to ensure that the
awards are implemented in a successful 4.7. The National Artist Award Council of
and impartial manner. Experts shall be composed of a maximum
of seven (7) members each of the seven (7)
areas/disciplines. The living National
4.3. The National Artist Award Secretariat Artists will automatically become members
shall commission art experts to form a
in addition to the forty-nine (49) selected
Special Research Group who shall verify members. These members will constitute
information submitted on nominees and
the first deliberation panel and will be
provide essential data. invited to evaluate the nominations and
materials submitted by the Special
They shall be selected for their Research Group.
specialization and familiarity with the works
and accomplishments of nominated artists. 4.8. Any member of the Council of Experts
who is nominated or related to a nominee
4.4. The Special Research Group shall be up to the fourth degree of consanguinity or
composed of ten (10) to twenty (20) affinity shall inhibit himself/herself from the
members who have expertise in one or deliberation process. Likewise, any
more fields or disciplines. member may decline to participate in the
deliberation for any reason or may be
4.5. The National Artist Award Council of removed for just cause upon
Experts shall be created before or during recommendation to the NCCA Board by at
the nomination period. It is tasked to screen least two thirds (2/3) of the members; in
nominees and recommend to the NCCA which case, the National Artist Award
and CCP Boards the candidates for the Secretariat shall again select the
replacements for those who decline or
Order of National Artists. It shall be
composed of highly regarded peers, resigned until the first deliberation panel is
completed.
scholars, (including cultural philosophers
and historians), academicians,
researchers, art critics, and other 4.9. The list of nominated members of the
knowledgeable individuals. A wider age- National Artist Award Council of Experts
range of experts who would have first-hand shall be reviewed by the National Artist
knowledge of achievements of nominees Award Secretariat as needed, for purposes
shall be considered. of adding new members or replacements.

4.6. The selection of the members of the 4.10. The members of the National Artist
National Artist Award Council of Award Council of Experts shall serve for a
Experts shall be based on the following fixed term of three (3) years.
criteria:
FINALS CONSTITUTION I ACJUCO 47

5. CRITERIA FOR SELECTION 6.2.5. Literature – poetry, fiction (short


story, novel and play); non-fiction (essay,
The Order of National Artists shall be given journalism, literary criticism and historical
literature).
to:

6.2.6. Film and Broadcast Arts – direction,


5.1 Living artists who are Filipino citizens at
the time of nomination, as well as those writing, production design,
cinematography, editing, camera work,
who died after the establishment of the
award in 1972 but were Filipino citizens at and/or performance.
the time of their death.
6.2.7. Architecture, Design and Allied Arts
– architecture design, interior design,
5.2 Artists who through the content and
form of their works have contributed in industrial arts design, landscape
architecture and fashion design.
building a Filipino sense of nationhood.

6.3. Nominations for the Order of National


5.3. Artists who have pioneered in a mode
of creative expression or style, thus, Artists may be submitted by government
and non-government cultural organizations
earning distinction and making an impact
on succeeding generations of artists. and educational institutions, as well as
private foundations and councils.

5.4. Artists who have created a substantial


6.4. Members of the Special Research
and significant body of works and/or
consistently displayed excellence in the Group, as well as agencies attached to the
practice of their art form thus enriching NCCA and CCP shall not submit
artistic expression or style. nominations.

5.5 Artists who enjoy broad acceptance 6.5. NCCA and CCP Board members and
through: consultants and NCCA and CCP officers
and staff are automatically disqualified from
being nominated.
5.5.1. prestigious national and/or
international recognition, such as
6.6. Nominations shall be accepted only
the Gawad CCP Para sa Sining,
CCP Thirteen Artists Award and when these are submitted in writing and
NCCA Alab ng Haraya with proper supporting documentation, as
follows:

5.5.2. critical acclaim and/or


reviews of their works 6.6.1. A cover letter signed by the
head or designated
representative of the nominating
5.5.3. respect and esteem from organization.
peers.
The cover letter shall be
6. NOMINATION PROCEDURE accompanied by a Board
Resolution approving the
6.1. The National Artist Award Secretariat nominee concerned with the said
shall announce the opening of nominations resolution signed by the
through media releases and letters to organization President and duly
qualified organizations. certified by the Board Secretary.

6.2. Candidates may be nominated under 6.6.2. A duly accomplished


one or more of the following categories: nomination form;

6.2.1. Dance – choreography, direction 6.6.3. A detailed curriculum vitae


and/or performance. of the nominee;

6.2.2. Music – composition, direction, 6.6.4. A list of the nominee’s


and/or performance. significant works categorized
according to the criteria;
6.2.3. Theater – direction, performance
and/or production design. 6.6.5. The latest photograph
(color or black and white) of the
nominee, either 5" x 7" or 8" x 11";
6.2.4. Visual Arts – painting, sculpture,
printmaking, photography, installation art,
mixed media works, illustration, 6.6.6. Pertinent information
comics/komiks, graphic arts, performance materials on the nominee’s
art and/or imaging. significant works (on CDs, VCDs
and DVDs);
FINALS CONSTITUTION I ACJUCO 48

6.6.7. Copies of published criteria of the Order of National Artists. In


reviews; extreme cases, the Second Deliberation
may add new names to the lists.
6.6.8. Any other document that
may be required. 7.6. The second deliberation panel may
recommend not to give award in any
6.7. Nominations received beyond the category if no nominee is found deserving.
The number of awardees shall also depend
announced deadline for the submission of
nominations shall not be considered. on the availability of funds. All decisions
and recommendations shall be in writing.

6.8. The National Artist Award Secretariat


7.7. The recommendations from the
shall announce the opening of nominations
through media releases. Second Deliberation Panel of the National
Artist Award Council of Experts shall then
be presented to the joint boards of NCCA
6.9. All inquiries and nominations shall be and CCP for final selection. The presentors
submitted to shall prepare their presentation in writing
together with an audio-visual presentation
The NATIONAL ARTIST AWARD SECRETARIAT or powerpoint presentation. Written
interpellations/opinions will be accepted
from selected critics. The review shall be
Office of the Artistic Director Cultural Center of the Philippines based on the ranking done by the Second
Roxas Boulevard, 1300 Pasay City or The NATIONAL ARTIST Deliberation. The voting shall be across
AWARD SECRETARIAT Office of the Deputy Executive disciplines. The National Artists will be
Director National Commission for Culture and the Arts 633 given the option whether to vote on all
General Luna Street, Intramuros, Manila categories or on his/her particular
discipline.
7. SCREENING AND SELECTION PROCESS
7.8. Proxy votes will not be allowed in the
7.1. The National Artist Award Secretariat Selection Process. Designation of
shall pre-screen the nominees based on permanent representatives of agencies
technical guideline items 5.1, 6.2, 6.3, 6.4, should be made at the outset to make them
6.5 and 6.6. The pre-screening shall not be regular Board members of NCCA and thus,
based on the accomplishments and merits may be allowed to cast votes.
of the nominee.
7.9. The list of awardees shall be submitted
7.2. The Special Research Group shall to the President of the Republic of the
accomplish its task within six (6) months. Philippines for confirmation, proclamation
The main objective is to verify the validity of and conferral.
the data, and evaluate the quality, true
value and significance of works according 8. PRESENTATION OF THE AWARDS
to the criteria. It shall come up with the
updated and comprehensive profiles of
nominees reflecting their most outstanding 8.1. The Order of National Artists shall not
achievements. be conferred more frequently than every
three (3) years.
7.3. The National Artist Award Secretariat
will meet to review the list of nominees for 8.2. The Order of National Artists shall be
oversights. Consequently, deserving conferred by the President of the
nominees shall be added to the list. Philippines on June 11 or any appropriate
date in fitting ceremonies to be organized
by the National Artist Secretariat.
7.4. The first deliberation panel (Council of
Experts) shall be intra-disciplinary. The
panelists shall be grouped according to 8.3. The medallion of the Order of National
their respective fields of expertise or Artists and citation shall be given to the
disciplines to shortlist the nominees in their honoree during the conferment ceremony.
disciplines or categories for presentation to The cash award of ₱100,000.00 in cheque
the second deliberation panel. shall be given immediately after the
ceremony or at another time and place as
requested by the honoree.
7.5. The second deliberation panel shall be
composed of a different set of experts from
the first deliberation panel three (3) experts 8.4. A posthumous conferral consisting of
each of the seven (7) areas/discipline and the medallion and citation shall be given to
may include members from varying the family or legal heir/s of the honoree.
backgrounds such as critics and The cash award of ₱75,000.00 in cheque
academicians. The achievements of each shall be given to the honoree’s legal heir/s
shortlisted nominee shall be presented by or a representative designated by the
one designated member of Council of family immediately after the ceremony or at
Experts. Then panel deliberates and ranks another time and place as requested by the
the shortlisted nominees according to the family. (Emphases supplied.)
order of precedence following the set
FINALS CONSTITUTION I ACJUCO 49

In 1996, the NCCA and the CCP created a National Artist The Committee on Honors shall, as a general rule, serve as a
Award Secretariat composed of the NCCA Executive Director screening committee to ensure that nominations received
as Chairperson, the CCP President as Vice-Chairperson, and from the various awards committees meet two tests: that
the NCCA Deputy Executive Director, the CCP Vice- there has not been an abuse of discretion in making the
President/Artistic Director, the NCCA National Artist Award nomination, and that the nominee is in good standing. Should
Officer and the CCP National Artist Award Officer as members. a nomination meet these criteria, a recommendation to the
They also centralized with the NCCA all financial resources President for conferment shall be made.
and management for the administration of the National Artists
Award. They added another layer to the selection process to The President of the Philippines takes the
involve and allow the participation of more members of the arts recommendations of the Committee on Honors in the
and culture sector of the Philippines in the selection of who highest consideration when making the final decision on
may be proclaimed a National Artist. the conferment of awards. (Emphasis supplied.)

On September 19, 2003, Executive Order No. 236, s. 2003, Executive Order No. 435, s. 2005, entitled Amending Section
entitled Establishing the Honors Code of the Philippines to 5(IV) of Executive Order No. 236 Entitled "Establishing the
Create an Order of Precedence of Honors Conferred and Honors Code of the Philippines to Create an Order of
for Other Purposes, was issued. The National Artists Award
Precedence of Honors Conferred and for Other Purposes"
was renamed the Order of National Artists and raised to the was subsequently issued on June 8, 2005. It amended the
level of a Cultural Order, fourth in precedence among the
wording of Executive Order No. 236, s. 2003, on the Order of
orders and decorations that comprise the Honors of the National Artists and clarified that the NCCA and the CCP "shall
Philippines. Executive Order No. 236, s. 2003, recognizes the advise the President on the conferment of the Order of National
vital role of the NCCA and the CCP in identifying Filipinos who Artists."
have made distinct contributions to arts and letters and states
that the National Artist recognition is conferred "upon the
recommendation of the Cultural Center of the Philippines and Controversy Surrounding the 2009 Order of National
the National Commission for Culture and the Arts."12 Executive Artists
Order No. 236, s. 2003, further created a Committee on Honors
to "assist the President in evaluating nominations for Petitioners alleged that on January 30, 2007, a joint meeting
recipients of Honors,"13 including the Order of National of the NCCA Board of Commissioners and the CCP Board of
Artists, and presidential awards. The Committee on Honors Trustees was held to discuss, among others, the evaluation of
has been allowed to "authorize relevant department or the 2009 Order of National Artists and the convening of the
government agencies to maintain Honors and/or Awards National Artist Award Secretariat. The nomination period
Committees to process nominations for Honors and/or was set for September 2007 to December 31, 2007, which was
Presidential Awards."14 In this connection, Section 2.4(A) of later extended to February 28, 2008. The pre-screening of
the Implementing Rules and Regulations 15 of Executive Order nominations was held from January to March 2008.16
No. 236, s. 2003, states:
On April 3, 2009, the First Deliberation Panel met. 17 A total of
2.4: Awards Committees 87 nominees18 were considered during the deliberation and a
preliminary shortlist19 of 32 names was compiled.
There shall be two types of awards committees: the
Committee on Honors and the various awards committees in On April 23, 2009, the Second Deliberation Panel purportedly
the various units of the government service. composed of an entirely new set of Council of Experts met and
shortlisted 13 out of the 32 names in the preliminary
A. The Committee on Honors shortlist.20 On May 6, 2009, the final deliberation was
conducted by the 30-member Final Deliberation Panel
comprised of the CCP Board of Trustees and the NCCA Board
The Committee on Honors serves as a National Awards
of Commissioners and the living National Artists. 21 From the 13
Committee. It is composed of the following: names in the second shortlist, a final list of four names was
agreed upon.22 The final list, according to rank, follows:
The Executive Secretary, Chairman

Name Art Field/Category Number of


The Secretary of Foreign Affairs, Vice-Chairman Votes

Head, Presidential Management Staff, member Manuel Conde (+) Film and Broadcast Arts 26
(Film)
Presidential Assistant for Historical Affairs, member Ramon Santos Music 19

Lazaro Francisco Literature 15


Chief of Presidential Protocol, member (+)

Chief of Protocol, DFA, member Federico Aguilar- Visual Arts 15


Alcuaz
All nominations from the various awards committees must be
submitted to the Committee on Honors via the Chancellery of On May 6, 2009, a letter, signed jointly by the Chairperson of
Philippine Orders and State Decorations. The Chancellery the NCCA, Undersecretary Vilma Labrador, and the President
shall process nominations for the consideration of the and Artistic Director of the CCP, Mr. Nestor Jardin, was sent to
Committee on Honors. The Committee on Honors shall screen the President.23 The letter stated, thus:
and recommend these nominations to the President.
FINALS CONSTITUTION I ACJUCO 50

May 6, 2009 The Committee on Honors thereafter submitted a


memorandum to then President Gloria Macapagal-Arroyo
Her Excellency GLORIA MACAPAGAL-ARROYO recommending the conferment of the Order of National Artists
on the four recommendees of the NCCA and the CCP Boards,
President of the Philippines
Malacañan Palace, Manila as well as on private respondents Guidote-Alvarez, Caparas,
Mañosa and Moreno. Acting on this recommendation,
Proclamation No. 1823 declaring Manuel Conde a National
Subject: 2009 Order of National Artist Awardees Artist was issued on June 30, 2009. Subsequently, on July 6,
2009, Proclamation Nos. 1824 to 1829 were issued
Dear President Arroyo: declaring Lazaro Francisco, Federico Aguilar Alcuaz and
private respondents Guidote-Alvarez, Caparas, Mañosa
and Moreno, respectively, as National Artists. This was
We are respectfully submitting a recommendation of the NCCA subsequently announced to the public by then Executive
Board of Trustees and CCP Board of Trustees for the Secretary Eduardo Ermita on July 29, 2009. 26
Proclamation of the following as 2009 Order of National Artists:
Convinced that, by law, it is the exclusive province of the
1. Mr. MANUEL CONDE+ (Posthumous) – NCCA Board of Commissioners and the CCP Board of
Film and Broadcast Arts Trustees to select those who will be conferred the Order
of National Artists and to set the standard for entry into that
2. Dr. RAMON SANTOS – Music select group, petitioners instituted this petition for prohibition,
certiorari and injunction (with prayer for restraining order)
praying that the Order of National Artists be conferred on
3. Mr. LAZARO FRANCISCO+ Dr. Santos and that the conferment of the Order of
(Posthumous) – Literature National Artists on respondents Guidote-Alvarez,
Caparas, Mañosa and Moreno be enjoined and declared to
4. Mr. FEDERICO AGUILAR-ALCUAZ – have been rendered in grave abuse of discretion.27
Visual Arts
In a Resolution28 dated August 25, 2009, the Court issued a
The above persons were identified by experts in the status quo order29 enjoining "public respondents" "from
various fields of arts and culture, including living National conferring the rank and title of the Order of National Artists on
Artists. An intensive selection process was observed following private respondents; from releasing the cash awards that
established practice. In the past, awards were presented by accompany such conferment and recognition; and from
the President at a Ceremony held at the Malacañan Palace holding the acknowledgment ceremonies for recognition of the
followed by a program called "Parangal" at the Cultural Center private respondents as National Artists."
of the Philippines. We also propose to continue with past
practice of celebrating the life and works of the four (4) Order What is the nature and scope of the power of the President
of National Artists through an exhibit that will open and a to confer the Order of the National Artists and how should
commemorative publication that will be released on the day of it be exercised? This is the essential issue presented in this
the proclamation. case. It will determine whether the proclamation of
respondents as National Artists is valid. Preliminary procedural
We respectfully suggest, subject to Her Excellency’s issues on the standing of the petitioners and the propriety of
availability, that the Proclamation be on June 11, 2009, if the remedies taken,30 however, call for resolution as a
possible at the Malacañan Palace. prerequisite to the discussion of the main question.

Thank you for your kind attention. Contention of the Parties

Very respectfully yours, A perusal of the pleadings submitted by the petitioners reveals
that they are an aggrupation of at least three groups, the
National Artists, cultural workers and academics, and the
(Sgd.)
Concerned Artists of the Philippines (CAP). The National
VILMA L. LABRADOR
Artists assert an "actual as well as legal interest in maintaining
Chairman
the reputation of the Order of National Artists."31 In particular,
National Commission for Culture and the Arts
they invoke their right to due process not to have the honor
they have been conferred with diminished by the irregular and
(Sgd.) questionable conferment of the award on respondents
NESTOR O. JARDIN Guidote-Alvarez, Caparas, Mañosa and Moreno. For
President and Artistic Director petitioners, this would adversely affect their right to live a
Cultural Center of the Philippines 24 meaningful life as it detracts not only from their right to
enjoy their honor as a fruit of their lifelong labor but also
According to respondents, the aforementioned letter was from the respect of their peers.32
referred by the Office of the President to the Committee on
Honors. Meanwhile, the Office of the President allegedly The cultural workers, academics and CAP claim to be Filipinos
received nominations from various sectors, cultural who are deeply concerned with the preservation of the
groups and individuals strongly endorsing private country’s rich cultural and artistic heritage. As taxpayers, they
respondents Cecile Guidote-Alvarez, Carlo Magno Jose are concerned about the use of public monies for illegal
Caparas, Francisco Mañosa and Jose Moreno. The appointments or spurious acts of discretion. 33
Committee on Honors purportedly processed these
nominations and invited resource persons to validate the
All of the petitioners claim that former President Macapagal-
qualifications and credentials of the nominees. 25
Arroyo gravely abused her discretion in disregarding the
results of the rigorous screening and selection process for the
Order of National Artists and in substituting her own choice for
FINALS CONSTITUTION I ACJUCO 51

those of the Deliberation Panels. According to petitioners, CCP Boards is simply to advise the President. The award of
the President’s discretion to name National Artists is not the Order of National Artists is the exclusive prerogative
absolute but limited. In particular, her discretion on the matter of the President who is not bound in any way by the
cannot be exercised in the absence of or against the recommendation of the NCCA and the CCP Boards. The
recommendation of the NCCA and the CCP. In adding the implementing rules and regulations or guidelines of the NCCA
names of respondents Caparas, Guidote-Alvarez, Mañosa and cannot restrict or limit the exclusive power of the President to
Moreno while dropping Dr. Santos from the list of conferees, select the recipients of the Order of National Artists. 39
the President’s own choices constituted the majority of the
awardees in utter disregard of the choices of the NCCA and For her part, in a letter40 dated March 11, 2010, respondent
the CCP and the arts and culture community which were Guidote-Alvarez manifested that she was waiving her right to
arrived at after a long and rigorous process of screening and file her comment on the petition and submitted herself to the
deliberation. Moreover, the name of Dr. Santos as National Court’s discretion and wisdom.
Artist for Music was deleted from the final list submitted by the
NCCA and the CCP Boards without clearly indicating the basis
thereof. For petitioners, the President’s discretion to name Respondent Mañosa manifested that his creations speak for
National Artists cannot be exercised to defeat the themselves as his contribution to Filipino cultural heritage and
recommendations made by the CCP and NCCA Boards after his worthiness to receive the award. Nonetheless, he
a long and rigorous screening process and with the benefit of expressed his conviction that the Order of National Artists is
expertise and experience. The addition of four names to the not a right but a privilege that he would willingly relinquish
final list submitted by the Boards of the CCP and the should he be found not worthy of it. 41
NCCA and the deletion of one name from the said list
constituted a substitution of judgment by the President Respondent Moreno did not file any pleading despite being
and a unilateral reconsideration without clear justification given several opportunities to do so. Hence, the Court
of the decision of the First, Second and Final Deliberation dispensed with his pleadings.42
Panels composed of experts.34
In a Resolution dated July 12, 2011, this Court gave due
Petitioners further argue that the choice of respondent course to the petition and required the parties to file their
GuidoteAlvarez was illegal and unethical because, as the then respective memoranda.43 Respondent Caparas filed his
Executive Director of the NCCA and presidential adviser on memorandum on September 8, 2011,44 the CCP filed its
culture and arts, she was disqualified from even being memorandum on September 19, 2011,45 respondent Mañosa
nominated.35 Moreover, such action on the part of the former on September 20, 2011,46 and the Office of the Solicitor
President constituted grave abuse of discretion as it gave General filed a manifestation stating that it is adopting its
preferential treatment to respondent Guidote-Alvarez by comment as its memorandum on September 21,
naming the latter a National Artist despite her not having been 2011.47 Respondent Moreno failed to file a Memorandum,
nominated and, thus, not subjected to the screening process hence, the Court resolved to dispense with the
provided by the rules for selection to the Order of National same.48Petitioners filed their Memorandum on May 14, 2012. 49
Artists. Her inclusion in the list by the President represented a
clear and manifest favor given by the President in that she was
exempted from the process that all other artists have to On the other hand, the original position of the Office of the
undergo. According to petitioners, it may be said that the Solicitor General (OSG) was similar to that of respondent
President used a different procedure to qualify respondent Caparas.50 In a subsequent manifestation,51 however, the
Guidote-Alvarez. This was clearly grave abuse of discretion for OSG stated that the current Board of Commissioners of the
being manifest and undue bias violative of the equal protection NCCA agree with the petitioners that the President cannot
clause.36 honor as a National Artist one who was not recommended by
the joint Boards of the NCCA and the CCP. The implementing
rules and regulations of Executive Order No. 236, s. 2003,
Respondent Caparas refutes the contention of the petitioning recognized the binding character of the recommendation of the
National Artists and insists that there could be no prejudice to NCCA and the CCP Boards and limited the authority of the
the latter. They remain to be National Artists and continue to Committee on Honors to the determination that (1) there has
receive the emoluments, benefits and other privileges been no grave abuse of discretion on the part of the NCCA and
pertaining to them by virtue of that honor. On the other hand, the CCP Boards in making the nomination, and (2) the
all the other petitioners failed to show any material and nominee is in good standing. Where a nomination meets the
personal injury or harm caused to them by the conferment of said two criteria, a recommendation to the President to confer
the Order of National Artists on respondents Guidote-Alvarez, the award shall be made.52
Caparas, Mañosa and Moreno. The rule on standing may not
be relaxed in favor of the petitioners as no question of
constitutionality has been raised and no issue of The OSG further argued that, while the President exercises
transcendental importance is involved.37 control over the NCCA and the CCP, the President has the
duty to faithfully execute the laws, including the NCCA-CCP
guidelines for selection of National Artists and the
Respondent Caparas further argues that the remedies of implementing rules of Executive Order No. 236, s. 2003.
prohibition and injunction are improper as the act sought to be Moreover, the laws recognize the expertise of the NCCA and
enjoined – the declaration of respondents Guidote-Alvarez, the CCP in the arts and tasked them to screen and select the
Caparas, Mañosa and Moreno as National Artists – had artists to be conferred the Order of National Artists. Their
already been consummated. In particular, respondent Caparas mandate is clear and exclusive as no other agency possesses
was already proclaimed National Artist through Proclamation such expertise.53
No. 1827 issued on July 6, 2009.38
The OSG also assailed the former President’s choice of
On the merits, respondent Caparas contends that no grave respondent Guidote-Alvarez for being contrary to Republic Act
abuse of discretion attended his proclamation as National No. 7356.54 Section 11 of the said law provides:
Artist. The former President considered the respective
recommendations of the NCCA and the CCP Boards and of
the Committee on Honors in eventually declaring him Sec. 11. Membership Restrictions. – During his/her term as
(Caparas) as National Artist. The function of the NCCA and the member of the Commission, a Commissioner shall not be
FINALS CONSTITUTION I ACJUCO 52

eligible for any grant, or such other financial aid from the Among the other petitioners, Prof. Gemino Abad presents a
Commission as an individual: Provided, however, That he/she unique valid personal and substantial interest. Like
may compete for grants and awards on the same level as other respondents Caparas, Mañosa and Moreno, he was among
artists one (1) year after his/her term shall have expired. the 87 nominees for the 2009 Order of National Artists. Like
respondent Moreno, he made it to the preliminary shortlist. As
The omission of the word "award" in the first portion of the he did not make it to the second shortlist, he was not
above provision appears to be unintentional as shown by the considered by the Final Deliberation Panel, more so by the
proviso which states that a member may compete for grants former President.
and awards only one year after his or her term shall have
expired. As such, respondent Guidote-Alvarez is restricted and It should be recalled too that respondent Guidote-Alvarez was
disqualified from being conferred the 2009 Order of National disqualified to be nominated for being the Executive Director of
Artists.55 the NCCA at that time while respondents Mañosa and Caparas
did not make it to the preliminary shortlist and respondent
The Court’s Ruling Moreno was not included in the second shortlist. Yet, the four
of them were treated differently and considered favorably when
they were exempted from the rigorous screening process of
Standing of the Petitioners the NCCA and the CCP and conferred the Order of National
Artists. The Committee on Honors and the former President
Standing is the determination of whether a specific person is effectively treated respondents Guidote-Alvarez, Caparas,
the proper party to bring a matter to the court for Mañosa and Moreno as a preferred class. The special
adjudication.56 The gist of the question of standing is whether treatment accorded to respondents Guidote-Alvarez, Caparas,
a party alleges such personal stake in the outcome of the Mañosa and Moreno fails to pass rational scrutiny. 60 No real
controversy as to assure that concrete adverseness which and substantial distinction between respondents and petitioner
sharpens the presentation of issues upon which the court Abad has been shown that would justify deviating from the
depends for illumination of difficult constitutional questions. 57 laws, guidelines and established procedures, and placing
respondents in an exceptional position. The undue
classification was not germane to the purpose of the law.
The parties who assail the constitutionality or legality of a Instead, it contradicted the law and well-established
statute or an official act must have a direct and personal guidelines, rules and regulations meant to carry the law into
interest. They must show not only that the law or any effect. While petitioner Abad cannot claim entitlement to the
governmental act is invalid, but also that they sustained or are Order of National Artists,61 he is entitled to be given an equal
in immediate danger of sustaining some direct injury as a result opportunity to vie for that honor. In view of the foregoing, there
of its enforcement, and not merely that they suffer thereby in was a violation of petitioner Abad’s right to equal protection, an
some indefinite way. They must show that they have been or interest that is substantial enough to confer him standing in this
are about to be denied some right or privilege to which they are case.
lawfully entitled or that they are about to be subjected to some
burdens or penalties by reason of the statute or act complained
of.58 As regards the other concerned artists and academics as well
as the CAP, their claim of deep concern for the preservation of
the country’s rich cultural and artistic heritage, while laudable,
In this case, we find that the petitioning National Artists will falls short of the injury in fact requirement of standing. Their
be denied some right or privilege to which they are entitled assertion constitutes a generalized grievance shared in a
as members of the Order of National Artists as a result of substantially equal measure by all or a large class of
the conferment of the award on respondents Guidote- citizens.62 Nor can they take refuge in their status as taxpayers
Alvarez, Caparas, Mañosa and Moreno. In particular, they as the case does not involve any illegal appropriation or
will be denied the privilege of exclusive membership in the taxation. A taxpayer’s suit is proper only when there is an
Order of National Artists. exercise of the spending or taxing power of the Congress. 63

In accordance with Section 2(a)59 of Executive Order No. 236, Nonetheless, as a reading of the petition shows that it has
s. 2003, the Order of National Artists is "an exclusive advanced an issue which deserves the attention of this Court
association of honored individuals." To ensure the exclusivity in view of its seriousness, novelty and weight as precedent, it
of the membership in the Order, a rigid nomination and behooves the Court to relax the rules on standing and to
screening process has been established with different resolve the issue presented before it. 64 Moreover, this issue is
sets of renowned artists and respected art critics invited of paramount interest,65 which further justifies a liberal stance
to sit as the Council of Experts for the First and Second on standing.
Deliberation Panels. Moreover, all living National Artists are
given a voice on who should be included in their exclusive club
as they automatically become members of the Final Propriety of the Remedies
Deliberation Panel that will vote on who should be included in
the final list to be submitted to the President for conferment of The present action is a petition for prohibition, certiorari,
the Order of National Artists. To allow the untrammeled injunction, restraining order and all other legal, just and
discretion and authority of the President to confer the equitable reliefs.
Order of National Artists without regard to the stringent
screening and rigorous selection process established by
the NCCA and the CCP will diminish, if not negate, the It has been held that the remedies of prohibition and injunction
exclusive nature of the said Order. It will unduly subject are preventive and, as such, cannot be availed of to restrain
the selection and conferment of the Order of National an act that is already fait accompli. 66 Where the act sought to
Artists to politics rather than to principles and procedures. be prohibited or enjoined has already been accomplished or
It will subvert the transparent and rigorous process and consummated, prohibition or injunction becomes moot. 67
allow entry to the exclusive Order of National Artists
through a secret backdoor of lobbying, back channeling Nevertheless, even if the principal issue is already moot, this
and political accommodation. Court may still resolve its merits for the future guidance of both
bench and bar. Courts will decide a question otherwise moot
FINALS CONSTITUTION I ACJUCO 53

and academic if it is "capable of repetition, yet evading The President’s power must be exercised in accordance
review."68 with existing laws. Section 17, Article VII of the
Constitution prescribes faithful execution of the laws by
the President:
It is an opportune time for the Court to assert its role as
republican schoolmaster,69 a teacher in a vital national
seminar.70 There are times when the controversy is of such Sec. 17. The President shall have control of all the
character that, to prevent its recurrence and to assure respect executive departments, bureaus and offices. He shall
for constitutional limitations, this Court must pass on the merits ensure that the laws be faithfully executed. (Emphasis
of a case.71 This is one such case. More than being a teaching supplied.)
moment, this is not the first time that the Order of National
Artists was conferred in the manner that is being assailed in The President’s discretion in the conferment of the Order
this case.72 If not addressed here and now, there is great of National Artists should be exercised in accordance with
probability that the central question involved in this case will the duty to faithfully execute the relevant laws. The faithful
haunt us again in the future. Every President may invoke execution clause is best construed as an obligation imposed
absolute presidential prerogative and thrust upon us National on the President, not a separate grant of power. 79 It simply
Artists after his or her own heart, in total disregard of the advise underscores the rule of law and, corollarily, the cardinal
of the CCP and the NCCA and the voice of the community of
principle that the President is not above the laws but is obliged
artists, resulting to repeated episodes of indignation and to obey and execute them.80 This is precisely why the law
uproar from the artists and the public.
provides that "administrative or executive acts, orders and
regulations shall be valid only when they are not contrary to the
Furthermore, if not corrected, such an act would give rise to laws or the Constitution."81
mischief and dangerous precedent whereby those in the
corridors of power could avoid judicial intervention and review
In this connection, the powers granted to the NCCA and the
by merely speedily and stealthily completing the commission CCP Boards in connection with the conferment of the Order of
of an illegality.73 National Artists by executive issuances were institutionalized
by two laws, namely, Presidential Decree No. 208 dated June
In any event, the present petition is also for certiorari and there 7, 1973 and Republic Act No. 7356. In particular, Proclamation
is no procedural bar for the Court to pass upon the question of No. 1144 dated May 15, 1973 constituted the CCP Board as
whether the proclamations of respondents Guidote-Alvarez, the National Artists Awards Committee and tasked it to
Caparas, Mañosa and Moreno as National Artists were "administer the conferment of the category of National Artist"
attended by grave abuse of presidential discretion. upon deserving Filipino artists with the mandate to "draft the
rules to guide its deliberations in the choice of National Artists":
Limits of the President’s Discretion
Proclamation No. 1001 dated April 27, 1972, creating the
The respective powers of the CCP Board of Trustees and of Award and Decoration of National Artist, is hereby amended
the NCCA Board of Commissioners with respect to the by creating a National Artists Awards Committee, hereinafter
conferment of the Order of National Artists are clear. They to administer the conferment of the category of National Artist
jointly administer the said award and, upon their upon those deserving thereof. The Committee, which shall be
recommendation or advice, the President confers the composed of members of the Board of Trustees of the Cultural
Center of the Philippines, shall organize itself immediately and
Order of National Artists.
shall draft the rules to guide its deliberations in the choice of
National Artists, to the end that those who have created a body
To "recommend" and to "advise" are synonymous. To of work in the arts and in letters capable of withstanding the
"recommend" is "to advise or counsel."74 To "advise" is "to give test of time will be so recognized. (Emphases supplied.)
an opinion or counsel, or recommend a plan or course of
action; also to give notice. To encourage, inform or
The authority of the CCP Board of Trustees as National Artists
acquaint."75 "Advise" imports that it is discretionary or optional
with the person addressed whether he will act on such advice Awards Committee was reiterated in Presidential Decree No.
208 dated June 7, 1973.
or not.76 This has been clearly explained in Cojuangco, Jr. v.
Atty. Palma77:
The function of the CCP Board of Trustees as National Artists
The "power to recommend" includes the power to give Awards Committee has been recognized under Republic Act
No. 7356:
"advice, exhortation or indorsement, which is essentially
persuasive in character, not binding upon the party to
whom it is made." (Emphasis supplied.) Sec. 18. The National Cultural Agencies. – The [NCCA] shall
coordinate with the national cultural agencies including but not
limited to the Cultural Center of the Philippines, the Institute of
Thus, in the matter of the conferment of the Order of National
Artists, the President may or may not adopt the Philippine Languages, the National Historical Institute, the
recommendation or advice of the NCCA and the CCP National Library, the National Museum, the Records
Boards. In other words, the advice of the NCCA and the Management and Archives Office. However, they shall
CCP is subject to the President’s discretion. continue operating under their respective charters or as
provided by law where provisions therein are not inconsistent
with the provisions of this Act. They shall serve as the national
Nevertheless, the President’s discretion on the matter is not repository and/or showcase, as the case may be, of the best
totally unfettered, nor the role of the NCCA and the CCP of Philippine culture and arts. For this purpose, these agencies
Boards meaningless. shall submit periodic reports, including recommendations to
the [NCCA]. (Emphasis supplied.)
Discretion is not a free-spirited stallion that runs and roams
wherever it pleases but is reined in to keep it from straying. In On the other hand, the NCCA has been given the following
its classic formulation, "discretion is not unconfined and mandate in connection with the conferment of cultural or arts
vagrant" but "canalized within banks that keep it from awards:
overflowing."78
FINALS CONSTITUTION I ACJUCO 54

Sec. 12. Mandate. – The Commission is hereby mandated to 2.4: Awards Committees
formulate and implement policies and plans in accordance with
the principles stated in Title 1 of this Act. There shall be two types of awards committees: the Committee
on Honors and the various awards committees in the various
(a) To encourage the continuing and balanced units of the government service.
development of a pluralistic culture by the people
themselves, it shall: A. The Committee on Honors

xxxx The Committee on Honors serves as a National Awards


Committee. It is composed of the following:
(4) extend recognition of artistic achievement through
awards, grants and services to artists and cultural The Executive Secretary, Chairman
groups which contribute significantly to the Filipino’s
cultural legacy;
The Secretary of Foreign Affairs, Vice-Chairman
xxxx
Head, Presidential Management Staff, member
Sec. 13. Powers and Functions. – To carry out its
mandate, the Commission shall exercise the Presidential Assistant for Historical Affairs, member
following powers and functions:
Chief of Presidential Protocol, member
xxxx
Chief of Protocol, DFA, member
(j) advise the President on matters pertaining to
culture and the arts, including the creation of a All nominations from the various awards committees must be
special decoration or award, for persons who have submitted to the Committee on Honors via the Chancellery of
significantly contributed to the development and Philippine Orders and State Decorations. The Chancellery
promotion of Philippine culture and arts; shall process nominations for the consideration of the
Committee on Honors. The Committee on Honors shall screen
(k) promulgate rules, regulations and undertake any and recommend these nominations to the President.
and all measures as may be necessary to implement
this Act. (Emphases supplied.) The Committee on Honors shall, as a general rule, serve as a
screening committee to ensure that nominations received from
By virtue of their respective statutory mandates in connection the various awards committees meet two tests: that there has
with the conferment of the National Artist Award, the NCCA not been an abuse of discretion in making the nomination, and
and the CCP decided to work together and jointly administer that the nominee is in good standing. Should a nomination
the National Artist Award. They reviewed the guidelines for the meet these criteria, a recommendation to the President for
nomination, selection and administration of the National Artist conferment shall be made.
Award, created a National Artist Award Secretariat, centralized
all financial resources and management for the administration The President of the Philippines takes the
of the National Artist Award, and added another layer to the recommendations of the Committee on Honors in the
selection process so that more members of the arts and culture highest consideration when making the final decision on
sector of the Philippines may be involved and participate in the the conferment of awards. (Emphasis supplied.)
selection of National Artists.
Pursuant to the above provision of the implementing rules of
We have held that an administrative regulation adopted Executive Order No. 236, s. 2003, the authority of the
pursuant to law has the force and effect of law.82 Thus, the Committee on Honors is limited to determining whether the
rules, guidelines and policies regarding the Order of National nominations submitted by a particular awards committee, in
Artists jointly issued by the CCP Board of Trustees and the this case, the joint NCCA and CCP Boards, have been tainted
NCCA pursuant to their respective statutory mandates have by abuse of discretion, and whether the nominees are in good
the force and effect of law. Until set aside, they are binding standing. Should the nominations meet these two criteria, the
upon executive and administrative agencies, 83 including the Committee on Honors shall make a recommendation to the
President himself/herself as chief executor of laws. In this President for conferment of the Order of National Artists.
connection, Section 2.5(A) of the Implementing Rules and
Regulations84 of Executive Order No. 236, s. 2003 provides:
In view of the various stages of deliberation in the selection
process and as a consequence of his/her duty to faithfully
2.5: General Guidelines for Awards Committees enforce the relevant laws, the discretion of the President in
the matter of the Order of National Artists is confined to
A. National Orders of Cultural and Scientific Merit the names submitted to him/her by the NCCA and the CCP
Boards. This means that the President could not have
considered conferment of the Order of National Artists on
The existing modalities of the NCCA for selecting recipients for any person not considered and recommended by the
the Order of National Artists, and the Gawad sa Manlilikha ng
NCCA and the CCP Boards. That is the proper import of the
Bayan, and of the NAST for selecting recipients of the Order of provision of Executive Order No. 435, s. 2005, that the NCCA
National Scientists, shall remain in force. (Emphases and the CCP "shall advise the President on the conferment of
supplied.) the Order of National Artists." Applying this to the instant case,
the former President could not have properly considered
Section 2.4(A) of the same implementing rules further states: respondents Guidote-Alvarez, Caparas, Mañosa and
Moreno, as their names were not recommended by the
FINALS CONSTITUTION I ACJUCO 55

NCCA and the CCP Boards. Otherwise, not only will the declares that, as the former President committed grave abuse
stringent selection and meticulous screening process be of discretion in issuing Proclamation Nos. 1826 to 1829 dated
rendered futile, the respective mandates of the NCCA and July 6, 2009, the said proclamations are invalid. However,
the CCP Board of Trustees under relevant laws to nothing in this Decision should be read as a disqualification on
administer the conferment of Order of National Artists, the part of respondents Guidote-Alvarez, Caparas, Mañosa
draft the rules and regulations to guide its deliberations, and Moreno to be considered for the honor of National Artist in
formulate and implement policies and plans, and the future, subject to compliance with the laws, rules and
undertake any and all necessary measures in that regard regulations governing said award.
will also become meaningless.
WHEREFORE, the petition is hereby GRANTED in PART.
Furthermore, with respect to respondent Guidote-Alvarez who Proclamation Nos. 1826 to 1829 dated July 6, 2009
was the Executive Director of the NCCA at that time, the proclaiming respondents Cecile Guidote-Alvarez, Carlo Magno
Guidelines expressly provides: Jose Caparas, Francisco Mañosa, and Jose Moreno,
respectively, as National Artists are declared INVALID and
6.5 NCCA and CCP Board members and consultants and
NCCA and CCP officers and staff are automatically disqualified SET ASIDE for having been issued with grave abuse of
from being nominated.85 discretion.

Respondent Guidote-Alvarez could not have even been SO ORDERED.


nominated, hence, she was not qualified to be considered and
conferred the Order of National Artists at that time. The
President’s discretion on the matter does not extend to
removing a legal impediment or overriding a legal restriction.

From the foregoing, the advice or recommendation of the


NCCA and the CCP Boards as to the conferment of the
Order of National Artists on Conde, Dr. Santos, Francisco
and Alcuaz was not binding on the former President but
only discretionary or optional for her whether or not to act
on such advice or recommendation. Also, by virtue of the
power of control, the President had the authority to alter
or modify or nullify or set aside such recommendation or
advice. It was well within the President’s power and
discretion to proclaim all, or some or even none of the
recommendees of the CCP and the NCCA Boards, without
having to justify his or her action. Thus, the exclusion of
Santos did not constitute grave abuse of discretion on the part
of the former President.

The conferment of the Order of National Artists on respondents


Guidote-Alvarez, Caparas, Mañosa and Moreno was an
entirely different matter.

There is grave abuse of discretion when an act is (1) done


contrary to the Constitution, the law or jurisprudence or (2)
executed whimsically, capriciously or arbitrarily, out of malice,
ill will or personal bias.86

There was a violation of the equal protection clause of the


Constitution87 when the former President gave
preferential treatment to respondents Guidote-Alvarez,
Caparas, Mañosa and Moreno.1âwphi1 The former
President’s constitutional duty to faithfully execute the laws
and observe the rules, guidelines and policies of the NCCA and
the CCP as to the selection of the nominees for conferment of
the Order of National Artists proscribed her from having a free
and uninhibited hand in the conferment of the said award. The
manifest disregard of the rules, guidelines and processes
of the NCCA and the CCP was an arbitrary act that unduly
favored respondents Guidote-Alvarez, Caparas, Mañosa
and Moreno. The conferment of the Order of National Artists
on said respondents was therefore made with grave abuse of
discretion and should be set aside.

While the Court invalidates today the proclamation of


respondents Guidote-Alvarez, Caparas, Mañosa and Moreno
as National Artists, such action should not be taken as a
pronouncement on whether they are worthy to be conferred
that honor. Only the President, upon the advise of the NCCA
and the CCP Boards, may determine that. The Court simply
FINALS CONSTITUTION I ACJUCO 56

[G.R. No. 149724. August 19, 2003] WHEREFORE, defendants DENR Secretary Antonio H.
Cerilles and Regional Executive Director Israel C. Gaddi are
DEPARTMENT OF ENVIRONMENT AND NATURAL hereby ordered to cease and desist from doing the act
RESOURCES, represented herein by its complained of, namely, to stop the transfer of DENR [Region]
Secretary, HEHERSON T. ALVAREZ, petitioner, vs. DENR 12 offices from Cotabato City to Korandal (Marbel), South
REGION 12 EMPLOYEES, represented by BAGUIDALI Cotabato.
KARIM, Acting President of COURAGE (DENR Region 12
Chapter), respondents. xxx xxx xxx.

DECISION SO ORDERED.[5]

YNARES-SANTIAGO, J.: Petitioner filed a Motion for Reconsideration with Motion to


Dismiss, raising the following grounds:
This is a petition for review assailing the Resolutions dated
May 31, 2000[1] of the Court of Appeals which dismissed the I.
petition for certiorari in CA-G.R. SP No. 58896, and its
Resolution dated August 20, 2001[2], which denied the motion The power to transfer the Regional Office of the Department of
for reconsideration. Environment and Natural Resources (DENR) is executive in
nature.
The facts are as follows:
II.
On November 15, 1999, Regional Executive Director of the
Department of Environment and Natural Resources for The decision to transfer the Regional Office is based on
Region XII, Israel C. Gaddi, issued a Memorandum[3] directing Executive Order No. 429, which reorganized Region XII.
the immediate transfer of the DENR XII Regional Offices
III.
from Cotabato City to Koronadal (formerly Marbel), South
Cotabato. The Memorandum was issued pursuant to DENR The validity of EO 429 has been affirmed by the Honorable
Administrative Order No. 99-14, issued by then DENR Supreme Court in the Case of Chiongbian vs. Orbos (1995)
Secretary Antonio H. Cerilles, which reads in part: 245 SCRA 255.
Subject: Providing for the Redefinition of Functions and IV.
Realignment of Administrative Units in the Regional and
Field Offices: Since the power to reorganize the Administrative Regions is
Executive in Nature citing Chiongbian, the Honorable Court
Pursuant to Executive Order No. 192, dated June 10, 1987 has no jurisdiction to entertain this petition.[6]
and as an interim administrative arrangement to improve the
efficiency and effectiveness of the Department of Environment On January 14, 2000, the trial court rendered judgment, the
and Natural Resources (DENR) in delivering its services dispositive portion of which reads:
pending approval of the government-wide reorganization by
Congress, the following redefinition of functions and CONSEQUENTLY, order is hereby issued ordering the
realignment of administrative units in the regional and field respondents herein to cease and desist from enforcing their
offices are hereby promulgated: Memorandum Order dated November 15, 1999 relative to the
transfer of the DENR Regional Offices from Region 12 to
Section 1. Realignment of Administrative Units: Region 11 at Koronadal, South Cotabato for being bereft of
legal basis and issued with grave abuse of discretion
The DENR hereby adopts a policy to establish at least one amounting to lack or excess of jurisdiction on their part,
Community Environment and Natural Resources Office and they are further ordered to return back the seat of the
(CENRO) or Administrative Unit per Congressional District DENR Regional Offices 12 to Cotabato City.
except in the Autonomous Region of Muslim Mindanao
(ARMM) and the National Capital Region (NCR). The Regional SO ORDERED.[7]
Executive Directors (REDs) are hereby authorized to
realign/relocate existing CENROs and implement this policy in Petitioners’ motion for reconsideration was denied in an Order
accordance with the attached distribution list per region which dated April 10, 2000. A petition for certiorari under Rule 65 was
forms part of this Order. Likewise, the following realignment filed before the Court of Appeals, docketed as CA-G.R. SP No.
and administrative arrangements are hereby adopted: 58896. The petition was dismissed outright for: (1) failure to
submit a written explanation why personal service was not
xxxxxxxxx done on the adverse party; (2) failure to attach affidavit of
service; (3) failure to indicate the material dates when copies
1.6. The supervision of the Provinces of South Cotabato of the orders of the lower court were received; (4) failure to
and Sarangani shall be transferred from Region XI to XII.[4] attach certified true copy of the order denying petitioners
motion for reconsideration; (5) for improper verification, the
Respondents, employees of the DENR Region XII who are
same being based on petitioners knowledge and belief, and (6)
members of the employees association, COURAGE,
wrong remedy of certiorari under Rule 65 to substitute a lost
represented by their Acting President, Baguindanai A. Karim,
appeal.[8]
filed with the Regional Trial Court of Cotabato, a petition for
nullity of orders with prayer for preliminary injunction. The motion for reconsideration was denied in a resolution
dated August 20, 2001.[9] Hence, this petition based on the
On December 8, 1999, the trial court issued a temporary
following assignment of errors:
restraining order enjoining petitioner from implementing the
assailed Memorandum. The dispositive portion of the Order I
reads:
FINALS CONSTITUTION I ACJUCO 57

RULES OF PROCEDURE CAN NOT BE USED TO DEFEAT Petitioner maintains that the assailed DAO-99-14 and the
THE ENDS OF SUBSTANTIAL JUSTICE implementing memorandum were valid and that the trial court
should have taken judicial notice of Republic Act No. 6734,
II otherwise known as An Organic Act for the Autonomous
Region in Muslim Mindanao, and its implementing
THE DECISION OF THE LOWER COURT DATED 14
Executive Order 429,[14] as the legal bases for the issuance of
JANUARY 2000 WHICH WAS AFFIRMED IN THE
the assailed DAO-99-14. Moreover, the validity of R.A. No.
QUESTIONED RESOLUTIONS OF THE COURT OF
6734 and E.O. 429 were upheld in the case of Chiongbian v.
APPEALS DATED 31 MAY 2000 AND 20 AUGUST 2001 IS
Orbos.[15] Thus, the respondents cannot, by means of an
PATENTLY ILLEGAL AND SHOULD BE NULLIFIED,
injunction, force the DENR XII Regional Offices to remain in
CONSIDERING THAT:
Cotabato City, as the exercise of the authority to transfer the
A. RESPONDENTS HAVE NO CAUSE OF ACTION AGAINST same is executive in nature.
PETITIONER AS THEY HAVE NO RIGHT TO CAUSE THE
It is apropos to reiterate the elementary doctrine of qualified
DENR REGION 12 OFFICE TO REMAIN IN COTABATO
political agency, thus:
CITY.
Under this doctrine, which recognizes the establishment
B. THE STATE DID NOT GIVE ITS CONSENT TO BE SUED.
of a single executive, all executive and administrative
C. THE DECISION OF THE LOWER COURT DATED 14 organizations are adjuncts of the Executive Department,
JANUARY 2000 IS CONTRARY TO THE RULE OF the heads of the various executive departments are
PRESUMPTION OF REGULARITY IN THE PERFORMANCE assistants and agents of the Chief Executive, and, except
OF OFFICIAL FUNCTIONS. in cases where the Chief Executive is required by the
Constitution or law to act in person or the exigencies of the
D. IN ANY EVENT, THE DECISION OF THE LOWER COURT situation demand that he act personally, the multifarious
DATED 14 JANUARY 2000 IS CONTRARY TO THE LETTER executive and administrative functions of the Chief
AND INTENT OF EXECUTIVE ORDER NO. 429 AND Executive are performed by and through the executive
REPUBLIC ACT NO. 6734. departments, and the acts of the Secretaries of such
departments, performed and promulgated in the regular
E. THE DETERMINATION OF THE PROPRIETY AND course of business, are, unless disapproved or reprobated
PRACTICALITY OF THE TRANSFER OF REGIONAL by the Chief Executive, presumptively the acts of the Chief
OFFICES IS INHERENTLY EXECUTIVE, AND THEREFORE, Executive.[16]
NON-JUSTICIABLE.[10]
This doctrine is corollary to the control power of the President
In essence, petitioner argues that the trial court erred in as provided for under Article VII, Section 17 of the 1987
enjoining it from causing the transfer of the DENR XII Regional Constitution, which reads:
Offices, considering that it was done pursuant to DENR
Administrative Order 99-14. Sec. 17. The President shall have control of all the executive
departments, bureaus, and offices. He shall ensure that the
The issues to be resolved in this petition are: (1) Whether laws be faithfully executed.
DAO-99-14 and the Memorandum implementing the same
were valid; and (2) Whether the DENR Secretary has the However, as head of the Executive Department, the President
authority to reorganize the DENR. cannot be expected to exercise his control (and supervisory)
powers personally all the time. He may delegate some of his
Prefatorily, petitioner prays for a liberal application of powers to the Cabinet members except when he is
procedural rules considering the greater interest of justice. required by the Constitution to act in person or the
exigencies of the situation demand that he acts
This Court is fully aware that procedural rules are not to be
personally.[17]
simply disregarded for these prescribed procedures ensure an
orderly and speedy administration of justice. However, it is In Buklod ng Kawaning EIIB v. Zamora,[18] this Court upheld
equally true that litigation is not merely a game of the continuing authority of the President to carry out the
technicalities. Time and again, courts have been guided by the reorganization in any branch or agency of the executive
principle that the rules of procedure are not to be applied in a department. Such authority includes the creation,
very rigid and technical manner, as rules of procedure are used alteration or abolition of public offices.[19] The Chief
only to help secure and not to override substantial Executives authority to reorganize the National
justice.[11] Thus, if the application of the Rules would tend to Government finds basis in Book III, Section 20 of E.O. No.
frustrate rather than promote justice, it is always within the 292, otherwise known as the Administrative Code of
power of this Court to suspend the rules, or except a particular 1987, viz:
case from its operation.[12]
Section 20. Residual Powers. Unless Congress provides
Despite the presence of procedural flaws, we find it necessary otherwise, the President shall exercise such other powers and
to address the issues because of the demands of public functions vested in the President which are provided for under
interest, including the need for stability in the public service and the laws and which are not specifically enumerated above or
the serious implications this case may cause on the effective which are not delegated by the President in accordance with
administration of the executive department. Although no law.
appeal was made within the reglementary period to appeal,
nevertheless, the departure from the general rule that the Further, in Larin v. Executive Secretary,[20] this Court had
extraordinary writ of certiorari cannot be a substitute for the lost occasion to rule:
remedy of appeal is justified because the execution of the
assailed decision would amount to an oppressive exercise of This provision speaks of such other powers vested in the
judicial authority.[13] President under the law. What law then gives him the power to
reorganize? It is Presidential Decree No. 1772 which
FINALS CONSTITUTION I ACJUCO 58

amended Presidential Decree No. 1416. These decrees Act: Provided, That only the provinces and cities voting
expressly grant the President of the Philippines the favorably in such plebiscite shall be included in the
continuing authority to reorganize the national Autonomous Region in Muslim Mindanao. The provinces and
government, which includes the power to group, cities which in the plebiscite do not vote for inclusion in the
consolidate bureaus and agencies, to abolish offices, to Autonomous Region shall remain in the existing administrative
transfer functions, to create and classify functions, regions: Provided, however, that the President may, by
services and activities and to standardize salaries and administrative determination, merge the existing regions.
materials. The validity of these two decrees is unquestionable.
The 1987 Constitution clearly provides that all laws, decrees, Pursuant to the authority granted by the aforequoted provision,
executive orders, proclamations, letters of instructions and then President Corazon C. Aquino issued on October 12, 1990
other executive issuances not inconsistent with this E.O. 429, Providing for the Reorganization of the
Constitution shall remain operative until amended, repealed or Administrative Regions in Mindanao. Section 4 thereof
revoked. So far, there is yet no law amending or repealing said provides:
decrees.
SECTION 4. REGION XII, to be known as CENTRAL
Applying the doctrine of qualified political agency, the MINDANAO, shall include the following provinces and cities:
power of the President to reorganize the National Government
Provinces
may validly be delegated to his cabinet members
exercising control over a particular executive Sultan Kudarat
department. Thus, in DOTC Secretary v. Mabalot,[21] we held
that the President through his duly constituted political agent Cotabato
and alter ego, the DOTC Secretary may legally and validly
decree the reorganization of the Department, particularly the South Cotabato
establishment of DOTC-CAR as the LTFRB Regional Office at
Cities
the Cordillera Administrative Region, with the concomitant
transfer and performance of public functions and Cotabato
responsibilities appurtenant to a regional office of the LTFRB.
General Santos
Similarly, in the case at bar, the DENR Secretary can validly
reorganize the DENR by ordering the transfer of the DENR The Municipality of Koronadal (Marinduque) in South
XII Regional Offices from Cotabato City to Koronadal, Cotabato shall serve as the regional center.
South Cotabato. The exercise of this authority by the
DENR Secretary, as an alter ego, is presumed to be the In Chiongbian v. Orbos, this Court stressed the rule that the
acts of the President for the latter had not expressly power of the President to reorganize the administrative
repudiated the same. regions carries with it the power to determine the regional
centers. In identifying the regional centers, the President
The trial court should have taken judicial notice of R.A. No. purposely intended the effective delivery of the field
6734, as implemented by E.O. No. 429, as legal basis of the services of government agencies.[23] The same intention
Presidents power to reorganize the executive department, can be gleaned from the preamble of the assailed DAO-99-14
specifically those administrative regions which did not vote for which the DENR sought to achieve, that is, to improve the
their inclusion in the ARMM. It is axiomatic that a court has the efficiency and effectiveness of the DENR in delivering its
mandate to apply relevant statutes and jurisprudence in services.
determining whether the allegations in a complaint establish a
cause of action. While it focuses on the complaint, a court It may be true that the transfer of the offices may not be timely
clearly cannot disregard decisions material to the proper considering that: (1) there are no buildings yet to house the
appreciation of the questions before it. [22] In resolving the regional offices in Koronadal, (2) the transfer falls on the month
motion to dismiss, the trial court should have taken cognizance of Ramadan, (3) the children of the affected employees are
of the official acts of the legislative, executive, and judicial already enrolled in schools in Cotabato City, (4) the Regional
departments because they are proper subjects of mandatory Development Council was not consulted, and (5) the
judicial notice as provided by Section 1 of Rule 129 of the Sangguniang Panglungsond, through a resolution, requested
Rules of Court, to wit: the DENR Secretary to reconsider the orders. However, these
concern issues addressed to the wisdom of the transfer rather
A court shall take judicial notice, without the introduction of than to its legality. It is basic in our form of government that the
evidence, of the existence and territorial extent of states, their judiciary cannot inquire into the wisdom or expediency of the
political history, forms of government and symbols of acts of the executive or the legislative department, [24] for each
nationality, the law of nations, the admiralty and maritime department is supreme and independent of the others, and
courts of the world and their seals, the political constitution and each is devoid of authority not only to encroach upon the
history of the Philippines, the official acts of the legislative, powers or field of action assigned to any of the other
executive and judicial departments of the Philippines, the department, but also to inquire into or pass upon the
laws of nature, the measure of time, and the geographical advisability or wisdom of the acts performed, measures taken
divisions. (Emphasis supplied) or decisions made by the other departments. [25]

Article XIX, Section 13 of R.A. No. 6734 provides: The Supreme Court should not be thought of as having been
tasked with the awesome responsibility of overseeing the
SECTION 13. The creation of the Autonomous Region in entire bureaucracy. Unless there is a clear showing of
Muslim Mindanao shall take effect when approved by a constitutional infirmity or grave abuse of discretion amounting
majority of the votes cast by the constituent units provided in to lack or excess of jurisdiction, the Courts exercise of the
paragraph (2) of Sec. 1 of Article II of this Act in a plebiscite judicial power, pervasive and limitless it may seem to be, still
which shall be held not earlier than ninety (90) days or later must succumb to the paramount doctrine of separation of
than one hundred twenty (120) days after the approval of this powers.[26] After a careful review of the records of the case, we
FINALS CONSTITUTION I ACJUCO 59

find that this jurisprudential element of abuse of discretion has


not been shown to exist.

WHEREFORE, in view of the foregoing, the petition for


review is GRANTED. The resolutions of the Court of Appeals
in CA-G.R. SP No. 58896 dated May 31, 2000 and August 20,
2001, as well as the decision dated January 14, 2000 of the
Regional Trial Court of Cotabato City, Branch 15, in Civil Case
No 389, are REVERSED and SET ASIDE. The permanent
injunction, which enjoined the petitioner from enforcing the
Memorandum Order of the DENR XII Regional Executive
Director, is LIFTED.

SO ORDERED.
FINALS CONSTITUTION I ACJUCO 60

G.R. No. 109406. September 11, 1998 excluding those created, maintained or acquired in pursuance
of a policy of the State enunciated in the Constitution, or by law
BLAQUERA VS ALCALA and those whose officers and employees are covered by the
Civil Service. (underscoring supplied)
DECISION
The PTrA is a GOCC created in pursuance of a policy of the
PURISIMA, J.: State. Section 9 of Presidential Decree No. 189 states that
to implement the policies and program of the Department
These are cases for certiorari and prohibition, challenging the (Dept. of Tourism), there is hereby created a Philippine
constitutionality and validity of Administrative Order Nos. 29 Tourism Authority, xxx. Likewise, Section 21 of the same
and 268 on various grounds. decree provides that all officials and employees of the
Authority, xxx, shall be subject to Civil Service Law, rules and
The facts in G.R. Nos. 109406, 110642, 111494, and 112056
regulations, and the coverage of the Wage and Position
are undisputed, to wit:
Classification Office.
Petitioners are officials and employees of several government
Furthermore, although Supplemental Rules and Regulations
departments and agencies who were paid incentive benefits
implementing R.A. #6971 was issued only on December 27,
for the year 1992, pursuant to Executive Order No.
1991, the law itself is clear that it pertains to private business
292[1] (EO 292), otherwise known as the Administrative Code
enterprises whose employees are covered by the Labor Code
of 1987, and the Omnibus Rules Implementing Book V[2]of
of the Philippines, as mentioned in the following provisions:
EO 292. On January 19, 1993, then President Fidel V.
Ramos (President Ramos) issued Administrative Order No. Section 5. Labor Management Committee. xxx that at the
29 (AO 29) authorizing the grant of productivity incentive request of any party to the negotiation, the National Wages and
benefits for the year 1992 in the maximum amount Productivity Commission of the Department of Labor and
of P1,000.00[3] and reiterating the prohibition[4] under Employment shall provide the necessary studies, xxx.
Section 7[5]of Administrative Order No. 268 (AO
268), enjoining the grant of productivity incentive benefits Section 8. Notification. - A business enterprise which adopts
without prior approval of the President. Section 4 of AO 29 a productivity incentive program shall submit copies of the
directed [a]ll departments, offices and agencies which same to the National Wages and Productivity Commission and
authorized payment of CY 1992 Productivity Incentive Bonus to the Bureau of Internal Revenue for their information and
in excess of the amount authorized under Section 1 hereof record.
[are hereby directed] to immediately cause the
return/refund of the excess within a period of six months to Section 9. Disputes and Grievances. - Whenever disputes,
commence fifteen (15) days after the issuance of this Order. In grievances, or other matters arise from the interpretation or
compliance therewith, the heads of the departments or implementation of the productivity incentive program, xxx may
agencies of the government concerned, who are the herein seek the assistance of the National Conciliation and Mediation
respondents, caused the deduction from petitioners salaries or Board of the Department of Labor and Employment for such
allowances of the amounts needed to cover the alleged purpose. xxx
overpayments. To prevent the respondents from making
Therefore, considering the foregoing, the PTrA is within the
further deductions from their salaries or allowances, the
exclusion provision of the Implementing Rules of RA #6971
petitioners have come before this Court to seek relief.
and so, it (PTrA) does not fall within its coverage as being
In G.R. No. 119597, the facts are different but the petition entitled to the productivity incentive bonus under RA #6971.
poses a common issue with the other consolidated cases. The
Secondly, Administrative Order No. 29 which is the basis for
petitioner, Association of Dedicated Employees of the
the grant of the productivity incentive bonus/benefits for CY
Philippine Tourism Authority (ADEPT), is an association of
1992 also expressly provides prohibiting payments of
employees of the Philippine Tourism Authority (PTA) who
similar benefits in future years unless duly authorized by the
were granted productivity incentive bonus for calendar year
President.
1992 pursuant to Republic Act No. 6971 (RA
6971), otherwise known as the Productivity Incentives Act Thirdly, the disallowance of the Auditor, PTrA has already
of 1990. Subject bonus was, however, disallowed by the been resolved when this Commission circularized thru COA
Corporate Auditor on the ground that it was prohibited Memorandum #92-758 dated April 3, 1992 the Supplemental
under Administrative Order No. 29 dated January 19, to Rules Implementing RA 6971 otherwise known as the
1993.[6] The disallowance of the bonus in question was finally Productivity Incentives Act of 1990. xxx
brought on appeal to the Commission on Audit (COA) which
denied the appeal in its Decision[7]of March 6, 1995, Lastly, considering the title of RA #6971, i.e. An Act to
ratiocinating, thus: encourage productivity and maintain industrial peace by
providing incentives to both labor and capital, and its
xxx Firstly, the provisions of RA #6971 insofar as the implementing rules and regulations prepared by the
coverage is concerned, refer to business enterprises Department of Labor and Employment and the Department
including government owned and/or controlled corporations of Finance, this Office concludes that said law/regulation
performing proprietary functions. pertains to agencies in the private sector whose
employees are covered by the Labor Code.
Section 1a of the Supplemental Rules Implementing RA
#6971 classified such coverage as: With the denial of its appeal, petitioner found its way here via
the petition in G.R. No. 119597, to seek relief from the
All business enterprises, with or without existing duly certified
aforesaid decision of COA.
labor organizations, including government owned and/or
controlled corporations performing proprietary functions which We will first resolve the issue on the applicability of RA 6971 to
are established solely for business or profit and accordingly petitioner ADEPT in G.R. No. 119597 before passing upon the
FINALS CONSTITUTION I ACJUCO 61

constitutionality or validity of Administrative Orders 29 and a. To assist the Department make a comprehensive survey of
268. the physical and natural tourism resources of the Philippines;
to establish the order of priority for development of said areas;
Section 3 of RA 6971, reads: to recommend to the President the proclamation of a tourist
zone; and to define and fix the boundaries of the zone;
SECTION 3. Coverage. This Act shall apply to all business
enterprises with or without existing and duly recognized or b. To formulate a development plan for each zone;
certified labor organizations, including government-owned and
controlled corporations performing proprietary functions. It c. To submit to the President through the National Economic
shall cover all employees and workers including casual, and Development Authority for review and approval all
regular, supervisory and managerial development plans before the same are enforced or
employees. (Underscoring ours) implemented;

Pursuant to Section 10[8] of RA 6971, the Secretary of Labor d. To submit to the President an Annual Progress Report;
and Secretary of Finance issued Supplemental Rules to
implement the said law, as follows: e. To assist the Department to determine the additional
capacity requirements for various tourist facilities and services;
Section 1. - Paragraph (a) Section 1, Rule II of the Rules to prepare a ten-year Tourism Priorities Plan; to update
Implementing RA 6971, shall be amended to read as follows: annually the ten year Tourism Priorities Plan.

Coverage. These Rules shall apply to: f. To gather, collate and analyze statistical data and other
pertinent information for the effective implementation of PD
(a) All business enterprises with or without existing duly 564.
certified labor organizations, including government-owned and
controlled corporations performing proprietary functions which 2. Acquisition and disposition of lands and other assets
are established solely for business or profit or gain and for tourist zone purposes
accordingly excluding those created, maintained or
acquired in pursuance of a policy of the state, enunciated a. To acquire possession and ownership of all lands
in the Constitution or by law, and those whose officers and transferred to it from other government corporations and
employees are covered by the Civil Service. (Underscoring institutions and any land having tourism potential and
ours) earmarked in the Tourism Priorities Plans for intensive
development into a tourist zone or as a part thereof, subject to
xxx the approval of the President.

Petitioner contends that the PTA is a government-owned and b. To acquire by purchase, by negotiation or by condemnation
controlled corporation performing proprietary function, and proceedings any private land within and without the tourist
therefore the Secretary of Labor and Employment and zones for any of the following reasons: (a) consolidation of
Secretary of Finance exceeded their authority in issuing the lands for tourist zone development purposes, (b) prevention of
aforestated Supplemental Rules Implementing RA 6971. land speculation in areas declared as tourist zones, (c)
acquisition of right of way to the zones, (d) protection of water
Government-owned and controlled corporations may perform shed areas and natural assets with tourism value, and (e) for
governmental or proprietary functions or both, depending on any other purpose expressly authorized under PD 564.
the purpose for which they have been created. If the purpose
is to obtain special corporate benefits or earn pecuniary c. For the purpose of providing land acquisition assistance to
profit, the function is proprietary. If it is in the interest of registered tourism enterprises, to sell, subdivide, resell, lease,
health, safety and for the advancement of public good and sublease, rent out, or otherwise, to said registered tourism
welfare, affecting the public in general, the function is enterprises under sufficiently soft terms for use specifically in
governmental.[9] Powers classified as proprietary are those the development of hotels, recreational facilities, and other
intended for private advantage and benefit. [10] tourist services.

The PTA was established by Presidential Decree No. 189, as d. To develop and/or subdivide any land in its name or
amended by Presidential Decree No. 564 (PD 564). undertake condominium projects thereon, and sell subdivision
lots or condominium units to private persons for investment
[11]
Its general purposes are: purposes.
1. To implement the policies and programs of the e. To take over or transfer to a registered tourism enterprise in
Department of Tourism (Department); accordance with law any lease on foreshore areas within a
tourist zone or adjacent thereto, in cases said areas are not
2. To develop tourist zones;
being utilized in accordance with the PTAs approved zone
3. To assist private enterprises in undertaking tourism projects; development plan and wherein the lessee concerned does not
agree to conform accordingly.
4. To operate and maintain tourist facilities;
f. To arrange for the reclamation of any land adjacent to or
5. To assure land availability for private investors in hotels and adjoining a tourist zone in coordination with appropriate
other tourist facilities; government agencies.

6. To coordinate all tourism project plans and operations. 3. Infrastructure development for tourist zone purposes

Its specific functions and powers [12] are: a. To contract, supervise and pay for infrastructure works and
civil works within a tourist zone owned and operated by the
1. Planning and development of tourism projects PTA.
FINALS CONSTITUTION I ACJUCO 62

b. To coordinate with appropriate government agencies the 7. Other powers and functions
development of infrastructure requirements supporting a
tourist zone. a. To engage or retain the services of financial, management,
legal, technical, and/or project consultants from the private or
c. To take water from any public stream, river, creek, lake, government sector.
spring, or waterfall and to alter, straighten, obstruct or increase
the flow of water in streams. b. To have the power to succeed by its corporate name.

4. Zone administration and control c. To adopt, alter, and use a corporate seal.

a. To formulate and implement zoning regulations. d. To sue and be sued under its corporate name.

b. To determine and regulate the enterprises to be established e. To enter into any contracts of any kind and description.
within a tourist zone.
f. To own or possess personal and/or real property.
c. To ensure, through the proper authorities concerned, the
g. To make, adopt and enforce rules and regulations to
ecological preservation, maintenance and/or rehabilitation of
execute its powers, duties and functions.
the common and the public areas within a tourist zone and the
environment thereof. h. To purchase, hold, and alienate shares of stock or bonds of
any corporation.
d. To identify and recommend to the President the
preservation and/or restoration of national monuments or I. To collect fees or charges as may be imposed under PD 564.
preserves; to arrange for the preservation and/or restoration of
the same with appropriate government agencies or with the j. To contract indebtedness and issue bonds.
private sector or with the owners themselves of said tourist
attractions; and to identify and recommend to the appropriate k. To fix and collect rentals for the lease, use or occupancy of
authorities concerned the declaration of tourist areas and lands, buildings, or other property owned or administered by
attractions as national monuments and preserves. PTA.

5. Project and investment promotions l. To do any and all acts and things necessary to carry out the
purposes for which the PTA is created.
a. To identify, develop, invest in, own, manage and operate
such projects as it may deem to be vital for recreation and rest Categorized in light of the foregoing provisions of law in point,
but not sufficiently attractive economically for private PTAs governmental functions include the first,
investment. third, fourth, and sixth of the aforesaid general
purposes. The second [13] and fifth general purposes fall under
b. To construct hotel buildings and other tourist facilities within its proprietary functions.
a tourist zone and in turn lease such facilities to registered
tourism enterprises for operation, management and With respect to PTAs specific functions and powers, the
maintenance. first and fourth are governmental in nature while the fifth
specific functions and powers are proprietary in
c. To organize, finance, invest in, manage and operate wholly- character. The second, third, sixth, and seventh specific
owned subsidiary corporations. functions and powers can be considered partly-governmental
and partly-proprietary, considering that 2(a), 2(b), 2(c), 2(d),
6. Direct assistance to registered enterprises 2(e), 3(a), 6(c), 6(d), 6(e), 7(h), 7(j), and 7(k) are proprietary
functions while 2(f), 3(b), 3(c), 6(a), 6(b), 6(f), 6(g), 6(h), 7(a),
a. To administer the tax and other incentives granted to
7(b), 7(c), 7(d), 7(f), 7(g), and 7(l) are governmental
registered enterprises.
functions. The specific functions and powers treated in 7(e)
b. To evaluate, approve and register or reject any and all and 7(i) may be classified either as proprietary or
tourism projects or enterprises established within the tourist governmental, depending on the circumstances under which
zones. they are exercised or performed.

c. To grant medium and long-term loans and/or re-lend any The aforecited powers and functions of PTA are
funds borrowed for the purpose to duly qualified registered predominantly governmental, principally geared towards the
tourism enterprises. development and promotion of tourism in the scenic Philippine
archipelago. But it is irrefutable that PTA also performs
d. To guarantee local and foreign borrowings of registered proprietary functions, as envisaged by its charter.
enterprises.
Reliance on the above analysis of the functions and powers of
e. To provide equity investments in the form of cash and/or PTA does not suffice for the determination of whether or not it
land. is within the coverage of RA 6971. For us to resolve the issues
raised here solely on the basis of the classification of PTAs
f. To extend technical, management and financial assistance powers and functions may lead to the rendition of judgment
to tourism projects. repugnant to the legislative intent and to established doctrines,
as well, such as on the prohibition against government workers
g. To identify, contact and assist in negotiations of suitable
to strike.[14] Under RA 6971, the workers have the right to
partners for both local and foreign investors interested in
strike.
investment or participation in the tourism industry.
To ascertain whether PTA is within the ambit of RA 6971,
h. To assist registered enterprises and prospective investors to
there is need to find out the legislative intent, and to refer to
have their papers processed with dispatch by government
other provisions of RA 6971 and other pertinent laws, that may
offices.
FINALS CONSTITUTION I ACJUCO 63

aid the Court in ruling on the right of officials and employees of bargaining unit(s); section 6[20] relates to existing collective
PTA to receive bonuses under RA 6971. bargaining agreements, and labor and management; section
7[21] speaks of strike or lockout; and section 9[22] purports to
Petitioner cites an entry in the journal of the House of seek the assistance of the National Conciliation and Mediation
Representatives to buttress its submission that PTA is within Board of the Department of Labor and Employment and
the coverage of RA 6971, to wit: include the name(s) of the voluntary arbitrators or panel of
voluntary arbitrator. All the aforecited provisions of law apply
Chairman Veloso: The intent of including government-owned
only to private corporations and government-owned and
and controlled corporations within the coverage of the Act is
controlled corporations organized under the general
the recognition of the principle that when government goes into
corporation law. Only they have collective bargaining agents,
business, it (divests) itself of its immunity from suit and goes
collective bargaining units, collective bargaining agreements,
down to the level of ordinary private enterprises and subjects
and the right to strike or lockout.
itself to the ordinary laws of the land just like ordinary private
enterprises. Now, when people work therefore in To repeat, employees of government corporations created
government-owned or controlled corporations, it is as if by special charters have neither the right to strike nor the
they are also, just like in the private sector, entitled to all right to bargain collectively, as defined in the Labor
the benefits of all laws that apply to workers in the private Code. The case of Social Security System Employees
sector. In my view, even including the right to organize, Association indicates the following remedy of government
bargain.... VELOSO (Bicameral Conference Committee on workers not allowed to strike or bargain collectively, to wit:
Labor and Employment, pp. 15-16)
Government employees may, therefore, through their unions
After a careful study, the Court is of the view, and so holds, that or associations, either petition the Congress for the betterment
contrary to petitioners interpretation, the government-owned of the terms and conditions of employment which are within the
and controlled corporations Mr. Chairman Veloso had in mind ambit of legislation or negotiate with the appropriate
were government-owned and controlled corporations government agencies for the improvement of those which are
incorporated under the general corporation law. This is so not fixed by law. If there be any unresolved grievances, the
because only workers in private corporations and dispute may be referred to the Public Sector Labor-
government-owned and controlled corporations, Management Council for appropriate action. But employees in
incorporated under the general corporation law, have the the civil service may not resort to strikes, walkouts and other
right to bargain (collectively). Those in government temporary work stoppages, like workers in the private sector,
corporations with special charter, which are subject to Civil to pressure the Government to accede to their
Service Laws, have no right to bargain (collectively), except demands. (supra, footnote 14, p. 698; italics ours)
where the terms and conditions of employment are not fixed by
law.[15] Their rights and duties are not comparable with those in It is a rule in statutory construction that every part of the statute
the private sector. must be interpreted with reference to the context, i.e., that
every part of the statute must be considered together with the
Since the terms and conditions of government employment are other parts, and kept subservient to the general intent of the
fixed by law, government workers cannot use the same whole enactment.[23] The provisions of RA 6971, taken
weapons employed by workers in the private sector to secure
concessions from their employers. The principle behind labor
together, reveal the legislative intent to include
unionism in private industry is that industrial peace cannot be only government-owned and controlled
secured through compulsion by law. Relations between private corporations performing proprietary functions
employers and their employees rest on an essentially voluntary within its coverage.
basis. Subject to the minimum requirements of wage laws and
other labor and welfare legislation, the terms and conditions of Every statute must be construed and harmonized with other
employment in the unionized private sector are settled through statutes as to form a uniform system of jurisprudence. [24] We
the process of collective bargaining. In government note Section 1, Rule X of the Omnibus Rules Implementing
employment, however, it is the legislature and, where Book V of EO 292, which reads:
properly given delegated power, the administrative heads
of government which fix the terms and conditions of SECTION 1. - Each department or agency of government,
employment. And this is effected through statutes or whether national or local, including bureaus and agencies,
administrative circulars, rules, and regulations, not state colleges and universities, and government owned and
through collective bargaining agreements. (Alliance of controlled corporations with original charters, shall establish its
Government Workers v. Minister of Labor and Employment, own Department or Agency Employee Suggestions and
124 SCRA 1) (italics ours) Incentives Award System in accordance with these Rules and
shall submit the same to the Commission for
Government corporations may be created by special charters approval. (Underscoring ours)
or by incorporation under the general corporation law. Those
created by special charters are governed by the Civil It is thus evident that PTA, being a government-owned and
Service Law while those incorporated under the general controlled corporation with original charter subject to
corporation law are governed by the Labor Code.[16] Civil Service Law, Rules and Regulations,[25] is already
within the scope of an incentives award system under
The legislative intent to place only government-owned and Section 1, Rule X of the Omnibus Rules Implementing EO
controlled corporations performing proprietary functions under 292 issued by the Civil Service
the coverage of RA 6971 is gleanable from the other provisions Commission (Commission). Since government-owned and
of the law. For instance, section 2[17] of said law envisions controlled corporations with original charters do have an
industrial peace and harmony and to provide incentive award system, Congress enacted a law that would
corresponding incentives to both labor and capital; section address the same concern of officials and employees of
4[18] refers to representatives of labor and management; government-owned and controlled corporations incorporated
section 5[19] mentions of collective bargaining agent(s) of the under the general corporation law.
FINALS CONSTITUTION I ACJUCO 64

All things studiedly considered in proper perspective, the Court interest in connection with, or in relation to, their official
finds no reversible error in the finding by respondent employment.
Commission that PTA is not within the purview of RA 6971. As
regards the promulgation of implementing rules and Sec. 36. Personnel Relations. - (1) It shall be the concern of
regulations, it bears stressing that the power of administrative the Commission to provide leadership and assistance in
officials to promulgate rules in the implementation developing employee relations programs in the department or
of the statute is necessarily limited to what is provided for in agencies.
the legislative enactment.[26] In the case under scrutiny, the
(2) Every Secretary or head of agency shall take all proper
Supplementary Rules Implementing RA 6971 issued by the
steps toward the creation of an atmosphere conducive to good
Secretary of Labor and Employment and the Secretary of
supervisor-employee relations and the improvement of
Finance accord with the intendment and provisions of RA
employee morale.
6971. Consequently, not being covered by RA
6971, AO 29 applies to the petitioner. Pursuant to the provision of Section 12(2), [28] Chapter 3, Book
V of EO 292, the Commission adopted and prescribed the
We now tackle the common issue posited by the consolidated Omnibus Rules Implementing Book V of EO 292 which, among
petitions on the constitutionality of AO 29 and AO 268. others, provide:
Petitioners contend and argue, that: Sec. 1. - Each department or agency of government, whether
national or local, including bureaus and agencies, state
I. AO 29 AND AO 268 ARE VIOLATIVE OF THE PROVISIONS
colleges and universities, and government owned and
OF EO 292 AND, HENCE, NULL AND VOID.
controlled corporations with original charters, shall establish
II. AO 29 AND AO 268 UNLAWFULLY USURP THE its own Department or Agency Employee Suggestions and
CONSTITUTIONAL AUTHORITY GRANTED SOLELY TO Incentives Award System in accordance with these Rules
THE CIVIL SERVICE COMMISSION. and shall submit the same to the Commission for
approval.
III. THE FORCED REFUND OF INCENTIVE PAY IS AN
UNCONSTITUTIONAL IMPAIRMENT OF A CONTRACTUAL Sec. 2. - The System is designed to encourage creativity,
OBLIGATION. innovativeness, efficiency, integrity and productivity in the
public service by recognizing and rewarding officials and
IV. ASSUMING, FOR THE SAKE OF ARGUMENT ONLY, employees, individually or in groups, for their suggestions,
THAT THE GRANT OF PRODUCTIVITY INCENTIVE inventions, superior accomplishments, and other personal
BENEFITS WAS INVALID, THE SAME SHOULD BE THE efforts which contribute to the efficiency, economy, or other
PERSONAL LIABILITY OF OFFICIALS DIRECTLY improvement in government operations, or for other
RESPONSIBLE THEREFOR IN ACCORDANCE WITH extraordinary acts of services in the public interest.
SECTION 9 OF AO 268.
xxx
Issued by the then President Corazon Aquino (President
Aquino) on July 25, 1987 in the exercise of her legislative Sec. 7. - The incentive awards shall consist of, though not
powers under the 1987 Constitution, [27] EO 292, or the limited to, the following:
Administrative Code of 1987, provided for the
xxx
following incentive award system:
(c) Productivity Incentive which shall be given to an employee
Sec. 31. Career and Personnel Development Plans. - Each or group of employees who has exceeded their targets or
department or agency shall prepare a career and personnel has incurred incremental improvement over existing
development plan which shall be integrated into a national plan targets.
by the Commission. Such career and personnel development
plans which shall include provisions on merit promotions, On February 21, 1992, President Aquino issued AO 268 which
performance evaluation, in-service training, including granted each official and employee of the government the
overseas and local scholarships and training grants, job productivity incentive benefits in a maximum amount
rotation, suggestions and incentive award systems, and equivalent to thirty percent (30%) of his one (1) month
such other provisions for employees health, welfare, basic salary but in no case shall such amount be less than
counseling, recreation and similar services. two thousand pesos (P2,000.00),[29] for those who have
rendered at least one year of service as of December 31,
Sec. 35. Employee Suggestions and Incentive Award
1991.[30]Said AO carried the prohibition, provided in Section 7
System. - There shall be established a government-wide
thereof, which reads:
employee suggestions and incentive awards system which
shall be administered under such rules, regulations, and SECTION 7. The productivity incentive benefits herein
standards as maybe promulgated by the Commission. authorized shall be granted only for Calendar Year
1991. Accordingly, all heads of agencies, including the
In accordance with rules, regulations, and standards governing boards of government-owned or -controlled
promulgated by the Commission, the President or the head corporations and financial institutions, are hereby strictly
of each department or agency is authorized to incur
prohibited from authorizing/granting productivity incentive
whatever necessary expenses involved in the honorary
benefits or other allowances of similar nature for Calendar Year
recognition of subordinate officers and employees of the
1992 and future years pending the result of a comprehensive
government who by their suggestions, inventions,
study being undertaken by the Office of the President in
superior accomplishment, and other personal efforts
coordination with the Civil Service Commission and the
contribute to the efficiency, economy, or other
Department of Budget and Management on the matter.
improvement of government operations, or who perform
such other extraordinary acts or services in the public
FINALS CONSTITUTION I ACJUCO 65

The formulation of the necessary implementing guidelines for Government, it is tasked to formulate and establish a system
Executive Order No. 486 dated 8 November 1991 establishing of incentives and rewards for officials and employees in the
a performance-based incentive system for government-owned public sector, alike.
or -controlled corporations shall likewise be included in the
comprehensive study referred to in the preceding paragraph. The functions of the Commission have been decentralized to
the different departments, offices, and agencies of the
On January 19, 1993, President Ramos issued AO 29 which government --
granted productivity incentive benefits to government
employees in the maximum amount of P1,000.00[31] for the SEC. 1. Declaration of Policy. -- The State shall insure and
calendar year 1992 but reiterated the proscription under promote the Constitutional mandate that appointments in the
Section 7 of AO 268, thus: Civil Service shall be made only according to merit and fitness;
that the Civil Service Commission, as the central personnel
SECTION 2. The prohibition prescribed under Section 7 of agency of the Government shall establish a career service,
Administrative Order No. 268 is hereby reiterated. Accordingly, adopt measures to promote morale, efficiency, integrity,
all heads of government offices/agencies, including responsiveness, and courtesy in the civil service, strengthen
government-owned and/or controlled corporations, as well as the merit and rewards system, integrate all human resources
their respective governing boards are hereby enjoined and development programs for all levels and ranks, and
prohibited from authorizing/granting Productivity Incentive institutionalize a management climate conducive to public
Benefits or any and all similar forms of allowances/benefits accountability; that public office is a public trust and public
without prior approval and authorization via officers and employees must at all times be accountable to the
Administrative Order by the Office of the people; and that personnel functions shall be decentralized,
President. Henceforth, anyone found violating any of the delegating the corresponding authority to the departments,
mandates in this Order, including all officials/employees and offices and agencies where such functions can be effectively
the COA Auditor-in-Charge of such government office/agency performed. (Section 1, Chapter I, Subtitle A, Title I, EO 292)
found to have taken part thereof, shall be accordingly and (underscoring ours)
severely dealt with in accordance with the applicable
provisions of existing penal laws. Specifically, implementation of the Employee Suggestions and
Incentive Award System has been decentralized to the
Consequently, all administrative authorizations to grant any President or to the head of each department or agency --
form of allowances/benefits and all forms of additional
compensation usually paid outside of the prescribed basic Sec. 35. Employee Suggestions and Incentive Award System.
salary under R.A. No. 6758, the Salary Standardization Law - There shall be established a government-wide employee
that are inconsistent with the legislated policy on the matter or suggestions and incentive awards system which shall be
are not covered by any legislative action are hereby revoked. administered under such rules, regulations, and standards as
maybe promulgated by the Commission.
The implementation of Executive Order No. 486 dated
November 8, 1991, as amended by Executive Order No. 518 In accordance with rules, regulations, and standards
dated May 29, 1992, is hereby deferred until a more promulgated by the Commission, the President or the head of
comprehensive and equitable scheme for the grant of the each department or agency is authorized to incur whatever
benefits that can be applied government-wide is formulated by necessary expenses involved in the honorary recognition of
the Department of Budget and Management. subordinate officers and employees of the government who by
their suggestions, inventions, superior accomplishment, and
Petitioners theorize that AO 29 and AO 268 violate EO 292 and other personal efforts contribute to the efficiency, economy, or
since the latter is a law, it prevails over executive other improvement of government operations, or who perform
issuances. Petitioners likewise assert that AO 29 and AO 268 such other extraordinary acts or services in the public interest
encroach upon the constitutional authority of the Civil Service in connection with, or in relation to, their official
Commission to adopt measures to strengthen the merit and employment. (EO 292) (underscoring ours)
rewards system and to promulgate rules, regulations and
standards governing the incentive awards system of the civil The President is the head of the government. Governmental
service. power and authority are exercised and implemented through
him. His power includes the control over executive
The Court is not impressed with petitioners’ submission. AO departments --
29 and AO 268 were issued in the valid exercise of
presidential control over the executive departments. The president shall have control of all the executive
departments, bureaus, and offices. He shall ensure that the
In establishing a Civil Service Commission, the 1987 laws be faithfully executed. (Section 17, Article VII, 1987
Constitution delineated its function, as follows: Constitution)

The Civil Service Commission, as the central personnel Control means the power of an officer to alter or modify or set
agency of the Government, shall establish a career service and aside what a subordinate officer had done in the performance
adopt measures to promote morale, efficiency, integrity, of his duties and to substitute the judgment of the former for
responsiveness, progressiveness, and courtesy in the civil that of the latter.[32] It has been held that [t]he President can,
service. It shall strengthen the merit and rewards system, by virtue of his power of control, review, modify, alter or nullify
integrate all human resources development programs for all any action, or decision, of his subordinate in the executive
levels and ranks, and institutionalize a management climate departments, bureaus, or offices under him. He can exercise
conducive to public accountability. It shall submit to the this power motu proprio without need of any appeal from any
President and the Congress an annual report on its personnel party.[33]
programs. (Section 3, Article IX, B, 1987 Constitution)
When the President issued AO 29 limiting the amount of
The Commission handles personnel matters of the incentive benefits, enjoining heads of government
government. As the central personnel agency of the agencies from granting incentive benefits without prior
FINALS CONSTITUTION I ACJUCO 66

approval from him, and directing the refund of the excess committing limited resources of government for the equal
over the prescribed amount, the President was just payment of incentives and awards. The President was
exercising his power of control over executive only exercising his power of control by modifying the acts
departments. This is decisively clear from the WHEREAS of the respondents who granted incentive benefits to their
CLAUSES of AO 268 and AO 29, to wit: employees without appropriate clearance from the Office
of the President, thereby resulting in the uneven
ADMINISTRATIVE ORDER NO. 268 distribution of government resources. In the view of the
President, respondents did a mistake which had to be
xxx
corrected. In so acting, the President exercised a
WHEREAS, the productivity incentive benefits granted by the constitutionally-protected prerogative --
different agencies are of varying amounts, causing
The Presidents duty to execute the law is of constitutional
dissension/demoralization on the part of those who had
origin. So, too, is his control of all executive departments. Thus
received less and those who have not yet received any such
it is, that department heads are men of his confidence. His is
benefit, thereby defeating the purpose for which the same
the power to appoint them; his, too, is the privilege to dismiss
should be granted; and
them at pleasure. Naturally, he controls and directs their
WHEREAS, there exists the need to regulate the grant of the acts. Implicit then is his authority to go over, confirm, modify or
productivity incentive benefits or other similar allowances in reverse the action taken by his department secretaries. In this
conformity with the policy on standardization of compensation context, it may not be said that the President cannot rule on
pursuant to Republic Act No. 6758; the correctness of a decision of a department
secretary. (Lacson-Magallanes Co., Inc. v. Pao, 21 SCRA 898)
x x x.
Neither can it be said that the President encroached upon the
ADMINISTRATIVE ORDER NO. 29 authority of the Commission on Civil Service to grant benefits
to government personnel. AO 29 and AO 268 did not revoke
xxx the privilege of employees to receive incentive benefits. The
same merely regulated the grant and amount thereof.
WHEREAS, the faithful implementation of statutes,
including the Administrative Code of 1987 and all laws Sound management and effective utilization of financial
governing all forms of additional compensation and resources of government are basically executive
personnel benefits is a Constitutional prerogative vested functions,[34] not the Commissions. Implicit is this recognition in
in the President of the Philippines under Section 17, EO 292, which states:
Article VII of the 1987 Constitution;
Sec. 35. Employee Suggestions and Incentive Award System.
WHEREAS, the Constitutional prerogative includes the - There shall be established a government-wide employee
determination of the rates, the timing and schedule of suggestions and incentive awards system which shall be
payment, and final authority to commit limited resources administered under such rules, regulations, and standards as
of government for the payment of personnel incentives, maybe promulgated by the Commission.
cash awards, productivity bonus, and other forms of
additional compensation and fringe benefits; In accordance with rules, regulations, and standards
promulgated by the Commission, the President or the head of
WHEREAS, some government agencies have overlooked each department or agency is authorized to incur whatever
said Constitutional prerogative and have unilaterally necessary expenses involved in the honorary recognition of
granted to their respective officials and employees subordinate officers and employees of the government who by
incentive awards; their suggestions, inventions, superior accomplishment, and
other personal efforts contribute to the efficiency, economy, or
WHEREAS, the Office of the President issued Administrative
other improvement of government operations, or who perform
Order No. 268, dated February 21, 1992, strictly prohibiting the
such other extraordinary acts or services in the public interest
grant of Productivity Incentive Bonus or other allowances of
in connection with, or in relation to, their official
similar nature for Calendar Year 1992 and future years pending
employment.(Chapter 5, Subtitle A, Book V) (underscoring
the issuance of the requisite authorization by the President;
ours)
WHEREAS, notwithstanding said prohibition some
Conformably, it is the President or the head of each
government offices/agencies and government-owned and/or
department or agency who is authorized to incur the
controlled corporations and financial institutions have granted
necessary expenses involved in the honorary recognition
productivity incentive benefits in varying nomenclature and
of subordinate officers and employees of the
amounts without the proper authorization/coordination with the
government. It is not the duty of the Commission to fix the
Office of the President;
amount of the incentives. Such function belongs to the
WHEREAS, the unilateral and uncoordinated grant of President or his duly empowered alter ego.
productivity incentive benefits gave rise to discontentment,
Anent petitioners contention that the forcible refund of
dissatisfaction and demoralization among government
incentive benefits is an unconstitutional impairment of a
personnel who have received less or have not received at all
contractual obligation, suffice it to state that [n]ot all contracts
such benefits;
entered into by the government will operate as a waiver of its
x x x. non-suability; distinction must be made between its sovereign
and proprietary acts (United States of America v. Ruiz, 136
The President issued subject Administrative Orders to SCRA 487).[35] The acts involved in this case are
regulate the grant of productivity incentive benefits and to governmental. Besides, the Court is in agreement with the
prevent discontentment, dissatisfaction and Solicitor General that the incentive pay or benefit is in the
demoralization among government personnel by
FINALS CONSTITUTION I ACJUCO 67

nature of a bonus which is not a demandable or enforceable WHEREFORE, the Petitions in G.R. Nos. 109406, 110642,
obligation. 111494, and 112056 are hereby DISMISSED, and as above
ratiocinated, further deductions from the salaries and
It is understood that the Judiciary, Civil Service Commission, allowances of petitioners are hereby ENJOINED.
Commission on Audit, Commission on Elections, and Office of
the Ombudsman, which enjoy fiscal autonomy, are not covered In G.R. No. 119597, the assailed Decision of respondent
by the amount fixed by the President. As explained in Bengzon Commission on Audit is AFFIRMED. No pronouncement as to
vs. Drilon (208 SCRA 133): costs.

As envisioned in the Constitution, the fiscal autonomy SO ORDERED.


enjoyed by the Judiciary, the Civil Service Commission,
the Commission on Audit, the Commission on Elections,
and the Office of the Ombudsman contemplates a
guarantee of full flexibility to allocate and utilize their
resources with the wisdom and dispatch that their needs
require. It recognizes the power and authority to levy, assess
and collect fees, fix rates of compensation not exceeding the
highest rates authorized by law for compensation and pay
plans of the government and allocate and disburse such sums
as may be provided by law or prescribed by them in the course
of the discharge of their functions.

Fiscal autonomy means freedom from outside control. If the


Supreme Court says it needs 100 typewriters but DBM rules
we need only 10 typewriters and sends its recommendations
to Congress without even informing us, the autonomy given by
the Constitution becomes an empty and illusory platitude.

The Judiciary, the Constitutional Commissions, and the


Ombudsman must have the independence and flexibility
needed in the discharge of their constitutional duties. The
imposition of restrictions and constraints on the manner the
independent constitutional offices allocate and utilize the funds
appropriated for their operations is anathema to fiscal
autonomy and violative not only of the express mandate of the
Constitution but especially as regards the Supreme Court, of
the independence and separation of powers upon which the
entire fabric of our constitutional system is based. In the
interest of comity and cooperation, the Supreme Court,
Constitutional Commissions, and the Ombudsman have so far
limited their objections to constant reminders. We now agree
with the petitioners that this grant of autonomy should cease to
be a meaningless provision.

Untenable is petitioners contention that the herein respondents


be held personally liable for the refund in question. Absent a
showing of bad faith or malice, public officers are not
personally liable for damages resulting from the performance
of official duties.[36]

Every public official is entitled to the presumption of good faith


in the discharge of official duties.[37] Absent any showing of bad
faith or malice, there is likewise a presumption of regularity in
the performance of official duties.[38]

In upholding the constitutionality of AO 268 and AO 29, the


Court reiterates the well-entrenched doctrine that in
interpreting statutes, that which will avoid a finding of
unconstitutionality is to be preferred.[39]

Considering, however, that all the parties here acted in good


faith, we cannot countenance the refund of subject incentive
benefits for the year 1992, which amounts the petitioners have
already received.Indeed, no indicia of bad faith can be
detected under the attendant facts and circumstances. The
officials and chiefs of offices concerned disbursed such
incentive benefits in the honest belief that the amounts given
were due to the recipients and the latter accepted the same
with gratitude, confident that they richly deserve such benefits.
FINALS CONSTITUTION I ACJUCO 68

[G.R. No. 131367. August 31, 2000] On August 15, 1996, the SBMA-PBAC issued a
resolution rejecting the bid of ICTSI because said bid does
HUTCHISON PORTS PHILIPPINES LIMITED, petitioner, not comply with the requirements of the tender documents and
vs. SUBIC BAY METROPOLITAN AUTHORITY, the laws of the Philippines. The said resolution also declared
INTERNATIONAL CONTAINER TERMINAL SERVICES that:
INC., ROYAL PORT SERVICES INC. and the EXECUTIVE
SECRETARY, respondents. RESOLVED FURTHER, that the winning bid be awarded to
HUTCHISON PORTS PHILIPPINES LIMITED (HPPL) and
DECISION that negotiations commence immediately with
HPPL (HUTCHISON) with a view to concluding an acceptable
YNARES-SANTIAGO, J.: agreement within 45 days of this date failing which negotiations
with RPSI (ROYAL) will commence with a view to concluding
On February 12, 1996, the Subic Bay Metropolitan an acceptable agreement within 45 days thereafter failing
Authority (or SBMA) advertised in leading national daily which there will be declared a failure of bids. [6] (Underscoring
newspapers and in one international publication, [1] an invitation supplied)
offering to the private sector the opportunity to develop and
operate a modern marine container terminal within the The following day, ICTSI filed a letter-appeal with
Subic Bay Freeport Zone. Out of seven bidders who SBMAs Board of Directors requesting the nullification and
responded to the published invitation, three were declared by reversal of the above-quoted resolution rejecting ICTSIs
the SBMA as qualified bidders after passing the pre- bid while awarding the same to HPPL. But even before the
qualification evaluation conducted by the SBMAs Technical SBMA Board could act on the appeal, ICTSI filed a similar
Evaluation Committee (or SBMA-TEC).These are: (1) appeal before the Office of the President.[7] On August 30,
International Container Terminal Services, Inc. (or ICTSI); (2) 1996, then Chief Presidential Legal Counsel (CPLC) Renato
a consortium consisting of Royal Port Services, Inc. and HPC L. Cayetano submitted a memorandum to then President Fidel
Hamburg Port Consulting GMBH (or RPSI); and (3) Hutchison V. Ramos, containing the following recommendations:
Ports Philippines Limited (or HPPL), representing a consortium
composed of HPPL, Guoco Holdings (Phils.), Inc. and Unicol
Management Services, Inc. All three qualified bidders were We therefore suggest that the President direct SBMA
required to submit their respective formal bid package on or Chairman Gordon to consider option number 4 that is to re-
before July 1, 1996 by the SBMAs Pre-qualification, Bids and evaluate the financial bids submitted by the parties, taking
Awards Committee (or SBMA-PBAC). into consideration all the following factors:

Thereafter, the services of three (3) international 1. Reinstate ICTSIs bid;


consultants[2] recommended by the World Bank for their
expertise were hired by SBMA to evaluate the business plans
submitted by each of the bidders, and to ensure that there 2. Disregard all arguments relating to monopoly;
would be a transparent and comprehensive review of the
submitted bids. The SBMA also hired the firm of Davis, 3. The re-evaluation must be limited to the parties’
Langdon and Seah Philippines, Inc. to assist in the evaluation financial bids.
of the bids and in the negotiation process after the winning
bidder is chosen. All the consultants, after such review and
evaluation unanimously concluded that HPPLs Business Plan 3.1 Considering that the parties business have been
was far superior to that of the two other bidders. [3] accepted (passed), strictly follow the criteria for bid
evaluation provided for in pars. (c) and (d), Part B (1)
However, even before the sealed envelopes containing of the Tender Document.
the bidders proposed royalty fees could be opened at the
appointed time and place, RPSI formally protested that 4. In the re-evaluation, the COA should actively participate to
ICTSI is legally barred from operating a second port in the determine which of the financial bids is more advantageous.
Philippines based on Executive Order No. 212 and
Department of Transportation and Communication
(DOTC) Order 95-863. RPSI thus requested that the financial 5. In addition, all the parties should be given ample opportunity
bid of ICTSI should be set aside.[4] to elucidate or clarify the components/justification for their
respective financial bids in order to ensure fair play and
Nevertheless, the opening of the sealed financial bids transparency in the proceedings.
proceeded under advisement relative to the protest signified by
RPSI. The financial bids, more particularly the proposed 6. The Presidents authority to review the final award shall
royalty fee of each bidder, was as follows: remain.[8] (Underscoring supplied)

ICTSI ------------US$57.80 TEU


The recommendation of CPLC Cayetano was
approved by President Ramos, and a copy of President
HPPL ------------US$20.50 TEU Ramos handwritten approval was sent to the SBMA Board of
Directors.Accordingly, the SBMA Board, with the concurrence
RPSI -------------US$15.08 TEU of representatives of the Commission on Audit, agreed to focus
the reevaluation of the bids in accordance with the evaluation
criteria and the detailed components contained in the Tender
The SBMA-PBAC decided to suspend the announcement of Document, including all relevant information gleaned from the
the winning bid, however, and instead gave ICTSI seven (7) bidding documents, as well as the reports of the three
days within which to respond to the letter-protest lodged by international experts and the consultancy firm hired by the
RPSI.The HPPL joined in RPSIs protest, stating that ICTSI SBMA.
should be disqualified because it was already operating
the Manila International Container Port (or MICP), which On September 19, 1996, the SBMA Board issued a
would give rise to inevitable conflict of interest between the Resolution, declaring:
MICP and the Subic Bay Container Terminal facility.[5]
FINALS CONSTITUTION I ACJUCO 69

NOW, THEREFORE, IT IS HEREBY RESOLVED that the bid aligned their judgment in congruence with, the opinion of the
that conforms to the Invitation to Tender, that has a realistic panel of experts and the SBMAs Technical Evaluation
Business Plan offering the greatest financial return to SBMA, Committee to the effect that HPPLs business is superior while
the best possible offer and the most advantageous to the that of ICTSIs appeared to be unrealistically high which may
government is that of HPPL and HPPL is accordingly selected eventually hinder the competitiveness of the SBMA port with
as the winning bidder and is hereby awarded the the rest of the world. Respondents averred that the panel of
concession for the operation and development of the Subic World Bank experts noted that ICTSIs high tariff rates at U.S.
Bay Container Terminal.[9] (Underscoring supplied) $119.00 per TEU is already higher by 37% through HPPL,
which could further increase by 20% in the first two (2) years
In a letter dated September 24, 1996, the SBMA Board and by 5% hike thereafter. In short, high tariffs would
of Directors submitted to the Office of the President the results discourage potential customers which may be translated into
of the re-evaluation of the bid proposals, to wit: low cargo volume that will eventually reduce financial return to
SBMA. Respondents asserted that HPPLs business plan
offers the greatest financial return which could be equated
SBMA, through the unanimous vote of all the Board Members, that over the five years, HPPL offers 1.25 billion pesos while
excluding the Chairman of the Board who voluntarily inhibited ICTSI offers P0.859 billion, and RPSI offers P.420 billion. Over
himself from participating in the re-evaluation, selected the the first ten years HPPL gives P2.430 billion, ICTSI tenders
HPPL bid as the winning bid, being: the conforming bid with P2.197 billion and RPSI has P1.632 billion.
a realistic Business Plan offering the greatest financial return
to the SBMA; the best possible offer in the market, and the
most advantageous to the government in accordance with the Viewed from this perspective alongside with the evidence on
Tender Document.[10] record, the undersigned panel does not find respondents to
have exceeded their discretion in awarding the bid to
HPPL.Consequently, it could not be said that respondents act
Notwithstanding the SBMA Boards recommendations had placed the government at a grossly disadvantageous
and action awarding the project to HPPL, then Executive plight that could have jeopardized the interest of the Republic
Secretary Ruben Torres submitted a memorandum to the of the Philippines.[13]
Office of the President recommending that another
rebidding be conducted.[11] Consequently, the Office of the
President issued a Memorandum directing the SBMA Board of On July 7, 1997, the HPPL, feeling aggrieved by the
Directors to refrain from signing the Concession Contract with SBMAs failure and refusal to commence negotiations and to
HPPL and to conduct a rebidding of the project.[12] execute the Concession Agreement despite its earlier
pronouncements that HPPL was the winning bidder, filed a
In the meantime, the Resident Ombudsman for the complaint[14] against SBMA before the Regional Trial Court
DOTC filed a complaint against members of the SBMA-PBAC (RTC) of Olongapo City, Branch 75, for specific performance,
before the Office of the Ombudsman for alleged violation of mandatory injunction and damages. In due time, ICTSI, RPSI
Section 3(e) of Republic Act No. 3019 for awarding the contract and the Office of the President filed separate Answers-in-
to HPPL. On April 16, 1997, the Evaluation and Preliminary Intervention[15] to the complaint opposing the reliefs sought by
Investigation Bureau of the Office of the Ombudsman issued a complainant HPPL.
Resolution absolving the members of the SBMA-PBAC of any
liability and dismissing the complaint against them, ruling thus: Complainant HPPL alleged and argued therein that a
binding and legally enforceable contract had been established
between HPPL and defendant SBMA under Article 1305 of
After an assiduous study of the respective contentions of both the Civil Code, considering that SBMA had repeatedly
parties, we are inclined to hold, as it is hereby held, that there declared and confirmed that HPPL was the winning
is no proof on record pinpointing respondents to have acted in bidder. Having accepted HPPLs offer to operate and develop
excess of their discretion when they awarded the bid to the proposed container terminal, defendant SBMA is duty-
HPPL. Records revealed that respondents, in the exercise of bound to comply with its obligation by commencing
their discretion in determining the financial packages offered negotiations and drawing up a Concession Agreement with
by the applicants, were guided by the expert report of Davis, plaintiff HPPL. HPPL also pointed out that the bidding
Langdon and Seah (DLS) that fairly evaluated which of the procedure followed by the SBMA faithfully complied with
bidders tender the greatest financial return to the existing laws and rules established by SBMA itself; thus, when
government. There is no showing that respondents had HPPL was declared the winning bidder it acquired the
abused their prerogatives. As succinctly set forth in the DLS exclusive right to negotiate with the SBMA. Consequently,
report it stated, among others, that, in assessing the full plaintiff HPPL posited that SBMA should be: (1) barred from
financial return to SBMA offered by the bidders, it is necessary conducting a re-bidding of the proposed project and/or
to consider the following critical matters: performing any such acts relating thereto; and (2) prohibited
from negotiating with any party other than plaintiff HPPL until
1. Royalty fees negotiations between HPPL and SBMA have been concluded
or in the event that no acceptable agreement could be arrived
at. Plaintiff HPPL also alleged that SBMAs continued refusal to
2. Volume of TEUs as affected by: negotiate the Concession Contract is a substantial
infringement of its proprietary rights, and caused damage and
a. Tariff rates; prejudice to plaintiff HPPL.
Hence, HPPL prayed that:
b. Marketing strategy;
(1) Upon the filing of this complaint, hearings be scheduled to
c. Port facilities; and determine the propriety of plaintiffs mandatory injunction
application which seeks to order defendant or any of its
d. Efficient reliable services. appropriate officers or committees to forthwith specify the date
as well as to perform any and all such acts (e.g. laying the
ground rules for discussion) for the commencement of
With the preceding parameters for the evaluation of bidders negotiations with plaintiff with the view to signing at the earliest
business plan, the respondents were fairly guided by, as they
FINALS CONSTITUTION I ACJUCO 70

possible time a Concession Agreement for the development which requires that a bidding be accomplished within a definite
and operation of the Subic Bay Container Terminal. time frame.

(2) Thereafter, judgment be rendered in favor of plaintiff and Truly, the matter of the deferment of the re-bidding on
against defendant: November 4, 1997 rests on the sound discretion of the
SBMA. For this Court to issue a cease-and-desist order would
2.1. Making permanent the preliminary mandatory injunction it be tantamount to an issuance of a Temporary Restraining
Order or a Writ of Preliminary Injunction. (Prado v. Veridiano II,
had issued;
G.R. No. 98118, December 6, 1991).

2.2. Ordering defendant to implement the Concession


Agreement it had executed with plaintiff in respect of the The Court notes that the Office of the President has not been
heard fully on the issues. Moreover, one of the intervenors is
development and operation of the proposed Subic Bay
Container Terminal; of the view that the issue of jurisdiction must be resolved first,
ahead of all the other issues.

2.3. Ordering defendant to pay for the cost of plaintiffs


WHEREFORE, and viewed from the foregoing considerations,
attorneys fees in the amount of P500,000.00, or as otherwise
proven during the trial. plaintiffs motion is DENIED.

Plaintiff prays for other equitable reliefs. [16] SO ORDERED.[20] (Underscoring supplied)

During the pre-trial hearing, one of the issues raised and Hence, this petition filed by petitioner (plaintiff below)
submitted for resolution was whether or not the Office of the HPPL against respondents SBMA, ICTSI, RPSI and the
President can set aside the award made by SBMA in favor Executive Secretary seeking to obtain a prohibitory
injunction. The grounds relied upon by petitioner HPPL to
of plaintiff HPPL and if so, can the Office of the President
direct the SBMA to conduct a re-bidding of the proposed justify the filing of the instant petition are summed up as
follows:
project.
While the case before the trial court was pending 29. It is respectfully submitted that to allow or for this
litigation, on August 4, 1997, the SBMA sent notices to plaintiff Honorable Court to otherwise refrain from restraining SBMA,
HPPL, ICTSI and RPSI requesting them to declare their during the pendency of this suit, from committing the
interest in participating in a rebidding of the proposed aforementioned act(s) which will certainly occur on 5
project.[17] On October 20, 1997, plaintiff HPPL received a copy December 1997 such action (or inaction) will work an injustice
of the minutes of the pre-bid conference which stated that the upon petitioner which has validly been announced as the
winning bidder would be announced on December 5, winning bidder for the operation of the Subic Bay Container
1997.[18] Then on November 4, 1997, plaintiff HPPL learned Terminal.
that the SBMA had accepted the bids of ICTSI and RPSI who
were the only bidders who qualified.
30. To allow or for this Honorable Court to otherwise refrain
In order to enjoin the rebidding while the case was still from restraining SBMA, during the pendency of this suit, from
pending, plaintiff HPPL filed a motion for maintenance of committing the aforementioned threatened acts would be in
the status quo[19] on October 28, 1997. The said motion was violation of petitioners rights in respect of the action it had filed
denied by the court a quo in an Order dated November 3, before the RTC of Olongapo City in Civil Case No. 243-O-97,
1997, to wit: and could render any judgment which may be reached by said
Court moot and ineffectual. As stated, the legal issues raised
by the parties in that proceedings are of far reaching
Plaintiff maintains that by voluntarily participating in this
importance to the national pride and prestige, and they impact
proceedings, the defendant and the intervenors have
on the integrity of government agencies engaged in
unqualifiedly agreed to submit the issue of the propriety,
international bidding of privatization projects. Its resolution on
legality and validity of the Office of the Presidents directive that
the merits by the trial court below and, thereafter, any further
the SBMA effect a rebidding of its concession contract or the
action to be taken by the parties before the appellate courts will
operation of the Subic Bay Container Terminal. As such, the
certainly benefit respondents and the entire Filipino people.[21]
status quo must be maintained in order not to thwart the courts
ability to resolve the issues presented. Further, the ethics of
the profession require that counsel should discontinue any act WHEREFORE, petitioner HPPL sought relief praying
which tends to render the issues academic. that:

The Opposition is anchored on lack of jurisdiction since the a) Upon the filing of this petition, the same be given due course
issuance of a cease-and-desist order would be tantamount to and a temporary restraining order and/or writ of preliminary
the issuance of a Temporary Restraining Order or a Writ of injunction be issued ex parte, restraining SBMA or any of its
Injunction which this Court cannot do in light of the provision of committees, or other persons acting under its control or
Section 21 of R.A. 7227 which states: direction or upon its instruction, from declaring any winner on 5
December 1997 or at any other date thereafter, in connection
with the rebidding for the privatization of the Subic Bay
Section 21. Injunction and Restraining Order. The
Container Terminal and/or for any, some or all of the
implementation of the projects for the conversion into
respondents to perform any such act(s) in pursuance thereof,
alternative productive uses of the military reservations are
until further orders from this Honorable Court;
urgent and necessary and shall not be restrained or enjoined
except by an order issued by the Supreme Court of the
Philippines. b) After appropriate proceedings, judgment be rendered in
favor of petitioner and against respondents --
During the hearing on October 30, 1997, SBMAs counsel
revealed that there is no law or administrative rule or regulation
FINALS CONSTITUTION I ACJUCO 71

(1) Ordering SBMA to desist from conducting any rebidding or Third. That there is an urgent and permanent necessity for the
in declaring the winner of any such rebidding in respect of the writ to prevent serious damage.[25]
development and operation of the Subic Bay Container
Terminal until the judgment which the RTC of Olongapo City
To our mind, petitioner HPPL has not sufficiently
may render in Civil Case No. 243-O-97 is resolved with finality; shown that it has a clear and unmistakable right to be
declared the winning bidder with finality, such that the
(2) Declaring null and void any award which SBMA may SBMA can be compelled to negotiate a Concession
announce or issue on 5 December 1997; and Contract. Though the SBMA Board of Directors, by resolution,
may have declared HPPL as the winning bidder, said award
(3) Ordering respondents to pay for the cost of suit. cannot be said to be final and unassailable. The SBMA
Board of Directors and other officers are subject to the
control and supervision of the Office of the President. All
Petitioner prays for other equitable reliefs. [22] projects undertaken by SBMA require the approval of the
President of the Philippines under Letter of Instruction No.
The instant petition seeks the issuance of an injunctive 620, which places the SBMA under its ambit as an
writ for the sole purpose of holding in abeyance the conduct by instrumentality, defined in Section 10 thereof as an agency of
respondent SBMA of a rebidding of the proposed SBICT the national government, not integrated within the department
project until the case for specific performance is resolved by framework, vested with special functions or jurisdiction by law,
the trial court. In other words, petitioner HPPL prays that endowed with some if not all corporate powers, administering
the status quo be preserved until the issues raised in the main special funds, and enjoying operational autonomy, usually
case are litigated and finally determined. Petitioner was through a charter. This term includes regulatory agencies,
constrained to invoke this Courts exclusive jurisdiction and chartered institutions and government owned and controlled
authority by virtue of the above-quoted Republic Act 7227, corporations.[26] (Underscoring supplied)
Section 21.
As a chartered institution, the SBMA is always under
On December 3, 1997, this Court granted petitioner the direct control of the Office of the President,
HPPLs application for a temporary restraining order enjoining particularly when contracts and/or projects undertaken by
the respondent SBMA or any of its committees, or other the SBMA entail substantial amounts of
persons acting under its control or direction or upon its money. Specifically, Letter of Instruction No. 620 dated
instruction, from declaring any winner on December 5, 1997 or October 27, 1997 mandates that the approval of the
at any other date thereafter, in connection with the rebidding President is required in all contracts of the national
for the privatization of the Subic Bay Container Terminal and/or government offices, agencies and instrumentalities,
for any, some or all of the respondents to perform any such act including government-owned or controlled corporations
or acts in pursuance thereof.[23] involving two million pesos (P2,000,000.00) and above,
awarded through public bidding or negotiation. The
There is no doubt that since this controversy arose, President may, within his authority, overturn or reverse any
precious time has been lost and a vital infrastructure project award made by the SBMA Board of Directors for justifiable
has in essense been mothballed to the detriment of all parties reasons. It is well-established that the discretion to accept
involved, not the least of which is the Philippine Government, or reject any bid, or even recall the award thereof, is of
through its officials and agencies, who serve the interest of the such wide latitude that the courts will not generally
nation. It is, therefore, imperative that the issues raised herein interfere with the exercise thereof by the executive
and in the court a quo be resolved without further delay so as department, unless it is apparent that such exercise of
not to exacerbate an already untenable situation. discretion is used to shield unfairness or injustice. When
the President issued the memorandum setting aside the award
At the outset, the application for the injunctive writ is only previously declared by the SBMA in favor of HPPL and
a provisional remedy, a mere adjunct to the main suit. [24] Thus, directing that a rebidding be conducted, the same was, within
it is not uncommon that the issues in the main action are the authority of the President and was a valid exercise of
closely intertwined, if not identical, to the allegations and his prerogative.Consequently, petitioner HPPL acquired no
counter allegations propounded by the opposing parties in clear and unmistakable right as the award announced by the
support of their contrary positions concerning the propriety or SBMA prior to the Presidents revocation thereof was not final
impropriety of the injunctive writ. While it is not our intention to and binding.
preempt the trial courts determination of the issues in the main
action for specific performance, this Court has a bounden duty There being no clear and unmistakable right on the part
to perform; that is, to resolve the matters before this Court in a of petitioner HPPL, the rebidding of the proposed project
manner that gives essence to justice, equity and good can no longer be enjoined as there is no material and
conscience. substantial invasion to speak of. Thus, there is no longer
any urgent or permanent necessity for the writ to prevent any
While our pronouncements are for the purpose only of perceived serious damage. In fine, since the requisites for the
determining whether or not the circumstances warrant the issuance of the writ of injunction are not present in the instant
issuance of the writ of injunction, it is inevitable that it may have case, petitioners application must be denied for lack of merit.[27]
some impact on the main action pending before the trial
court. Nevertheless, without delving into the merits of the main Finally, we focus on the matter of whether or not
case, our findings herein shall be confined to the necessary petitioner HPPL has the legal capacity to even seek redress
issues attendant to the application for an injunctive writ. from this Court. Admittedly, petitioner HPPL is a foreign
corporation, organized and existing under the laws of the
For an injunctive writ to be issued, the following British Virgin Islands. While the actual bidder was a consortium
requisites must be proven: composed of petitioner, and two other corporations, namely,
Guoco Holdings (Phils.) Inc. and Unicol Management
First. That the petitioner/applicant must have a clear and Servises, Inc., it is only petitioner HPPL that has brought the
unmistakable right. controversy before the Court, arguing that it is suing only on an
isolated transaction to evade the legal requirement that foreign
corporations must be licensed to do business in the Philippines
Second. That there is a material and substantial invasion of to be able to file and prosecute an action before Philippines
such right. courts.
FINALS CONSTITUTION I ACJUCO 72

The maelstrom of this issue is whether participating in


the bidding is a mere isolated transaction, or did it constitute
engaging in or transacting business in the Philippines such that
petitioner HPPL needed a license to do business in the
Philippines before it could come to court.
There is no general rule or governing principle laid down
as to what constitutes doing or engaging in or transacting
business in the Philippines. Each case must be judged in the
light of its peculiar circumstances. [28] Thus, it has often been
held that a single act or transaction may be considered as
doing business when a corporation performs acts for which it
was created or exercises some of the functions for which it was
organized. The amount or volume of the business is of no
moment, for even a singular act cannot be merely incidental or
casual if it indicates the foreign corporations intention to do
business.[29]
Participating in the bidding process constitutes doing
business because it shows the foreign corporations intention
to engage in business here. The bidding for the concession
contract is but an exercise of the corporations reason for
creation or existence. Thus, it has been held that a foreign
company invited to bid for IBRD and ADB international projects
in the Philippines will be considered as doing business in the
Philippines for which a license is required. In this regard, it is
the performance by a foreign corporation of the acts for which
it was created, regardless of volume of business, that
determines whether a foreign corporation needs a license or
not.[30]
The primary purpose of the license requirement is to
compel a foreign corporation desiring to do business within the
Philippines to submit itself to the jurisdiction of the courts of the
state and to enable the government to exercise jurisdiction
over them for the regulation of their activities in this
country.[31] If a foreign corporation operates a business in the
Philippines without a license, and thus does not submit itself to
Philippine laws, it is only just that said foreign corporation be
not allowed to invoke them in our courts when the need
arises. While foreign investors are always welcome in this land
to collaborate with us for our mutual benefit, they must be
prepared as an indispensable condition to respect and be
bound by Philippine law in proper cases, as in the one at
bar.[32] The requirement of a license is not intended to put
foreign corporations at a disadvantage, for the doctrine of lack
of capacity to sue is based on considerations of sound public
policy.[33] Accordingly, petitioner HPPL must be held to be
incapacitated to bring this petition for injunction before this
Court for it is a foreign corporation doing business in the
Philippines without the requisite license.
WHEREFORE, in view of all the foregoing, the instant
petition is hereby DISMISSED for lack of merit. Further, the
temporary restraining order issued on December 3, 1997 is
LIFTED and SET ASIDE. No costs.
SO ORDERED.
FINALS CONSTITUTION I ACJUCO 73

[G. R. No. 143481. February 15, 2002] authorized under Joint Resolution No. 01 in two tranches, as
follows:
NATIONAL ELECTRIFICATION
ADMINISTRATION, petitioner, vs. COMMISSION ON
SEC. 2. Full Implementation. The Department of Budget and
AUDIT, respondent. Management is hereby directed to implement in full in FY 1997
the remaining balance of said Salary Schedule after the partial
DECISION implementation made of the same in 1994, 1995 and 1996 to
civilian and uniformed personnel, as follows:
CARPIO, J.:
The Case
1. For Civilian Personnel
This is a petition for certiorari under Rule 65 of the 1997
Rules of Civil Procedure with prayer for preliminary injunction a. Effective January 1, 1997 = in
and temporary restraining order, to reverse and set aside accordance with the Fourth Interim
Decision No. 2000-132 dated May 16, 2000 of the Commission Salary Schedule hereto attached and
on Audit[1] (Commission for brevity) in RE: Appeal of Mr. marked as Annex A of this Order. The
Conrado Estrella III, Administrator, National Electrification adjustment shall be to the designated
Administration (NEA) Quezon City, for the lifting of the salary step of the employee in the
disallowance on the payment of accelerated increases under salary grade allocation of his position
Joint Resolution No. 01 totaling P14,155,342.00. The as of December 31, 1996;
dispositive portion of the Decision reads:
b. Effective November 1, 1997 = in
Premises considered, the instant appeal has to be, as it is accordance with the attached Salary
hereby denied for lack of legal basis. Consequently, the Notice Schedule marked as Annex B of this
of Disallowance issued by the NEA Auditor covering the Order. The adjustment shall be to the
subject disbursement is hereby sustained. Accordingly, all designated salary step of the
NEA officials and employees who received compensation and employee in the salary grade
allowances in violation of the provisions of Executive Order No. allocation of his position as of October
389 and National Budget Circular No. 458 are hereby directed 31, 1997.
to refund the same within a period of one year after the x x x.
promulgation of this decision. NEA management is enjoined to
effect said refund under the supervision of the NEA Auditor The Department of Budget and Management (DBM for
who shall ensure the proper and strict implementation of this brevity) issued Implementing Guidelines under National
decision.[2] Budget Circular No. 458 (NBC No. 458), series of
1997, reiterating the schedule of payments in EO 389.
The Antecedent Facts In January 1997, NEA implemented the salary increases
prescribed for the year 1997 pursuant to Joint Resolution No.
Petitioner National Electrification Administration (NEA for 01. However, NEA did not implement the salary increases in
brevity) is a government-owned and controlled corporation accordance with the schedule of payment specified in EO 389
created under Presidential Decree No. 269, as and NBC No. 458. Instead, NEA implemented in one lump
amended. NEA is charged with the responsibility of sum beginning January 1, 1997 the salary increases
organizing, financing and regulating electric cooperatives required to be paid in two tranches, the first tranche
throughout the country. on January 1, 1997 and the second tranche on November
1, 1997. Otherwise stated, NEA accelerated the
On July 1, 1989, Republic Act No. 6758 (RA 6758), implementation of the salary increase by paying the
entitled An Act Prescribing a Revised Compensation and second tranche starting January 1, 1997 instead
Position Classification System in the Government and For of November 1, 1997.
Other Purposes, took effect. RA 6758 provided, among others,
a salary schedule for all government positions, appointive On September 26, 1997, the Commissions resident
or elective, including positions in government-owned or auditor in NEA issued a Notice of Suspension requiring the
controlled corporations and government financial institutions. submission of the legal basis for the full implementation of the
new salary schedule effective January 1, 1997 instead
In response to pressing economic difficulties and the of November 1, 1997. The NEA failed to submit the basis for
need to alleviate the plight of government personnel, the its advance implementation of the prescribed salary
Senate and the House of Representatives passed on March 3, rates. Thus, the Commissions resident auditor issued on May
1994 Joint Resolution No. 01 entitled Urging the President of 14 and 27, 1998, Notices of Disallowance Nos. 98-010-101
the Philippines to Revise the Existing Compensation and and 98-011-101, respectively. The resident auditor issued
Position Classification System in the Government and to another Notice of Disallowance on September 18,
Implement the Same Initially Effective January 1, 1998. On September 28, 1998 the resident auditor denied
1994. Approved by then President Fidel V. Ramos on March 7, NEAs September 23, 1998 request to reconsider the
1994, Joint Resolution No. 01 adjusted the salary schedule of disallowance. Consequently, NEA appealed to the Corporate
all officials and employees of the government. Paragraph 10 of Audit Office II of the Commission but the appeal was denied
Joint Resolution No. 01 provides that the new salary on February 5, 1999. On March 12, 1999, NEA filed an appeal
schedule shall be implemented within four (4) with the Commission en banc but the latter denied the same
years beginning in 1994. on May 16, 2000 and sustained the disallowance made by the
resident auditor.
On December 28, 1996, then President Fidel V. Ramos
issued Executive Order No. 389 (EO 389) Hence, this Petition.
entitled Implementing the Fourth and Final Year Salary
Increases Authorized by Joint Senate and House of Ruling of the Commission on Audit
Representatives Resolution No. 01, Series of 1994. EO 389
directed payment of the fourth and final salary increases In sustaining the disallowance made by the resident
auditor, the Commission explained thus:
FINALS CONSTITUTION I ACJUCO 74

After a careful evaluation of the facts and pertinent laws constitute unbridled authority to government agencies to spend
obtaining in this case, this Commission finds the instant the appropriated amounts as they may wish.
appeal bereft of merit. Pursuant to Article 29 (1) of the 1987
Constitution No money shall be paid out of the Treasury except Pursuant to the provisions on National Government
in pursuance of an appropriation made by law. Also, under Budgeting[5] found in the Revised Administrative Code of 1987
R.A. 8244, a law appropriating twenty-seven billion pesos for (Administrative Code), appropriations for Personal Services
the fourth and final year of implementation of the salary are not itemized. Thus, the 1997 GAA contains a lump sum
increases pursuant to the Senate-House of Representatives appropriation of P210, 766,000.00 for NEAs Personal
Resolution No. 01 Series of 1994 for all National Government Services, broken down into P37, 476,000.00 for General
civilian and uniformed personnel, it is specifically provided that Administration and Support, P103, 855,000.00 for Support to
the salary increases shall be effective on the following Operations, and P69, 435,000.00 for Operations. There is no
schedule of payments: itemization of Personal Services in the 1997 GAA, and
nothing is mentioned therein about the acceleration or full
payment of the Salary Standardization Law II.
1. Effective January 1, 1997 for the first 50% of the
unimplemented balance as of December 31, The itemization of Personal Services is
1996; and prepared after the enactment of the annual GAA and
requires the approval of the President. Thus, Section 23,
2. Effective November 1, 1997 the remaining fifty Chapter 4, Book IV of the Administrative Code provides that:
percent (50%) of said unimplemented balance
to effect full salary adjustment.
SEC. 23. Content of the General Appropriations Act. The
Perusal of the provision of E.O. No. 389 and National Budget General Appropriations Act shall be presented in the form of
Circular No. 458 Series of 1997 would show the same budgetary programs and projects for each agency of the
effectivity dates or schedule of payments. Suffice it to say, that government, with the corresponding appropriations for each
the aforequoted provisions of law treating on the subject salary program and project, including statutory provisions of specific
implementation is clear and unequivocal such that there could agency or general applicability. The General Appropriations
never be any room for a different interpretation regarding the Act shall not contain any itemization of personal services,
effectivity dates except that which is explicitly stated which shall be prepared by the Secretary after enactment
therein. Thus, when the NEA effected full implementation of of the General Appropriations Act, for consideration and
the new salary schedule on January 1, 1997, instead approval of the President. (Emphasis supplied)
of November 1, 1997, NEA was, then, clearly acting in
violation of the mandates of the law. Consequently, said
Further, the execution of the annual GAA is subject to a
wrongful implementation must be struck down for being program of expenditure to be approved by the President and
baseless and unlawful, and all its employees who received the this approved program of expenditure is the basis for the fund
undue increases must necessarily return the amount thus release. Thus, Section 34, Chapter 5, Book IV of the
received. Administrative Code states that
The Issues
Sec. 34. Program of Expenditure - The Secretary of Budget
In its Memorandum,[3] NEA avers that the Commission shall recommend to the President the years program of
committed grave abuse of discretion amounting to lack or expenditure for each agency of the government on the basis of
excess of jurisdiction in disallowing the increased salaries of authorized appropriations.The approved expenditure
NEAs officials and employees for the period January 1, program shall constitute the basis for fund release during
1997 to October 31, 1997 for the following reasons: the fiscal period, subject to such policies, rules and
regulations as may be approved by the
1. NEAs accelerated implementation of SSL II is in President. (Emphasis supplied)
accordance with law, Joint Senate-House of
Representatives Resolution No. 01 dated
Moreover, Section 60, Chapter 7, Book VI of the
March 3, 1994, particularly Section 10 thereof
Administrative Code provides that no portion of the
x x x.
appropriations in the GAA shall be used for payment of any
2. The fund to pay such increase had the salary increase or adjustment unless specifically authorized by
imprimatur of the DBM and was included in the law or appropriate budget circular. It reads:
General Appropriations Act of 1997 (R.A. 8250)
x x x.[4] SEC. 60. Restrictions on Salary Increases. No portion of the
appropriations provided in the General Appropriations Act
In the main, NEA argues that it may accelerate the
shall be used for payment of any salary increase or
implementation of the salary increases for the year 1997 due
adjustment unless specifically authorized by law or
to the availability of funds
appropriate budget circular nor shall any appropriation for
The Courts Ruling salaries authorized in the General Appropriations Act, save as
otherwise provided for under the Compensation and Position
The Petition has no merit. Classification Act, be paid unless the positions have been
classified by the Budget Commission. (Emphasis supplied)
First, we find that NEAs accelerated implementation of
the Salary Standardization Law II is not in accordance with
law. Finally, Section 33 of the 1997 GAA itself expressly
provides that the salary increases authorized by the Senate-
We reject NEAs claim that Republic Act No. 8250, House of Representatives Joint Resolution No. 01 or the
otherwise known as the General Appropriations Act of 1997 Salary Standardization Law II are subject to approval by the
(1997 GAA), serves as legal basis for NEAs accelerated President. It reads:
implementation of the last phase of the Salary Standardization
Law II. The 1997 GAA is not self-executory so as to serve as
Sec. 33. Compensation Adjustment and Productivity Incentive
outright legal authority for NEA to spend what had been
Benefits. The amount authorized for Compensation
appropriated for NEAs Personal Services under the 1997 Adjustment and Productivity Incentive Benefits shall be used
GAA. Budgetary appropriations under the GAA do not
FINALS CONSTITUTION I ACJUCO 75

for the adjustment in basic salary and associated benefits of authorizes GOCCs with insufficient funds to partially
national government personnel pursuant to Joint Resolution implement the prescribed salary increases in a uniform and
No. 01, s. 1994 of Congress, as well as Productivity Incentive non-discriminatory manner. Nothing in Section 10
Benefits as may be approved by the President: PROVIDED, authorizes GOCCs with sufficient funds to accelerate the
That such compensation adjustment shall be fully implemented prescribed schedule of salary increases. Clearly, Section
within FY 1997: PROVIDED, FURTHER, That transportation 10 of EO 389 does not authorize, expressly or impliedly, the
allowance, if any, shall be deducted from or reduced by the advance implementation of the salary increases just because
salary adjustment: PROVIDED, FURTHERMORE, That a GOCC has the available funds.
compensation adjustment for government-owned or controlled
corporations and local government units shall be charged to NEA also contends that its accelerated implementation
their corporate and local funds, respectively: xxx. (Emphasis of the salary increases is supported by the Memorandum of
supplied) the Office of the President dated November 7, 1995, the
subject of which reads, xxx: Authorizing the Acceleration of the
Implementation of the Revised Compensation and Position
Clearly, NEA cannot automatically spend its authorized Classification Plan provided in Senate-House of
appropriation for Personal Services under the 1997 GAA. The Representatives Joint Resolution No. 01 Adopted and
Budget Secretary must first prepare an itemization of the Approved on 07 March 1994 to Government-Owned and/or
Personal Services, and submit the same for approval of the Controlled Corporations (GOCCs) and Government Financial
President. Next, the Budget Secretary must recommend to the Institutions (GFIs). According to NEA, the Memorandum allows
President NEAs program of expenditure for the current year full implementation of salary increases x x x not earlier
based on NEAs authorized appropriation. The President may than November 1, 1996. The specific provision referred to by
approve the expenditure program subject to certain policies NEA reads as follows:
and rules. The salary adjustments as well as the associated
benefits granted by the Salary Standardization Law II are,
under the 1997 GAA, expressly subject to the Presidents The three tranches scheme for GOCCs are as follows:
approval. Appropriations for salary increases or adjustments
shall be released as specifically authorized by law or FIRST - effective not earlier than 01 November 1997 at an
appropriate budget circular, which in this case is National amount as may be determined by the governing Board of the
Budget Circular No. 458. Hence, compliance with said budget GOCC concerned, provided such amount shall not exceed
circular is mandatory. 30% of the unimplemented balance of said Salary Schedule;
The rules on National Government Budgeting as
prescribed by the Administrative Code are not idle or empty SECOND - the 30% of the said balance or any lower amount
exercises. The mere approval by Congress of the GAA does as may be determined by the governing Board of the
not instantly make the funds available for spending by the concerned GOCC may be implemented not earlier than 01
Executive Department. The funds authorized for disbursement April 1996; and
under the GAA are usually still to be collected during the fiscal
year. The revenue collections of the government, largely from THIRD- the remaining balance may be implemented not
taxes, may fall short of the approved budget, as has been the earlier than 01 November 1996. (Emphasis supplied)
normal occurrence almost every year.
This puts the Executive Department in a dilemma: borrow The Memorandum, which allows full implementation of
money to bridge the deficit, or cut down on spending even if the salary increases NOT earlier than November 1, 1996,
the expenditure is authorized by the general appropriations does not automatically accelerate the staggered salary
law. Borrowing money locally puts an upward pressure on increases for 1997. On the contrary, the Memorandum
interest rates, while borrowing from abroad increases our specifically provides that accelerated implementation can be
foreign debt stock and eventually puts a downward pressure availed of by GOCCs and GFIs x x x only upon prior
on the peso. On the other hand, cutting down on spending approval of the DBM. In order to secure such prior approval
impairs the delivery of basic services and dampens the from the DBM, GOCCs and GFIs must submit an application
economy. The Executive Department must balance for acceleration to the DBM which will evaluate and act on the
carefully these economic and social factors, and to do this same on the basis of nine terms and conditions specifically
it must calibrate government disbursements to match, as much enumerated in the Memorandum. The Memorandum provides
as possible, receipt of revenues. This is the rationale behind thus:
the rules on National Government Budgeting.
Next, NEA argues that an intention to exempt adequately The GOCC and GFI can avail of the above accelerated
funded government-owned or controlled corporations (GOCCs implementation only upon prior approval by the DBM. For
for brevity) from the two-tranche payment can be gleaned from this purpose, GOCC and GFI will submit an application for
the last paragraph of Section 10 of EO 389 which reads: acceleration to DBM which will evaluate and act on same on
the basis of the following terms and conditions:
GOCCs, GFIs and LGUs which do not have adequate or
sufficient funds to pay the salary increases prescribed herein, 1. the GOCC and GFI shall have never been
may only partially implement the established rate; Provided, seriously/critically assailed to have caused or
That, any partial implementation should be fixed at a uniform contributed to the economic problems of the
percentage such that no official or employee shall receive a country as evidenced by duly verified/proven
percentage adjustment higher than that of any other facts presented in a responsible published
official/employee in the same corporate entity and local public criticism;
government unit. 2. That it must not have received any subsidy or
other forms of financial support from the
The interpretation placed by NEA on Section 10 does not national government in financing its operation
find support in the text thereof expressium facit cessare or in the implementation of projects for the last
tacitum what is expressed puts an end to that which is three (3) years;
implied.[6]Section 10 refers only to GOCCs with insufficient
funds to pay the salary increases. Section 10 expressly 3. that its operational performance for the same
period, as well as its present financial position,
FINALS CONSTITUTION I ACJUCO 76

is indicative that the concerned GOCC and GFI executive order to be issued by the Office of the
will remain financially viable and capable of President. (Emphasis supplied)
financing its operations;
4. that it has actually remitted all mandatory As the administrative head of the government, the
dividends to the national government through President is vested with the power to execute, administer
the National Treasury equivalent to 50% of its and carry out laws into practical operation. Hence, the
net income pursuant to R.A. No. 7656, dated Court has held that -
09 November 1993, and has no unpaid taxes
due the national government or local While Congress is vested with the power to enact laws, the
government units, and their respective President executes the laws. The executive power is vested
agencies and instrumentalities; in the President. It is generally defined as the power to enforce
and administer the laws.It is the power of carrying (out) the
5. that all advances made by the national laws into practical operation and enforcing their due
government for debt service and other observance.[10]
obligations shall have been accordingly
liquidated;
There could be no doubt that EO 389 has been issued
6. that it has not incurred any losses from on authority and within the confines of the law. Joint Resolution
operations for the last three (3) years; No. 01 established a time frame of four years [11] for the
implementation of the Salary Standardization Law II.
7. that the financial position and earning Consonant with this time frame, the initial implementation was
performance of the GOCC and GFI shall in no effected in 1994 through Executive Order No. 164; in 1995
case be affected by SSL acceleration; through Executive Order No. 218; in 1996 through Executive
8. that the accelerated implementation herein Order No. 290 and clarified by Presidential Memorandum to
authorized shall strictly be based on the the Secretary of Budget and Management dated November 7,
Position Allocation List (PAL) specifically 1995. For the fourth and final year, Executive Order No. 389
approved by the DBM for such GOCC and GFI dated December 28, 1996 was issued by the President. Oddly,
pursuant to R.A. No. 6758, or Organizational NEA does not question the authority of the President to issue
Structure and Staffing Pattern pursuant to the executive orders implementing the Salary Standardization
existing budgeting laws, and shall be based on Law II previous to EO 389. Apparently, NEA complied with the
the 33-grade Salary Schedule; and previous executive orders implementing Joint Resolution No.
01.
9. that no funding support shall be required from
the national government nor funds already NEA argues that the Commission failed to take note that
released and earmarked for a specific purpose RA 8244, which provides for the same schedule of payment as
be used therefore. Funds for the purpose shall EO 389 and NBC No. 458, is intended only for all national
solely be sourced from corporate funds: government civilian and uniformed personnel and not GOCCs
and GFIs. A reading of the decision of the Commission would
show that reference to RA 8244 by the Commission was
x x x. (Emphasis supplied) resorted to give effect to the relevant law and rules. Since RA
8244 and EO 389 are in pari materia, relating as they are to
Evidently, in order to avail of the benefits of accelerated the fourth year implementation of the salary increases
implementation, NEA must secure the approval of the DBM by authorized by Joint Resolution No. 01, the Commission applied
complying with the terms and conditions prescribed by the said law and rules in harmony with each other. The
Memorandum. NEA failed to do this. Absent any authority or Commission thus stated that a perusal of RA 8244, EO 389
approval from the DBM or the President authorizing NEA to and NBC No. 458 would show the same effectivity dates or
accelerate implementation of the last phase of the salary schedule of payments.
increase, NEAs accelerated payment is without legal basis.
Similarly untenable is NEAs contention that the
Neither could NEA successfully assail the authority of the Commission acted beyond the scope of its functions in
President to issue EO 389. The Administrative Code has determining whether or not NEA violated the law. According to
unequivocally vested the President with rule-making powers in NEA, the Commission exceeded its authority in inquiring
the form of executive orders, administrative orders, whether NEAs advance release of the salary increases
proclamations, memorandum orders and circulars and general violated certain laws considering that the Commissions power
or special orders.[7] An executive order, like the one prescribing is limited to a determination of whether or not there is a law
the salary schedules, is defined in the Administrative Code as appropriating funds for that purpose. To support this theory,
follows: NEA cites Guevara vs. Gimenez,[12] wherein the Supreme
Court allegedly outlined the scope of authority of the
Commission as follows:
Sec. 2. Executive Orders. Acts of the President providing for
rules of a general or permanent character in implementation or
execution of constitutional or statutory powers shall be Under the Constitution, the authority of the Auditor General in
promulgated in executive orders.[8] (Italics supplied) connection with the expenditures of the government is limited
to the auditing of expenditures of fund or property pertaining
to, or held in trust by, the government or the provinces or
Joint Resolution No. 01 expressly acknowledges the municipalities thereof. xxx xxx Such function is limited to a
authority of the President to revise the existing compensation determination of whether there is a law appropriating funds for
and position classification under the standards and guidelines a given purpose.
provided by said Resolution.[9] Further, paragraph 13 of the
Resolution states that:
The ruling in Guevara has already been overturned by
the Court in Caltex Philippines, Inc. vs. Commission on
(13) Implementing Guidelines - The Department of Budget and Audit,[13] as follows:
Management shall prepare and issue the necessary guidelines
for the implementation of the revised compensation and
position classification system consistent with the governing
FINALS CONSTITUTION I ACJUCO 77

The ruling on this particular point, quoted by petitioner from the proposed budget cannot be deemed sufficient authority to
cases of Guevara vs. Gimenez and Ramos vs. Aquino, are no execute the same in disregard of the relevant orders and
longer controlling as the two (2) were decided in the light of the circulars providing for its manner of execution. The budget
1935 Constitution. process is a cycle of sequential and interrelated budget
activities regularly recurring within a specific time frame (a
xxx. As observed by one of the Commissioners of the 1986 twelve-month period called fiscal year).[15]
Constitutional Commission, Fr. Joaquin G. Bernas: The DBMs approval of NEAs proposed budget is only a
part of the first phase of the entire budget process which
It should be noted, however, that whereas under Article XI, consists of four major phases, namely: Budget Preparation,
Section 2, of the 1935 Constitution the Auditor General could Budget Authorization, Budget Execution and Budget
not correct irregular, unnecessary, excessive or extravagant Accountability.[16] After approval of the proposed budget by the
expenditures of public funds but could only bring [the matter] DBM, the same is submitted to Congress for evaluation and
to the attention of the proper administrative officer, under the inclusion in the appropriations law which sets forth the
1987 Constitution, as also under the 1973 Constitution, the authorized appropriations of the departments and
Commission on Audit can promulgate accounting and auditing agencies. However, this authorization does not include the
rules and regulations including those for the prevention and authority to disburse. A program of expenditures is first
disallowance of irregular, unnecessary, excessive, prepared showing approved programs and projects. An
extravagant, or unconscionable expenditures or uses of itemization of personal services is also prepared listing
government funds and properties. Hence, since the authorized itemized positions and their corresponding
Commission on Audit must ultimately be responsible for the classifications and authorized salaries. As clearly stated in
enforcement of these rules and regulations, the failure to Section 60, Chapter 7, Book VI of the Administrative Code, no
comply with these regulations can be a ground for disapproving portion of the appropriations in the GAA shall be used for
the payment of a proposed expenditure. payment of any salary increase or adjustment unless
specifically authorized by law or appropriate budget
circular.[17] NBC No. 458 is the appropriate budget circular
Indeed, the powers of the Commission as provided in the
referred to by the law with respect to the payment of the last
1987 Constitution are broader and more extensive. Section 2,
phase of the Salary Standardization Law II.
Paragraph D, Article IX of the 1987 Constitution reads:
Third, under our system of government all executive
Sec. 2. (1) The Commission on Audit shall have the power, departments, bureaus and offices are under the control of the
authority and duty to examine, audit, and settle all accounts President of the Philippines. This precept is embodied in
pertaining to the revenue and receipts of, and expenditures or Article VII, Section 17 of the Constitution which provides as
uses of funds and property, owned or held in trust by, or follows:
pertaining to, the government, or any of its subdivisions,
agencies, or instrumentalities, including government-owned Sec. 17. The President shall have control of all the executive
and controlled corporations with original charters and on a departments, bureaus and offices. He shall ensure that the
post-audit basis: (a) constitutional bodies, commissions and laws be faithfully executed.
offices that have been granted fiscal autonomy under this
Constitution; (b) autonomous state colleges and universities;
(c) other government-owned or controlled corporations and The presidential power of control over the executive
their subsidiaries; and (d) such non-governmental entities branch of government extends to all executive employees from
receiving subsidy or equity, directly or indirectly, from or Cabinet Secretary to the lowliest clerk.[18] The constitutional
through the Government, which are required by law or the vesture of this power in the President is self-executing and
granting institution to submit to such audit as a condition of does not require statutory implementation, nor may its exercise
subsidy or equity. x x x. be limited, much less withdrawn, by the legislature. [19]
Executive officials who are subordinate to the President
(2) The Commission shall have exclusive authority, subject to should not trifle with the Presidents constitutional power of
the limitations in the Article, to define the scope of its audit and control over the executive branch. There is only one Chief
examination, establish the techniques and methods required Executive who directs and controls the entire executive
therefor, and promulgate accounting and auditing rules and branch[20], and all other executive officials must implement in
regulations, including those for the prevention and good faith his directives and orders. This is necessary to
disallowance of irregular, unnecessary, excessive, provide order, efficiency and coherence in carrying out the
extravagant, or unconscionable expenditures, or uses of plans, policies and programs of the executive branch.
government funds and properties.
This case would not have arisen had NEA complied in
good faith with the directives and orders of the President in the
[14]
The Constitution and existing laws mandate the implementation of the last phase of the Salary Standardization
Commission to audit all government agencies, including Law II. The directives and orders are clearly and manifestly in
government-owned or controlled corporations. The accordance with all relevant laws. The reasons advanced by
Constitution specifically vests in the Commission the authority NEA in disregarding the Presidents directives and orders are
to determine whether government entities comply with laws patently flimsy, even ill-conceived. This cannot be
and regulations in the disbursement of government funds and countenanced as it will result in chaos and disorder in the
to disallow illegal or irregular disbursements of government executive branch to the detriment of public service.
funds.
WHEREFORE, the instant petition is DISMISSED for
Second, there is no merit in NEAs contention that the lack of merit and the Decision of the Commission on Audit
DBM, upon its approval of NEAs proposed budget, had dated May 16, 2000 is AFFIRMED in toto.
effectively stamped its imprimatur on the accelerated
implementation of the salary increases starting January 1, SO ORDERED.
1997 because NEAs proposed budget for 1997 included funds
for such accelerated implementation. This is not the approval
contemplated by the Presidential Memorandum
dated November 7, 1995, which requires compliance with
specific terms and conditions. The DBMs approval of NEAs
FINALS CONSTITUTION I ACJUCO 78

[G.R. No. 132988. July 19, 2000] SECTION 1. All government departments and agencies,
including state universities and colleges, government-owned
AQUILINO Q. PIMENTEL JR., petitioner, vs. Hon. and controlled corporations and local governments units will
ALEXANDER AGUIRRE in his capacity as Executive identify and implement measures in FY 1998 that will reduce
Secretary, Hon. EMILIA BONCODIN in her capacity as total expenditures for the year by at least 25% of
Secretary of the Department of Budget and authorized regular appropriations for non-personal services
Management, respondents. items, along the following suggested areas:
ROBERTO PAGDANGANAN, intervenor.
1. Continued implementation of the streamlining
DECISION policy on organization and staffing by deferring
action on the following:
PANGANIBAN, J.:
a. Operationalization of new agencies;
The Constitution vests the President with the power of
supervision, not control, over local government units
(LGUs). Such power enables him to see to it that LGUs and b. Expansion of organizational units and/or creation of
their officials execute their tasks in accordance with law. While positions;
he may issue advisories and seek their cooperation in solving
economic difficulties, he cannot prevent them from performing c. Filling of positions; and
their tasks and using available resources to achieve their
goals. He may not withhold or alter any authority or power
d. Hiring of additional/new consultants, contractual and casual
given them by the law. Thus, the withholding of a portion of
personnel, regardless of funding source.
internal revenue allotments legally due them cannot be
directed by administrative fiat.
2. Suspension of the following activities:
The Case
a. Implementation of new capital/infrastructure
Before us is an original Petition for Certiorari and projects, except those which have already been
Prohibition seeking (1) to annul Section 1 of Administrative contracted out;
Order (AO) No. 372, insofar as it requires local government
units to reduce their expenditures by 25 percent of their
b. Acquisition of new equipment and motor
authorized regular appropriations for non-personal
services; and (2) to enjoin respondents from vehicles;
implementing Section 4 of the Order, which withholds a
portion of their internal revenue allotments. c. All foreign travels of government personnel,
except those associated with scholarships and
On November 17, 1998, Roberto Pagdanganan, through trainings funded by grants;
Counsel Alberto C. Agra, filed a Motion for Intervention/Motion
to Admit Petition for Intervention,[1] attaching thereto his
Petition in Intervention[2] joining petitioner in the reliefs d. Attendance in conferences abroad where the
sought. At the time, intervenor was the provincial governor of cost is charged to the government except those
Bulacan, national president of the League of Provinces of the clearly essential to Philippine commitments in the
Philippines and chairman of the League of Leagues of Local international field as may be determined by the
Governments. In a Resolution dated December 15, 1998, the Cabinet;
Court noted said Motion and Petition.
e. Conduct of trainings/workshops/seminars,
The Facts and the Arguments except those conducted by government training
institutions and agencies in the performance of
On December 27, 1997, the President of the Philippines their regular functions and those that are funded
issued AO 372. Its full text, with emphasis on the assailed by grants;
provisions, is as follows:
f. Conduct of cultural and social celebrations and
"ADMINISTRATIVE ORDER NO. 372 sports activities, except those associated with the
Philippine Centennial celebration and those
ADOPTION OF ECONOMY MEASURES IN GOVERNMENT involving regular competitions/events;
FOR FY 1998
g. Grant of honoraria, except in cases where it
WHEREAS, the current economic difficulties brought about by constitutes the only source of compensation from
the peso depreciation requires continued prudence in government received by the person concerned;
government fiscal management to maintain economic stability
and sustain the country's growth momentum; h. Publications, media advertisements and related
items, except those required by law or those
WHEREAS, it is imperative that all government agencies adopt already being undertaken on a regular basis;
cash management measures to match expenditures with
available resources; i. Grant of new/additional benefits to employees,
except those expressly and specifically authorized
NOW, THEREFORE, I, FIDEL V. RAMOS, President of the by law; and
Republic of the Philippines, by virtue of the powers vested in
me by the Constitution, do hereby order and direct: j. Donations, contributions, grants and gifts, except
those given by institutions to victims of calamities.
FINALS CONSTITUTION I ACJUCO 79

3. Suspension of all tax expenditure The Issues


subsidies to all GOCCs and LGUs [3]
The Petition submits the following issues for the
4. Reduction in the volume of consumption of Court's resolution:
fuel, water, office supplies, electricity and
other utilities "A. Whether or not the president committed grave abuse
5. Deferment of projects that are of discretion [in] ordering all LGUS to adopt a 25% cost
encountering significant implementation reduction program in violation of the LGU’s fiscal
problems autonomy

6. Suspension of all realignment of funds and "B. Whether or not the president committed grave abuse
the use of savings and reserves of discretion in ordering the withholding of 10% of the
LGU’s IRA"
SECTION 2. Agencies are given the flexibility to identify the
specific sources of cost-savings, provided the 25% minimum In sum, the main issue is whether (a) Section 1 of AO
savings under Section 1 is complied with. 372, insofar as it "directs" LGUs to reduce their
expenditures by 25 percent; and (b) Section 4 of the same
SECTION 3. A report on the estimated savings generated from issuance, which withholds 10 percent of their internal
these measures shall be submitted to the Office of the revenue allotments, are valid exercises of the President's
President, through the Department of Budget and power of general supervision over local governments.
Management, on a quarterly basis using the attached format.
Additionally, the Court deliberated on the question
whether petitioner had the locus standi to bring this suit,
SECTION 4. Pending the assessment and evaluation despite respondents' failure to raise the issue. [4] However, the
by the Development Budget Coordinating Committee intervention of Roberto Pagdanganan has rendered academic
of the emerging fiscal situation, the amount any further discussion on this matter.
equivalent to 10% of the internal revenue allotment
to local government units shall be withheld. The Court's Ruling

SECTION 5. The Development Budget Coordination The Petition is partly meritorious.


Committee shall conduct a monthly review of the fiscal
position of the National Government and if necessary, Main Issue: Validity of AO 372 Insofar as LGUs Are
shall recommend to the President the imposition of Concerned
additional reserves or the lifting of previously imposed
Before resolving the main issue, we deem it important
reserves.
and appropriate to define certain crucial concepts: (1) the
SECTION 6. This Administrative Order shall take effect scope of the President's power of general supervision over
January 1, 1998 and shall remain valid for the entire year local governments and (2) the extent of the local governments'
unless otherwise lifted. autonomy.

Scope of President's Power of Supervision over LGUs


DONE in the City of Manila, this 27th day of December, in the
year of our Lord, nineteen hundred and ninety-seven." Section 4 of Article X of the Constitution confines the
President's power over local governments to one of
Subsequently, on December 10, 1998, President Joseph general supervision. It reads as follows:
E. Estrada issued AO 43, amending Section 4 of AO 372, by
reducing to five percent (5%) the amount of internal "Sec. 4. The President of the Philippines shall exercise general
revenue allotment (IRA) to be withheld from the LGUs. supervision over local governments. x x x"
Petitioner contends that the President, in issuing AO 372,
was in effect exercising the power of control over LGUs. The This provision has been interpreted to exclude the power
Constitution vests in the President, however, only the power of of control. In Mondano v. Silvosa,[5] the Court contrasted the
general supervision over LGUs, consistent with the principle of President's power of supervision over local government
local autonomy. Petitioner further argues that the directive to officials with that of his power of control over executive officials
withhold ten percent (10%) of their IRA is in contravention of of the national government. It was emphasized that the two
Section 286 of the Local Government Code and of Section 6, terms -- supervision and control -- differed in meaning and
Article X of the Constitution, providing for the automatic extent. The Court distinguished them as follows:
release to each of these units its share in the national internal
revenue.
"x x x In administrative law, supervision means overseeing
The solicitor general, on behalf of the respondents, or the power or authority of an officer to see that subordinate
claims on the other hand that AO 372 was issued to alleviate officers perform their duties. If the latter fail or neglect to fulfill
the "economic difficulties brought about by the peso them, the former may take such action or step as prescribed
devaluation" and constituted merely an exercise of the by law to make them perform their duties. Control, on the other
President's power of supervision over LGUs. It allegedly does hand, means the power of an officer to alter or modify or
not violate local fiscal autonomy, because it nullify or set aside what a subordinate officer ha[s] done in
merely directs local governments to identify measures that will the performance of his duties and to substitute the judgment of
reduce their total expenditures for non-personal services by at the former for that of the latter."[6]
least 25 percent. Likewise, the withholding of 10 percent of the
LGUs IRA does not violate the statutory prohibition on the In Taule v. Santos,[7] we further stated that the Chief
imposition of any lien or holdback on their revenue shares, Executive wielded no more authority than that of checking
because such withholding is "temporary in nature pending the whether local governments or their officials were performing
assessment and evaluation by the Development Coordination their duties as provided by the fundamental law and by
Committee of the emerging fiscal situation." statutes. He cannot interfere with local governments, so
FINALS CONSTITUTION I ACJUCO 80

long as they act within the scope of their control over their acts in the sense that he can substitute
authority. "Supervisory power, when contrasted with control, is their judgments with his own.[21]
the power of mere oversight over an inferior body; it does not
include any restraining authority over such body,"[8] we said.
Decentralization of power, on the other hand, involves an
In a more recent case, Drilon v. Lim,[9] the difference abdication of political power in the favor of local government
between control and supervision was further units declared to be autonomous. In that case, the
delineated. Officers in control lay down the rules in the autonomous government is free to chart its own destiny and
performance or accomplishment of an act. If these rules are shape its future with minimum intervention from central
not followed, they may, in their discretion, order the act undone authorities. According to a constitutional author,
or redone by their subordinates or even decide to do it decentralization of power amounts to 'self-immolation,' since in
themselves. On the other hand, supervision does not cover that event, the autonomous government becomes accountable
such authority. Supervising officials merely see to it that the not to the central authorities but to its constituency." [22]
rules are followed, but they themselves do not lay down such
rules, nor do they have the discretion to modify or replace Under the Philippine concept of local autonomy, the
them. If the rules are not observed, they may order the work national government has not completely relinquished all its
done or redone, but only to conform to such rules. They may powers over local governments, including autonomous
not prescribe their own manner of execution of the act. They regions. Only administrative powers over local affairs are
have no discretion on this matter except to see to it that the delegated to political subdivisions. The purpose of the
rules are followed. delegation is to make governance more directly responsive
and effective at the local levels. In turn, economic, political and
Under our present system of government, executive social development at the smaller political units are expected
power is vested in the President.[10] The members of the to propel social and economic growth and development. But to
Cabinet and other executive officials are merely alter egos. As enable the country to develop as a whole, the programs and
such, they are subject to the power of control of the President, policies effected locally must be integrated and coordinated
at whose will and behest they can be removed from office; or towards a common national goal. Thus, policy-setting for the
their actions and decisions changed, suspended or entire country still lies in the President and Congress. As we
reversed.[11] In contrast, the heads of political subdivisions are stated in Magtajas v. Pryce Properties Corp., Inc., municipal
elected by the people. Their sovereign powers emanate from governments are still agents of the national government. [23]
the electorate, to whom they are directly accountable. By
constitutional fiat, they are subject to the Presidents The Nature of AO 372
supervision only, not control, so long as their acts are
exercised within the sphere of their legitimate powers. By the Consistent with the foregoing jurisprudential precepts, let
same token, the President may not withhold or alter any us now look into the nature of AO 372. As its preambular
authority or power given them by the Constitution and the law. clauses declare, the Order was a "cash management
measure" adopted by the government "to match expenditures
Extent of Local Autonomy with available resources," which were presumably depleted at
the time due to "economic difficulties brought about by the
Hand in hand with the constitutional restraint on the peso depreciation." Because of a looming financial crisis, the
President's power over local governments is the state policy of President deemed it necessary to "direct all government
ensuring local autonomy.[12] agencies, state universities and colleges, government-owned
In Ganzon v. Court of Appeals,[13] we said that local and controlled corporations as well as local governments to
autonomy signified "a more responsive and accountable local reduce their total expenditures by at least 25 percent along
government structure instituted through a system of suggested areas mentioned in AO 372.
decentralization."The grant of autonomy is intended to "break Under existing law, local government units, in addition
up the monopoly of the national government over the affairs of to having administrative autonomy in the exercise of their
local governments, x x x not x x x to end the relation of functions, enjoy fiscal autonomy as well. Fiscal autonomy
partnership and interdependence between the central means that local governments have the power to create their
administration and local government units x x own sources of revenue in addition to their equitable share in
x." Paradoxically, local governments are still subject to the national taxes released by the national government, as well
regulation, however limited, for the purpose of enhancing as the power to allocate their resources in accordance with
self-government.[14] their own priorities. It extends to the preparation of their
Decentralization simply means the devolution of budgets, and local officials in turn have to work within the
national administration, not power, to local constraints thereof. They are not formulated at the national
governments. Local officials remain accountable to the level and imposed on local governments, whether they are
central government as the law may provide. [15] The difference relevant to local needs and resources or not. Hence, the
between decentralization of administration and that of power necessity of a balancing of viewpoints and the harmonization
was explained in detail in Limbona v. Mangelin[16] as follows: of proposals from both local and national officials, [24] who in
any case are partners in the attainment of national goals.

"Now, autonomy is either decentralization of Local fiscal autonomy does not however rule out any
administration or decentralization of power. There is manner of national government intervention by way of
decentralization of administration when the central government supervision, in order to ensure that local programs, fiscal and
delegates administrative powers to political subdivisions in otherwise, are consistent with national goals. Significantly, the
order to broaden the base of government power and in the President, by constitutional fiat, is the head of the economic
process to make local governments 'more responsive and and planning agency of the government, [25] primarily
accountable,'[17] and 'ensure their fullest development as self- responsible for formulating and implementing continuing,
reliant communities and make them more effective partners in coordinated and integrated social and economic policies, plans
the pursuit of national development and social progress.' [18] At and programs[26] for the entire country. However, under the
the same time, it relieves the central government of the burden Constitution, the formulation and the implementation of such
of managing local affairs and enables it to concentrate on policies and programs are subject to "consultations with the
national concerns. The President exercises 'general appropriate public agencies, various private sectors, and local
supervision'[19] over them, but only to 'ensure that local government units." The President cannot do so unilaterally.
affairs are administered according to law.'[20] He has no
FINALS CONSTITUTION I ACJUCO 81

Consequently, the Local Government Code provides: [27] Withholding a Part of LGUs' IRA
Section 4 of AO 372 cannot, however, be upheld. A
"x x x [I]n the event the national government incurs an basic feature of local fiscal autonomy is the automatic release
unmanaged public sector deficit, the President of the of the shares of LGUs in the national internal revenue. This is
Philippines is hereby authorized, upon the recommendation of mandated by no less than the Constitution. [28] The Local
[the] Secretary of Finance, Secretary of the Interior and Local Government Code[29] specifies further that the release shall be
Government and Secretary of Budget and Management, and made directly to the LGU concerned within five (5) days after
subject to consultation with the presiding officers of both every quarter of the year and "shall not be subject to any lien
Houses of Congress and the presidents of the liga, to make the or holdback that may be imposed by the national government
necessary adjustments in the internal revenue allotment of for whatever purpose."[30] As a rule, the term "shall" is a word
local government units but in no case shall the allotment be of command that must be given a compulsory meaning.[31] The
less than thirty percent (30%) of the collection of national provision is, therefore, imperative.
internal revenue taxes of the third fiscal year preceding the
current fiscal year x x x." Section 4 of AO 372, however, orders the withholding,
effective January 1, 1998, of 10 percent of the LGUs' IRA
There are therefore several requisites before the "pending the assessment and evaluation by the
President may interfere in local fiscal matters: (1) an Development Budget Coordinating Committee of the
unmanaged public sector deficit of the national government; emerging fiscal situation" in the country. Such withholding
(2) consultations with the presiding officers of the Senate and clearly contravenes the Constitution and the law. Although
the House of Representatives and the presidents of the temporary, it is equivalent to a holdback, which means
various local leagues; and (3) the corresponding "something held back or withheld, often temporarily." [32] Hence,
recommendation of the secretaries of the Department of the "temporary" nature of the retention by the national
Finance, Interior and Local Government, and Budget and government does not matter. Any retention is prohibited.
Management. Furthermore, any adjustment in the allotment In sum, while Section 1 of AO 372 may be upheld as an
shall in no case be less than thirty percent (30%) of the advisory effected in times of national crisis, Section 4 thereof
collection of national internal revenue taxes of the third fiscal has no color of validity at all. The latter provision effectively
year preceding the current one. encroaches on the fiscal autonomy of local
Petitioner points out that respondents failed to comply governments. Concededly, the President was well-
with these requisites before the issuance and the intentioned in issuing his Order to withhold the LGUs IRA, but
implementation of AO 372. At the very least, they did not even the rule of law requires that even the best intentions must be
try to show that the national government was suffering from an carried out within the parameters of the Constitution and the
unmanageable public sector deficit. Neither did they claim law. Verily, laudable purposes must be carried out by legal
having conducted consultations with the different leagues of methods.
local governments.Without these requisites, the President has Refutation of Justice Kapunan's Dissent
no authority to adjust, much less to reduce, unilaterally the
LGU's internal revenue allotment. Mr. Justice Santiago M. Kapunan dissents from our
Decision on the grounds that, allegedly, (1) the Petition is
The solicitor general insists, however, that AO 372 is premature; (2) AO 372 falls within the powers of the President
merely directory and has been issued by the President as chief fiscal officer; and (3) the withholding of the LGUs IRA
consistent with his power of supervision over local is implied in the President's authority to adjust it in case of an
governments. It is intended only to advise all government unmanageable public sector deficit.
agencies and instrumentalities to undertake cost-reduction
measures that will help maintain economic stability in the First, on prematurity. According to the Dissent, when "the
country, which is facing economic difficulties. Besides, it does conduct has not yet occurred and the challenged construction
not contain any sanction in case of noncompliance. Being has not yet been adopted by the agency charged with
merely an advisory, therefore, Section 1 of AO 372 is well administering the administrative order, the determination of the
within the powers of the President. Since it is not a scope and constitutionality of the executive action in advance
mandatory imposition, the directive cannot be characterized as of its immediate adverse effect involves too remote and
an exercise of the power of control. abstract an inquiry for the proper exercise of judicial function."
While the wordings of Section 1 of AO 372 have a rather This is a rather novel theory -- that people should await
commanding tone, and while we agree with petitioner that the the implementing evil to befall on them before they can
requirements of Section 284 of the Local Government Code question acts that are illegal or unconstitutional. Be it
have not been satisfied, we are prepared to accept the solicitor remembered that the real issue here is whether the
general's assurance that the directive to "identify and Constitution and the law are contravened by Section 4 of
implement measures x x x that will reduce total AO 372, not whether they are violated by the acts
expenditures x x x by at least 25% of authorized regular implementing it. In the unanimous en banc case Taada v.
appropriation" is merely advisory in character, and does Angara,[33] this Court held that when an act of the legislative
not constitute a mandatory or binding order that interferes with department is seriously alleged to have infringed the
local autonomy. The language used, while authoritative, does Constitution, settling the controversy becomes the duty of this
not amount to a command that emanates from a boss to a Court. By the mere enactment of the questioned law or the
subaltern. approval of the challenged action, the dispute is said to have
ripened into a judicial controversy even without any other overt
Rather, the provision is merely an advisory to prevail act. Indeed, even a singular violation of the Constitution and/or
upon local executives to recognize the need for fiscal restraint the law is enough to awaken judicial duty. Said the Court:
in a period of economic difficulty. Indeed, all concerned would
do well to heed the President's call to unity, solidarity and "In seeking to nullify an act of the Philippine Senate on the
teamwork to help alleviate the crisis. It is understood, however, ground that it contravenes the Constitution, the petition no
that no legal sanction may be imposed upon LGUs and their doubt raises a justiciable controversy. Where an action of the
officials who do not follow such advice. It is in this light that we legislative branch is seriously alleged to have infringed the
sustain the solicitor general's contention in regard to Section 1. Constitution, it becomes not only the right but in fact the duty
of the judiciary to settle the dispute. 'The question thus posed
is judicial rather than political. The duty (to adjudicate) remains
FINALS CONSTITUTION I ACJUCO 82

to assure that the supremacy of the Constitution is Notably, Justice Kapunan recognizes the need for
upheld.'[34] Once a 'controversy as to the application or "interaction between the national government and the LGUs at
interpretation of a constitutional provision is raised before this the planning level," in order to ensure that "local development
Court x x x , it becomes a legal issue which the Court is bound plans x x x hew to national policies and standards." The
by constitutional mandate to decide.' [35] problem is that no such interaction or consultation was
xxxxxxxxx ever held prior to the issuance of AO 372. This is why the
"As this Court has repeatedly and firmly emphasized in many petitioner and the intervenor (who was a provincial governor
cases,[36] it will not shirk, digress from or abandon its sacred and at the same time president of the League of Provinces of
duty and authority to uphold the Constitution in matters that the Philippines and chairman of the League of Leagues of
involve grave abuse of discretion brought before it in Local Governments) have protested and instituted this
appropriate cases, committed by any officer, agency, action. Significantly, respondents do not deny the lack of
instrumentality or department of the government." consultation.
In the same vein, the Court also held in Tatad v. In addition, Justice Kapunan cites Section 287[40] of the
Secretary of the Department of Energy:[37] LGC as impliedly authorizing the President to withhold the IRA
of an LGU, pending its compliance with certain
"x x x Judicial power includes not only the duty of the courts to requirements.Even a cursory reading of the provision reveals
settle actual controversies involving rights which are legally that it is totally inapplicable to the issue at bar. It directs LGUs
demandable and enforceable, but also the duty to determine to appropriate in their annual budgets 20 percent of their
whether or not there has been grave abuse of discretion respective IRAs for development projects. It speaks of no
amounting to lack or excess of jurisdiction on the part of any positive power granted the President to priorly withhold any
branch or instrumentality of government. The courts, as amount. Not at all.
guardians of the Constitution, have the inherent authority to WHEREFORE, the Petition is GRANTED. Respondents
determine whether a statute enacted by the legislature and their successors are hereby
transcends the limit imposed by the fundamental law. Where permanently PROHIBITED from implementing Administrative
the statute violates the Constitution, it is not only the right but Order Nos. 372 and 43, respectively dated December 27, 1997
the duty of the judiciary to declare such act unconstitutional and December 10, 1998, insofar as local government units are
and void." concerned.

By the same token, when an act of the President, who in SO ORDERED.


our constitutional scheme is a coequal of Congress, is
seriously alleged to have infringed the Constitution and the
laws, as in the present case, settling the dispute becomes the
duty and the responsibility of the courts.
Besides, the issue that the Petition is premature has not
been raised by the parties; hence it is deemed
waived. Considerations of due process really prevents its use
against a party that has not been given sufficient notice of its
presentation, and thus has not been given the opportunity to
refute it.[38]
Second, on the President's power as chief fiscal officer of the
country. Justice Kapunan posits that Section 4 of AO 372
conforms with the President's role as chief fiscal officer, who
allegedly "is clothed by law with certain powers to ensure the
observance of safeguards and auditing requirements, as well
as the legal prerequisites in the release and use of IRAs, taking
into account the constitutional and statutory mandates." [39] He
cites instances when the President may lawfully intervene in
the fiscal affairs of LGUs.
Precisely, such powers referred to in the Dissent have
specifically been authorized by law and have not been
challenged as violative of the Constitution. On the other hand,
Section 4 of AO 372, as explained earlier, contravenes explicit
provisions of the Local Government Code (LGC) and the
Constitution. In other words, the acts alluded to in the Dissent
are indeed authorized by law; but, quite the opposite, Section
4 of AO 372 is bereft of any legal or constitutional basis.
Third, on the President's authority to adjust the IRA of
LGUs in case of an unmanageable public sector deficit. It must
be emphasized that in striking down Section 4 of AO 372, this
Court is not ruling out any form of reduction in the IRAs of
LGUs. Indeed, as the President may make necessary
adjustments in case of an unmanageable public sector deficit,
as stated in the main part of this Decision, and in line with
Section 284 of the LGC, which Justice Kapunan cites. He,
however, merely glances over a specific requirement in the
same provision -- that such reduction is subject to
consultation with the presiding officers of both Houses of
Congress and, more importantly, with the presidents of
the leagues of local governments.
FINALS CONSTITUTION I ACJUCO 83

[G.R. No. 130775. September 27, 2004] Panlungsod of Caloocan City, and Secretary Barbers.[7] Rayos
alleged that he was elected President of the Liga Caloocan
THE NATIONAL LIGA NG MGA BARANGAY, represented Chapter in the elections held on 14 June 1997 by the members
by ALEX L. DAVID in his capacity as National President of the Caloocan Chapter pursuant to their Resolution/Petition
and for his own Person, President ALEX L. No. 001-97.[8] On 18 July 1997, the presiding judge granted the
DAVID, petitioners, vs. HON. VICTORIA ISABEL A. TRO, enjoining therein respondents David, Quimpo and
PAREDES, Presiding Judge, Regional Trial Court, Branch Secretary Barbers from proceeding with the synchronized
124, Caloocan City, and THE DEPARTMENT OF INTERIOR elections for the Provincial and Metropolitan Chapters of
and LOCAL GOVERNMENT, represented the HON. the Liga scheduled on 19 July 1997, but only for the purpose
SECRETARY ROBERT Z. BARBERS and MANUEL A. of maintaining the status quo and effective for a period not
RAYOS, respondents. exceeding seventy-two (72) hours.[9]

[G.R. No. 131939. September 27, 2004] Eventually, on 18 July 1997, at petitioner Davids
LEANDRO YANGOT, BONIFACIO LACWASAN and BONY instance, Special Civil Action (SCA) No. C-512 pending before
TACIO, petitioners, vs. DILG Secretary ROBERT Z. Branch 126 was consolidated with SCA No. C-508 pending
BARBERS and DILG Undersecretary MANUEL before Branch 124.[10]
SANCHEZ, respondents.
Before the consolidation of the cases, on 25 July 1997,
DECISION
the DILG through respondent Secretary Barbers, filed in
Tinga, J.:
SCA No. C-512 an Urgent Motion,[11] invoking the Presidents
power of general supervision over all local government units
At bottom, the present petition inquires into the essential and seeking the following reliefs:
nature of the Liga ng mga Barangay and questions the extent
of the power of Secretary of the Department of Interior and
WHEREFORE, in the interest of the much-needed delivery of
Local Government (DILG), as alter ego of the President. More
basic services to the people, the maintenance of public order
immediately, the petition disputes the validity of the
and to further protect the interests of the forty-one thousand
appointment of the DILG as the interim caretaker of the Liga
barangays all over the country, herein respondent respectfully
ng mga Barangay.
prays:

On 11 June 1997, private respondent Manuel A. Rayos


a) That the Department of the Interior and Local
[as petitioner therein], Punong Barangay of Barangay 52,
Government (DILG), pursuant to its delegated
District II, Zone 5, District II, Caloocan City, filed a petition for
power of general supervision, be appointed as
prohibition and mandamus, with prayer for a writ of preliminary
the Interim Caretaker to manage and
injunction and/or temporary restraining order and damages
administer the affairs of the Liga, until such
before the Regional Trial Court (RTC) of Caloocan, [1] alleging
time that the new set of National Liga Officers
that respondent therein Alex L. David [now petitioner], Punong
shall have been duly elected and assumed
Barangay of Barangay 77, Zone 7, Caloocan City and then
office; ...[12]
president of the Liga Chapter of Caloocan City and of the Liga
ng mga Barangay National Chapter, committed certain
irregularities in the notice, venue and conduct of the proposed The prayer for injunctive reliefs was anchored on the
synchronized Liga ng mga Barangay elections in following grounds: (1) the DILG Secretary exercises the power
1997. According to the petition, the irregularities consisted of of general supervision over all government units by virtue of
the following: (1) the publication of the notice in the Manila Administrative Order No. 267 dated 18 February 1992; (2) the
Bulletin but without notifying in writing the individual punong Liga ng mga Barangay is a government organization; (3) undue
barangays of Caloocan City;[2](2) the Notice of Meeting dated interference by some local elective officials during the
08 June 1997 for the Liga Chapter of Caloocan City did not Municipal and City Chapter elections of the Liga ng mga
specify whether the meeting scheduled on 14 June 1997 was Barangay; (4) improper issuance of confirmations of the
to be held at 8:00 a.m. or 8:00 p.m., and worse, the meeting elected Liga Chapter officers by petitioner David and the
was to be held in Lingayen, Pangasinan; [3] and (3) the deadline National LigaBoard; (5) the need for the DILG to provide
for the filing of the Certificates of Candidacy having been set remedies measured in view of the confusion and chaos
at 5:00 p.m. of the third day prior to the above election day, or sweeping the Liga ng mga Barangay and the incapacity of the
on 11 June 1997,[4] Rayos failed to meet said deadline since National Liga Board to address the problems properly.
he was not able to obtain a certified true copy of the COMELEC
Certificate of Canvas and Proclamation of Winning Candidate, On 31 July 1997, petitioner David opposed the
which were needed to be a delegate, to vote and be voted for DILGs Urgent Motion, claiming that the DILG, being a
in the Liga election. On 13 June 1997, the Executive Judge respondent in the case, is not allowed to seek any sanction
issued a temporary restraining order (TRO), effective for against a co-respondent like David, such as by filing a cross-
seventy-two (72) hours, enjoining the holding of the general claim, without first seeking leave of court. [13] He also alleged
membership and election meeting of Liga Chapter that the DILGs request to be appointed interim caretaker
of Caloocan City on 14 June 1975.[5] constitutes undue interference in the internal affairs of
the Liga, since the Liga is not subject to DILG control and
However, the TRO was allegedly not properly served on supervision.[14]
herein petitioner David, and so the election for the officers of
the Liga-Caloocan was held as scheduled.[6] Petitioner David Three (3) days after filing its Urgent Motion, on 28 July
was proclaimed President of the Liga-Caloocan, and thereafter 1997, and before it was acted upon by the lower court, the
took his oath and assumed the position of ex-officio member of DILG through then Undersecretary Manuel Sanchez, issued
the Sangguniang Panlungsod of Caloocan. Memorandum Circular No. 97-176.[15] It cited the reported
violations of the Liga ng mga Barangay Constitution and By-
On 17 July 1997, respondent Rayos filed a second Laws by David and widespread chaos and confusion among
petition, this time for quo warranto, mandamus and prohibition, local government officials as to who were the qualified ex-
with prayer for a writ of preliminary injunction and/or temporary officio Liga members in their
restraining order and damages, against David, Nancy Quimpo, respective sangunians.[16] Pending the appointment of the
Presiding Officer of the Sangguniang DILG as the Interim Caretaker of the Liga ng mga Barangay by
FINALS CONSTITUTION I ACJUCO 84

the court and until the officers and board members of the judge issued the questioned order, in mockery of the justice
national Liga Chapter have been elected and have assumed system. He implied that Secretary Barbers knew about
office, the Memorandum Circular directed all provincial respondent judges questioned order even before it was
governors, vice governors, city mayors, city vice mayors, promulgated.[23]
members of the sangguniang panlalawigan and panlungsod,
DILG regional directors and other concerned officers, as On 11 August 1997, the DILG issued Memorandum
follows: Circular No. 97-193,[24] providing supplemental guidelines for
the 1997 synchronized elections of the provincial and
1. All concerned are directed not to recognize and/or honor any metropolitan chapters and for the election of the national
Liga Presidents of the Provincial and Metropolitan Chapters as chapter of the Liga ng mga Barangay. The Memorandum
ex-officio members of the sanggunian concerned until further Circular set the synchronized elections for the provincial and
notice from the Courts or this Department; metropolitan chapters on 23 August 1997 and for the national
chapter on 06 September 1997.
2. All concerned are directed to disregard any pronouncement
and/or directive issued by Mr. Alex David on any issue or On 12 August 1997, the DILG issued a Certificate of
matter relating to the affairs of the Liga ng mga Barangay until Appointment[25] in favor of respondent Rayos as president
further notice from the Courts or this Department. [17] of the Liga ng mga Barangay of Caloocan City. The
appointment purportedly served as Rayos’ legal basis for ex-
On 04 August 1997, public respondent Judge Victoria officio membership in the Sangguniang
Isabel A. Paredes issued the assailed order, [18] the pertinent Panlungsod of Caloocan City and to qualify and participate in
portions of which read, thus: the forthcoming National Chapter Election of the Liga ng mga
Barangay.[26]
The authority of the DILG to exercise general supervisory
jurisdiction over local government units, including the different On 23 August 1997, the DILG conducted the
leagues created under the Local Government Code of 1991 synchronized elections of Provincial and
(RA 7160) finds basis in Administrative Order No. 267 dated Metropolitan Liga Chapters. Thereafter, on 06 September
February 18, 1992. Specifically, Section 1 (a) of the said 1997, the National Liga Chapter held its election of officers and
Administrative Order provides a broad premise for the board of directors, wherein James Marty L. Lim was elected as
supervisory power of the DILG. Administratively, the DILGs President of the National Liga.[27]
supervision has been tacitly recognized by the local
barangays, municipalities, cities and provinces as shown by On 01 October 1997, public respondent judge denied
the evidences presented by respondent David himself (See Davids motion for reconsideration,[28] ruling that there was no
Annexes A to C). The fact that the DILG has sought to refer the factual or legal basis to reconsider the appointment of the DILG
matters therein to the National Liga Board/Directorate does as interim caretaker of the National Liga Board and to cite
not ipso facto mean that it has lost jurisdiction to act directly Secretary Barbers in contempt of court.[29]
therein. Jurisdiction is conferred by law and cannot be claimed
or lost through agreements or inaction by individuals. What On 10 October 1997, petitioners filed the instant Petition
respondent David may term as interference should for Certiorari[30] under Rule 65 of the Rules of Court, seeking to
caretakership be allowed, this Court would rather view as a annul public respondent judges orders of 04 August 1997 and
necessary and desirable corollary to the exercise of 01 October 1997. They dispute the latters opinion on the power
supervision.[19] of supervision of the President under the Constitution, through
the DILG over local governments, which is the same as that of
Political motivations must not preclude, hamper, or obstruct the the DILGs as shown by its application of the power on the Liga
delivery of basic services and the perquisites of public ng mga Barangay. Specifically, they claim that the public
service. In this case, the fact of confusion arising from respondent judges’ designation of the DILG as interim
conflicting appointments, non-action, and uninformed or caretaker and the acts which the DILG sought to
wavering decisions of the incumbent National Liga implement pursuant to its designation as such are beyond
Board/Directorate, having been satisfactorily established, the scope of the Chief Executives power of supervision.
cannot simply be brushed aside as being politically motivated
or arising therefrom. It is incumbent, therefore, that the DILG To support the petition, petitioners argue that under
exercise a more active role in the supervision of the affairs and Administrative Order No. 267, Series of 1992, the power of
operations of the National Liga Board/ Directorate at least until general supervision of the President over local government
such time that the regular National Liga Board/Directorate may units does not apply to the Liga and its various chapters
have been elected, qualified and assumed office. [20] precisely because the Liga is not a local government unit,
contrary to the stance of the respondents. [31]
xxx
Section 507 of the Local Government Code (Republic Act
WHEREFORE, premises considered, the Urgent Motion of the No. 7160)[32] provides that the Liga shall be governed by its
DILG for appointment as interim caretaker, until such time that own Constitution and By-laws. Petitioners posit that the duly
the regularly elected National Liga Board of Directors shall elected officers and directors of the National Liga elected in
have qualified and assumed office, to manage and administer 1994 had a vested right to their positions and could only be
the affairs of the National Liga Board, is hereby GRANTED. [21] removed therefrom for cause by affirmative vote of two-thirds
(2/3) of the entire membership pursuant to
On 11 August 1997, petitioner David filed an urgent the Liga Constitution and By-Laws, and not by mere issuances
motion for the reconsideration of the assailed order and to of the DILG, even if bolstered by the dubious authorization of
declare respondent Secretary Barbers in contempt of respondent judge.[33] Thus, petitioners claim that the
Court.[22] David claimed that the 04 August 1997 order divested questioned order divested the then incumbent officers and
the duly elected members of the Board of Directors of directors of the Liga of their right to their respective offices
the Liga National Directorate of their positions without due without due process of law.
process of law. He also wanted Secretary Barbers declared in
contempt for having issued, through his Undersecretary, Assuming the Liga could be subsumed under the term
Memorandum Circular No. 97-176, even before respondent local governments, over which the President, through the DILG
FINALS CONSTITUTION I ACJUCO 85

Secretary, has the power of supervision, [34] petitioners point In the interim, another petition, this time for Prohibition
out that still there is no legal or constitutional basis for the with Prayer for a Temporary Restraining Order, [53] was filed by
appointment of the DILG as interim caretaker. [35] They stress several presidents of Liga Chapters, praying that this Court
that the actions contemplated by the DILG as interim caretaker declare the DILG Secretary and Undersecretary are not vested
go beyond supervision, as what it had sought and obtained with any constitutional or legal power to exercise control or
was authority to alter, modify, nullify or set aside the actions of even supervision over the National Liga ng mga Barangay, nor
the Liga Board of Directors and even to substitute its judgment to take over the functions of its officers or suspend its
over that of the latter which are all clearly one of constitution; and declare void any and all acts committed by
control.[36] Petitioners question the appointment of Rayos respondents therein in connection with their caretakership of
as Liga-Caloocan President since at that time petitioner David the Liga.[54] The petition was consolidated with G.R. No.
was occupying that position which was still the subject of the 130775, but it was eventually dismissed because the
quo warranto proceedings Rayos himself had petitioners failed to submit an affidavit of service and proof of
instituted.[37] Petitioners likewise claim that DILG service of the petition.[55]
Memorandum Circular No. 97-193, providing supplemental
guidelines for the synchronized elections of the Liga, replaced Meanwhile, on 01 December 1998, petitioner David died
the implementing rules adopted by the Liga pursuant to its and was substituted by his legal representatives. [56]
Constitution and By-laws.[38] In fact, even before its
appointment as interim caretaker, DILG specifically enjoined
ISSUES:
all heads of government units from recognizing petitioner
David and/or honoring any of his pronouncements relating to
the Liga.[39] Petitioners have raised a number of issues.[57] Integrated
and simplified, these issues boil down to the question of
whether or not respondent Judge acted with grave abuse
Petitioners rely on decision in Taule v. Santos,[40] which,
of discretion in appointing the DILG as interim caretaker
they claim, already passed upon the extent of authority of the
to administer and manage the affairs of the
then Secretary of Local Government over the katipunan ng
National Liga Board, per its order dated 04 August 1997.[58] In
mga barangay or the barangay councils, as it specifically ruled
turn, the resolution of the question of grave abuse of discretion
that the Secretary [of Local Government] has no authority to
entails a couple of definitive issues, namely: (1) whether
pass upon the validity or regularity of the election of officers of
the Liga ng mga Barangay is a government organization that
the katipunan.[41]
is subject to the DILG Secretarys power of supervision over
local governments as the alter ego of the President, and (2)
For his part, respondent Rayos avers that since the whether the respondent Judges designation of the DILG as
Secretary of the DILG supervises the acts of local officials by interim caretaker of the Liga has invested the DILG with control
ensuring that they act within the scope of their prescribed over the Liga and whether DILG Memorandum Circular No.
powers and functions and since members of the various 97-176, issued before it was designated as
leagues, such as the Liga in this case, are themselves officials such interim caretaker, and DILG Memorandum Circular No.
of local government units, it follows that the Liga members 97-193 and other acts which the DILG made in its capacity as
are subject to the power of supervision of the DILG.[42] He interim caretaker of the Liga, involve supervision or control of
adds that as the DILGs management and administration of the Liga.
the Liga affairs was limited only to the conduct of the elections,
its actions were consistent with its rule-making power and
However, the Court should first address the question of
power of supervision under existing laws. [43] He asserts that in
mootness which intervenor Lim raised because, according to
assailing the appointment of the DILG as interim
him, during the pendency of the present petition a general
caretaker, petitioners failed to cite any provision of positive law
election was held; the new set of officers and directors had
in support of their stance. Thus, he adds, if a law is silent,
assumed their positions; and that supervening events the DILG
obscure or insufficient, a judge may apply a rule he sees fit to
had turned-over the management and administration of
resolve the issue, as long as the rule chosen is in harmony with
the Liga to new Liga officers and directors.[59] Respondent
general interest, order, morals and public policy, [44] in
Rayos has joined him in this regard. [60] Forthwith, the Court
consonance with Article 9 of the Civil Code. [45]
declares that these supervening events have not rendered the
instant petition moot, nor removed it from the jurisdiction of this
On the other hand, it is quite significant that the Solicitor Court.
General has shared petitioners’ position. He states that the
DILGs act of managing and administering the affairs of the
This case transcends the elections ordered and
National Liga Board are not merely acts of supervision but
conducted by the DILG as interim caretaker of the Liga and
plain manifestations of control and direct takeover of the
the Liga officers and directors who were elected to replace
functions of the National Liga Board,[46] going beyond the limits
petitioner David and the former officers. At the core of the
of the power of general supervision of the President over local
petition is the validity of the DILGs caretakership of
governments.[47] Moreover, while the Liga may be deemed a
the Liga and the official acts of the DILG as such caretaker
government organization, it is not strictly a local government
which exceeded the bounds of supervision and were exercise
unit over which the DILG has supervisory power. [48]
of control. At stake in this case is the realization of the
constitutionally ensconced principle of local government
Meanwhile, on 24 September 1998, James Marty L. Lim, autonomy;[61] the statutory objective to enhance the
the newly elected President of the National Liga, filed a Motion capabilities of barangays and municipalities by providing them
for Leave to File Comment in Intervention,[49] with opportunities to participate actively in the implementation of
his Comment in Intervention attached,[50] invoking the validity national programs and projects;[62] and the promotion of the
of the DILGs actions relative to the conduct of avowed aim to ensure the independence and non-partisanship
the Liga elections.[51] In addition, he sought the dismissal of the of the Liga ng mga Barangay. The mantle of local autonomy
instant petition on the following grounds: (1) the issue of validity would be eviscerated and remain an empty buzzword if
or invalidity of the questioned order has been rendered moot unconstitutional, illegal and unwarranted intrusions in the
and academic by the election of Liga officers; (2) the turn-over affairs of the local governments are tolerated and left
of the administration and management of Liga affairs to unchecked.
the Liga officers; and (3) the recognition and acceptance by
the members of the Liga nationwide.[52]
FINALS CONSTITUTION I ACJUCO 86

Indeed, it is the declared policy of the State that its The presidents of the Liga at the municipal, city and
territorial and political subdivisions should enjoy genuine provincial levels, automatically become ex-officio members of
meaningful local autonomy to enable them to attain their fullest the Sangguniang Bayan, Sangguniang
development as self-reliant communities and make them more Panlungsod and SangguniangPanlalawigan, respectively. Th
effective partners in the attainment of national goals. [63] In the ey shall serve as such only during their term of office as
case of De Leon v. Esguerra,[64] the Court ruled that even presidents of the Liga chapters, which in no case shall be
barangays are meant to possess genuine and meaningful beyond the term of office of the sanggunianconcerned.[75]
local autonomy so that they may develop fully as self-
reliant communities.[65] The Liga ng mga Barangay has one principal aim,
namely: to promote the development of barangays and secure
Furthermore, well-entrenched is the rule that courts will the general welfare of their inhabitants. [76] In line with this,
decide a question otherwise moot and academic if it is capable the Liga is granted the following functions and duties:
of repetition, yet evading review.[66] For the question of whether
the DILG may validly be appointed as interim caretaker, or a) Give priority to programs designed for the total
assume a similar position and perform acts pursuant thereto, development of the barangays and in
is likely to resurrect again, and yet the question may not be consonance with the policies, programs and
decided before the actual assumption, or the termination of projects of the national government;
said assumption even.
b) Assist in the education of barangay residents for
So too, dismissing the petition on the ground of mootness people’s participation in local government
could lead to the wrong impression that the challenged order administration in order to promote untied and
and issuances are valid. Verily, that does not appear to be the concerted action to achieve country-wide
correct conclusion to make since by applying opposite development goals;
precedents to the issues the outcome points to invalidating the
assailed order and memorandum circulars.
c) Supplement the efforts of government in creating
gainful employment within the barangay;
The resolution of the issues of whether the Liga ng mga
Barangay is subject to DILG supervision, and whether the
d) Adopt measures to promote the welfare of
questioned caretakership order of the respondent judge
barangay officials;
and the challenged issuances and acts of the DILG
constitute control in derogation of the Constitution,
necessitates a brief overview of the barangay, as the lowest e) Serve as forum of the barangays in order to
LGU, and the Liga, as a vehicle of governance and forge linkages with government and non-
coordination. governmental organizations and thereby
promote the social, economic and political well-
being of the barangays; and
As the basic political unit, the barangay serves as the
primary planning and implementing unit of government
policies, plans, programs, projects and activities in the f) Exercise such other powers and perform such
community, and as a forum wherein the collective views of the other duties and functions which will bring
people may be expressed, crystallized and considered, and about stronger ties between barangays and
where disputes may be amicably settled.[67] promote the welfare of the barangay
inhabitants.[77]
On the other hand, the Liga ng mga Barangay[68] is the
organization of all barangays, the primary purpose of which is The Ligas are primarily governed by the provisions
the determination of the representation of the Liga in of the Local Government Code. However, they are
the sanggunians, and the ventilation, articulation, and empowered to make their own constitution and by-laws to
crystallization of issues affecting barangay government govern their operations. Sec. 507 of the Code provides:
administration and securing solutions thereto, through proper
and legal means.[69]The Liga ng mga Barangay shall have Sec. 507. Constitution and By-Laws of the Liga and the
chapters at the municipal, city and provincial and metropolitan Leagues. - All other matters not herein otherwise provided for
political subdivision levels.[70] The municipal and city chapters affecting the internal organization of the leagues of local
of the Liga are composed of the barangay representatives government units shall be governed by their respective
from the municipality or city concerned. The presidents of the constitution and by-laws which are hereby made suppletory to
municipal and city chapters of the Liga form the provincial or the provision of this Chapter: Provided, That said Constitution
metropolitan political subdivision chapters of the Liga. The and By-laws shall always conform to the provision of the
presidents of the chapters of the Liga in highly urbanized cities, Constitution and existing laws.
provinces and the Metro Manila area and other metropolitan
political subdivisions constitute the National Liga ng mga Pursuant to the Local Government Code, the Liga ng
Barangay.[71] mga Barangay adopted its own Constitution and By-
Laws. It provides that the corporate powers of
As conceptualized in the Local Government Code, the Liga, expressed or implied, shall be vested in the board of
the barangay is positioned to influence and direct the directors of each level of the Liga which shall:
development of the entire country. This was heralded by the
adoption of the bottom-to-top approach process of a) Have jurisdiction over all officers, directors and committees
development which requires the development plans of of the said Liga; including the power of appointment,
the barangay to be considered in the development plans of the assignment and delegation;
municipality, city or province,[72]whose plans in turn are to be
taken into account by the central government[73] in its plans for b) Have general management of the business, property, and
the development of the entire country. [74] The Liga is the funds of said Liga;
vehicle assigned to make this new development approach
materialize and produce results.
FINALS CONSTITUTION I ACJUCO 87

c) Prepare and approve a budget showing anticipated receipts We rule in the affirmative. In Opinion No. 41, Series of 1995,
and expenditures for the year, including the plans or schemes the Department of Justice ruled that the liga ng mga
for funding purposes; and barangay is a government organization, being an
association, federation, league or union created by law or
d) Have the power to suspend or remove from office any officer by authority of law, whose members are either appointed
or member of the said board on grounds cited and in the or elected government officials. The Local Government
manner provided in hereinunder provisions. [78] Code defines the liga ng mga barangay as an organization of
all barangays for the primary purpose of determining the
representation of the liga in the sanggunians, and for
The National Liga Board of Directors promulgated the
ventilating, articulating and crystallizing issues affecting
rules for the conduct of its Ligas general elections.[79] And, as
barangay government administration and securing, through
early as 28 April 1997, the Liga National Chapter had already
proper and legal means, solutions thereto. [91]
scheduled its general elections on 14 June 1997. [80]

The rationale for making the Liga subject to DILG


The controlling provision on the issues at hand is Section
supervision is quite evident, whether from the perspectives of
4, Article X of the Constitution, which reads in part:
logic or of practicality. The Liga is an aggroupment
of barangays which are in turn represented therein by their
Sec. 4 The President of the Philippines shall exercise general respective punong barangays. The representatives of
supervision over local governments. the Liga sit in an ex officio capacity at the municipal, city and
provincial sanggunians. As such, they enjoy all the powers
The 1935, 1973 and 1987 Constitutions uniformly and discharge all the functions of regular municipal
differentiate the Presidents power of supervision over local councilors, city councilors or provincial board members,
governments and his power of control of the executive as the case may be. Thus, the Liga is the vehicle through
departments bureaus and offices.[81] Similar to the counterpart which the barangay participates in the enactment of
provisions in the earlier Constitutions, the provision in the 1987 ordinances and formulation of policies at all the legislative
Constitution provision has been interpreted to exclude the local levels higher than the sangguniang barangay, at the
power of control.[82] same time serving as the mechanism for the bottom-to-top
approach of development.
In the early case of Mondano v. Silvosa, et
al.,[83] this Court defined supervision as overseeing, or the In the case at bar, even before the respondent Judge
power or authority of an officer to see that subordinate officers designated the DILG as interim caretaker of the Liga, on 28
perform their duties, and to take such action as prescribed by July 1997, it issued Memorandum Circular No. 97-176,
law to compel his subordinates to perform their duties. Control, directing local government officials not to recognize David as
on the other hand, means the power of an officer to alter or the National Liga President and his pronouncements relating
modify or nullify or set aside what a subordinate officer had to the affairs of the Liga. Not only was the action premature, it
done in the performance of his duties and to substitute the even smacked of superciliousness and injudiciousness. The
judgment of the former for that of the latter.[84] In Taule v. DILG is the topmost government agency which maintains
Santos,[85] the Court held that the Constitution permits the coordination with, and exercises supervision over local
President to wield no more authority than that of checking government units and its multi-level leagues. As such, it
whether a local government or its officers perform their duties should be forthright, circumspect and supportive in its
as provided by statutory enactments.[86]Supervisory power, dealings with the Ligas especially the Liga ng mga
when contrasted with control, is the power of mere oversight Barangay. The indispensable role played by the latter in the
over an inferior body; it does not include any restraining development of the barangays and the promotion of the
authority over such body.[87] welfare of the inhabitants thereof deserve no less than the full
support and respect of the other agencies of government. As
The case of Drilon v. Lim[88] clearly defined the extent of the Court held in the case of San Juan v. Civil Service
supervisory power, thus: Commission,[92] our national officials should not only comply
with the constitutional provisions on local autonomy but should
The supervisor or superintendent merely sees to it that the also appreciate the spirit of liberty upon which these provisions
rules are followed, but he himself does not lay down such rules, are based.[93]
nor does he have the discretion to modify or replace them. If
the rules are not observed, he may order the work done or re- When the respondent judge eventually appointed the
done but only to conform to the prescribed rules. He may not DILG as interim caretaker to manage and administer the affairs
prescribe his own manner for the doing of the act. He has no of the Liga, she effectively removed the management from the
judgment on this matter except to see that the rules are National Liga Board and vested control of the Liga on the
followed[89] DILG. Even a cursory glance at the DILGs prayer for
appointment as interim caretaker of the Liga to manage and
In Section 4, Article X of the Constitution applicable to administer the affairs of the Liga, until such time that the
the Liga ng mga Barangay? Otherwise put, is the Liga legally new set of National Liga officers shall have been duly elected
susceptible to DILG suspension? and assumed office reveals that what the DILG wanted was to
take control over the Liga. Even if said caretakership was
contemplated to last for a limited time, or only until a new set
This question was resolved in Bito-Onon v. of officers assume office, the fact remains that it was a
Fernandez,[90] where the Court ruled that the Presidents power conferment of control in derogation of the Constitution.
of the general supervision, as exercised therein by the DILG
Secretary as his alter ego, extends to the Liga ng mga
Barangay. With his Department already appointed as interim
caretaker of the Liga, Secretary Barbers nullified the results of
the Liga elections and promulgated DILG Memorandum
Does the Presidents power of general supervision extend Circular No. 97-193 dated 11 August 1997, where he laid down
to the liga ng mga barangay, which is not a local the supplemental guidelines for the 1997 synchronized
government unit? elections of the provincial and metropolitan chapters and for
the election of the national chapter of the Liga ng mga
Barangay; scheduled dates for the new provincial,
FINALS CONSTITUTION I ACJUCO 88

metropolitan and national chapter elections; and appointed xxx


respondent Rayos as president of Liga-Caloocan Chapter.
xxx. The amendment of the GUIDELINES is more than an
These acts of the DILG went beyond the sphere of exercise of the power of supervision but is an exercise of the
general supervision and constituted direct interference power of control, which the President does not have over the
with the political affairs, not only of the Liga, but more LIGA. Although the DILG is given the power to prescribe rules,
importantly, of the barangay as an institution. The election regulations and other issuances, the Administrative Code limits
of Liga officers is part of the Ligas internal organization, for its authority to merely monitoring compliance by local
which the latter has already provided guidelines. In government units of such issuances. To monitor means to
succession, the DILG assumed stewardship and jurisdiction watch, observe or check and is compatible with the power of
over the Liga affairs, issued supplemental guidelines for the supervision of the DILG Secretary over local governments,
election, and nullified the effects of the Liga-conducted which is limited to checking whether the local government unit
elections. Clearly, what the DILG wielded was the power of concerned or the officers thereof perform their duties as per
control which even the President does not have. statutory enactments. Besides, any doubt as to the power of
the DILG Secretary to interfere with local affairs should be
Furthermore, the DILG assumed control when it resolved in favor of the greater autonomy of the local
appointed respondent Rayos as president of the Liga- government.[95]
Caloocan Chapter prior to the newly scheduled
general Liga elections, although petitioner Davids term In Taule,[96] the Court ruled that the Secretary of Local
had not yet expired. The DILG substituted its choice, who Government had no authority to pass upon the validity or
was Rayos, over the choice of majority of the punong regularity of the election of officers of katipunan ng mga
barangay of Caloocan, who was the incumbent President, barangay or barangay councils. In that case, a protest was
petitioner David. The latter was elected and had in fact been lodged before the Secretary of Local Government regarding
sitting as an ex-officio member of the sangguniang several irregularities in, and seeking the nullification of, the
panlungsod in accordance with the Liga Constitution and By- election of officers of the Federation of Associations of
Laws. Yet, the DILG extended the appointment to respondent Barangay Councils (FABC) of Catanduanes. Then Local
Rayos although it was aware that the position was the subject Government Secretary Luis Santos issued a resolution
of a quo warranto proceeding instituted by Rayos himself, nullifying the election of officers and ordered a new one to be
thereby preempting the outcome of that case. It was bad conducted. The Court ruled:
enough that the DILG assumed the power of control, it was
worse when it made use of the power with evident bias and Construing the constitutional limitation on the power of general
partiality. supervision of the President over local governments, we hold
that respondent Secretary has no authority to pass upon the
As the entity exercising supervision over the Liga ng mga validity or regularity of the officers of the katipunan. To allow
Barangay, the DILGs authority over the Liga is limited to respondent Secretary to do so will give him more power
seeing to it that the rules are followed, but it cannot lay down than the law or the Constitution grants. It will in effect give
such rules itself, nor does it have the discretion to modify or him control over local government officials for it will permit him
replace them. In this particular case, the most that the DILG to interfere in a purely democratic and non-partisan activity
could do was review the acts of the incumbent officers of aimed at strengthening the barangay as the basic component
the Liga in the conduct of the elections to determine if they of local governments so that the ultimate goal of fullest
committed any violation of the Ligas Constitution and By-laws autonomy may be achieved. In fact, his order that the new
and its implementing rules. If the National Liga Board and its elections to be conducted be presided by the Regional Director
officers had violated Liga rules, the DILG should have ordered is a clear and direct interference by the Department with the
the Liga to conduct another election in accordance with political affairs of the barangays which is not permitted by the
the Ligas own rules, but not in obeisance to DILG-dictated limitation of presidential power to general supervision over
guidelines. Neither had the DILG the authority to remove the local governments.[97]
incumbent officers of the Liga and replace them, even
temporarily, with unelected Liga officers. All given, the Court is convinced that the assailed order
was issued with grave abuse of discretion while the acts
Like the local government units, the Liga ng mga of the respondent Secretary, including DILG Memorandum
Barangay is not subject to control by the Chief Executive Circulars No. 97-176 and No. 97-193, are unconstitutional
or his alter ego. and ultra vires, as they all entailed the conferment or
exercise of control a power which is denied by the
In the Bito-Onon[94] case, this Court held that DILG Constitution even to the President.
Memorandum Circular No. 97-193, insofar as it authorized the
filing of a petition for review of the decision of the Board of WHEREFORE, the Petition is GRANTED. The Order of
Election Supervisors (BES) with the regular courts in a post- the Regional Trial Court dated 04 August 1997 is SET ASIDE
proclamation electoral protest, involved the exercise of control for having been issued with grave abuse of discretion
as it in effect amended the guidelines already promulgated by amounting to lack or excess of jurisdiction. DILG Memorandum
the Liga.The decision reads in part: Circulars No. 97-176 and No. 97-193, are declared VOID for
being unconstitutional and ultra vires.
xxx. Officers in control, lay down the rules in the doing of an
act. If they are not followed, it is discretionary on his part to No pronouncements as to costs.
order the act undone or redone by his subordinate or he may
even decide to do it himself. Supervision does not cover such SO ORDERED.
authority. Supervising officers merely see to it that the rules are
followed, but he himself does not lay down such rules, nor does
he have the discretion to modify or replace them. If the rules
are not observed, he may order the work done or re-done to
conform for to the prescribed rules. He cannot prescribe his
own manner the doing of the act.
FINALS CONSTITUTION I ACJUCO 89

[G.R. No. 131429. August 4, 1999] In the meantime, on 10 October 1997, Bermudez
together with his co-petitioners Arturo Llobrera and Claudio
OSCAR BERMUDEZ, ARTURO A. LLOBRERA and Dayaon, the Second Assistant Provincial Prosecutor and the
CLAUDIO L. DAYAON, petitioners, vs. EXECUTIVE Fourth Assistant Provincial Prosecutor of Tarlac, respectively,
SECRETARY RUBEN TORRES, BUDGET SECRETARY filed with the Regional Trial Court of Tarlac, a petition for
SALVADOR ENRIQUEZ, JR., JUSTICE SECRETARY prohibition and/or injunction, and mandamus, with a prayer for
TEOFISTO GUINGONA, JR., and ATTY. CONRADO the issuance of a writ of injunction/temporary restraining order,
QUIAOIT, respondents. against herein respondents, challenging the appointment of
Quiaoit primarily on the ground that the appointment lacks
the recommendation of the Secretary of Justice
DECISION
prescribed under the Revised Administrative Code of
VITUG, J.: 1987. After hearing, the trial court considered the petition
submitted for resolution and, in due time, issued its now
assailed order dismissing the petition. The subsequent move
The validity and legality of the appointment of by petitioners to have the order reconsidered met with a denial.
respondent Conrado Quiaoit to the post of Provincial
Prosecutor of Tarlac by then President Fidel V. Ramos is Hence, the instant recourse.
assailed in this petition for review on certiorari on a pure
question of law which prays for the reversal of the The core issue for consideration is whether or not the
Order,[1] dated 20 October 1997, of the Regional Trial Court absence of a recommendation of the Secretary of Justice
(Branch 63) of Tarlac, Tarlac, dismissing the petition for to the President can be held fatal to the appointment of
prohibition and/or injunction and mandamus, with a prayer for respondent Conrado Quiaoit. This question would, in turn,
the issuance of a writ of injunction/temporary restraining order, pivot on the proper understanding of the provision of the
instituted by herein petitioners. Revised Administrative Code of 1987 (Book IV, Title III,
Chapter II, Section 9) to the effect that-
The occurrence of a vacancy in the Office of the
Provincial Prosecutor of Tarlac impelled the main contestants
All provincial and city prosecutors and their assistants shall be
in this case, petitioner Oscar Bermudez and respondent
appointed by the President upon the recommendation of the
Conrado Quiaoit, to take contrasting views on the proper
Secretary.
interpretation of a provision in the 1987 Revised Administrative
Code. Bermudez, the First Assistant Provincial Prosecutor
of Tarlac and Officer-In-Charge of the Office of the Petitioners contend that an appointment of a provincial
Provincial Prosecutor, was a recommendee[2] of then prosecutor mandatorily requires a prior recommendation of the
Justice Secretary Teofisto Guingona, Jr., for the position of Secretary of Justice endorsing the intended appointment
Provincial Prosecutor. Quiaoit, on the other hand, would citing, by analogy, the case of San Juan vs. CSC[5] where the
appear to have had the support of then Representative Jose Court held:
Yap of the Second Legislative District of Tarlac.[3] On 30
June 1997, Quiaoit emerged the victor when he was "x x x The DBM may appoint only from the list of qualified
appointed by President Ramos to the coveted office. Quiaoit recommendees nominated by the Governor. If none is
received a certified xerox copy of his appointment and, on 21 qualified, he must return the list of nominees to the Governor
July 1997, took his oath of office before Executive Judge Angel explaining why no one meets the legal requirements and ask
Parazo of the Regional Trial Court (Branch 65) of Tarlac, for new recommendees who have the necessary eligibilities
Tarlac. On 23 July 1997, Quiaoit assumed office and and qualifications.
immediately informed the President, as well as the Secretary
of Justice and the Civil Service Commission, of that
assumption. Bermudez refused to vacate the Office of The Provincial Budget Officer (PBO) is expected to
Provincial Prosecutor claiming that the original copy of synchronize his work with DBM.[6] (Emphasis supplied.)
Quiaoits appointment had not yet been released by the
Secretary of Justice.[4] Quiaoit, nonetheless, performed the Insisting on the application of San Juan, petitioners call
functions and duties of the Office of Provincial Prosecutor by attention to the tenor of Executive Order No. 112[7] -
issuing office orders and memoranda, signing resolutions on
preliminary investigations, and filing several informations
before the courts. Quiaoit had since been regularly receiving Section 1. All budget officers of provinces, cities and
the salary, RATA and other emoluments of the office. municipalities shall be appointed henceforth by the Minister of
Budget and Management upon recommendation of the
On 17 September 1997, Bermudez and Quiaoit were local chief executive concernedx x x.
summoned to Manila by Justice Secretary Guingona. The
three met at the Department of Justice and, following the That, they claim, can be likened to the aforequoted provision
conference, Bermudez was ordered to wind up his cases until of the Revised Administrative Code of 1987. Respondents
15 October 1997 and to turn-over the contested office to argue differently.
Quiaoit the next day.
The legislative intent is, of course, primordial. There is
In his First Indorsement, dated 22 September 1997, for no hard-and-fast rule in ascertaining whether the language in
the Chief State prosecutor, Assistant Chief State Prosecutor a statute should be considered mandatory or directory, and the
Nilo Mariano transmitted the original copy of Quiaoits application of a ruling in one particular instance may not
appointment to the Regional State Prosecutor Carlos de Leon, necessarily be apt in another[8] for each must be determined
Region III, at San Fernando, Pampanga. In turn, in his Second on the basis of the specific law in issue and the peculiar
Indorsement, dated 02 October 1997, Regional State circumstances attendant to it. More often than not, the
Prosecutor de Leon forwarded to Quiaoit said original copy of problem, in the final analysis, is firmed up and addressed on a
his appointment. On the basis of the transmittal letter of case-to-case basis. The nature, structure and aim of the law
Regional State Prosecutor de Leon, Quiaoit, as directed, again itself is often resorted to in looking at the legislative
so assumed office on 16 October 1997. On even date, intent. Generally, it is said that if no consequential rights or
Bermudez was detailed at the Office of the Regional State liabilities depend on it and no injury can result from ignoring it,
Prosecutor, Region III, in San Fernando, Pampanga. and that the purpose of the legislature can be accomplished in
a manner other than that prescribed when substantially the
FINALS CONSTITUTION I ACJUCO 90

same results can be obtained, then the statute should be innocuous position involves the application of a most important
regarded merely as directory, rather than as mandatory, in constitutional policy and principle, that of local autonomy. We
character.[9] have to obey the clear mandate on local autonomy. Where a
law is capable of two interpretations, one in favor of
An appointment to a public office is the unequivocal act centralized power in Malacaang and the other beneficial to
of designating or selecting by one having the authority therefor local autonomy, the scales must be weighed in favor of
of an individual to discharge and perform the duties and autonomy.
functions of an office or trust.[10] The appointment is deemed
complete once the last act required of the appointing
authority has been complied with and its acceptance xxxxxxxxx
thereafter by the appointee in order to render it
effective.[11]Appointment necessarily calls for an exercise of When the Civil Service Commission interpreted the
discretion on the part of the appointing authority. [12] In recommending power of the Provincial Governor as purely
Pamantasan ng Lungsod ng Maynila vs. Intermediate directory, it went against the letter and spirit of the
Appellate Court,[13] reiterated in Flores vs. Drilon,[14] this Court constitutional provisions on local autonomy.If the DBM
has held: Secretary jealously hoards the entirety of budgetary powers
and ignores the right of local governments to develop self-
The power to appoint is, in essence, discretionary. The reliance and resoluteness in the handling of their own funds,
appointing power has the right of choice which he may exercise the goal of meaningful local autonomy is frustrated and set
freely according to his judgment, deciding for himself who is back.[25]
best qualified among those who have the necessary
qualifications and eligibilities. It is a prerogative of the The Court there has explained that the President merely
appointing power x x x[15] exercises general supervision over local government units and
local officials;[26] hence, in the appointment of a Provincial
Indeed, it may rightly be said that the right of choice is the Budget Officer, the executive department, through the
heart of the power to appoint.[16] In the exercise of the power Secretary of Budget and Management, indeed had to
of appointment, discretion is an integral part thereof. share the questioned power with the local government.

When the Constitution[17] or the law[18] clothes the In the instant case, the recommendation of the
President with the power to appoint a subordinate officer, such Secretary of Justice and the appointment of the President
conferment must be understood as necessarily carrying with it are acts of the Executive Department itself, and there is no
an ample discretion of whom to appoint. It should be here sharing of power to speak of, the latter being deemed for all
pertinent to state that the President is the head of intents and purposes as being merely an extension of the
government whose authority includes the power of personality of the President.
control over all executive departments, bureaus and WHEREFORE, the petition is DENIED. No costs.
offices. Control means the authority of an empowered officer
to alter or modify, or even nullify or set aside, what a SO ORDERED.
subordinate officer has done in the performance of his duties,
as well as to substitute the judgment of the latter, [19] as and
when the former deems it to be appropriate. Expressed in
another way, the President has the power to
assume directly the functions of an executive department,
bureau and office.[20] It can accordingly be inferred therefrom
that the President can interfere in the exercise of
discretion of officials under him or altogether ignore their
recommendations.[21]
It is the considered view of the Court, given the above
disquisition, that the phrase upon recommendation of the
Secretary, found in Section 9, Chapter II, Title III, Book IV, of
the Revised Administrative Code, should be interpreted, as it
is normally so understood, to be a mere advise,
exhortation or indorsement, which is essentially persuasive
in character and not binding or obligatory upon the party to
whom it is made.[22] The recommendation is here nothing
really more than advisory in nature.[23] The President, being
the head of the Executive Department, could very well
disregard or do away with the action of the departments,
bureaus or offices even in the exercise of discretionary
authority, and in so opting, he cannot be said as having acted
beyond the scope of his authority.
The doctrine in San Juan, relied upon by petitioners, is
tangential. While the tenor of the legal provision in Executive
Order No. 112 has some similarity with the provision in the
1987 Administrative Code in question, it is to be pointed out,
however, that San Juan,[24] in construing the law, has
distinctively given stress to the constitutional mandate on local
autonomy; thus:

The issue before the Court is not limited to the validity of the
appointment of one Provincial Budget Officer. The tug of war
between the Secretary of Budget and Management and the
Governor of the premier province of Rizal over a seemingly
FINALS CONSTITUTION I ACJUCO 91

G.R. No. 79974 December 17, 1987 The President shall nominate and, with the consent of the
Commission on Appointments, appoint the heads of the
ULPIANO P. SARMIENTO III AND JUANITO G. executive departments, ambassadors, other public
ministers and consuls, or officers of the armed forces
ARCILLA, petitioners,
vs. from the rank of colonel or naval captain, and other
SALVADOR MISON, in his capacity as COMMISSIONER officers whose appointments are vested in him in this
OF THE BUREAU OF CUSTOMS, AND GUILLERMO Constitution. He shall also appoint all other officers of the
CARAGUE, in his capacity as SECRETARY OF THE Government whose appointments are not otherwise provided
DEPARTMENT OF BUDGET, respondents, COMMISSION for by law, and those whom he may be authorized by law to
ON APPOINTMENTS, intervenor. appoint. The Congress may, by law, vest the appointment of
other officers lower in rank in the President alone, in the courts,
or in the heads of the departments, agencies, commissions or
PADILLA, J.: boards.

Once more the Court is called upon to delineate constitutional The President shall have the power to make appointments
boundaries. In this petition for prohibition, the petitioners, who during the recess of the Congress, whether voluntary or
are taxpayers, lawyers, members of the Integrated Bar of the compulsory, but such appointments shall be effective
Philippines and professors of Constitutional Law, seek to only until disapproval by the Commission on
enjoin the respondent Salvador Mison from performing the Appointments or until the next adjournment of the
functions of the Office of Commissioner of the Bureau of Congress.
Customs and the respondent Guillermo Carague, as Secretary
of the Department of Budget, from effecting disbursements in It is readily apparent that under the provisions of the 1987
payment of Mison's salaries and emoluments, on the ground Constitution, just quoted, there are four (4) groups of officers
that Mison's appointment as Commissioner of the Bureau whom the President shall appoint. These four (4) groups, to
of Customs is unconstitutional by reason of its not having which we will hereafter refer from time to time, are:
been confirmed by the Commission on Appointments. The
respondents, on the other hand, maintain the constitutionality First, the heads of the executive departments, ambassadors,
of respondent Mison's appointment without the confirmation of other public ministers and consuls, officers of the armed forces
the Commission on Appointments. from the rank of colonel or naval captain, and other officers
whose appointments are vested in him in this Constitution; 2
Because of the demands of public interest, including the need
for stability in the public service, the Court resolved to give due Second, all other officers of the Government whose
course to the petition and decide, setting aside the finer appointments are not otherwise provided for by law; 3
procedural questions of whether prohibition is the proper
remedy to test respondent Mison's right to the Office of
Commissioner of the Bureau of Customs and of whether the Third, those whom the President may be authorized by law to
appoint;
petitioners have a standing to bring this suit.

By the same token, and for the same purpose, the Court Fourth, officers lower in rank 4 whose appointments the
allowed the Commission on Appointments to intervene and file Congress may by law vest in the President alone.
a petition in intervention. Comment was required of
respondents on said petition. The comment was filed, followed The first group of officers is clearly appointed with the
by intervenor's reply thereto. The parties were also heard in consent of the Commission on Appointments.
oral argument on 8 December 1987. Appointments of such officers are initiated by nomination and,
if the nomination is confirmed by the Commission on
This case assumes added significance because, at bottom Appointments, the President appoints. 5
line, it involves a conflict between two (2) great departments of
government, the Executive and Legislative Departments. It The second, third and fourth groups of officers are the present
also occurs early in the life of the 1987 Constitution. bone of contention. Should they be appointed by the
President with or without the consent (confirmation) of the
Commission on Appointments? By following the accepted
The task of the Court is rendered lighter by the existence of
relatively clear provisions in the Constitution. In cases like this, rule in constitutional and statutory construction that an express
enumeration of subjects excludes others not enumerated, it
we follow what the Court, speaking through Mr. Justice (later,
Chief Justice) Jose Abad Santos stated in Gold Creek Mining would follow that only those appointments to positions
Corp. vs. Rodriguez, 1 that: expressly stated in the first group require the consent
(confirmation) of the Commission on Appointments. But we
need not rely solely on this basic rule of constitutional
The fundamental principle of constitutional construction. We can refer to historical background as well
construction is to give effect to the intent of as to the records of the 1986 Constitutional Commission to
the framers of the organic law and of the determine, with more accuracy, if not precision, the intention of
people adopting it. The intention to which the framers of the 1987 Constitution and the people adopting
force is to be given is that which is it, on whether the appointments by the President, under the
embodied and expressed in the second, third and fourth groups, require the consent
constitutional provisions themselves. (confirmation) of the Commission on Appointments. Again, in
this task, the following advice of Mr. Chief Justice J. Abad
The Court will thus construe the applicable constitutional Santos in Gold Creek is apropos:
provisions, not in accordance with how the executive or the
legislative department may want them construed, but in In deciding this point, it should be borne in mind that a
accordance with what they say and provide. constitutional provision must be presumed to have been
framed and adopted in the light and understanding of prior and
Section 16, Article VII of the 1987 Constitution says: existing laws and with reference to them. "Courts are bound to
presume that the people adopting a constitution are familiar
FINALS CONSTITUTION I ACJUCO 92

with the previous and existing laws upon the subjects to which The proceedings in the 1986 Constitutional Commission
its provisions relate, and upon which they express their support this conclusion. The original text of Section 16,
judgment and opinion in its adoption." (Barry vs. Truax 13 N.D., Article VII, as proposed by the Committee on the Executive of
131; 99 N.W., 769,65 L. R. A., 762.) 6 the 1986 Constitutional Commission, read as follows:

It will be recalled that, under Sec. 10, Article VII of the 1935 Section 16. The president shall nominate and, with the consent
Constitution, it is provided that — of a Commission on Appointment, shall appoint the heads of
the executive departments and bureaus, ambassadors, other
xxx xxx xxx public ministers and consuls, or officers of the armed forces
from the rank of colonel or naval captain and all other officers
of the Government whose appointments are not otherwise
(3) The President shall nominate and with the consent of the provided for by law, and those whom he may be authorized by
Commission on Appointments, shall appoint the heads of the law to appoint. The Congress may by law vest the appointment
executive departments and bureaus, officers of the army from of inferior officers in the President alone, in the courts, or in the
the rank of colonel, of the Navy and Air Forces from the rank heads of departments 7 [Emphasis supplied].
of captain or commander, and all other officers of the
Government whose appointments are not herein otherwise
provided for, and those whom he may be authorized by law to The above text is almost a verbatim copy of its
appoint; but the Congress may by law vest the appointment of counterpart provision in the 1935 Constitution. When the
inferior officers, in the President alone, in the courts, or in the frames discussed on the floor of the Commission the proposed
heads of departments. text of Section 16, Article VII, a feeling was manifestly
expressed to make the power of the Commission on
Appointments over presidential appointments more limited
(4) The President shall havethe power to make appointments than that held by the Commission in the 1935 Constitution.
during the recess of the Congress, but such appointments shall Thus-
be effective only until disapproval by the Commission on
Appointments or until the next adjournment of the Congress.
Mr. Rama: ... May I ask that Commissioner Monsod be
recognized
xxx xxx xxx
The President: We will call Commissioner Davide later.
(7) ..., and with the consent of the Commission on
Appointments, shall appoint ambassadors, other public
ministers and consuls ... Mr. Monsod: With the Chair's indulgence, I just want to take a
few minutes of our time to lay the basis for some of the
amendments that I would like to propose to the Committee this
Upon the other hand, the 1973 Constitution provides that- morning.

Section 10. The President shall appoint the heads of bureaus xxx xxx xxx
and offices, the officers of the Armed Forces of the Philippines
from the rank of Brigadier General or Commodore, and all
other officers of the government whose appointments are not On Section 16, I would like to suggest that the power of the
herein otherwise provided for, and those whom he may be Commission on Appointments be limited to the department
authorized by law to appoint. However, the Batasang heads, ambassadors, generals and so on but not to the
Pambansa may by law vest in the Prime Minister, members levels of bureau heads and colonels.
of the Cabinet, the Executive Committee, Courts, Heads of
Agencies, Commissions, and Boards the power to appoint xxx xxx xxx 8 (Emphasis supplied.)
inferior officers in their respective offices.
In the course of the debates on the text of Section 16, there
Thus, in the 1935 Constitution, almost all presidential were two (2) major changes proposed and approved by the
appointments required the consent (confirmation) of the Commission. These were (1) the exclusion of the
Commission on Appointments. It is now a sad part of our appointments of heads of bureaus from the requirement
political history that the power of confirmation by the of confirmation by the Commission on Appointments; and
Commission on Appointments, under the 1935 Constitution, (2) the exclusion of appointments made under the second
transformed that commission, many times, into a venue of sentence 9 of the section from the same requirement. The
"horse-trading" and similar malpractices. records of the deliberations of the Constitutional Commission
show the following:
On the other hand, the 1973 Constitution, consistent with the
authoritarian pattern in which it was molded and remolded by MR. ROMULO: I ask that Commissioner Foz be recognized
successive amendments, placed the absolute power of
appointment in the President with hardly any check on the
THE PRESIDENT: Commissioner Foz is recognized
part of the legislature.

MR. FOZ: Madam President, my proposed amendment is on


Given the above two (2) extremes, one, in the 1935 page 7, Section 16, line 26 which is to delete the words "and
Constitution and the other, in the 1973 Constitution, it is not
bureaus," and on line 28 of the same page, to change the
difficult for the Court to state that the framers of the 1987 phrase 'colonel or naval captain to MAJOR GENERAL OR
Constitution and the people adopting it, struck a "middle
REAR ADMIRAL. This last amendment which is co-authored
ground" by requiring the consent (confirmation) of the by Commissioner de Castro is to put a period (.) after the word
Commission on Appointments for the first group of ADMIRAL, and on line 29 of the same page, start a new
appointments and leaving to the President, without such sentence with: HE SHALL ALSO APPOINT, et cetera.
confirmation, the appointment of other officers, i.e., those in the
second and third groups as well as those in the fourth group,
i.e., officers of lower rank.
FINALS CONSTITUTION I ACJUCO 93

MR. REGALADO: May we have the amendments one by one. THE PRESIDENT: The Acting Floor Leader is recognized.
The first proposed amendment is to delete the words "and
bureaus" on line 26. THE PRESIDENT: Commissioner Foz is recognized
MR. FOZ: Madam President, this is the third proposed
MR. FOZ: That is correct. amendment on page 7, line 28. 1 propose to put a period (.)
after 'captain' and on line 29, delete 'and all' and substitute it
MR. REGALADO: For the benefit of the other Commissioners, with HE SHALL ALSO APPOINT ANY.
what would be the justification of the proponent for such a
deletion? MR. REGALADO: Madam President, the Committee accepts
the proposed amendment because it makes it clear that those
other officers mentioned therein do not have to be confirmed
MR. FOZ: The position of bureau director is actually quite low by the Commission on Appointments.
in the executive department, and to require further confirmation
of presidential appointment of heads of bureaus would subject MR. DAVIDE: Madam President.
them to political influence.
THE PRESIDENT: Commissioner Davide is recognized.
MR. REGALADO: The Commissioner's proposed amendment xxx xxx xxx
by deletion also includes regional directors as distinguished
from merely staff directors, because the regional directors MR. DAVIDE: So would the proponent accept an amendment
have quite a plenitude of powers within the regions as to his amendment, so that after "captain" we insert the
distinguished from staff directors who only stay in the office. following words: AND OTHER OFFICERS WHOSE
MR. FOZ: Yes, but the regional directors are under the APPOINTMENTS ARE VESTED IN HIM IN THIS
supervisiopn of the staff bureau directors. CONSTITUTION?
FR. BERNAS: It is a little vague.
xxx xxx xxx
MR. DAVIDE: In other words, there are positions provided for
MR. MAAMBONG: May I direct a question to Commissioner in the Constitution whose appointments are vested in the
Foz? The Commissioner proposed an amendment to delete President, as a matter of fact like those of the different
'and bureaus on Section 16. Who will then appoint the bureau constitutional commissions.
directors if it is not the President? FR. BERNAS: That is correct. This list of officials found in
MR. FOZ: It is still the President who will appoint them but their Section 16 is not an exclusive list of those appointments which
appointment shall no longer be subject to confirmation by constitutionally require confirmation of the Commission on
the Commission on Appointments. Appointments,
MR. DAVIDE: That is the reason I seek the incorporation of the
MR. MAAMBONG: In other words, it is in line with the same words I proposed.
answer of Commissioner de Castro?
FR. BERNAS: Will Commissioner Davide restate his proposed
MR. FOZ: Yes. amendment?

MR. MAAMBONG: Thank you. MR. DAVIDE: After 'captain,' add the following: AND OTHER
OFFICERS WHOSE APPOINTMENTS ARE VESTED IN HIM
IN THIS CONSTITUTION.
THE PRESIDENT: Is this clear now? What is the reaction of FR. BERNAS: How about:"AND OTHER OFFICERS WHOSE
the Committee? APPOINTMENTS REQUIRE CONFIRMATION UNDER THIS
CONSTITUTION"?
xxx xxx xxx
MR. DAVIDE: Yes, Madam President that is modified by the
Committee.
MR. REGALADO: Madam President, the Committee feels that
this matter should be submitted to the body for a vote. FR. BERNAS: That will clarify things.

MR. DE CASTRO: Thank you. THE PRESIDENT: Does the Committee accept?

MR. REGALADO: Just for the record, of course, that excludes


MR. REGALADO: We will take the amendments one by
those officers which the Constitution does not require
one. We will first vote on the deletion of the phrase 'and
confirmation by the Commission on Appointments, like the
bureaus on line 26, such that appointments of bureau directors
members of the judiciary and the Ombudsman.
no longer need confirmation by the Commission on
MR. DAVIDE: That is correct. That is very clear from the
Appointment.
modification made by Commissioner Bernas.
THE PRESIDENT: So we have now this proposed amendment
Section 16, therefore, would read: 'The President shall
of Commissioners Foz and Davide.
nominate, and with the consent of a Commission on
xxx xxx xxx
Appointments, shall appoint the heads of the executive
departments, ambassadors. . . .
THE PRESIDENT: Is there any objection to this proposed
amendment of Commissioners Foz and Davide as accepted by
THE PRESIDENT: Is there any objection to delete the phrase the Committee? (Silence) The Chair hears none; the
'and bureaus' on page 7, line 26? (Silence) The Chair hears amendment, as amended, is approved 10 (Emphasis
none; the amendments is approved. supplied).

xxx xxx xxx It is, therefore, clear that appointments to the second and
third groups of officers can be made by the President
without the consent (confirmation) of the Commission on
MR. ROMULO: Madam President. Appointments.
FINALS CONSTITUTION I ACJUCO 94

limitations or qualifications must be clearly stated in order to be


It is contended by amicus curiae, Senator Neptali Gonzales, recognized. But, it is only in the first sentence of Sec. 16, Art.
that the second sentence of Sec. 16, Article VII reading- VII where it is clearly stated that appointments by the President
to the positions therein enumerated require the consent of the
He (the President) shall also appoint all other officers of the Commission on Appointments.
Government whose appointments are not otherwise provided
for by law and those whom he may be authorized by law to As to the fourth group of officers whom the President can
appoint . . . . (Emphasis supplied) with particular reference to appoint, the intervenor Commission on Appointments
the word "also," implies that the President shall "in like manner" underscores the third sentence in Sec. 16, Article VII of the
appoint the officers mentioned in said second sentence. 1987 Constitution, which reads:
In other words, the President shall appoint the officers
mentioned in said second sentence in the same manner as he The Congress may, by law, vest the appointment of other
appoints officers mentioned in the first sentence, that is, by officers lower in rank in the President alone, in the courts, or in
nomination and with the consent (confirmation) of the the heads of departments, agencies, commissions, or boards.
Commission on Appointments. [Emphasis supplied].

Amicus curiae's reliance on the word "also" in said and argues that, since a law is needed to vest the appointment
second sentence is not necessarily supportive of the of lower-ranked officers in the President alone, this implies
conclusion he arrives at. For, as the Solicitor General that, in the absence of such a law, lower-ranked officers have
argues, the word "also" could mean "in addition; as well; to be appointed by the President subject to confirmation by the
besides, too" (Webster's International Dictionary, p. 62, 1981 Commission on Appointments; and, if this is so, as to lower-
edition) which meanings could, on the contrary, stress that the ranked officers, it follows that higher-ranked officers should be
word "also" in said second sentence means that the appointed by the President, subject also to confirmation by the
President, in addition to nominating and, with the consent of Commission on Appointments.
the Commission on Appointments, appointing the officers
enumerated in the first sentence, can appoint (without such The respondents, on the other hand, submit that the third
consent (confirmation) the officers mentioned in the second sentence of Sec. 16, Article VII, abovequoted, merely declares
sentence- that, as to lower-ranked officers, the Congress may by law vest
their appointment in the President, in the courts, or in the heads
Rather than limit the area of consideration to the possible of the various departments, agencies, commissions, or boards
meanings of the word "also" as used in the context of said in the government. No reason however is submitted for the use
second sentence, the Court has chosen to derive significance of the word "alone" in said third sentence.
from the fact that the first sentence speaks of nomination by
the President and appointment by the President with the The Court is not impressed by both arguments. It is of the
consent of the Commission on Appointments, whereas, the considered opinion, after a careful study of the deliberations of
second sentence speaks only of appointment by the the 1986 Constitutional Commission, that the use of the word
President. And, this use of different language in two (2) alone" after the word "President" in said third sentence of Sec.
sentences proximate to each other underscores a difference in 16, Article VII is, more than anything else, a slip or lapsus in
message conveyed and perceptions established, in line with draftmanship. It will be recalled that, in the 1935 Constitution,
Judge Learned Hand's observation that "words are not pebbles the following provision appears at the end of par. 3, section 1
in alien juxtaposition" but, more so, because the recorded 0, Article VII thereof —
proceedings of the 1986 Constitutional Commission clearly
and expressly justify such differences.
...; but the Congress may by law vest the appointment of
inferior officers, in the President alone, in the courts, or in the
As a result of the innovations introduced in Sec. 16, Article VII heads of departments. [Emphasis supplied].
of the 1987 Constitution, there are officers whose
appointments require no confirmation of the Commission on
Appointments, even if such officers may be higher in rank, The above provision in the 1935 Constitution appears
compared to some officers whose appointments have to be immediately after the provision which makes practically all
confirmed by the Commission on Appointments under the first presidential appointments subject to confirmation by the
sentence of the same Sec. 16, Art. VII. Thus, to illustrate, the Commission on Appointments, thus-
appointment of the Central Bank Governor requires no
confirmation by the Commission on Appointments, even if he 3. The President shall nominate and with the consent of the
is higher in rank than a colonel in the Armed Forces of the Commission on Appointments, shall appoint the heads of the
Philippines or a consul in the Consular Service. executive departments and bureaus, officers of the Army from
the rank of colonel, of the Navy and Air Forces from the rank
But these contrasts, while initially impressive, merely of captain or commander, and all other officers of the
underscore the purposive intention and deliberate judgment of Government whose appointments are not herein provided for,
the framers of the 1987 Constitution that, except as to those and those whom he may be authorized by law to appoint;...
officers whose appointments require the consent of the
Commission on Appointments by express mandate of the first In other words, since the 1935 Constitution subjects, as a
sentence in Sec. 16, Art. VII, appointments of other officers are general rule, presidential appointments to confirmation by the
left to the President without need of confirmation by the Commission on Appointments, the same 1935 Constitution
Commission on Appointments. This conclusion is inevitable, if saw fit, by way of an exception to such rule, to provide that
we are to presume, as we must, that the framers of the 1987 Congress may, however, by law vest the appointment of
Constitution were knowledgeable of what they were doing and inferior officers (equivalent to 11 officers lower in rank" referred
of the foreseable effects thereof. to in the 1987 Constitution) in the President alone, in the
courts, or in the heads of departments,
Besides, the power to appoint is fundamentally executive
or presidential in character. Limitations on or qualifications In the 1987 Constitution, however, as already pointed out, the
of such power should be strictly construed against them. Such clear and expressed intent of its framers was to exclude
FINALS CONSTITUTION I ACJUCO 95

presidential appointments from confirmation by the Commissioner of Customs shall be appointed by the
Commission on Appointments, except appointments to President of the Philippines (Emphasis supplied.)
offices expressly mentioned in the first sentence of Sec.
16, Article VII. Consequently, there was no reason to use in
Of course, these laws (Rep. Act No. 1937 and PD No. 34) were
the third sentence of Sec. 16, Article VII the word "alone" after approved during the effectivity of the 1935 Constitution, under
the word "President" in providing that Congress may by law which the President may nominate and, with the consent of the
vest the appointment of lower-ranked officers in the President Commission on Appointments, appoint the heads of bureaus,
alone, or in the courts, or in the heads of departments, because like the Commissioner of the Bureau of Customs.
the power to appoint officers whom he (the President) may be
authorized by law to appoint is already vested in the
President, without need of confirmation by the After the effectivity of the 1987 Constitution, however, Rep. Act
Commission on Appointments, in the second sentence of No. 1937 and PD No. 34 have to be read in harmony with Sec.
the same Sec. 16, Article VII. 16, Art. VII, with the result that, while the appointment of the
Commissioner of the Bureau of Customs is one that devolves
on the President, as an appointment he is authorizedby law to
Therefore, the third sentence of Sec. 16, Article VII could have make, such appointment, however, no longer needs the
stated merely that, in the case of lower-ranked officers, the confirmation of the Commission on Appointments.
Congress may by law vest their appointment in the President,
in the courts, or in the heads of various departments of the
government. In short, the word "alone" in the third Consequently, we rule that the President of the Philippines
sentence of Sec. 16, Article VII of the 1987 Constitution, as acted within her constitutional authority and power in
a literal import from the last part of par. 3, section 10, appointing respondent Salvador Mison, Commissioner of
Article VII of the 1935 Constitution, appears to be the Bureau of Customs, without submitting his
redundant in the light of the second sentence of Sec. 16, nomination to the Commission on Appointments for
Article VII. And, this redundancy cannot prevail over the clear confirmation. He is thus entitled to exercise the full authority
and positive intent of the framers of the 1987 Constitution that and functions of the office and to receive all the salaries and
presidential appointments, except those mentioned in the first emoluments pertaining thereto.
sentence of Sec. 16, Article VII, are not subject to confirmation
by the Commission on Appointments. WHEREFORE, the petition and petition in intervention should
be, as they are, hereby DISMISSED. Without costs.
Coming now to the immediate question before the Court, it is
evident that the position of Commissioner of the Bureau of SO ORDERED.
Customs (a bureau head) is not one of those within the
first group of appointments where the consent of the
Commission on Appointments is required. As a matter of
fact, as already pointed out, while the 1935 Constitution
includes "heads of bureaus" among those officers whose
appointments need the consent of the Commission on
Appointments, the 1987 Constitution on the other hand,
deliberately excluded the position of "heads of bureaus" from
appointments that need the consent (confirmation) of the
Commission on Appointments.

Moreover, the President is expressly authorized by law to


appoint the Commissioner of the Bureau of Customs. The
original text of Sec. 601 of Republic Act No. 1937, otherwise
known as the Tariff and Customs Code of the Philippines,
which was enacted by the Congress of the Philippines on 22
June 1957, reads as follows:

601. Chief Officials of the Bureau.-The Bureau of Customs


shall have one chief and one assistant chief, to be known
respectively as the Commissioner (hereinafter known as the
'Commissioner') and Assistant Commissioner of Customs, who
shall each receive an annual compensation in accordance with
the rates prescribed by existing laws. The Assistant
Commissioner of Customs shall be appointed by the proper
department head.

Sec. 601 of Republic Act No. 1937, was amended on 27


October 1972 by Presidential Decree No. 34, amending the
Tariff and Customs Code of the Philippines. Sec. 601, as thus
amended, now reads as follows:

Sec. 601. Chief Officials of the Bureau of Customs.-The


Bureau of Customs shall have one chief and one assistant
chief, to be known respectively as the Commissioner
(hereinafter known as Commissioner) and Deputy
Commissioner of Customs, who shall each receive an annual
compensation in accordance with the rates prescribed by
existing law. The Commissioner and the Deputy
FINALS CONSTITUTION I ACJUCO 96

G.R. No. 86439 April 13, 1989 Barely a year after Mison, the Court is again confronted with a
similar question, this time, whether or not the appointment by
MARY CONCEPCION BAUTISTA, petitioner, the President of the Chairman of the Commission on Human
Rights (CHR), an "independent office" created by the 1987
vs.
SENATOR JOVITO R. SALONGA, COMMISSION ON Constitution, is to be made with or without the confirmation of
APPOINTMENTS COMMITTEE ON JUSTICE, JUDICIAL the Commission on Appointments (CA, for brevity). Once
AND BAR COUNCIL AND HUMAN RIGHTS AND HESIQUIO more, as in Mison, the Court will resolve the issue irrespective
R. MALLILLIN, respondents. of the parties involved in the litigation, mindful that what really
matters are the principles that will guide this Administration and
others in the years to come.
Mary Concepcion Bautista for and in her own behalf.
Since the position of Chairman of the Commission on Human
Christine A.Tomas Espinosa for private respondent Hesiquio Rights is not among the positions mentioned in the first
R. Mallillin sentence of Sec. 16, Art. VII of the 1987 Constitution,
appointments to which are to be made with the confirmation of
PADILLA, J.: the Commission on Appointments, it follows that the
appointment by the President of the Chairman of the (CHR), is
to be made without the review or participation of the
The Court had hoped that its decision in Sarmiento III vs. Commission on Appointments.
Mison, 1 would have settled the question of which
appointments by the President, under the 1987 Constitution,
are to be made with and without the review of the Commission To be more precise, the appointment of the Chairman and
on Appointments. The Mison case was the first major case Members of the Commission on Human Rights is not
under the 1987 Constitution and in construing Sec. 16, Art. VII specifically provided for in the Constitution itself, unlike the
of the 1987 Constitution which provides: Chairmen and Members of the Civil Service Commission, the
Commission on Elections and the Commission on Audit,
whose appointments are expressly vested by the Constitution
The President shall nominate and, with the consent of the in the President with the consent of the Commission on
Commission on Appointments, appoint the heads of the Appointments. 2
executive departments, ambassadors, other public ministers
and consuls, or officers of the armed forces from the rank of
colonel or naval captain, and other officers whose The President appoints the Chairman and Members of the
appointments are vested in him in this Constitution. He shall Commission on Human Rights pursuant to the second
also appoint all other officers of the Government whose sentence in Section 16, Art. VII, that is, without the confirmation
appointments are not otherwise provided for by law, and those of the Commission on Appointments because they are among
whom he may be authorized by law to appoint. The Congress the officers of government "whom he (the President) may be
may, by law, vest the appointment of other officers lower in authorized by law to appoint." And Section 2(c), Executive
rank in the President alone, in the courts, or in the heads of the Order No. 163, 5 May 1987, authorizes the President to
departments, agencies, commissions or boards. appoint the Chairman and Members of the Commission on
Human Rights. It provides:
The President shall have the power to
make appointments during the recess of (c) The Chairman and the Members of the
the Congress, whether voluntary or Commission on Human Rights shall be
compulsory, but such appointments shall appointed by the President for a term of
be effective only until disapproval by the seven years without reappointment.
Commission on Appointments or until the Appointment to any vacancy shall be only
next adjournment of the Congress. for the unexpired term of the predecessor.

this Court, drawing extensively from the proceedings of the The above conclusions appear to be plainly evident and,
1986 Constitutional Commission and the country's experience therefore, irresistible. However, the presence in this case of
under the 1935 and 1973 Constitutions, held that only those certain elements — absent in the Mison case — makes
appointments expressly mentioned in the first sentence of Sec. necessary a closer scrutiny. The facts are therefore essential.
16, Art. VII are to be reviewed by the Commission on
Appointments, namely, "the heads of the executive On 27 August 1987, the President of the
department, ambassadors, other public ministers and consuls, Philippines designated herein petitioner Mary Concepcion
or officers of the armed forces from the rank of colonel or naval Bautista as "Acting Chairman, Commission on Human Rights."
captain, and other officers whose appointments are vested in The letter of designation reads:
him in this Constitution." All other appointments by the
President are to be made without the participation of the
Commission on Appointments. Accordingly, in the Mison case, 27 August 1987
the appointment of therein respondent Salvador M. Mison as
head of the Bureau of Customs, without the confirmation of the M a d a m:
Commission on Appointments, was held valid and in
accordance with the Constitution. You are hereby designated ACTING CHAIRMAN,
COMMISSION ON HUMAN RIGHTS, to succeed the late
The Mison case doctrine did not foreclose contrary opinions. Senator Jose W. Diokno and Justice J. B. L. Reyes.
So with the very provisions of Sec. 16, Art. VII as designed by
the framers of the 1987 Constitution. But the Constitution, as Very truly HON. MARY CONCEPCION BAUTISTA 3
construed by this Court in appropriate cases, is the supreme
law of the land. And it cannot be over-stressed that the strength
of the Constitution, with all its imperfections, lies in the respect Realizing perhaps the need for a permanent chairman and
and obedience accorded to it by the people, especially the members of the Commission on Human Rights, befitting an
officials of government, who are the subjects of its commands. independent office, as mandated by the Constitution, 4 the
FINALS CONSTITUTION I ACJUCO 97

President of the Philippines on 17 December 1988 extended had originally held merely in an acting capacity beginning 27
to petitioner Bautista a permanent appointment as Chairman August 1987.
of the Commission. The appointment letter is as follows:
On 9 January 1989, petitioner Bautista received a letter from
17 December 1988 the Secretary of the Commission on Appointments requesting
her to submit to the Commission certain information and
The Honorable documents as required by its rules in connection with the
confirmation of her appointment as Chairman of the
The Chairman
Commission on Human Rights Commission on Human Rights. 7 On 10 January 1989, the
Pasig, Metro Manila Commission on Appointments' Secretary again wrote
petitioner Bautista requesting her presence at a meeting of the
Commission on Appointments Committee on Justice, Judicial
M a d a m: and Bar Council and Human Rights set for 19 January 1989 at
9 A.M. at the Conference Room, 8th Floor, Kanlaon Tower I,
Pursuant to the provisions of existing laws, the following are Roxas Boulevard, Pasay City that would deliberate on her
hereby appointed to the positions indicated opposite their appointment as Chairman of the Commission on Human
respective names in the Commission on Human Rights: Rights. 8

MARY CONCEPCION BAUTISTA — Chairman On 13 January 1989, petitioner Bautista wrote to the Chairman
ABELARDO L. APORTADERA, JR — Member of the Commission on Appointments stating, for the reasons
SAMUEL SORIANO — Member therein given, why she considered the Commission on
HESIQUIO R. MALLILLIN — Member Appointments as having no jurisdiction to review her
NARCISO C. MONTEIRO — Member appointment as Chairman of the Commission on Human
Rights. The petitioner's letter to the Commission on
Appointments' Chairman reads:
By virtue hereof, they may qualify and enter upon the
performance of the duties of the office furnishing this Office and
the Civil Service Commission with copies of their oath of office. January 13, 1 989

Very truly yours, SENATE PRESIDENT JOVITO R. SALONGA


Chairman
Commission on Appointments
CORAZON C. AQUINO 5 Senate, Manila

It is to be noted that by virtue of such appointment, petitioner S i r:


Bautista was advised by the President that she could qualify
and enter upon the performance of the duties of the office of
Chairman of the Commission on Human Rights, requiring her We acknowledge receipt of the communication from the
to furnish the office of the President and the Civil Service Commission on Appointments requesting our appearance on
Commission with copies of her oath of office. January 19, 1989 for deliberation on our appointments.

On 22 December 1988, before the Chief Justice of this Court, We respectfully submit that the appointments of the
Hon. Marcelo B. Fernan, petitioner Bautista took her oath of Commission commissioners of the Human Rights Commission
office by virtue of her appointment as Chairman of the are not subject to confirmation by the Commission on
Commission on Human Rights. The full text of the oath of office Appointments.
is as follows:
The Constitution, in Article VII Section 16 which expressly
OATH OF OFFICE vested on the President the appointing power, has expressly
mentioned the government officials whose appointments are
subject to the confirmation of the Commission on
I, MARY CONCEPCION BAUTISTA of 3026 General G. del Appointments of Congress. The Commissioners of the
Pilar Street, Bangkal, Makati, Metro Manila having been Commission on Human Rights are not included among those.
appointed to the position of CHAIRMAN of the Commission on
Human Rights, do solemnly swear that I will discharge to the
best of my ability all the duties and responsibilities of the office Where the confirmation of the Commission on Appointments is
to which I have been appointed; uphold the Constitution of the required, as in the case of the Constitutional Commissions
Republic of the Philippines, and obey all the laws of the land such as the Commission on Audit, Civil Service Commission
without mental reservation or purpose of evasion. and the Commission on Elections, it was expressly provided
that the nominations will be subject to confirmation of
Commission on Appointments. The exclusion again of the
SO HELP ME GOD. Commission on Human Rights, a constitutional office, from this
enumeration is a clear denial of authority to the Commission
MARY CONCEPCION BAUTISTA on Appointments to review our appointments to the
Commission on Human Rights.
SUBSCRIBED AND SWORN TO before me this 22nd day of
December in the year of Our Lord, 1988 in Manila. Furthermore, the Constitution specifically provides that this
Commission is an independent office which:
Immediately, after taking her oath of office as Chairman of the
Commission on Human Rights, petitioner Bautista discharged a. must investigate all forms of human rights violations
the functions and duties of the Office of Chairman of the involving civil and political rights;
Commission on Human Rights which, as previously stated, she
FINALS CONSTITUTION I ACJUCO 98

b. shall monitor the government's compliance in all our treaty RAOUL V. VICTORINO
obligations on human rights. We submit that, the monitoring of Secretary 11
all agencies of government, includes even Congress itself, in
the performance of its functions which may affect human rights;
On the same date (1 February 1989), the Commission on
Appointments' Secretary informed petitioner Bautista that the
c. may call on all agencies of government for the motion for reconsideration of the disapproval of her "ad
implementation of its mandate. interim appointment as Chairman of the Commission on
Human Rights" was denied by the Commission on
The powers of the Commission on Appointments is in fact a Appointments. The letter reads as follows:
derogation of the Chief Executive's appointing power and
therefore the grant of that authority to review a valid exercise 1 February 1989
of the executive power can never be presumed. It must be
expressly granted. ATTY. MARY CONCEPCION BAUTISTA
Commission on Human Rights
The Commission on Appointments has no jurisdiction under Integrated Bar of the Philippines
the Constitution to review appointments by the President of Bldg. Pasig, Metro Manila
Commissioners of the Commission on Human Rights.
Dear Atty. Bautista:
In view of the foregoing considerations, as Chairman of an
independent constitutional office. I cannot submit myself to the
Pursuant to Sec. 6 (a), Chapter II of the Rules of the
Commission on Appointments for the purpose of confirming or Commission on Appointments, the denial by the Commission
rejecting my appointment.
on Appointments, assembled in plenary (session) earlier
today, of Senator Mamintal A.J. Tamano's motion for
Very truly yours, reconsideration of the disapproval of your ad
interim appointment as Chairperson of the Commission on
Human Rights is respectfully conveyed.
MARY CONCEPCION BAUTISTA
Chairman9
Thank you for your attention.
In respondent Commission's comment (in this case), dated 3
February 1989, there is attached as Annex 1 a letter of the Very truly yours,
Commission on Appointments' Secretary to the Executive
Secretary, Hon. Catalino Macaraig, Jr. making reference to the RAOUL V. VICTORINO
"ad interim appointment which Her Excellency extended to Secretary 12
Atty. Mary Concepcion Bautista on 14 January 1989 as
Chairperson of the Commission on Human Rights" 10 and
informing Secretary Macaraig that, as previously conveyed to In Annex 3 of respondent Commission's same comment, dated
him in a letter of 25 January 1989, the Commission on 3 February 1989, is a news item appearing in the 3 February
Appointments disapproved petitioner Bautista's "ad 1989 issue of the "Manila Standard" reporting that the
interim appointment' as Chairperson of the Commission on President had designated PCHR Commissioner Hesiquio R.
Human Rights in view of her refusal to submit to the jurisdiction Mallillin as "Acting Chairman of the Commission" pending the
of the Commission on Appointments. The letter reads: resolution of Bautista's case which had been elevated to the
Supreme Court. The news item is here quoted in full, thus —
1 February 1989
Aquino names replacement for MaryCon
HON. CATALINO MACARAIG, JR.
Executive Secretary President Aquino has named replacement for Presidential
Malacanang, Manila Commission on Human Rights Chairman Mary Concepcion
Bautista whose appointment was rejected anew by the
Congressional commission on appointments.
S i r:

The President designated PCHR commissioner Hesiquio R.


This refers to the ad interim appointment which Her Excellency Mallillin as acting chairman of the Commission pending the
extended to Atty. Mary Concepcion Bautista on 14 January
resolution of Bautista's case which had been elevated to the
1989 as Chairperson of the Commission on Human Rights. Supreme Court.

As we conveyed to you in our letter of 25 January 1989, the The President's action followed after Congressional
Commission on Appointments, assembled in plenary (session)
Commission on Appointments Chairman, Senate President
on the same day, disapproved Atty. Bautista's ad Jovito Salonga declared Bautista can no longer hold on to her
interim appointment as Chairperson of the Commission on
position after her appointment was not confirmed for the
Human Rights in view of her refusal to submit to the jurisdiction second time.
of the Commission on Appointments.

For all practical purposes, Salonga said Bautista can be


This is to inform you that the Commission on Appointments,
accused of usurpation of authority if she insists to stay on her
likewise assembled in plenary (session) earlier today, denied office.
Senator Mamintal A. J. Tamano's motion for reconsideration of
the disapproval of Atty. Bautista's ad interim appointment as
Chairperson of the Commission on Human Rights. In effect, the President had asked Bautista to vacate her office
and give way to Mallillin (Mari Villa) 13
Very truly yours,
FINALS CONSTITUTION I ACJUCO 99

On 20 January 1989, or even before the respondent the 1987 Constitution and the doctrine in Mison which is here
Commission on Appointments had acted on her "ad reiterated.
interimappointment as Chairman of the Commission on
Human Rights" petitioner Bautista filed with this Court the
The threshold question that has really come to the fore is
present petition for certiorari with a prayer for the immediate whether the President, subsequent to her act of 17 December
issuance of a restraining order, to declare "as unlawful and 1988, and after petitioner Bautista had qualified for the office
unconstitutional and without any legal force and effect any to which she had been appointed, by taking the oath of office
action of the Commission on Appointments as well as of the and actually assuming and discharging the functions and
Committee on Justice, Judicial and Bar Council and Human duties thereof, could extend another appointment to the
Rights, on the lawfully extended appointment of the petitioner petitioner on 14 January 1989, an "ad interim appointment" as
as Chairman of the Commission on Human Rights, on the termed by the respondent Commission on Appointments or
ground that they have no lawful and constitutional authority to any other kind of appointment to the same office of Chairman
confirm and to review her appointment." 14
of the Commission on Human Rights that called for
confirmation by the Commission on Appointments.
The prayer for temporary restraining order was "to enjoin the
respondent Commission on Appointments not to proceed The Court, with all due respect to both the Executive and
further with their deliberation and/or proceedings on the
Legislative Departments of government, and after careful
appointment of the petitioner ... nor to enforce, implement or deliberation, is constrained to hold and rule in the negative.
act on any order, resolution, etc. issued in the course of their
When Her Excellency, the President converted petitioner
deliberations." 15 Bautista's designation as Acting Chairman to a permanent
appointment as Chairman of the Commission on Human
Respondents were required to file comment within ten (10) Rights on 17 December 1988, significantly she advised
days. 16 On 7 February 1989, petitioner filed an amended Bautista (in the same appointment letter) that, by virtue of such
petition, with urgent motion for restraining order, impleading appointment, she could qualify and enter upon the
Commissioner Hesiquio R. Mallillin the designated acting performance of the duties of the office (of Chairman of the
chairman as party respondent and praying for the nullification Commission on Human Rights). All that remained for Bautista
of his appointment. The succeeding day, a supplemental to do was to reject or accept the appointment. Obviously, she
urgent ex-parte motion was filed by petitioner seeking to accepted the appointment by taking her oath of office before
restrain respondent Mallillin from continuing to exercise the the Chief Justice of the Supreme Court, Hon. Marcelo B.
functions of chairman and to refrain from demanding courtesy Fernan and assuming immediately thereafter the functions and
resignations from officers or separating or dismissing duties of the Chairman of the Commission on Human
employees of the Commission. Rights. Bautista's appointment therefore on 17 December
1988 as Chairman of the Commission on Human Rights was a
completed act on the part of the President. To paraphrase the
Acting on petitioner's amended petition and supplemental
urgent ex-parte motion, the Court resolved to issue a great jurist, Mr. Chief Justice Marshall, in the celebrated case
temporary restraining order directing respondent Mallillin to of Marbury vs. Madison. 23
cease and desist from effecting the dismissal, courtesy
resignation, i removal and reorganization and other similar xxx xxx xxx
personnel actions. 17 Respondents were likewise required to
comment on said amended petition with allowance for The answer to this question seems an obvious one. The
petitioner to file a reply within two (2) days from receipt of a appointment being the sole act of the President, must be
copy thereof. completely evidenced, when it is shown that he has done
everything to be performed by him.
Respondents Senator Salonga, the Commission on
Appointments the Committee on J & BC and Human Rights xxx xxx xxx
filed a comment to the amended petition on 21 February
1989. 18 Petitioner filed her reply. 19 On 24 February 1989,
respondent Mallillin filed a separate comment. 20 The Court Some point of time must be taken when the power of the
required petitioner to reply to respondent Mallillin's comment executive over an officer, not removable at his will must cease.
. 21 Petitioner filed her reply. 22 That point of time must be when the constitutional power of
appointment has been exercised. And this power has been
exercised when the last act, required from the person
In deference to the Commission on Appointments, an possessing the power, has been performed. ....
instrumentality of a co-ordinate and co-equal branch of
government, the Court did not issue a temporary restraining
order directed against it. However, this does not mean that the xxx xxx xxx
issues raised by the petition, as met by the respondents'
comments, will not be resolved in this case. The Court will not But having once made the appointment, his (the President's)
shirk from its duty as the final arbiter of constitutional issues, in power over the office is terminated in all cases, where by law
the same way that it did not in Mison. the officer is not removable by him. The right to the office is
then in the person appointed, and he has the absolute,
As disclosed by the records, and as previously adverted to, it unconditional power of accepting or rejecting it.
is clear that petitioner Bautista was extended by Her
Excellency, the President a permanent appointment as xxx xxx xxx
Chairman of the Commission on Human Rights on 17
December 1988. Before this date, she was merely the "Acting
Chairman" of the Commission. Bautista's appointment on 17 THE "APPOINTMENT" OF PETITIONER BAUTISTA ON 14
December 1988 is an appointment that was for the President JANUARY 1989
solely to make, i.e., not an appointment to be submitted for
review and confirmation (or rejection) by the Commission on It is respondent Commission's submission that the President,
Appointments. This is in accordance with Sec. 16, Art. VII of after the appointment of 17 December 1988 extended to
petitioner Bautista, decided to extend another appointment (14
FINALS CONSTITUTION I ACJUCO 100

January 1989) to petitioner Bautista, this time, submitting such made, in the first place, without the participation of the
appointment (more accurately, nomination) to the Commission Commission on Appointments, was then and there a complete
on Appointments for confirmation. And yet, it seems obvious and finished act, which, upon the acceptance by Bautista, as
enough, both in logic and in fact, that no new or further shown by her taking of the oath of office and actual assumption
appointment could be made to a position already filled by a of the duties of said office, installed her, indubitably and
previously completed appointment which had been accepted unequivocally, as the lawful Chairman of the Commission on
by the appointee, through a valid qualification and assumption Human Rights for a term of seven (7) years. There was thus
of its duties. no vacancy in the subject office on 14 January 1989 to which
an appointment could be validly made. In fact, there is no
Respondent Commission vigorously contends that, granting vacancy in said office to this day.
that petitioner's appointment as Chairman of the Commission
on Human Rights is one that, under Sec. 16, Art. VII of the Nor can respondents impressively contend that the new
Constitution, as interpreted in the Mison case, is solely for the appointment or re-appointment on 14 January 1989 was an ad
President to make, yet, it is within the president's prerogative interim appointment, because, under the Constitutional
to voluntarily submit such appointment to the Commission on design, ad interim appointments do not apply to appointments
Appointment for confirmation. The mischief in this contention, solely for the President to make, i.e., without the participation
as the Court perceives it, lies in the suggestion that the of the Commission on Appointments. Ad interim appointments,
President (with Congress agreeing) may, from time to by their very nature under the 1987 Constitution, extend only
time move power boundaries, in the Constitution differently to appointments where the review of the Commission on
from where they are placed by the Constitution. Appointments is needed. That is why ad interim appointments
are to remain valid until disapproval by the Commission on
The Court really finds the above contention difficult of Appointments or until the next adjournment of Congress; but
acceptance. Constitutional Law, to begin with, is concerned appointments that are for the President solely to make, that is,
with power not political convenience, wisdom, exigency, or without the participation of the Commission on Appointments,
even necessity. Neither the Executive nor the Legislative can not be ad interim appointments.
(Commission on Appointments) can create power where the
Constitution confers none. The evident constitutional intent is EXECUTIVE ORDER NO. 163-A, 30 JUNE 1987, PROVIDING
to strike a careful and delicate balance, in the matter of THAT THE TENURE OF THE CHAIRMAN AND MEMBERS
appointments to public office, between the President and OF THE COMMISSION ON HUMAN RIGHTS SHALL BE AT
Congress (the latter acting through the Commission on THE PLEASURE OF THE PRESIDENT IS
Appointments). To tilt one side or the other of the scale is to UNCONSTITUTIONAL.
disrupt or alter such balance of power. In other words, to the
extent that the Constitution has blocked off certain Respondent Mallillin contends that with or without confirmation
appointments for the President to make with the participation
by the Commission on Appointments, petitioner Bautista, as
of the Commission on Appointments, so also has the Chairman of the Commission on Human Rights, can be
Constitution mandated that the President can confer no power removed from said office at anytime, at the pleasure of the
of participation in the Commission on Appointments over other President; and that with the disapproval of Bautista's
appointments exclusively reserved for her by the Constitution. appointment (nomination) by the Commission on
The exercise of political options that finds no support in the Appointments, there was greater reason for her removal by the
Constitution cannot be sustained. President and her replacement with respondent Mallillin Thus,
according to respondent Mallillin the petition at bar has become
Nor can the Commission on Appointments, by the actual moot and academic.
exercise of its constitutionally delimited power to review
presidential appointments, create power to confirm We do not agree that the petition has become moot and
appointments that the Constitution has reserved to the academic. To insist on such a posture is akin to deluding
President alone. Stated differently, when the appointment is oneself that day is night just because the drapes are drawn and
one that the Constitution mandates is for the President to make
the lights are on. For, aside from the substantive questions of
without the participation of the Commission on Appointments, constitutional law raised by petitioner, the records clearly show
the executive's voluntary act of submitting such appointment to
that petitioner came to this Court in timely manner and has not
the Commission on Appointments and the latter's act of shown any indication of abandoning her petition.
confirming or rejecting the same, are done without or in excess
of jurisdiction.
Reliance is placed by respondent Mallillin on Executive Order
No. 163-A, 30 June 1987, full text of which is as follows:
EVEN IF THE PRESIDENT MAY VOLUNTARILY SUBMIT TO
THE COMMISSION ON APPOINTMENTS AN
APPOINTMENT THAT UNDER THE CONSTITUTION WHEREAS, the Constitution does not prescribe the term of
SOLELY BELONGS TO HER, STILL, THERE WAS NO office of the Chairman and Members of the Commission on
VACANCY TO WHICH AN APPOINTMENT COULD BE Human Rights unlike those of other Constitutional
MADE ON 14 JANUARY 1989 Commissions;

Under this heading, we will assume, ex gratia argumenti, that NOW, THEREFORE, I, CORAZON C. AQUINO, President of
the Executive may voluntarily allow the Commission on the Philippines, do hereby order:
Appointments to exercise the power of review over an
appointment otherwise solely vested by the Constitution in the SECTION 1. Section 2, sub-paragraph (c) of Executive Order
President. Yet, as already noted, when the President No. 163 is hereby amended to read as follows:
appointed petitioner Bautista on 17 December 1988 to the
position of Chairman of the Commission on Human Rights with
the advice to her that by virtue of such appointment (not, until The Chairman and Members of the Commission on Human
confirmed by the Commission on Appointments), she could Rights shall be appointed by the President. Their tenure in
qualify and enter upon the performance of her duties after office shall be at the pleasure of the President.
taking her oath of office, the presidential act of appointment to
the subject position which, under the Constitution, is to be
FINALS CONSTITUTION I ACJUCO 101

SEC. 2. This Executive Order shall take effect immediately. Indeed, the Court finds it extremely difficult to conceptualize
DONE in the City of Manila, this 30th day of June, in the year how an office conceived and created by the Constitution to be
of Our Lord, nineteen hundred and eighty-seven. independent as the Commission on Human Rights-and vested
with the delicate and vital functions of investigating violations
(Sgd.) CORAZON C. AQUINO of human rights, pinpointing responsibility and recommending
President of the Philippines sanctions as well as remedial measures therefor, can truly
function with independence and effectiveness, when
the tenure in office of its Chairman and Members is made
By the President: dependent on the pleasure of the President. Executive Order
No. 163-A, being antithetical to the constitutional mandate of
(Sgd.) JOKER P. ARROYO independence for the Commission on Human Rights has to be
Executive Secretary 24 declared unconstitutional.

Previous to Executive Order No. 163-A, or on 5 May 1987, The Court is not alone in viewing Executive Order No. 163-A
Executive Order No. 163 25 was issued by the President, Sec. as containing the seeds of its constitutional destruction. The
2(c) of which provides: proceedings in the 1986 Constitutional Commission clearly
point to its being plainly at war with the constitutional intent of
independence for the Commission. Thus —
Sec. 2(c). The Chairman and the Members of the Commission
on Human Rights shall be appointed by the President for a
term of seven years without reappointment. Appointments to MR. GARCIA (sponsor). Precisely, one of the reasons why it is
any vacancy shall be only for the unexpired term of the important for this body to be constitutionalized is the fact that
predecessor. regardless of who is the President or who holds the executive
power, the human rights issue is of such importance that it
should be safeguarded and it should be independent of
It is to be noted that, while the earlier executive order (No. 163) political parties or powers that are actually holding the reins of
speaks of a term of office of the Chairman and Members of the government. Our experience during the martial law period
Commission on Human Rights — which is seven (7) years made us realize how precious those rights are and, therefore,
without reappointment — the later executive order (163-A) these must be safeguarded at all times.
speaks of the tenure in office of the Chairman and Members of
the Commission on Human Rights, which is "at the pleasure of
the President." xxx xxx xxx

Tenure in office should not be confused with term of office. As MR. GARCIA. I would like to state this fact: Precisely we do not
Mr. Justice (later, Chief Justice) Concepcion in his concurring want the term or the power of the Commission on Human
opinion in Alba vs. Evangelista, 26 stated: Rights to be coterminous with the president, because the
President's power is such that if he appoints a certain
commissioner and that commissioner is subject to the
The distinction between "term" and "tenure" is important, for, President, therefore, any human rights violations committed
pursuant to the Constitution, "no officer or employee in the Civil under the person's administration will be subject to presidential
Service may be removed or suspended except for cause, as pressure. That is what we would like to avoid — to make the
provided by law" (Art. XII, section 4), and this fundamental protection of human rights go beyond the fortunes of different
principle would be defeated if Congress could legally make the political parties or administrations in power. 28
tenure of some officials dependent upon the pleasure of the
President, by clothing the latter with blanket authority to
replace a public officer before the expiration of his term. 27 xxx xxx xxx

When Executive Order No. 163 was issued, the evident MR. SARMIENTO (sponsor). Yes, Madam President. I
purpose was to comply with the constitutional provision that conferred with the honorable Chief Justice Concepcion and
"the term of office and other qualifications and disabilities of retired Justice J.B.L. Reyes and they believe that there should
the Members of the Commission (on Human Rights) shall be be an independent Commission on Human Rights free from
provided by law" (Sec. 17(2), Art. XIII, 1987 Constitution). executive influence because many of the irregularities on
human rights violations are committed by members of the
armed forces and members of the executive branch of the
As the term of office of the Chairman (and Members) of the government. So as to insulate this body from political
Commission on Human Rights, is seven (7) years, without interference, there is a need to constitutionalize it. 29
reappointment, as provided by Executive Order No. 163, and
consistent with the constitutional design to give the
Commission the needed independence to perform and xxx xxx xxx
accomplish its functions and duties, the tenure in office of said
Chairman (and Members) cannot be later made dependent on MR. SARMIENTO: On the inquiry on whether there is a need
the pleasure of the President. for this to be constitutionalized, I would refer to a previous
inquiry that there is still a need for making this a constitutional
Nor can respondent Mallillin find support in the majority opinion body free or insulated from interference. I conferred with former
in the Alba case, supra, because the power of the President, Chief Justice Concepcion and the acting chairman of the
sustained therein, to replace a previously appointed vice- Presidential Committee on Human Rights, retired Justice
mayor of Roxas City given the express provision in Sec. 8, J.B.L. Reyes, and they are one in saying that this body should
Rep. Act No. 603 (creating the City of Roxas) stating that the be constitutionalized so that it will be free from executive
vice-mayor shall serve at the pleasure of the President, can control or interferences, since many of the abuses are
find no application to the Chairman of an INDEPENDENT committed by the members of the military or the armed
OFFICE, created not by statute but by the Constitution itself. forces. 30
Besides, unlike in the Alba case, here the Constitution has
decreed that the Chairman and Members of the Commission xxx xxx xxx
on Human Rights shall have a "term of office."
FINALS CONSTITUTION I ACJUCO 102

MR. SARMIENTO. Yes, Congress can create this body, but as If there are charges against Bautista for misfeasance or
I have said, if we leave it to Congress, this commission will be malfeasance in office, charges may be filed against her with
within the reach of politicians and of public officers and that to the Ombudsman. If he finds a prima facie case against her, the
me is dangerous. We should insulate this body from political corresponding information or informations can be filed with the
control and political interference because of the nature of its Sandiganbayan which may in turn order her suspension from
functions to investigate all forms of human rights violations office while the case or cases against her are pending before
which are principally committed by members of the military, by said court. 37 This is due process in action. This is the way of a
the Armed Forces of the Philippines. 31 government of laws and not of men.

xxx xxx xxx A FINAL WORD

MR. GARCIA. The critical factor here is political control, and It is to the credit of the President that, in deference to the rule
normally, when a body is appointed by Presidents who may of law, after petitioner Bautista had elevated her case to this
change, the commission must remain above these changes in Tribunal, Her Excellency merely designated an Acting
political control. Secondly, the other important factor to Chairman for the Commission on Human Rights (pending
consider are the armed forces, the police forces which have decision in this case) instead of appointing another permanent
tremendous power at their command and, therefore, we would Chairman. The latter course would have added only more legal
need a commission composed of men who also are beyond difficulties to an already difficult situation.
the reach of these forces and the changes in political
administration. 32 WHEREFORE, the petition is GRANTED. Petitioner Bautista
is declared to be, as she is, the duly appointed Chairman of the
xxx xxx xxx Commission on Human Rights and the lawful incumbent
thereof, entitled to all the benefits, privileges and emoluments
MR MONSOD. Yes, It is the committee's position that this of said office. The temporary restraining order heretofore
issued by the Court against respondent Mallillin enjoining him
proposed special body, in order to function effectively, must be
invested with an independence that is necessary not only for from dismissing or terminating personnel of the Commission
on Human Rights is made permanent.
its credibility but also for the effectiveness of its work. However,
we want to make a distinction in this Constitution. May be what
happened was that it was referred to the wrong committee. In SO ORDERED.
the opinion of the committee, this need not be a commission
that is similar to the three constitutional commissions like the
COA, the COMELEC, and the Civil Service. It need not be in
that article. 33

xxx xxx xxx

MR. COLAYCO. The Commissioners earlier objection was that


the Office of the President is not involved in the project. How
sure are we that the next President of the Philippines will be
somebody we can trust? Remember, even now there is a
growing concern about some of the bodies, agencies and
commission created by President Aquino. 34

xxx xxx xxx

.... Leaving to Congress the creation of the Commission on


Human Rights is giving less importance to a truly fundamental
need to set up a body that will effectively enforce the rules
designed to uphold human rights. 35

PETITIONER BAUTISTA MAY OF COURSE BE REMOVED


BUT ONLY FOR CAUSE

To hold, as the Court holds, that petitioner Bautista is the lawful


incumbent of the office of Chairman of the Commission on
Human Rights by virtue of her appointment, as such, by the
President on 17 December 1988, and her acceptance thereof,
is not to say that she cannot be removed from office before the
expiration of her seven (7) year term. She certainly can be
removed but her removal must be for cause and with her right
to due process properly safeguarded. In the case of NASECO
vs. NLRC, 36 this Court held that before a rank-and-file
employee of the NASECO, a government-owned corporation,
could be dismissed, she was entitled to a hearing and due
process. How much more, in the case of the Chairman of
a constitutionally mandated INDEPENDENT OFFICE, like the
Commission on Human Rights.
FINALS CONSTITUTION I ACJUCO 103

G.R. No. 91636 April 23, 1992 (confirmation) of the Commission on


Appointments.
PETER JOHN D. CALDERON, petitioner,
vs. . . . Consequently, we rule that the
BARTOLOME CARALE, in his capacity as Chairman of the President of the Philippines acted within
National Labor Relations Commission, EDNA BONTO her constitutional authority and power in
PEREZ, LOURDES C. JAVIER, ERNESTO G. LADRIDO III, appointing respondent Salvador Mison,
MUSIB M. BUAT, DOMINGO H. ZAPANTA, VICENTE S.E. Commissioner of the Bureau of Customs,
VELOSO III, IRENEO B. BERNARDO, IRENEA E. CENIZA, without submitting his nomination to the
LEON G. GONZAGA, JR., ROMEO B. PUTONG, ROGELIO Commission on Appointments for
I. RAYALA, RUSTICO L. DIOKNO, BERNABE S. BATUHAN confirmation. . . .
and OSCAR N. ABELLA, in their capacity as
Commissioners of the National Labor Relations . . . In the 1987 Constitution, however, as
Commission, and GUILLERMO CARAGUE, in his capacity already pointed out, the clear and
as Secretary of Budget and Management, respondents. expressed intent of its framers was to
exclude presidential appointments from
confirmation by the Commission on
Appointments, except appointments to
PADILLA, J.: offices expressly mentioned in the first
sentence of Sec. 16, Art.VII. Consequently,
there was no reason to use in the third
Controversy is focused anew on Sec. 16, Art. VII of the 1987 sentence of Sec. 16, Article VII the word
Constitution which provides: "alone" after the word "President" in
providing that Congress may by law vest
Sec. 16. The President shall nominate and, the appointment of lower-ranked officers in
with the consent of the Commission on the President alone, or in the courts, or in
Appointments, appoint the heads of the the heads of departments, because the
executive departments, ambassadors, power to appoint officers whom he (the
other public ministers and consuls, or president) may be authorized by law to
officers of the armed forces from the rank appoint is already vested in the President,
of colonel or naval captain, and other without need of confirmation by the
officers whose appointments are vested in Commission on Appointments, in the
him in this Constitution. He shall also second sentence of the same Sec. 16,
appoint all other officers of the Government Article VII." (emphasis supplied)
whose appointments are not otherwise
provided for by law, and those whom he Next came Mary Concepcion Bautista v. Salonga, 3 this time
may be authorized by law to appoint. The involving the appointment of the Chairman of the Commission
Congress may, by law, vest the on Human Rights. Adhering to the doctrine in Mison, the Court
appointment of other officers lower in rank explained:
in the President alone, in the courts, or in
the heads of departments, agencies,
commissions, or boards. . . . Since the position of Chairman of the
Commission on Human Rights is not
among the positions mentioned in the first
The President shall have the power to sentence of Sec. 16, Art. VII of the 1987
make appointments during the recess of Constitution, appointments to which are to
the Congress, whether voluntary or be made with the confirmation of the
compulsory, but such appointments shall Commission on Appointments, it follows
be effective only until disapproval by the that the appointment by the President of the
Commission on Appointments or until the Chairman of the CHR is to be made without
next adjournment of the Congress. 1 the review or participation of the
Commission on Appointments. To be more
The power of the Commission on Appointments (CA for precise, the appointment of the Chairman
brevity) to confirm appointments, contained in the aforequoted and Members of the Commission on
paragraph 1 of Sec. 16, Art. VII, was first construed Human Rights is not specifically provided
in Sarmiento III vs. Mison 2 as follows: for in the Constitution itself, unlike the
Chairmen and Members of the Civil Service
Commission, the Commission on Elections
. . . it is evident that the position of and the Commission on Audit, whose
Commissioner of the Bureau of Customs (a appointments are expressly vested by the
bureau head) is not one of those within the Constitution in the president with the
first group of appointments where the consent of the Commission on
consent of the Commission on Appointments. The president appoints the
Appointments is required. As a matter of Chairman and Members of The
fact, as already pointed out, while the 1935 Commission on Human Rights pursuant to
Constitution includes "heads of bureaus" the second sentence in Section 16, Art. VII,
among those officers whose appointments that is, without the confirmation of the
need the consent of the Commission on Commission on Appointments because
Appointments, the 1987 Constitution, on they are among the officers of government
the other hand, deliberately excluded the "whom he (the President) may be
position of "heads of bureaus" from authorized by law to appoint." And Section
appointments that need the consent 2(c), Executive Order No. 163, 5 May 1987,
authorizes the President to appoint the
FINALS CONSTITUTION I ACJUCO 104

Chairman and Members of the performance of the duties of the office. After said
Commission on Human Rights. appointments, then Labor Secretary Franklin Drilon issued
Administrative Order No. 161, series of 1989, designating the
places of assignment of the newly appointed commissioners.
Consistent with its rulings in Mison and Bautista, in Teresita
Quintos Deles, et al. v. The Commission on Constitutional
Commissions, et al.,4 the power of confirmation of the This petition for prohibition questions the constitutionality and
Commission on Appointments over appointments by the legality of the permanent appointments extended by the
President of sectoral representatives in Congress was upheld President of the Philippines to the respondents Chairman and
because: Members of the National Labor Relations Commission
(NLRC), without submitting the same to the Commission on
. . . Since the seats reserved for sectoral Appointments for confirmation pursuant to Art. 215 of the Labor
representatives in paragraph 2, Section 5, Code as amended by said RA 6715.
Art. VI may be filled by appointment by the
President by express provision of Section Petitioner insists on a mandatory compliance with RA 6715
7, Art. XVIII of the Constitution, it is which has in its favor the presumption of validity. RA 6715 is
indubitable that sectoral representatives to not, according to petitioner, an encroachment on the
the House of Representatives are among appointing power of the executive contained in Section 16, Art.
the "other officers whose appointments are VII, of the Constitution, as Congress may, by law, require
vested in the President in this Constitution," confirmation by the Commission on Appointments of other
referred to in the first sentence of Section officers appointed by the President additional to those
16, Art. VII whose appointments are mentioned in the first sentence of Section 16 of Article VII of
subject to confirmation by the Commission the Constitution. Petitioner claims that
on Appointments. the Mison and Bautista rulings are not decisive of the issue in
this case for in the case at bar, the President issued permanent
From the three (3) cases above-mentioned, these doctrines appointments to the respondents without submitting them to
are deducible: the CA for confirmation despite passage of a law (RA 6715)
which requires the confirmation by the Commission on
Appointments of such appointments.
1. Confirmation by the Commission on Appointments is
required only for presidential appointees mentioned in the first
sentence of Section 16, Article VII, including, those officers The Solicitor General, on the other hand, contends that RA
whose appointments are expressly vested by the Constitution 6715 which amended the Labor Code transgressesSection 16,
itself in the president (like sectoral representatives to Congress Article VII by expanding the confirmation powers of the
and members of the constitutional commissions of Audit, Civil Commission on Appointments without constitutional
Service and Election). basis. Mison and Bautista laid the issue to rest, says the
Solicitor General, with the following exposition:

2. Confirmation is not required when the President appoints


other government officers whose appointments are not As interpreted by this Honorable Court in
otherwise provided for by law or those officers whom he may the Mison case, confirmation by the
be authorized by law to appoint (like the Chairman and Commission on Appointments is required
exclusively for the heads of executive
Members of the Commission on Human Rights). Also, as
observed in Mison, when Congress creates inferior offices but departments, ambassadors, public
ministers, consuls, officers of the armed
omits to provide for appointment thereto, or provides in an
unconstitutional manner for such appointments, the officers forces from the rank of colonel or naval
are considered as among those whose appointments are not captain, and other officers whose
otherwise provided for by law. appointments are vested in the President
by the Constitution, such as the members
of the various Constitutional Commissions.
Sometime in March 1989, RA 6715 (Herrera-Veloso Law), With respect to the other officers whose
amending the Labor Code (PD 442) was approved. It provides appointments are not otherwise provided
in Section 13 thereof as follows: for by the law and to those whom the
President may be authorized by law to
xxx xxx xxx appoint, no confirmation by the
Commission on Appointments is required.
The Chairman, the Division Presiding
Commissioners and other Commissioners Had it been the intention to allow Congress
shall all be appointed by the President, to expand the list of officers whose
subject to confirmation by the Commission appointments must be confirmed by the
on Appointments. Appointments to any Commission on Appointments, the
vacancy shall come from the nominees of Constitution would have said so by adding
the sector which nominated the the phrase "and other officers required by
predecessor. The Executive Labor Arbiters law" at the end of the first sentence, or the
and Labor Arbiters shall also be appointed phrase, "with the consent of the
by the President, upon recommendation of Commission on Appointments" at the end
the Secretary of Labor and Employment, of the second sentence. Evidently, our
and shall be subject to the Civil Service Constitution has significantly omitted to
Law, rules and regulations. 5 provide for such additions.

Pursuant to said law (RA 6715), President Aquino appointed The original text of Section 16 of Article VII
the Chairman and Commissioners of the NLRC representing of the present Constitution as embodied in
the public, workers and employers sectors. The appointments Resolution No. 517 of the Constitutional
stated that the appointees may qualify and enter upon the Commission reads as follows:
FINALS CONSTITUTION I ACJUCO 105

"The President shall enumerated in Section 16, but the word


nominate and, with the nominate does not any more appear in the
consent of the 2nd and 3rd sentences. Therefore, the
Commission on president's appointment pursuant to the
Appointments, shall 2nd and 3rd sentences needs no
appoint the heads of confirmation. 6
the executive
departments and The only issue to be resolved by the Court in the present case
bureaus, ambassadors, is whether or not Congress may, by law, require confirmation
other public ministers by the Commission on Appointments of appointments
and consuls, or officers extended by the president to government officers additional to
of the armed forces those expressly mentioned in the first sentence of Sec. 16, Art.
from the rank of captain
VII of the Constitution whose appointments require
or commander, and all confirmation by the Commission on Appointments.
other officers of the
Government whose
appointments are not To resolve the issue, we go back to Mison where the Court
herein otherwise stated:
provided for by law, and
those whom he may be . . . there are four (4) groups of officers
authorized by law to whom the President shall appoint. These
appoint. The Congress four (4) groups, to which we will hereafter
may by law vest the refer from time to time, are:
appointment of inferior
officers in the President
alone, in the courts or in First, the heads of the
the heads of the executive departments,
department." ambassadors, other
public ministers and
consuls, officers of the
Three points should be noted regarding armed forces from the
sub-section 3 of Section 10 of Article VII of rank of colonel or naval
the 1935 Constitution and in the original captain, and other
text of Section 16 of Article VII of the officers whose
present Constitution as proposed in appointments are
Resolution No. 517. vested in him in this
Constitution;
First, in both of them, the appointments of
heads of bureaus were required to be Second, all other
confirmed by the Commission on officers of the
Appointments. Government whose
appointments are not
Second, in both of them, the appointments otherwise provided for
of other officers, "whose appointments are by law;
not otherwise provided for by law to
appoint" are expressly made subject to Third, those whom the
confirmation by the Commission on president may be
Appointments. However, in the final version authorized by law to
of Resolution No. 517, as embodied in appoint;
Section 16 of Article VII of the present
Constitution, the appointment of the above
mentioned officers (heads of bureaus; Fourth, officers lower in
other officers whose appointments are not rank whose
provided for by law; and those whom he appointments the
may be authorized by law to appoint) are Congress may by law
excluded from the list of those officers vest in the President
whose appointments are to be confirmed alone. 7
by the Commission on Appointments. This
amendment, reflected in Section 16 of Mison also opined:
Article VII of the Constitution, clearly shows
the intent of the framers to exclude such
appointments from the requirement of In the course of the debates on the text of
confirmation by the Commission on Section 16, there were two (2) major
Appointments. changes proposed and approved by the
Commission. These were (1) the exclusion
of the appointments of heads of bureaus
Third, under the 1935 Constitution the word from the requirement of confirmation by the
"nominate" qualifies the entire Subsection Commission on Appointments; and (2) the
3 of Section 10 of Article VII thereof. exclusion of appointments made under the
second sentence of the section from the
Respondent reiterates that if confirmation same requirement. . . .
is required, the three (3) stage process of
nomination, confirmation and appointment The second sentence of Sec. 16, Art. VII refers to all other
operates. This is only true of the first group officers of the government whose appointments are not
FINALS CONSTITUTION I ACJUCO 106

otherwise provided for by law and those whom the President ascertainment of the meaning of the phrase
may be authorized by law to appoint. "which shall not be diminished during their
continuance in office," found in Section 9,
Article VIII of the Constitution, referring to
Indubitably, the NLRC Chairman and Commissioners fall
within the second sentence of Section 16, Article VII of the the salaries of judicial officers.
Constitution, more specifically under the "third groups" of
appointees referred to in Mison, i.e. those whom the President xxx xxx xxx
may be authorized by law to appoint. Undeniably, the
Chairman and Members of the NLRC are not among the The rule is recognized
officers mentioned in the first sentence of Section 16, Article elsewhere that the
VII whose appointments requires confirmation by the legislature cannot pass
Commission on Appointments. To the extent that RA 6715 any declaratory act, or
requires confirmation by the Commission on Appointments of act declaratory of what
the appointments of respondents Chairman and Members of the law was before its
the National Labor Relations Commission, it is unconstitutional passage, so as to give it
because: any binding weight with
the courts. A legislative
1) it amends by legislation, the first sentence of Sec. 16, Art. definition of a word as
VII of the Constitution by adding thereto appointments used in a statute is not
requiring confirmation by the Commission on Appointments; conclusive of its
and meaning as used
elsewhere; otherwise,
the legislature would be
2) it amends by legislation the second sentence of Sec. 16, Art.
VII of the Constitution, by imposing the confirmation of the usurping a judicial
Commission on Appointments on appointments which are function in defining a
otherwise entrusted only with the President. term. (11 Am. Jur., 914,
emphasis supplied).

Deciding on what laws to pass is a legislative prerogative.


The legislature cannot,
Determining their constitutionality is a judicial function. The
Court respects the laudable intention of the legislature. upon passing law which
Regretfully, however, the constitutional infirmity of Sec. 13 of violates a constitutional
RA 6715 amending Art. 215 of the Labor Code, insofar as it provision, validate it so
requires confirmation of the Commission on Appointments as to prevent an attack
over appointments of the Chairman and Member of the thereon in the courts, by
National Labor Relations Commission (NLRC) is, as we see it, a declaration that it
shall be so construed
beyond redemption if we are to render fealty to the mandate of
the Constitution in Sec. 16, Art. VII thereof. as not to violate the
constitutional inhibition.
(11 Am., Jur., 919,
Supreme Court decisions applying or interpreting the emphasis supplied).
Constitution shall form part of the legal system of the
Philippines.8 No doctrine or principle of law laid down by the
We have already said that the Legislature
Court in a decision rendered en banc or in division may be
modified or reversed except by the Court sitting en banc.9 under our form of government is assigned
the task and the power to make and enact
laws, but not to interpret them. This is more
. . . The interpretation upon a law by this true with regard to the interpretation of the
Court constitutes, in a way, a part of the law basic law, the Constitution, which is not
as of the date that law was originally within the sphere of the Legislative
passed, since this Court's construction department. If the Legislature may declare
merely establishes the contemporaneous what a law means, or what a specific
legislative intent that the law thus portion of the Constitution means,
construed intends to effectuate. The settled especially after the courts have in actual
rule supported by numerous authorities is a case ascertained its meaning by
restatement of the legal maxim "legis interpretation and applied it in a decision,
interpretado legis vim obtinent" — the this would surely cause confusion and
interpretation placed upon the written law instability in judicial processes and court
by a competent court has the force of decisions. Under such a system, a final
law. 10 court determination of a case based on a
judicial interpretation of the law or of the
The rulings in Mison, Bautista and Quintos-Deles have Constitution may be undermined or even
interpreted Art. VII, Sec. 16 consistently in one manner. Can annulled by a subsequent and different
legislation expand a constitutional provision after the Supreme interpretation of the law or of the
Court has interpreted it? Constitution by the Legislative department
that would be neither wise nor desirable,
being clearly violative of the fundamental
In Endencia and Jugo vs. David, 11 the Court held: principles of our constitutional system of
government, particularly those governing
By legislative fiat as enunciated in Section the separation of powers. 14(Emphasis
13, Republic Act No. 590, Congress says supplied)
that taxing the salary of a judicial officer is
not a decrease of compensation. This is a Congress, of course, must interpret the Constitution, must
clear example of interpretation or estimate the scope of its constitutional powers when it sets out
FINALS CONSTITUTION I ACJUCO 107

to enact legislation and it must take into account the relevant


constitutional prohibitions. 15

. . . The Constitution did not change with


public opinion.

It is not only the same words, but the same


in meaning . . . and as long as it it speaks
not only in the same words, but with the
same meaning and intent with which it
spoke when it came from the hands of its
framers, and was voted and adopted by the
people . . . 16

The function of the Court in passing upon an act of Congress


is to "lay the article of the Constitution which is invoked beside
the statute which is challenged and to decide whether the latter
squares with the former" and to "announce its considered
judgment upon the question." 17

It can not be overlooked that Sec. 16, Art. VII of the 1987
Constitution was deliberately, not unconsciously, intended by
the framers of the 1987 Constitution to be a departure from the
system embodied in the 1935 Constitution where the
Commission on Appointments exercised the power of
confirmation over almost all presidential appointments, leading
to many cases of abuse of such power of confirmation.
Subsection 3, Section 10, Art. VII of the 1935 Constitution
provided:

3. The President shall nominate and with


the consent of the Commission on
Appointments, shall appoint the heads of
the executive departments and bureaus,
officers of the Army from the rank of
colonel, of the Navy and Air Forces from
the rank of captain or commander, and all
other officers of the Government whose
appointments are not herein otherwise
provided for, and those whom he may be
authorized by law to appoint; . . .

The deliberate limitation on the power of confirmation of the


Commission on Appointments over presidential appointments,
embodied in Sec. 16, Art. VII of the 1987 Constitution, has
undoubtedly evoked the displeasure and disapproval of
members of Congress. The solution to the apparent problem,
if indeed a problem, is not judicial or legislative but
constitutional. A future constitutional convention or Congress
sitting as a constituent (constitutional) assembly may then
consider either a return to the 1935 Constitutional provisions
or the adoption of a hybrid system between the 1935 and 1987
constitutional provisions. Until then, it is the duty of the Court
to apply the 1987 Constitution in accordance with what it says
and not in accordance with how the legislature or the executive
would want it interpreted.

WHEREFORE, the petition is DISMISSED. Art. 215 of the


Labor Code as amended by RA 6715 insofar as it requires the
confirmation of the Commission on Appointments of
appointments of the Chairman and Members of the National
Labor Relations Commission (NLRC) is hereby declared
unconstitutional and of no legal force and effect.

SO ORDERED.
FINALS CONSTITUTION I ACJUCO 108

[G.R. No. 107369. August 11, 1999] (a) Police Officer I to Senior Police Officer IV - Appointed by
the PNP regional director for regional personnel or by the Chief
JESULITO A. MANALO, petitioner, vs. PEDRO G. of the PNP for the national headquarters personnel and
SISTOZA, REGINO ARO III, NICASIO MA. CUSTODIO, attested by the Civil Service Commission;
GUILLERMO DOMONDON, RAYMUNDO L. LOGAN,
WILFREDO R. REOTUTAR, FELINO C. PACHECO, JR., (b) Inspector to Superintendent - Appointed by the Chief of the
RUBEN J. CRUZ, GERONIMO B. VALDERRAMA, PNP, as recommended by their immediate superiors, and
MERARDO G. ABAYA, EVERLINO B. NARTATEZ, attested by the Civil Service Commission;
ENRIQUE T. BULAN, PEDRO J. NAVARRO, DOMINADOR
M. MANGUBAT, RODOLFO M. GARCIA and HONORABLE
SALVADOR M. ENRIQUEZ II In His Capacity as Secretary (c) Senior Superintendent to Deputy Director General -
of Budget and Management, respondents. Appointed by the President upon recommendation of the Chief
of the PNP, with the proper endorsement by the Chairman of
the Civil Service Commission and subject to confirmation by
DECISION the Commission on Appointments; and
PURISIMA, J.:
(d) Director General - Appointed by the President from among
The case at bar is not of first impression. The issue the senior officers down to the rank of chief superintendent in
posed concerning the limits of the power of the Commission on the service, subject to confirmation by the Commission on
Appointments to confirm appointments issued by the Chief Appointments; Provided, That the Chief of the PNP shall serve
Executive has been put to rest in a number of cases. The court a tour of duty not to exceed four (4) years; Provided, further,
finds no basis for departing from the ruling laid down in those That, in times of war or other national emergency declared by
cases. Congres, the President may extend such tour of duty.
(underlining supplied).
In this special civil action for Prohibition under Rule 65 of
the Revised Rules of Court, petitioners question the In accordance therewith, on March 10, 1992, the
constitutionality and legality of the permanent appointments President of the Philippines, through then Executive Secretary
issued by former President Corazon C. Aquino to the Franklin M. Drilon, promoted the fifteen (15) respondent police
respondent senior officers of the Philippine National Police officers herein, by appointing them to positions in the Philippine
who were promoted to the ranks of Chief Superintendent and National Police with the rank of Chief Superintendent to
Director without their appointments submitted to the Director[2], namely:
Commission on Appointments for confirmation under Section
16, Article VII of the 1987 Constitution and Republic Act 6975
otherwise known as the Local Government Act of Chief Supt. PEDRO G. SISTOZA - Director
1990. Impleaded in the case is the former Secretary of Budget
and Management Salvador M. Enriquez III, who approved and Chief Supt. REGINO ARO III - Director
effected the disbursements for the salaries and other
emoluments of subject police officers.
Chief Supt. NICASIO MA. CUSTODIO - Director
The antecedents facts are as follows:
Chief Supt. GUILLERMO DOMONDON - Director
On December 13, 1990, Republic Act 6975 creating the
Department of Interior and Local Government was signed into
law by former President Corazon C. Aquino. Pertinent Chief Supt. RAYMUNDO L. LOGAN - Director
provisions of the said Act read:
Senior Supt. WILFREDO REOTUTAR - Chief Superintendent
Sec. 26. Powers, Functions and Term of Office of the PNP
Chief. - The command and direction of the PNP shall be vested Senior Supt. FELINO C. PACHECO, JR. - Chief
in the Chief of the PNP who shall have the power to direct and Superintendent
control tactical as well as strategic movements, deployment,
placement, utilization of the PNP or any of its units and
personal, including its equipment, facilities and other Senior Supt. RUBEN J. CRUZ - Chief Superintendent
resources. Such command and direction of the Chief of the
PNP may be delegated to subordinate officials with respect to Senior Supt. GERONIMO B. VALDERRAMA - Chief
the units under their respective commands, in accordance with Superintendent
the rules and regulations prescribed by the Commission. The
Chief of the PNP shal also have the power to issue detailed
implementing policies and instructions regarding personnel, Senior Supt. MERARDO G. ABAYA - Chief Superintendent
funds, properties, records, correspondence and such other
matters as may be necesary to effectively carry out the Senior Supt. EVERLINO NARTATEZ - Chief Superintendent
functions, powers and duties of the Bureau. The Chief of the
PNP shall be appointed by the President from among the
senior officers down to the rank of the chief superintendent, Senior Supt. ENRIQUE T. BULAN - Chief Superintendent
subject to confirmation by the Commission on
Appointments: Provided, That the Chief of the PNP shall serve Senior Supt. PEDRO J. NAVARRO - Chief Superintendent
a term of office not to exceed four (4) years: Provided, further,
That in times of war or other national emergency declared by
Senior Supt. DOMINADOR MANGUBAT - Chief
Congress, the President may extend such term of
Superintendent
office. [1] (underlining supplied).

Senior Supt. RODOLFO M. GARCIA - Chief Superintendent


Sec.31. Appointment of PNP Officers and Members. - The
appointment of the officers and members of the PNP shall be
effected in the following manner:
FINALS CONSTITUTION I ACJUCO 109

The appointments of respondent police officers were in a heads of the executive departments, ambassadors, other
permanent capacity. Their letters of appointment stated in part public ministers and consuls, or officers of the armed forces
: from the rank of colonel or naval captain, and other officers
whose appointments are vested in him in this Constitution. He
By virtue hereof, they may qualify and enter upon the shall also appoint all other officers of the Government whose
performance of the duties of the office, furnishing this office appointments are not otherwise provided for by law, and those
and the Civil Service Commission with copies of their oath of whom he may be authorized by law to appoint. The Congress
office.[3] may, by law, vest the appointment of other officers lower in
rank in the President alone, in the courts, or in the heads of
departments, agencies, commissions, or boards.
Without their names submitted to the Commission on
Appointments for confirmation, the said police officers took
their oath of office and assumed their respective The President shall have the power to make appointments
positions. Thereafter, the Department of Budget and during the recess of the Congress, whether voluntary or
Management, under the then Secretary Salvador M. Enriquez compulsory, but such appointments shall be effective only until
III, authorized disbursements for their salaries and other disapproval by the Commission on Appointments or until the
emoluments. next adjournment of the Congress.

On October 21, 1992, the petitioner brought before this The aforecited provision of the Constitution has been the
Court this present original petition for prohibition, as a taxpayer subject of several cases on the issue of the restrictive function
suit, to assail the legality of subject appointments and of the Commission on Appointments with respect to the
disbursements made therefor. appointing power of the President. This court touched upon the
historical antecedent of the said provision in the case of
Petitioner contends that:
Sarmiento III vs. Mison[5] in which it was ratiocinated upon that
Section 16 of Article VII of the 1987 Constitution requiring
I. Respondent officers, in assuming their offices and confirmation by the Commission on Appointments of certain
discharging the functions attached thereto, despite their invalid appointments issued by the President contemplates a system
appointments, in view of the failure to secure the required of checks and balances between the executive and legislative
confirmation of the Commission on Appointments as required branches of government. Experience showed that when
by the Constitution and the law, are acting without or in excess almost all presidential appointments required the consent of
of their jurisdiction or with grave abuse of discretion, the Commission on Appointments, as was the case under the
considering that : 1935 Constitution, the commission became a venue of horse-
trading and similar malpractices.[6] On the other hand, placing
A. Republic Act 6975 is a valid law that duly requires absolute power to make appointments in the President with
confirmation of the appointments of officers from the rank of hardly any check by the legislature, as what happened under
senior superintendent and higher by the Commission on 1973 Constitution, leads to abuse of such power. Thus was
Appointments; perceived the need to establish a middle ground between the
1935 and 1973 Constitutions. The framers of the 1987
Constitution deemed it imperative to subject certain high
B. The Philippine National Police is akin to the Armed Forces positions in the government to the power of confirmation of the
where the Constitution specifically requires confirmation by the Commission on Appointments and to allow other positions
Commission on Appointments. within the exclusive appointing power of the President.
Conformably, as consistently interpreted and ruled in the
II. Respondent Secretary in allowing and/or effecting
leading case of Sarmiento III vs. Mison[7], and in the
disbursements in favor of respondent officers despite the
subsequent cases of Bautista vs. Salonga[8], Quintos-Deles vs.
unconstitutionality and illegality of their appointments is acting
Constitutional Commission[9], and Calderon vs. Carale[10];
without or in excess of his jurisdiction or with grave abuse of
under Section 16, Article VII, of the Constitution, there are four
discretion.
groups of officers of the government to be appointed by the
President:
The petition must fail. It is not impressed with merit.
Petitioner theorizes that Republic Act 6975 enjoys the First, the heads of the executive departments, ambassadors,
presumption of constitutionality and that every statute passed other public ministers and consuls, officers of the armed forces
by Congress is presumed to have been carefully studied and from the rank of colonel or naval captain, and other officers
considered before its enactment. He maintains that the respect whose appointments are vested in him in this Constitution;
accorded to each department of the government requires that
the court should avoid, as much as possible, deciding Second, all other officers of the Government whose
constitutional questions. appointments are not otherwise provided for by law;
The Court agrees with petitioner. However, it is equally
demanded from the courts, as guardians of the Constitution, to Third, those whom the President may be authorized by law to
see to it that every law passed by Congress is not repugnant appoint;
to the organic law.Courts have the inherent authority to
determine whether a statute enacted by the legislature Fourth, officers lower in rank whose appointments the
transcends the limit delineated by the fundamental Congress may by law vest in the President alone.
law.[4] When it does, the courts will not hesitate to strike down
such unconstitutional law.
It is well-settled that only presidential appointments
The power to make appointments is vested in the Chief belonging to the first group require the confirmation by the
Executive by Section 16, Article VII of the Constitution, which Commission on Appointments. The appointments of
provides: respondent officers who are not within the first category, need
not be confirmed by the Commission on Appointments. As held
Section 16. The President shall nominate and, with the in the case of Tarrosa vs. Singson[11], Congress cannot by law
consent of the Commission on Appointments, appoint the expand the power of confirmation of the Commission on
FINALS CONSTITUTION I ACJUCO 110

Appointments and require confirmation of appointments of for the salaries and other emoluments of the respondent police
other government officials not mentioned in the first sentence officers whose appointments are valid.
of Section 16 of Article VII of the 1987 Constitution.
WHEREFORE, for lack of merit, the petition under
Consequently, unconstitutional are Sections 26 and 31 consideration is hereby DISMISSED. No pronouncement as to
of Republic Act 6975 which empower the Commission on costs.
Appointments to confirm the appointments of public officials
whose appointments are not required by the Constitution to be SO ORDERED.
confirmed. But the unconstitutionality of the aforesaid sections
notwithstanding, the rest of Republic Act 6975 stands. It is well-
settled that when provisions of law declared void are severable
from the main statute and the removal of the unconstitutional
provisions would not affect the validity and enforceability of the
other provisions, the statute remains valid without its voided
sections.[12]
It is petitioners submission that the Philippine National
Police is akin to the Armed Forces of the Philippines and
therefore, the appointments of police officers whose rank is
equal to that of colonel or naval captain require confirmation by
the Commission on Appointments.

This contention is equally untenable. The Philippine


National Police is separate and distinct from the Armed Forces
of the Philippines. The Constitution, no less, sets forth the
distinction. Under Section 4 of Article XVI of the 1987
Constitution,

The Armed Forces of the Philippines shall be composed of a


citizen armed force which shall undergo military training and
service, as may be provided by law. It shall keep a regular
force necessary for the security of the State.

On the other hand, Section 6 of the same Article of the


Constitution ordains that:

The State shall establish and maintain one police force, which
shall be national in scope and civilian in character to be
administered and controlled by a national police
commission. The authority of local executives over the police
units in their jurisdiction shall be provided by law.

To so distinguish the police force from the armed forces,


Congress enacted Republic Act 6975 which states in part:

Section 2. Declaration of policy - It is hereby declared to be the


policy of the State to promote peace and order, ensure public
safety and further strengthen local government capability
aimed towards the effective delivery of the basic services to
the citizenry through the establishment of a highly efficient and
competent police force that is national in scope and civilian in
character. xxx

The policy force shall be organized, trained and equipped


primarily for the performance of police functions. Its national
scope and civilian character shall be paramount. No element
of the police force shall be military nor shall any position thereof
be occupied by active members of the Armed Forces of the
Philippines.

Thereunder, the police force is different from and


independent of the armed forces and the ranks in the military
are not similar to those in the Philippine National Police. Thus,
directors and chief superintendents of the PNP, such as the
herein respondent police officers, do not fall under the first
category of presidential appointees requiring the confirmation
by the Commission on Appointments.
In view of the foregoing disquisition and conclusion, the
respondent former Secretary Salvador M. Enriquez III of the
Department of Budget and Management, did not act with grave
abuse of discretion in authorizing and effecting disbursements
FINALS CONSTITUTION I ACJUCO 111

[G.R. No. 153881. March 24, 2003] enforcement and not merely that he suffers thereby in some
indefinite way.[2]
ELPIDIO G. SORIANO III, petitioner, vs. REUBEN S. LISTA,
DOMINGO T. ESTERA, ELPIDIO B. PADAMA, MIGUEL C. The instant petition cannot even be classified as a
TABARES, ARTHUR N. GOSINGAN, EFREN L. TADURAN, taxpayers suit because petitioner has no interest as such and
CESAR A. SARILE, DANILO M. VILDA and HONORABLE this case does not involve the exercise by Congress of its
EMILIA T. BONCODIN, in her capacity as Secretary of taxing power.
Budget and Management, respondents.
Assuming arguendo that petitioner has the legal
personality to question the subject appointments, the petition
DECISION will nevertheless fail. As aptly pointed out by the Solicitor
General, the PCG used to be administered and maintained as
CORONA, J.: a separate unit of the Philippine Navy under Section 4 of RA
5173. It was subsequently placed under the direct supervision
Before us is a Petition for Prohibition under Rule 65 of and control of the Secretary of the Department of National
the Rules of Court questioning the constitutionality and legality Defense (DND) pursuant to Section 4 of PD 601. Eventually, it
of the permanent appointments, made by President Gloria was integrated into the Armed Forces of the Philippines (AFP)
Macapagal-Arroyo, of public respondents to different positions as a major subordinate unit of the Philippine Navy under
in the Philippine Coast Guard and their subsequent Section 54 of Chapter 8, Sub-title II, Title VIII, Book IV of EO
assumption of office without confirmation by the Commission 292, as amended.
on Appointments under the 1987 Constitution.
However, on March 30, 1998, after the aforesaid
The petition impleads Hon. Emilia T. Boncodin in her changes in the charter of the PCG, then President Fidel V.
capacity as Secretary of the Department of Budget and Ramos, in the exercise of his statutory authority to reorganize
Management (DBM). Petitioner, Elpidio G. Soriano, filed the the Office of the President, issued EO 475 transferring the
instant petition as member of the Integrated Bar of the PCG from the DND to the Office of the President. He later on
Philippines and as a taxpayer. again transferred the PCG from the Office of the President
to the Department of Transportation and
Public respondents were promoted to different ranks in Communications (DOTC).
the Philippine Coast Guard (PCG) on different dates as follows:
Now that the PCG is under the DOTC and no longer part
Reuben S. Lista Vice Admiral, Philippine Coast Guard of the Philippine Navy or the Armed Forces of the Philippines,
the promotions and appointments of respondent officers of the
Domingo T. Estera Rear Admiral, Philippine Coast Guard PCG, or any PCG officer from the rank of captain and higher
Miguel C. Tabares Commodore, Philippine Coast Guard for that matter, do not require confirmation by the CA.

Arthur N. Gosingan Commodore, Philippine Coast Guard Section 16, Article VII of the 1987 Constitution provides:

Efren L. Taduran Naval Captain, Philippine Coast Guard Section 16. The President shall nominate and, with the
consent of the Commission on Appointments, appoint the
Cesar A. Sarile Naval Captain, Philippine Coast Guard
heads of the executive departments, ambassadors, other
Danilo M. Vilda Naval Captain, Philippine Coast Guard public ministers and consuls, or officers of the armed forces
from the rank of colonel or naval captain, and other officers
Elpidio B. Padama Commodore, Philippine Coast Guard whose appointments are vested in him in this Constitution. He
shall also appoint all other officers of the Government whose
Petitioner bewails the fact that despite the non- appointments are not otherwise provided for by law, and those
submission of their names to the Commission on whom he may be authorized by law to appoint. The Congress
Appointments (CA) for confirmation, all of the said respondent may, by law, vest the appointment of other officers lower in
officers of the PCG had assumed their duties and rank in the President alone, in the courts, or in the heads of
functions. According to petitioner, their respective departments, agencies, commissions, or boards.
appointments are illegal and unconstitutional for failure to
undergo the confirmation process in the CA. Thus, they should
be prohibited from discharging their duties and functions as The President shall have the power to make appointments
such officers of the PCG. during the recess of the Congress, whether voluntary or
compulsory, but such appointments shall be effective only until
In the same vein, petitioner opines that there is no legal disapproval by the Commission on Appointments or until the
basis for the DBM to allow the disbursement of the salaries and next adjournment of the Congress.
emoluments of respondent officers of the PCG. Accordingly,
he prays that respondent Secretary Boncodin be ordered to
It is clear from the foregoing provision of the Constitution
desist from allowing such disbursements until the confirmation
that only appointed officers from the rank of colonel or naval
of their respective appointments by the CA.
captain in the armed forces require confirmation by the CA.
At the outset, the Court finds petitioner to be without any The rule is that the plain, clear and unambiguous language of
legal personality to file the instant petition. We have ruled that the Constitution should be construed as such and should not
a private citizen is allowed to raise constitutional questions only be given a construction that changes its meaning.[3]
if he can show that he has personally suffered some actual or
The enumeration of appointments subject to confirmation
threatened injury as a result of the allegedly illegal conduct of
by the CA under Section 16, Article VII of the 1987 Constitution
the government, the injury is fairly traceable to the challenged
is exclusive. The clause officers of the armed forces from the
action and the injury is likely to be redressed by a favorable
rank of colonel or naval captain refers to military
action.[1] In the case at bar, petitioner has failed to clearly
officers alone. This is clear from the deliberations of the
demonstrate that he has personally suffered actual or
Constitutional Commission on the proposed text of said
threatened injury. It should be emphasized that a party
Section 16, Article VII of the Constitution. Since the promotions
bringing a suit challenging the constitutionality of an act or
and appointments of respondent officers are not covered by
statute must show not only that the law or act is invalid, but
the above-cited provision of the Constitution, the same need
also that he has sustained or is in immediate, or imminent
not be confirmed by the CA.[4]
danger of sustaining some direct injury as a result of its
FINALS CONSTITUTION I ACJUCO 112

Accordingly, the Court declares that no grave abuse of


discretion amounting to lack or excess of jurisdiction was
committed by respondent officers of the PCG. Their
assumption to office as well as the disbursement of their
respective salaries and other emoluments by the respondent
Secretary of the DBM are hereby declared valid and legal.
WHEREFORE, the petition is hereby DISMISSED.
SO ORDERED.
FINALS CONSTITUTION I ACJUCO 113

AQUILINO Q. PIMENTEL, G.R. No. 164978 Avelino J. National Defense 23 August 2004
JR., Cruz, Jr.
EDGARDO J. ANGARA, Present: Rene C. Villa Agrarian Reform 23 August 2004
JUAN PONCE ENRILE, Davide, Jr., C.J., Joseph H. Tourism 23 August 2004
LUISA P. EJERCITO- Puno, Durano
ESTRADA, Panganiban, Michael T. Environment and 23 August 2004
JINGGOY E. ESTRADA, Quisumbing, Defensor Natural Resources
PANFILO M. LACSON, Ynares-Santiago,
ALFREDO S. LIM, Sandoval-Gutierrez,
The appointment papers are uniformly worded as follows:
JAMBY A.S. MADRIGAL, Carpio,
and Austria-Martinez,
SERGIO R. OSMEA III, Corona,
Sir:
Petitioners, Carpio Morales,
Pursuant to the provisions of
- versus -
existing laws, you are hereby appointed
EXEC. SECRETARY Callejo, Sr.,
ACTING SECRETARY, DEPARTMENT
EDUARDO
OF (appropriate department) vice (name of
R. ERMITA, FLORENCIO Azcuna,
person replaced).
B. ABAD,
AVELINO J. CRUZ, JR., Tinga,
By virtue hereof, you may qualify
MICHAEL T. DEFENSOR, Chico-Nazario, and
and enter upon the performance of the
JOSEPH H. DURANO, Garcia, JJ.
duties and functions of the office, furnishing
RAUL M. GONZALEZ,
this Office and the Civil Service
ALBERTO G. ROMULO,
Commission with copies of your Oath of
RENE C. VILLA, and Promulgated:
Office. (signed) Gloria Arroyo
ARTHUR C. YAP,
Respondents took their oath of office and assumed
Respondents. October 13, 2005
duties as acting secretaries.
x---------------------------------------------------
--x
On 8 September 2004, Aquilino Q. Pimentel, Jr. (Senator
Pimentel), Edgardo J. Angara (Senator Angara), Juan Ponce
DECISION
Enrile (Senator Enrile), Luisa P. Ejercito-Estrada (Senator
Ejercito-Estrada), Jinggoy E. Estrada (Senator Estrada),
CARPIO, J.:
Panfilo M. Lacson (Senator Lacson), Alfredo S. Lim (Senator
Lim), Jamby A.S. Madrigal (Senator Madrigal), and Sergio R.
Osmea, III (Senator Osmea) (petitioners) filed the present
petition as Senators of the Republic of the Philippines.
The Case

Congress adjourned on 22 September 2004. On 23


This is a petition for certiorari and prohibition[1] with a
September 2004, President Arroyo issued ad
prayer for the issuance of a writ of preliminary injunction to
interim appointments[3] to respondents as secretaries of the
declare unconstitutional the appointments issued by President
departments to which they were previously appointed in an
Gloria Macapagal-Arroyo (President Arroyo) through
acting capacity. The appointment papers are uniformly worded
Executive Secretary Eduardo R. Ermita (Secretary Ermita) to
as follows:
Florencio B. Abad, Avelino J. Cruz, Jr., Michael T. Defensor,
Joseph H. Durano, Raul M. Gonzalez, Alberto G. Romulo,
Sir:
Rene C. Villa, and Arthur C. Yap (respondents) as acting
secretaries of their respective departments. The petition also
Pursuant to the provisions of
seeks to prohibit respondents from performing the duties of
existing laws, you are hereby appointed
department secretaries.
SECRETARY [AD INTERIM],
DEPARTMENT OF (appropriate
department).
Antecedent Facts
By virtue hereof, you may qualify
and enter upon the performance of the
The Senate and the House of Representatives
duties and functions of the office, furnishing
(Congress) commenced their regular session on 26 July 2004.
this Office and the Civil Service
The Commission on Appointments, composed of Senators and
Commission with copies of your oath of
Representatives, was constituted on 25 August 2004.
office.
(signed)Gloria Arroyo
Meanwhile, President Arroyo issued
appointments[2] to respondents as acting secretaries of their
Issue
respective departments.

Appointee Department Date of The petition questions the constitutionality of


Appointment President Arroyos appointment of respondents as acting
Arthur C. Yap Agriculture 15 August 2004 secretaries without the consent of the Commission on
Alberto G. Foreign Affairs 23 August 2004 Appointments while Congress is in session.
Romulo
Raul M. Justice 23 August 2004
Gonzalez The Courts Ruling
Florencio B. Education 23 August 2004
Abad
FINALS CONSTITUTION I ACJUCO 114

Appointments as a body (rather than individual members of the


The petition has no merit. Congress) may possess standing in this case. [10]

Preliminary Matters Petitioners, on the other hand, state that the Court can exercise
its certiorari jurisdiction over unconstitutional acts of the
President.[11] Petitioners further contend that they possess
On the Mootness of the Petition standing because President Arroyos appointment of
department secretaries in an acting capacity while Congress is
in session impairs the powers of Congress. Petitioners
The Solicitor General argues that the petition is moot because cite Sanlakas v. Executive Secretary[12] as basis, thus:
President Arroyo had extended to respondents ad
interim appointments on 23 September 2004 immediately after To the extent that the powers of
the recess of Congress. Congress are impaired, so is the power of
each member thereof, since his office
As a rule, the writ of prohibition will not lie to enjoin confers a right to participate in the exercise
acts already done.[4] However, as an exception to the rule on of the powers of that institution.
mootness, courts will decide a question otherwise moot if it is
capable of repetition yet evading review. [5] An act of the Executive which
In the present case, the mootness of the petition does not bar injures the institution of Congress causes a
its resolution. The question of the constitutionality of the derivative but nonetheless substantial
Presidents appointment of department secretaries in an acting injury, which can be questioned by a
capacity while Congress is in session will arise in every such member of Congress. In such a case, any
appointment. member of Congress can have a resort to
the courts.

On the Nature of the Power to Appoint

Considering the independence of the Commission on


The power to appoint is essentially executive in nature, and the Appointments from Congress, it is error for petitioners to claim
legislature may not interfere with the exercise of this executive standing in the present case as members of Congress.
power except in those instances when the Constitution President Arroyos issuance of acting appointments while
expressly allows it to interfere.[6] Limitations on the executive Congress is in session impairs no power of Congress. Among
power to appoint are construed strictly against the the petitioners, only the following are members of the
legislature.[7] The scope of the legislatures interference in the Commission on Appointments of the 13th Congress: Senator
executives power to appoint is limited to the power to prescribe Enrile as Minority Floor Leader, Senator Lacson as Assistant
the qualifications to an appointive office. Congress cannot Minority Floor Leader, and Senator Angara, Senator Ejercito-
appoint a person to an office in the guise of prescribing Estrada, and Senator Osmea as members.
qualifications to that office. Neither may Congress impose on
the President the duty to appoint any particular person to an Thus, on the impairment of the prerogatives of
office.[8] members of the Commission on Appointments, only Senators
However, even if the Commission on Appointments is Enrile, Lacson, Angara, Ejercito-Estrada, and Osmea have
composed of members of Congress, the exercise of its powers standing in the present petition. This is in contrast to Senators
is executive and not legislative. The Commission on Pimentel, Estrada, Lim, and Madrigal, who, though vigilant in
Appointments does not legislate when it exercises its power to protecting their perceived prerogatives as members of
give or withhold consent to presidential appointments. Thus: Congress, possess no standing in the present petition.

xxx The Commission on Appointments is a


creature of the Constitution. Although its The Constitutionality of President Arroyos Issuance
membership is confined to members of of Appointments to Respondents as Acting Secretaries
Congress, said Commission is
independent of Congress. The powers of
the Commission do not come from Petitioners contend that President Arroyo should not
Congress, but emanate directly from the have appointed respondents as acting secretaries because in
Constitution. Hence, it is not an agent of case of a vacancy in the Office of a Secretary, it is only an
Congress. In fact, the functions of the Undersecretary who can be designated as Acting
Commissioner are purely executive in Secretary.[13] Petitioners base their argument on Section 10,
nature. xxx[9] Chapter 2, Book IV of Executive Order No. 292 (EO
292),[14]which enumerates the powers and duties of the
undersecretary. Paragraph 5 of Section 10 reads:

SEC. 10. Powers and Duties of


the Undersecretary. - The Undersecretary
On Petitioners Standing shall:

xxx
The Solicitor General states that the present petition is a quo
warranto proceeding because, with the exception of Secretary (5) Temporarily discharge the
Ermita, petitioners effectively seek to oust respondents for duties of the Secretary in the latters
unlawfully exercising the powers of department secretaries. absence or inability to discharge his duties
The Solicitor General further states that petitioners may not for any cause or in case of vacancy of the
claim standing as Senators because no power of the said office, unless otherwise provided by
Commission on Appointments has been infringed upon or law. Where there are more than one
violated by the President. xxx If at all, the Commission on Undersecretary, the Secretary shall
allocate the foregoing powers and duties
FINALS CONSTITUTION I ACJUCO 115

among them. The President shall likewise position, unless he is already in the
make the temporary designation of Acting government service in which case he shall
Secretary from among them; and receive only such additional compensation
as, with his existing salary, shall not exceed
xxx the salary authorized by law for the position
filled. The compensation hereby authorized
shall be paid out of the funds appropriated
Petitioners further assert that while Congress is in session, for the office or agency concerned.
there can be no appointments, whether regular or acting, to a (3) In no case shall a temporary
vacant position of an office needing confirmation by the designation exceed one (1)
Commission on Appointments, without first having obtained its year. (Emphasis supplied)
consent.[15]

In sharp contrast, respondents maintain that the President can


issue appointments in an acting capacity to department Petitioners and respondents maintain two diametrically
secretaries without the consent of the Commission on opposed lines of thought. Petitioners assert that the President
Appointments even while Congress is in session. Respondents cannot issue appointments in an acting capacity to department
point to Section 16, Article VII of the 1987 Constitution. Section secretaries while Congress is in session because the law does
16 reads: not give the President such power. In contrast, respondents
insist that the President can issue such appointments because
SEC. 16. The President shall no law prohibits such appointments.
nominate and, with the consent of the
Commission on Appointments, appoint the The essence of an appointment in an acting capacity is its
heads of the executive departments, temporary nature. It is a stop-gap measure intended to fill an
ambassadors, other public ministers and office for a limited time until the appointment of a permanent
consuls, or officers of the armed forces occupant to the office.[16] In case of vacancy in an office
from the rank of colonel or naval captain, occupied by an alter ego of the President, such as the office of
and other officers whose appointments are a department secretary, the President must necessarily
vested in him in this Constitution. He shall appoint an alter ego of her choice as acting secretary before
also appoint all other officers of the the permanent appointee of her choice could assume office.
Government whose appointments are not
otherwise provided for by law, and those Congress, through a law, cannot impose on the
whom he may be authorized by law to President the obligation to appoint automatically the
appoint. The Congress may, by law, vest undersecretary as her temporary alter ego. An alter ego,
the appointment of other officers lower in whether temporary or permanent, holds a position of great trust
rank in the President alone, in the courts, or and confidence. Congress, in the guise of prescribing
in the heads of departments, agencies, qualifications to an office, cannot impose on the President who
commissions, or boards. her alter ego should be.

The President shall have the The office of a department secretary may become
power to make appointments during the vacant while Congress is in session. Since a department
recess of the Congress, whether voluntary secretary is the alter ego of the President, the acting appointee
or compulsory, but such appointments shall to the office must necessarily have the Presidents confidence.
be effective only until disapproval by the Thus, by the very nature of the office of a department
Commission on Appointments or until the secretary, the President must appoint in an acting capacity a
next adjournment of the Congress. person of her choice even while Congress is in session. That
person may or may not be the permanent appointee, but
practical reasons may make it expedient that the acting
appointee will also be the permanent appointee.
Respondents also rely on EO 292, which devotes a
chapter to the Presidents power of appointment. Sections 16 The law expressly allows the President to make such
and 17, Chapter 5, Title I, Book III of EO 292 read: acting appointment. Section 17, Chapter 5, Title I, Book III of
EO 292 states that [t]he President may temporarily designate
SEC. 16. Power of an officer already in the government service or any other
Appointment. The President shall competent person to perform the functions of an office in the
exercise the power to appoint such executive branch. Thus, the President may even appoint in an
officials as provided for in the acting capacity a person not yet in the government service, as
Constitution and laws. long as the President deems that person competent.

SEC. 17. Power to Issue Petitioners assert that Section 17 does not apply to
Temporary Designation. (1) The President appointments vested in the President by the Constitution,
may temporarily designate an officer because it only applies to appointments vested in the
already in the government service or President by law. Petitioners forget that Congress is not the
any other competent person to perform only source of law. Law refers to the Constitution, statutes or
the functions of an office in the acts of Congress, municipal ordinances, implementing rules
executive branch, appointment to which issued pursuant to law, and judicial decisions. [17]
is vested in him by law, when: (a) the
officer regularly appointed to the office Finally, petitioners claim that the issuance of appointments in
is unable to perform his duties by an acting capacity is susceptible to abuse. Petitioners fail to
reason of illness, absence or any other consider that acting appointments cannot exceed one year as
cause; or (b) there exists a vacancy[.] expressly provided in Section 17(3), Chapter 5, Title I, Book III
of EO 292. The law has incorporated this safeguard to prevent
(2) The person designated shall abuses, like the use of acting appointments as a way to
receive the compensation attached to the circumvent confirmation by the Commission on Appointments.
FINALS CONSTITUTION I ACJUCO 116

In distinguishing ad interim appointments from


appointments in an acting capacity, a noted textbook writer on
constitutional law has observed:

Ad-interim appointments must be


distinguished from appointments in an
acting capacity. Both of them are effective
upon acceptance. But ad-interim
appointments are extended only during a
recess of Congress, whereas acting
appointments may be extended any time
there is a vacancy. Moreover ad-interim
appointments are submitted to the
Commission on Appointments for
confirmation or rejection; acting
appointments are not submitted to the
Commission on Appointments. Acting
appointments are a way of temporarily
filling important offices but, if abused, they
can also be a way of circumventing the
need for confirmation by the Commission
on Appointments.[18]

However, we find no abuse in the present case. The absence


of abuse is readily apparent from President Arroyos issuance
of ad interim appointments to respondents immediatelyupon
the recess of Congress, way before the lapse of one year.

WHEREFORE, we DISMISS the present petition


for certiorari and prohibition.

SO ORDERED.
FINALS CONSTITUTION I ACJUCO 117

[A.M. No. 98-5-01-SC. November 9, 1998] Also pertinent although not specifically discussed is Section 9
In Re Appointments dated March 30, 1998 of Hon. Mateo of the same Article VIII which provides that for the lower courts,
A. Valenzuela and Hon. Placido B. Vallarta as Judges of the President shall issue the appointments - from a list of at
the Regional Trial Court of Branch 62, Bago City and of least three nominees prepared by the Council for every
Branch 24, Cabanatuan City, respectively. vacancy - within ninety days from the submission of the list.

DECISION The view was then expressed by Senior associate Justice


Florenz D. Regalado, Consultant of the Council, who had been
NARVASA, CJ.: a member of the Committee of the Executive Department and
of the Committee on the Judicial Department of the 1986
The question presented for resolution in the Constitutional Commission, that on the basis of the
administrative matter at bar is whether, during the period of the Commission's records, the election ban had no application to
ban on appointments imposed by Section 15, Article VII of the appointments to the Court of Appeals. Without any extended
Constitution, the President is nonetheless required to fill discussion or any prior research and study on the part of the
vacancies in the judiciary, in view of Sections 4(1) and 9 of other Members of the JBC, this hypothesis was accepted, and
Article VIII. A corollary question is whether he can make was then submitted to the President for consideration, together
appointments to the judiciary during the period of the ban in the with the Council's nominations for eight (8) vacancies in the
interest of public service. Court of Appeals.

Resolution of the issues is needful; it will preclude a


On April 6, 1998 the Chief Justice received an official
recurrence of any conflict in the matter of nominations and
communication from the Executive Secretary transmitting the
appointments to the Judiciary - as that here involved - between
appointments of eight (8) Associate Justices of the Court of
the Chief Executive, on the one hand, and on the other, the
Appeals all of which had been duly signed on March 11, 1998
Supreme Court and the Judicial and Bar Council over which
by His Excellency, the President. In view of the fact that all the
the Court exercises general supervision and wields specific
appointments had been signed on March 11, 1998 - the day
powers including the assignment to it of other functions and
immediately before the commencement of the ban on
duties in addition to its principal one of recommending
appointments imposed by Section 15, Article VII of the
appointees to the Judiciary, and the determination of its
Constitution - which impliedly but no less clearly indicated that
Members' emoluments.[1]
the President's Office did not agree with the hypothesis that
I The Relevant Facts appointments to the Judiciary were not covered by said ban,
the Chief Justice resolved to defer consideration of
The Resolution of the Court En Banc, handed down on nominations for the vacancy in the Supreme Court created by
May 14, 1998, sets out the relevant facts and is for that reason the retirement of Associate Justice Ricardo J. Francisco,
hereunder reproduce in full. specially considering that the Court had scheduled sessions in
Baguio City in April, 1998, that the legislature's representatives
to the JBC were occupied with the forthcoming elections, and
Referred to the Court En Banc by the Chief Justice are the
that a member of the Council was going on a trip out of the
appointments signed by His Excellency the President under
country.
the date of March 30, 1998 of Hon. Mateo A. Valenzuela and
Hon. Placido B. Vallarta as Judges of the Regional Trial Court
of Branch 62, Bago City and of Branch 24, Cabanatuan City, On May 4, 1998, the Chief Justice received a letter from the
respectively. The appointments were received at the Chief President, addressed to the JBC requesting transmission of
Justice's chambers on May 12, 1998. The referral was made the "list of final nominees" for the vacancy "no later than
in view of the serious constitutional issue concerning said Wednesday, May 6, 1998," in view of the duty imposed on him
appointments arising from the pertinent antecedents. by the Constitution "to fill up the vacancy *** within ninety (90)
days from February 13, 1998, the date the present vacancy
occurred."
The issue was first ventilated at the meeting of the Judicial and
Bar Council on March 9, 1998. The meeting had been called,
according to the Chief Justice as Ex Officio Chairman, to On May 5, 1998, Secretary of Justice Silvestre Bello III
discuss the question raised by some sectors about the requested the Chief Justice for "guidance" respecting the
"constitutionality of *** appointments" to the Court of Appeals, expressed desire of the "regular members" of the JBC to hold
specifically, in light of the forthcoming presidential a meeting immediately to fill up the vacancy in the Court in line
elections. Attention was drawn to Section 15, Article VII of the with the President's letter of May 4. The Chief Justice advised
Constitution reading as follows: Secretary Bello to await the reply that he was drafting to the
President's communication, a copy of which he would give to
the Secretary the following day.
"SEC 15. Two months immediately before the next presidential
elections and up to the end of his term, a President or Acting
President shall not make appointments, except temporary On May 6, 1998 the Chief Justice sent his reply to the
appointments to executive positions when continued President. He began by stating that no sessions had been
vacancies therein will prejudice public service or endanger scheduled for the Council after the May elections for the reason
public safety." that apparently the President's Office did not share the view
posited by the JBC that Section 15, Article VII of the
Constitution had no application to JBC-recommended
On the other hand, appointments to fill vacancies in the
appointments - the appointments to the Court of Appeals
Supreme court during the period mentioned in the provision
having been all uniformly dated March 11, 1998, before the
just quoted could seemingly be justified by another provision of
commencement of the prohibition in said provision - thus giving
the same Constitution.Section 4(1) of Article VIII which states:
rise to the "need to undertake further study of the matter,"
prescinding from "the desire to avoid any constitutional issue
"SEC 4 (1) The Supreme Court shall be composed of a Chief regarding the appointment to the mentioned vacancy" and the
Justice and fourteen Associate Justices. ***. Any vacancy shall further fact that "certain senior members of the Court of
be filled within ninety days from the occurrence thereof." Appeals *** (had) asked the Council to reopen the question of
their exclusion on account of age from such (final) list." He
closed with the assurance that the JBC expected to deliberate
FINALS CONSTITUTION I ACJUCO 118

on the nominations "forthwith upon the completion of the The Chief Justice replied to the letter the following day, May 8,
coming elections." The letter was delivered to Malacaang at 1998. Since the Chief Justice's letter explains the issue quite
about 5 o'clock in the afternoon of May 6, 1998, and a copy plainly, it is here quoted in full.
given to the Office of Justice Secretary Bello shortly before that
hour. "Thank you for your letter of May 7, 1998, responding to my
own communication of May 6, 1998 which, I would like to say,
It would appear, however, that the Justice Secretary and the reflects the collective sentiments of my colleagues in the
regular members of the Council had already taken action Supreme Court.Knowing how busy you are, I will deal
without awaiting the Chief Justice's promised response to the straightaway with the points set out in your letter.
President's letter of May 4, 1998. On that day, May 6, 1998,
they met at some undisclosed place, deliberated, and came to The dating of the latest appointments to the Court of Appeals
an agreement on a resolution which they caused to be reduced was adverted to merely to explain how we in the Court and the
to writing and thereafter signed. In that two-page Resolution JBC came to have the impression that you did not share the
they drew attention to Section 4 (1), Article VIII of the view expressed in the JBC minutes of March 9, 1998 'that there
Constitution (omitting any mention of Section 15, Article VII) as is no election ban with regard to the JBC appointments.' Be this
well as to the President's letter of May 4 in which he as it may, the Court feels that there is a serious question
"emphatically requested that the required list of final nominee
concerning the matter in light of the seemingly inconsistent
be submitted to him;" and pointing out that the "Council would provisions of the Constitution. The first of these is Section 15,
be remiss in its duties" should it fail to submit the nominations,
Article VII, which reads:
closed with an appeal that the Chief Justice convene the
Council for the purpose "on May 7, 1998, at 2:00 o'clock in the
afternoon." This Resolution they transmitted to the Chief 'SEC. 15. Two months immediately before the next presidential
Justice together with their letter, also dated May 6, in which elections and up to the end of his term, a President or Acting
they emphasized that "we are pressed for time" again drawing President shall not make appointments, except temporary
attention to Section 4 (1). In Article VIII of the Constitution (and appointments to executive positions when continued
again omitting any reference to Section 15, Article VII). They vacancies therein will prejudice public service or endanger
ended their letter with the following intriguing paragraph: public safety.'

"Should the Chief Justice be not disposed to call for the The second is Section 4(1) of Article VIII which states:
meeting aforesaid, the undersigned members constituting the
majority will be constrained to convene the Council for the 'SEC 4(1) The Supreme Court shall be composed of a Chief
purpose of complying with its Constitutional mandate." Justice and fourteen Associate Justices. ***. Any vacancy shall
be filled within ninety days from the occurrence thereof.'
It seems evident, as just intimated, that the resolution and the
covering letter were deliberated on, prepared and signed hours As you can see, Your Excellency, Section 15 of Article VII
before delivery of the Chief Justice's letter to the President and imposes a direct prohibition on the President: he "shall not
the Justice Secretary. make appointments" within the period mentioned, and since
there is no specification of which appointments are proscribed,
Since the Members of the Council appeared determined to the same may be considered as applying to all appointments
hold a meeting regardless of the Chief Justice's wishes, the of any kind and nature. This is the general rule then, the only
latter convoked the Council to a meeting at 3 o'clock in the exception being only as regards "executive positions" as to
afternoon of May 7, 1998.Present at the meeting were Chief which "temporary appointments" may be made within the
Justice, Secretary Bello, ex officio member and the regular interdicted period "when continued vacancies therein will
members of the Council; Justice Regino Hermosisima, Atty. prejudice public service or endanger public safety." As the
Teresita Cruz Sison, Judge Cesar C. Peralejo. Also present on exception makes reference only to "executive" positions, it
the invitation of the Chief Justice, were Justices Hilario G. would seem that "judicial" positions are covered by the general
Davide, Jr., Flerida Ruth P. Romero, Josue N. Bellosillo, rule.
Reynato S. Puno, Jose C. Vitug, Vicente V. Mendoza, Artemio
V. Panganiban, Antonio M. Martinez, Leonardo A. Quisumbing On the other hand, Section 4 (1) of Article VIII, requires that
and Fidel P. Purisima. The Chief Justice reviewed the events any vacancy in the Supreme Court "shall be filled within ninety
leading to the session, and after discussion, the body agreed days from the occurrence thereof." Unlike Section 15, Article
to give the President time to answer the Chief Justice's letter VII, the duty of filling the vacancy is not specifically imposed on
of May 6, 1998. the President; hence, it may be inferred that it is a duty shared
by the Judicial and Bar council and the President.
On May 7, 1998, the Chief Justice received a letter from His
Excellency the President in reply to his letter of May 6 (which Now, in view of the general prohibition in the first-quoted
the President said had been "received early this provision, how is the requirement of filling vacancies in the
morning"). The President expressed the view that "the Court within ninety days to be construed? One interpretation
election-ban provision (Article VII, Sec. 15) *** applies only that immediately suggests itself is that Section 4(1), Article VIII
to executive appointments or appointments in the executive is a general provision while Section 15, Article VII is a particular
branch of government," the whole article being "entitled one; that is to say, normally, when there are no presidential
'EXECUTIVE DEPARTMENT.'" He also observed that further elections - which after all occur only every six years - Section
proof of his theory "is the fact that appointments to the judiciary 4(1), Article VIII shall apply: vacancies in the Supreme Court
have special, specific provisions applicable to them" (citing shall be filled within 90 days; but when (as now) there are
Article VIII, Sec. 4 [1] and Article VIII, Section 9. In view presidential elections, the prohibition in Section 15, Article VII
thereof, he "firmly and respectfully reiterate(d) *** (his) request comes into play: the President shall not make any
for the Judicial and Bar Council to transmit *** the final list of appointments. The reason for said prohibition, according to Fr.
nominees for the lone Supreme Court vacancy." J. Bernas, S.J., an authority on Constitutional Law and himself
a member of the Constitutional Commission, is "(I)n order not
to tie the hands of the incoming President through midnight
FINALS CONSTITUTION I ACJUCO 119

appointments." Another interpretation is that put forth in the justiciable issue before the Court, an issue of sufficient
Minutes of the JBC Meeting of March 9, 1998. importance to warrant consideration and adjudication on the
merits.
I must emphasize that the validity of any appointment to the
Supreme Court at this time hinges on the correct interpretation Accordingly, the Court Resolved to (1) CONSIDER the case at
of the foregoing sections of the Constitution. On account of the bar an administrative matter and cause it to be appropriately
importance of the question, I consulted the Court about it but, docketed; (2) to DIRECT the Clerk of Court to immediately
as I stated in my letter of May 6, 1998, "it declined to take any serve copies of this Resolution on (a) the Office of the
position, since obviously there had not been enough time to President, (b) the Office of the Solicitor General, (c) Hon.
deliberate on the same *** (although it) did agree that further Mateo A. Valenzuela, and (d) Hon. Placido B. Vallarta (at their
study was necessary ***." addresses recorded in the Judicial and Bar Council); and (3) to
REQUIRE the Office of the President, the Office of the Solicitor
Since the question has actually come up, and its importance General, Hon. Mateo A. Valenzuela, and Hon. Placido B.
cannot be gainsaid, and it is the Court that is empowered under Vallarta to file their comments on this Resolution within fifteen
the Constitution to make an authoritative interpretation of its (15) days from notice thereof.
(provisions) or of those of any other law. I believe that the Court
may now perhaps consider the issue ripe for determination and The Court further Resolved that (1) pending the foregoing
come to grips with it, to avoid any possible polemics proceedings and the deliberation by the Court on the matter,
concerning the matter. However the Court resolves the issue, and until further orders, no action be taken on the
no serious prejudice will be done. Should the Court rule that appointments of Hon. Valenzuela and Hon. Vallarta which in
the President is indeed prohibited to make appointments in a the meantime shall be held in abeyance and not given any
presidential election year, then any appointment attempted effect and said appointees shall refrain from taking their oath
within the proscribed period would be void anyway. If the Court of office; and that (2) exercising its power of supervision over
should adjudge that the ban has no application to the Judicial and Bar Council, said Council and its ex officio and
appointments to the Supreme Court, the JBC may submit regular Members herein mentioned be INSTRUCTED, as they
nominations and the President may make the appointment are hereby INSTRUCTED, to defer all action on the matter of
forthwith upon such adjudgment. nominations to fill up the lone vacancy in the Supreme Court
or any other vacancy until further orders.
The matter is a delicate one, quite obviously, and must thus be
dealt with with utmost circumspection, to avoid any question SO ORDERED.
regarding the validity of an appointment to the Court at this
time, or any accusation of "midnight" appointments or rash,
hasty action on the part of the JBC or the President.

In view thereof, and upon the advice and consent of the


Members of the Court, I am requesting the regular Members of
the Judicial and Bar Council to defer action on the matter until
further advice by the Court. I earnestly make the same request
of you, Your Excellency, I assure you, however, that as befits
a matter in which the Chief Executive has evinced much
interest, my colleagues and I will give it preferential and
expeditious attention and consideration. To this end, I intend to
convene the Court by next week, at the latest."

On May 8, 1998, again on the insistence of the regular


Members of the JBC, another meeting was held at which were
present the Chief Justice, the Secretary of Justice and the
three regular Members above mentioned, as well as Justices
Hilario G. Davide, Jr., Flerida Ruth P. Romero, Josue N.
Bellosillo, Reynato S. Puno, Jose C. Vitug, Santiago M.
Kapunan, Vicente V. Mendoza, Artemio V. Panganiban,
Antonio M. Martinez, Leonardo A. Quisumbing and Fidel P.
Purisima. The meeting closed with a resolution that "the
constitutional provisions *** (in question) be referred to the
Supreme Court En Banc for appropriate action, together with
the request that the Supreme Court consider that the ninety-
day period stated in Section 4 (1), Article VIII be suspended or
interrupted in view of the peculiar circumstances ***."

On May 12, 1998, the Chief Justice received from Malacaang


the appointments of two (2) Judges of the Regional Trial Court
mentioned above. This places on the Chief Justice the
obligation of acting thereon; i.e., transmitting the appointments
to the appointees so that they might take their oaths and
assume their duties of their office. The trouble is that in doing
so, the Chief Justice runs the risk of acting in a manner
inconsistent with the Constitution, for these appointments
appear prima facie, at least, to be expressly prohibited by
Section 15, Article VII of the charter. This circumstance, and
the referral of the constitutional question to the Court in virtue
of the Resolution of May 8, 1998, supra, operate to raise a
FINALS CONSTITUTION I ACJUCO 120

GR 191002 importance to the Nation, because the appointment of the


De casto vs. JBC Chief Justice is any Presidents most important appointment.
x-------------------------------------------------------------------------------x A precedent frequently cited is In Re Appointments
Dated March 30, 1998 of Hon. Mateo A. Valenzuela and Hon.
DECISION Placido B. Vallarta as Judges of the Regional Trial Court of
Branch 62, Bago City and of Branch 24, Cabanatuan City,
BERSAMIN, J.: respectively (Valenzuela),[7] by which the Court held that
Section 15, Article VII prohibited the exercise by the President
The compulsory retirement of Chief Justice Reynato S. Puno of the power to appoint to judicial positions during the period
by May 17, 2010 occurs just days after the coming presidential therein fixed.
elections on May 10, 2010. Even before the event actually
happens, it is giving rise to many legal dilemmas. May the In G.R. No. 191002, De Castro submits that the
incumbent President appoint his successor, considering that conflicting opinions on the issue expressed by legal
Section 15, Article VII (Executive Department) of the luminaries one side holds that the incumbent President is
Constitution prohibits the President or Acting President from prohibited from making appointments within two months
making appointments within two months immediately before immediately before the coming presidential elections and until
the next presidential elections and up to the end of his the end of her term of office as President on June 30, 2010,
term, except temporary appointments to executive positions while the other insists that the prohibition applies only to
when continued vacancies therein will prejudice public service appointments to executive positions that may influence the
or endanger public safety? What is the relevance of Section 4 election and, anyway, paramount national interest justifies the
(1), Article VIII (Judicial Department) of the Constitution, which appointment of a Chief Justice during the election ban has
provides that any vacancy in the Supreme Court shall be filled impelled the JBC to defer the decision to whom to send its list
within 90 days from the occurrence thereof, to the matter of the of at least three nominees, whether to the incumbent President
appointment of his successor? May the Judicial and Bar or to her successor.[8] He opines that the JBC is thereby
Council (JBC) resume the process of screening the candidates arrogating unto itself the judicial function that is not conferred
nominated or being considered to succeed Chief Justice Puno, upon it by the Constitution, which has limited it to the task of
and submit the list of nominees to the incumbent President recommending appointees to the Judiciary, but has not
even during the period of the prohibition under Section 15, empowered it to finally resolve constitutional questions, which
Article VII? Does mandamus lie to compel the submission of is the power vested only in the Supreme Court under the
the shortlist of nominees by the JBC? Constitution. As such, he contends that the JBC acted with
grave abuse of discretion in deferring the submission of the list
Precs of the Consolidated Cases of nominees to the President; and that a final and definitive
resolution of the constitutional questions raised above would
Petitioners Arturo M. De Castro and John G. Peralta diffuse (sic) the tension in the legal community that would go a
respectively commenced G.R. No. 191002[1] and G.R. No. long way to keep and maintain stability in the judiciary and the
191149[2] as special civil actions for certiorari and mandamus, political system.[9]
praying that the JBC be compelled to submit to the incumbent
President the list of at least three nominees for the position of In G.R. No. 191032, Soriano offers the view that the
the next Chief Justice. JBC committed a grave abuse of discretion amounting to lack
or excess of its jurisdiction when it resolved unanimously on
In G.R. No. 191032,[3] Jaime N. Soriano, via his January 18, 2010 to open the search, nomination, and
petition for prohibition, proposes to prevent the JBC from selection process for the position of Chief Justice to succeed
conducting its search, selection and nomination proceedings Chief Justice Puno, because the appointing authority for the
for the position of Chief Justice. position of Chief Justice is the Supreme Court itself, the
Presidents authority being limited to the appointment of the
In G.R. No. 191057, a special civil action Members of the Supreme Court. Hence, the JBC should not
for mandamus,[4] the Philippine Constitution Association intervene in the process, unless a nominee is not yet a Member
(PHILCONSA) wants the JBC to submit its list of nominees for of the Supreme Court.[10]
the position of Chief Justice to be vacated by Chief Justice
Puno upon his retirement on May 17, 2010, because the
incumbent President is not covered by the prohibition that For its part, PHILCONSA observes in its petition in
applies only to appointments in the Executive Department. G.R. No. 191057 that unorthodox and exceptional
circumstances spawned by the discordant interpretations, due
In Administrative Matter No. 10-2-5-SC,[5] petitioner Estelito M. perhaps to a perfunctory understanding, of Sec. 15, Art. VII in
Mendoza, a former Solicitor General, seeks a ruling from the relation to Secs. 4(1), 8(5) and 9, Art. VIII of the Constitution
Court for the guidance of the JBC on whether Section 15, have bred a frenzied inflammatory legal debate on the
Article VII applies to appointments to the Judiciary. constitutional provisions mentioned that has divided the bench
and the bar and the general public as well, because of its
In G.R. No. 191342,[6] which the Court consolidated on March dimensional impact to the nation and the people, thereby
9, 2010 with the petitions earlier filed, petitioners Amador Z. fashioning transcendental questions or issues affecting the
Tolentino, Jr. and Roland B. Inting, Integrated Bar of the JBCs proper exercise of its principal function of recommending
Philippines (IBP) Governors for Southern Luzon and Eastern appointees to the Judiciary by submitting only to the President
Visayas, respectively, want to enjoin and restrain the JBC from (not to the next President) a list of at least three nominees
submitting a list of nominees for the position of Chief Justice to prepared by the Judicial and Bar Council for every vacancy
the President for appointment during the period provided for in from which the members of the Supreme Court and judges of
Section 15, Article VII. the lower courts may be appointed. [11] PHILCONSA further
believes and submits that now is the time to revisit and
All the petitions now before the Court pose as the principal review Valenzuela, the strange and exotic Decision of the
legal question whether the incumbent President can appoint Court en banc.[12]
the successor of Chief Justice Puno upon his retirement. That
question is undoubtedly impressed with transcendental Peralta states in his petition in G.R. No. 191149
that mandamus can compel the JBC to immediately transmit to
the President, within a reasonable time, its nomination list for
FINALS CONSTITUTION I ACJUCO 121

the position of chief justice upon the mandatory retirement of


Chief Justice Reynato S. Puno, in compliance with its
mandated duty under the Constitution in the event that the The Judicial and Bar Council (JBC)
Court resolves that the President can appoint a Chief Justice announces the opening for application or
even during the election ban under Section 15, Article VII of recommendation, of the position of CHIEF
the Constitution.[13] JUSTICE OF THE SUPREME COURT,
which will be vacated on 17 May
The petitioners in G.R. No. 191342 insist that there is 2010 upon the retirement of the incumbent
an actual controversy, considering that the JBC has initiated Chief Justice, HON. REYNATO S. PUNO.
the process of receiving applications for the position of Chief
Justice and has in fact begun the evaluation process for the Applications or recommendations
applications to the position, and is perilously near completing for this position must be submitted not later
the nomination process and coming up with a list of nominees than 4 February 2010 (Thursday) to the
for submission to the President, entering into the period of the JBC Secretariat xxx:
ban on midnight appointments on March 10, 2010, which only The announcement was published on January 20,
highlights the pressing and compelling need for a writ of 2010 in the Philippine Daily Inquirer and The Philippine
prohibition to enjoin such alleged ministerial function of Star.[17]
submitting the list, especially if it will be cone within the period
of the ban on midnight appointments.[14] Conformably with its existing practice, the JBC automatically
Antecedents considered for the position of Chief Justice the five most senior
of the Associate Justices of the Court, namely: Associate
These cases trace their genesis to the controversy Justice Antonio T. Carpio; Associate Justice Renato C.
that has arisen from the forthcoming compulsory retirement of Corona; Associate Justice Conchita Carpio Morales; Associate
Chief Justice Puno on May 17, 2010, or seven days after the Justice Presbitero J. Velasco, Jr.; and Associate Justice
presidential election. Under Section 4(1), in relation to Section Antonio Eduardo B. Nachura. However, the last two declined
9, Article VIII, that vacancy shall be filled within ninety days their nomination through letters dated January 18,
from the occurrence thereof from a list of at least three 2010 and January 25, 2010, respectively.[18]
nominees prepared by the Judicial and Bar Council for every
vacancy. Others either applied or were nominated. Victor Fernandez,
the retired Deputy Ombudsman for Luzon, applied, but later
On December 22, 2009, Congressman Matias V. formally withdrew his name from consideration through his
Defensor, an ex officio member of the JBC, addressed a letter letter dated February 8, 2010. Candidates who accepted their
to the JBC, requesting that the process for nominations to the nominations without conditions were Associate Justice Renato
office of the Chief Justice be commenced immediately. C. Corona; Associate Justice Teresita J. Leonardo-De Castro;
Associate Justice Arturo D. Brion; and Associate Justice
In its January 18, 2010 meeting en banc, therefore, Edilberto G. Sandoval (Sandiganbayan). Candidates who
the JBC passed a resolution,[15] which reads: accepted their nominations with conditions were Associate
Justice Antonio T. Carpio and Associate Justice Conchita
The JBC, in its en banc meeting Carpio Morales.[19] Declining their nominations were Atty.
of January 18, 2010, unanimously agreed Henry Villarica (via telephone conversation with the Executive
to start the process of filling up the position Officer of the JBC on February 5, 2010) and Atty. Gregorio M.
of Chief Justice to be vacated on May 17, Batiller, Jr. (via telephone conversation with the Executive
2010 upon the retirement of the incumbent Officer of the JBC on February 8, 2010).[20]
Chief Justice Honorable Reynato S. Puno.
The JBC excluded from consideration former RTC Judge
It will publish the opening of the Florentino Floro (for failure to meet the standards set by the
position for applications or JBC rules); and Special Prosecutor Dennis Villa-Ignacio of the
recommendations; deliberate on the list of Office of the Ombudsman (due to cases pending in the Office
candidates; publish the names of of the Ombudsman).[21]
candidates; accept comments on or
opposition to the applications; conduct In its meeting of February 8, 2010, the JBC resolved to proceed
public interviews of candidates; and to the next step of announcing the names of the following
prepare the shortlist of candidates. candidates to invite the public to file their sworn complaint,
written report, or opposition, if any, not later than February 22,
As to the time to submit this shortlist 2010, to wit: Associate Justice Carpio, Associate Justice
to the proper appointing authority, in the Corona, Associate Justice Carpio Morales, Associate Justice
light of the Constitution, existing laws and Leonardo-De Castro, Associate Justice Brion, and Associate
jurisprudence, the JBC welcomes and will Justice Sandoval. The announcement came out in
consider all views on the matter. the Philippine Daily Inquirer and The Philippine Star issues
of February 13, 2010.[22]
18 January 2010.
Issues

Although it has already begun the process for the


(sgd.) filling of the position of Chief Justice Puno in accordance with
MA. LUISA D. VILLARAMA its rules, the JBC is not yet decided on when to submit to the
Clerk of Court & President its list of nominees for the position due to the
Ex-Officio Secretary controversy now before us being yet unresolved. In the
Judicial and Bar Council meanwhile, time is marching in quick step towards May 17,
2010 when the vacancy occurs upon the retirement of Chief
Justice Puno.
As a result, the JBC opened the position of Chief Justice for
application or recommendation, and published for that purpose The actions of the JBC have sparked a vigorous
its announcement dated January 20, 2010,[16] viz: debate not only among legal luminaries, but also among non-
FINALS CONSTITUTION I ACJUCO 122

legal quarters, and brought out highly disparate opinions on a. Does Section 15, Article VII of the
whether the incumbent President can appoint the next Chief Constitution apply to appointments to
Justice or not. Petitioner Mendoza notes that positions in the Judiciary under
in Valenzuela, which involved the appointments of two judges Section 9, Article VIII of the
of the Regional Trial Court, the Court addressed this issue now Constitution?
before us as an administrative matter to avoid any possible
polemics concerning the matter, but he opines that the b. May President Gloria Macapagal-Arroyo
polemics leading to Valenzuela would be miniscule [sic] make appointments to the Judiciary
compared to the polemics that have now erupted in regard to after March 10, 2010, including that for
the current controversy, and that unless put to a halt, and this the position of Chief Justice after Chief
may only be achieved by a ruling from the Court, the integrity Justice Puno retires on May 17, 2010?
of the process and the credibility of whoever is appointed to the
position of Chief Justice, may irreparably be impaired. [23] G.R. No. 191149

Accordingly, we reframe the issues as submitted by each a. Does the JBC have the discretion to
petitioner in the order of the chronological filing of their withhold the submission of the short
petitions. list to President Gloria Macapagal-
Arroyo?

G.R. No. 191342


G.R. No. 191002
a. Does the JBC have the authority to
submit the list of nominees to the
a. Does the JBC have the power and incumbent President without
authority to resolve the constitutional committing a grave violation of the
question of whether the incumbent Constitution and jurisprudence
President can appoint a Chief Justice prohibiting the incumbent President
during the election ban period? from making midnight appointments
two months immediately preceding the
b. Does the incumbent President have the next presidential elections until the
power and authority to appoint during end of her term?
the election ban the successor of
Chief Justice Puno when he vacates b. Is any act performed by the JBC,
the position of Chief Justice on his including the vetting of the candidates
retirement on May 17, 2010? for the position of Chief Justice,
constitutionally invalid in view of the
JBCs illegal composition allowing
G.R. No. 191032 each member from the Senate and the
House of Representatives to have one
a. Is the power to appoint the Chief Justice vote each?
vested in the Supreme Court en banc?

G.R. No. 191057 On February 16, 2010, the Court directed the JBC
and the Office of the Solicitor General (OSG) to comment on
a. Is the constitutional prohibition against the consolidated petitions, except that filed in G.R. No.
appointment under Section 15, Article 191342.
VII of the Constitution applicable only
to positions in the Executive On February 26, 2010, the JBC submitted its
Department? comment, reporting therein that the next stage of the process
for the selection of the nominees for the position of Chief
b. Assuming that the prohibition under Justice would be the public interview of the candidates and the
Section 15, Article VII of the preparation of the short list of candidates, including the
Constitution also applies to members interview of the constitutional experts, as may be needed. [24] It
of the Judiciary, may such stated:[25]
appointments be excepted because
they are impressed with public interest
or are demanded by the exigencies of Likewise, the JBC has yet to take a
public service, thereby justifying these position on when to submit the
appointments during the period of shortlist to the proper appointing
prohibition? authority, in light of Section 4 (1),
Article VIII of the Constitution,
c. Does the JBC have the authority to which provides that vacancy in the
decide whether or not to include and Supreme Court shall be filled within
submit the names of nominees who ninety (90) days from the
manifested interest to be nominated occurrence thereof, Section 15,
for the position of Chief Justice on the Article VII of the Constitution
understanding that his/her nomination concerning the ban on Presidential
will be submitted to the next President appointments two (2) months
in view of the prohibition against immediately before the next
presidential appointments from March presidential elections and up to the
11, 2010 until June 30, 2010? end of his term and Section 261 (g),
Article XXII of the Omnibus Election
A. M. No. 10-2-5-SC Code of the Philippines.
FINALS CONSTITUTION I ACJUCO 123

12. Since the Honorable Supreme Court is Supreme Court should be so reduced that it will have no
the final interpreter of the Constitution, quorum, or should the voting on a particular important question
the JBC will be guided by its decision requiring expeditious resolution be divided; [34] and
in these consolidated Petitions and that Valenzuela also recognized that the filling of vacancies in
Administrative Matter. the Judiciary is undoubtedly in the public interest, most
especially if there is any compelling reason to justify the
making of the appointments during the period of the
On February 26, 2010, the OSG also submitted its prohibition.[35]
comment, essentially stating that the incumbent President can
appoint the successor of Chief Justice Puno upon his Lastly, the OSG urges that there are now undeniably
retirement by May 17, 2010. compelling reasons for the incumbent President to appoint the
next Chief Justice, to wit: (a) a deluge of cases involving
The OSG insists that: (a) a writ of prohibition cannot sensitive political issues is quite expected; [36] (b) the Court acts
issue to prevent the JBC from performing its principal function as the Presidential Electoral Tribunal (PET), which, sitting en
under the Constitution to recommend appointees in the banc, is the sole judge of all contests relating to the election,
Judiciary; (b) the JBCs function to recommend is a continuing returns, and qualifications of the President and Vice President
process, which does not begin with each vacancy or end with and, as such, has the power to correct manifest errors on the
each nomination, because the goal is to submit the list of statement of votes (SOV) and certificates of canvass
nominees to Malacaang on the very day the vacancy (COC);[37] (c) if history has shown that during ordinary
arises;[26] the JBC was thus acting within its jurisdiction when it times the Chief Justice was appointed immediately upon the
commenced and set in motion the process of selecting the occurrence of the vacancy, from the time of the effectivity of
nominees to be submitted to the President for the position of the Constitution, there is now even more reason to appoint the
Chief Justice to be vacated by Chief Justice Puno; [27] (c) next Chief Justice immediately upon the retirement of Chief
petitioner Sorianos theory that it is the Supreme Court, not the Justice Puno;[38] and (d) should the next Chief Justice come
President, who has the power to appoint the Chief Justice, is from among the incumbent Associate Justices of the Supreme
incorrect, and proceeds from his misinterpretation of the Court, thereby causing a vacancy, it also becomes incumbent
phrase members of the Supreme Court found in Section 9, upon the JBC to start the selection process for the filling up of
Article VIII of the Constitution as referring only to the Associate the vacancy in accordance with the constitutional mandate.[39]
Justices, to the exclusion of the Chief Justice; [28] (d) a writ
of mandamus can issue to compel the JBC to submit the list of On March 9, 2010, the Court admitted the following
nominees to the President, considering that its duty to prepare comments/oppositions-in-intervention, to wit:
the list of at least three nominees is unqualified, and the
submission of the list is a ministerial act that the JBC is (a) The opposition-in-intervention
mandated to perform under the Constitution; as such, the JBC, dated February 22, 2010 of Atty. Peter
the nature of whose principal function is executive, is not Irving Corvera (Corvera);[40]
vested with the power to resolve who has the authority to
appoint the next Chief Justice and, therefore, has no discretion (b) The opposition-in-intervention
to withhold the list from the President; [29] and (e) a writ dated February 22, 2010 of Atty.
of mandamus cannot issue to compel the JBC to include or Christian Robert S. Lim (Lim);
exclude particular candidates as nominees, considering that
there is no imperative duty on its part to include in or exclude (c) The opposition-in-intervention
from the list particular individuals, but, on the contrary, the dated February 23, 2010 of Atty.
JBCs determination of who it nominates to the President is an Alfonso V. Tan, Jr. (Tan);
exercise of a discretionary duty.[30]
(d) The comment/opposition-in-
The OSG contends that the incumbent President intervention dated March 1, 2010 of
may appoint the next Chief Justice, because the prohibition the National Union of Peoples
under Section 15, Article VII of the Constitution does not apply Lawyers (NUPL);
to appointments in the Supreme Court. It argues that any
vacancy in the Supreme Court must be filled within 90 days (e) The opposition-in-intervention
from its occurrence, pursuant to Section 4(1), Article VIII of the dated February 25, 2010 of Atty.
Constitution; [31] that in their deliberations on the mandatory Marlou B. Ubano (Ubano);
period for the appointment of Supreme Court Justices, the
framers neither mentioned nor referred to the ban against (f) The opposition-in-intervention
midnight appointments, or its effects on such period, or vice dated February 25, 2010 of Integrated
versa;[32] that had the framers intended the prohibition to apply Bar of the Philippines-Davao del Sur
to Supreme Court appointments, they could have easily Chapter and its Immediate Past
expressly stated so in the Constitution, which explains why the President, Atty. Israelito P. Torreon
prohibition found in Article VII (Executive Department) was not (IBP- Davao del Sur);
written in Article VIII (Judicial Department); and that the
framers also incorporated in Article VIII ample restrictions or (g) The opposition-in-intervention
limitations on the Presidents power to appoint members of the dated February 26, 2010 of Atty.
Supreme Court to ensure its independence from political Mitchell John L. Boiser (Boiser);
vicissitudes and its insulation from political pressures, [33] such
as stringent qualifications for the positions, the establishment (h)The consolidated comment/opposition-
of the JBC, the specified period within which the President shall in-intervention dated February 26,
appoint a Supreme Court Justice. 2010 of BAYAN Chairman Dr.
Carolina P. Araullo; BAYAN Secretary
The OSG posits that although Valenzuela involved General Renato M. Reyes, Jr.;
the appointment of RTC Judges, the situation now refers to the Confederation for Unity, Recognition
appointment of the next Chief Justice to which the prohibition and Advancement of Government
does not apply; that, at any rate, Valenzuela even recognized Employees (COURAGE) Chairman
that there might be the imperative need for an appointment Ferdinand Gaite; Kalipunan ng
during the period of the ban, like when the membership of the Damayang Mahihirap (KADAMAY)
FINALS CONSTITUTION I ACJUCO 124

Secretary General Gloria Arellano; ice is imperative for the stability of the judicial system and the
Alyansa ng Nagkakaisang Kabataan political situation in the country when the election-related
ng Samayanan Para sa Kaunlaran questions reach the Court as false, because there is an
(ANAKBAYAN) Chairman Ken existing law on filling the void brought about by a vacancy in
Leonard Ramos; Tayo ang Pag-asa the office of Chief Justice; that the law is Section 12 of the
Convenor Alvin Peters; League of Judiciary Act of 1948, which has not been repealed by Batas
Filipino Students (LFS) Chairman Pambansa Blg. 129 or any other law; that a temporaryor
James Mark Terry Lacuanan Ridon; an acting Chief Justice is not anathema to judicial
National Union of Students of the independence; that the designation of an acting Chief Justice
Philippines (NUSP) Chairman Einstein is not only provided for by law, but is also dictated by practical
Recedes, College Editors Guild of the necessity; that the practice was intended to be enshrined in the
Philippines (CEGP) Chairman Vijae 1987 Constitution, but the Commissioners decided not to write
Alquisola; and Student Christian it in the Constitution on account of the settled practice; that the
Movement of the Philippines (SCMP) practice was followed under the 1987 Constitution, when, in
Chairman Ma. Cristina Angela 1992, at the end of the term of Chief Justice Marcelo B. Fernan,
Guevarra (BAYAN et al.); Associate Justice Andres Narvasa assumed the position as
Acting Chief Justice prior to his official appointment as Chief
(i) The opposition-in-intervention Justice; that said filling up of a vacancy in the office of the Chief
dated March 3, 2010 of Walden F. Justice was acknowledged and even used by analogy in the
Bello and Loretta Ann P. Rosales case of the vacancy of the Chairman of the Commission on
(Bello et al.); and Elections, per Brillantes v. Yorac, 192 SCRA 358; and that the
history of the Supreme Court has shown that this rule of
(j) The consolidated comment/opposition- succession has been repeatedly observed and has become a
in-intervention dated March 4, 2010 of part of its tradition.
the Women Trial Lawyers
Organization of the Philippines Intervenors Ubano, Boiser, NUPL, Corvera, and Lim
(WTLOP), represented by Atty. maintain that the Omnibus Election Code penalizes as an
Yolanda Quisumbing-Javellana; Atty. election offense the act of any government official who
Belleza Alojado Demaisip; Atty. appoints, promotes, or gives any increase in salary or
Teresita Gandionco-Oledan; Atty. Ma. remuneration or privilege to any government official or
Verena Kasilag-Villanueva; Atty. employee during the period of 45 days before a regular
Marilyn Sta. Romana; Atty. Leonila de election; that the provision covers all appointing heads,
Jesus; and Atty. Guinevere de Leon officials, and officers of a government office, agency or
(WTLOP). instrumentality, including the President; that for the incumbent
President to appoint the next Chief Justice upon the retirement
of Chief Justice Puno, or during the period of the ban under
Intervenors Tan, WTLOP, BAYAN et al., Corvera, IBP Davao the Omnibus Election Code, constitutes an election offense;
del Sur, and NUPL take the position that De Castros petition that even an appointment of the next Chief Justice prior to the
was bereft of any basis, because under Section 15, Article VII, election ban is fundamentally invalid and without effect
the outgoing President is constitutionally banned from making because there can be no appointment until a vacancy occurs;
any appointments from March 10, 2010 until June 30, 2010, and that the vacancy for the position can occur only by May 17,
including the appointment of the successor of Chief Justice 2010.
Puno. Hence, mandamus does not lie to compel the JBC to
submit the list of nominees to the outgoing President if the Intervenor Boiser adds that De Castros prayer to
constitutional prohibition is already in effect. Tan adds that the compel the submission of nominees by the JBC to the
prohibition against midnight appointments was applied by the incumbent President is off-tangent because the position of
Court to the appointments to the Judiciary made by then Chief Justice is still not vacant; that to speak of a list, much
President Ramos, with the Court holding that the duty of the more a submission of such list, before a vacancy occurs is
President to fill the vacancies within 90 days from occurrence glaringly premature; that the proposed advance appointment
of the vacancies (for the Supreme Court) or from the by the incumbent President of the next Chief Justice will be
submission of the list (for all other courts) was not an excuse unconstitutional; and that no list of nominees can be submitted
to violate the constitutional prohibition. by the JBC if there is no vacancy.

Intervenors Tan, Ubano, Boiser, Corvera, NULP, All the intervenors-oppositors submit that Section 15, Article
BAYAN et al., and Bello et al. oppose the insistence VII makes no distinction between the kinds of appointments
that Valenzuela recognizes the possibility that the President made by the President; and that the Court, in Valenzuela, ruled
may appoint the next Chief Justice if exigent circumstances that the appointments by the President of the two judges during
warrant the appointment, because that recognition is obiter the prohibition period were void.
dictum; and aver that the absence of a Chief Justice or even
an Associate Justice does not cause epic damage or absolute Intervenor WTLOP posits that Section 15, Article VII
disruption or paralysis in the operations of the Judiciary. They of the 1987 Constitution does not apply only to the
insist that even without the successor of Chief Justice Puno appointments in the Executive Department, but also to judicial
being appointed by the incumbent President, the Court is appointments, contrary to the submission of PHILCONSA; that
allowed to sit and adjudge en banc or in divisions of three, five Section 15 does not distinguish; and that Valenzuela already
or seven members at its discretion; that a full membership of interpreted the prohibition as applicable to judicial
the Court is not necessary; that petitioner De Castros fears are appointments.
unfounded and baseless, being based on a mere possibility, Intervenor WTLOP further posits that petitioner
the occurrence of which is entirely unsure; that it is not in the Sorianos contention that the power to appoint the Chief Justice
national interest to have a Chief Justice whose appointment is is vested, not in the President, but in the Supreme Court, is
unconstitutional and, therefore, void; and that such a situation utterly baseless, because the Chief Justice is also a Member
will create a crisis in the judicial system and will worsen an of the Supreme Court as contemplated under Section 9, Article
already vulnerable political situation. VIII; and that, at any rate, the term members was interpreted
in Vargas v. Rillaroza (G.R. No. L-1612, February 26, 1948) to
refer to the Chief Justice and the Associate Justices of the
FINALS CONSTITUTION I ACJUCO 125

Supreme Court; that PHILCONSAs prayer that the Court pass avoid obstructing the efficient functioning of public officials and
a resolution declaring that persons who manifest their interest offices involved in public service. It is required, therefore, that
as nominees, but with conditions, shall not be considered the petitioner must have a personal stake in the outcome of the
nominees by the JBC is diametrically opposed to the controversy, for, as indicated in Agan, Jr. v. Philippine
arguments in the body of its petition; that such glaring International Air Terminals Co., Inc.:[42]
inconsistency between the allegations in the body and the relief
prayed for highlights the lack of merit of PHILCONSAs petition; The question on legal standing is
that the role of the JBC cannot be separated from the whether such parties have alleged such
constitutional prohibition on the President; and that the Court a personal stake in the outcome of the
must direct the JBC to follow the rule of law, that is, to submit controversy as to assure that concrete
the list of nominees only to the next duly elected President after adverseness which sharpens the
the period of the constitutional ban against midnight presentation of issues upon which the
appointments has expired. court so largely depends for
illumination of difficult constitutional
Oppositor IBP Davao del Sur opines that the questions.[43] Accordingly, it has been
JBC because it is neither a judicial nor a quasi-judicial held that the interest of a person
body has no duty under the Constitution to resolve the assailing the constitutionality of a
question of whether the incumbent President can appoint a statute must be direct and personal. He
Chief Justice during the period of prohibition; that even if the must be able to show, not only that the
JBC has already come up with a short list, it still has to bow to law or any government act is invalid, but
the strict limitations under Section 15, Article VII; that should also that he sustained or is in imminent
the JBC defer submission of the list, it is not arrogating unto danger of sustaining some direct injury
itself a judicial function, but simply respecting the clear as a result of its enforcement, and not
mandate of the Constitution; and that the application of the merely that he suffers thereby in some
general rule in Section 15, Article VII to the Judiciary does not indefinite way. It must appear that the
violate the principle of separation of powers, because said person complaining has been or is
provision is an exception. about to be denied some right or
privilege to which he is lawfully entitled
Oppositors NUPL, Corvera, Lim and BAYAN et or that he is about to be subjected to
al. state that the JBCs act of nominating appointees to the some burdens or penalties by reason of
Supreme Court is purely ministerial and does not involve the the statute or act complained of.[44]
exercise of judgment; that there can be no default on the part
of the JBC in submitting the list of nominees to the President, It is true that as early as in 1937, in People v.
considering that the call for applications only begins from the Vera,[45] the Court adopted the direct injury test for determining
occurrence of the vacancy in the Supreme Court; and that the whether a petitioner in a public action had locus standi. There,
commencement of the process of screening of applicants to fill the Court held that the person who would assail the validity of
the vacancy in the office of the Chief Justice only begins from a statute must have a personal and substantial interest in the
the retirement on May 17, 2010, for, prior to this date, there is case such that he has sustained, or will sustain direct injury as
no definite legal basis for any party to claim that the submission a result. Vera was followed in Custodio v. President of the
or non-submission of the list of nominees to the President by Senate,[46] Manila Race Horse Trainers Association v. De la
the JBC is a matter of right under law. Fuente,[47] Anti-Chinese League of the Philippines v.
Felix,[48] and Pascual v. Secretary of Public Works.[49]
The main question presented in all the filings
herein because it involves two seemingly conflicting provisions Yet, the Court has also held that the requirement
of the Constitution imperatively demands the attention and of locus standi, being a mere procedural technicality, can be
resolution of this Court, the only authority that can resolve the waived by the Court in the exercise of its discretion. For
question definitively and finally. The imperative demand rests instance, in 1949, in Araneta v. Dinglasan,[50] the Court
on the ever-present need, first, to safeguard the liberalized the approach when the cases had transcendental
independence, reputation, and integrity of the entire Judiciary, importance. Some notable controversies whose petitioners did
particularly this Court, an institution that has been not pass the direct injury test were allowed to be treated in the
unnecessarily dragged into the harsh polemics brought on by same way as in Araneta v. Dinglasan.[51]
the controversy; second, to settle once and for all the doubt
about an outgoing Presidents power to appoint to the Judiciary In the 1975 decision in Aquino v. Commission on
within the long period starting two months before the Elections,[52] this Court decided to resolve the issues raised by
presidential elections until the end of the presidential term; the petition due to their far-reaching implications, even if the
and third, to set a definite guideline for the JBC to follow in the petitioner had no personality to file the suit. The liberal
discharge of its primary office of screening and nominating approach of Aquino v. Commission on Elections has been
qualified persons for appointment to the Judiciary. adopted in
several notable cases, permitting ordinary citizens, legislators
Thus, we resolve. , and civic
organizations to bring their suits involving the constitutionality
Ruling of the Court or validity of laws, regulations, and rulings. [53]

Locus Standi of Petitioners However, the assertion of a public right as a


predicate for challenging a supposedly illegal or
The preliminary issue to be settled is whether or not unconstitutional executive or legislative action rests on the
the petitioners have locus standi. theory that the petitioner represents the public in general.
Although such petitioner may not be as adversely affected by
Black defines locus standi as a right of appearance the action complained against as are others, it is enough that
in a court of justice on a given question. [41] In public or he sufficiently demonstrates in his petition that he is entitled to
constitutional litigations, the Court is often burdened with the protection or relief from the Court in the vindication of a public
determination of the locus standi of the petitioners due to the right.
ever-present need to regulate the invocation of the intervention
of the Court to correct any official action or policy in order to
FINALS CONSTITUTION I ACJUCO 126

Quite often, as here, the petitioner in a public action the body that has constitutional supervision and authority over
sues as a citizen or taxpayer to gain locus standi. That is not them and other members of the legal profession.[61]
surprising, for even if the issue may appear to concern only the The Court rules that the petitioners have each
public in general, such capacities nonetheless equip the demonstrated adequate interest in the outcome of the
petitioner with adequate interest to sue. In David v. controversy as to vest them with the requisite locus standi. The
Macapagal-Arroyo,[54] the Court aptly explains why: issues before us are of transcendental importance to the
people as a whole, and to the petitioners in particular. Indeed,
Case law in most jurisdictions now the issues affect everyone (including the petitioners),
allows both citizen and taxpayer standing in regardless of ones personal interest in life, because they
public actions. The distinction was first laid concern that great doubt about the authority of the incumbent
down in Beauchamp v. Silk,[55] where it was President to appoint not only the successor of the retiring
held that the plaintiff in a taxpayers suit is incumbent Chief Justice, but also others who may serve in the
in a different category from the plaintiff in a Judiciary, which already suffers from a far too great number of
citizens suit. In the former, the plaintiff is vacancies in the ranks of trial judges throughout the country.
affected by the expenditure of public
funds, while in the latter, he is but the In any event, the Court retains the broad discretion
mere instrument of the public to waive the requirement of legal standing in favor of any
concern. As held by the New York petitioner when the matter involved has transcendental
Supreme Court in People ex rel Case v. importance, or otherwise requires a liberalization of the
Collins:[56] In matter of mere public right, requirement.[62]
howeverthe people are the real partiesIt
is at least the right, if not the duty, of Yet, if any doubt still lingers about the locus standi of
every citizen to interfere and see that a any petitioner, we dispel the doubt now in order to remove any
public offence be properly pursued and obstacle or obstruction to the resolution of the essential issue
punished, and that a public grievance be squarely presented herein. We are not to shirk from
remedied. With respect to taxpayers discharging our solemn duty by reason alone of an obstacle
suits, Terr v. Jordan[57] held that the right more technical than otherwise. In Agan, Jr. v.Philippine
of a citizen and a taxpayer to maintain International Air Terminals Co., Inc.,[63] we pointed out:
an action in courts to restrain the Standing is a peculiar concept in constitutional law because in
unlawful use of public funds to his some cases, suits are not brought by parties who have been
injury cannot be denied.[58] personally injured by the operation of a law or any other
government act but by concerned citizens, taxpayers or voters
who actually sue in the public interest. But even if, strictly
Petitioners De Castro (G.R. No. 191002), Soriano speaking, the petitioners are not covered by the definition, it is
(G.R. No. 191032) and Peralta (G.R. No. 191149) all assert still within the wide discretion of the Court to waive the
their right as citizens filing their petitions on behalf of the public requirement and so remove the impediment to its addressing
who are directly affected by the issue of the appointment of the and resolving the serious constitutional questions raised. [64]
next Chief Justice. De Castro and Soriano further claim
standing as taxpayers, with Soriano averring that he is affected Justiciability
by the continuing proceedings in the JBC, which involve
unnecessary, if not, illegal disbursement of public funds. [59] Intervenor NUPL maintains that there is no actual
case or controversy that is appropriate or ripe for adjudication,
PHILCONSA alleges itself to be a non-stock, non- considering that although the selection process commenced
profit organization existing under the law for the purpose of by the JBC is going on, there is yet no final list of nominees;
defending, protecting, and preserving the Constitution and hence, there is no imminent controversy as to whether such list
promoting its growth and flowering. It also alleges that the must be submitted to the incumbent President, or reserved for
Court has recognized its legal standing to file cases on submission to the incoming President.
constitutional issues in several cases. [60]
Intervenor Tan raises the lack of any actual
In A.M. No. 10-2-5-SC, Mendoza states that he is a justiciable controversy that is ripe for judicial determination,
citizen of the Philippines, a member of the Philippine Bar pointing out that petitioner De Castro has not even shown that
engaged in the active practice of law, and a former Solicitor the JBC has already completed its selection process and is
General, former Minister of Justice, former Member of the now ready to submit the list to the incumbent President; and
Interim Batasang Pambansa and the Regular Batasang that petitioner De Castro is merely presenting a hypothetical
Pambansa, and former member of the Faculty of the College scenario that is clearly not sufficient for the Court to exercise
of Law of the University of the Philippines. its power of judicial review.

The petitioners in G.R. No. 191342 are the Intervenors Corvera and Lim separately opine that
Governors of the Integrated Bar of the Philippines (IBP) De Castros petition rests on an overbroad and vague allegation
for Southern Luzon and Eastern Visayas. They allege that they of political tension, which is insufficient basis for the Court to
have the legal standing to enjoin the submission of the list of exercise its power of judicial review.
nominees by the JBC to the President, for [a]n adjudication of
the proper interpretation and application of the constitutional Intervenor BAYAN et al. contend that the petitioners
ban on midnight appointments with regard to respondent JBCs are seeking a mere advisory opinion on what the JBC and the
function in submitting the list of nominees is well within the President should do, and are not invoking any issues that are
concern of petitioners, who are duty bound to ensure that justiciable in nature.
obedience and respect for the Constitution is upheld, most
especially by government offices, such as respondent JBC, Intervenors Bello et al. submit that there exist no
who are specifically tasked to perform crucial functions in the conflict of legal rights and no assertion of opposite legal claims
whole scheme of our democratic institution. They further allege in any of the petitions; that PHILCONSA does not allege any
that, reposed in them as members of the Bar, is a clear legal action taken by the JBC, but simply avers that the conditional
interest in the process of selecting the members of the manifestations of two Members of the Court, accented by the
Supreme Court, and in the selection of the Chief Justice, divided opinions and interpretations of legal experts, or
considering that the person appointed becomes a member of associations of lawyers and law students on the issues
FINALS CONSTITUTION I ACJUCO 127

published in the daily newspapers are matters of paramount already began, or that are reasons persuading the JBC to
and transcendental importance to the bench, bar and general desist from the rest of the process.
public; that PHILCONSA fails not only to cite any legal duty or
allege any failure to perform the duty, but also to indicate what We need not await the occurrence of the vacancy
specific action should be done by the JBC; that Mendoza does by May 17, 2010 in order for the principal issue to ripe for
not even attempt to portray the matter as a controversy or judicial determination by the Court. It is enough that one
conflict of rights, but, instead, prays that the Court should rule alleges conduct arguably affected with a constitutional interest,
for the guidance of the JBC; that the fact that the Court but seemingly proscribed by the Constitution. A reasonable
supervises the JBC does not automatically imply that the Court certainty of the occurrence of the perceived threat to a
can rule on the issues presented in the Mendoza petition, constitutional interest is sufficient to afford a basis for bringing
because supervision involves oversight, which means that the a challenge, provided the Court has sufficient facts before it to
subordinate officer or body must first act, and if such action is enable it to intelligently adjudicate the issues. [65]Herein, the
not in accordance with prescribed rules, then, and only then, facts are not in doubt, for only legal issues remain.
may the person exercising oversight order the action to be
redone to conform to the prescribed rules; that the Mendoza Substantive Merits
petition does not allege that the JBC has performed a specific
act susceptible to correction for being illegal or I
unconstitutional; and that the Mendoza petition asks the Court Prohibition under Section 15, Article VII does not apply
to issue an advisory ruling, not to exercise its power of to appointments to fill a vacancy in the Supreme Court
supervision to correct a wrong act by the JBC, but to declare or to other appointments to the Judiciary
the state of the law in the absence of an actual case or
controversy.
Two constitutional provisions are seemingly in
We hold that the petitions set forth an actual case or conflict.
controversy that is ripe for judicial determination. The reality is
that the JBC already commenced the proceedings for the The first, Section 15, Article VII (Executive
selection of the nominees to be included in a short list to be Department), provides:
submitted to the President for consideration of which of them
will succeed Chief Justice Puno as the next Chief Justice. Section 15. Two months
Although the position is not yet vacant, the fact that the JBC immediately before the next presidential
began the process of nomination pursuant to its rules and elections and up to the end of his term, a
practices, although it has yet to decide whether to submit the President or Acting President shall not
list of nominees to the incumbent outgoing President or to the make appointments, except temporary
next President, makes the situation ripe for judicial appointments to executive positions when
determination, because the next steps are the public interview continued vacancies therein will prejudice
of the candidates, the preparation of the short list of public service or endanger public safety.
candidates, and the interview of constitutional experts, as may
be needed.
The other, Section 4 (1), Article VIII (Judicial
A part of the question to be reviewed by the Court is Department), states:
whether the JBC properly initiated the process, there being an
insistence from some of the oppositors-intervenors that the Section 4. (1). The Supreme Court
JBC could only do so once the vacancy has occurred (that is, shall be composed of a Chief Justice and
after May 17, 2010). Another part is, of course, whether the fourteen Associate Justices. It may sit en
JBC may resume its process until the short list is prepared, in banc or in its discretion, in division of three,
view of the provision of Section 4(1), Article VIII, which five, or seven Members. Any vacancy shall
unqualifiedly requires the President to appoint one from the be filled within ninety days from the
short list to fill the vacancy in the Supreme Court (be it the Chief occurrence thereof.
Justice or an Associate Justice) within 90 days from the
occurrence of the vacancy.
In the consolidated petitions, the petitioners, with the
The ripeness of the controversy for judicial exception of Soriano, Tolentino and Inting, submit that the
determination may not be doubted. The challenges to the incumbent President can appoint the successor of Chief
authority of the JBC to open the process of nomination and to Justice Puno upon his retirement on May 17, 2010, on the
continue the process until the submission of the list of ground that the prohibition against presidential appointments
nominees; the insistence of some of the petitioners to compel under Section 15, Article VII does not extend to appointments
the JBC through mandamus to submit the short list to the in the Judiciary.
incumbent President; the counter-insistence of the intervenors
to prohibit the JBC from submitting the short list to the The Court agrees with the submission.
incumbent President on the ground that said list should be
submitted instead to the next President; the strong position that First. The records of the deliberations of the
the incumbent President is already prohibited under Section Constitutional Commission reveal that the framers devoted
15, Article VII from making any appointments, including those time to meticulously drafting, styling, and arranging the
to the Judiciary, starting on May 10, 2010 until June 30, 2010; Constitution. Such meticulousness indicates that the
and the contrary position that the incumbent President is not organization and arrangement of the provisions of the
so prohibited are only some of the real issues for Constitution were not arbitrarily or whimsically done by the
determination. All such issues establish the ripeness of the framers, but purposely made to reflect their intention and
controversy, considering that for some the short list must be manifest their vision of what the Constitution should contain.
submitted before the vacancy actually occurs by May 17,
2010. The outcome will not be an abstraction, or a merely The Constitution consists of 18 Articles, three of
hypothetical exercise. The resolution of the controversy will which embody the allocation of the awesome powers of
surely settle with finality the nagging questions that are government among the three great departments, the
preventing the JBC from moving on with the process that it Legislative (Article VI), the Executive (Article VII), and the
Judicial Departments (Article VIII). The arrangement was a
FINALS CONSTITUTION I ACJUCO 128

true recognition of the principle of separation of powers that He also wished to ensure that that number
underlies the political structure, as Constitutional would not be reduced for any appreciable
Commissioner Adolfo S. Azcuna (later a worthy member of the length of time (even only temporarily), and
Court) explained in his sponsorship speech: to this end proposed that any vacancy must
be filled within two months from the date
We have in the political part of this that the vacancy occurs. His proposal to
Constitution opted for the separation of have a 15-member Court was not initially
powers in government because we believe adopted. Persisting however in his desire
that the only way to protect freedom and to make certain that the size of the Court
liberty is to separate and divide the would not be decreased for any substantial
awesome powers of government. Hence, period as a result of vacancies, Lerum
we return to the separation of powers proposed the insertion in the provision
doctrine and the legislative, executive and (anent the Courts membership) of the
judicial departments.[66] same mandate that IN CASE OF ANY
VACANCY, THE SAME SHALL BE FILLED
WITHIN TWO MONTHS FROM
OCCURRENCE THEREOF. He later
As can be seen, Article VII is devoted to the agreed to suggestions to make the period
Executive Department, and, among others, it lists the powers three, instead of two, months. As thus
vested by the Constitution in the President. The presidential amended, the proposal was approved. As
power of appointment is dealt with in Sections 14, 15 and 16 it turned out, however, the Commission
of the Article. ultimately agreed on a fifteen-member
Court. Thus it was that the section fixing
Article VIII is dedicated to the Judicial Department the composition of the Supreme Court
and defines the duties and qualifications of Members of the came to include a command to fill up
Supreme Court, among others. Section 4(1) and Section 9 of any vacancy therein within 90 days from
this Article are the provisions specifically providing for the its occurrence.
appointment of Supreme Court Justices. In particular, Section
9 states that the appointment of Supreme Court Justices can In this connection, it may be pointed
only be made by the President upon the submission of a list of out that that instruction that any
at least three nominees by the JBC; Section 4(1) of the Article vacancy shall be filled within ninety days
mandates the President to fill the vacancy within 90 days from (in the last sentence of Section 4 (1) of
the occurrence of the vacancy. Article VIII) contrasts with the prohibition in
Section 15, Article VII, which is couched in
Had the framers intended to extend the prohibition stronger negative language - that a
contained in Section 15, Article VII to the appointment of President or Acting President shall
Members of the Supreme Court, they could have explicitly not make appointments
done so. They could not have ignored the meticulous ordering
of the provisions. They would have easily and surely written The commission later approved a
the prohibition made explicit in Section 15, Article VII as being proposal of Commissioner Hilario G.
equally applicable to the appointment of Members of the Davide, Jr. (now a Member of this Court) to
Supreme Court in Article VIII itself, most likely in Section 4 (1), add to what is now Section 9 of Article VIII,
Article VIII. That such specification was not done only reveals the following paragraph: WITH RESPECT
that the prohibition against the President or Acting President TO LOWER COURTS, THE PRESIDENT
making appointments within two months before the next SHALL ISSUE THE APPOINTMENT
presidential elections and up to the end of the Presidents or WITHIN NINETY DAYS FROM THE
Acting Presidents term does not refer to the Members of the SUBMISSION OF THE LIST (of nominees
Supreme Court. by the Judicial and Bar Council to the
President). Davide stated that his purpose
was to provide a uniform rule for lower
courts. According to him, the 90-day period
Although Valenzuela[67] came to hold that the should be counted from submission of the
prohibition covered even judicial appointments, it cannot be list of nominees to the President in view of
disputed that the Valenzuela dictum did not firmly rest on the the possibility that the President might
deliberations of the Constitutional Commission. Thereby, the reject the list submitted to him and the JBC
confirmation made to the JBC by then Senior Associate Justice thus need more time to submit a new one.
Florenz D. Regalado of this Court, a former member of the
Constitutional Commission, about the prohibition not being On the other hand, Section 15,
intended to apply to the appointments to the Judiciary, which Article VII - which in effect deprives the
confirmation Valenzuela even expressly mentioned, should President of his appointing power two
prevail. months immediately before the next
presidential elections up to the end ofhis
Relevantly, Valenzuela adverted to the intent of the framers in term - was approved without discussion.[68]
the genesis of Section 4 (1), Article VIII, viz:
However, the reference to the records of the Constitutional
V . Intent of the Constitutional Commission did not advance or support the result
Commission in Valenzuela. Far to the contrary, the records disclosed the
express intent of the framers to enshrine in the Constitution,
The journal of the Commission upon the initiative of Commissioner Eulogio Lerum, a
which drew up the present Constitution command [to the President] to fill up any vacancy therein within
discloses that the original proposal was to 90 days from its occurrence, which
have an eleven-member Supreme Court. even Valenzuela conceded.[69] The exchanges
Commissioner Eulogio Lerum wanted to during deliberations of the Constitutional Commission
increase the number of Justices to fifteen. on October 8, 1986 further show that the filling of a vacancy in
FINALS CONSTITUTION I ACJUCO 129

the Supreme Court within the 90-day period was statute which are inconsistent be
a true mandate for the President, viz: harmonized at a sacrifice of the legislative
intention. It may be that two provisions are
MR. DE CASTRO. I understand that irreconcilable; if so, the one which
our justices now in the Supreme Court, expresses the intent of the law-makers
together with the Chief Justice, are only 11. should control. And the arbitrary rule has
been frequently announced that where
MR. CONCEPCION. Yes. there is an irreconcilable conflict between
the different provisions of a statute, the
MR. DE CASTRO. And the second provision last in order of position will
sentence of this subsection reads: Any prevail, since it is the latest expression of
vacancy shall be filled within ninety the legislative will. Obviously, the rule is
days from the occurrence thereof. subject to deserved criticism. It is seldom
applied, and probably then only where an
MR. CONCEPCION. That is right. irreconcilable conflict exists between
different sections of the same act, and after
MR. DE CASTRO. Is this now a all other means of ascertaining the
mandate to the executive to fill the meaning of the legislature have been
vacancy? exhausted. Where the conflict is between
two statutes, more may be said in favor of
MR. CONCEPCION. That is right. the rules application, largely because of the
That is borne out of the fact that in the principle of implied repeal.
past 30 years, seldom has the Court had
a complete complement.[70]
Moreover, the usage in Section 4(1), Article VIII of In this connection, PHILCONSAs urging of a revisit
the word shall an imperative, operating to impose a duty that and a review of Valenzuela is timely and
may be enforced[71] should not be disregarded. Thereby, appropriate. Valenzuela arbitrarily ignored the express intent
Sections 4(1) imposes on the President the imperative duty to of the Constitutional Commission to have Section 4 (1), Article
make an appointment of a Member of the Supreme Court VIII stand independently of any other provision, least of all one
within 90 days from the occurrence of the vacancy. The failure found in Article VII. It further ignored that the two provisions
by the President to do so will be a clear disobedience to the had no irreconcilable conflict, regardless of Section 15, Article
Constitution. VII being couched in the negative. As judges, we are not to
The 90-day limitation fixed in Section 4(1), Article VIII unduly interpret, and should not accept an interpretation that
for the President to fill the vacancy in the Supreme Court was defeats the intent of the framers.[73]
undoubtedly a special provision to establish a definite
mandate for the President as the appointing power, and cannot Consequently, prohibiting the incumbent President from
be defeated by mere judicial interpretation in Valenzuela to the appointing a Chief Justice on the premise that Section 15,
effect that Section 15, Article VII prevailed because it was Article VII extends to appointments in the Judiciary cannot be
couched in stronger negative language. Such interpretation sustained. A misinterpretation like Valenzuela should not be
even turned out to be conjectural, in light of the records of the allowed to last after its false premises have been exposed. [74] It
Constitutional Commissions deliberations on Section 4 (1), will not do to merely distinguish Valenzuela from these cases,
Article VIII. for the result to be reached herein is entirely incompatible with
what Valenzuela decreed. Consequently,
How Valenzuela justified its pronouncement and Valenzuela now deserves to be quickly sent to the dustbin of
result is hardly warranted. According to an authority on the unworthy and forgettable.
statutory construction:[72]
We reverse Valenzuela.
xxx the court should seek to avoid
any conflict in the provisions of the statute Second. Section 15, Article VII does not apply as well
by endeavoring to harmonize and reconcile to all other appointments in the Judiciary.
every part so that each shall be effective. It
is not easy to draft a statute, or any other There is no question that one of the reasons
writing for that matter, which may not in underlying the adoption of Section 15 as part of Article VII was
some manner contain conflicting to eliminate midnight appointments from being made by
provisions. But what appears to the reader an outgoing Chief Executive in the mold of the appointments
to be a conflict may not have seemed so to dealt with in the leading case of Aytona v. Castillo.[75] In fact,
the drafter. Undoubtedly, each provision in Valenzuela, the Court so observed, stating that:
was inserted for a definite reason. Often by
considering the enactment in its entirety, xxx it appears that Section 15,
what appears to be on its face a conflict Article VII is directed against two types of
may be cleared up and the provisions appointments: (1) those made for buying
reconciled. votes and (2) those made for partisan
considerations. The first refers to those
Consequently, that construction appointments made within the two months
which will leave every word operative will preceding a Presidential election and are
be favored over one which leaves some similar to those which are declared election
word or provision meaningless because of offenses in the Omnibus Election
inconsistency. But a word should not be Code, viz.:
given effect, if to do so gives the statute a
meaning contrary to the intent of the xxx
legislature. On the other hand, if full effect
cannot be given to the words of a statute, The second type of appointments
they must be made effective as far as prohibited by Section 15, Article VII
possible. Nor should the provisions of a consists of the so-
FINALS CONSTITUTION I ACJUCO 130

called midnight appointments. In Aytona v. to executive positions when continued


Castillo, it was held that after the vacancies will prejudice public service or
proclamation of Diosdado Macapagal as endanger public safety. Obviously, the
duly elected President, President Carlos P. article greatly restricts the appointing
Garcia, who was defeated in his bid for power of the President during the period of
reelection, became no more than a the ban.
caretaker administrator whose duty was to
prepare for the orderly transfer of authority Considering the respective reasons
to the incoming President. Said the Court: for the time frames for filling vacancies in
the courts and the restriction on the
The filling up of President's power of appointment, it is this
vacancies in important Courts view that, as a general proposition,
positions, if few, and so in case of conflict, the former should yield
spaced as to afford some to the latter. Surely, the prevention of vote-
assurance of deliberate buying and similar evils outweighs the need
action and careful for avoiding delays in filling up of court
consideration of the need for vacancies or the disposition of some
the appointment and cases. Temporary vacancies can abide the
appointee's qualifications period of the ban which, incidentally and as
may undoubtedly be earlier pointed out, comes to exist only
permitted. But the issuance once in every six years.Moreover, those
of 350 appointments in one occurring in the lower courts can be filled
night and the planned temporarily by designation. But prohibited
induction of almost all of appointments are long-lasting and
them in a few hours before permanent in their effects. They may, as
the inauguration of the new earlier pointed out, in fact influence the
President may, with some results of elections and, for that reason,
reason, be regarded by the their making is considered an election
latter as an abuse of offense.[76]
Presidential prerogatives,
the steps taken being
apparently a mere partisan Given the background and rationale for the
effort to fill all vacant prohibition in Section 15, Article VII, we have no doubt that the
positions irrespective of Constitutional Commission confined the prohibition to
fitness and other conditions, appointments made in the Executive Department. The framers
and thereby to deprive the did not need to extend the prohibition to appointments in the
new administration of an Judiciary, because their establishment of the JBC and their
opportunity to make the subjecting the nomination and screening of candidates for
corresponding judicial positions to the unhurried and deliberate prior process
appointments. of the JBC ensured that there would no longer
be midnight appointments to the Judiciary. If midnight
As indicated, the Court recognized appointments in the mold of Aytona were made in haste and
that there may well be appointments to with irregularities, or made by an outgoing Chief Executive in
important positions which have to be made the last days of his administration out of a desire to subvert the
even after the proclamation of the new policies of the incoming President or for partisanship, [77] the
President. Such appointments, so long appointments to the Judiciary made after the establishment of
as they are few and so spaced as to the JBC would not be suffering from such defects because of
afford some assurance of deliberate the JBCs prior processing of candidates. Indeed, it is axiomatic
action and careful consideration of the in statutory construction that the ascertainment of the purpose
need for the appointment and the of the enactment is a step in the process of ascertaining the
appointees qualifications, can be made intent or meaning of the enactment, because the reason for the
by the outgoing President. Accordingly, enactment must necessarily shed considerable light on the law
several appointments made by President of the statute, i.e., the intent; hence, the enactment should be
Garcia, which were shown to have been construed with reference to its intended scope and purpose,
well considered, were upheld. and the court should seek to carry out this purpose rather than
to defeat it.[78]
Section 15, Article VII has a
broader scope than the Aytona ruling. It Also, the intervention of the JBC eliminates the
may not unreasonably be deemed to danger that appointments to the Judiciary can be made for the
contemplate not only midnight purpose of buying votes in a coming presidential election, or of
appointments those made obviously for satisfying partisan considerations. The experience from the
partisan reasons as shown by their time of the establishment of the JBC shows that even
number and the time of their making but candidates for judicial positions at any level backed by people
also appointments presumed made for influential with the President could not always be assured of
the purpose of influencing the outcome being recommended for the consideration of the President,
of the Presidential election. because they first had to undergo the vetting of the JBC and
pass muster there. Indeed, the creation of the JBC
On the other hand, the exception in was precisely intended to de-politicize the Judiciary by doing
the same Section 15 of Article VII allowing away with the intervention of the Commission on
appointments to be made during the period Appointments. This insulating process was absent from
of the ban therein provided is much the Aytona midnight appointment.
narrower than that recognized
in Aytona. The exception allows only the Third. As earlier stated, the non-applicability of
making of temporary appointments Section 15, Article VII to appointments in the Judiciary was
FINALS CONSTITUTION I ACJUCO 131

confirmed by then Senior Associate Justice Regalado to the and Legislative Departments. Such a holding will tie the
JBC itself when it met on March 9, 1998 to discuss the Judiciary and the Supreme Court to the fortunes or misfortunes
question raised by some sectors about the constitutionality of of political leaders vying for the Presidency in a presidential
xxx appointments to the Court of Appeals in light of the election. Consequently, the wisdom of having the new
forthcoming presidential elections. He assured that on the President, instead of the current incumbent President, appoint
basis of the (Constitutional) Commissions records, the election the next Chief Justice is itself suspect, and cannot ensure
ban had no application to appointments to the Court of judicial independence, because the appointee can also
Appeals.[79] This confirmation was accepted by the JBC, which become beholden to the appointing authority. In contrast, the
then submitted to the President for consideration the appointment by the incumbent President does not run the
nominations for the eight vacancies in the Court of Appeals.[80] same risk of compromising judicial independence, precisely
because her term will end by June 30, 2010.
The fault of Valenzuela was that it accorded no
weight and due consideration to the confirmation of Justice Sixth. The argument has been raised to the effect
Regalado. Valenzuela was weak, because it relied on that there will be no need for the incumbent President to
interpretation to determine the intent of the framers rather than appoint during the prohibition period the successor of Chief
on the deliberations of the Constitutional Commission. Much of Justice Puno within the context of Section 4 (1), Article VIII,
the unfounded doubt about the Presidents power to appoint because anyway there will still be about 45 days of the 90 days
during the period of prohibition in Section 15, Article VII could mandated in Section 4(1), Article VIII remaining.
have been dispelled since its promulgation on November 9,
1998, had Valenzuela properly acknowledged and relied on The argument is flawed, because it is focused only
the confirmation of a distinguished member of the on the coming vacancy occurring from Chief Justice Punos
Constitutional Commission like Justice Regalado. retirement by May 17, 2010. It ignores the need to apply
Section 4(1) to every situation of a vacancy in the Supreme
Fourth. Of the 23 sections in Article VII, three (i.e., Court.
Section 14, Section15, and Section 16) concern the appointing
powers of the President. The argument also rests on the fallacious
assumption that there will still be time remaining in the 90-day
Section 14 speaks of the power of period under Section 4(1), Article VIII. The fallacy is easily
the succeeding President to revoke appointments made by an demonstrable, as the OSG has shown in its comment.
Acting President,[81] and evidently refers only to appointments
in the Executive Department. It has no application to Section 4 (3), Article VII requires the regular elections
appointments in the Judiciary, to be held on the second Monday of May, letting the elections
because temporary or acting appointments can only fall on May 8, at the earliest, or May 14, at the latest. If the
undermine the independence of the Judiciary due to their being regular presidential elections are held on May 8, the period of
revocable at will.[82] The letter and spirit of the Constitution the prohibition is 115 days. If such elections are held on May
safeguard that independence. Also, there is no law in the 14, the period of the prohibition is 109 days. Either period of
books that authorizes the revocation of appointments in the the prohibition is longer than the full mandatory 90-day period
Judiciary. Prior to their mandatory retirement or resignation, to fill the vacancy in the Supreme Court. The result is that there
judges of the first and second level courts and the Justices of are at least 19 occasions (i.e., the
the third level courts may only be removed for cause, but the difference between the shortest possible period of the ban of
Members of the Supreme Court may be removed only by 109 days and the 90-day mandatory period for appointments)
impeachment. in which the outgoing President would be in no position to
comply with the constitutional duty to fill up a vacancy in the
Section 16 covers only the presidential appointments Supreme Court. It is safe to assume that the framers of the
that require confirmation by the Commission on Appointments. Constitution could not have intended such an absurdity. In fact,
Thereby, the Constitutional Commission restored the in their deliberations on the mandatory period for the
requirement of confirmation by the Commission on appointment of Supreme Court Justices under Section 4 (1),
Appointments after the requirement was removed from the Article VIII, the framers neither discussed, nor mentioned, nor
1973 Constitution. Yet, because of Section 9 of Article VIII, the referred to the ban against midnight appointments under
restored requirement did not include appointments to the Section 15, Article VII, or its effects on the 90-day period,
Judiciary.[83] or vice versa. They did not need to, because they never
intended Section 15, Article VII to apply to a vacancy in the
Section 14, Section 15, and Section 16 are obviously Supreme Court, or in any of the lower courts.
of the same character, in that they affect the power of the
President to appoint. The fact that Section 14 and Section 16 Seventh. As a matter of fact, in an extreme case, we
refer only to appointments within the Executive Department can even raise a doubt on whether a JBC list is necessary at
renders conclusive that Section 15 also applies only to the all for the President any President to appoint a Chief Justice if
Executive Department. This conclusion is consistent with the the appointee is to come from the ranks of the sitting justices
rule that every part of the statute must be interpreted with of the Supreme Court.
reference to the context, i.e. that every part must be Sec. 9, Article VIII says:
considered together with the other parts, and kept subservient
to the general intent of the whole enactment. [84] It is absurd to xxx. The Members of the Supreme
assume that the framers deliberately situated Section Court xxx shall be appointed by the
15 between Section 14 and Section 16, if they intended President from a list of at least three
Section 15 to cover all kinds of presidential appointments. If nominees prepared by the Judicial and Bar
that was their intention in respect of appointments to the Council for any vacancy. Such
Judiciary, the framers, if only to be clear, would have easily and appointments need no confirmation.
surely inserted a similar prohibition in Article VIII, most likely xxx
within Section 4 (1) thereof.

Fifth. To hold like the Court did in Valenzuela that The provision clearly refers to an appointee coming into the
Section 15 extends to appointments to the Judiciary further Supreme Court from the outside, that is, a non-member of the
undermines the intent of the Constitution of ensuring the Court aspiring to become one. It speaks of candidates for the
independence of the Judicial Department from the Executive Supreme Court, not of those who are already members or
FINALS CONSTITUTION I ACJUCO 132

sitting justices of the Court, all of whom have previously been Justice is not yet appointed, or in which the incumbent Chief
vetted by the JBC. Justice is unable to perform the duties and powers of the
office. It ought to be remembered, however, that it was enacted
Can the President, therefore, appoint any of the because the Chief Justice appointed under the 1935
incumbent Justices of the Court as Chief Justice? Constitution was subject to the confirmation of the Commission
on Appointments, and the confirmation process might take
The question is not squarely before us at the longer than expected.
moment, but it should lend itself to a deeper analysis if and
when circumstances permit. It should be a good issue for the The appointment of the next Chief Justice by the
proposed Constitutional Convention to consider in the light of incumbent President is preferable to having the Associate
Senate President Juan Ponce Enriles statement that the Justice who is first in precedence take over. Under the
President can appoint the Chief Justice from among the sitting Constitution, the heads of the Legislative and Executive
justices of the Court even without a JBC list. Departments are popularly elected, and whoever are elected
and proclaimed at once become the leaders of their respective
II Departments. However, the lack of any appointed occupant of
The Judiciary Act of 1948 the office of Chief Justice harms the independence of the
Judiciary, because the Chief Justice is the head of the entire
The posture has been taken that no urgency exists for the Judiciary. The Chief Justice performs functions absolutely
President to appoint the successor of Chief Justice Puno, significant to the life of the nation. With the entire Supreme
considering that the Judiciary Act of 1948 can still address the Court being the Presidential Electoral Tribunal, the Chief
situation of having the next President appoint the successor. Justice is the Chairman of the Tribunal. There being no
obstacle to the appointment of the next Chief Justice, aside
from its being mandatory for the incumbent President to make
Section 12 of the Judiciary Act of 1948 states: within the 90-day period from May 17, 2010, there is no
justification to insist that the successor of Chief Justice Puno
Section 12. Vacancy in Office of be appointed by the next President.
Chief Justice. In case of a vacancy in the
office of Chief Justice of the Supreme Court Historically, under the present Constitution, there has
or of his inability to perform the duties and been no wide gap between the retirement and the resignation
powers of his office, they shall devolve of an incumbent Chief Justice, on one hand, and the
upon the Associate Justice who is first in appointment to and assumption of office of his successor, on
precedence, until such disability is the other hand. As summarized in the comment of the OSG,
removed, or another Chief Justice is the chronology of succession is as follows:
appointed and duly qualified. This provision
shall apply to every Associate Justice who 1. When Chief Justice Claudio
succeeds to the office of Chief Justice. Teehankee retired on April 18, 1988,
Chief Justice Pedro Yap was
appointed on the same day;
The provision calls for an Acting Chief Justice in the
event of a vacancy in the office of the Chief Justice, or in the 2. When Chief Justice Yap retired
event that the Chief Justice is unable to perform his duties and on July 1, 1988, Chief Justice Marcelo
powers. In either of such circumstances, the duties and powers Fernan was appointed on the same
of the office of the Chief Justice shall devolve upon the day;
Associate Justice who is first in precedence until a new Chief
Justice is appointed or until the disability is removed. 3. When Chief Justice Fernan resigned
on December 7, 1991, Chief Justice
Notwithstanding that there is no pressing need to dwell on this Andres Narvasa was appointed the
peripheral matter after the Court has hereby resolved the following day, December 8, 1991;
question of consequence, we do not find it amiss to confront
the matter now. 4. When Chief Justice Narvasa retired
on November 29, 1998, Chief Justice
We cannot agree with the posture. Hilario Davide, Jr. was sworn into
office the following early morning
A review of Sections 4(1) and 9 of Article VIII shows that the of November 30, 1998;
Supreme Court is composed of a Chief Justice and 14
Associate Justices, who all shall be appointed by the President 5. When Chief Justice Davide retired on
from a list of at least three nominees prepared by the JBC for December 19, 2005, Chief Justice
every vacancy, which appointments require no confirmation by Artemio Panganiban was appointed
the Commission on Appointments. With reference to the Chief the next day, December 20, 2005; and
Justice, he or she is appointed by the President as Chief
Justice, and the appointment is never in an acting capacity. 6. When Chief Justice Panganiban
The express reference to a Chief Justice abhors the idea that retired on December 6, 2006, Chief
the framers contemplated an Acting Chief Justice to head the Justice Reynato S. Puno took his oath
membership of the Supreme Court. Otherwise, they would as Chief Justice
have simply written so in the Constitution. Consequently, to at midnight of December 6, 2006.[85]
rely on Section 12 of the Judiciary Act of 1948 in order to
forestall the imperative need to appoint the next Chief Justice III
soonest is to defy the plain intent of the Constitution. Writ of mandamus does not lie against the JBC

For sure, the framers intended the position of Chief May the JBC be compelled to submit the list of nominees to the
Justice to be permanent, not one to be occupied in an acting President?
or temporary capacity. In relation to the scheme of things
under the present Constitution, Section 12 of the Judiciary Act Mandamus shall issue when any tribunal, corporation, board,
of 1948 only responds to a rare situation in which the new Chief officer or person unlawfully neglects the performance of an act
FINALS CONSTITUTION I ACJUCO 133

that the law specifically enjoins as a duty resulting from an recommending nominees to the Judiciary, that is, in submitting
office, trust, or station.[86] It is proper when the act against the list to the President.
which it is directed is one addressed to the discretion of the
tribunal or officer. Mandamus is not available to direct the The distinction between a ministerial act and a
exercise of a judgment or discretion in a particular way. [87] discretionary one has been delineated in the following manner:

For mandamus to lie, the following requisites must be The distinction between a ministerial and
complied with: (a) the plaintiff has a clear legal right to the act discretionary act is well delineated.
demanded; (b) it must be the duty of the defendant to perform A purely ministerial act or duty is one
the act, because it is mandated by law; (c) the defendant which an officer or tribunal performs in
unlawfully neglects the performance of the duty enjoined by a given state of facts, in a prescribed
law; (d) the act to be performed is ministerial, not discretionary; manner, in obedience to the mandate of
and (e) there is no appeal or any other plain, speedy and a legal authority, without regard to or
adequate remedy in the ordinary course of law. the exercise of his own judgment upon
the propriety or impropriety of the act
Section 8(5) and Section 9, Article VIII, mandate the done. If the law imposes a duty upon a
JBC to submit a list of at least three nominees to the President public officer and gives him the right to
for every vacancy in the Judiciary: decide how or when the duty shall be
performed, such duty is
Section 8. xxx discretionary and not ministerial.
The duty is ministerial only when the
(5) The Council shall have the discharge of the same requires neither
principal function of recommending the exercise of official discretion or
appointees to the Judiciary. xxx judgment.[89]

Section 9. The Members of the


Supreme Court and judges of lower courts Accordingly, we find no sufficient grounds to grant the petitions
shall be appointed by the President from a for mandamus and to issue a writ of mandamus against the
list of at least three nominees prepared JBC. The actions for that purpose are premature, because it is
by the Judicial and Bar Council for every clear that the JBC still has until May 17, 2010, at the latest,
vacancy. Such appointments need no within which to submit the list of nominees to the President to
confirmation. fill the vacancy created by the compulsory retirement of Chief
Justice Puno.
For the lower courts, the President
shall issue the appointments within
ninety days from the submission of the IV
list. Writ of prohibition does not lie against the JBC

However, Section 4(1) and Section 9, Article VIII, In light of the foregoing disquisitions, the conclusion
mandate the President to fill the vacancy in the Supreme Court is ineluctable that only the President can appoint the Chief
within 90 days from the occurrence of the vacancy, and within Justice. Hence, Sorianos petition for prohibition in G.R. No.
90 days from the submission of the list, in the case of the lower 191032, which proposes to prevent the JBC from intervening
courts. The 90-day period is directed at the President, not at in the process of nominating the successor of Chief Justice
the JBC. Thus, the JBC should start the process of selecting Puno, lacks merit.
the candidates to fill the vacancy in the Supreme
Court before the occurrence of the vacancy. On the other hand, the petition for prohibition in G.R.
No. 191342 is similarly devoid of merit. The challenge mounted
Under the Constitution, it is mandatory for the JBC to against the composition of the JBC based on the allegedly
submit to the President the list of nominees to fill a vacancy in unconstitutional allocation of a vote each to the ex
the Supreme Court in order to enable the President to appoint officio members from the Senate and the House of
one of them within the 90-day period from the occurrence of Representatives, thereby prejudicing the chances of some
the vacancy. The JBC has no discretion to submit the list to the candidates for nomination by raising the minimum number of
President after the vacancy occurs, because that shortens the votes required in accordance with the rules of the JBC, is not
90-day period allowed by the Constitution for the President to based on the petitioners actual interest, because they have not
make the appointment. For the JBC to do so will be alleged in their petition that they were nominated to the JBC to
unconscionable on its part, considering that it will fill some vacancies in the Judiciary. Thus, the petitioners
thereby effectively and illegally deprive the President of the lack locus standi on that issue.
ample time granted under the Constitution to reflect on the
qualifications of the nominees named in the list of the JBC
before making the appointment. WHEREFORE, the Court:

The duty of the JBC to submit a list of 1. Dismisses the petitions


nominees before the start of the Presidents mandatory 90-day for certiorari and mandamus in G.R. No. 191002 and G.R. No.
period to appoint is ministerial, but its selection of the 191149, and the petition for mandamus in G.R. No. 191057 for
candidates whose names will be in the list to be submitted to being premature;
the President lies within the discretion of the JBC. The object
of the petitions for mandamus herein should only refer to the 2. Dismisses the petitions for prohibition in G.R.
duty to submit to the President the list of nominees for every No. 191032 and G.R. No. 191342 for lack of merit; and
vacancy in the Judiciary, because in order to constitute
unlawful neglect of duty, there must be an unjustified delay in 3. Grants the petition in A.M. No. 10-2-5-SC and,
performing that duty.[88] For mandamus to lie against the JBC, accordingly, directs the Judicial and Bar Council:
therefore, there should be an unexplained delay on its part in
FINALS CONSTITUTION I ACJUCO 134

(a) To resume its proceedings for the


nomination of candidates to fill the
vacancy to be created by the
compulsory retirement of Chief Justice
Reynato S. Puno by May 17, 2010;

(b) To prepare the short list of nominees for


the position of Chief Justice;

(c) To submit to the incumbent President


the short list of nominees for the
position of Chief Justice on or before
May 17, 2010; and

(d) To continue its proceedings for


the nomination of candidates to fill
other vacancies in the Judiciary and
submit to the President the short list of
nominees corresponding thereto in
accordance with this decision.

SO ORDERED.
FINALS CONSTITUTION I ACJUCO 135

G.R. No. 203372 June 16, 2015 Atty. Venturanza is a Petition for Review on
Certiorari,3 assailing the Decision4 dated 31 August 2012 and
ATTY. CHELOY E. VELICARIA-GARAFIL, Petitioner, Resolution5 dated 12 March 2013 of the CA in CA-G.R. SP No.
123659. G.R. No. 209138 filed by Villanueva and Rosquita is
vs.
OFFICE OF THE PRESIDENT and HON. SOLICITOR a Petition for Certiorari,6 seeking to nullify the Decision7 dated
GENERAL JOSE ANSELMO I. CADIZ, Respondents. 28 August 2013 of the CA in CA-G.R. SP Nos. 123662,
123663, and 123664.8 Villanueva and Rosquita filed a Petition-
in-Intervention in the consolidated cases before the CA. G.R.
x-----------------------x No. 212030 is a Petition for Review on Certiorari, 9 assailing the
Decision10 dated 31 August 2012 of the CA in CAG.R. SP No.
G.R. No. 206290 123664 and Resolution11 dated 7 April 2014 of the CA in
CAG.R. SP Nos. 123662, 123663, and 123664. 12
ATTY. DINDO G. VENTURANZA, Petitioner,
vs. Facts of the Cases
OFFICE OF THE PRESIDENT, LEILA M. DE LIMA, in her
capacity as the Secretary of the Department of Justice, Prior to the conduct of the May 2010 elections, then President
CLARO A. ARELLANO, in his capacity as the Prosecutor Gloria Macapagal-Arroyo (President Macapagal-Arroyo)
General, and RICHARD ANTHONY D. FADULLON, in his issued more than 800 appointments to various positions in
capacity as the Officer-in-Charge of the Office of the City several government offices.
Prosecutor of Quezon City,Respondents.
The ban on midnight appointments in Section 15, Article VII of
x-----------------------x the 1987 Constitution reads:

G.R. No. 209138 Two months immediately before the next presidential elections
and up to the end of his term, a President or Acting President
IRMA A. VILLANUEVA and FRANCISCA B. shall not make appointments, except temporary appointments
ROSQUITA, Petitioners, to executive positions when continued vacancies therein will
vs. prejudice public service or endanger public safety.
COURT OF APPEALS and THE OFFICE OF THE
PRESIDENT, Respondents. Thus, for purposes of the 2010 elections, 10 March 2010 was
the cutoff date for valid appointments and the next day, 11
x-----------------------x March 2010, was the start of the ban on midnight
appointments. Section 15, Article VII of the 1987 Constitution
recognizes as an exception to the ban on midnight
G.R. No. 212030 appointments only "temporary appointments to executive
positions when continued vacancies therein will prejudice
EDDIE U. TAMONDONG, Petitioner, public service or endanger public safety." None of the
vs. petitioners claim that their appointments fall under this
EXECUTIVE SECRETARY PAQUITO N. OCHOA, exception.
JR., Respondent.
Appointments
DECISION
G.R. No. 203372
CARPIO, J.:
The paper evidencing Atty. Velicaria-Garafil's appointment as
The present consolidated cases involve four petitions: G.R. State Solicitor II at the OSG was dated 5 March 2010.13 There
No. 203372 with Atty. Cheloy E. Velicaria-Garafil (Atty. was a transmittal letter dated 8 March 2010 of the appointment
Velicaria-Garafil), who was appointed State Solicitor II at the paper from the Office of the President (OP), but this transmittal
Office of the Solicitor General (OSG), as petitioner; G.R. No. letter was received by the Malacañang Records Office (MRO)
206290 with Atty. Dindo G. Venturanza (Atty. Venturanza), only on 13 May 2010. There was no indication as to the OSG's
who was appointed Prosecutor IV (City Prosecutor) of Quezon date of receipt of the appointment paper. On 19 March 2010,
City, as petitioner; G.R. No. 209138 with Irma A. Villanueva the OSG's Human Resources Department called up Atty.
(Villanueva), who was appointed Administrator for Visayas of Velicaria-Garafil to schedule her oath-taking. Atty. Velicaria-
the Board of Administrators of the Cooperative Development Garafil took her oath of office as State Solicitor II on 22 March
Authority (CDA), and Francisca B. Rosquita (Rosquita), who 2010 and assumed her position on 6 April 2010.
was appointed Commissioner of the National Commission of
Indigenous Peoples (NCIP), as petitioners; and G.R. No. G.R. No. 206290
212030 with Atty. Eddie U. Tamondong (Atty. Tamondong),
who was appointed member of the Board of Directors of the
Subic Bay Metropolitan Authority (SBMA), as petitioner. All The paper evidencing Atty. Venturanza's appointment as
petitions question the constitutionality of Executive Order No. Prosecutor IV (City Prosecutor) of Quezon City was dated 23
2 (EO 2) for being inconsistent with Section 15, Article VII of February 2010.14 It is apparent, however, that it was only on 12
the 1987 Constitution. March 2010 that the OP, in a letter dated 9 March 2010,
transmitted Atty. Venturanza's appointment paper to then
Department of Justice (DOJ) Secretary Alberto C.
Petitioners seek the :reversal of the separate Decisions of the Agra.15 During the period between 23 February and 12 March
Court of Appeals (CA) that dismissed their petitions and upheld 2010, Atty. Venturanza, upon verbal advice from Malacañang
the constitutionality of EO 2. G.R. No. 203372 filed by Atty. of his promotion but without an official copy of his appointment
Velicaria-Garafil is a Petition for Review on paper, secured clearances from the Civil Service Commission
Certiorari,1 assailing the Decision2 dated 31 August 2012 of (CSC),16 Sandiganbayan,17 and the DOJ.18 Atty. Venturanza
the CA in CA-G.R. SP No. 123662. G.R. No. 206290 filed by
FINALS CONSTITUTION I ACJUCO 136

took his oath of office on 15 March 2010, and assumed office Issuance of EO 2
on the same day.
On 30 June 2010, President Benigno S. Aquino III (President
G.R. No. 209138 Aquino) took his oath of office as President of the Republic of
the Philippines. On 30 July 2010, President Aquino issued EO
2 recalling, withdrawing, and revoking appointments issued by
The paper evidencing Villanueva's appointment as
Administrator for Visayas of the Board of Administrators of the President Macapagal-Arroyo which violated the constitutional
ban on midnight appointments.
CDA was dated 3 March 2010.19 There was no transmittal
letter of the appointment paper from the OP. Villanueva took
her oath of office on 13 April 2010. The entirety of EO 2 reads:

The paper evidencing Rosquita's appointment as EXECUTIVE ORDER NO. 2


Commissioner, representing Region I and the Cordilleras, of
the NCIP was dated 5 March 2010.20 Like Villanueva, there RECALLING, WITHDRAWING, AND REVOKING
was no transmittal letter of the appointment paper from the OP. APPOINTMENTS ISSUED BY THE PREVIOUS
Rosquita took her oath of office on 18 March 2010. G.R. No.
ADMINISTRATION IN VIOLATION OF THE
212030 CONSTITUTIONAL BAN ON MIDNIGHT APPOINTMENTS,
AND FOR OTHER PURPOSES.
The paper evidencing Atty. Tamondong's appointment as
member, representing the private sector, of the SBMA Board
WHEREAS, Sec. 15, Article VII of the 1987 Constitution
of Directors was dated 1 March 2010.21 Atty. Tamondong provides that "Two months immediately before the next
admitted that the appointment paper was received by the
presidential elections and up to the end of his term, a President
Office of the SBMA Chair on 25 March 201022 and that he took or Acting President shall not make appointments, except
his oath of office on the same day.23 He took another oath of temporary appointments to executive positions when
office on 6 July 2010 as "an act of extra caution because of the continued vacancies therein will prejudice public service or
rising crescendo of noise from the new political mandarins endanger public safety."; WHEREAS, in the case of "In re:
against the so-called 'midnight appointments."'24 Appointments dated March 30, 1998 of Hon. Mateo Valenzuela
and Hon. Vallarta as Judges of the Regional Trial Court of
To summarize, the pertinent dates for each petitioner are as Branch 62 of Bago City and Branch 24 of Cabanatuan City,
follows: respectively" (A.M. No. 98-5-01-SC Nov. 9, 1998), the
Supreme Court interpreted this provision to mean that the
President is neither required to make appointments nor
G.R. No. Date of Date of Date Date Assumpti allowed to do so during the two months immediately before the
Appointm Transmit of of on of next presidential elections and up to the end of her term. The
ent Letter tal Letter Recei Oath Office only known exceptions to this prohibition are (1) temporary
pt by of appointments in the executive positions when continued
MRO Offic vacancies will prejudice public service or endanger public
e safety and in the light of the recent Supreme Court decision in
the case of De Castro, et al. vs. JBC and PGMA, G.R. No.
22 191002, 17 March 2010, (2) appointments to the Judiciary;
203372
13 Marc
(Atty. 5 March 8 March 6 April
May h
Velicaria- 2010 2010 2010 WHEREAS, Section 261 of the Omnibus Election Code
2010 201
Garafil) provides that:
0

15
206290 12 "Section 261. Prohibited Acts.-The following shall be guilty of
23 Marc
(Atty. 9 March Marc 15 March an election offense:
February h
Venturan 2010 h 2010
2010 201
za) 2010
0 (g) Appointments of new employees,
creation of new position, promotion, or
13 giving salary increases. - During the period
209138
3 March 4 May April of forty-five days before a regular election
(Villanuev
2010 2010 201 and thirty days before a special election.
a)
0

18 (1) Any head, official or appointing officer of


209138 13 Marc a government office, agency or
5 March instrumentality, whether national or local,
(Rosquita May h
2010 including government-owned or controlled
) 2010 201
0 corporations, who appoints or hires any
new employee, whether provisional,
25 temporary or casual, or creates and fills any
Marc new position, except upon prior authority to
h the Commission. The Commission shall not
212030 201 grant the authority sought unless it is
(Atty. 1 March 0 satisfied that the position to be filled is
Tamondo 2010 and essential to the proper functioning of the
ng) 6 office or agency concerned, and that the
July position shall not be filled in a manner that
201 may influence the election.
0
FINALS CONSTITUTION I ACJUCO 137

As an exception to the foregoing positions covered or otherwise affected are hereby declared
provisions, a new employee may be vacant.
appointed in the case of urgent need:
SECTION 3. Temporary designations. - When necessary to
Provided, however, that notice of the maintain efficiency in public service and ensure the continuity
appointment shall be given to the of government operations, the Executive Secretary may
Commission within three days from the designate an officer-in-charge (OIC) to perform the duties and
date of the appointment. Any appointment discharge the responsibilities of any of those whose
or hiring in violation of this provision shall appointment has been recalled, until the replacement of the
be null and void. OIC has been appointed and qualified.

(2) Any government official who promotes SECTION 4. Repealing Clause. - All executive issuances,
or gives any increase of salary or orders, rules and regulations or part thereof inconsistent with
remuneration or privilege to any the provisions of this Executive Order are hereby repealed or
government official or employee, including modified accordingly.
those in government-owned or controlled
corporations."; SECTION 5. Separability Clause. - If any section or provision
of this executive order shall be declared unconstitutional or
WHEREAS, it appears on record that a number of invalid, the other sections or provision not affected thereby
appointments were made on or about 10 March 2010 in shall remain in full force and effect.
complete disregard of the intent and spirit of the constitutional
ban on midnight appointment and which deprives the new SECTION 6. Effectivity. - This Executive order shall take effect
administration of the power to make its own appointment; immediately.

WHEREAS, based on established jurisprudence, an DONE in the City of Manila, this 30th day of July, in the year
appointment is deemed complete only upon acceptance of the Two Thousand and Ten.
appointee;

By the President:
WHEREAS, in order to strengthen the civil service system, it is
necessary to uphold the principle that appointments to the civil
service must be made on the basis of merit and fitness, it is (Sgd.) PAQUITO N. OCHOA, JR.
imperative to recall, withdraw, and revoke all appointments Executive Secretary25
made in violation of the letter and spirit of the law;
(Sgd.) BENIGNO S. AQUINO III
NOW, THEREFORE, I, BENIGNO S. AQUINO III, by virtue of
the powers vested in me by the Constitution as President of Effect of the Issuance of EO 2
the Philippines, do hereby order and direct that:
G.R. No. 203372
SECTION 1. Midnight Appointments Defined. - The following
appointments made by the former President and other
appointing authorities in departments, agencies, offices, and On 5 August 2010, Jose Anselmo Cadiz assumed office as
instrumentalities, including government-owned or controlled Solicitor General (Sol. Gen. Cadiz). On 6 August 2010, Sol.
corporations, shall be considered as midnight appointments: Gen. Cadiz instructed a Senior Assistant Solicitor General to
inform the officers and employees affected by EO 2 that they
were terminated from service effective the next day.
(a) Those made on or after March 11, 2010,
including all appointments bearing dates
prior to March 11, 2010 where the Atty. Velicaria-Garafil reported for work on 9 August 2010
appointee has accepted, or taken his oath, without any knowledge of her termination. She was made to
or assumed public office on or after March return the office-issued laptop and cellphone, and was told that
11, 2010, except temporary appointments her salary ceased as of 7 August 2010. On 12 August 2010,
in the executive positions when continued Atty. Velicaria-Garafil was informed that her former secretary
vacancies will prejudice public service or at the OSG received a copy of a memorandum on her behalf.
endanger public safety as may be The memorandum, dated 9 August 2010, bore the subject
determined by the appointing authority. "Implementation of Executive Order No. 2 dated 30 July 2010"
and was addressed to the OSG's Director of Finance and
Management Service.
(b) Those made prior to March 11, 2010,
but to take effect after said date or
appointments to office that would be vacant Atty. Velicaria-Garafil filed a petition for certiorari (G.R. No.
only after March 11, 2010. 193327) before this Court on 1 September 2010. The petition
prayed for the nullification of EO 2, and for her reinstatement
as State Solicitor II without loss of seniority, rights and
(c) Appointments and promotions made privileges, and with full backwages from the time that her salary
during the period of 45 days prior to the was withheld.26
May 10, 2010 elections in violation of
Section 261 of the Omnibus Election Code.
G.R. No. 206290
SECTION 2. Recall, Withdraw, and Revocation of Midnight
Appointments. Midnight appointments, as defined under On 1 September 2010, Atty. Venturanza received via facsimile
Section 1, are hereby recalled, withdrawn, and revoked. The transmission an undated copy of DOJ Order No. 556. DOJ
Order No. 556, issued by DOJ Secretary Leila M. De Lima
FINALS CONSTITUTION I ACJUCO 138

(Sec. De Lima), designated Senior Deputy State Prosecutor 4. Whether EO 2 violated the Civil Service Rules on
Richard Anthony D. Fadullon (Pros. Fadullon) as Officer-in- Appointment.33
Charge of the Office of the City Prosecutor in Quezon City. In
a letter to Sec. De Lima dated 15 September 2010, Atty.
This Court gave the CA the authority to resolve all pending
Venturanza asked for clarification of his status, duties, and matters and applications, and to decide the issues as if these
functions since DOJ Order No. 556 did not address the same. cases were originally filed with the CA.
Atty. Venturanza also asked for a status quo ante order to
prevent Pros. Fadullon ·from usurping the position and
functions of the City Prosecutor of Quezon City. Atty. Rulings of the CA
Venturanza also wrote a letter to President Aquino on the same
day, and sought reaffirmation of his promotion as City Even though the same issues were raised in the different
Prosecutor of Quezon City. petitions, the CA promulgated separate Decisions for the
petitions. The CA consistently ruled that EO 2 is constitutional.
On 6 October 2010, Atty. Venturanza received a letter dated The CA, however, issued different rulings as to the evaluation
25 August 2010 from Sec. De Lima which directed him to of the circumstances of petitioners' appointments. In the cases
relinquish the office to which he was appointed, and to cease of Attys. Velicaria-Garafil and Venturanza, the CA stated that
from performing its functions. the OP should consider the circumstances of their
appointments. In the cases of Villanueva, Rosquita, and Atty.
Tamondong, the CA explicitly stated that · the revocation of
Atty. Venturanza filed a Petition for Certiorari, Prohibition, their appointments was proper because they were midnight
Mandamus with Urgent Prayer for Status Quo Ante Order, appointees.
Temporary Restraining Order and/or Preliminary Mandatory
Injunction (G.R. No. 193 867) before this Court on 14 October
2010.27 G.R. No. 203372 (CA-G.R. SP No. 123662)

G.R. No. 209138 The CA promulgated its Decision in CA-G.R. SP No. 123662
on 31 August 2012. The CA ruled that EO 2 is not
unconstitutional. However, the CA relied on Sales v.
The OP withheld the salaries of Villanueva and Rosquita on Carreon34 in ruling that the OP should evaluate whether Atty.
the basis of EO 2. On 3 August 2010, Villanueva and Rosquita Velicaria-Garafil's appointment had extenuating circumstances
sought to intervene in G.R. No. 192991.28 On 1 October 2010, that might make it fall outside the ambit of EO 2.
Executive Secretary Paquito N. Ochoa, Jr. revoked Rosquita's
appointment as NCIP Commissioner.29 On 13 October 2010,
Villanueva and Rosquita notified this Court that they wanted to The dispositive portion of the CA's Decision reads:
intervene in Atty. Tamondong's petition (G.R. No. 192987)
instead. WHEREFORE, the petition for certiorari and mandamus [is]
DENIED.
G.R. No. 212030
Executive Order No. 2, dated July 30, 2010, is NOT
Atty. Tamondong was removed from the SBMA Board of unconstitutional.
Directors on 30 July 2010. He filed a petition for prohibition,
declaratory relief and preliminary injunction with prayer for The issue on whether or not to uphold petitioner's appointment
temporary restraining order (G.R. No. 192987) before this as State Solicitor II at the OSG is hereby referred to the Office
Court on 9 August 2010. The petition prayed for the prohibition of the President which has the sole authority and discretion to
of the implementation of EO 2, the declaration of his pass upon the same.
appointment as legal, and the declaration of EO 2 as
unconstitutional.30
SO ORDERED.35
Referral to CA
G.R. No. 212030 (CA-G.R. SP No. 123664)
There were several petitions31 and motions for
intervention32 that challenged the constitutionality of EO 2. On 31 August 2012, the CA promulgated its Decision in CA-
G.R. SP No. 123664. The dispositive portion reads as follows:
On 31 January 2012, this Court issued a Resolution referring
the petitions, motions for intervention, as well as various WHEREFORE, premises considered, the instant Petition is
letters, to the CA for further proceedings, including the hereby DISMISSED. Executive Order No. 2 is hereby declared
reception and assessment of the evidence from all parties. We NOT UNCONSTITUTIONAL. Accordingly, the revocation of
defined the issues as follows: Atty. Eddie Tamondong's appointment as Director of Subic Bay
Metropolitan Authority is VALID for being a midnight
appointment.
1. Whether the appointments of the petitioners and
intervenors were midnight appointments within the
coverage of EO 2; SO ORDERED.39

2. Whether all midnight appointments, including The Issues for Resolution


those of petitioners and intervenors, were invalid;
We resolve the following issues in these petitions: (1) whether
3 . Whether the appointments of the petitioners and petitioners' appointments violate Section 15, Article VII of the
intervenors were made with undue haste, hurried 1987 Constitution, and (2) whether EO 2 is constitutional.
maneuvers, for partisan reasons, and not in Ruling of the Court
accordance with good faith; and
FINALS CONSTITUTION I ACJUCO 139

The petitions have no merit. All of petitioners' appointments are Under the circumstances above described, what with the
midnight appointments and are void for violation of Section 15, separation of powers, this Court resolves that it must decline
Article VII of the 1987 Constitution. EO 2 is constitutional. to disregard the Presidential .Administrative Order No. 2,
Villanueva and Rosquita, petitioners in G.R. No. 209138, did cancelling such "midnight" or "last-minute" appointments.
not appeal the CA's ruling under Rule 45, but instead filed a
petition for certiorari under Rule 65. This procedural error alone Of course the Court is . aware of many precedents to the effect
warrants an outright dismissal of G.R. No. 209138. Even if it that once an appointment has been issued, it cannot be
were correctly filed under Rule 45, the petition should still be reconsidered, specially where the appointee has qualified. But
dismissed for being filed out of time. 40 There was also no none of them refer to mass ad interim appointments (three
explanation as to why they did not file a motion for hundred and fifty), issued in the last hours of an outgoing Chief
reconsideration of the CA's Decision. Midnight Appointments Executive, in a setting similar to that outlined herein. On the
other hand, the authorities admit of exceptional circumstances
This ponencia and the dissent both agree that the facts in all justifying revocation and if any circumstances justify
these cases show that "none of the petitioners have shown that revocation, those described herein should fit the exception.
their appointment papers (and transmittal letters) have been
issued (and released) before the ban."41The dates of receipt by Incidentally, it should be stated that the underlying reason for
the MRO, which in these cases are the only reliable evidence
denying the power to revoke after the appointee has qualified
of actual transmittal of the appointment papers by President is the latter's equitable rights. Yet it is doubtful if such equity
Macapagal-Arroyo, are dates clearly falling during the
might be successfully set up in the present situation,
appointment ban. Thus, this ponencia and the dissent both considering the rush conditional appointments, hurried
agree that all the appointments in these cases are midnight maneuvers and other happenings detracting from that degree
appointments in violation of Section 15, Article VII of the 1987 of good faith, morality and propriety which form the basic
Constitution. foundation of claims to equitable relief. The appointees, it might
be argued, wittingly or unwittingly cooperated with the
Constitutionality of EO 2 stratagem to beat the deadline, whatever the resultant
consequences to the dignity and efficiency of the public
Based on prevailing jurisprudence, appointment to a service. Needless to say, there are instances wherein not only
government post is a process that takes several steps to strict legality, but also fairness, justice and righteousness
complete. Any valid appointment, including one made under should be taken into account.43
the exception provided in Section 15, Article VII of the 1987
Constitution, must consist of the President signing an During the deliberations for the 1987 Constitution, then
appointee's appointment paper to a vacant office, the official Constitutional Commissioner (now retired Supreme Court
transmittal of the appointment paper (preferably through the Chief Justice) Hilario G. Davide, Jr. referred to this Court's
MRO), receipt of the appointment paper by the appointee, and ruling in Aytona and stated that his proposal seeks to prevent
acceptance of the appointment by the appointee evidenced by a President, whose term is about to end, from preempting his
his or her oath of office or his or her assumption to office. successor by appointing his own people to sensitive positions.

Aytona v. Castillo (Aytona)42 is the basis for Section 15, Article MR. DAVIDE: The idea of the proposal is that about the end of
VII of the 1987 Constitution. Aytona defined "midnight or last the term of the President, he may prolong his rule indirectly by
minute" appointments for Philippine appointing people to these sensitive positions, like the
jurisprudence.1âwphi1 President Carlos P. Garcia submitted commissions, the Ombudsman, the judiciary, so he could
on 29 December 1961, his last day in office, 350 appointments, perpetuate himself in power even beyond his term of office;
including that of Dominador R. Aytona for Central Bank therefore foreclosing the right of his successor to make
Governor. President Diosdado P. Macapagal assumed office appointments to these positions. We should realize that the
on 30 December 1961, and issued on 31 December 1961 term of the President is six years and under what we had voted
Administrative Order No. 2 recalling, withdrawing, and on, there is no reelection for him. Yet he can continue to rule
cancelling all appointments made by President Garcia after 13 the country through appointments made about the end of his
December 1961 (President Macapagal's proclamation date). term to these sensitive positions.44
President Macapagal appointed Andres V. Castillo as Central
Bank Governor on 1 January 1962. This Court dismissed The 1986 Constitutional Commission put a definite period, or
Aytona's quo warranto proceeding against Castillo, and upheld
an empirical value, on Aytona's intangible "stratagem to beat
Administrative Order No. 2's cancellation of the "midnight or the deadline," and also on the act of "preempting the
last minute" appointments. We wrote:
President's successor," which shows a lack of "good faith,
morality and propriety." Subject to only one exception,
x x x But the issuance of 350 appointments in one night and appointments made during this period are thus automatically
the planned induction of almost all of them a few hours before prohibited under the Constitution, regardless of the appointee's
the inauguration of the new President may, with some reason, qualifications or even of the President's motives. The period for
be regarded by the latter as an abuse of Presidential prohibited appointments covers two months before the
prerogatives, the steps taken being apparently a mere partisan elections until the end of the President's term. The
effort to fill all vacant positions irrespective of fitness and other Constitution, with a specific exception, ended the President's
conditions, and thereby to deprive the new administration of an power to appoint "two months immediately before the next
opportunity to make the corresponding appointments. presidential elections." For an appointment to be valid, it must
be made outside of the prohibited period or, failing that, fall
x x x Now it is hard to believe that in signing 350 appointments under the specified exception.
in one night, President Garcia exercised such "double care"
which was required and expected of him; and therefore, there The dissent insists that, during the prohibited period, an
seems to be force to the contention that these appointments appointment should be viewed in its "narrow sense." In its
fall beyond the intent and spirit of the constitutional provision narrow sense, an appointment is not a process, but is only an
granting to the Executive authority to issue ad interim "executive act that the President unequivocally exercises
appointments. pursuant to his discretion."45 The dissent makes acceptance of
the appointment inconsequential. The dissent holds that an
FINALS CONSTITUTION I ACJUCO 140

appointment is void if the appointment is made before the ban strictly applied on appointments made just before or during the
but the transmittal and acceptance are made after the ban. appointment ban.
However, the dissent holds that an appointment is valid, or
"efficacious," if the appointment and transmittal are made
In attempting to extricate itself from the obvious consequences
before the ban even if the acceptance is made after the ban. of its selective application, the dissent glaringly contradicts
In short, the dissent allows an appointment to take effect during itself:
the ban, as long as the President signed and transmitted the
appointment before the ban, even if the appointee never
received the appointment paper before the ban and accepted Thus, an acceptance is still necessary in order for the
the appointment only during the ban. appointee to validly assume his post and discharge the
functions of his new office, and thus make the appointment
effective. There can never be an instance where the
The dissent's view will lead to glaring absurdities. Allowing the appointment of an incumbent will automatically result in his
dissent's proposal that an appointment is complete merely resignation from his present post and his subsequent
upon the signing of an appointment paper and its transmittal, assumption of his new position; or where the President can
excluding the appointee's acceptance from the appointment simply remove an incumbent from his current office by
process, will lead to the absurdity that, in case of non- appointing him to another one. I stress that acceptance through
acceptance, the position is considered occupied and nobody
oath or any positive act is still indispensable before any
else may be appointed to it. Moreover, an incumbent public assumption of office may occur.46 (Emphasis added)
official, appointed to another public office by the President, will
automatically be deemed to occupy the new public office and
to have automatically resigned from his first office upon The dissent proposes that this Court ignore well-settled
transmittal of his appointment paper, even if he refuses to jurisprudence during the appointment ban, but apply the same
accept the new appointment. This will result in chaos in public jurisprudence outside of the appointment ban.
service.
[T]he well-settled rule in our jurisprudence, that an
Even worse, a President who is unhappy with an incumbent appointment is a process that begins with the selection by the
public official can simply appoint him to another public office, appointing power and ends with acceptance of the
effectively removing him from his first office without due appointment by the appointee, stands. As early as the 1949
process. The mere transmittal of his appointment paper will case of Lacson v. Romero, this Court laid down the rule that
remove the public official from office without due process and acceptance by the appointee is the last act needed to make an
even without cause, in violation of the Constitution. appointment complete. The Court reiterated this rule in the
1989 case of Javier v. Reyes. In the 1996 case of Garces v.
Court of Appeals, this Court emphasized that acceptance by
The dissent's proferred excuse (that the appointee is not the appointee is indispensable to complete an appointment.
alluded to in Section 15, Article VII) for its rejection of The 1999 case of Bermudez v. Executive Secretary, cited in
"acceptance by the appointee" as an integral part of the the ponencia, affirms this standing rule in our jurisdiction, to
appointment process ignores the reason for the limitation of the
wit:
President's power to appoint, which is .to prevent the outgoing
President from continuing to rule the country indirectly after the
end of his term. The 1986 Constitutional Commission installed "The appointment is deemed complete once the last act
a definite cut-off date as an objective and unbiased marker required of the appointing authority has been complied with
against which this once-in-every-six-years prohibition should and its acceptance thereafter by the appointee in order to
be measured. render it effective."47

The dissent's assertion that appointment should be viewed in The dissent's assertion creates a singular exception to the
its narrow sense (and is not a process) only during the well-settled doctrine that appointment is a process that begins
prohibited period is selective and time-based, and ignores well- with the signing of the appointment paper, followed by the
settled jurisprudence. For purposes of complying with the time transmittal and receipt of the appointment paper, and becomes
limit imposed by the appointment ban, the dissent' s position complete with the acceptance of the appointment. The dissent
cuts short the appointment process to the signing of the makes the singular exception that during the constitutionally
appointment paper and its transmittal, excluding the receipt of mandated ban on appointments, acceptance is not necessary
the appointment paper and acceptance of the appointment by to complete the appointment. The dissent gives no reason why
the appointee. this Court should make such singular exception, which is
contrary to the express provision of the Constitution prohibiting
the President from making appointments during the ban. The
The President exercises only one kind of appointing power. dissent's singular exception will allow the President, during the
There is no need to differentiate the exercise of the President's
ban on appointments, to remove from office incumbents
appointing power outside, just before, or during the without cause by simply appointing them to another office and
appointment ban. The Constitution allows the President to
transmitting the appointment papers the day before the ban
exercise the power of appointment during the period not begins, appointments that the incumbents cannot refuse
covered by the appointment ban, and disallows (subject to an because their acceptance is not required during the ban.
exception) the President from exercising the power of Adoption by this Court of the dissent's singular exception will
appointment during the period covered by the appointment certainly wreak havoc on the civil service.
ban. The concurrence of all steps in the appointment process
is admittedly required for appointments outside the
appointment ban. There is no justification whatsoever to The following elements should always concur in the making of
remove acceptance as a requirement in the appointment a valid (which should be understood as both complete and
process for appointments just before the start of the effective) appointment: (1) authority to appoint and evidence of
appointment ban, or during the appointment ban in the exercise of the authority; (2) transmittal of the appointment
appointments falling within the exception. The existence of the paper and evidence of the transmittal; (3) a vacant position at
appointment ban makes no difference in the power of the the time of appointment; and (4) receipt of the appointment
President to appoint; it is still the same power to appoint. In paper and acceptance of the appointment by the appointee
fact, considering the purpose of the appointment ban, the who possesses all the qualifications and none of the
concurrence of all steps in the appointment process must be disqualifications. The concurrence of all these elements should
FINALS CONSTITUTION I ACJUCO 141

always apply, regardless of when the appointment is made, a. Receive, record and screen all incoming
whether outside, just before, or during the appointment ban. correspondence, telegrams, documents and papers,
These steps in the appointment process should always concur and
and operate as a single process. There is no valid appointment
if the process lacks even one step. And, unlike the dissent's (1) Forward those of a personal and
proposal, there is no need to further distinguish between an unofficial nature to the President's Private
effective and an ineffective appointment when an appointment Office; and
is valid.

(2) Distribute those requiring action within


Appointing Authority the Office or requiring staff work prior to
presentation to the President to the
The President's exercise of his power to appoint officials is appropriate units within the Office.
provided for in the Constitution and laws. 48 Discretion is an
integral part in the exercise of the power of
b. Follow up on correspondence forwarded to entities
appointment.49 Considering that appointment calls for a outside the Office to assure that prompt replies are
selection, the appointing power necessarily exercises a made and copies thereof furnished the Office.
discretion. According to Woodbury, J., "the choice of a person
to fill an office constitutes the essence of his appointment," and
Mr. Justice Malcolm adds that an "[a]ppointment to office is c. Dispatch outgoing correspondence and telegrams.
intrinsically an executive act involving the exercise of
discretion." In Pamantasan ng Lungsod ng Maynila v. d. Have custody of records of the Office, except
Intermediate Appellate Court we held: personal papers of the President, and keep them in
such condition as to meet the documentary and
The power to appoint is, in essence, discretionary. The reference requirements of the Office.
appointing power has the right of choice which he may exercise
freely according to his judgment, deciding for himself who is e. Keep and maintain a filing and records system for
best qualified among those who have the necessary acts, memoranda, orders, circulars, correspondence
qualifications and eligibilities. It is a prerogative of the and other documents affecting the Office for ready
appointing power x x x x reference and use.

Indeed, the power of choice is the heart of the power to f. Issue certified true copies of documents on file in
appoint. Appointment involves an exercise of discretion of the Division m accordance with prevailing standard
whom to appoint; it is not a ministerial act of issuing operating procedure.
appointment papers to the appointee. In other words, the
choice of the appointee is a fundamental component of the
appointing power. g. Keep a separate record of communications or
documents of confidential nature.
Hence, when Congress clothes the President with the power
to appoint an officer, it (Congress) cannot at the same time limit h. Have custody of the Great Seal of the Republic of
the choice of the President to only one candidate. Once the the Philippines.
power of appointment is conferred on the President, such
conferment necessarily carries the discretion of whom to i. Prepare and submit to the approving authority,
appoint. Even on the pretext of prescribing the qualifications of periodic disposition schedules of non-current records
the officer, Congress may not abuse such power as to divest which have no historical, legal and/or claim value.
the appointing authority, directly or indirectly, of his discretion
to pick his own choice. Consequently, when the qualifications
prescribed by Congress can only be met by one individual, j. With the approval of the Executive Secretary, assist
such enactment effectively eliminates the discretion of the other offices in the installation or improvement of
appointing power to choose and constitutes an irregular their records management system; and
restriction on the power of appointment. 50
k. Give instructions or deliver lectures and conduct
Transmittal practical training to in-service trainees from other
offices and to students from educational institutions
on records management.51
It is not enough that the President signs the appointment
paper. There should be evidence that the President intended
the appointment paper to be issued. It could happen that an The Records Division was elevated to an Office in
appointment paper may be dated and signed by the President 1975, with the addition of the following functions:
months before the appointment ban, but never left his locked
drawer for the entirety of his term. Release of the appointment 1. Maintain and control vital documents and
paper through the MRO is an unequivocal act that signifies the essential records to support the functions
President's intent of its issuance. of the OP in its day to day activities;

The MRO was created by Memorandum Order No. 1, Series of 2. Monitor the flow of communications' from
1958, Governing the Organization and Functions of the their time of receipt up to their dispatch;
Executive Office and General Matters of Procedure Therein.
Initially called the Records Division, the MRO functioned as an
3. Service the documentary, information
administrative unit of the Executive Office. Memorandum
and reference requirements of top
Order No. 1 assigned the following functions:
management and action officers of the OP,
and the reference and research needs of
FINALS CONSTITUTION I ACJUCO 142

other government agencies and the release of the original copy of the appointment paper signed
general public; by the President.

4. Ensure the proper storage, Q: Can you briefly illustrate the procedure for the release of the
maintenance, protection and preservation original copy of the appointment paper signed by the
of vital and presidential documents, and the President?
prompt disposal of obsolete and valueless
records;
A: After an appointment paper is signed by the President, the
Office of the Executive Secretary (OES) forwards the
5. Effect the prompt appointment paper bearing the stamp mark, barcode, and
publication/dissemination of laws, hologram of the Office of the President, together with a
presidential issuances and classified transmittal letter, to the MRO for official release. Within the
documents; same day, the MRO sends the original copy of the appointment
paper together with the transmittal letter and a delivery receipt
6. Provide computerized integrated records which contains appropriate spaces for the name of the
management support services for easy addressee, the date released, and the date received by the
reference and retrieval of data and addressee. Only a photocopy of the appointment is retained for
information; and the MRO's official file.

7. To be able to represent the OP and OP Q: What is the basis for the process you just discussed?
officials in response to Subpoena Duces
Tecum and Testificandum served by courts A: The Service Guide of the MRO.
and other investigating bodies.52
xxxx
For purposes of verification of the appointment paper's
existence and authenticity, the appointment paper must bear
Q: What is the legal basis for the issuance of the MRO Service
the security marks (i.e., handwritten signature of the President, Guide, if any?
bar code, etc.) and must be accompanied by a transmittal letter
from the MRO.
A: The MRO Service Guide was issued pursuant to
Memorandum Circular No. 35, Series of 2003 and
The testimony of Mr. Mariani to Dimaandal, Director IV of the Memorandum Circular No. 133, Series of 2007.
MRO, underscores the purpose of the release of papers
through his office.
xxxx
Q: What are the functions of the MRO?
Q: Do you exercise any discretion in the release of documents
forwarded to the MRO for transmittal to various offices?
A: The MRO is mandated under Memorandum Order No. 1,
series of 1958 to (1) receive, record, and screen all incoming
correspondence, telegrams, documents, and papers; (2) follow A: No. We are mandated to immediately release all documents
up on correspondence forwarded to entities outside the Office and correspondence forwarded to us for transmittal.
of the President ("OP") to assure that prompt replies are made
and copies thereof furnished the OP; (3) timely dispatch all Q: If a document is forwarded by the OES to the MRO today,
outgoing documents and correspondence; (4) have custody of when is it officially released by the MRO to the department or
records of the OP, except personal papers of the President, agency concerned?
and keep them in such condition as to meet the documentary
and reference requirements of the Office; (5) keep and
maintain a filing and records system for Acts, Memoranda, A: The document is released within the day by the MRO if the
Orders, Circulars, correspondence, and other pertinent addressee is within Metro Manila. For example, in the case of
documents for ready reference and use; ( 6) issue certified the appointment paper of Dindo Venturanza, the OES
copies of documents on file as requested and in accordance forwarded to the MRO on March 12, 2010 his original
with prevailing standard operating procedures; (7) maintain appointment paper dated February 23, 2010 and the
and control vital documents and essential records to support transmittal letter dated March 9, 2010 prepared by the OES.
the OP in its day-to-day activities; (8) monitor the flow of The MRO released his appointment paper on the same day or
communications from the time of receipt up to their dispatch; on March 12, 2010, and was also received by the DOJ on
and (9) other related functions. March 12, 2010 as shown by the delivery receipt.

xxxx Q: What is the effect if a document is released by an office or


department within Malacañan without going through the MRO?
Q: As you previously mentioned, the MRO is the custodian of
all documents emanating from Malacañang pursuant to its A: If a document does not pass through the MRO contrary to
mandate under Memorandum Order No. 1, Series of 1958. Is established procedure, the MRO cannot issue a certified true
the MRO required to follow a specific procedure in dispatching copy of the same because as far as the MRO is concerned, it
outgoing documents? does not exist in our official records, hence, not an official
document from the Malacañang. There is no way of verifying
the document's existence and authenticity unless the
A: Yes. document is on file with the MRO even if the person who claims
to have in his possession a genuine document furnished to him
Q: Is this procedure observed for the release of an appointment personally by the President. As a matter of fact, it is only the
paper signed by the President? A: Yes. It is observed for the MRO which is authorized to issue certified true copies of
documents emanating from Malacañan being the official
FINALS CONSTITUTION I ACJUCO 143

custodian and central repository of said documents. Not even Q: Do you have any evidence to show this steep rise?
the OES can issue a certified true copy of documents prepared
by them. A: Yes. I prepared a Certification showing these statistics and
the graphical representation thereof.
Q: Why do you say that, Mr. Witness?
Q: If those documents will be shown to you, will you be able to
A: Because the MRO is the so-called "gatekeeper" of the recognize them?
Malacañang Palace. All incoming and outgoing documents
and correspondence must pass through the MRO. As the A: Yes.
official custodian, the MRO is in charge of the official release
of documents.
Q: I am showing you a Certification containing the number of
presidential appointees per month since January 2009 until
Q: What if an appointment paper was faxed by the Office of the June 2010, and a graphical representation thereof. Can you go
Executive Secretary to the appointee, is that considered an over these documents and tell us the relation of these
official release by the MRO? documents to the ones you previously mentioned?

A: No. It is still the MRO which will furnish the original copy of A: These are [sic] the Certification with the table of statistics I
the appointment paper to the appointee. That appointment prepared after we counted the appointments, as well as the
paper is, at best, only an "advanced copy." graph thereof.

Q: Assuming the MRO has already received the original xxxx


appointment paper signed by the President together with the
transmittal letter prepared by the OES, you said that the MRO
is bound to transmit these documents immediately, that is, on Q: Out of the more than 800 appointees made in March 2010,
the same day? how many appointment papers and transmittal letters were
released through the MRO?
A: Yes.
A: Only 133 appointment papers were released through the
MRO.
Q: Were there instances when the President, after the original
appointment paper has already been forwarded to the MRO,
recalls the appointment and directs the MRO not to transmit Q: In some of these transmittal letters and appointment papers
the documents? which were not released through the MRO but apparently
through the OES, there were portions on the stamp of the OES
which supposedly indicated the date and time it was actually
A: Yes, there were such instances. received by the agency or office concerned but were curiously
left blank, is this regular or irregular?
Q: How about if the document was already transmitted by the
MRO, was there any instance when it was directed to recall the A: It is highly irregular.
appointment and retrieve the documents already transmitted?
A: Yes, but only in a few instances. Sometimes, when the MRO
messenger is already in transit or while he is already in the Q: Why do you say so?
agency or office concerned, we get a call to hold the delivery.
Q: You previously outlined the procedure governing the A: Usually, if the document released by the MRO, the delivery
transmittal of original copies of appointment papers to the receipt attached to the transmittal letter is filled out completely
agency or office concerned. Would you know if this procedure because the dates when the original appointment papers were
was followed by previous administrations? actually received are very material. It is a standard operating
procedure for the MRO personnel to ask the person receiving
A: Yes. Since I started working in the MRO in 1976, the the documents to write his/her name, his signature, and the
procedure has been followed. However, it was unusually date and time when he/she received it.
disregarded when the appointments numbering more than 800
were made by then President Arroyo in March 2010. The MRO Q: So, insofar as these transmittal letters and appointment
did not even know about some of these appointments and we papers apparently released by the OES are concerned, what
were surprised when we learned about them in the is the actual date when the agency or the appointee concerned
newspapers. received it?

Q: You mentioned that then President Arroyo appointed more A: I cannot answer. There is no way of knowing when they
than 800 persons in the month of March alone. How were you were actually received because the date and time were
able to determine this number? deliberately or inadvertently left blank.

A: My staff counted all the appointments made by then Q: Can we say that the date appearing on the face of the
President An-oyo within the period starting January 2009 until transmittal letters or the appointment papers is the actual date
June 2010. when it was released by the OES?

Q: What did you notice, if any, about these appointments? A: We cannot say that for sure. That is why it is very unusual
that the person who received these documents did not indicate
A: There was a steep rise in the number of appointments made the date and time when it was received because these details
by then President Arroyo in the month of March 2010 are very important.53
compared to the other months.
FINALS CONSTITUTION I ACJUCO 144

The MRO's exercise of its mandate does not prohibit the (b) The portion rubber stamped by the
President or the Executive Secretary from giving the Office of the Executive Secretary located at
appointment paper directly to the appointee. However, a the back of the last page of the letter
problem may arise if an appointment paper is not coursed showing receipt by Ma. Carissa O.
through the MRO and the appointment paper is lost or the Coscuella with blank spaces for the date
appointment is questioned. The appointee would then have to and time when it was actually received as
prove that the appointment paper was directly given to him. Exhibit "2-L-3";

Dimaandal's counsel made this manifestation about xxxx


petitioners' appointment papers and their transmittal:
8. A) The Transmittal Letter pertinent to the
Your Honors, we respectfully request for the following appointments of x x x FRANCISCA
markings to be made: BESTOYONG-ROSQUITA dated March 8,
2010 but turned over to the MRO on May
1. A) The Transmittal Letter pertinent to the 13, 2010 as Exhibit "2-T" for the
appointment of petitioner DINDO VENTURANZA respondents;
dated March 9, 2010 as Exhibit "2-F" for the
respondents; xxxx

B) The delivery receipt attached in front of (c) The portion with the name
the letter bearing the date March 12, 2010 "FRANCISCA BESTOYONGROSQUIT A"
as Exhibit "2-F-l"; as "Commissioner, Representing Region I
and the Cordilleras" as Exhibit "2-T-3·"
C) The Appointment Paper of DINDO
VENTURANZA dated February 23, 2010 (d) The portion rubber stamped by the
as Exhibit "2-G" for the respondents; Office of the Executive Secretary at the
back thereof showing receipt by Masli A.
2. A) The Transmittal Letter pertinent to the Quilaman of NCIP-QC on March 15, 2010
appointment of CHELOY E. VELICARIA-GARAFIL as Exhibit "2-T-4;"
turned over to the MRO on May 13, 2010 consisting
of seven (7) pages as Exhibits "2-H," "2-H-l," "2-H-2," xxxx
"2-H-3," "2-H-4," "2-H-5," and "2-H-6" respectively for
the respondents; D) The Appointment Paper of FRANCISCA
BESTOYONGROSQUIT A dated March 5, 2010 as
i. The portion with the name Exhibit "2-W" for the respondents;
"CHELOY E.
VELICARIAGARAFIL" as "State 9. A) The Transmittal Letter pertinent to the
Solicitor II, Office of the Solicitor appointment of IRMA A. VILLANUEVA as
General" located on the first page Administrator for Visayas, Board of Administrators,
of the letter as Exhibit "2-H-7;" Cooperative Development Authority, Department of
Finance dated March 8, 2010 as Exhibit "2-X" for the
ii. The portion rubber stamped by respondents;
the Office of the Executive
Secretary located at the back of (a) The portion rubber stamped by the
the last page of the -letter Office of the Executive Secretary at the
showing receipt by the DOJ with back thereof showing receipt by DOF with
blank spaces for the date and
blank spaces for the date and time when it
time when it was actually was actually received as Exhibit "2-X-1 ;"
received as Exhibit "2-H-8;"

B) The Appointment Paper of IRMA A. VILLANUEVA


B) The Appointment Paper of CHELOY E. dated March 3, 2010 as Exhibit "2-Y" for the
VELICARIA-GARAFIL dated March 5, respondents.54
2010 as Exhibit "2-I" for the respondents;

The testimony of Ellenita G. Gatbunton, Division Chief of File


xxxx Maintenance and Retrieval Division of the MRO, supports
Dimaandal's counsel's manifestation that the transmittal of
4. A) The Transmittal Letter pertinent to the petitioners' appointment papers is questionable.
appointment of EDDIE U. TAMONDONG dated 8
March 2010 but turned over to the MRO only on May Q: In the case of Cheloy E. Velicaria-Garafil, who was
6, 2010 consisting of two (2) pages as Exhibits "2-L" appointed as State Solicitor II of the Office of the Solicitor
and "2-L-l" respectively for the respondents;
General, was her appointment paper released through the
MRO?
(a) The portion with the name "EDDIE U.
TAMONDONG" as "Member, representing A: No. Her appointment paper dated March 5, 2010, with its
the Private Sector, Board of Directors" as
corresponding transmittal letter, was merely turned over to the
Exhibit "2-L-2"; MRO on May 13, 2010. The transmittal letter that was turned
over to the MRO was already stamped "released" by the Office
FINALS CONSTITUTION I ACJUCO 145

of the Executive Secretary, but the date and time as to when it were indispensable, then a loss of the original appointment
was actually received were unusually left blank. paper, which could be brought about by negligence, accident,
fraud, fire or theft, corresponds to a loss of the
office.56 However, in case of loss of the original appointment
Q: What is your basis?
paper, the appointment must be evidenced by a certified true
copy issued by the proper office, in this case the MRO. Vacant
A: The transmittal letter and appointment paper turned over to Position
the MRO.
An appointment can be made only to a vacant office. An
xxxx appointment cannot be made to an occupied office. The
incumbent must first be legally removed, or his appointment
Q: In the case of Eddie U. Tamondong, who was appointed as validly terminated, before one could be validly installed to
member of the Board of Directors of Subic Bay Metropolitan succeed him.57
Authority, was her [sic] appointment paper released through
the MRO? To illustrate: in Lacson v. Romero,58 Antonio Lacson (Lacson)
occupied the post of provincial fiscal of Negros Oriental. He
A: No. His appointment paper dated March 1, 2010, with its was later nominated and confirmed as provincial fiscal of
corresponding transmittal letter, was merely turned over to the Tarlac. The President nominated and the Commission on
MRO on May 6, 2010. The transmittal letter that was turned Appointments confirmed Honorio Romero (Romero) as
over to the MRO was already stamped "released" by the Office provincial fiscal of Negros Oriental as Lacson's replacement.
of the Executive Secretary, but the date and time as to when it Romero took his oath of office, but Lacson neither accepted
was actually received were unusually left blank. the appointment nor assumed office as provincial fiscal of
Tarlac. This Court ruled that Lacson remained as provincial
fiscal of Negros Oriental, having declined the appointment as
Q: What is your basis? provincial fiscal of Tarlac. There was no vacancy to which
Romero could be legally appointed; hence, Romero's
A: The transmittal letter and appointment paper turned over to appointment as provincial fiscal ofNegros Oriental vice Lacson
the MRO. was invalid.

xxxx The appointment to a government post like that of provincial


fiscal to be complete involves several steps. First, comes the
nomination by the President. Then to make that nomination
Q: In the case of Francisca Bestoyong-Resquita who was valid and permanent, the Commission on Appointments of the
appointed as Commissioner of the National Commission on Legislature has to confirm said nomination. The last step is the
Indigenous Peoples, representing Region 1 and the acceptance thereof by the appointee by his assumption of
Cordilleras, was her appointment paper released thru the office. The first two steps, nomination and confirmation,
MRO? constitute a mere offer of a post. They are acts of the Executive
and Legislative departments of the Government. But the last
A: No. Her appointment paper dated March 5, 2010, with its necessary step to make the appointment complete and
corresponding transmittal letter, was merely turned over to the effective rests solely with the appointee himself. He may or he
MRO on May 13, 2010. The transmittal letter that was turned may not accept the appointment or nomination. As held in the
over to the MRO was already stamped "released" by the Office case of Borromeo vs. Mariano, 41 Phil. 327, "there is no power
of the Executive Secretary and received on March 15, 2010. in this country which can compel a man to accept an office."
Consequently, since Lacson has declined to accept his
appointment as provincial fiscal of Tarlac and no one can
Q: What is your basis?
compel him to do so, then he continues as provincial fiscal of
Negros Oriental and no vacancy in said office was created,
A: The transmittal letter and appointment paper turned over to unless Lacson had been lawfully removed as such fiscal of
the MRO. Negros Oriental.59

xxxx Paragraph (b ), Section 1 of EO 2 considered as midnight


appointments those appointments to offices that will only be
Q: In the case of Irma A. Villanueva who was appointed as vacant on or after 11 March 2010 even though the
Administrator for Visayas of the Cooperative Development appointments are made prior to 11 March 2010. EO 2
Authority, was her appointment paper released thru the MRO? remained faithful to the intent of Section 15, Article VII of the
1987 Constitution: the outgoing President is prevented from
continuing to rule the country indirectly after the end of his
A: No. Her appointment paper dated March 3, 2010, with its term.
corresponding transmittal letter, was merely turned over to the
MRO on May 4, 2010. The transmittal letter that was turned
over to the MRO was already stamped "released" by the Office Acceptance by the Qualified Appointee
of the Executive Secretary, but the date and time as to when it
was actually received were unusually left blank. Acceptance is indispensable to complete an appointment.
Assuming office and taking the oath amount to acceptance of
Q: What is your basis? the appointment.60 An oath of office is a qualifying requirement
for a public office, a prerequisite to the full investiture of the
office.61
A: The transmittal letter and appointment paper turned over to
the MR0.55
Javier v. Reyes62 is instructive in showing how acceptance is
indispensable to complete an appointment. On 7 November
The possession of the original appointment paper is not 1967, petitioner Isidro M. Javier (Javier) was appointed by then
indispensable to authorize an appointee to assume office. If it Mayor Victorino B. Aldaba as the Chief of Police of Malolos,
FINALS CONSTITUTION I ACJUCO 146

Bulacan. The Municipal Council confirmed and approved


Javier's appointment on the same date. Javier took his oath of
office on 8 November 1967, and subsequently discharged the
rights, prerogatives, and duties of the office. On 3 January
1968, while the approval of Javier's appointment was pending
with the CSC, respondent Purificacion C. Reyes (Reyes), as
the new mayor of Malolos, sent to the . CSC a letter to recall
Javier's appointment. Reyes also designated Police Lt.
Romualdo F. Clemente as Officer-in-Charge of the police
department. The CSC approved Javier's appointment as
permanent on 2 May 1968, and even directed Reyes to
reinstate Javier. Reyes, on the other hand, pointed to the
appointment of Bayani Bernardo as Chief of Police of Malolos,
Bulacan on 4 September 1967. This Court ruled that Javier's
appointment prevailed over that of Bernardo. It cannot be said
that Bernardo accepted his appointment because he never
assumed office or took his oath.

Excluding the act of acceptance from the appointment process


leads us to the very evil which we seek to avoid (i.e., antedating
of appointments). Excluding the act of acceptance will only
provide more occasions to honor the Constitutional provision
in the breach. The inclusion of acceptance by the appointee as
an integral part of the entire appointment process prevents the
abuse of the Presidential power to appoint. It is relatively easy
to antedate appointment papers and make it appear that they
were issued prior to the appointment ban, but it is more difficult
to simulate the entire appointment process up until acceptance
by the appointee.

Petitioners have failed to show compliance with all four


elements of a valid appointment. They cannot prove with
certainty that their appointment papers were transmitted before
the appointment ban took effect. On the other hand, petitioners
admit that they took their oaths of office during the appointment
ban.

Petitioners have failed to raise any valid ground for the Court
to declare EO 2, or any part of it, unconstitutional.
Consequently, EO 2 remains valid and constitutional.

WHEREFORE, the petitions in G.R. Nos. 203372, 206290, and


212030 are DENIED, and the petition in G.R. No. 209138 is
DISMISSED. The appointments of petitioners Atty. Cheloy E.
Velicaria-Garafil (G.R. No. 203372), Atty. Dindo G. Venturanza
(G.R. No. 206290), Irma A. Villanueva, and Francisca B.
Rosquita (G.R. No. 209138), and Atty. Eddie U. Tamondong
(G.R. No. 212030) are declared VOID. We DECLARE that
Executive Order No. 2 dated 30 July 2010 is VALID and
CONSTITUTIONAL.

SO ORDERED
FINALS CONSTITUTION I ACJUCO 147

[G.R. No. 131136. February 28, 2001] Ayala, and Flordeliza Oriazel, filed with the CSC a claim for
payment of their salaries, alleging that although their
CONRADO L. DE RAMA, petitioner, vs. THE COURT OF appointments were declared permanent by Conrado Gulim,
APPEALS (NINTH DIVISION, THE CIVIL SERVICE Director II of the CSC Field Office based in Quezon, petitioner
COMMISSION), ELADIO MARTINEZ, DIVINO DE JESUS, de Rama withheld the payment of their salaries and benefits
MORELL AYALA, ARISTEO CATALLA, DAISY PORTA, pursuant to Office Order No. 95-01, which was issued on June
FLORDELIZA ORIASEL, GRACIELA GLORY, FELECIDAD 30, 1995, wherein the appointments of the said fourteen (14)
ORINDAY, MA. PETRA MUFFET LUCE, ELSA MARINO, employees were recalled.
BERNARDITA MENDOZA, JANE MACATANGAY, ADELFO
GLODOVIZA and FLORINO RAMOS, respondents. Based on the documents submitted by Marino, Ayala and
Oriazel, the Legal and Quasi-Judicial Division of the CSC
issued an Order[2] finding that since the claimants-employees
DECISION had assumed their respective positions and performed their
duties pursuant to their appointments, they are therefore
YNARES-SANTIAGO, J.:
entitled to receive the salaries and benefits appurtenant to their
positions. Citing Rule V, Section 10 of the Omnibus
Upon his assumption to the position of Mayor of Rules[3] which provides, in part, that if the appointee has
Pagbilao, Quezon, petitioner Conrado L. de Rama wrote a assumed the duties of the position, he shall be entitled to
letter dated July 13, 1995 to the Civil Service Commission (or receive his salary at once without awaiting the approval of his
CSC), seeking the recall of the appointments of fourteen (14) appointment by the Commission, the CSC Legal and Quasi-
municipal employees, namely: Judicial Division ruled that the said employees cannot be
deprived of their salaries and benefits by the unilateral act of
NAME POSITION DATE OF the newly-assumed mayor.
APPOINTMENT
On April 30, 1996, the CSC denied petitioners request for
Eladio Martinez Registration Officer I June 1, 1995 the recall of the appointments of the fourteen employees, for
lack of merit. The CSC also cited Rule V, Sections 9 and 10 of
the Omnibus Rules, and declared that the appointments of the
Divino de Jesus Bookbinder III June 1, 1995 said employees were issued in accordance with pertinent
laws. Thus, the same were effective immediately, and cannot
Morell Ayala Accounting Clerk III June 16, 1995 be withdrawn or revoked by the appointing authority until
disapproved by the CSC. The CSC also dismissed petitioners
allegation that these were midnight appointments, pointing out
Daisy Porta Clerk IV June 27, 1995 that the Constitutional provision relied upon by petitioner
prohibits only those appointments made by an outgoing
Aristeo Catalla Gen. Services Officer June 19, 1995 President and cannot be made to apply to local elective
officials. Thus, the CSC opined, the appointing authority can
validly issue appointments until his term has expired, as long
Elsa Marino Mun. Agriculturist June 19, 1995
as the appointee meets the qualification standards for the
position.[4]
Gracella Glory Bookkeeper II June 27, 1995
The CSC upheld the validity of the appointments on the
ground that they had already been approved by the Head of
Ma. Petra Muffet Lucce Accounting Clerk III June 27, 1995 the CSC Field Office in Lucena City, and for petitioners failure
to present evidence that would warrant the revocation or recall
Felicidad Orindag Accounting Clerk II June 27, 1995 of the said appointments.
Petitioner moved for the reconsideration of the CSCs
Bernardita Mendoza Agricultural Technologist June 27, 1995 Resolution, as well as the Order of the CSC Legal and Quasi-
Judicial Division, averring that the CSC was without
Flordeliza Oriazel Clerk I June 27, 1995 jurisdiction: (1) to refuse to revoke the subject appointments;
and (2) to uphold the validity of said appointments, even
assuming that there was failure to present evidence that would
Jane Macatangay Day Care Worker I June 27, 1995 prove that these appointments contravened existing laws or
rules. He also posited that the CSC erred in finding the
Adolfo Glodoviza Utility Worker II June 27, 1995 appointments valid despite the existence of circumstances
showing that the same were fraudulently issued and
processed.
Florencio Ramos Utility Foreman June 27, 1995[1]
On November 21, 1996, the CSC denied petitioners
Petitioner de Rama justified his recall request on the
motion for reconsideration. The CSC reiterated its ruling that:
allegation that the appointments of the said employees were
midnight appointments of the former mayor, Ma. Evelyn S.
Abeja, done in violation of Article VII, Section 15 of the 1987 In the absence of any showing that these alleged midnight
Constitution, which provides: appointments were defective in form and in substance, nor is
there evidence presented to show that subject appointments
were issued in contravention of law or rules, these
Section 15. Two months immediately before the next
appointments are deemed valid and in effect.
presidential elections and up to the end of his term,
a President or Acting President shall not make appointments,
except temporary appointments to executive positions when xxxxxxxxx
continued vacancies therein will prejudice public service or
endanger public safety. (Underscoring supplied) Mayor de Rama failed to present evidence that subject
appointments should be revoked or recalled because of any of
While the matter was pending before the CSC, three of the abovementioned grounds enumerated. As a matter of fact,
the above-named employees, namely: Elsa Marino, Morell said appointments were even approved by the Head, Civil
FINALS CONSTITUTION I ACJUCO 148

Service Field Office, Lucena City when submitted for I. No screening process and no criteria were adopted by the
attestation. In the absence of a clear showing that these Personnel Selection Board in nominating the respondents;
appointments were issued in violation of any of these grounds,
the Commission has no other recourse but to uphold their
II. No posting in three (3) conspicuous public places of notice
validity. (Underscoring supplied) of vacancy as required by the rules and the law;

The CSC also cited the Supreme Court ruling in the case III. Merit and fitness requirements were not observed by the
of Aquino v. Civil Service Commission[5] wherein this Court
selection board and by the appointing authority as required by
held that: the Civil Service rules;

It is well-settled that once an appointment is issued and the IV. Petitioner has valid grounds to recall the appointments of
moment the appointee assumes a position in the civil service
respondents.[8]
under a completed appointment, he acquires a legal, not
merely equitable right (to the position), which is protected not
only by statute, but also by the Constitution, and cannot be In a Resolution dated October 20, 1997, the Court of
taken away from him either by revocation of the appointment, Appeals denied the motion for reconsideration.
or by removal, except for cause, and with previous notice and
hearing. (Emphasis supplied) Hence, the instant petition for review on certiorari on the
following assigned errors:

Consequently, petitioner filed a petition for review before


the Court of Appeals, arguing that the CSC arrived at the I. THE PUBLIC RESPONDENT COURT OF APPEALS,
erroneous conclusion after it ignored his supplement to the GRAVELY AND SERIOUSLY ERRED IN FINDING THAT THE
consolidated appeal and motion for reconsideration wherein he CIVIL SERVICE COMMISSION WAS CORRECT IN NOT
laid out evidence showing that the subject appointments were UPHOLDING THE PETITIONERS RECALL OF THE
obtained through fraud. APPOINTMENTS OF PRIVATE RESPONDENTS IN THE
FACE OF FRAUD AND VIOLATION OF RULES AND LAWS
After reviewing the facts and issues raised by petitioner, ON ISSUANCE OF APPOINTMENTS.
the Court of Appeals issued a Resolution[6] dated May 16, 1997
which held that there was no abuse of the power of II. THE PUBLIC RESPONDENT COURT OF APPEALS
appointment on the part of the outgoing mayor. SERIOUSLY ERRED IN FINDING THAT THE PARTICULAR
GROUNDS NAMELY:
The Court of Appeals further held that the fact that the
appointments of Marino, Ayala, Ramos, Mendoza and Glory
were made more than four (4) months after the publication of I. No screening process and no criteria were
the vacancies to which they were appointed is of no adopted by the Personnel Selection Board
moment. Setting aside petitioners suppositions, the Court of in nominating the respondents;
Appeals ruled that Republic Act No. 7041 does not provide that
every appointment to the local government service must be II. No posting in three (3) conspicuous public
made within four (4) months from publication of the places of notice of vacancy as required by
vacancies. It cited Section 80 of said Act, to wit: the rules and the law;

Section 80. Public Notice of Vacancy: Personnel Selection III. Merit and fitness requirements were not
Board. (a) Whenever a local chief executive decides to fill a observed by the selection board and by the
vacant career position, there shall be posted notices of the appointing authority as required by the Civil
vacancy in at least three (3) conspicuous public places in the Service rules;
local government unit concerned for a period of not less than
fifteen (15) days.
IV. Petitioner has valid grounds to recall the
appointments of respondents.
(b) There shall be established in every province, city or
municipality a personnel selection board to assist the local
chief executive in the judicious and objective selection of ARE NEW ISSUES BECAUSE THE GROUNDS FOR
personnel for employment as well as for promotion, and in the RECALL OF THE APPOINTMENTS BY THE PETITIONER
formulation of such policies as would contribute to employee WERE PRECISELY THE VIOLATION OF LAWS AND
welfare. REGULATIONS ON ISSUANCE OF APPOINTMENTS AS
RAISED BEFORE THE RESPONDENT CIVIL SERVICE
COMMISSION.
(c) The personnel selection board shall be headed by the local
sanggunian concerned. A representative of the Civil Service
Commission, if any, and the personnel officer of the local Petitioner assails the findings of both the CSC and the
government unit concerned shall be ex officio members of the Court of Appeals for being contrary to law and not being
board.[7] supported by the evidence on record.

This argument is too specious to be given credence. The


Likewise, neither did the CSCs own Circular Order No. records reveal that when the petitioner brought the matter of
27, Section 7, Series of 1991, require that vacant positions recalling the appointments of the fourteen (14) private
published in a government quarterly must be filled up before respondents before the CSC, the only reason he cited to justify
the advent of the succeeding quarter. his action was that these were midnight appointments that are
forbidden under Article VII, Section 15 of the
On the basis of all the foregoing findings, the Court of Constitution. However, the CSC ruled, and correctly so, that
Appeals denied for lack of merit the petition for review. the said prohibition applies only to presidential
Petitioner filed a motion for reconsideration arguing that appointments. In truth and in fact, there is no law that prohibits
the appellate court erred in upholding the CSCs resolutions local elective officials from making appointments during the
despite the following defects: last days of his or her tenure. Petitioner certainly did not raise
FINALS CONSTITUTION I ACJUCO 149

the issue of fraud on the part of the outgoing mayor who made the supplemental pleading, or even to consider the averments
the appointments. Neither did he allege that the said therein.
appointments were tainted by irregularities or anomalies that
breached laws and regulations governing appointments. His Secondly, a supplemental pleading must state
solitary reason for recalling these appointments was that they transactions, occurrences or events which took place since the
were, to his personal belief, midnight appointments which the time the pleading sought to be supplemented was filed. In the
outgoing mayor had no authority to make. instant case, petitioner alleged fraud and irregularities that
supposedly occurred contemporaneous to the execution of the
Even in petitioners consolidated appeal and motion for appointments. They should have been raised at the very first
reconsideration, he did not make any assertion that these opportunity. They are not new events which petitioner could
appointments were violative of civil service rules and not have originally included as grounds for the recall of the
procedures. Indeed, he harped on the CSCs alleged lack of appointments.
jurisdiction to refuse to recall the subject appointments. After
first invoking the authority of the CSC to approve or affirm his Accordingly, the CSC, as well as the Court of Appeals,
act, he then contradicted himself by arguing that the CSC had found that the allegations in his supplemental pleading did not
no jurisdiction to do so, but only after the CSC had ruled that constitute new evidence that can be the proper subject of a
the recall was without legal basis. He emphasized that he supplemental pleading.These were old facts and issues which
alone has sole discretion to appoint and recall the appointment he failed to raise earlier. Consequently, the CSC and the Court
of municipal employees, an authority which, he stressed, the of Appeals did not err in refusing to give credence to the
CSC cannot usurp. Yet, nowhere in said pleading did he cite supplemental pleading.
any other ground, much less present proof that would warrant
Be that as it may, these alleged irregularities were
the recall of said appointments.
considered by the CSC and the Court of Appeals as new
Perhaps realizing the weakness of his arguments, albeit issues which were raised for the first time on appeal. It is rather
belatedly, petitioner filed a supplement to the appeal and too late for petitioner to raise these issues for the first time on
motion for reconsideration where, for the very first time, he appeal. It is well-settled that issues or questions of fact cannot
alleged that the appointments were fraught with irregularities be raised for the first time on appeal. [10] We have consistently
for failing to comply with CSC rules and held that matters, theories or arguments not brought out in the
regulations. Nevertheless, the CSC overruled petitioners original proceedings cannot be considered on review or appeal
assertions, holding that no new evidence had been presented where they are raised for the first time.[11] To consider the
to warrant a reversal of its earlier resolution. alleged facts and arguments raised belatedly in the
supplemental pleading to the appeal at this very late stage in
Thus, in a petition for review before the Court of Appeals, the proceedings would amount to trampling on the basic
petitioner questioned the CSCs conclusion because it had principles of fair play, justice and due process. [12]
ignored the allegations and documents he presented in the
supplement to his earlier consolidated appeal and motion for The grounds for the recall of the appointments that
reconsideration. He argued that these form part of the records petitioner raised in his supplemental pleading to the
of the case and that the CSC erred in failing to consider the consolidated appeal and motion for reconsideration are that:
assertions he raised therein. The appellate court, however, (1) the rules on screening of applicants based on adopted
agreed with the CSC when it ruled that the documents criteria were not followed; (2) there was no proper posting of
presented by petitioner in the supplemental pleading did not notice of vacancy; and (3) the merit and fitness requirements
constitute new evidence that would convince the CSC to set by the civil service rules were not observed. These are
reverse its earlier ruling.In fine, the Court of Appeals, as did the grounds that he could have stated in his order of recall, but
CSC, simply dismissed petitioners allegations and documents which he did not. Neither did he raise said grounds in his
attached to the supplemental pleading for they did not original appeal, but only by way of a supplemental
constitute new evidence that a court, board or tribunal may pleading. Failure of the petitioner to raise said grounds and to
entertain. present supporting documents constitute a waiver thereof, and
the same arguments and evidence can no longer be
Herein lies the inconsistency of petitioners entertained on appeal before the CSC, nor in the Court of
arguments. He faults the Court of Appeals and the CSC for Appeals, and much less in a petition for review before the
ignoring his supplemental pleading, while at the same time Supreme Court.[13] In fine, the raising of these factual issues
arguing that the grounds for recall such as violations of laws for the first time in a pleading which is supplemental only to an
and regulations on issuance of appointments are not new appeal is barred by estoppel.[14]
issues because he had timely raised them before the CSC.
Petitioner asks this Court to appreciate and consider
There is no question that parties may file supplemental these factual issues. It must be recalled that the jurisdiction of
pleadings to supply deficiencies in aid of an original pleading, the Supreme Court in a petition for review on certiorari under
but which should not entirely substitute the latter. [9] The Rule 45 of the Revised Rules of Court is limited to reviewing
propriety and substance of supplemental pleadings are only errors of law, not of fact.[15] That is, of course, unless the
prescribed under Rule 10, Section 6 of the 1997 Rules of Civil factual findings assailed by petitioner are devoid of support by
Procedure, which provides: the evidence on record or the impugned judgment is based on
a misapprehension of facts.[16]
Sec. 6. Supplemental Pleadings. - Upon motion of a party the A thorough perusal of the records reveal that the CSCs
court may, upon reasonable notice and upon such terms as ruling is supported by the evidence and the law. The fourteen
are just, permit him to serve a supplemental pleading setting (14) employees were duly appointed following two meetings of
forth transactions, occurrences or events which have the Personnel Selection Board held on May 31 and June 26,
happened since the date of the pleading sought to be 1995. There is no showing that any of the private respondents
supplemented. The adverse party may plead thereto within ten were not qualified for the positions they were appointed
(10) days from notice of the order admitting the supplemental to. Moreover, their appointments were duly attested to by the
pleading. Head of the CSC field office at Lucena City. By virtue thereof,
they had already assumed their appointive positions even
Supplemental pleadings must be with reasonable notice, before petitioner himself assumed his elected position as town
and it is discretionary upon the court or tribunal to allow the mayor. Consequently, their appointments took effect
same or not. Thus, the CSC was under no obligation to admit
FINALS CONSTITUTION I ACJUCO 150

immediately and cannot be unilaterally revoked or recalled by ruled, however, that the constitutional prohibition on so-called
petitioner. midnight appointments, specifically those made within two (2)
months immediately prior to the next presidential elections,
It has been held that upon the issuance of an applies only to the President or Acting President.
appointment and the appointees assumption of the position in
the civil service, he acquires a legal right which cannot be If ever there were other procedural or legal requirements
taken away either by revocation of the appointment or by that were violated in implementing the appointments of the
removal except for cause and with previous notice and private respondents, the same were not seasonably brought
hearing.[17] Moreover, it is well-settled that the person before the Civil Service Commission. These cannot be raised
assuming a position in the civil service under a completed for the first time on appeal.
appointment acquires a legal, not just an equitable, right to the
position. This right is protected not only by statute, but by the WHEREFORE, in view of all the foregoing, the instant
Constitution as well, which right cannot be taken away by either petition for review is DENIED and the Resolution of the Court
revocation of the appointment, or by removal, unless there is of Appeals in CA-G.R. SP No. 42896 affirming CSC
valid cause to do so, provided that there is previous notice and Resolutions Nos. 96-2828 and 96-7527 is hereby
hearing.[18] AFFIRMED in toto.

Petitioner admits that his very first official act upon No pronouncement as to costs.
assuming the position of town mayor was to issue Office Order
SO ORDERED.
No. 95-01 which recalled the appointments of the private
respondents. There was no previous notice, much less a
hearing accorded to the latter. Clearly, it was petitioner who
acted in undue haste to remove the private respondents
without regard for the simple requirements of due process of
law. In doing so, he overstepped the bounds of his
authority. While he argues that the appointing power has the
sole authority to revoke said appointments, there is no debate
that he does not have blanket authority to do so. Neither can
he question the CSCs jurisdiction to affirm or revoke the recall.
Rule V, Section 9 of the Omnibus Implementing
Regulations of the Revised Administrative Code specifically
provides that an appointment accepted by the appointee
cannot be withdrawn or revoked by the appointing authority
and shall remain in force and in effect until disapproved by the
Commission. Thus, it is the CSC that is authorized to recall an
appointment initially approved, but only when such
appointment and approval are proven to be in disregard of
applicable provisions of the civil service law and regulations. [19]
Moreover, Section 10 of the same rule provides:

Sec. 10. An appointment issued in accordance with pertinent


laws and rules shall take effect immediately upon its issuance
by the appointing authority, and if the appointee has assumed
the duties of the position, he shall be entitled to receive his
salary at once without awaiting the approval of his appointment
by the Commission. The appointment shall remain effective
until disapproved by the Commission.In no case shall an
appointment take effect earlier than he date of its issuance.

Section 20 of Rule VI also provides:

Sec. 20. Notwithstanding the initial approval of an


appointment, the same may be recalled on any of the following
grounds:

(a) Non-compliance with the procedures/criteria


provided in the agencys Merit Promotion Plan;
(b) Failure to pass through the agencys
Selection/Promotion Board;
(c) Violation of the existing collective agreement
between management and employees relative
to promotion; or
(d) Violation of other existing civil service law, rules
and regulations.
Accordingly, the appointments of the private respondents
may only be recalled on the above-cited grounds. And yet, the
only reason advanced by the petitioner to justify the recall was
that these were midnight appointments. The CSC correctly
FINALS CONSTITUTION I ACJUCO 151

G.R. No. 149036 April 2, 2002 Director IV of the EID and to Cinco as Director III also of the
EID, designating Cinco Officer-in-Charge of the EID and
MA. J. ANGELINA G. MATIBAG, petitioner, reassigning petitioner to the Law Department. COMELEC EID
Commissioner-in-Charge Mehol K. Sadain objected to
vs.
ALFREDO L. BENIPAYO, RESURRECCION Z. BORRA, petitioner’s reassignment in a Memorandum dated April 14,
FLORENTINO A. TUASON, JR., VELMA J. CINCO, and 200112 addressed to the COMELEC en banc. Specifically,
GIDEON C. DE GUZMAN in his capacity as Officer-In- Commissioner Sadain questioned Benipayo’s failure to consult
Charge, Finance Services Department of the Commission the Commissioner-in-Charge of the EID in the reassignment of
on Elections, respondents. petitioner.

CARPIO, J.: On April 16, 2001, petitioner requested Benipayo to reconsider


her relief as Director IV of the EID and her reassignment to the
Law Department.13 Petitioner cited Civil Service Commission
The Case Memorandum Circular No. 7 dated April 10, 2001, reminding
heads of government offices that "transfer and detail of
Before us is an original Petition for Prohibition with prayer for employees are prohibited during the election period beginning
the issuance of a writ of preliminary injunction and a temporary January 2 until June 13, 2001." Benipayo denied her request
restraining order under Rule 65 of the 1997 Rules of Civil for reconsideration on April 18, 2001, 14 citing COMELEC
Procedure. Petitioner Ma. J. Angelina G. Matibag ("Petitioner" Resolution No. 3300 dated November 6, 2000, which states in
for brevity) questions the constitutionality of the appointment part:
and the right to hold office of the following: (1) Alfredo L.
Benipayo ("Benipayo" for brevity) as Chairman of the "NOW, THEREFORE, the Commission on Elections
Commission on Elections ("COMELEC" for brevity); and (2) by virtue of the powers conferred upon it by the
Resurreccion Z. Borra ("Borra" for brevity) and Florentino A. Constitution, the Omnibus Election Code and other
Tuason, Jr. ("Tuason" for brevity) as COMELEC election laws, as an exception to the foregoing
Commissioners. Petitioner also questions the legality of the prohibitions, has RESOLVED, as it is hereby
appointment of Velma J. Cinco1 ("Cinco" for brevity) as Director RESOLVED, to appoint, hire new employees or fill
IV of the COMELEC’s Education and Information Department new positions and transfer or reassign its personnel,
("EID" for brevity). when necessary in the effective performance of its
mandated functions during the prohibited period,
The Facts provided that the changes in the assignment of its
field personnel within the thirty-day period before
election day shall be effected after due notice and
On February 2, 1999, the COMELEC en banc appointed hearing."
petitioner as "Acting Director IV" of the EID. On February 15,
2000, then Chairperson Harriet O. Demetriou renewed the
appointment of petitioner as Director IV of EID in a Petitioner appealed the denial of her request for
"Temporary" capacity. On February 15, 2001, Commissioner reconsideration to the COMELEC en banc in a Memorandum
Rufino S.B. Javier renewed again the appointment of petitioner dated April 23, 2001.15 Petitioner also filed an administrative
to the same position in a "Temporary" capacity.2 and criminal complaint16 with the Law Department17 against
Benipayo, alleging that her reassignment violated Section 261
(h) of the Omnibus Election Code, COMELEC Resolution No.
On March 22, 2001, President Gloria Macapagal Arroyo 3258, Civil Service Memorandum Circular No. 07, s. 001, and
appointed, ad interim, Benipayo as COMELEC Chairman,3and other pertinent administrative and civil service laws, rules and
Borra4 and Tuason5 as COMELEC Commissioners, each for a regulations.
term of seven years and all expiring on February 2, 2008.
Benipayo took his oath of office and assumed the position of
COMELEC Chairman. Borra and Tuason likewise took their During the pendency of her complaint before the Law
oaths of office and assumed their positions as COMELEC Department, petitioner filed the instant petition questioning the
Commissioners. The Office of the President submitted to the appointment and the right to remain in office of Benipayo, Borra
Commission on Appointments on May 22, 2001 the ad and Tuason, as Chairman and Commissioners of the
interim appointments of Benipayo, Borra and Tuason for COMELEC, respectively. Petitioner claims that the ad
confirmation.6 However, the Commission on Appointments did interim appointments of Benipayo, Borra and Tuason violate
not act on said appointments. the constitutional provisions on the independence of the
COMELEC, as well as on the prohibitions on temporary
appointments and reappointments of its Chairman and
On June 1, 2001, President Arroyo renewed the ad members. Petitioner also assails as illegal her removal as
interim appointments of Benipayo, Borra and Tuason to the Director IV of the EID and her reassignment to the Law
same positions and for the same term of seven years, expiring Department. Simultaneously, petitioner challenges the
on February 2, 2008.7 They took their oaths of office for a designation of Cinco as Officer-in-Charge of the EID.
second time. The Office of the President transmitted on June Petitioner, moreover, questions the legality of the
5, 2001 their appointments to the Commission on disbursements made by COMELEC Finance Services
Appointments for confirmation.8 Department Officer-in-Charge Gideon C. De Guzman to
Benipayo, Borra and Tuason by way of salaries and other
Congress adjourned before the Commission on Appointments emoluments.
could act on their appointments. Thus, on June 8, 2001,
President Macapagal Arroyo renewed again the ad In the meantime, on September 6, 2001, President Macapagal
interim appointments of Benipayo, Borra and Tuason to the Arroyo renewed once again the ad interimappointments of
same positions.9 The Office of the President submitted their Benipayo as COMELEC Chairman and Borra and Tuason as
appointments for confirmation to the Commission on Commissioners, respectively, for a term of seven years
Appointments.10 They took their oaths of office anew. expiring on February 2, 2008.18 They all took their oaths of
office anew.
In his capacity as COMELEC Chairman, Benipayo issued a
Memorandum dated April 11, 200111 addressed to petitioner as
FINALS CONSTITUTION I ACJUCO 152

The Issues We are not persuaded.

The issues for resolution of this Court are as follows: Benipayo reassigned petitioner from the EID, where she was
Acting Director, to the Law Department, where she was placed
1. Whether or not the instant petition satisfies all the on detail service.20 Respondents claim that the reassignment
was "pursuant to x x x Benipayo’s authority as Chairman of the
requirements before this Court may exercise its power of
judicial review in constitutional cases; Commission on Elections, and as the Commission’s Chief
Executive Officer."21 Evidently, respondents anchor the legality
of petitioner’s reassignment on Benipayo’s authority as
2. Whether or not the assumption of office by Benipayo, Borra Chairman of the COMELEC. The real issue then turns on
and Tuason on the basis of the ad interimappointments issued whether or not Benipayo is the lawful Chairman of the
by the President amounts to a temporary appointment COMELEC. Even if petitioner is only an Acting Director of the
prohibited by Section 1 (2), Article IX-C of the Constitution; EID, her reassignment is without legal basis if Benipayo is not
the lawful COMELEC Chairman, an office created by the
3. Assuming that the first ad interim appointments and the first Constitution.
assumption of office by Benipayo, Borra and Tuason are legal,
whether or not the renewal of their ad interim appointments On the other hand, if Benipayo is the lawful COMELEC
and subsequent assumption of office to the same positions Chairman because he assumed office in accordance with the
violate the prohibition on reappointment under Section 1 (2), Constitution, then petitioner’s reassignment is legal and she
Article IX-C of the Constitution; has no cause to complain provided the reassignment is in
accordance with the Civil Service Law. Clearly, petitioner has
4. Whether or not Benipayo’s removal of petitioner from her a personal and material stake in the resolution of the
position as Director IV of the EID and her reassignment to the constitutionality of Benipayo’s assumption of office. Petitioner’s
Law Department is illegal and without authority, having been personal and substantial injury, if Benipayo is not the lawful
done without the approval of the COMELEC as a collegial COMELEC Chairman, clothes her with the requisite locus
body; standi to raise the constitutional issue in this petition.

5. Whether or not the Officer-in-Charge of the COMELEC’s Respondents harp on petitioner’s belated act of questioning
Finance Services Department, in continuing to make the constitutionality of the ad interim appointments of
disbursements in favor of Benipayo, Borra, Tuason and Cinco, Benipayo, Borra and Tuason. Petitioner filed the instant
is acting in excess of jurisdiction. petition only on August 3, 2001, when the first ad
interimappointments were issued as early as March 22, 2001.
However, it is not the date of filing of the petition that
First Issue: Propriety of Judicial Review determines whether the constitutional issue was raised at the
earliest opportunity. The earliest opportunity to raise a
Respondents assert that the petition fails to satisfy all the four constitutional issue is to raise it in the pleadings before a
requisites before this Court may exercise its power of judicial competent court that can resolve the same, such that, "if it is
review in constitutional cases. Out of respect for the acts of the not raised in the pleadings, it cannot be considered at the trial,
Executive department, which is co-equal with this Court, and, if not considered at the trial, it cannot be considered on
respondents urge this Court to refrain from reviewing the appeal."22 Petitioner questioned the constitutionality of the ad
constitutionality of the ad interim appointments issued by the interim appointments of Benipayo, Borra and Tuason when
President to Benipayo, Borra and Tuason unless all the four she filed her petition before this Court, which is the earliest
requisites are present. These are: (1) the existence of an actual opportunity for pleading the constitutional issue before a
and appropriate controversy; (2) a personal and substantial competent body. Furthermore, this Court may determine, in the
interest of the party raising the constitutional issue; (3) the exercise of sound discretion, the time when a constitutional
exercise of the judicial review is pleaded at the earliest issue may be passed upon.23 There is no doubt petitioner
opportunity; and (4) the constitutional issue is the lis mota of raised the constitutional issue on time.
the case.19 Respondents argue that the second, third and
fourth requisites are absent in this case. Respondents maintain Moreover, the legality of petitioner’s reassignment hinges on
that petitioner does not have a personal and substantial the constitutionality of Benipayo’s ad interimappointment and
interest in the case because she has not sustained a direct assumption of office. Unless the constitutionality of
injury as a result of the ad interim appointments of Benipayo, Benipayo’s ad interim appointment and assumption of office is
Borra and Tuason and their assumption of office. Respondents resolved, the legality of petitioner’s reassignment from the EID
point out that petitioner does not claim to be lawfully entitled to to the Law Department cannot be determined. Clearly, the lis
any of the positions assumed by Benipayo, Borra or Tuason. mota of this case is the very constitutional issue raised by
Neither does petitioner claim to be directly injured by the petitioner.
appointments of these three respondents.
In any event, the issue raised by petitioner is of paramount
Respondents also contend that petitioner failed to question the importance to the public. The legality of the directives and
constitutionality of the ad interim appointments at the earliest decisions made by the COMELEC in the conduct of the May
opportunity. Petitioner filed the petition only on August 3, 2001 14, 2001 national elections may be put in doubt if the
despite the fact that the ad interimappointments of Benipayo, constitutional issue raised by petitioner is left unresolved. In
Borra and Tuason were issued as early as March 22, 2001. keeping with this Court’s duty to determine whether other
Moreover, the petition was filed after the third time that these agencies of government have remained within the limits of the
three respondents were issued ad interim appointments. Constitution and have not abused the discretion given them,
this Court may even brush aside technicalities of procedure
Respondents insist that the real issue in this case is the legality and resolve any constitutional issue raised. 24 Here the
of petitioner’s reassignment from the EID to the Law petitioner has complied with all the requisite technicalities.
Department. Consequently, the constitutionality of the ad Moreover, public interest requires the resolution of the
interim appointments is not the lis mota of this case. constitutional issue raised by petitioner.
FINALS CONSTITUTION I ACJUCO 153

Second Issue: The Nature of an Ad Interim Appointment appointments shall be effective only until disapproval
by the Commission on Appointments or until the next
Petitioner argues that an ad interim appointment to the adjournment of the Congress." (Emphasis supplied)
COMELEC is a temporary appointment that is prohibited by
Section 1 (2), Article IX-C of the Constitution, which provides Thus, the ad interim appointment remains effective until such
as follows: disapproval or next adjournment, signifying that it can no
longer be withdrawn or revoked by the President. The fear that
the President can withdraw or revoke at any time and for any
"The Chairman and the Commissioners shall be appointed by
the President with the consent of the Commission on reason an ad interim appointment is utterly without basis.
Appointments for a term of seven years without reappointment.
Of those first appointed, three Members shall hold office for More than half a century ago, this Court had already ruled that
seven years, two Members for five years, and the last an ad interim appointment is permanent in character.
Members for three years, without reappointment. Appointment In Summers vs. Ozaeta,25 decided on October 25, 1948, we
to any vacancy shall be only for the unexpired term of the held that:
predecessor. In no case shall any Member be appointed or
designated in a temporary or acting capacity." (Emphasis "x x x an ad interim appointment is one made in
supplied) pursuance of paragraph (4), Section 10, Article VII of
the Constitution, which provides that the ‘President
Petitioner posits the view that an ad interim appointment can shall have the power to make appointments during
be withdrawn or revoked by the President at her pleasure, and the recess of the Congress, but such appointments
can even be disapproved or simply by-passed by the shall be effective only until disapproval by the
Commission on Appointments. For this reason, petitioner Commission on Appointments or until the next
claims that an ad interim appointment is temporary in adjournment of the Congress.’ It is an appointment
character and consequently prohibited by the last sentence of permanent in nature, and the circumstance that it is
Section 1 (2), Article IX-C of the Constitution. subject to confirmation by the Commission on
Appointments does not alter its permanent character.
An ad interim appointment is disapproved certainly
Based on petitioner’s theory, there can be no ad
interim appointment to the COMELEC or to the other two for a reason other than that its provisional period has
expired. Said appointment is of course
constitutional commissions, namely the Civil Service
Commission and the Commission on Audit. The last sentence distinguishable from an ‘acting’ appointment which is
of Section 1 (2), Article IX-C of the Constitution is also found in merely temporary, good until another permanent
Article IX-B and Article IX-D providing for the creation of the appointment is issued." (Emphasis supplied)
Civil Service Commission and the Commission on Audit,
respectively. Petitioner interprets the last sentence of Section The Constitution imposes no condition on the effectivity of
1 (2) of Article IX-C to mean that the ad interim appointee an ad interim appointment, and thus an ad interimappointment
cannot assume office until his appointment is confirmed by the takes effect immediately. The appointee can at once assume
Commission on Appointments for only then does his office and exercise, as a de jure officer, all the powers
appointment become permanent and no longer temporary in pertaining to the office. In Pacete vs. Secretary of the
character. Commission on Appointments,26 this Court elaborated on the
nature of an ad interim appointment as follows:
The rationale behind petitioner’s theory is that only an
appointee who is confirmed by the Commission on "A distinction is thus made between the exercise of
Appointments can guarantee the independence of the such presidential prerogative requiring confirmation
COMELEC. A confirmed appointee is beyond the influence of by the Commission on Appointments when Congress
the President or members of the Commission on Appointments is in session and when it is in recess. In the former,
since his appointment can no longer be recalled or the President nominates, and only upon the consent
disapproved. Prior to his confirmation, the appointee is at the of the Commission on Appointments may the person
mercy of both the appointing and confirming powers since his thus named assume office. It is not so with reference
appointment can be terminated at any time for any cause. In to ad interim appointments. It takes effect at once.
the words of petitioner, a Sword of Damocles hangs over the The individual chosen may thus qualify and perform
head of every appointee whose confirmation is pending with his function without loss of time. His title to such office
the Commission on Appointments. is complete. In the language of the Constitution, the
appointment is effective ‘until disapproval by the
We find petitioner’s argument without merit. Commission on Appointments or until the next
adjournment of the Congress.’"

An ad interim appointment is a permanent appointment


because it takes effect immediately and can no longer be Petitioner cites Black’s Law Dictionary which defines the term
withdrawn by the President once the appointee has qualified "ad interim" to mean "in the meantime" or "for the time being."
Hence, petitioner argues that an ad interim appointment is
into office. The fact that it is subject to confirmation by the
Commission on Appointments does not alter its permanent undoubtedly temporary in character. This argument is not new
and was answered by this Court in Pamantasan ng Lungsod
character. The Constitution itself makes an ad
interim appointment permanent in character by making it ng Maynila vs. Intermediate Appellate Court,27 where we
effective until disapproved by the Commission on explained that:
Appointments or until the next adjournment of Congress. The
second paragraph of Section 16, Article VII of the Constitution "x x x From the arguments, it is easy to see why the petitioner
provides as follows: should experience difficulty in understanding the situation.
Private respondent had been extended several ‘ad interim’
"The President shall have the power to make appointments which petitioner mistakenly understands as
appointments during the recess of the Congress, appointments temporary in nature. Perhaps, it is the literal
whether voluntary or compulsory, but such translation of the word ‘ad interim’ which creates such belief.
The term is defined by Black to mean "in the meantime" or "for
FINALS CONSTITUTION I ACJUCO 154

the time being". Thus, an officer ad interim is one appointed to is the Constitution itself that places the Sword of Damocles
fill a vacancy, or to discharge the duties of the office during the over the heads of the ad interimappointees.
absence or temporary incapacity of its regular incumbent
(Black’s Law Dictionary, Revised Fourth Edition, 1978). But
While an ad interim appointment is permanent and irrevocable
such is not the meaning nor the use intended in the context of except as provided by law, an appointment or designation in a
Philippine law. In referring to Dr. Esteban’s appointments, the temporary or acting capacity can be withdrawn or revoked at
term is not descriptive of the nature of the appointments given the pleasure of the appointing power. 31A temporary or acting
to him. Rather, it is used to denote the manner in which said appointee does not enjoy any security of tenure, no matter how
appointments were made, that is, done by the President of the briefly. This is the kind of appointment that the Constitution
Pamantasan in the meantime, while the Board of Regents, prohibits the President from making to the three independent
which is originally vested by the University Charter with the constitutional commissions, including the COMELEC. Thus, in
power of appointment, is unable to act. x x x." (Emphasis Brillantes vs. Yorac,32 this Court struck down as
supplied)
unconstitutional the designation by then President Corazon
Aquino of Associate Commissioner Haydee Yorac as Acting
Thus, the term "ad interim appointment", as used in letters of Chairperson of the COMELEC. This Court ruled that:
appointment signed by the President, means a permanent
appointment made by the President in the meantime that
"A designation as Acting Chairman is by its very
Congress is in recess. It does not mean a temporary terms essentially temporary and therefore revocable
appointment that can be withdrawn or revoked at any time. The
at will. No cause need be established to justify its
term, although not found in the text of the Constitution, has revocation. Assuming its validity, the designation of
acquired a definite legal meaning under Philippine the respondent as Acting Chairman of the
jurisprudence. The Court had again occasion to explain the Commission on Elections may be withdrawn by the
nature of an ad interim appointment in the more recent case President of the Philippines at any time and for
of Marohombsar vs. Court of Appeals,28where the Court whatever reason she sees fit. It is doubtful if the
stated: respondent, having accepted such designation, will
not be estopped from challenging its withdrawal.
"We have already mentioned that an ad interim
appointment is not descriptive of the nature of the xxx
appointment, that is, it is not indicative of whether the
appointment is temporary or in an acting capacity,
rather it denotes the manner in which the The Constitution provides for many safeguards to the
appointment was made. In the instant case, the independence of the Commission on Elections,
appointment extended to private respondent by then foremost among which is the security of tenure of its
MSU President Alonto, Jr. was issued without members. That guarantee is not available to the
condition nor limitation as to tenure. The permanent respondent as Acting Chairman of the Commission
status of private respondent’s appointment as on Elections by designation of the President of the
Executive Assistant II was recognized and attested Philippines."
to by the Civil Service Commission Regional Office
No. 12. Petitioner’s submission that private Earlier, in Nacionalista Party vs. Bautista,33 a case decided
respondent’s ad interim appointment is synonymous under the 1935 Constitution, which did not have a provision
with a temporary appointment which could be validly prohibiting temporary or acting appointments to the
terminated at any time is clearly untenable. Ad COMELEC, this Court nevertheless declared unconstitutional
interim appointments are permanent but their terms the designation of the Solicitor General as acting member of
are only until the Board disapproves them." the COMELEC. This Court ruled that the designation of an
(Emphasis supplied) acting Commissioner would undermine the independence of
the COMELEC and hence violate the Constitution. We
An ad interim appointee who has qualified and assumed office declared then: "It would be more in keeping with the intent,
becomes at that moment a government employee and purpose and aim of the framers of the Constitution to appoint
therefore part of the civil service. He enjoys the constitutional a permanent Commissioner than to designate one to act
protection that "[n]o officer or employee in the civil service shall temporarily." (Emphasis supplied)
be removed or suspended except for cause provided by
law."29 Thus, an ad interim appointment becomes complete In the instant case, the President did in fact appoint permanent
and irrevocable once the appointee has qualified into office. Commissioners to fill the vacancies in the COMELEC, subject
The withdrawal or revocation of an ad interim appointment is only to confirmation by the Commission on Appointments.
possible only if it is communicated to the appointee before the Benipayo, Borra and Tuason were extended permanent
moment he qualifies, and any withdrawal or revocation appointments during the recess of Congress. They were not
thereafter is tantamount to removal from office. 30 Once an appointed or designated in a temporary or acting capacity,
appointee has qualified, he acquires a legal right to the office unlike Commissioner Haydee Yorac in Brillantes vs.
which is protected not only by statute but also by the Yorac34 and Solicitor General Felix Bautista in Nacionalista
Constitution. He can only be removed for cause, after notice Party vs. Bautista.35 The ad interim appointments of Benipayo,
and hearing, consistent with the requirements of due process. Borra and Tuason are expressly allowed by the Constitution
which authorizes the President, during the recess of Congress,
An ad interim appointment can be terminated for two causes to make appointments that take effect immediately.
specified in the Constitution. The first cause is the disapproval
of his ad interim appointment by the Commission on While the Constitution mandates that the COMELEC "shall be
Appointments. The second cause is the adjournment of independent"36 , this provision should be harmonized with the
Congress without the Commission on Appointments acting on President’s power to extend ad interim appointments. To hold
his appointment. These two causes are resolutory conditions that the independence of the COMELEC requires the
expressly imposed by the Constitution on all ad Commission on Appointments to first confirm ad
interim appointments. These resolutory conditions constitute, interim appointees before the appointees can assume office
in effect, a Sword of Damocles over the heads of ad will negate the President’s power to make ad
interim appointees. No one, however, can complain because it interim appointments. This is contrary to the rule on statutory
FINALS CONSTITUTION I ACJUCO 155

construction to give meaning and effect to every provision of otherwise would result from prolonged vacancies in
the law. It will also run counter to the clear intent of the framers government offices, including the three constitutional
of the Constitution. commissions. In his concurring opinion in Guevara vs.
Inocentes,38 decided under the 1935 Constitution, Justice
The original draft of Section 16, Article VII of the Constitution - Roberto Concepcion, Jr. explained the rationale behind ad
on the nomination of officers subject to confirmation by the interim appointments in this manner:
Commission on Appointments - did not provide for ad interim
appointments. The original intention of the framers of the "Now, why is the lifetime of ad interim appointments
Constitution was to do away with ad interim appointments so limited? Because, if they expired before the
because the plan was for Congress to remain in session session of Congress, the evil sought to be avoided –
throughout the year except for a brief 30-day compulsory interruption in the discharge of essential functions –
recess. However, because of the need to avoid disruptions in may take place. Because the same evil would result
essential government services, the framers of the Constitution if the appointments ceased to be effective during the
thought it wise to reinstate the provisions of the 1935 session of Congress and before its adjournment.
Constitution on ad interim appointments. The following Upon the other hand, once Congress has adjourned,
discussion during the deliberations of the Constitutional the evil aforementioned may easily be conjured by
Commission elucidates this: the issuance of other ad interim appointments or
reappointments." (Emphasis supplied)
"FR. BERNAS: X x x our compulsory recess now is
only 30 days. So under such circumstances, is it Indeed, the timely application of the last sentence of Section
necessary to provide for ad interim appointments? 16, Article VII of the Constitution barely avoided the interruption
Perhaps there should be a little discussion on that. of essential government services in the May 2001 national
elections. Following the decision of this Court in Gaminde vs.
xxx Commission on Appointments,39 promulgated on December
13, 2000, the terms of office of constitutional officers first
appointed under the Constitution would have to be counted
MS. AQUINO: My concern is that unless this problem starting February 2, 1987, the date of ratification of the
is addressed, this might present problems in terms of Constitution, regardless of the date of their actual appointment.
anticipating interruption of government business, By this reckoning, the terms of office of three Commissioners
considering that we are not certain of the length of of the COMELEC, including the Chairman, would end on
involuntary recess or adjournment of the Congress. February 2, 2001.40
We are certain, however, of the involuntary
adjournment of the Congress which is 30 days, but
we cannot leave to conjecture the matter of Then COMELEC Chairperson Harriet O. Demetriou was
involuntary recess. appointed only on January 11, 2000 to serve, pursuant to her
appointment papers, until February 15, 2002, 41 the original
expiry date of the term of her predecessor, Justice Bernardo
FR. BERNAS: That is correct, but we are trying to P. Pardo, who was elevated to this Court. The original expiry
look for a formula. I wonder if the Commissioner has date of the term of Commissioner Teresita Dy-Liacco Flores
a formula x x x. was also February 15, 2002, while that of Commissioner Julio
F. Desamito was November 3, 2001.42 The original expiry
xxx dates of the terms of office of Chairperson Demetriou and
Commissioners Flores and Desamito were therefore supposed
to fall after the May 2001 elections. Suddenly and
MR. BENGZON: Madam President, apropos of the unexpectedly, because of the Gaminde ruling, there were
matter raised by Commissioner Aquino and after three vacancies in the seven-person COMELEC, with national
conferring with the Committee, Commissioner elections looming less than three and one-half months away.
Aquino and I propose the following amendment as To their credit, Chairperson Demetriou and Commissioner
the last paragraph of Section 16, the wordings of Flores vacated their offices on February 2, 2001 and did not
which are in the 1935 Constitution: THE PRESIDENT question any more before this Court the applicability of
SHALL HAVE THE POWER TO MAKE the Gaminderuling to their own situation.
APPOINTMENTS DURING THE RECESS OF
CONGRESS WHETHER IT BE VOLUNTARY OR
COMPULSORY BUT SUCH APPOINTMENTS In a Manifestation43 dated December 28, 2000 filed with this
SHALL BE EFFECTIVE ONLY UNTIL Court in the Gaminde case, Chairperson Demetriou stated that
DISAPPROVAL BY THE COMMISSION ON she was vacating her office on February 2, 2001, as she
APPOINTMENTS OR UNTIL THE NEXT believed any delay in choosing her successor might create a
ADJOURNMENT OF THE CONGRESS. "constitutional crisis" in view of the proximity of the May 2001
national elections. Commissioner Desamito chose to file a
petition for intervention44 in the Gaminde case but this Court
This is otherwise called the ad interim appointments. denied the intervention. Thus, Commissioner Desamito also
vacated his office on February 2, 2001.
xxx
During an election year, Congress normally goes on voluntary
THE PRESIDENT: Is there any objection to the recess between February and June considering that many of
proposed amendment of Commissioners Aquino and the members of the House of Representatives and the Senate
Bengzon, adding a paragraph to the last paragraph run for re-election. In 2001, the Eleventh Congress adjourned
of Section 16? (Silence) The Chair hears none; the from January 9, 2001 to June 3, 2001.45 Concededly, there was
amendment is approved."37 (Emphasis supplied) no more time for Benipayo, Borra and Tuason, who were
originally extended ad interim appointments only on March 22,
2001, to be confirmed by the Commission on Appointments
Clearly, the reinstatement in the present Constitution of the ad before the May 14, 2001 elections.
interim appointing power of the President was for the purpose
of avoiding interruptions in vital government services that
FINALS CONSTITUTION I ACJUCO 156

If Benipayo, Borra and Tuason were not extended ad vacancies in the COMELEC are precisely staggered to insure
interim appointments to fill up the three vacancies in the that the majority of its members hold confirmed appointments,
COMELEC, there would only have been one division and not one President will appoint all the COMELEC
functioning in the COMELEC instead of two during the May members.50 In the instant case, the Commission on
2001 elections. Considering that the Constitution requires that Appointments had long confirmed four 51 of the incumbent
"all x x x election cases shall be heard and decided in COMELEC members, comprising a majority, who could now
division",46 the remaining one division would have been be removed from office only by impeachment. The special
swamped with election cases. Moreover, since under the constitutional safeguards that insure the independence of the
Constitution motions for reconsideration "shall be decided by COMELEC remain in place.52 The COMELEC enjoys fiscal
the Commission en banc", the mere absence of one of the four autonomy, appoints its own officials and employees, and
remaining members would have prevented a quorum, a less promulgates its own rules on pleadings and practice.
than ideal situation considering that the Commissioners are Moreover, the salaries of COMELEC members cannot be
expected to travel around the country before, during and after decreased during their tenure.
the elections. There was a great probability that disruptions in
the conduct of the May 2001 elections could occur because of
In fine, we rule that the ad interim appointments extended by
the three vacancies in the COMELEC. The successful conduct the President to Benipayo, Borra and Tuason, as COMELEC
of the May 2001 national elections, right after the tumultuous Chairman and Commissioners, respectively, do not constitute
EDSA II and EDSA III events, was certainly essential in temporary or acting appointments prohibited by Section 1 (2),
safeguarding and strengthening our democracy. Article IX-C of the Constitution.

Evidently, the exercise by the President in the instant case of Third Issue: The Constitutionality of Renewals of Appointments
her constitutional power to make ad interimappointments
prevented the occurrence of the very evil sought to be avoided
by the second paragraph of Section 16, Article VII of the Petitioner also agues that assuming the first ad interim
Constitution. This power to make ad interim appointments is appointments and the first assumption of office by Benipayo,
lodged in the President to be exercised by her in her sound Borra and Tuason are constitutional, the renewal of the their
judgment. Under the second paragraph of Section 16, Article ad interim appointments and their subsequent assumption of
VII of the Constitution, the President can choose either of two office to the same positions violate the prohibition on
modes in appointing officials who are subject to confirmation reappointment under Section 1 (2), Article IX-C of the
by the Commission on Appointments. First, while Congress is Constitution, which provides as follows:
in session, the President may nominate the prospective
appointee, and pending consent of the Commission on "The Chairman and the Commissioners shall be
Appointments, the nominee cannot qualify and assume office. appointed by the President with the consent of the
Second, during the recess of Congress, the President may Commission on Appointments for a term of seven
extend an ad interim appointment which allows the appointee years without reappointment. Of those first
to immediately qualify and assume office. appointed, three Members shall hold office for seven
years, two Members for five years, and the last
Whether the President chooses to nominate the prospective members for three years, without reappointment. X x
appointee or extend an ad interim appointment is a matter x." (Emphasis supplied)
within the prerogative of the President because the
Constitution grants her that power. This Court cannot inquire Petitioner theorizes that once an ad interim appointee is by-
into the propriety of the choice made by the President in the passed by the Commission on Appointments, his ad interim
exercise of her constitutional power, absent grave abuse of appointment can no longer be renewed because this will
discretion amounting to lack or excess of jurisdiction on her violate Section 1 (2), Article IX-C of the Constitution which
part, which has not been shown in the instant case. prohibits reappointments. Petitioner asserts that this is
particularly true to permanent appointees who have assumed
The issuance by Presidents of ad interim appointments to the office, which is the situation of Benipayo, Borra and Tuason if
COMELEC is a long-standing practice. Former President their ad interim appointments are deemed permanent in
Corazon Aquino issued an ad interim appointment to character.
Commissioner Alfredo E. Abueg.47 Former President Fidel V.
Ramos extended ad interim appointments to Commissioners There is no dispute that an ad interim appointee disapproved
Julio F. Desamito, Japal M. Guiani, Graduacion A. Reyes- by the Commission on Appointments can no longer be
Claravall and Manolo F. Gorospe.48 Former President Joseph extended a new appointment. The disapproval is a final
Estrada also extended ad interim appointments to decision of the Commission on Appointments in the exercise
Commissioners Abdul Gani M. Marohombsar, Luzviminda of its checking power on the appointing authority of the
Tancangco, Mehol K. Sadain and Ralph C. Lantion. 49 President. The disapproval is a decision on the merits, being a
refusal by the Commission on Appointments to give its consent
The President’s power to extend ad interim appointments may after deliberating on the qualifications of the appointee. Since
indeed briefly put the appointee at the mercy of both the the Constitution does not provide for any appeal from such
appointing and confirming powers. This situation, however, is decision, the disapproval is final and binding on the appointee
only for a short period - from the time of issuance of the ad as well as on the appointing power. In this instance, the
interim appointment until the Commission on Appointments President can no longer renew the appointment not because
gives or withholds its consent. The Constitution itself sanctions of the constitutional prohibition on reappointment, but because
this situation, as a trade-off against the evil of disruptions in of a final decision by the Commission on Appointments to
vital government services. This is also part of the check-and- withhold its consent to the appointment.
balance under the separation of powers, as a trade-off against
the evil of granting the President absolute and sole power to An ad interim appointment that is by-passed because of lack
appoint. The Constitution has wisely subjected the President’s of time or failure of the Commission on Appointments to
appointing power to the checking power of the legislature. organize is another matter. A by-passed appointment is one
that has not been finally acted upon on the merits by the
This situation, however, does not compromise the Commission on Appointments at the close of the session of
independence of the COMELEC as a constitutional body. The Congress. There is no final decision by the Commission on
FINALS CONSTITUTION I ACJUCO 157

Appointments to give or withhold its consent to the Section 16, Article VII of the Constitution, and not because a
appointment as required by the Constitution. Absent such reappointment is prohibited under Section 1 (2), Article IX-C of
decision, the President is free to renew the ad interim the Constitution. A by-passed ad interim appointment can be
appointment of a by-passed appointee. This is recognized in revived by a new ad interim appointment because there is no
Section 17 of the Rules of the Commission on Appointments, final disapproval under Section 16, Article VII of the
which provides as follows: Constitution, and such new appointment will not result in the
appointee serving beyond the fixed term of seven years.
"Section 17. Unacted Nominations or Appointments
Returned to the President. Nominations or Section 1 (2), Article IX-C of the Constitution provides that
appointments submitted by the President of the "[t]he Chairman and the Commissioners shall be appointed x x
Philippines which are not finally acted upon at the x for a term of seven years without reappointment." (Emphasis
close of the session of Congress shall be returned to supplied) There are four situations where this provision will
the President and, unless new nominations or apply. The first situation is where an ad interim appointee to
appointments are made, shall not again be the COMELEC, after confirmation by the Commission on
considered by the Commission." (Emphasis Appointments, serves his full seven-year term. Such person
supplied) cannot be reappointed to the COMELEC, whether as a
member or as a chairman, because he will then be actually
Hence, under the Rules of the Commission on Appointments, serving more than seven years. The second situation is where
the appointee, after confirmation, serves a part of his term and
a by-passed appointment can be considered again if the
President renews the appointment. then resigns before his seven-year term of office ends. Such
person cannot be reappointed, whether as a member or as a
chair, to a vacancy arising from retirement because a
It is well settled in this jurisdiction that the President can renew reappointment will result in the appointee also serving more
the ad interim appointments of by-passed appointees. Justice than seven years. The third situation is where the appointee is
Roberto Concepcion, Jr. lucidly explained in his concurring confirmed to serve the unexpired term of someone who died or
opinion in Guevara vs. Inocentes53why by-passed ad resigned, and the appointee completes the unexpired term.
interim appointees could be extended new appointments, thus: Such person cannot be reappointed, whether as a member or
chair, to a vacancy arising from retirement because a
"In short, an ad interim appointment ceases to be reappointment will result in the appointee also serving more
effective upon disapproval by the Commission, than seven years.
because the incumbent can not continue holding
office over the positive objection of the Commission. The fourth situation is where the appointee has previously
It ceases, also, upon "the next adjournment of the served a term of less than seven years, and a vacancy arises
Congress", simply because the President may then from death or resignation. Even if it will not result in his serving
issue new appointments - not because of implied more than seven years, a reappointment of such person to
disapproval of the Commission deduced from its serve an unexpired term is also prohibited because his
inaction during the session of Congress, for, under situation will be similar to those appointed under the second
the Constitution, the Commission may affect sentence of Section 1 (2), Article IX-C of the Constitution. This
adversely the interim appointments only by action, provision refers to the first appointees under the Constitution
never by omission. If the adjournment of Congress whose terms of office are less than seven years, but are barred
were an implied disapproval of ad from ever being reappointed under any situation. Not one of
interimappointments made prior thereto, then the these four situations applies to the case of Benipayo, Borra or
President could no longer appoint those so by- Tuason.
passed by the Commission. But, the fact is that the
President may reappoint them, thus clearly indicating
that the reason for said termination of the ad The framers of the Constitution made it quite clear that any
interim appointments is not the disapproval thereof person who has served any term of office as COMELEC
member – whether for a full term of seven years, a truncated
allegedly inferred from said omission of the
Commission, but the circumstance that upon said term of five or three years, or even for an unexpired term of
any length of time – can no longer be reappointed to the
adjournment of the Congress, the President is free to
make ad interim appointments or reappointments." COMELEC. Commissioner Foz succinctly explained this intent
(Emphasis supplied) in this manner:

"MR. FOZ. But there is the argument made in the


Guevara was decided under the 1935 Constitution from where
the second paragraph of Section 16, Article VII of the present concurring opinion of Justice Angelo Bautista in the
Constitution on ad interim appointments was case of Visarra vs. Miraflor, to the effect that the
lifted verbatim.54 The jurisprudence under the 1935 prohibition on reappointment applies only when the
Constitution governing ad interim appointments by the term or tenure is for seven years. But in cases where
President is doubtless applicable to the present Constitution. the appointee serves only for less than seven years,
The established practice under the present Constitution is that he would be entitled to reappointment. Unless we put
the qualifying words "without reappointment" in the
the President can renew the appointments of by-passed ad
interim appointees. This is a continuation of the well- case of those appointed, then it is possible that an
interpretation could be made later on their case, they
recognized practice under the 1935 Constitution, interrupted
only by the 1973 Constitution which did not provide for a can still be reappointed to serve for a total of seven
years.
Commission on Appointments but vested sole appointing
power in the President.
Precisely, we are foreclosing that possibility by
The prohibition on reappointment in Section 1 (2), Article IX-C making it clear that even in the case of those first
appointed under the Constitution, no reappointment
of the Constitution applies neither to disapproved nor by-
passed ad interim appointments. A disapproved ad can be made."55 (Emphasis supplied)
interim appointment cannot be revived by another ad
interimappointment because the disapproval is final under
FINALS CONSTITUTION I ACJUCO 158

In Visarra vs. Miraflor,56 Justice Angelo Bautista, in Commission in the same manner (as) provided for
his concurring opinion, quoted Nacionalista vs. De the Judiciary; 2) fixed term of office without
Vera57that a "[r]eappointment is not prohibited when reappointment on a staggered basis to ensure
a Commissioner has held office only for, say, three continuity of functions and to minimize the
or six years, provided his term will not exceed nine opportunity of the President to appoint all the
years in all." This was the interpretation despite the members during his incumbency; 3) prohibition to
express provision in the 1935 Constitution that a decrease salaries of the members of the
COMELEC member "shall hold office for a term of Commissions during their term of office; and 4)
nine years and may not be reappointed." appointments of members would not require
confirmation."59 (Emphasis supplied)
To foreclose this interpretation, the phrase "without
reappointment" appears twice in Section 1 (2), Article IX-C of There were two important amendments subsequently made by
the present Constitution. The first phrase prohibits the Constitutional Commission to these four features. First, as
reappointment of any person previously appointed for a term discussed earlier, the framers of the Constitution decided to
of seven years. The second phrase prohibits reappointment of require confirmation by the Commission on Appointments of all
any person previously appointed for a term of five or three appointments to the constitutional commissions. Second, the
years pursuant to the first set of appointees under the framers decided to strengthen further the prohibition on
Constitution. In either case, it does not matter if the person serving beyond the fixed seven-year term, in the light of a
previously appointed completes his term of office for the former chair of the Commission on Audit remaining in office for
intention is to prohibit any reappointment of any kind. 12 years despite his fixed term of seven years. The following
exchange in the deliberations of the Constitutional
However, an ad interim appointment that has lapsed by Commission is instructive:
inaction of the Commission on Appointments does not
constitute a term of office. The period from the time the ad "MR. SUAREZ: These are only clarificatory
interim appointment is made to the time it lapses is neither a questions, Madam President. May I call the
fixed term nor an unexpired term. To hold otherwise would sponsor’s attention, first of all, to Section 2 (2) on the
mean that the President by his unilateral action could start and Civil Service Commission wherein it is stated: "In no
complete the running of a term of office in the COMELEC case shall any Member be appointed in a temporary
without the consent of the Commission on Appointments. This or acting capacity." I detect in the Committee’s
interpretation renders inutile the confirming power of the proposed resolutions a constitutional hangover, if I
Commission on Appointments. may use the term, from the past administration. Am I
correct in concluding that the reason the Committee
The phrase "without reappointment" applies only to one who introduced this particular provision is to avoid an
incident similar to the case of the Honorable
has been appointed by the President and confirmed by the
Commission on Appointments, whether or not such person Francisco Tantuico who was appointed in an acting
completes his term of office. There must be a confirmation by capacity as Chairman of the Commission on Audit for
the Commission on Appointments of the previous appointment about 5 years from 1975 until 1980, and then in 1980,
before the prohibition on reappointment can apply. To hold was appointed as Chairman with a tenure of another
otherwise will lead to absurdities and negate the President’s 7 years. So, if we follow that appointment to (its)
power to make ad interim appointments. logical conclusion, he occupied that position for
about 12 years in violation of the Constitution?

In the great majority of cases, the Commission on


Appointments usually fails to act, for lack of time, on the ad MR. FOZ: It is only one of the
interim appointments first issued to appointees. If such ad considerations. Another is really to make sure that
interim appointments can no longer be renewed, the President any member who is appointed to any of the
will certainly hesitate to make ad interim appointments commissions does not serve beyond 7
years."60 (Emphasis supplied)
because most of her appointees will effectively be disapproved
by mere inaction of the Commission on Appointments. This will
nullify the constitutional power of the President to make ad Commissioner Christian Monsod further clarified the
interim appointments, a power intended to avoid disruptions in prohibition on reappointment in this manner:
vital government services. This Court cannot subscribe to a
proposition that will wreak havoc on vital government services. "MR. MONSOD. If the (Commissioner) will read the
whole Article, she will notice that there is no
The prohibition on reappointment is common to the three reappointment of any kind and, therefore as a whole
constitutional commissions. The framers of the present there is no way that somebody can serve for more
Constitution prohibited reappointments for two reasons. The than seven years. The purpose of the last sentence
first is to prevent a second appointment for those who have is to make sure that this does not happen by including
been previously appointed and confirmed even if they served in the appointment both temporary and acting
for less than seven years. The second is to insure that the capacities."61 (Emphasis supplied)
members of the three constitutional commissions do not serve
beyond the fixed term of seven years. As reported in Plainly, the prohibition on reappointment is intended to insure
the Journal of the Constitutional Commission, Commissioner that there will be no reappointment of any kind. On the other
Vicente B. Foz, who sponsored58 the proposed articles on the hand, the prohibition on temporary or acting appointments is
three constitutional commissions, outlined the four important intended to prevent any circumvention of the prohibition on
features of the proposed articles, to wit: reappointment that may result in an appointee’s total term of
office exceeding seven years. The evils sought to be avoided
"Mr. Foz stated that the Committee had introduced by the twin prohibitions are very specific - reappointment of any
basic changes in the common provision affecting the kind and exceeding one’s term in office beyond the maximum
three Constitutional Commissions, and which are: 1) period of seven years.
fiscal autonomy which provides (that) appropriations
shall be automatically and regularly released to the
FINALS CONSTITUTION I ACJUCO 159

Not contented with these ironclad twin prohibitions, the framers Petitioner claims that Benipayo has no authority to remove her
of the Constitution tightened even further the screws on those as Director IV of the EID and reassign her to the Law
who might wish to extend their terms of office. Thus, the word Department. Petitioner further argues that only the COMELEC,
"designated" was inserted to plug any loophole that might be acting as a collegial body, can authorize such reassignment.
exploited by violators of the Constitution, as shown in the Moreover, petitioner maintains that a reassignment without her
following discussion in the Constitutional Commission: consent amounts to removal from office without due process
and therefore illegal.
"MR. DE LOS REYES: On line 32, between the
words "appointed" and "in", I propose to insert the Petitioner’s posturing will hold water if Benipayo does not
words OR DESIGNATED so that the whole sentence possess any color of title to the office of Chairman of the
will read: "In no case shall any Member be appointed COMELEC. We have ruled, however, that Benipayo is the de
OR DESIGNATED in a temporary or acting capacity." jure COMELEC Chairman, and consequently he has full
authority to exercise all the powers of that office for so long as
THE PRESIDING OFFICER (Mr. Trenas): What does his ad interim appointment remains effective. Under Section 7
the Committee say? (4), Chapter 2, Subtitle C, Book V of the Revised Administrative
Code, the Chairman of the COMELEC is vested with the
following power:
MR. FOZ: But it changes the meaning of this
sentence. The sentence reads: "In no case shall any
Member be appointed in a temporary or acting "Section 7. Chairman as Executive Officer; Powers
capacity." and Duties. The Chairman, who shall be the Chief
Executive Officer of the Commission, shall:

MR. DE LOS REYES: Mr. Presiding Officer, the


reason for this amendment is that some lawyers xxx
make a distinction between an appointment and a
designation. The Gentleman will recall that in the (4) Make temporary assignments, rotate and transfer
case of Commissioner on Audit Tantuico, I think his personnel in accordance with the provisions of the
term exceeded the constitutional limit but the Minister Civil Service Law." (Emphasis supplied)
of Justice opined that it did not because he was only
designated during the time that he acted as The Chairman, as the Chief Executive of the COMELEC, is
Commissioner on Audit. So, in order to erase that expressly empowered on his own authority to transfer or
distinction between appointment and designation, we
reassign COMELEC personnel in accordance with the Civil
should specifically place the word so that there will Service Law. In the exercise of this power, the Chairman is not
be no more ambiguity. "In no case shall any Member
required by law to secure the approval of the COMELEC en
be appointed OR DESIGNATED in a temporary or banc.
acting capacity."

Petitioner’s appointment papers dated February 2, 1999,


MR. FOZ: The amendment is accepted, Mr.
February 15, 2000 and February 15, 2001, attached as
Presiding Officer. Annexes "X", "Y" and "Z" to her Petition, indisputably show that
she held her Director IV position in the EID only in an acting or
MR. DE LOS REYES: Thank you. temporary capacity.64 Petitioner is not a Career Executive
Service (CES) officer, and neither does she hold Career
THE PRESIDING OFFICER (Mr. Trenas): Is there Executive Service Eligibility, which are necessary
any objection? (Silence) The Chair hears none; the qualifications for holding the position of Director IV as
amendment is approved."62 prescribed in the Qualifications Standards (Revised 1987)
issued by the Civil Service Commission.65 Obviously, petitioner
does not enjoy security of tenure as Director IV. In Secretary
The ad interim appointments and subsequent renewals of of Justice Serafin Cuevas vs. Atty. Josefina G. Bacal,66 this
appointments of Benipayo, Borra and Tuason do not violate Court held that:
the prohibition on reappointments because there were no
previous appointments that were confirmed by the
Commission on Appointments. A reappointment presupposes "As respondent does not have the rank appropriate
a previous confirmed appointment. The same ad for the position of Chief Public Attorney, her
interim appointments and renewals of appointments will also appointment to that position cannot be considered
not breach the seven-year term limit because all the permanent, and she can claim no security of tenure
in respect of that position. As held in Achacoso v.
appointments and renewals of appointments of Benipayo,
Borra and Tuason are for a fixed term expiring on February 2, Macaraig:
2008.63 Any delay in their confirmation will not extend the
expiry date of their terms of office. Consequently, there is no ‘It is settled that a permanent appointment
danger whatsoever that the renewal of the ad can be issued only ‘to a person who meets
interim appointments of these three respondents will result in all the requirements for the position to
any of the evils intended to be exorcised by the twin which he is being appointed, including the
prohibitions in the Constitution. The continuing renewal of appropriate eligibility prescribed.’
the ad interim appointment of these three respondents, for so Achacoso did not. At best, therefore, his
long as their terms of office expire on February 2, 2008, does appointment could be regarded only as
not violate the prohibition on reappointments in Section 1 (2), temporary. And being so, it could be
Article IX-C of the Constitution. withdrawn at will by the appointing authority
and ‘at a moment’s notice’, conformably to
Fourth Issue: Respondent Benipayo’s Authority to Reassign established jurisprudence x x x.
Petitioner
The mere fact that a position belongs to the
Career Service does not automatically
FINALS CONSTITUTION I ACJUCO 160

confer security of tenure on its occupant teachers, within the election period except
even if he does not possess the required upon approval of the Commission.
qualifications. Such right will have to
depend on the nature of his appointment,
WHEREAS, the aforequoted provisions are
which in turn depends on his eligibility or applicable to the national and local elections on May
lack of it. A person who does not have the 14, 2001;
requisite qualifications for the position
cannot be appointed to it in the first place,
or as an exception to the rule, may be WHEREAS, there is an urgent need to appoint,
appointed to it merely in an acting capacity transfer or reassign personnel of the Commission on
in the absence of appropriate eligibles. The Elections during the prohibited period in order that it
appointment extended to him cannot be can carry out its constitutional duty to conduct free,
regarded as permanent even if it may be so orderly, honest, peaceful and credible elections;
designated x x x.’"
"NOW, THEREFORE, the Commission on Elections
Having been appointed merely in a temporary or acting by virtue of the powers conferred upon it by the
capacity, and not possessed of the necessary qualifications to Constitution, the Omnibus Election Code and other
hold the position of Director IV, petitioner has no legal basis in election laws, as an exception to the foregoing
claiming that her reassignment was contrary to the Civil prohibitions, has RESOLVED, as it is hereby
Service Law. This time, the vigorous argument of petitioner that RESOLVED, to appoint, hire new employees or fill
a temporary or acting appointment can be withdrawn or new positions and transfer or reassign its personnel,
revoked at the pleasure of the appointing power happens to when necessary in the effective performance of its
apply squarely to her situation. mandated functions during the prohibited period,
provided that the changes in the assignment of
its field personnel within the thirty-day period before
Still, petitioner assails her reassignment, carried out during the election day shall be effected after due notice and
election period, as a prohibited act under Section 261 (h) of the hearing." (Emphasis supplied)
Omnibus Election Code, which provides as follows:

The proviso in COMELEC Resolution No. 3300, requiring due


"Section 261. Prohibited Acts. The following shall be
notice and hearing before any transfer or reassignment can be
guilty of an election offense: made within thirty days prior to election day, refers only to
COMELEC field personnel and not to head office personnel
xxx like the petitioner. Under the Revised Administrative
Code,69 the COMELEC Chairman is the sole officer specifically
(h) Transfer of officers and employees in the civil vested with the power to transfer or reassign COMELEC
service - Any public official who makes or causes any personnel. The COMELEC Chairman will logically exercise the
transfer or detail whatever of any officer or employee authority to transfer or reassign COMELEC personnel
in the civil service including public school teachers, pursuant to COMELEC Resolution No. 3300. The
within the election period except upon prior approval COMELEC en banc cannot arrogate unto itself this power
of the Commission." because that will mean amending the Revised Administrative
Code, an act the COMELEC en banc cannot legally do.

Petitioner claims that Benipayo failed to secure the approval of


the COMELEC en banc to effect transfers or reassignments of COMELEC Resolution No. 3300 does not require that every
COMELEC personnel during the election period. 67 Moreover, transfer or reassignment of COMELEC personnel should carry
the concurrence of the COMELEC as a collegial body.
petitioner insists that the COMELEC en banc must concur to
every transfer or reassignment of COMELEC personnel during Interpreting Resolution No. 3300 to require such concurrence
will render the resolution meaningless since the COMELEC en
the election period.
banc will have to approve every personnel transfer or
reassignment, making the resolution utterly useless.
Contrary to petitioner’s allegation, the COMELEC did in fact Resolution No. 3300 should be interpreted for what it is, an
issue COMELEC Resolution No. 3300 dated November 6, approval to effect transfers and reassignments of personnel,
2000,68 exempting the COMELEC from Section 261 (h) of the without need of securing a second approval from the
Omnibus Election Code. The resolution states in part: COMELEC en banc to actually implement such transfer or
reassignment.
"WHEREAS, Sec. 56 and Sec. 261, paragraphs (g)
and (h), of the Omnibus Election Code provides as The COMELEC Chairman is the official expressly authorized
follows: by law to transfer or reassign COMELEC personnel. The
person holding that office, in a de jure capacity, is Benipayo.
xxx The COMELEC en banc, in COMELEC Resolution No. 3300,
approved the transfer or reassignment of COMELEC
personnel during the election period. Thus, Benipayo’s order
Sec. 261. Prohibited Acts. The following reassigning petitioner from the EID to the Law Department
shall be guilty of an election offense: does not violate Section 261 (h) of the Omnibus Election Code.
For the same reason, Benipayo’s order designating Cinco
xxx Officer-in-Charge of the EID is legally unassailable.

(h) Transfer of officers and employees in Fifth Issue: Legality of Disbursements to Respondents
the civil service – Any public official who
makes or causes any transfer or detail Based on the foregoing discussion, respondent Gideon C. De
whatever of any officer or employee in the Guzman, Officer-in-Charge of the Finance Services
civil service including public school Department of the Commission on Elections, did not act in
FINALS CONSTITUTION I ACJUCO 161

excess of jurisdiction in paying the salaries and other


emoluments of Benipayo, Borra, Tuason and Cinco.

WHEREFORE, the petition is dismissed for lack of merit. Costs


against petitioner.

SO ORDERED.
FINALS CONSTITUTION I ACJUCO 162

G.R. No. 112745 October 16, 1997 This is a report in the case of Assistant
Commissioner AQUILINO T. LARIN of the
AQUILINO T. LARIN, petitioner, Excise Tax Service, Bureau of Internal
Revenue, a presidential appointee, one of
vs.
THE EXECUTIVE SECRETARY, SECRETARY OF those convicted in Criminal Case Nos.
FINANCE, COMMISSIONER OF THE BUREAU OF 14208-14209, entitled "People of the
INTERNAL REVENUE AND THE COMMITTEE CREATED Philippines vs. Aquilino T. Larin, et. al."
TO INVESTIGATE THE ADMINISTRATIVE COMPLAINT referred to the Department of Finance by
AGAINST AQUILINO T. LARIN, COMPOSED OF the Commissioner of Internal Revenue.
FRUMENCIO A. LAGUSTAN, JOSE B. ALEJANDRINO AND
JAIME M. MAZA, respondents. The cases against Pareno and Evangelista
are being acted upon by the Bureau of
TORRES, JR., J.: Internal Revenue as they are non-
presidential appointees.

Challenged in this petition is the validity of petitioner's removal


from service as Assistant Commissioner of the Excise Tax xxx xxx xxx
Service of the Bureau of Internal Revenue. Incidentally, he
questions Memorandum Order No. 164 issued by the Office of It is clear from the foregoing that Mr. Larin
the President, which provides for the creation of "A Committee has been found beyond reasonable doubt
to Investigate the Administrative Complaint Against Aquilino T. to have committed acts constituting grave
Larin, Assistant Commissioner, Bureau of Internal Revenue" misconduct. Under the Civil Service Laws
as well as the investigation made in pursuance thereto, and and Rules which require only
Administrative Order No. 101 dated December 2, 1993 which preponderance of evidence, grave
found him guilty of grave misconduct in the administrative misconduct is punishable by dismissal.
charge and imposed upon him the penalty of dismissal from
office. Acting by authority of the President, Sr. Deputy Executive
Secretary Leonardo A. Quisumbing issued Memorandum
Likewise, petitioner seeks to assail the legality of Executive Order No. 164 dated August 25, 1993 which provides for the
Order No. 132, issued by President Ramos on October 26, creation of an Executive Committee to investigate the
1993, which provides for the "Streamlining of the Bureau of administrative charge against herein petitioner Aquilino T.
Internal Revenue," and of its implementing rules issued by the Larin. It states thus:
Bureau of Internal Revenue, namely: a) Administrative Order
No. 4-93, which provides for the "Organizational Structure and
A Committee is hereby created to
Statement of General Functions of Offices in the National investigate the administrative complaint
Office" and b) Administrative Order No. 5-93, which provides filed against Aquilino T. Larin, Assistant
for "Redefining the Areas of Jurisdiction and Renumbering of Commissioner, Bureau of Internal
Regional And District Offices." Revenue, to be composed of:

The antecedent facts of the instant case as succinctly related Atty. Frumencio A.
by the Solicitor General are as follows: Lagustan — Chairman
Assistant Executive
On September 18, 1992,1 a decision was rendered by the Secretary for
Sandiganbayan convicting herein petitioner Aquilino T. Larin, Legislation
Revenue Specific Tax Officer, then Assistant Commissioner of
the Bureau of Internal Revenue and his co-accused (except Mr. Jose B. Alejandro
Justino E. Galban, Jr.) of the crimes of violation of Section 268 — Member
(4) of the National Internal Revenue Code and Section 3 (e) of Presidential Assistant
R.A. 3019 in Criminal Cases Nos. 14208-14209, entitled
"People of the Philippines, Plaintiff vs. Aquilino T. Larin,
Teodoro T. Pareno, Justino E. Galban, Jr. and Potenciana N. Atty. Jaime M. Maza —
Evangelista, Accused," the dispositive portion of the judgment Member
reads: Assistant
Commissioner for
Inspector Services
WHEREFORE, judgment is now rendered
Bureau of Internal
in Criminal Cases Nos. 14208 and 14209 Revenue
convicting accused Assistant
Commissioner for Specific Tax AQUILINO
T. LARIN, Chief of the Alcohol Tax Division The Committee shall have all the powers
TEODORO P. PARENO, and Chief of the and prerogatives of (an) investigating
Revenue Accounting Division committee under the Administrative Code
POTENCIANA M. EVANGELISTA: of 1987 including the power to summon
witnesses, administer oath or take
testimony or evidence relevant to the
xxx xxx xxx investigation by subpoena ad testificandum
and subpoena duces tecum.
SO ORDERED.
xxx xxx xxx
The fact of petitioner's conviction was reported to the President
of the Philippines by the then Acting Finance Secretary Leong
The Committee shall convene immediately,
through a memorandum dated June 4, 1993. The conduct the investigation in the most
memorandum states, inter alia:
FINALS CONSTITUTION I ACJUCO 163

expeditious manner, and terminate the ground as the Office of the Ombudsman had already taken
same as soon as practicable from its first cognizance of the case and had caused the filing only of the
scheduled date of hearing. criminal charges against him, b) by res judicata, c) by double
jeopardy, and d) because to proceed with the case would be
xxx xxx xxx redundant, oppressive and a plain persecution against him.

Consequently, the Committee directed the petitioner to Meanwhile, the President issued the challenged Executive
Order No. 132 dated October 26, 1993 which mandates for the
respond to the administrative charge leveled against him
through a letter dated September 17, 1993, thus: streamlining of the Bureau of Internal Revenue. Under said
order, some positions and functions are either abolished,
renamed, decentralized or transferred to other offices, while
Presidential Memorandum Order No. 164 other offices are also created. The Excise Tax Service or the
dated August 25, 1993, a xerox copy of Specific Tax Service, of which petitioner was the Assistant
which is hereto attached for your ready Commissioner, was one of those offices that was abolished by
reference, created an Investigation said executive order.
Committee to look into the charges against
you which are also the subject of the
Criminal Cases No. 14208 and 14209 The corresponding implementing rules of Executive Order No.
entitled People of the Philippines 132, namely, Revenue Administrative Orders Nos. 4-93 and 5-
vs. Aquilino T . Larin, et. al. 93, were subsequently issued by the Bureau of Internal
Revenue.

The Committee has in its possession a


certified true copy of the Decision of the On October 27, 1993, or one day after the promulgation of
Sandiganbayan in the above-mentioned Executive Order No. 132, the President appointed the following
cases. as BIR Assistant Commissioners:

Pursuant to Presidential Memorandum 1. Bernardo A. Frianeza


Order No. 164, you are hereby directed to
file your position paper on the 2. Dominador L. Galura
aforementioned charges within seven (7)
days from receipt hereof . . . . 3. Jaime D. Gonzales

Failure to file the required position paper


4. Lilia C. Guillermo
shall be considered as a waiver on your
part to submit such paper or to be heard, in
which case, the Committee shall deem the 5. Rizalina S. Magalona
case submitted on the basis of the
documents and records at hand. 6. Victorino C.
Mamalateo
In compliance, petitioner submitted a letter dated September
30, 1993 which was addressed to Atty. Frumencio A. Lagustan, 7. Jaime M. Maza
the Chairman of the Investigating Committee. In said latter, he
asserts that,
8. Antonio N.
Pangilinan
The case being sub-judice, I may not,
therefore, comment on the merits of the
issues involved for fear of being cited in 9. Melchor S. Ramos
contempt of Court. This position paper is
thus limited to furnishing the Committee 10. Joel L. Tan-Torres
pertinent documents submitted with the
Supreme Court and other tribunal which
took cognizance of the case in the past, as Consequently, the President, in the assailed Administrative
follows: Order No. 101 dated December 2, 1993, found petitioner guilty
of grave misconduct in the administrative charge and imposed
upon him the penalty of dismissal with forfeiture of his leave
xxx xxx xxx credits and retirement benefits including disqualification for
reappointment in the government service.
The foregoing documents readily show that
am not administratively liable or criminally Aggrieved, petitioner filed directly with this Court the instant
culpable of the charges leveled against me, petition on December 13, 1993 to question basically his alleged
and that the aforesaid cases are mere unlawful removal from office.
persecutions caused to be filed and are
being orchestrated by taxpayers who were
prejudiced by multi-million peso On April 17, 1996 and while the instant petition is pending, this
assessments I caused to be issued against Court set aside the conviction of petitioner in Criminal Case
them in my official capacity as Assistant Nos. 14208 and 14209.
Commissioner, Excise Tax Office of the
Bureau of Internal Revenue. In his petition, petitioner challenged the authority of the
President to dismiss him from office. He argued that in so far
In the same letter, petitioner claims that the administrative as presidential appointees who are Career Executive Service
complaint against him is already barred: a) on jurisdictional Officers are concerned, the President exercises only the power
of control not the power to remove. He also averred that the
FINALS CONSTITUTION I ACJUCO 164

administrative investigation conducted under Memorandum Order No. 164, which created a committee to investigate the
Order No. 164 is void as it violated his right to due process. administrative charge against petitioner, was issued pursuant
According to him, the letter of the Committee dated September to the power of removal of the President. This power of
17, 1993 and his position paper dated September 30, 1993 are removal, however, is not an absolute one which accepts no
not sufficient for purposes of complying with the requirements reservation. It must be pointed out that petitioner is a career
of due process. He alleged that he was not informed of the service officer. Under the Administrative Code of 1987, career
administrative charges leveled against him nor was he given service is characterized by the existence of security of tenure,
official notice of his dismissal. as contra-distinguished from non-career service whose tenure
is co-terminus with that of the appointing authority or subject to
Petitioner likewise claimed that he was removed as a result of his pleasure, or limited to a period specified by law or to the
the reorganization made by the Executive Department in the duration of a particular project for which purpose the
BIR pursuant to Executive Order No. 132. Thus, he assailed employment was made. As a career service officer, petitioner
enjoys the right to security of tenure. No less than the 1987
said Executive Order No. 132 and its implementing rules,
namely, Revenue Administrative Orders 4-93 and 5-93 for Constitution guarantees the right of security of tenure of the
employees of the civil service. Specifically, Section 36 of P.D.
being ultra vires. He claimed that there is yet no law enacted
by Congress which authorizes the reorganization by the No. 807, as amended, otherwise known as Civil Service
Executive Department of executive agencies, particularly the Decree of the Philippines, is emphatic that career service
Bureau of Internal Revenue. He said that the reorganization officers and employees who enjoy security of tenure may be
sought to be effected by the Executive Department on the removed only for any of the causes enumerated in said law. In
basis of E.O. No. 132 is tainted with bad faith in apparent other words, the fact that petitioner is a presidential appointee
violation of Section 2 of R.A. 6656, otherwise known as the Act does not give the appointing authority the license to remove
him at will or at his pleasure for it is an admitted fact that he is
Protecting the Security of Tenure of Civil Service Officers and
Employees in the Implementation of Government likewise a career service officer who under the law is the
recipient of tenurial protection, thus, may only be removed for
Reorganization.
a cause and in accordance with procedural due process.

On the other hand. respondents contended that since


petitioner is a presidential appointee, he falls under the Was petitioner then removed from office for a legal cause
under a valid proceeding?
disciplining authority of the President. They also contended
that E.O. No. 132 and its implementing rules were validly
issued pursuant to Sections 48 and 62 of Republic Act No. Although the proceedings taken complied with the
7645. Apart from this, the other legal bases of E.O. No. 132 as requirements of procedural due process, this Court, however,
stated in its preamble are Section 63 of E.O. No. 127 considers that petitioner was not dismissed for a valid cause.
(Reorganizing the Ministry of Finance), and Section 20, Book
III of E.O. No. 292, otherwise known as the Administrative
It should be noted that what precipitated the creation of the
Code of 1987. In addition, it is clear that in Section 11 of R.A. investigative committee to look into the administrative charge
No. 6656 future reorganization is expressly contemplated and against petitioner is his conviction by the Sandiganbayan in
nothing in said law that prohibits subsequent reorganization Criminal Case Nos. 14208 and 14209. As admitted by the
through an executive order. Significantly, respondents clarified respondents, the administrative case against petitioner is
that petitioner was not dismissed by virtue of EO 132. based on the Sandiganbayan Decision of September 18, 1992.
Respondents claimed that he was removed from office Thus, in the Administrative Order No. 101 issued by Senior
because he was found guilty of grave misconduct in the
Deputy Executive Secretary Quisumbing which found
administrative cases filed against him. petitioner guilty of grave misconduct, it clearly states that:

The ultimate issue to be resolved in the instant case falls on This pertains to the administrative charge
the determination of the validity of petitioner's dismissal from against Assistant Commissioner Aquilino
office. Incidentally, in order to resolve this matter, it is T. Larin of the Bureau of Internal Revenue,
imperative that We consider these questions: a) Who has the
for grave misconduct by virtue of a
power to discipline the petitioner?, b) Were the proceedings Memorandum signed by Acting Secretary
taken pursuant to Memorandum Order No. 164 in accord with
Leong of the Department of Finance, on
due process?, c) What is the effect of petitioner's acquittal in the basis of a decision handed down by the
the criminal case to his administrative charge?, d) Does the Hon. Sandiganbayan convicting Larin, et.
President have the power to reorganize the BIR or to issue the al. in Criminal Case Nos. 14208 and
questioned E.O. NO. 132?, and e) Is the reorganization of BIR 14209.4
pursuant to E.O. No. 132 tainted with bad faith?

In a nutshell, the criminal cases against petitioner refer to his


At the outset, it is worthy to note that the position of Assistant alleged violation of Section 268 (4) of the National Internal
Commissioner of the BIR is part of the Career Executive Revenue Code and of Section 3 (e) of R.A. No. 3019 as a
Service.2 Under the law,3 Career Executive Service officers, consequence of his act of favorably recommending the grant
namely, Undersecretary, Assistant Secretary, Bureau Director, of tax credit to Tanduay Distillery, Inc.. The pertinent portion of
Assistant Bureau Director, Regional Director, Assistant
the judgment of the Sandiganbayan reads:
Regional Director, Chief of Department Service and other
officers of equivalent rank as may be identified by the Career
Executive Service Board, are all appointed by the President. As above pointed out, the accused had
Concededly, petitioner was appointed as Assistant conspired in knowingly preparing false
Commissioner in January, 1987 by then President Aquino. memoranda and certification in order to
Thus, petitioner is a presidential appointee who belongs to effect a fraud upon taxes due to the
career service of the Civil Service. Being a presidential government. By their separate acts which
appointee, he comes under the direct disciplining authority of had resulted in an appropriate tax credit of
the President. This is in line with the well settled principle that P180,701,682.00 in favor of Tanduay. The
the "power to remove is inherent in the power to appoint" government had been defrauded of a tax
conferred to the President by Section 16, Article VII of the revenue — for the full amount, if one is to
Constitution. Thus, it is ineluctably clear that Memorandum look at the availments or utilization thereof
FINALS CONSTITUTION I ACJUCO 165

(Exhibits "AA" to "AA- 31-a"), or for a documents attached as annexes to his letter, all of which are
substantial portion thereof evidences supporting his defense. Prior to this, he received a
(P73,000,000.00) if we are to rely on the letter dated September 17, 1993 from the Investigation
letter of Deputy Commissioner Eufracio D. Committee requiring him to explain his side concerning the
Santos (Exhibits "21" for all the accused). charge. It can not therefore be argued that petitioner was
denied of due process.
As pointed out above, the confluence of
acts and omissions committed by accused Let us now examine Executive Order No. 132.
Larin, Pareno and Evangelista adequately
prove conspiracy among them for no other As stated earlier, with the issuance of Executive Order No. 132,
purpose than to bring about a tax credit some of the positions and offices, including the office of Excise
which Tanduay did not deserve. These Tax Services of which petitioner was the Assistant
misrepresentations as to how much Commissioner, were abolished or otherwise decentralized.
Tanduay had paid in ad valorem taxes Consequently, the President released the list of appointed
obviously constituted a fraud of tax revenue Assistant Commissioners of the BIR. Apparently, petitioner
of the government . . . .5 was not included.

However, it must be stressed at this juncture that the conviction We do not agree.
of petitioner by the Sandiganbayan was set aside by this Court
in our decision promulgated on April 17, 1996 in G.R. Nos.
108037-38 and 107119-20. We specifically ruled in no Under its preamble, E.O. No. 132 lays down the legal bases of
uncertain terms that: a) petitioner can not be held negligent in its issuance, namely: a) Section 48 and 62 of R.A. No. 7645,
relying on the certification of a co-equal unit in the BIR, b) it is b) Section 63 of E.O. No. 127, and c) Section 20, Book III of
not incumbent upon Larin to go beyond the certification made E.O. No. 292.
by the Revenue Accounting Division that Tanduay Distillery,
Inc. had paid the ad valorem taxes, c) there is nothing irregular Section 48 of R.A. 7645 provides that:
or anything false in Larin's marginal note on the memorandum
addressed to Pareno, the Chief of Alcohol Tax Division who
was also one of the accused, but eventually acquitted, in the Sec. 48. Scaling Down and Phase Out of
said criminal cases, and d) there is no proof of actual Activities of Agencies Within the Executive
agreement between the accused, including petitioner, to Branch. — The heads of departments,
commit the illegal acts charged. We are emphatic in our bureaus and offices and agencies are
resolution in said cases that there is nothing "illegal with the hereby directed to identify their respective
acts committed by the petitioner(s)." We also declare that activities which are no longer essential in
"there is no showing that petitioner(s) had acted irregularly, or the delivery of public services and which
performed acts outside of his (their) official functions." may be scaled down, phased out or
Significantly, these acts which. We categorically declare to be abolished, subject to civil service rules and
not unlawful and improper in G.R. Nos. 108037-38 and G.R. regulations. . . . Actual scaling down,
Nos. 107119-20 are the very same acts for which petitioner is phasing out or abolition of the activities
held to be administratively responsible. Any charge of shall be effected pursuant to Circulars or
malfeasance or misfeasance on the part of the petitioner is Orders issued for the purpose by the Office
clearly belied by our conclusion in said cases. In the light of of the President. (emphasis ours)
this decisive pronouncement, We see no reason for the
administrative charge to continue — it must, thus, be Said provision clearly mentions the acts of "scaling down,
dismissed. phasing out and abolition" of offices only and does not cover
the creation of offices or transfer of functions. Nevertheless,
We are not unaware of the rule that since administrative cases the act of creating and decentralizing is included in the
are independent from criminal actions for the same act or subsequent provision of Section 62, which provides that:
omission, the dismissal or acquittal of the criminal charge does
not foreclose the institution of administrative action nor carry Sec. 62. Unauthorized organizational
with it the relief from administrative liability. 6 However, the charges. — Unless otherwise created by
circumstantial setting of the instant case sets it miles apart from law or directed by the President of the
the foregoing rule and placed it well within the exception. Philippines, no organizational unit of
Corollarily, where the very basis of the administrative case charges in key positions in any department
against petitioner is his conviction in the criminal action which or agency shall be authorized in their
was later on set aside by this Court upon a categorical and respective organization structures and be
clear finding that the acts for which he was administratively funded from appropriations by this Act.
held liable are not unlawful and irregular, the acquittal of the (emphasis ours)
petitioner in the criminal case necessarily entails the dismissal
of the administrative action against him, because in such a
case, there is no more basis nor justifiable reason to maintain The foregoing provision evidently shows that the President is
the administrative suit. authorized to effect organizational charges including the
creation of offices in the department or agency concerned.

On the aspect of procedural due process, suffice it to say that


petitioner was given every chance to present his side. The rule The contention of petitioner that the two provisions are riders
is well settled that the essence of due process in administrative deserves scant consideration. Well settled is the rule that every
proceedings is that a party be afforded a reasonable law has in its favor the presumption of constitutionality. 8 Unless
opportunity to be heard and to submit any evidence he may and until a specific provision of the law is declared invalid and
have in support of his defense.7 The records clearly show that unconstitutional, the same is valid and biding for all intents and
on October 1, 1993 petitioner submitted his letter-response purposes.
dated September 30, 1993 to the administrative charge filed
against him. Aside from his letter, he also submitted various
FINALS CONSTITUTION I ACJUCO 166

Another legal basis of E.O. No. 132 is Section 20, Book III of As a general rule, a reorganization is
E.O. No. 292 which states: carried out in "good faith" if it is for the
purpose of economy or to make
bureaucracy more efficient. In that event no
Sec. 20. Residual Powers. — Unless
Congress provides otherwise, the dismissal or separation actually occurs
President shall exercise such other powers because the position itself ceases to exist.
and functions vested in the President which And in that case the security of tenure
are provided for under the laws and which would not be a Chinese wall. Be that as it
are not specifically enumerated above or may, if the abolition which is nothing else
which are not delegated by the President in but a separation or removal, is done for
accordance with law. (emphasis ours) political reasons or purposely to defeat
security of tenure, or otherwise not in good
faith, no valid abolition takes place and
This provision speaks of such other powers vested in the whatever abolition is done is void ab initio.
President under the law. What law then which gives him the There is an invalid abolition as where there
power to reorganize? It is Presidential Decree No. 17729 which is merely a change of nomenclature of
amended Presidential Decree No. 1416. These decrees positions or where claims of economy are
expressly grant the President of the Philippines the continuing belied by the existence of ample funds. 11
authority to reorganize the national government, which
includes the power to group, consolidate bureaus and
agencies, to abolish offices, to transfer functions, to create and In this regard, it is worth mentioning that Section 2 of R. A. No.
classify functions, services and activities and to standardize 6656 lists down the circumstances evidencing bad faith in the
salaries and materials. The validity of these two decrees are removal of employees as a result of the reorganization, thus:
unquestionable. The 1987 Constitution clearly provides that
"all laws, decrees, executive orders, proclamations, letters of Sec. 2. No officer or employee in the career
instructions and other executive issuances not inconsistent service shall be removed except for a valid
with this Constitution shall remain operative until amended, cause and after due notice and hearing. A
repealed or revoked."10 So far, there is yet no law amending or valid cause for removal exists when,
repealing said decrees. Significantly, the Constitution itself pursuant to a bona fide reorganization, a
recognizes future reorganizations in the government as what position has been abolished or rendered
is revealed in Section 16 of Article XVIII, thus: redundant or there is a need to merge,
divide, or consolidate positions in order to
meet the exigencies of the service, or other
Sec. 16. Career civil service employees
separated from service not for cause but as lawful causes allowed by the Civil Service
Law. The existence of any or some of the
a result of the . . . reorganization following
the ratification of this Constitution shall be following circumstances may be
entitled to appropriate separation pay . . . considered as evidence of bad faith in the
removals made as a result of the
reorganization, giving rise to a claim for
However, We can not consider E.O. No. 127 signed on reinstatement or reappointment by an
January 30, 1987 as a legal basis for the reorganization of the aggrieved party:
BIR. E.O. No. 127 should be related to the second paragraph
of Section 11 of Republic Act No. 6656.
a) Where there is a significant increase in
the number of positions in the new staffing
Section 11 provides inter alia: pattern of the department or agency
concerned;
xxx xxx xxx
b) Where an office is abolished and another
In the case of the 1987 reorganization of performing substantially the same
the executive branch, all departments and functions is created;
agencies which are authorized by
executive orders promulgated by the c) Where incumbents are replaced by
President to reorganize shall have ninety those less qualified in terms of status of
days from the approval of this act within appointment, performance and merit;
which to implement their respective
reorganization plans in accordance with the
provisions of this Act. (emphasis ours) d) Where there is a reclassification of
offices in the department or agency
concerned and the reclassified offices
Executive Order No. 127 was part of the 1987 reorganization perform substantially the same functions as
contemplated under said provision. Obviously, it had become the original offices;
stale by virtue of the expiration of the ninety day deadline
period. It can not thus be used as a proper basis for the
reorganization of the BIR. Nevertheless, as shown earlier, e) Where the removal violates the order of
there are other legal bases to sustain the authority of the separation provided in Section 3 hereof.
President to issue the questioned E.O. NO. 132.
A reading of some of the provisions of the questioned E.O. No.
While the President's power to reorganize can not be denied, 132 clearly leads us to an inescapable conclusion that there
this does not mean however that the reorganization itself is are circumstances considered as evidences of bad faith in the
properly made in accordance with law. Well-settled is the rule reorganization of the BIR.
that reorganization is regarded as valid provided it is pursued
in good faith. Thus, in Dario vs. Mison, this Court has had the Section 1.1.2 of said executive order provides that:
occasion to clarify that:
FINALS CONSTITUTION I ACJUCO 167

1.1.2 The Intelligence and Investigation


Office and the Inspection Service are
abolished. An Intelligence and
Investigation Service is
hereby created to absorb the same
functions of the abolished office and
service. . . . (emphasis ours)

This provision is a clear illustration of the circumstance


mentioned in Section 2 (b) of R.A. No. 6656 that an office is
abolished and another one performing substantially the same
function is created.

Another circumstance is the creation of services and divisions


in the BIR resulting to a significant increase in the number of
positions in the said bureau as contemplated in paragraph (a)
of Section 2 of R.A. No. 6656. Under Section 1.3 of E.O. No.
132, the Information Systems Group has two newly created
Systems Services. Aside from this, six new divisions are also
created. Under Section 1.2.1, three more divisions of the
Assessment Service are formed. With these newly created
offices, there is no doubt that a significant increase of positions
will correspondingly follow.

Furthermore, it is perceivable that the non-reappointment of


the petitioner as Assistant Commissioner violates Section 4 of
R.A. No. 6656. Under said provision, officers holding
permanent appointments are given preference for appointment
to the new positions in the approved staffing pattern
comparable to their former positions or in case there are not
enough comparable positions to positions next lower in rank. It
is undeniable that petitioner is a career executive officer who
is holding a permanent position. Hence, he should have been
given preference for appointment in the position of Assistant
Commissioner. As claimed by petitioner, Antonio Pangilinan
who was one of those appointed as Assistant Commissioner,
"is an outsider of sorts to the Bureau, not having been an
incumbent officer of the Bureau at the time of the
reorganization." We should not lose sight of the second
paragraph of Section 4 of R.A. No. 6656 which explicitly states
that no new employees shall be taken in until all permanent
officers shall have been appointed for permanent position.

IN VIEW OF THE FOREGOING, the petition is granted, and


petitioner is hereby reinstated to his position as Assistant
Commissioner without loss of seniority rights and shall be
entitled to full backwages from the time of his separation from
service until actual reinstatement unless, in the meanwhile, he
would have reached the compulsory retirement age of sixty-
five years in which case, he shall be deemed to have retired at
such age and entitled thereafter to the corresponding
retirement benefits.

SO ORDERED.

Narvasa, C.J., Davide, Jr., Romero, Bellosillo, Melo, Puno,


Vitug, Kapunan, Mendoza, Francisco, Hermosisima, Jr. and
Panganiban, JJ., concur.
FINALS CONSTITUTION I ACJUCO 168

G.R. No. 206666 January 21, 2015 declares the forfeiture in favor of the government of the
following:
ATTY. ALICIA RISOS-VIDAL, Petitioner,
ALFREDO S. LIM Petitioner-Intervenor, (1) The total amount of Five Hundred Forty[-]Two
vs. Million Seven Hundred Ninety[-]One Thousand
COMMISSION ON ELECTIONS and JOSEPH EJERCITO Pesos (₱545,291,000.00), with interest and income
ESTRADA, Respondents. earned, inclusive of the amount of Two Hundred
Million Pesos (₱200,000,000.00), deposited in the
DECISION name and account of the Erap Muslim Youth
Foundation.

LEONARDO-DE CASTRO, J.:


(2) The amount of One Hundred Eighty[-]Nine Million
Pesos (₱189,000,000.00), inclusive of interests and
Before the Court are (1) a Petition for Certiorari filed under Rule income earned, deposited in the Jose Velarde
64, in relation to Rule 65, both of the Revised Rules of Court, account.
by Atty. Alicia Risos-Vidal (Risos-Vidal), which essentially
prays for the issuance of the writ of certiorari annulling and
setting aside the April 1, 20131 and April 23, 20132 Resolutions (3) The real property consisting of a house and lot
dubbed as "Boracay Mansion" located at #100 11th
of the Commission on Elections (COMELEC), Second Division
and En bane, respectively, in SPA No. 13-211 (DC), entitled Street, New Manila, Quezon City.
"Atty. Alicia Risos-Vidal v. Joseph Ejercito Estrada" for having
been rendered with grave abuse of discretion amounting to The cash bonds posted by accused Jose "Jinggoy" Estrada
lack or excess of jurisdiction; and (2) a Petition-in- and Atty. Edward S. Serapio are hereby ordered cancelled and
Intervention3 filed by Alfredo S. Lim (Lim), wherein he prays to released to the said accused or their duly authorized
be declared the 2013 winning candidate for Mayor of the City representatives upon presentation of the original receipt
of Manila in view of private respondent former President evidencing payment thereof and subject to the usual
Joseph Ejercito Estrada’s (former President Estrada) accounting and auditing procedures. Likewise, the hold-
disqualification to run for and hold public office. departure orders issued against the said accused are hereby
recalled and declared functus oficio. 4
The Facts
On October 25, 2007, however, former President Gloria
The salient facts of the case are as follows: Macapagal Arroyo (former President Arroyo) extended
executive clemency, by way of pardon, to former President
Estrada. The full text of said pardon states:
On September 12, 2007, the Sandiganbayan convicted former
President Estrada, a former President of the Republic of the
Philippines, for the crime of plunder in Criminal Case No. MALACAÑAN PALACE
26558, entitled "People of the Philippines v. Joseph Ejercito MANILA
Estrada, et al." The dispositive part of the graft court’s decision
reads: By the President of the Philippines

WHEREFORE, in view of all the foregoing, judgment is hereby PARDON


rendered in Criminal Case No. 26558 finding the accused,
Former President Joseph Ejercito Estrada, GUILTY beyond WHEREAS, this Administration has a policy of releasing
reasonable doubt of the crime of PLUNDER, defined in and inmates who have reached the age of seventy (70),
penalized by Republic Act No. 7080, as amended. On the other
hand, for failure of the prosecution to prove and establish their
guilt beyond reasonable doubt, the Court finds the accused WHEREAS, Joseph Ejercito Estrada has been under detention
Jose "Jinggoy" Estrada and Atty. Edward S. Serapio NOT for six and a half years,
GUILTY of the crime of plunder, and accordingly, the Court
hereby orders their ACQUITTAL. WHEREAS, Joseph Ejercito Estrada has publicly committed to
no longer seek any elective position or office,
The penalty imposable for the crime of plunder under Republic
Act No. 7080, as amended by Republic Act No. 7659, is IN VIEW HEREOF and pursuant to the authority conferred
Reclusion Perpetua to Death. There being no aggravating or upon me by the Constitution, I hereby grant executive
mitigating circumstances, however, the lesser penalty shall be clemency to JOSEPH EJERCITO ESTRADA, convicted by the
applied in accordance with Article 63 of the Revised Penal Sandiganbayan of Plunder and imposed a penalty of Reclusion
Code. Accordingly, the accused Former President Joseph Perpetua. He is hereby restored to his civil and political rights.
Ejercito Estrada is hereby sentenced to suffer the penalty of
Reclusion Perpetua and the accessory penalties of civil
interdiction during the period of sentence and perpetual The forfeitures imposed by the Sandiganbayan remain in force
absolute disqualification. and in full, including all writs and processes issued by the
Sandiganbayan in pursuance hereof, except for the bank
account(s) he owned before his tenure as President.
The period within which accused Former President Joseph
Ejercito Estrada has been under detention shall be credited to
him in full as long as he agrees voluntarily in writing to abide Upon acceptance of this pardon by JOSEPH EJERCITO
by the same disciplinary rules imposed upon convicted ESTRADA, this pardon shall take effect.
prisoners.
Given under my hand at the City of Manila, this 25th Day of
Moreover, in accordance with Section 2 of Republic Act No. October, in the year of Our Lord, two thousand and seven.
7080, as amended by Republic Act No. 7659, the Court hereby
FINALS CONSTITUTION I ACJUCO 169

Gloria M. Arroyo (sgd.) (a) Those sentenced by final judgment for an offense
involving moral turpitude or for an offense punishable
By the President: by one (1) year or more of imprisonment, within two
(2) years after serving sentence; (b) Those removed
from office as a result of an administrative case;
IGNACIO R. BUNYE (sgd.)
Acting Executive Secretary5
(c) Those convicted by final judgment for violating the
oath of allegiance to the Republic;
On October 26, 2007, at 3:35 p.m., former President Estrada
"received and accepted"6 the pardon by affixing his signature
(d) Those with dual citizenship;
beside his handwritten notation thereon.

On November 30, 2009, former President Estrada filed a (e) Fugitives from justice in criminal or nonpolitical
Certificate of Candidacy7 for the position of President. During cases here or abroad;
that time, his candidacy earned three oppositions in the
COMELEC: (1) SPA No. 09-024 (DC), a "Petition to Deny Due (f) Permanent residents in a foreign country or those
Course and Cancel Certificate of Candidacy" filed by Rev. Elly who have acquired the right to reside abroad and
Velez B. Lao Pamatong, ESQ; (2) SPA No. 09-028 (DC), a continue to avail of the same right after the effectivity
petition for "Disqualification as Presidential Candidate" filed by of this Code; and
Evilio C. Pormento (Pormento); and (3) SPA No. 09-104 (DC),
a "Petition to Disqualify Estrada Ejercito, Joseph M.from
(g) The insane or feeble minded. (Emphasis
Running as President due to Constitutional Disqualification supplied.)
and Creating Confusion to the Prejudice of Estrada, Mary Lou
B" filed by Mary Lou Estrada. In separate Resolutions 8 dated
January 20, 2010 by the COMELEC, Second Division, Sec. 12, Omnibus Election Code:
however, all three petitions were effectively dismissed on the
uniform grounds that (i) the Constitutional proscription on Section 12. Disqualifications. - Any person who has been
reelection applies to a sitting president; and (ii) the pardon declared by competent authority insane or incompetent, or has
granted to former President Estrada by former President been sentenced by final judgmentfor subversion, insurrection,
Arroyo restored the former’s right to vote and be voted for a rebellion, or for any offense for which he has been sentenced
public office. The subsequent motions for reconsideration to a penalty of more than eighteen months or for a crime
thereto were denied by the COMELEC En banc. involving moral turpitude, shall be disqualified to be a
candidate and to hold any public office, unless he has been
After the conduct of the May 10, 2010 synchronized elections, given plenary pardon or granted amnesty. (Emphases
however, former President Estrada only managed to garner supplied.)
the second highest number of votes.
In a Resolution dated April 1, 2013,the COMELEC, Second
Of the three petitioners above-mentioned, only Pormento Division, dismissed the petition for disqualification, the fallo of
sought recourse to this Court and filed a petition for certiorari, which reads:
which was docketed as G.R. No. 191988, entitled "Atty. Evilio
C. Pormento v. Joseph ‘ERAP’ Ejercito Estrada and WHEREFORE, premises considered, the instant petition is
Commission on Elections." But in a Resolution9 dated August hereby DISMISSED for utter lack of merit.12
31, 2010, the Court dismissed the aforementioned petition on
the ground of mootness considering that former President
Estrada lost his presidential bid. The COMELEC, Second Division, opined that "[h]aving taken
judicial cognizance of the consolidated resolution for SPA No.
09-028 (DC) and SPA No. 09-104 (DC) and the 10 May 2010
On October 2, 2012, former President Estrada once more En Banc resolution affirming it, this Commission will not be
ventured into the political arena, and filed a Certificate of labor the controversy further. Moreso, [Risos-Vidal] failed to
Candidacy,10 this time vying for a local elective post, that ofthe present cogent proof sufficient to reverse the standing
Mayor of the City of Manila. pronouncement of this Commission declaring categorically that
[former President Estrada’s] right to seek public office has
On January 24, 2013, Risos-Vidal, the petitioner in this case, been effectively restored by the pardon vested upon him by
filed a Petition for Disqualification against former President former President Gloria M. Arroyo. Since this Commission has
Estrada before the COMELEC. The petition was docketed as already spoken, it will no longer engage in disquisitions of a
SPA No. 13-211 (DC). Risos Vidal anchored her petition on the settled matter lest indulged in wastage of government
theory that "[Former President Estrada] is Disqualified to Run resources."13
for Public Office because of his Conviction for Plunder by the
Sandiganbayan in Criminal Case No. 26558 entitled ‘People of The subsequent motion for reconsideration filed by Risos-Vidal
the Philippines vs. Joseph Ejercito Estrada’ Sentencing Him to was denied in a Resolution dated April 23, 2013.
Suffer the Penalty of Reclusion Perpetuawith Perpetual
Absolute Disqualification."11 She relied on Section 40 of the
Local Government Code (LGC), in relation to Section 12 of the On April 30, 2013, Risos-Vidal invoked the Court’s jurisdiction
Omnibus Election Code (OEC), which state respectively, that: by filing the present petition. She presented five issues for the
Court’s resolution, to wit:
Sec. 40, Local Government Code:
I. RESPONDENT COMELEC COMMITTED GRAVE
ABUSE OF DISCRETION AMOUNTING TO LACK
SECTION 40. Disqualifications.- The following persons are OR EXCESS OF JURISDICTION IN HOLDING
disqualified from running for any elective local position: THAT RESPONDENT ESTRADA’S PARDON WAS
NOT CONDITIONAL;
FINALS CONSTITUTION I ACJUCO 170

II. RESPONDENT COMELEC COMMITTED GRAVE President Estrada is qualified to vote and be voted for in public
ABUSE OF DISCRETION AMOUNTING TO LACK office as a result of the pardon granted to him by former
OR EXCESS OF JURISDICTION IN NOT FINDING President Arroyo.
THAT RESPONDENT ESTRADA IS DISQUALIFIED
TO RUN AS MAYOR OF MANILA UNDER SEC. 40 In her petition, Risos-Vidal starts her discussion by pointing out
OF THE LOCAL GOVERNMENTCODE OF 1991 that the pardon granted to former President Estrada was
FOR HAVING BEEN CONVICTED OF PLUNDER, conditional as evidenced by the latter’s express acceptance
AN OFFENSE INVOLVING MORAL TURPITUDE; thereof. The "acceptance," she claims, is an indication of the
conditional natureof the pardon, with the condition being
III. RESPONDENT COMELEC COMMITTED embodied in the third Whereas Clause of the pardon, i.e.,
GRAVE ABUSE OF DISCRETION AMOUNTING TO "WHEREAS, Joseph Ejercito Estrada has publicly committed
LACK OR EXCESS OF JURISDICTION IN to no longer seek any elective position or office." She explains
DISMISSING THE PETITION FOR that the aforementioned commitment was what impelled
DISQUALIFICATION ON THE GROUND THAT THE former President Arroyo to pardon former President Estrada,
CASE INVOLVES THE SAME OR SIMILAR ISSUES without it, the clemency would not have been extended. And
IT ALREADY RESOLVED IN THE CASES OF any breach thereof, that is, whenformer President Estrada filed
"PORMENTO VS. ESTRADA", SPA NO. 09-028 his Certificate of Candidacy for President and Mayor of the City
(DC) AND IN "RE: PETITION TO DISQUALIFY of Manila, he breached the condition of the pardon; hence, "he
ESTRADA EJERCITO, JOSEPH M. FROM ought to be recommitted to prison to serve the unexpired
RUNNING AS PRESIDENT, ETC.," SPA NO. 09-104 portion of his sentence x x x and disqualifies him as a
(DC); candidate for the mayoralty [position] of Manila." 16

IV. RESPONDENT COMELEC COMMITTED Nonetheless, Risos-Vidal clarifies that the fundamental basis
GRAVE ABUSE OF DISCRETION AMOUNTING TO upon which former President Estrada mustbe disqualified from
LACK OR EXCESS OF JURISDICTION IN NOT running for and holding public elective office is actually the
RULING THAT RESPONDENT ESTRADA’S proscription found in Section 40 of the LGC, in relation to
PARDON NEITHER RESTORED HIS RIGHT OF Section 12 ofthe OEC. She argues that the crime of plunder is
SUFFRAGE NOR REMITTED HIS PERPETUAL both an offense punishable by imprisonment of one year or
ABSOLUTE DISQUALIFICATION FROM SEEKING more and involving moral turpitude; such that former President
PUBLIC OFFICE; and Estrada must be disqualified to run for and hold public elective
office.
V. RESPONDENT COMELEC COMMITTED
GRAVE ABUSE OF DISCRETION AMOUNTING TO Even with the pardon granted to former President Estrada,
LACK OR EXCESS OF JURISDICTION IN NOT however, Risos-Vidal insists that the same did not operate to
HAVING EXERCISED ITS POWER TO MOTU make available to former President Estrada the exception
PROPRIO DISQUALIFY RESPONDENT ESTRADA provided under Section 12 of the OEC, the pardon being
IN THE FACE OF HIS PATENT merely conditional and not absolute or plenary. Moreover,
DISQUALIFICATION TO RUN FOR PUBLIC Risos-Vidal puts a premium on the ostensible requirements
OFFICE BECAUSE OF HIS PERPETUAL AND provided under Articles 36 and 41 of the Revised Penal Code,
ABSOLUTE DISQUALIFICATION TO SEEK to wit:
PUBLIC OFFICE AND TO VOTE RESULTING
FROM HIS CRIMINAL CONVICTION FOR ART. 36. Pardon; its effects.– A pardon shall not work the
PLUNDER.14 restoration of the right to hold publicoffice, or the right of
suffrage, unless such rights be expressly restored by the terms
While this case was pending beforethe Court, or on May 13, of the pardon.
2013, the elections were conducted as scheduled and former
President Estrada was voted into office with 349,770 votes cast A pardon shall in no case exempt the culprit from the payment
in his favor. The next day, the local board of canvassers of the civil indemnity imposed upon him by the sentence.
proclaimed him as the duly elected Mayor of the City of Manila.

xxxx
On June 7, 2013, Lim, one of former President Estrada’s
opponents for the position of Mayor, moved for leave to
intervene in this case. His motion was granted by the Court in ART. 41. Reclusion perpetua and reclusion temporal – Their
a Resolution15 dated June 25, 2013. Lim subscribed to Risos- accessory penalties.– The penalties of reclusion perpetua and
Vidal’s theory that former President Estrada is disqualified to reclusion temporal shall carry with them that of civil interdiction
run for and hold public office as the pardon granted to the latter for life or during the period of the sentence as the case may
failed to expressly remit his perpetual disqualification. Further, be, and that of perpetual absolute disqualification which the
given that former President Estrada is disqualified to run for offender shall suffer even though pardoned as to the principal
and hold public office, all the votes obtained by the latter should penalty, unless the same shall have been expressly remitted
be declared stray, and, being the second placer with 313,764 in the pardon. (Emphases supplied.)
votes to his name, he (Lim) should be declared the rightful
winning candidate for the position of Mayor of the City of She avers that in view of the foregoing provisions of law, it is
Manila. not enough that a pardon makes a general statement that such
pardon carries with it the restoration of civil and political rights.
The Issue By virtue of Articles 36 and 41, a pardon restoring civil and
political rights without categorically making mention what
specific civil and political rights are restored "shall not work to
Though raising five seemingly separate issues for resolution,
restore the right to hold public office, or the right of suffrage;
the petition filed by Risos-Vidal actually presents only one nor shall it remit the accessory penalties of civil interdiction and
essential question for resolution by the Court, that is, whether perpetual absolute disqualification for the principal penalties of
or not the COMELEC committed grave abuse of discretion reclusion perpetua and reclusion temporal." 17 In other words,
amounting to lack or excess of jurisdiction in ruling that former
FINALS CONSTITUTION I ACJUCO 171

she considers the above constraints as mandatory Court;" that he "was granted an absolute pardon and thereby
requirements that shun a general or implied restoration of civil restored to his full civil and political rights, including the right to
and political rights in pardons. seek public elective office such as the mayoral (sic) position in
the City of Manila;" that "the majority decision in the case of
Risos-Vidal cites the concurring opinions of Associate Justices Salvacion A. Monsanto v. Fulgencio S. Factoran, Jr.,which was
Teodoro R. Padilla and Florentino P. Feliciano in Monsanto v. erroneously cited by both Vidal and Lim as authority for their
Factoran, Jr.18 to endorse her position that "[t]he restoration of respective claims, x x x reveal that there was no discussion
the right to hold public office to one who has lost such right by whatsoever in the ratio decidendi of the Monsanto case as to
reason of conviction in a criminal case, but subsequently the alleged necessity for an expressed restoration of the ‘right
pardoned, cannot be left to inference, no matter how intensely to hold public office in the pardon’ as a legal prerequisite to
arguable, but must be statedin express, explicit, positive and remove the subject perpetual special disqualification;" that
specific language." moreover, the "principal question raised in this Monsanto case
is whether or not a public officer, who has been granted an
absolute pardon by the Chief Executive, is entitled to
Applying Monsantoto former President Estrada’s case, Risos- reinstatement toher former position without need of a new
Vidal reckons that "such express restoration is further appointment;" that his "expressed acceptance [of the pardon]
demanded by the existence of the condition in the [third] is not proof that the pardon extended to [him] is conditional and
[W]hereas [C]lause of the pardon x x x indubitably indicating not absolute;" that this case is a mere rehash of the casesfiled
that the privilege to hold public office was not restored to against him during his candidacy for President back in 2009-
him."19 2010; that Articles 36 and 41 of the Revised Penal Code
"cannot abridge or diminish the pardoning power of the
On the other hand, the Office ofthe Solicitor General (OSG) for President expressly granted by the Constitution;" that the text
public respondent COMELEC, maintains that "the issue of of the pardon granted to him substantially, if not fully, complied
whether or not the pardon extended to [former President with the requirement posed by Article 36 of the Revised Penal
Estrada] restored his right to run for public office had already Code as it was categorically stated in the said document that
been passed upon by public respondent COMELEC way back he was "restored to his civil and political rights;" that since
in 2010 via its rulings in SPA Nos. 09-024, 09-028 and 09-104, pardon is an act of grace, it must be construed favorably in
there is no cogent reason for it to reverse its standing favor of the grantee;25 and that his disqualification will result in
pronouncement and declare [former President Estrada] massive disenfranchisement of the hundreds of thousands of
disqualified to run and be voted as mayor of the City of Manila Manileños who voted for him.26
in the absence of any new argument that would warrant its
reversal. To be sure, public respondent COMELEC correctly The Court's Ruling
exercised its discretion in taking judicial cognizance of the
aforesaid rulings which are known toit and which can be
The petition for certiorari lacks merit.
verified from its own records, in accordance with Section 2,
Rule 129 of the Rules of Court on the courts’ discretionary
power to take judicial notice of matters which are of public Former President Estrada was granted an absolute pardon
knowledge, orare capable of unquestionable demonstration, or that fully restored allhis civil and political rights, which naturally
ought to be known to them because of their judicial includes the right to seek public elective office, the focal point
functions."20 of this controversy. The wording of the pardon extended to
former President Estrada is complete, unambiguous, and
Further, the OSG contends that "[w]hile at first glance, it is unqualified. It is likewise unfettered by Articles 36 and 41 of the
apparent that [former President Estrada’s] conviction for Revised Penal Code. The only reasonable, objective, and
plunder disqualifies him from running as mayor of Manila under constitutional interpretation of the language of the pardon is
Section 40 of the [LGC], the subsequent grant of pardon to him, that the same in fact conforms to Articles 36 and 41 of the
however, effectively restored his right to run for any public Revised Penal Code. Recall that the petition for disqualification
office."21 The restoration of his right to run for any public office filed by Risos-Vidal against former President Estrada,
docketed as SPA No. 13-211 (DC), was anchored on Section
is the exception to the prohibition under Section 40 of the LGC,
as provided under Section 12 of the OEC. As to the seeming 40 of the LGC, in relation to Section 12 of the OEC, that is,
having been convicted of a crime punishable by imprisonment
requirement of Articles 36 and 41 of the Revised Penal Code,
i.e., the express restoration/remission of a particular right to be of one year or more, and involving moral turpitude, former
stated in the pardon, the OSG asserts that "an airtight and rigid President Estrada must be disqualified to run for and hold
interpretation of Article 36 and Article 41 of the [RPC] x x x public elective office notwithstanding the fact that he is a
would be stretching too much the clear and plain meaning of grantee of a pardon that includes a statement expressing "[h]e
the aforesaid provisions."22 Lastly, taking into consideration is hereby restored to his civil and political rights." Risos-Vidal
the third Whereas Clause of the pardon granted to former theorizes that former President Estrada is disqualified from
running for Mayor of Manila inthe May 13, 2013 Elections, and
President Estrada, the OSG supports the position that it "is not
an integral part of the decree of the pardon and cannot remains disqualified to hold any local elective post despite the
presidential pardon extended to him in 2007 by former
therefore serve to restrict its effectivity."23
President Arroyo for the reason that it (pardon) did not
expressly provide for the remission of the penalty of perpetual
Thus, the OSG concludes that the "COMELEC did not commit absolute disqualification, particularly the restoration of his
grave abuse of discretion amounting to lack or excess of (former President Estrada) right to vote and bevoted upon for
jurisdiction in issuing the assailed Resolutions." 24 public office. She invokes Articles 36 and 41 of the Revised
Penal Code as the foundations of her theory.
For his part, former President Estrada presents the following
significant arguments to defend his stay in office: that "the It is insisted that, since a textual examination of the pardon
factual findings of public respondent COMELEC, the given to and accepted by former President Estrada does not
Constitutional body mandated to administer and enforce all actually specify which political right is restored, it could be
laws relative to the conduct of the elections, [relative to the inferred that former President Arroyo did not deliberately intend
absoluteness of the pardon, the effects thereof, and the to restore former President Estrada’s rights of suffrage and to
eligibility of former President Estrada to seek public elective hold public office, orto otherwise remit the penalty of perpetual
office] are binding [and conclusive] on this Honorable Supreme
FINALS CONSTITUTION I ACJUCO 172

absolute disqualification. Even if her intention was the contrary, SR. TAN. Madam President, lines 7 to 9 state:
the same cannot be upheld based on the pardon’s text.
However, the power to grant executive clemency for violations
The pardoning power of the President cannot be limited by of corrupt practices laws may be limited by legislation.
legislative action.
I suggest that this be deletedon the grounds that, first,
The 1987 Constitution, specifically Section 19 of Article VII and violations of corrupt practices may include a very little offense
Section 5 of Article IX-C, provides that the President of the like stealing ₱10; second, which I think is more important, I get
Philippines possesses the power to grant pardons, along with the impression, rightly or wrongly, that subconsciously we are
other acts of executive clemency, to wit: drafting a constitution on the premise that all our future
Presidents will bebad and dishonest and, consequently, their
acts will be lacking in wisdom. Therefore, this Article seems to
Section 19. Except in cases of impeachment, or as otherwise
provided in this Constitution, the President may grant contribute towards the creation of an anti-President
Constitution or a President with vast responsibilities but no
reprieves, commutations, and pardons, and remit fines and
forfeitures, after conviction by final judgment. corresponding power except to declare martial law. Therefore,
I request that these lines be deleted.

He shall also have the power to grant amnesty with the


MR. REGALADO. Madam President,may the Committee react
concurrence of a majority of all the Members of the Congress.
to that?

xxxx
THE PRESIDENT. Yes, please.

Section 5. No pardon, amnesty, parole, or suspension of


sentence for violation of election laws, rules, and regulations MR. REGALADO. This was inserted here on the resolution of
Commissioner Davide because of the fact that similar to the
shall be granted by the President without the favorable
recommendation of the Commission. provisions on the Commission on Elections, the
recommendation of that Commission is required before
executive clemency isgranted because violations of the
It is apparent from the foregoing constitutional provisions that election laws go into the very political life of the country.
the only instances in which the President may not extend
pardon remain to be in: (1) impeachment cases; (2) cases that
With respect to violations of our Corrupt Practices Law, we felt
have not yet resulted in a final conviction; and (3) cases
involving violations of election laws, rules and regulations in that it is also necessary to have that subjected to the same
condition because violation of our Corrupt Practices Law may
which there was no favorable recommendation coming from
the COMELEC. Therefore, it can be argued that any act of be of such magnitude as to affect the very economic systemof
Congress by way of statute cannot operate to delimit the the country. Nevertheless, as a compromise, we provided here
pardoning power of the President. that it will be the Congress that will provide for the classification
as to which convictions will still require prior recommendation;
after all, the Congress could take into account whether or not
In Cristobal v. Labrador27 and Pelobello v. Palatino,28 which the violation of the Corrupt Practices Law is of such magnitude
were decided under the 1935 Constitution,wherein the as to affect the economic life of the country, if it is in the millions
provision granting pardoning power to the President shared or billions of dollars. But I assume the Congress in its collective
similar phraseology with what is found in the present 1987 wisdom will exclude those petty crimes of corruption as not to
Constitution, the Court then unequivocally declared that require any further stricture on the exercise of executive
"subject to the limitations imposed by the Constitution, the clemency because, of course, there is a whale of a difference
pardoning power cannot be restricted or controlled by if we consider a lowly clerk committing malversation of
legislative action." The Court reiterated this pronouncement in government property or funds involving one hundred pesos.
Monsanto v. Factoran, Jr.29 thereby establishing that, under But then, we also anticipate the possibility that the corrupt
the present Constitution, "a pardon, being a presidential practice of a public officer is of such magnitude as to have
prerogative, should not be circumscribed by legislative action." virtually drained a substantial portion of the treasury, and then
Thus, it is unmistakably the long-standing position of this Court he goes through all the judicial processes and later on, a
that the exercise of the pardoning power is discretionary in the President who may have close connections with him or out of
President and may not be interfered with by Congress or the improvident compassion may grant clemency under such
Court, except only when it exceeds the limits provided for by conditions. That is why we left it to Congress to provide and
the Constitution. make a classification based on substantial distinctions
between a minor act of corruption or an act of substantial
This doctrine of non-diminution or non-impairment of the proportions. SR. TAN. So, why do we not just insert the word
President’s power of pardon by acts of Congress, specifically GROSS or GRAVE before the word "violations"?
through legislation, was strongly adhered to by an
overwhelming majority of the framers of the 1987 Constitution MR. REGALADO. We feel that Congress can make a better
when they flatly rejected a proposal to carve out an exception distinction because "GRAVE" or "GROSS" can be
from the pardoning power of the President in the form of misconstrued by putting it purely as a policy.
"offenses involving graft and corruption" that would be
enumerated and defined by Congress through the enactment
of a law. The following is the pertinent portion lifted from the MR. RODRIGO. Madam President.
Record of the Commission (Vol. II):
THE PRESIDENT. Commissioner Rodrigo is recognized.
MR. ROMULO. I ask that Commissioner Tan be recognized to
introduce an amendment on the same section. MR. RODRIGO. May I speak in favor of the proposed
amendment?
THE PRESIDENT. Commissioner Tan is recognized.
THE PRESIDENT. Please proceed.
FINALS CONSTITUTION I ACJUCO 173

MR. RODRIGO. The power to grant executive clemency is Madam President, over and over again, we have been saying
essentially an executive power, and that is precisely why it is and arguing before this Constitutional Commission that we are
called executive clemency. In this sentence, which the emasculating the powers of the presidency, and this provision
amendment seeks to delete, an exception is being made. to me is another clear example of that. So, I speak against this
Congress, which is the legislative arm, is allowed to intrude into provision. Even the 1935 and the 1973 Constitutions do not
this prerogative of the executive. Then it limits the power of provide for this kind of provision.
Congress to subtract from this prerogative of the President to
grant executive clemency by limiting the power of Congress to I am supporting the amendment by deletion of Commissioner
only corrupt practices laws. There are many other crimes more Tan.
serious than these. Under this amendment, Congress cannot
limit the power of executive clemency in cases of drug
addiction and drug pushing which are very, very serious crimes MR. ROMULO. Commissioner Tingson would like to be
that can endanger the State; also, rape with murder, recognized.
kidnapping and treason. Aside from the fact that it is a
derogation of the power of the President to grant executive THE PRESIDENT. Commissioner Tingson is recognized.
clemency, it is also defective in that it singles out just one kind
of crime. There are far more serious crimes which are not
included. MR. TINGSON. Madam President, I am also in favor of the
amendment by deletion because I am in sympathy with the
stand of Commissioner Francisco "Soc" Rodrigo. I do believe
MR. REGALADO. I will just make one observation on that. We and we should remember that above all the elected or
admit that the pardoning power is anexecutive power. But even appointed officers of our Republic, the leader is the President.
in the provisions on the COMELEC, one will notice that I believe that the country will be as the President is, and if we
constitutionally, it is required that there be a favorable systematically emasculate the power of this presidency, the
recommendation by the Commission on Elections for any time may come whenhe will be also handcuffed that he will no
violation of election laws. longer be able to act like he should be acting.

At any rate, Commissioner Davide, as the principal proponent So, Madam President, I am in favor of the deletion of this
of that and as a member of the Committee, has explained in particular line.
the committee meetings we had why he sought the inclusion
of this particular provision. May we call on Commissioner
Davide to state his position. MR. ROMULO. Commissioner Colayco would like to be
recognized.
MR. DAVIDE. Madam President.
THE PRESIDENT. Commissioner Colayco is recognized.
THE PRESIDENT. Commissioner Davide is recognized.
MR. COLAYCO. Thank you very much, Madam President.
MR. DAVIDE. I am constrained to rise to object to the proposal.
We have just approved the Article on Accountability of Public I seldom rise here to object to or to commend or to recommend
Officers. Under it, it is mandated that a public office is a public the approval of proposals, but now I find that the proposal of
trust, and all government officers are under obligation to Commissioner Tan is worthy of approval of this body.
observe the utmost of responsibility, integrity, loyalty and
efficiency, to lead modest lives and to act with patriotism and Why are we singling out this particular offense? There are
justice. other crimes which cast a bigger blot on the moral character of
the public officials.
In all cases, therefore, which would go into the verycore of the
concept that a public office is a public trust, the violation is itself Finally, this body should not be the first one to limit the almost
a violation not only of the economy but the moral fabric of absolute power of our Chief Executive in deciding whether to
public officials. And that is the reason we now want that if there pardon, to reprieve or to commute the sentence rendered by
is any conviction for the violation of the Anti-Graft and Corrupt the court.
Practices Act, which, in effect, is a violation of the public trust
character of the public office, no pardon shall be extended to
the offender, unless some limitations are imposed. I thank you.

Originally, my limitation was, it should be with the concurrence THE PRESIDENT. Are we ready to vote now?
of the convicting court, but the Committee left it entirely to the
legislature to formulate the mechanics at trying, probably, to MR. ROMULO. Commissioner Padilla would like to be
distinguish between grave and less grave or serious cases of recognized, and after him will be Commissioner Natividad.
violation of the Anti-Graft and Corrupt Practices Act. Perhaps
this is now the best time, since we have strengthened the
THE PRESIDENT. Commissioner Padilla is recognized.
Article on Accountability of Public Officers, to accompany it
with a mandate that the President’s right to grant executive
clemency for offenders or violators of laws relating to the MR. PADILLA. Only one sentence, Madam President. The
concept of a public office may be limited by Congress itself. Sandiganbayan has been called the Anti-Graft Court, so if this
is allowed to stay, it would mean that the President’s power
togrant pardon or reprieve will be limited to the cases decided
MR. SARMIENTO. Madam President.
by the Anti-Graft Court, when as already stated, there are
many provisions inthe Revised Penal Code that penalize more
THE PRESIDENT. Commissioner Sarmiento is recognized. serious offenses.

MR. SARMIENTO. May I briefly speak in favor of the Moreover, when there is a judgment of conviction and the case
amendment by deletion. merits the consideration of the exercise of executive clemency,
FINALS CONSTITUTION I ACJUCO 174

usually under Article V of the Revised Penal Code the judge ART. 36. Pardon; its effects.– A pardon shall not work the
will recommend such exercise of clemency. And so, I am in restoration of the right to hold publicoffice, or the right of
favor of the amendment proposed by Commissioner Tan for suffrage, unless such rights be expressly restored by the terms
the deletion of this last sentence in Section 17. of the pardon.

THE PRESIDENT. Are we ready to vote now, Mr. Floor A pardon shall in no case exempt the culprit from the payment
Leader? of the civil indemnity imposed upon him by the sentence.

MR. NATIVIDAD. Just one more. xxxx

THE PRESIDENT. Commissioner Natividad is recognized. ART. 41. Reclusion perpetua and reclusion temporal – Their
accessory penalties.– The penalties of reclusion perpetua and
MR. NATIVIDAD. I am also against this provision which will reclusion temporal shall carry with them that of civil interdiction
again chip more powers from the President. In case of other for life or during the period of the sentence as the case may
criminals convicted in our society, we extend probation to them be, and that of perpetual absolute disqualification which the
offender shall suffer even though pardoned as to the principal
while in this case, they have already been convicted and we
offer mercy. The only way we can offer mercy to them is penalty, unless the same shall have been expressly remitted
in the pardon. (Emphases supplied.)
through this executive clemency extended to them by the
President. If we still close this avenue to them, they would be
prejudiced even worse than the murderers and the more A rigid and inflexible reading of the above provisions of law, as
vicious killers in our society. I do not think they deserve this proposed by Risos-Vidal, is unwarranted, especially so if it will
opprobrium and punishment under the new Constitution. defeat or unduly restrict the power of the President to grant
executive clemency.
I am in favor of the proposed amendment of Commissioner
Tan. It is well-entrenched in this jurisdiction that where the words of
a statute are clear, plain, and free from ambiguity, it must be
MR. ROMULO. We are ready tovote, Madam President. given its literal meaning and applied without attempted
interpretation. Verba legis non est recedendum. From the
words of a statute there should be no departure. 31 It is this
THE PRESIDENT. Is this accepted by the Committee? Court’s firm view that the phrase in the presidential pardon at
issue which declares that former President Estrada "is hereby
MR. REGALADO. The Committee, Madam President, prefers restored to his civil and political rights" substantially complies
to submit this to the floor and also because of the objection of with the requirement of express restoration.
the main proponent, Commissioner Davide. So we feel that the
Commissioners should vote on this question. The Dissent of Justice Marvic M.V.F. Leonen agreed with
Risos Vidal that there was no express remission and/or
VOTING restoration of the rights of suffrage and/or to hold public office
in the pardon granted to former President Estrada, as required
by Articles 36 and 41 of the Revised Penal Code.
THE PRESIDENT. As many as are in favor of the proposed
amendment of Commissioner Tan to delete the last sentence
of Section 17 appearing on lines 7, 8 and 9, please raise their Justice Leonen posits in his Dissent that the aforementioned
hand. (Several Members raised their hand.) codal provisions must be followed by the President, as they do
not abridge or diminish the President’s power to extend
clemency. He opines that they do not reduce the coverage of
As many as are against, please raise their hand. (Few the President’s pardoning power. Particularly, he states:
Members raised their hand.)
Articles 36 and 41 refer only to requirements of convention or
The results show 34 votes in favor and 4 votes against; the form. They only provide a procedural prescription. They are not
amendment is approved.30 (Emphases supplied.) concerned with areas where or the instances when the
President may grant pardon; they are only concerned with how
The proper interpretation of Articles he or she is to exercise such power so that no other
governmental instrumentality needs to intervene to give it full
effect.
36 and 41 of the Revised Penal Code.

All that Articles 36 and 41 do is prescribe that, if the President


The foregoing pronouncements solidify the thesis that Articles wishes to include in the pardon the restoration of the rights of
36 and 41 of the Revised Penal Code cannot, in any way, serve suffrage and to hold public office, or the remission of the
to abridge or diminish the exclusive power and prerogative of accessory penalty of perpetual absolute disqualification,he or
the President to pardon persons convicted of violating penal she should do so expressly. Articles 36 and 41 only ask that
statutes. the President state his or her intentions clearly, directly, firmly,
precisely, and unmistakably. To belabor the point, the
The Court cannot subscribe to Risos-Vidal’s interpretation that President retains the power to make such restoration or
the said Articles contain specific textual commands which must remission, subject to a prescription on the manner by which he
be strictly followed in order to free the beneficiary of or she is to state it.32
presidential grace from the disqualifications specifically
prescribed by them. With due respect, I disagree with the overbroad statement that
Congress may dictate as to how the President may exercise
Again, Articles 36 and 41 of the Revised Penal Code provides: his/her power of executive clemency. The form or manner by
which the President, or Congress for that matter, should
exercise their respective Constitutional powers or prerogatives
FINALS CONSTITUTION I ACJUCO 175

cannot be interfered with unless it is so provided in the renunciation of any and all foreign citizenship before
Constitution. This is the essence of the principle of separation any public officer authorized to administer an oath;
of powers deeply ingrained in our system of government which
"ordains that each of the three great branches of government
(3) Those appointed to any public office shall
has exclusive cognizance of and is supreme in matters falling subscribe and swear an oath of allegiance to the
within its own constitutionally allocated sphere."33 Moreso, this Republic of the Philippines and its duly constituted
fundamental principle must be observed if noncompliance with authorities prior to their assumption of office:
the form imposed by one branch on a co-equal and coordinate Provided, That they renounce their oath of allegiance
branch will result into the diminution of an exclusive to the country where they took that oath; (4) Those
Constitutional prerogative. intending to practice their profession in the
Philippines shall apply with the proper authority for a
For this reason, Articles 36 and 41 of the Revised Penal Code license or permit to engage in such practice; and
should be construed in a way that will give full effect to the
executive clemency granted by the President, instead of (5) That right to vote or be elected or appointed to
indulging in an overly strict interpretation that may serve to any public office in the Philippines cannot be
impair or diminish the import of the pardon which emanated exercised by, or extended to, those who:
from the Office of the President and duly signed by the Chief
Executive himself/herself. The said codal provisions must be
construed to harmonize the power of Congress to define (a) are candidates for or are occupying any
crimes and prescribe the penalties for such crimes and the public office in the country of which theyare
power of the President to grant executive clemency. All that the naturalized citizens; and/or
said provisions impart is that the pardon of the principal penalty
does notcarry with it the remission of the accessory penalties (b) are in active service as commissioned
unless the President expressly includes said accessory or non commissioned officers in the armed
penalties in the pardon. It still recognizes the Presidential forces of the country which they are
prerogative to grant executive clemency and, specifically, to naturalized citizens. (Emphases supplied.)
decide to pardon the principal penalty while excluding its
accessory penalties or to pardon both. Thus, Articles 36 and
41 only clarify the effect of the pardon so decided upon by the No less than the International Covenant on Civil and Political
President on the penalties imposedin accordance with law. Rights, to which the Philippines is a signatory, acknowledges
the existence of said right. Article 25(b) of the Convention
states: Article 25
A close scrutiny of the text of the pardon extended to former
President Estrada shows that both the principal penalty of
reclusion perpetua and its accessory penalties are included in Every citizen shall have the right and the opportunity, without
the pardon. The first sentence refers to the executive clemency any of the distinctions mentioned in Article 2 and without
extended to former President Estrada who was convicted by unreasonable restrictions:
the Sandiganbayan of plunder and imposed a penalty of
reclusion perpetua. The latter is the principal penalty pardoned xxxx
which relieved him of imprisonment. The sentence that
followed, which states that "(h)e is hereby restored to his civil
and political rights," expressly remitted the accessory penalties (b) To vote and to be electedat genuine periodic elections
that attached to the principal penalty of reclusion perpetua. which shall be by universal and equal suffrage and shall be
Hence, even if we apply Articles 36 and 41 of the Revised held by secret ballot, guaranteeing the free expression of the
Penal Code, it is indubitable from the textof the pardon that the will of the electors[.] (Emphasis supplied.)
accessory penalties of civil interdiction and perpetual absolute
disqualification were expressly remitted together with the Recently, in Sobejana-Condon v. Commission on
principal penalty of reclusion perpetua. Elections,35 the Court unequivocally referred to the right to
seek public elective office as a political right, to wit:
In this jurisdiction, the right toseek public elective office is
recognized by law as falling under the whole gamut of civil and Stated differently, it is an additional qualification for elective
political rights. office specific only to Filipino citizens who re-acquire their
citizenship under Section 3 of R.A. No. 9225. It is the operative
Section 5 of Republic Act No. 9225,34 otherwise known as the act that restores their right to run for public office. The
"Citizenship Retention and Reacquisition Act of 2003," reads petitioner’s failure to comply there with in accordance with the
as follows: exact tenor of the law, rendered ineffectual the Declaration of
Renunciation of Australian Citizenship she executed on
September 18, 2006. As such, she is yet to regain her political
Section 5. Civil and Political Rights and Liabilities.– Those who right to seek elective office. Unless she executes a sworn
retain or reacquire Philippine citizenship under this Act shall renunciation of her Australian citizenship, she is ineligible to
enjoy full civil and political rights and be subject to all attendant run for and hold any elective office in the Philippines.
liabilities and responsibilities under existing laws of the (Emphasis supplied.)
Philippines and the following conditions: (1) Those intending to
exercise their right of suffrage must meet the requirements
under Section 1, Article V of the Constitution, Republic Act No. Thus, from both law and jurisprudence, the right to seek public
9189, otherwise known as "The Overseas Absentee Voting Act elective office is unequivocally considered as a political right.
of 2003" and other existing laws; Hence, the Court reiterates its earlier statement that the pardon
granted to former President Estrada admits no other
interpretation other than to mean that, upon acceptance of the
(2) Those seeking elective public office in the pardon granted tohim, he regained his FULL civil and political
Philippines shall meet the qualifications for holding rights – including the right to seek elective office.
such public office as required by the Constitution and
existing laws and, at the time of the filing of the
certificate of candidacy, make a personal and sworn On the other hand, the theory of Risos-Vidal goes beyond the
plain meaning of said penal provisions; and prescribes a formal
FINALS CONSTITUTION I ACJUCO 176

requirement that is not only unnecessary but, if insisted upon, unprecedented. In Jalosjos, Jr. v. Commission on
could be in derogation of the constitutional prohibition relative Elections,37 the Court acknowledged the aforementioned
to the principle that the exercise of presidential pardon cannot provision as one of the legal remedies that may be availed of
be affected by legislative action. to disqualify a candidate in a local election filed any day after
the last day for filing of certificates of candidacy, but not later
Risos-Vidal relied heavily on the separate concurring opinions than the date of proclamation.38 The pertinent ruling in the
in Monsanto v. Factoran, Jr.36 to justify her argument that an Jalosjos case is quoted as follows:
absolute pardon must expressly state that the right to hold
public office has been restored, and that the penalty of What is indisputably clear is that false material representation
perpetual absolute disqualification has been remitted. of Jalosjos is a ground for a petition under Section 78.
However, since the false material representation arises from a
This is incorrect. crime penalized by prision mayor, a petition under Section 12
ofthe Omnibus Election Code or Section 40 of the Local
Government Code can also be properly filed. The petitioner
Her reliance on said opinions is utterly misplaced. Although the has a choice whether to anchor his petition on Section 12 or
learned views of Justices Teodoro R. Padilla and Florentino P. Section 78 of the Omnibus Election Code, or on Section 40 of
Feliciano are to be respected, they do not form partof the the Local Government Code. The law expressly provides
controlling doctrine nor to be considered part of the law of the multiple remedies and the choice of which remedy to adopt
land. On the contrary, a careful reading of the majority opinion belongs to petitioner.39 (Emphasis supplied.)
in Monsanto, penned by no less than Chief Justice Marcelo B.
Fernan, reveals no statement that denotes adherence to a
The third preambular clause of the pardon did not operate to
stringent and overly nuanced application of Articles 36 and 41
of the Revised Penal Code that will in effect require the make the pardon conditional.
President to use a statutorily prescribed language in extending
executive clemency, even if the intent of the President can Contrary to Risos-Vidal’s declaration, the third preambular
otherwise be deduced from the text or words used in the clause of the pardon, i.e., "[w]hereas, Joseph Ejercito Estrada
pardon. Furthermore, as explained above, the pardon here is has publicly committed to no longer seek any elective position
consistent with, and not contrary to, the provisions of Articles or office," neither makes the pardon conditional, nor militate
36 and 41. against the conclusion that former President Estrada’s rights to
suffrage and to seek public elective office have been restored.
The disqualification of former President Estrada under Section
40 of the LGC in relation to Section 12 of the OEC was This is especially true as the pardon itself does not explicitly
removed by his acceptance of the absolute pardon granted to impose a condition or limitation, considering the unqualified
him. use of the term "civil and political rights"as being restored.
Jurisprudence educates that a preamble is not an essential
Section 40 of the LGC identifies who are disqualified from part of an act as it is an introductory or preparatory clause that
running for any elective local position. Risos-Vidal argues that explains the reasons for the enactment, usually introduced by
former President Estrada is disqualified under item (a), to wit: the word "whereas."40 Whereas clauses do not form part of a
statute because, strictly speaking, they are not part of the
operative language of the statute.41 In this case, the whereas
(a) Those sentenced by final judgment for an offense involving clause at issue is not an integral part of the decree of the
moral turpitude or for an offense punishable by one (1) year or pardon, and therefore, does not by itself alone operate to make
more of imprisonment, within two (2) years after serving the pardon conditional or to make its effectivity contingent upon
sentence[.] (Emphasis supplied.) the fulfilment of the aforementioned commitment nor to limit the
scope of the pardon.
Likewise, Section 12 of the OEC provides for similar
prohibitions, but it provides for an exception, to wit: On this matter, the Court quotes with approval a relevant
excerpt of COMELEC Commissioner Maria Gracia Padaca’s
Section 12. Disqualifications. – x x x unless he has been given separate concurring opinion in the assailed April 1, 2013
plenary pardon or granted amnesty. (Emphasis supplied.) Resolution of the COMELEC in SPA No. 13-211 (DC), which
captured the essence of the legal effect of preambular
paragraphs/whereas clauses, viz:
As earlier stated, Risos-Vidal maintains that former President
Estrada’s conviction for plunder disqualifies him from running
for the elective local position of Mayor of the City of Manila The present dispute does not raise anything which the 20
under Section 40(a) of the LGC. However, the subsequent January 2010 Resolution did not conclude upon. Here,
absolute pardon granted to former President Estrada Petitioner Risos-Vidal raised the same argument with respect
effectively restored his right to seek public elective office. This to the 3rd "whereas clause" or preambular paragraph of the
is made possible by reading Section 40(a) of the LGC in decree of pardon. It states that "Joseph Ejercito Estrada has
relation to Section 12 of the OEC. publicly committed to no longer seek any elective position or
office." On this contention, the undersigned reiterates the ruling
of the Commission that the 3rd preambular paragraph does not
While it may be apparent that the proscription in Section 40(a) have any legal or binding effect on the absolute nature of the
of the LGC is worded in absolute terms, Section 12 of the OEC pardon extended by former President Arroyo to herein
provides a legal escape from the prohibition – a plenary pardon Respondent. This ruling is consistent with the traditional and
or amnesty. In other words, the latter provision allows any customary usage of preambular paragraphs. In the case of
person who has been granted plenary pardon or amnesty after Echegaray v. Secretary of Justice, the Supreme Court ruled on
conviction by final judgment of an offense involving moral the legal effect of preambular paragraphs or whereas clauses
turpitude, inter alia, to run for and hold any public office, on statutes. The Court stated, viz.:
whether local or national position.
Besides, a preamble is really not an integral part of a law. It is
Take notice that the applicability of Section 12 of the OEC to merely an introduction to show its intent or purposes. It cannot
candidates running for local elective positions is not be the origin of rights and obligations. Where the meaning of a
FINALS CONSTITUTION I ACJUCO 177

statute is clear and unambiguous, the preamble can neither meaning, it was not only unconditional, it was unrestricted in
expand nor restrict its operation much less prevail over its text. scope, complete and plenary in character, as the term "political
rights"adverted to has a settled meaning in law and
jurisprudence.
If former President Arroyo intended for the pardon to be
conditional on Respondent’s promise never to seek a public
office again, the former ought to have explicitly stated the same With due respect, I disagree too with Justice Leonen that the
in the text of the pardon itself. Since former President Arroyo omission of the qualifying word "full" can be construed as
did not make this an integral part of the decree of pardon, the excluding the restoration of the rights of suffrage and to hold
Commission is constrained to rule that the 3rd preambular public office. There appears to be no distinction as to the
clause cannot be interpreted as a condition to the pardon coverage of the term "full political rights" and the term "political
extended to former President Estrada.42 (Emphasis supplied.) rights" used alone without any qualification. How to ascribe to
the latter term the meaning that it is "partial" and not "full" defies
Absent any contrary evidence, former President Arroyo’s one’s understanding. More so, it will be extremely difficult to
silence on former President Estrada’s decision torun for identify which of the political rights are restored by the pardon,
President in the May 2010 elections against, among others, the when the text of the latter is silent on this matter. Exceptions to
candidate of the political party of former President Arroyo, after the grant of pardon cannot be presumed from the absence of
the qualifying word "full" when the pardon restored the "political
the latter’s receipt and acceptance of the pardon speaks
volume of her intention to restore him to his rights to suffrage rights" of former President Estrada without any exclusion or
reservation.
and to hold public office.

Therefore, there can be no other conclusion but to say that the


Where the scope and import of the executive clemency
extended by the President is in issue, the Court must turn to pardon granted to former President Estrada was absolute in
the absence of a clear, unequivocal and concrete factual basis
the only evidence available to it, and that is the pardon itself.
From a detailed review ofthe four corners of said document, upon which to anchor or support the Presidential intent to grant
nothing therein gives an iota of intimation that the third a limited pardon.
Whereas Clause is actually a limitation, proviso, stipulation or
condition on the grant of the pardon, such that the breach of To reiterate, insofar as its coverageis concerned, the text of the
the mentioned commitment not to seek public office will result pardon can withstand close scrutiny even under the provisions
ina revocation or cancellation of said pardon. To the Court, of Articles 36 and 41 of the Revised Penal Code.
what it is simply is a statement of fact or the prevailing situation
at the time the executive clemency was granted. It was not The COMELEC did not commit grave abuse of discretion
used as a condition to the efficacy orto delimit the scope of the amounting to lack or excess of jurisdiction in issuing the
pardon. assailed Resolutions.

Even if the Court were to subscribe to the view that the third In light of the foregoing, contrary to the assertions of Risos-
Whereas Clausewas one of the reasons to grant the pardon, Vidal, the COMELEC did not commit grave abuse of discretion
the pardon itself does not provide for the attendant amounting to lack or excess of jurisdiction in issuing the
consequence of the breach thereof. This Court will be hard put assailed Resolutions.
to discern the resultant effect of an eventual infringement. Just
like it will be hard put to determine which civil or political rights
were restored if the Court were to take the road suggested by The Court has consistently held that a petition for
Risos-Vidal that the statement "[h]e is hereby restored to his certiorariagainst actions of the COMELEC is confined only to
civil and political rights" excludes the restoration of former instances of grave abuse of discretion amounting to patentand
President Estrada’s rights to suffrage and to hold public office. substantial denial of due process, because the COMELEC is
The aforequoted text ofthe executive clemency granted does presumed to be most competent in matters falling within its
not provide the Court with any guide asto how and where to domain.43
draw the line between the included and excluded political
rights. As settled in jurisprudence, grave abuse of discretion is the
arbitrary exercise of power due to passion, prejudice or
Justice Leonen emphasizes the point that the ultimate issue for personal hostility; or the whimsical, arbitrary, or capricious
resolution is not whether the pardon is contingent on the exercise of power that amounts to an evasion or refusal to
condition that former President Estrada will not seek janother perform a positive duty enjoined by law or to act at all in
elective public office, but it actually concerns the coverage of contemplation of law. For an act to be condemned as having
the pardon – whether the pardon granted to former President been done with grave abuse of discretion, such an abuse must
Estrada was so expansive as to have restored all his political be patent and gross.44
rights, inclusive of the rights of suffrage and to hold public
office. Justice Leonen is of the view that the pardon in question The arguments forwarded by Risos-Vidal fail to adequately
is not absolute nor plenary in scope despite the statement that demonstrate any factual or legal bases to prove that the
former President Estrada is "hereby restored to his civil and assailed COMELEC Resolutions were issued in a "whimsical,
political rights," that is, the foregoing statement restored to arbitrary or capricious exercise of power that amounts to an
former President Estrada all his civil and political rights except evasion orrefusal to perform a positive duty enjoined by law"
the rights denied to him by the unremitted penalty of perpetual or were so "patent and gross" as to constitute grave abuse of
absolute disqualification made up of, among others, the rights discretion.
of suffrage and to hold public office. He adds that had the
President chosen to be so expansive as to include the rights of
suffrage and to hold public office, she should have been more On the foregoing premises and conclusions, this Court finds it
clear on her intentions. unnecessary to separately discuss Lim's petition-in-
intervention, which substantially presented the same
arguments as Risos-Vidal's petition.
However, the statement "[h]e is hereby restored to his civil and
political rights," to the mind of the Court, iscrystal clear – the
pardon granted to former President Estrada was absolute, WHEREFORE, the petition for certiorari and petition-
inintervention are DISMISSED. The Resolution dated April 1,
FINALS CONSTITUTION I ACJUCO 178

2013 of the Commission on Elections, Second Division, and


the Resolution dated April 23, 2013 of the Commission on
Elections, En bane, both in SPA No. 13-211 (DC), are
AFFIRMED.

SO ORDERED.
FINALS CONSTITUTION I ACJUCO 179

G.R. No. L-1278 January 21, 1949 WHEREAS, charges have been presented in the
courts against many members of these resistance
LORETO BARRIOQUINTO and NORBERTO forces, for such acts;
JIMENEZ, petitioners,
vs. WHEREAS, the fact that such acts were committed
ENRIQUE A. FERNANDEZ, ANTONIO BELMONTE and in furtherance of the resistance to the enemy is not a
FELICISIMO OCAMPO, as Commissioners of the valid defense under the laws of the Philippines;
Fourteenth Guerrilla Amnesty Commission, respondents.
WHEREAS, the persons so accused should not be
Roseller T. Lim for petitioners. regarded as criminals but rather as patriots and
Antonio Belmonte for respondents. heroes who have rendered invaluable service to the
nation; and
FERIA, J.:
WHEREAS, it is desirable that without the least
This is a special action of mandamus instituted by the possible delay, these persons be freed form the
indignity and the jeopardy to which they are now
petitioners against the respondents who composed the 14th
Guerrilla Amnesty Commission, to compel the latter to act and being subjected;
decide whether or not the petitioners are entitled to the benefits
of amnesty. NOW, THEREFORE, I Manuel Roxas, President of
the Philippines in accordance with the provisions of
Petitioners Norberto Jimenez and Loreto Barrioquinto were Article VII, section 10, paragraph 6 of the
Constitution, do hereby declare and proclaim an
charged with the crime of murder. As the latter had not yet
been arrested the case proceeded against the former, and amnesty inn favor of al persons who committed any
after trial Court of First Instance of Zamboanga sentenced act penalized under the Revised Penal Code in
Jimenez to life imprisonment. Before the period for perfecting furtherance of the resistance to the enemy or against
an appeal had expired, the defendant Jimenez became aware persons aiding in the war effort of the enemy, and
of the Proclamation No. 8, dated September 7, 1946, which committed during the period from December 8, 1941
grants amnesty in favor of all persons who may be charged to the date when each particular area of the
with an act penalized under the Revised Penal Code in Philippines was actually liberated from the enemy
furtherance of the resistance to the enemy or against persons control and occupation. This amnesty shall not apply
to crimes against chastity or to acts committed from
aiding in the war efforts of the enemy, and committed during
the period from December 8, 1941, to the date when particular purely personal motives.
area of the Philippines where the offense was actually
committed was liberated from enemy control and occupation, It is further proclaimed and declared that in order to
and said Jimenez decided to submit his case to the Guerrilla determine who among those against whom charges
Amnesty Commission presided by the respondents herein, and have been filed before the courts of the Philippines
the other petitioner Loreto Barrioquinto, who had then been or against whom charges may be filed in the future,
already apprehended, did the same. come within the terms of this amnesty, Guerrilla
Amnesty Commissions, simultaneously to be
After a preliminary hearing had started, the Amnesty established , shall examine the facts and
Commission, prescribed by the respondents, issued on circumstance surrounding each case and, if
January 9, 1947, an order returning the cases of the petitioners necessary, conduct summary hearings of witnesses
to the Court of First Instance of Zamboanga, without deciding both for the complainant and the accused. These
Commissions shall decided each case and, upon
whether or not they are entitled to the benefits of he said
Amnesty Proclamation, on the ground that inasmuch as neither finding that it falls within the terms of this
proclamation, the Commissions shall so declare and
Barrioquinto nor Jimenez have admitted having committed the
offense, because Barrioquinto alleged that it was Hipolito this amnesty shall immediately be effective as to the
accused, who shall forthwith be released or
Tolentino who shot and killed the victim, they cannot invoke the
benefits of amnesty. discharged.

The Amnesty Proclamation of September 7, 1946, issued by The theory of the respondents, supported by the dissenting
opinion, is predicated on a wrong conception of the nature or
the President with the concurrence of Congress of the
Philippines, reads in part as follows: character of an amnesty. Amnesty must be distinguished from
pardon.

WHEREAS, since the inception of the war until the


Pardon is granted by the Chief Executive and as such it is a
liberation of the different areas comprising the
territory of the Philippines, volunteer armed forces of private act which must be pleaded and proved by the person
Filipinos and for of other nationalities operated as pardoned, because the courts take no notice thereof; while
guerrillas and other patriotic individuals and groups amnesty by Proclamation of the Chief Executive with the
pursued activities in opposition to the forces and concurrence of Congress, and it is a public act of which the
agents of the Japanese Empire in the invasion and courts should take judicial notice. Pardon is granted to one
occupation of the Philippines; after conviction; while amnesty is granted to classes of persons
or communities who may be guilty of political offenses,
generally before or after the institution of the criminal
WHEREAS, members of such forces, in their prosecution and sometimes after conviction. Pardon looks
determined efforts to resist the enemy, and to bring forward and relieves the offender from the consequences of an
about his ultimate defeat, committed acts penalized offense of which he has been convicted, that is, it abolished or
under the Revised Penal Code; forgives the punishment, and for that reason it does ""nor work
the restoration of the rights to hold public office, or the right of
suffrage, unless such rights be expressly restored by the terms
of the pardon," and it "in no case exempts the culprit from the
FINALS CONSTITUTION I ACJUCO 180

payment of the civil indemnity imposed upon him by the of a defendant at the time of arraignment or the hearing of the
sentence" article 36, Revised Penal Code). while amnesty case about said motive, can not generally be considered and
looks backward and abolishes and puts into oblivion the relied on, specially if there is evidence to the contrary, as the
offense itself, it so overlooks and obliterates the offense with true expression of the reason o motive he had at the time of
which he is charged that the person released by amnesty committing the offense. Because such statements or testimony
stands before the law precisely as though he had committed may be an afterthought or colored by the interest he may have
no offense. (section 10[6], Article VII, Philippine Constitution; to suit his defense or the purpose for which he intends to
State vs. Blalock, 62 N.C., 242, 247; In re Briggs, 135 N.C., achieve with such declaration. Hence it does not stand to
118; 47 S.E. 402., 403; Ex parte Law, 35 GA., 285, 296; State reason and logic to say, as the dissenting opinion avers, that
ex rel Anheuser—Busch Brewing Ass'n. vs. Eby, 170 Mo., 497; unless the defendant admits at the investigation or hearing
71 S.W 52, 61; Burdick vs United States, N.Y., 35 S. Ct., 267; having committed the offense with which he is charged, and
271; 236 U.S., 79; 59 Law. ed., 476.) states that he did it in furtherance of the resistance to the
enemy, and not for purely personal motive, it is impossible for
In view of the foregoing, we are of the opinion and so hold that, the court of Commission to verify the motive for the
commission of the offense, because only the accused could
in order to entitle a person to the benefits of the Amnesty
Proclamation of September 7, 1946, it is not necessary that he explain of the offense, because only the accused could explain
should, as a condition precedent or sine qua non, admit having his belief and intention or the motive of committing the offense.
committed the criminal act or offense with which he is charged
and allege the amnesty as a defense; it is sufficient that the There is no necessity for an accused to admit his responsibility
evidence either of the complainant or the accused, shows that for the commission of a criminal act before a court of Amnesty
the offense committed comes within the terms of said Amnesty Commission may investigate and extend or not to him the
Proclamation. Hence, it is not correct to say that "invocation of benefits of amnesty. The fact that he pleads not guilty or that
the benefits of amnesty is in the nature of a plea of confession he has not committed the act with which he is charged, does
and avoidance." Although the accused does not confess the not necessarily prove that he is not guilty thereof.
imputation against him, he may be declared by the courts or Notwithstanding his denial, the evidence for the prosecution or
the Amnesty Commissions entitled to the benefits. For, complainant may show the contrary, as it is generally the case
whether or not he admits or confesses having committed the in criminal proceedings, and what should in such a case be
offense with which he is charged, the Commissions should, if determined is whether or not the offense committed is of
necessary or requested by the interested party, conduct political character. The plea of not having committed the
summary hearing of the witnesses both for the complainants offense made by an accused simply means that he can not be
and the accused, on whether he has committed the offense in convicted of the offense charged because he is not guilty
furtherance of the resistance to the enemy, or against persons thereof, and, even if the evidence would show that he is,
aiding in the war efforts of the enemy, and decide whether he because he has committed it in furtherance of the resistance
is entitled to the benefits of amnesty and to be "regarded as a to the enemy or against persons a ding in the war efforts of the
patriot or hero who have rendered invaluable services to the enemy, and not for purely political motives.
nation,," or not, in accordance with the terms of the Amnesty
Proclamation. since the Amnesty Proclamation is a public act,
According to Administrative Order No. 11 of October 2, 1946,
the courts as well as the Amnesty Commissions created creating the Amnesty Commissions, issued by the President of
thereby should take notice of the terms of said Proclamation
the Philippines, cases pending in the Courts of First Instance
and apply the benefits granted therein to cases coming within of the province in which the accused claims the benefits of
their province or jurisdiction, whether pleaded or claimed by Amnesty Proclamation, and cases already decided by said
the person charged with such offenses or not, if the evidence courts but not yet elevated on appeal to the appellate courts,
presented show that the accused is entitled to said benefits. shall be passed upon and decided by the respective Amnesty
Commission, and cases pending appeal shall be passed upon
The right to the benefits of amnesty, once established by the by the Seventh Amnesty Commission. Under the theory of the
evidence presented either by the complainant or prosecution, respondents and the writer oft he dissenting opinion, the
or by the defense, can not be waived, because it is of public Commissions should refuse to comply with the directive of said
interest that a person who is regarded by the Amnesty Administrative Order, because is almost all cases pending in
Proclamation which has the force of a law, not only as the Court of First Instance, and all those pending appeal form
innocent, for he stands in the eyes of the law as if he had never the sentence of said courts, the defendants must not have
committed any punishable offense because of the amnesty, pleaded guilty or admitted having committed the offense
but as a patriot or hero, can not be punishment as a criminal. charged for otherwise, they would not or could not have
Just as the courts of justice can not convict a person who, appealed from the judgment of the Courts of First Instance. To
according to the evidence, has committed an act not hold that a Amnesty Commission should not proceed to the
punishable by law, although he confesses being guilty thereof, investigation and act and decide whether the offense with
so also and a fortiori they can not convict a person considered which an accused was charged comes within the Amnesty
by law not a criminal, but as a patriot and hero, for having Proclamation if he does not admit or confess having committed
rendered invaluable services to the nation inn committing such it would be to defeat the purpose for which the Amnesty
an act. Proclamation was issued and the Amnesty Commission were
established. If the courts have to proceed to the trail or hearing
While it is true that the evidence must show that the offense of a case and decide whether the offense committed by the
charged was against chastity and was committed in defendant comes within the terms of the Amnesty
Proclamation although the defendant has plead not guilty,
furtherance of the resistance against the enemy, for otherwise,
it is to be naturally presumed that is has been committed for there is no reason why the Amnesty Commissions can not do
so. Where a defendant to admit or confess having committed
purely personal motive, it is nonetheless true that though the
motive as a mental impulse is state of mind or subjective, it the offense or being responsible therefor before he can invoke
need not be testified to be the defendant himself at his the benefit of amnesty, as there is no law which makes such
arraignment or hearing of the case. Generally the motive for admission or confession not admissible as evidence against
the commission of an offense is established by the testimony him in the courts of justices in case the Amnesty Commission
of witnesses on the acts or statements of the accused before finds that the offense does not come within the terms of the
or immediately after the commission of the offense, deeds or Amnesty Proclamation, nobody or few would take the risk of
submitting their case to said Commission.
words hat may express it or from which his motive or reason
for committing it may be inferred. The statement of testimony
FINALS CONSTITUTION I ACJUCO 181

Besides, in the present case, the allegation of Loreto


Barrioquinto that the offended party or victim was shot and
killed by Agapito Hipolito , does not necessarily bar the
respondents from finding, after the summary hearing of the
witnesses for the complaints and the accused, directed in the
said Amnesty Proclamation and Administrative Order No. 11,
that the petitioners are responsible for the killing of the victim,
either as principals by cooperation, inducement or
conspiration, or as accessories before as well as after the fact,
but that they are entitled to the benefits of amnesty, because
they were members of the same group of guerrilleros who
killed the victim in furtherance of the resistance to the enemy
or against persons aiding in the war efforts of the enemy.

Wherefore, the respondents are hereby ordered to


immediately proceed to hear and decide the application for
amnesty of petitioners Barrioquinto and Jimenez, unless
amnesty of petitioners Barrioquinto and Jimenez, unless the
courts have in the meantime already decided, expressly and
finally, the question whether or not they are entitled to the
benefits of the Amnesty Proclamation No. 8 of September 7,
1946. So ordered.
FINALS CONSTITUTION I ACJUCO 182

G.R. No. L-18184 January 31, 1963 defendant Jaime Garcia had any participation in the
complex crime charged. Neither does the evidence
GAUDENCIO VERA, RESTITUTO FIGUERAS, LORENZO reveal that he admitted or disclaimed any role
therein. Consequently, there would be no room,
AMBAS, JUSTO FLORIDO, PAULINO BAYRAN AND
JAYME GARCIA, petitioners, either for his conviction, or for the application of the
vs. provisions of the aforementioned amnesty
PEOPLE OF THE PHILIPPINES and COURT OF proclamation.
APPEALS, respondents.
FOR ALL THE FOREGOING CONSIDERATIONS,
De Mesa & De Mesa for petitioners. this Eighth Guerrilla Amnesty Commission denies the
Office of the Solicitor General for respondents. defendants motion for reconsideration and maintains
its order contained in its decisions, to return the case
to the Court of First Instance of Quezon for the latter
BARRERA, J.: to act on it accordingly, not only because of lack of
jurisdiction, but also because, even if it has
In the Court of First Instance of Quezon, petitioners Gaudencio jurisdiction, the defendants are not entitled to the
Vera, Restituto Figueras, Lorenzo Ambas, Justo Florido, benefits of the amnesty proclamation.
Paulino Bayran, and 92 others, as John Does, were charged
with the complex crime of kidnapping with murder of Amadeo From this order of the Commission, petitioners appealed to the
Lozanes, alias Azarcon. Upon petitioners' motion, invoking the Court of Appeals. The latter, on July 27, 1959, certified the
benefits of Amnesty Proclamation of the President, series of appeal to us, in view of the legal issue involved, namely,
1946, the case was referred to the Eighth Guerrilla Amnesty whether or not persons invoking the benefit of amnesty should
Commission, which actually tried it. first admit having committed the crime of which they were
accused. On August 13, 1959 we ordered the docketing of the
During the hearing, none of the petitioner-defendants admitted appeal in this court (G.R. No. L-15803). However, on
having committed the crime charged. In fact, Gaudencio Vera, petitioners' motion to return the record of the case to the Court
the only defendant who took the witness stand, instead of of Appeals (on the ground that the appeal was originally
admitting the killing of the deceased Lozanes categorically coursed to said Court, due to "factual issues to the effect that
denied it. Hence, the Commission, in its decision of January the death of Amado Lozanes did not spring from personal
12, 1956, held that it could not take cognizance of the case, on motive or on account of rivalry between guerrilla units, but
the ground that the benefits of the Amnesty Proclamation, owing to the fact that said decedent had aided in the war efforts
could be invoked only by defendants in a criminal case who, of the enemy, by having been a member of the Jap-sponsored
admitting the commission of the crime, plead that said Philippine Constabulary organization, and by having been one
commission was in pursuance of the resistance movement and of those who arrested and subsequently massacred, innocent
perpetrated against persons who aided the enemy during the civilians and guerrillas in Catanauan, Quezon"), we ordered
Japanese occupation. Consequently, the Commission ordered the return of said record to said Court.
that the case be remanded to the court of origin for trial. A
motion for reconsideration filed by petitioners was denied by On November 16, 1960, the Court of Appeals rendered a
the Commission in its order dated January 11, 1957, which decision, affirming the Order of the Commission, stating in part,
partly reads: as follows:

The Commission is convinced that the motive for the Appellants stressed in their aforementioned motion
kidnapping and killing of Lt. Amadeo Lozanes of the for reconsideration that they had impliedly admitted
Hunters was the keen rivalry between the Vera's their participation in the killing of Amadeo Lozanes.
Guerrilla Party and the Hunter's ROTC Guerrilla But mere implied admission is not sufficient, for
organizations. It is noteworthy that the Hunters were Administrative Order No. 144 of the Department of
driven away by General Vera from Pitogo in Justice, dated October 11, 1950, amending
December, 1944, and that after said kidnapping and Administrative Order No. 179 thereof, and issued on
killing on February 13 and 14, 1945, Mayor Ramon November 17, 1949, explicitly directs that "where the
Isaac of Unisan, was in turn kidnapped by the offense charged against any person is not one
Hunters. Leopoldo Miciano, secretary of Col. de Luna against chastity but is covered by the Revised Penal
of the Vera's Guerrilla Party, testified that General Code, and the offense took place between December
Vera told him of his (Vera's) suspicion that Mayor 8, 1941 and the date of the liberation of the province
Isaac was kidnapped by way of reprisal as he, Vera, or city where the offense is alleged to have been
had ordered the liquidation of Lt. Lozanes committed, "in order that the Amnesty Commission
(dinispatcha). may take cognizance of the case, the accused or
respondent must allege or claim verbally or in writing
In any event, since it is an established fact that when that he committed the acts charged against him in
Lozanes was kidnapped, tortured, and later killed, he furtherance of the resistance movement or against
was actually a lieutenant of the Hunter's ROTC persons who aided in the war efforts of the enemy',
Guerrilla Organizations then engaged in the for amnesty presupposes the commission of a
resistance movement, it may not be said with any crime".
amount of truth that the aforesaid killing was to
further the resistance movement at the time, as the xxx xxx xxx
defense intimate. Rather, the killing of Lt. Lozanes of
the Hunters ROTC Guerrilla would tend to weaken
commensurately the resistance movement against Therefore, and since appellants did not claim verbally
the Japanese invaders. or in writing that they committed the offense with
which they were charged in furtherance of the
resistance movement or against persons who aided
The Commission noted, however, that nowhere in in the war efforts of the enemy, but on the contrary,
the evidence of record has it been shown that as already stated, herein appellants had verbally
FINALS CONSTITUTION I ACJUCO 183

denied the charge against them, their case should be proclamation imposes certain conditions, as in this
tried by the ordinary courts of justice. Hence, the 8th case, it is incumbent upon the accused to prove the
Guerrilla Amnesty Commission could not take existence of such conditions. The invocation of
cognizance of their case. amnesty is in the nature of a plea of confession and
avoidance, which means that the pleader admits the
Wherefore, the parties respectfully pray that the allegations against him but disclaims liability therefor
foregoing stipulation of facts be admitted and on account of intervening facts which, if proved,
approved by this Honorable Court, without prejudice would being the crime charged uithin the scope of the
to the parties adducing other evidence to prove their amnesty proclamation. (Emphasis supplied)
case not covered by this stipulation of
facts. 1äwphï1.ñët At any rate, the facts established before the Commission do
not bring this case within the terms of Amnesty Proclamation
Appellants, however, want us to rule one way or the No. 8. Note that said proclamation extends its provisions to "all
other, as to the factual question that the death of persons who committed any act penalized under the Revised
Amadeo Lozanes did not spring from personal Penal Code in furtherance of the resistance to the enemy or
motive or on account of rivalry of guerrilla units but against persons aiding in the war effort of the enemy." As found
by the Commission, the killing of the deceased (Lozañes) was
owing to the fact that the said decedent had aided in
the war efforts of the enemy. Without shirking from not in furtherance of the resistance movement, but was due to
the rivalry between the Hunter's Guerrilla, to which he
our duty to make a finding or pronouncement on a
question of fact, we are constrained not to make a belonged, and the Vera's Guerrilla of petitioners.
pronouncement on this question, in view of our ruling
stated earlier that the Commission is not competent Neither may petitioners rely on the case of Buyco v. People, et
to take cognizance of this case, for the reasons al. (L-6327, July 29, 1954) because in said case, we held that
already stated, but it should be the ordinary courts of petitioner was not entitled to the benefits of the Amnesty
justice. Any ruling that we would make now on the Proclamation not only because "the evidence did not suffice to
factual issue postulated by appellants would not only show that appellant had acted in the manner contemplated in
be premature and prejudicial, but also useless, the amnesty proclamation," namely, that he killed the
because this case proceeded from a body (the deceased Luis Gonzales due to his being an enemy
Commission) that has no jurisdiction to entertain the collaborator, but also because if petitioner's version was true
same. It may be stated, in this connection, that that he had no participation whatsoever in the killing of the
jurisdiction could be raised at any stage of the deceased, then he "had committed no crime whatsoever, and,
proceedings. hence, there would be no room, either for his conviction or for
the application of the provisions of the aforementioned
amnesty application," which, in effect, reiterates our previous
WHEREFORE, the decision and order appealed
from are hereby affirmed. ruling in the Llanitaand Guillermo cases, supra, that amnesty
cannot be invoked, where the accused actually denies the
commission of the offense charged.
IT IS SO ORDERED.
We find no merit in petitioner's claim that the Court of Appeals
Their motion for reconsideration of said decision having been erred in applying or citing Department of Justice Administrative
denied, petitioners instituted the present petition for review. Order No. 144, series of 1950,1 considering that the latter was
issued precisely pursuant to our ruling in the
Petitioners contend (as they did in the Court of Appeals), that aforesaid Llanita and Guillermo cases.
to be entitled to the benefits of Amnesty Proclamation No. 8,
dated September 7, 1946, it is not necessary for them to admit Petitioners also argue that the Court of Appeals erred in
the commission of the crime charged, citing in support of their declining to resolve the factual issues they had raised before
submission the cases of Barrioquinto, et al. vs. Fernandez, et it. The argument is untenable, because as the appellate court
al. (L-1278, January 21, 1949, 82 Phil. 642), Provincial Fiscal correctly pointed out, any ruling that it would make on the
of Ilocos Norte v. De los Santos, et al. (L-2502, December 1, factual issues presented by petitioners "would not only be
1949, 85 Phil. 77) and Viray v. Amnesty Commission, et al. (L- premature and prejudicial, but also useless, because this case
2540, January 28, 1960, 85 Phil. 354), to the effect that "in proceeded from a body (the Commission) that had no
order to entitle a person to the benefits of Amnesty jurisdiction to entertain the same."
Proclamation (No. 8) of September 7, 1946, it is not necessary
that he should, as a condition precedent or sine qua non, admit
having committed the criminal act or offense with which he is WHEREFORE, finding no error in the decision of the Court of
Appeals sought to be reviewed, the same is hereby affirmed,
charged, and allege the amnesty as a defense; it is sufficient
that the evidence, either of the complainant or the accused, with costs against the petitioners. So ordered.
shows that the offense committed comes within the terms of
said Amnesty Proclamation."

But said cases have been superseded and deemed overruled


by the subsequent cases of People v. Llanita, et al.(L-2082,
April 26, 1950, 86 Phil. 219) and People v. Guillermo, et al. (L-
2188, May 18, 1950, 86 Phil. 395), wherein we held that —

It is rank inconsistency for appellant to justify an act,


or seek forgiveness for an act which, according to
him, he has not committed. Amnesty presupposes
the commission of a crime, and when an accused
maintains that he has not committed a crime, he
cannot have any use for amnesty. Where an amnesty
FINALS CONSTITUTION I ACJUCO 184

G.R. No. 47941. December 7, 1940.] No. 11 of the municipality of Malabon, Province of Rizal.

MIGUEL CRISTOBAL, Petitioner, v. ALEJO LABRADOR, The antecedents which form the factual background of this
ET AL., Respondents. election controversy are briefly narrated as follows:chanrob1es
virtual 1aw library
Victoriano Yamzon for Petitioner.
On March 15, 1930, the Court of First Instance of Rizal found
E. Voltaire Garcia for respondent Santos. Teofilo C. Santos, respondent herein, guilty of the crime of
estafa and sentenced him to six months of arresto mayor and
SYLLABUS the accessories provided by law, to return to the offended
parties, Toribio Alarcon and Emilio Raymundo, the amounts
1. CONSTITUTIONAL LAW; PARDONING POWER OF P375 and P125, respectively, with subsidiary imprisonment in
CHIEF EXECUTIVE. — Paragraph 6 of section 11 of Article VII case of insolvency, and to pay the costs. On appeal, this court,
of our Constitution, provides:" (6) The President shall have the on December 20, 1930, confirmed the judgment of conviction.
power to grant reprieves, commutations, and pardons, and Accordingly, he was confined in the provincial jail of Pasig,
remit fines and forfeitures, after conviction, for all offenses, Rizal, from March 14, 1932 to August 18, 1932 and paid the
except in cases of impeachment, upon such conditions and corresponding costs of trial. As to his civil liability consisting in
with such restrictions and limitations as he may deem proper the return of the two amounts aforestated, the same was
to impose. He shall have the power to grant amnesty with the condoned by the complainants. Notwithstanding his conviction,
concurrence of the National Assembly." It should be observed Teofilo C. Santos continued to be a registered elector in the
that there are two limitations upon the exercise of this municipality of Malabon, Rizal, and was, for the period
constitutional prerogative by the Chief Executive, namely: (a) comprised between 1934 and 1937, seated as the municipal
that the power be exercised after conviction; and (b) that such president of that municipality. On August 22, 1938,
power does not extend to cases of impeachment. Subject to Commonwealth Act No. 357, otherwise known as the Election
the limitations imposed by the Constitution, the pardoning Code, was approved by the National Assembly, section 94,
power cannot be restricted of controlled by legislative action. It paragraph (b) of which disqualifies the respondent from voting
must remain where the sovereign authority has placed it and for having been "declared by final judgment guilty of any crime
must be exercised by the highest authority to whom it is against property." In view of this provision, the respondent
entrusted. An absolute pardon not only blots out the crime forthwith applied to His Excellency, the President, for an
committed, but removes all disabilities resulting from the absolute pardon, his petition bearing date of August 15, 1939.
conviction. Upon the favorable recommendation of the Secretary of
Justice, the Chief Executive, on December 24, 1939, granted
2. ID.; ID.; CASE AT BAR. — In the present case, the disability the said petition, restoring the respondent to his "full civil and
is the result of conviction without there would be no basis for political rights, except that with respect to the right to hold
disqualification from voting. Imprisonment is not the only public office or employment, he will be eligible for appointment
punishment which the law imposes upon those who violate its only to positions which are clerical or manual in nature and
command. There are accessory and resultant disabilities, and involving no money or property responsibility." virtua1aw
the pardoning power likewise extends to such disabilities. library
When granted after the term of imprisonment has expired,
absolute pardon removes all that is left of the consequences of On November 16, 1940, the herein petitioner, Miguel Cristobal,
conviction. In the present case, while the pardon extended to filed a petition for the exclusion of the name of Teofilo C.
respondent S is conditional in the sense that "he will be eligible Santos from the list of voters in precinct No. 11 of Malabon,
for appointment only to positions which are clerical or manual Rizal, on the ground that the latter is disqualified under
in nature involving no money or property responsibility," it is paragraph (b) of section 94 of Commonwealth Act No. 357.
absolute in so far as it "restores the respondent to full civil and After hearing, the court below rendered its decision on
political rights." (Pardon, Exhibit 1, extended December 24, November 28, 1940, the dispositive portion of which reads as
1939.) While there are cases in the United States which hold follows:jgc:chanrobles.com.ph
that the pardoning power does not restore the privilege of
voting, this is because, as stated by the learned judge below, "Without going further into a discussion of all the other minor
in the United States the right of suffrage is a matter exclusively points and questions raised by the petitioner, the court
in the hands of the State and not in the hands of the Federal declares that the pardon extended in favor of the respondent
Government (decision, page 9). Even then, there are cases to on December 24, 1939, has had the effect of excluding the
the contrary (Jones v. Board of registrars, 56 Miss, 766; respondent from the disqualification created by section 94,
Hildreth v. Heath, 1 I11. App., 82). Upon the other hand, the subsection (b) of the New Election Code. The petition for
suggestion that the disqualification imposed in paragraph (b) exclusion of the respondent Teofilo C. Santos should be, as it
of section 94 of Commonwealth Act No. 357, does not fall hereby is, denied. Let there be no costs." virtua1aw library
within the purview of the pardoning power of the Chief
Executive, would lead to the impairment of the pardoning Petitioner Cristobal has filed the present petition
power of the Chief Executive, not contemplated in the for certiorari in which he impugns the decision of the court
Constitution, and would lead furthermore to the result that there below on the several grounds stated in the petition.
would be no way of restoring the political privilege in a case of
this nature except through legislative action. It is the contention of the petitioner that the pardon granted by
His Excellency, the President of the Philippines, to the
respondent, Teofilo C. Santos, did not restore the said
DECISION respondent to the full enjoyment of his political rights, because
(a) the pardoning power of the Chief Executive does not apply
to legislative prohibitions; (b) the pardoning power here would
LAUREL, J.: amount to an unlawful exercise by the Chief Executive of a
legislative function; and (c) the respondent having served his
sentence and all the accessory penalties imposed by law, there
This is a petition for a writ of certiorari to review the decision of was nothing to pardon. All these propositions involve an inquiry
the Court of First Instance of Rizal in its election case No. 7890, into the primary question of the nature and extent of the
rendered on November 28, 1940, sustaining the right of Teofilo pardoning power vested in the Chief Executive of the Nation
C. Santos to remain in the list of registered voters in precinct by the Constitution.
FINALS CONSTITUTION I ACJUCO 185

Paragraph 6 of section 11 of Article VII of our Constitution,


provides:jgc:chanrobles.com.ph

"(6) The President shall have the power to grant reprieves,


commutations, and pardons, and remit fines and forfeitures,
after conviction, for all offenses, except in cases of
impeachment, upon such conditions and with such restrictions
and limitations as he may deem pro to impose. He shall have
the power to grant amnesty with the concurrence of the
National Assembly." virtua1aw library

It should be observed that there are two limitations upon the


exercise of this constitutional prerogative by the Chief
Executive, namely: (a) that the power be exercised after
conviction; and (b) that such power does not extend cases of
impeachment. Subject to the limitations imposed by the
Constitution, the pardoning power cannot be restricted or
controlled by legislative action. It must remain where the
sovereign authority has placed it and must be exercised by the
highest authority to whom it is entrusted. An absolute pardon
not only blots out the crime committed, but removes all
disabilities resulting from the conviction. In the present case,
the disability is the result of conviction without which there
would be no basis for disqualification from voting.
Imprisonment is not the only punishment which the law
imposes upon those who violate its command. There are
accessory and resultant disabilities, and the pardoning power
likewise extends to such disabilities. When granted after the
term of imprisonment has expired, absolute pardon removes
all that is left of the consequences f conviction. In the present
case, while the pardon extended to respondent Santos is
conditional in the sense that "he will be eligible for appointment
only to positions which a e clerical or manual in nature involving
no money or property responsibility," it is absolute insofar as it
"restores the respondent to full civil and political rights."
(Pardon, Exhibit 1, extended December 24, 1939.) While there
are cases in the United States which hold that the pardoning
power does not restore the privilege of voting, this is because,
as stated by the learned judge below, in the United States the
right of suffrage is a matter exclusively in the hands of the State
and not in the hands of the Federal Government (Decision,
page 9). Even then, there are cases to the contrary (Jones v.
Board of Registrars, 56 Miss. 766; Hildreth v. Heath, 1 Ill. App.
82). Upon other hand, the suggestion that the disqualification
imposed in paragraph (b) of section 94 of Commonwealth Act
No. 357, does not fall within the purview of the pardoning
power of the Chief Executive, would lead to the impairment of
the pardoning power of the Chief Executive, not contemplated
in the Constitution, and would lead furthermore to the result
that there would be no way of restoring the political privilege in
a case of this nature except through legislative action.
FINALS CONSTITUTION I ACJUCO 186

April 7, 1922 criminal acts may nevertheless constitute proof that the
attorney does not possess a good moral character and is not a
In Re MARCELINO LONTOK fit or proper person to retain his license to practice law. (People
vs. Burton [1907], 39 Colo., 164; People vs. George [1900],186
Ill., 122; Nelson vs. Com. [1908],128 Ky., 779; Case of In re —
Ramon Diokno for respondent. ——— [1881],86 N.Y., 563.)
Attorney-General Villa-Real for the Government.
The celebrated case of Ex parte Garland [1866], 4 Wall., 380,
MALCOLM, J.: is directly in point. The petitioner in this case applied for a
license to practice law in the United States courts, without first
The Attorney-General asks that an order issue for the removal taking an oath to the effect that he had never voluntarily given
of Marcelino Lontok from his office of lawyer in the Philippine aid to any government hostile to the United States, as required
Islands, because of having been convicted of the crime of by statute. The petitioner, it seems, had been a member of the
bigamy. The respondent lawyer, in answer, prays that the Conferate Congress, during the secession of the South, but
charges be dismissed, and bases his plea principally on a had been pardons by the President of the United States. It was
pardon issued to him by former Governor-General Harrison. held, buy a divided court, that to exclude the petitioner from the
practice of law for the offense named would be to enforce a
punishment for the offense, notwithstanding the pardon which
Marcelino Lontok was convicted by the Court of First Instance the court had no right to do; and the opinion of the court, in
of Zambales of the crime of bigamy. This judgement was part, said:
affirmed on appeal to the Supreme Court, while a further
attempt to get the case before the United States Supreme
Court was unsuccessful. On February 9, 1921, a pardon was A pardon reaches both the punishment prescribed for
issued by the Governor-General of the following tenor: the offense and the guilt of the offender; and when
the pardon is full, it releases the punishment and
blots out of existence the guilt, so that in the eye of
By virtue of the authority conferred upon me by the the law the offender is an innocent as if he had never
Philippine Organic Act on August 29, 1916, the committed the offense. If granted before conviction,
sentence in the case of Marcelino Lontok convicted it prevents any of the penalties and disabilities,
by the Court of First Instance of Zambales of bigamy consequent upon conviction, from attaching; if
and sentenced on February 27, 1918, to granted after conviction, it removes the penalties and
imprisonment for eight years, to suffer the accessory disabilities, and restores him to all his civil rights; it
penalties prescribed by law, and to pay the costs of makes him, as it were, a new man, and gives him a
the proceedings, which sentence was, on September new credit and capacity.
8, 1919, confirmed by the Supreme Court is hereby
remitted, on condition that he shall not again be guilty
of any misconduct. There is only this limitation to its operation; it does
not restore offices forfeited, or property or interest
vested in others in consequence of the conviction
The particular provision of the Code of Civil Procedure, upon and judgement.
which the Attorney-General relies in asking for the disbarment
of Attorney Lontok, provides that a member of the bar may be
removed or suspended form his office of lawyer by the Although much which is contained in the opinion of the four
Supreme Court "by reason of his conviction of a crime involving dissenting justices, in the Garland case, appeals powerfully to
moral turpitude." (Sec. 21) That conviction of the crime of the minds of the court, we feel ourselves under obligation to
bigamy involves moral turpitude, within the meaning of the law, follow the rule laid down by the majority decision of the higher
cannot be doubted. The debatable question relates to the court. We do this with the more grace when we recall that
effect of the pardon by the Governor-General. On the one according to the article 130 of the Penal Code, one of the
hand, it is contended by the Government that while the pardon different ways by which criminal liability is extinguished is by
removes the legal infamy of the crime, it cannot wash out the pardon. We must also remember that the motion for
moral stain; on the other hand, it is contended by the disbarment is based solely on the judgement of conviction for
respondent that the pardon reaches the offense for which he a crime of which the respondent has been pardoned, We must
was convicted and blots it out so that he may not be looked also remember that the motion for disbarment is based solely
upon as guilty of it. on the judgment of conviction for crime of which the respondent
has been pardoned, and that the language of the pardon is not
such as to amount to a conditional pardon similar in nature to
The cases are not altogether clear as to just what effect a a parole. It may be mentioned however, in this connection, that
pardon has on the right of a court of disbar an attorney for if Marcelino Lontok should again be guilty of any misconduct,
conviction of a felony. On close examination, however, it will the condition of his pardon would be violated, and he would
be found that the apparent conflict in the decisions is more then become subject to disbarment.
apparent than real, and arises from differences in the nature of
the charges on which the proceedings to disbar are based.
Where preceedings to strike an attorney's name from the rolls It results, therefore, that the petition of the Attorney-General
are founded on, and depend alone, on a statute making the cannot be granted, and that the proceedings must be
fact of a conviction for a felony ground for disbarment, it has dismissed. Costs shall be taxed as provided by section 24 of
been held that a pardon operates to wipe out the conviction the Code of Civil Procedure. So ordered.
and is a bar to any proceeding for the disbarment of the
attorney after the pardon has been granted. (In re Emmons
[1915], 29 Cal. App., 121; Scott vs. State [1894], 6 Tex. Civ.
App., 343). But where proceedings to disbar an attorney are
founded on the professional misconduct involved in a
transaction which has culminated in a conviction of felony, it
has been held that while the effect of the pardon is to relieve
him of the penal consequences of his act, it does not operate
as a bar to the disbarment proceedings, inasmuch as the
FINALS CONSTITUTION I ACJUCO 187

G.R. No. 76872 July 23, 1987 Decree No. 772 (interfering with police functions).
Some of these charges were Identified in the NBI
WILFREDO TORRES Y SUMULONG, petitioner, report as having been dismissed. The NBI report did
not purport to be a status report on each of the
vs.
HON. NEPTALI A. GONZALES, THE CHAIRMAN, BOARD charges there listed and Identified.
OF PARDONS AND PAROLE, and THE DIRECTOR,
BUREAU OF PRISONS, respondents. 4. On 4 June 1986, the respondent Minister of Justice
wrote to the President of the Philippines informing
FELICIANO, J.: her of the Resolution of the Board recommending
cancellation of the conditional pardon previously
granted to petitioner.
This is an original petition for habeas corpus filed on behalf of
petitioner Wilfredo S. Torres, presently confined at the National
Penitentiary in Muntinlupa. We issued the writ and during the 5. On 8 September 1986, the President cancelled the
conditional pardon of the petitioner.
hearing and from the return filed by the respondents through
the Solicitor General, and other pleadings in this case, the
following facts emerged: 6. On 10 October 1986, the respondent Minister of
Justice issued "by authority of the President" an
Order of Arrest and Recommitment against
1. Sometime before 1979 (no more specific date
appears in the records before this Court), petitioner petitioner. The petitioner was accordingly arrested
was convicted by the Court of First Instance of Manila and confined in Muntinlupa to serve the unexpired
of the crime of estafa (two counts) and was portion of his sentence.
sentenced to an aggregate prison term of from
eleven (11) years, ten (10) months and twenty-two Petitioner now impugns the validity of the Order of Arrest and
(22) days to thirty-eight (38) years, nine (9) months Recommitment. He claims that he did not violate his
and one (1) day, and to pay an indemnity of conditional pardon since he has not been convicted by final
P127,728.75 (Criminal Cases Nos. 68810, 91041 judgment of the twenty (20) counts of estafa charged in
and F-138107). These convictions were affirmed by Criminal Cases Nos. Q-19672 and Q-20756 nor of the crime of
the Court of Appeals (CA-G.R. Nos. 14773-CR and sedition in Criminal Case No. Q-22926.3 Petitioner also
17694-CR). The maximum sentence would expire on contends that he was not given an opportunity to be heard
2 November 2000.1 before he was arrested and recommitted to prison, and
accordingly claims he has been deprived of his rights under the
2. On 18 April 1979, a conditional pardon was due process clause of the Constitution.
granted to the petitioner by the President of the
Philippines on condition that petitioner would "not The issue that confronts us therefore is whether or not
again violate any of the penal laws of the Philippines. conviction of a crime by final judgment of a court is necessary
Should this condition be violated, he will be before the petitioner can be validly rearrested and recommitted
proceeded against in the manner prescribed by for violation of the terms of his conditional pardon and
law."2 Petitioner accepted the conditional pardon and accordingly to serve the balance of his original sentence.
was consequently released from confinement.
This issue is not novel. It has been raised before this Court
3. On 21 May 1986, the Board of Pardons and Parole three times in the past. This Court was first faced with this issue
(the "Board") resolved to recommend to the in Tesoro Director of Prison.4 Tesoro, who had been convicted
President the cancellation of the conditional pardon of the crime of falsification of public documents, was granted a
granted to the petitioner. In making its parole by the then Governor-General. One of the conditions of
recommendation to the President, the Board relied the parole required the parolee "not [to] commit any other crime
upon the decisions of this Court in Tesoro vs. and [to] conduct himself in an orderly manner." 5 Two years
Director of Prisons (68 Phil. 154 [1939]) after the grant of parole, Tesoro was charged before the
and Espuelas vs. Provincial Warden of Bohol (108 Justice of the Peace Court of San Juan, Rizal, with the crime
Phil. 356 [1960]). The evidence before the Board of adultery said to have been committed with the wife of
showed that on 22 March 1982 and 24 June 1982, Tesoro's brother-in-law. The fiscal filed with the Court of First
petitioner had been charged with twenty counts of Instance the corresponding information which, however, was
estafa in Criminal Cases Nos. Q-19672 and Q- dismissed for non-appearance of the complainant. The
20756, which cases were then (on 21 May 1986) complainant then went before the Board of Indeterminate
pending trial before the Regional Trial Court of Rizal Sentence and charged Tesoro with violation of the conditions
(Quezon City). The record before the Board also of his parole. After investigation by the parole officer, and on
showed that on 26 June 1985, petitioner had been the basis of his report, the Board recommended to the
convicted by the Regional Trial Court of Rizal President of the Philippines the arrest and recommitment of the
(Quezon City) of the crime of sedition in Criminal petitioner. Tesoro contended, among other things, that a
Case No. Q-22926; this conviction was then pending "judicial pronouncement to the effect that he has committed a
appeal before the Intermediate Appellate Court. The crime" is necessary before he could properly be adjudged as
Board also had before it a letter report dated 14 having violated his conditional parole.
January 1986 from the National Bureau of
Investigation ("NBI"), addressed to the Board, on the Addressing this point, this Court, speaking through then Mr.
petitioner. Per this letter, the records of the NBI Justice Moran, held that the determination of whether the
showed that a long list of charges had been brought conditions of Tesoro's parole had been breached rested
against the petitioner during the last twenty years for exclusively in the sound judgment of the Governor-General
a wide assortment of crimes including estafa, other and that such determination would not be reviewed by the
forms of swindling, grave threats, grave coercion, courts. As Tesoro had consented to place his liberty on parole
illegal possession of firearms, ammunition and upon the judgment of the power that had granted it, we held
explosives, malicious mischief, violation of Batas
that "he [could not] invoke the aid of the courts, however
Pambansa Blg. 22, and violation of Presidential erroneous the findings may be upon which his recommitment
FINALS CONSTITUTION I ACJUCO 188

was ordered."6 Thus, this Court held that by accepting the of his parole, an executive order for his arrest and
terms under which the parole had been granted, Tesoro had in remandment to prison should at once issue, and be
effect agreed that the Governor-General's determination conclusive upon him. 9
(rather than that of the regular courts of law) that he had
breached one of the conditions of his parole by committing In Espuelas vs. Provincial Warden of Bohol,10 the petitioner
adultery while he was conditionally at liberty, was binding and had been convicted of the crime of inciting to sedition. While
conclusive upon him. In reaching this conclusion, this Court serving his sentence, he was granted by the President a
relied upon Section 64 (i) of the Revised Administrative Code conditional pardon "on condition that he shall not again violate
which empowered the Governor-General any of the penal laws of the Philippines."11 Espuelas accepted
the conditional pardon and was released from confinement.
to grant to convicted prisoners reprieves or pardons, Sometime thereafter, he was convicted by the Justice of the
either plenary or partial, conditional or unconditional; Peace Court in Tagbilaran, Bohol, of the crime of usurpation of
to suspend sentences without parole, remit fines, and authority. He appealed to the Court of First Instance. Upon
order the discharge of any convicted person upon motion of the provincial fiscal, the Court of First Instance
parole, subject to such conditions as he may impose; dismissed the case provisionally, an important prosecution
and to authorize the arrest and recommitment of any witness not having been available on the day set for trial. A few
such person who, in his judgment, shall fail to comply months later, upon recommendation of the Board of Pardons
with the condition or conditions, of his pardon, and Parole, the President ordered his recommitment to prison
parole or suspension of sentence. (Emphasis to serve the unexpired period of his original sentence.
supplied)
The Court in Espuelas reaffirmed the continuing force and
In Sales vs. Director of Prisons,7 the petitioner had been effect of Section 64 (i) of the Revised Administrative Code. This
convicted of the crime of frustrated murder. After serving a little Court, quoting Tesoro and Sales, ruled that:
more than two years of his sentence, he was given a
conditional pardon by the President of the Philippines, "the Due process is not necessarily judicial The appellee
condition being that he shall not again violate any of the penal had had his day in court and been afforded the
laws of the Philippines and that, should this condition be opportunity to defend himself during his trial for the
violated, he shall be proceeded against in the manner crime of inciting to sedition, with which he was
prescribed by law."8 Eight years after the grant of his charged, that brought about or resulted in his
conditional pardon, Sales was convicted of estafa and conviction, sentence and confinement in the
sentenced to three months and eleven days of arresto mayor. penitentiary. When he was conditionally pardoned it
He was thereupon recommitted to prison to serve the
was a generous exercise by the Chief Executive of
unexpired portion of his original sentence. Sales raised before his constitutional prerogative. The acceptance
this Court two principal contentions. Firstly, he argued that
thereof by the convict or prisoner carrie[d] with it the
Section 64 (i) of the Revised Administrative Code had been authority or power of the Executive to determine
repealed by Article 159 of the Revised Penal Code. He whether a condition or conditions of the pardon has
contended, secondly, that Section 64 (i) was in any case or have been violated. To no other department of the
repugnant to the due process clause of the Constitution (Article Government [has] such power been intrusted. 12
III [1], 1935 Constitution). This Court, through Mr. Justice
Ozaeta speaking for the majority, rejected both contentions of
Sales. The status of our case law on the matter under consideration
may be summed up in the following propositions:
Sales held, firstly, that Article 159 of the Revised Penal Code
did not repeal Section 64 (i) Revised Administrative Code. It 1. The grant of pardon and the determination of the
was pointed out that Act No. 4103, the Indeterminate Sentence terms and conditions of a conditional pardon are
Law, which was enacted subsequent to the Revised Penal purely executive acts which are not subject to judicial
Code, expressly preserved the authority conferred upon the scrutiny.
President by Section 64. The Court also held that Article 159
and Section 64 (i) could stand together and that the proceeding 2. The determination of the occurrence of a breach
under one provision did not necessarily preclude action under of a condition of a pardon, and the proper
the other. Sales held, secondly, that Section 64 (i) was not consequences of such breach, may be either a
repugnant to the constitutional guarantee of due process. This purely executive act, not subject to judicial scrutiny
Court in effect held that since the petitioner was a convict "who under Section 64 (i) of the Revised Administrative
had already been seized in a constitutional was been Code; or it may be a judicial act consisting of trial for
confronted by his accusers and the witnesses against him-, and conviction of violation of a conditional pardon
been convicted of crime and been sentenced to punishment under Article 159 of the Revised Penal Code. Where
therefor," he was not constitutionally entitled to another judicial the President opts to proceed under Section 64 (i) of
determination of whether he had breached the condition of his the Revised Administrative Code, no judicial
parole by committing a subsequent offense. Thus: pronouncement of guilt of a subsequent crime is
necessary, much less conviction therefor by final
[a] statute [like Section 64 (i)] supervenes to avoid judgment of a court, in order that a convict may be
the necessity for any action by the courts in the recommended for the violation of his conditional
premises. The executive clemency under it is pardon.
extended upon the conditions named in it, and he
accepts it upon those conditions. One of these is that 3. Because due process is not semper et
the governor may withdraw his grace in a certain unique judicial process, and because the
contingency, and another is that the governor shall conditionally pardoned convict had already been
himself determine when that contingency has accorded judicial due process in his trial and
arisen. It is as if the convict, with full competency to conviction for the offense for which he was
bind himself in the premises, had expressly conditionally pardoned, Section 64 (i) of the Revised
contracted and agreed, that, whenever the governor Administrative Code is not afflicted with a
should conclude that he had violated the conditions constitutional vice.
FINALS CONSTITUTION I ACJUCO 189

We do not believe we should depart from the clear and well


understood rules and doctrine on this matter.

It may be emphasized that what is involved in the instant case


is not the prosecution of the parolee for a subsequent
offense in the regular course of administration of the criminal
law. What is involved is rather the ascertainment of whether
the convict has breached his undertaking that he would "not
again violate any of the penal laws of the Philippines" for
purposes of reimposition upon him of the remitted portion of
his original sentence. The consequences that we here deal
with are the consequences of an ascertained breach of the
conditions of a pardon. A convict granted conditional pardon,
like the petitioner herein, who is recommitted must of course
be convicted by final judgment of a court of the subsequent
crime or crimes with which he was charged before the criminal
penalty for such subsequent offense(s) can be imposed upon
him. Again, since Article 159 of the Revised Penal Code
defines a distinct, substantive, felony, the parolee or convict
who is regarded as having violated the provisions thereof must
be charged, prosecuted and convicted by final judgment before
he can be made to suffer the penalty prescribed in Article
159.1avvphi1

Succinctly put, in proceeding against a convict who has been


conditionally pardoned and who is alleged to have breached
the conditions of his pardon, the Executive Department has two
options: (i) to proceed against him under Section 64 (i) of the
Revised Administrative Code; or (ii) to proceed against him
under Article 159 of the Revised Penal Code which imposes
the penalty of prision correccional, minimum period, upon a
convict who "having been granted conditional pardon by the
Chief Executive, shall violate any of the conditions of such
pardon." Here, the President has chosen to proceed against
the petitioner under Section 64 (i) of the Revised Administrative
Code. That choice is an exercise of the President's executive
prerogative and is not subject to judicial scrutiny.

WHEREFORE, this Petition is hereby DISMISSED. No


pronouncement as to costs.

SO ORDERED.
FINALS CONSTITUTION I ACJUCO 190

[G.R. No. 141284. August 15, 2000] personnel whose training, skill, discipline and firepower prove
INTEGRATED BAR OF THE PHILIPPINES, petitioner, well-above the present capability of the local police alone to
vs. HON. RONALDO B. ZAMORA, GEN. PANFILO M. handle. The deployment of a joint PNP NCRPO-Philippine
LACSON, GEN. EDGAR B. AGLIPAY, and GEN. ANGELO Marines in the conduct of police visibility patrol in urban areas
REYES, respondents. will reduce the incidence of crimes specially those perpetrated
by active or former police/military personnel.
DECISION
4. MISSION:
KAPUNAN, J.:
The PNP NCRPO will organize a provisional Task Force to
At bar is a special civil action for certiorari and prohibition conduct joint NCRPO-PM visibility patrols to keep Metro
with prayer for issuance of a temporary restraining order Manila streets crime-free, through a sustained street patrolling
seeking to nullify on constitutional grounds the order of to minimize or eradicate all forms of high-profile crimes
President Joseph Ejercito Estrada commanding the especially those perpetrated by organized crime syndicates
deployment of the Philippine Marines (the Marines) to join the whose members include those that are well-trained, disciplined
Philippine National Police (the PNP) in visibility patrols around and well-armed active or former PNP/Military personnel.
the metropolis.
In view of the alarming increase in violent crimes in Metro 5. CONCEPT IN JOINT VISIBILITY PATROL OPERATIONS:
Manila, like robberies, kidnappings and carnappings, the
President, in a verbal directive, ordered the PNP and the a. The visibility patrols shall be conducted jointly by the
Marines to conduct joint visibility patrols for the purpose of NCRPO [National Capital Regional Police Office] and the
crime prevention and suppression. The Secretary of National Philippine Marines to curb criminality in Metro Manila and to
Defense, the Chief of Staff of the Armed Forces of the preserve the internal security of the state against insurgents
Philippines (the AFP), the Chief of the PNP and the Secretary and other serious threat to national security, although the
of the Interior and Local Government were tasked to execute primary responsibility over Internal Security Operations still
and implement the said order. In compliance with the rests upon the AFP.
presidential mandate, the PNP Chief, through Police Chief
Superintendent Edgar B. Aglipay, formulated Letter of
Instruction 02/2000[1] (the LOI) which detailed the manner by b. The principle of integration of efforts shall be applied to
which the joint visibility patrols, called Task eradicate all forms of high-profile crimes perpetrated by
Force Tulungan, would be conducted.[2] Task organized crime syndicates operating in Metro Manila. This
Force Tulungan was placed under the leadership of the Police concept requires the military and police to work cohesively and
Chief of Metro Manila. unify efforts to ensure a focused, effective and holistic
approach in addressing crime prevention. Along this line, the
Subsequently, the President confirmed his previous role of the military and police aside from neutralizing crime
directive on the deployment of the Marines in a Memorandum, syndicates is to bring a wholesome atmosphere wherein
dated 24 January 2000, addressed to the Chief of Staff of the delivery of basic services to the people and development is
AFP and the PNP Chief.[3] In the Memorandum, the President achieved. Hand-in-hand with this joint NCRPO-Philippine
expressed his desire to improve the peace and order situation Marines visibility patrols, local Police Units are responsible for
in Metro Manila through a more effective crime prevention the maintenance of peace and order in their locality.
program including increased police patrols.[4] The President
further stated that to heighten police visibility in the metropolis, c. To ensure the effective implementation of this project, a
augmentation from the AFP is necessary. [5] Invoking his
provisional Task Force TULUNGAN shall be organized to
powers as Commander-in-Chief under Section 18, Article VII provide the mechanism, structure, and procedures for the
of the Constitution, the President directed the AFP Chief of
integrated planning, coordinating, monitoring and assessing
Staff and PNP Chief to coordinate with each other for the the security situation.
proper deployment and utilization of the Marines to assist the
PNP in preventing or suppressing criminal or lawless
violence.[6] Finally, the President declared that the services of xxx.[8]
the Marines in the anti-crime campaign are merely temporary
in nature and for a reasonable period only, until such time when The selected areas of deployment under the LOI are:
the situation shall have improved.[7] Monumento Circle, North Edsa (SM City), Araneta Shopping
Center, Greenhills, SM Megamall, Makati Commercial Center,
The LOI explains the concept of the PNP-Philippine LRT/MRT Stations and the NAIA and Domestic Airport.[9]
Marines joint visibility patrols as follows:
On 17 January 2000, the Integrated Bar of the
xxx Philippines (the IBP) filed the instant petition to annul LOI
02/2000 and to declare the deployment of the Philippine
Marines, null and void and unconstitutional, arguing that:
2. PURPOSE:
I
The Joint Implementing Police Visibility Patrols between the
PNP NCRPO and the Philippine Marines partnership in the THE DEPLOYMENT OF THE PHILIPPINE MARINES IN
conduct of visibility patrols in Metro Manila for the suppression METRO MANILA IS VIOLATIVE OF THE CONSTITUTION, IN
of crime prevention and other serious threats to national THAT:
security.
A) NO EMERGENCY SITUATION OBTAINS IN METRO
3. SITUATION: MANILA AS WOULD JUSTIFY, EVEN ONLY REMOTELY,
THE DEPLOYMENT OF SOLDIERS FOR LAW
Criminal incidents in Metro Manila have been perpetrated not ENFORCEMENT WORK; HENCE, SAID DEPLOYMENT IS IN
only by ordinary criminals but also by organized syndicates DEROGATION OF ARTICLE II, SECTION 3 OF THE
whose members include active and former police/military CONSTITUTION;
FINALS CONSTITUTION I ACJUCO 191

B) SAID DEPLOYMENT CONSTITUTES AN INSIDIOUS existence of an actual and appropriate case; (2) a personal and
INCURSION BY THE MILITARY IN A CIVILIAN FUNCTION substantial interest of the party raising the constitutional
OF GOVERNMENT (LAW ENFORCEMENT) IN question; (3) the exercise of judicial review is pleaded at the
DEROGATION OF ARTICLE XVI, SECTION 5 (4), OF THE earliest opportunity; and (4) the constitutional question is
CONSTITUTION; the lis mota of the case.[12]

C) SAID DEPLOYMENT CREATES A DANGEROUS


TENDENCY TO RELY ON THE MILITARY TO PERFORM
The IBP has not sufficiently complied with the requisites
THE CIVILIAN FUNCTIONS OF THE GOVERNMENT.
of standing in this case.

II
Legal standing or locus standi has been defined as a
IN MILITARIZING LAW ENFORCEMENT IN METRO MANILA, personal and substantial interest in the case such that the party
THE ADMINISTRATION IS UNWITTINGLY MAKING THE has sustained or will sustain direct injury as a result of the
MILITARY MORE POWERFUL THAN WHAT IT SHOULD governmental act that is being challenged. [13] The term interest
REALLY BE UNDER THE CONSTITUTION.[10] means a material interest, an interest in issue affected by the
decree, as distinguished from mere interest in the question
involved, or a mere incidental interest.[14] The gist of the
Asserting itself as the official organization of Filipino question of standing is whether a party alleges such personal
lawyers tasked with the bounden duty to uphold the rule of law stake in the outcome of the controversy as to assure that
and the Constitution, the IBP questions the validity of the concrete adverseness which sharpens the presentation of
deployment and utilization of the Marines to assist the PNP in issues upon which the court depends for illumination of difficult
law enforcement. constitutional questions.[15]
Without granting due course to the petition, the Court in In the case at bar, the IBP primarily anchors its standing
a Resolution,[11] dated 25 January 2000, required the Solicitor on its alleged responsibility to uphold the rule of law and the
General to file his Comment on the petition. On 8 February Constitution. Apart from this declaration, however, the IBP
2000, the Solicitor General submitted his Comment. asserts no other basis in support of its locus standi. The mere
The Solicitor General vigorously defends the invocation by the IBP of its duty to preserve the rule of law and
constitutionality of the act of the President in deploying the nothing more, while undoubtedly true, is not sufficient to clothe
Marines, contending, among others, that petitioner has no it with standing in this case. This is too general an interest
legal standing; that the question of deployment of the Marines which is shared by other groups and the whole citizenry. Based
is not proper for judicial scrutiny since the same involves a on the standards above-stated, the IBP has failed to present a
political question; that the organization and conduct of police specific and substantial interest in the resolution of the case. Its
visibility patrols, which feature the team-up of one police officer fundamental purpose which, under Section 2, Rule 139-A of
and one Philippine Marine soldier, does not violate the civilian the Rules of Court, is to elevate the standards of the law
supremacy clause in the Constitution. profession and to improve the administration of justice is alien
to, and cannot be affected by the deployment of the Marines. It
The issues raised in the present petition are: (1) Whether should also be noted that the interest of the National President
or not petitioner has legal standing; (2) Whether or not the of the IBP who signed the petition, is his alone, absent a formal
Presidents factual determination of the necessity of calling the board resolution authorizing him to file the present action. To
armed forces is subject to judicial review; and, (3) Whether or be sure, members of the BAR, those in the judiciary included,
not the calling of the armed forces to assist the PNP in joint have varying opinions on the issue. Moreover, the IBP,
visibility patrols violates the constitutional provisions on civilian assuming that it has duly authorized the National President to
supremacy over the military and the civilian character of the file the petition, has not shown any specific injury which it has
PNP. suffered or may suffer by virtue of the questioned
governmental act. Indeed, none of its members, whom the IBP
The petition has no merit. purportedly represents, has sustained any form of injury as a
result of the operation of the joint visibility patrols. Neither is it
First, petitioner failed to sufficiently show that it is in
alleged that any of its members has been arrested or that their
possession of the requisites of standing to raise the issues in
civil liberties have been violated by the deployment of the
the petition. Second, the President did not commit grave abuse
Marines. What the IBP projects as injurious is the supposed
of discretion amounting to lack or excess of jurisdiction nor did
militarization of law enforcement which might threaten
he commit a violation of the civilian supremacy clause of the
Philippine democratic institutions and may cause more harm
Constitution.
than good in the long run. Not only is the presumed injury not
The power of judicial review is set forth in Section 1, personal in character, it is likewise too vague, highly
Article VIII of the Constitution, to wit: speculative and uncertain to satisfy the requirement of
standing. Since petitioner has not successfully established a
direct and personal injury as a consequence of the questioned
Section 1. The judicial power shall be vested in one Supreme act, it does not possess the personality to assail the validity of
Court and in such lower courts as may be established by law. the deployment of the Marines. This Court, however, does not
categorically rule that the IBP has absolutely no standing to
Judicial power includes the duty of the courts of justice to settle raise constitutional issues now or in the future. The IBP must,
actual controversies involving rights which are legally by way of allegations and proof, satisfy this Court that it has
demandable and enforceable, and to determine whether or not sufficient stake to obtain judicial resolution of the controversy.
there has been grave abuse of discretion amounting to lack or
excess of jurisdiction on the part of any branch or Having stated the foregoing, it must be emphasized that
instrumentality of the Government. this Court has the discretion to take cognizance of a suit which
does not satisfy the requirement of legal standing when
paramount interest is involved.[16] In not a few cases, the Court
When questions of constitutional significance are raised, has adopted a liberal attitude on the locus standi of a petitioner
the Court can exercise its power of judicial review only if the where the petitioner is able to craft an issue of transcendental
following requisites are complied with, namely: (1) the significance to the people.[17] Thus, when the issues raised are
FINALS CONSTITUTION I ACJUCO 192

of paramount importance to the public, the Court may brush cannot be said to exclude the Presidents exercising as
aside technicalities of procedure.[18] In this case, a reading of Commander-in-Chief powers short of the calling of the armed
the petition shows that the IBP has advanced constitutional forces, or suspending the privilege of the writ of habeas
issues which deserve the attention of this Court in view of their corpus or declaring martial law, in order to keep the peace, and
seriousness, novelty and weight as precedents.Moreover, maintain public order and security.
because peace and order are under constant threat and
lawless violence occurs in increasing tempo, undoubtedly xxx[21]
aggravated by the Mindanao insurgency problem, the legal
controversy raised in the petition almost certainly will not go Nonetheless, even if it is conceded that the power
away. It will stare us in the face again. It, therefore, behooves involved is the Presidents power to call out the armed forces
the Court to relax the rules on standing and to resolve the issue to prevent or suppress lawless violence, invasion or rebellion,
now, rather than later. the resolution of the controversy will reach a similar result.
We now address the Solicitor Generals argument that the
issue involved is not susceptible to review by the judiciary
The President did not commit grave abuse of discretion in calling out the Marines. because it involves a political question, and thus,
not justiciable.
As a general proposition, a controversy is justiciable if it
In the case at bar, the bone of contention concerns the
refers to a matter which is appropriate for court review. [22] It
factual determination of the President of the necessity of calling
pertains to issues which are inherently susceptible of being
the armed forces, particularly the Marines, to aid the PNP in
decided on grounds recognized by law. Nevertheless, the
visibility patrols. In this regard, the IBP admits that the
Court does not automatically assume jurisdiction over actual
deployment of the military personnel falls under the
constitutional cases brought before it even in instances that are
Commander-in-Chief powers of the President as stated in
ripe for resolution. One class of cases wherein the Court
Section 18, Article VII of the Constitution, specifically, the
hesitates to rule on are political questions. The reason is that
power to call out the armed forces to prevent or suppress
political questions are concerned with issues dependent upon
lawless violence, invasion or rebellion. What the IBP
the wisdom, not the legality, of a particular act or measure
questions, however, is the basis for the calling of the Marines
being assailed. Moreover, the political question being a
under the aforestated provision. According to the IBP, no
function of the separation of powers, the courts will not
emergency exists that would justify the need for the calling of
normally interfere with the workings of another co-equal branch
the military to assist the police force. It contends that no
unless the case shows a clear need for the courts to step in to
lawless violence, invasion or rebellion exist to warrant the
uphold the law and the Constitution.
calling of the Marines. Thus, the IBP prays that this Court
review the sufficiency of the factual basis for said troop As Taada v. Cuenco[23] puts it, political questions refer to
[Marine] deployment.[19] those questions which, under the Constitution, are to be
decided by the people in their sovereign capacity, or in regard
The Solicitor General, on the other hand, contends that
to which full discretionary authority has been delegated to the
the issue pertaining to the necessity of calling the armed forces
legislative or executive branch of government. Thus, if an issue
is not proper for judicial scrutiny since it involves a political
is clearly identified by the text of the Constitution as matters for
question and the resolution of factual issues which are beyond
discretionary action by a particular branch of government or to
the review powers of this Court.
the people themselves then it is held to be a political
As framed by the parties, the underlying issues are the question. In the classic formulation of Justice Brennan in Baker
scope of presidential powers and limits, and the extent of v. Carr,[24] [p]rominent on the surface of any case held to
judicial review. But, while this Court gives considerable weight involve a political question is found a textually demonstrable
to the parties formulation of the issues, the resolution of the constitutional commitment of the issue to a coordinate political
controversy may warrant a creative approach that goes department; or a lack of judicially discoverable and
beyond the narrow confines of the issues raised. Thus, while manageable standards for resolving it; or the impossibility of
the parties are in agreement that the power exercised by the deciding without an initial policy determination of a kind clearly
President is the power to call out the armed forces, the Court for nonjudicial discretion; or the impossibility of a courts
is of the view that the power involved may be no more than the undertaking independent resolution without expressing lack of
maintenance of peace and order and promotion of the general the respect due coordinate branches of government; or an
welfare.[20] For one, the realities on the ground do not show that unusual need for unquestioning adherence to a political
there exist a state of warfare, widespread civil unrest or decision already made; or the potentiality of embarassment
anarchy. Secondly, the full brunt of the military is not brought from multifarious pronouncements by various departments on
upon the citizenry, a point discussed in the latter part of this the one question.
decision. In the words of the late Justice Irene Cortes
The 1987 Constitution expands the concept of judicial
in Marcos v. Manglapus:
review by providing that (T)he Judicial power shall be vested
in one Supreme Court and in such lower courts as may be
More particularly, this case calls for the exercise of the established by law. Judicial power includes the duty of the
Presidents powers as protector of the peace. [Rossiter, The courts of justice to settle actual controversies involving rights
American Presidency]. The power of the President to keep the which are legally demandable and enforceable, and to
peace is not limited merely to exercising the commander-in- determine whether or not there has been a grave abuse of
chief powers in times of emergency or to leading the State discretion amounting to lack or excess of jurisdiction on the
against external and internal threats to its existence. The part of any branch or instrumentality of the
President is not only clothed with extraordinary powers in times Government.[25] Under this definition, the Court cannot agree
of emergency, but is also tasked with attending to the day-to- with the Solicitor General that the issue involved is a political
day problems of maintaining peace and order and ensuring question beyond the jurisdiction of this Court to review. When
domestic tranquility in times when no foreign foe appears on the grant of power is qualified, conditional or subject to
the horizon. Wide discretion, within the bounds of law, in limitations, the issue of whether the prescribed qualifications or
fulfilling presidential duties in times of peace is not in any way conditions have been met or the limitations respected, is
diminished by the relative want of an emergency specified in justiciable - the problem being one of legality or validity, not its
the commander-in-chief provision. For in making the President wisdom.[26] Moreover, the jurisdiction to delimit constitutional
commander-in-chief the enumeration of powers that follow boundaries has been given to this Court. [27] When political
FINALS CONSTITUTION I ACJUCO 193

questions are involved, the Constitution limits the the President shall submit a report in person or in writing to the
determination as to whether or not there has been a grave Congress. The Congress, voting jointly, by a vote of at least a
abuse of discretion amounting to lack or excess of jurisdiction majority of all its Members in regular or special session, may
on the part of the official whose action is being questioned. [28] revoke such proclamation or suspension, which revocation
shall not be set aside by the President. Upon the initiative of
By grave abuse of discretion is meant simply capricious the President, the Congress may, in the same manner, extend
or whimsical exercise of judgment that is patent and gross as such proclamation or suspension for a period to be determined
to amount to an evasion of positive duty or a virtual refusal to by the Congress, if the invasion or rebellion shall persist and
perform a duty enjoined by law, or to act at all in contemplation public safety requires it.
of law, as where the power is exercised in an arbitrary and
despotic manner by reason of passion or hostility. [29] Under this
definition, a court is without power to directly decide matters The Congress, if not in session, shall within twenty-four hours
over which full discretionary authority has been delegated. But following such proclamation or suspension, convene in
while this Court has no power to substitute its judgment for that accordance with its rules without need of a call.
of Congress or of the President, it may look into the question
of whether such exercise has been made in grave abuse of The Supreme Court may review, in an appropriate proceeding
discretion.[30] A showing that plenary power is granted either filed by any citizen, the sufficiency of the factual basis of the
department of government, may not be an obstacle to judicial proclamation of martial law or the suspension of the privilege
inquiry, for the improvident exercise or abuse thereof may give of the writ or the extension thereof, and must promulgate its
rise to justiciable controversy.[31] decision thereon within thirty days from its filing.
When the President calls the armed forces to prevent or
suppress lawless violence, invasion or rebellion, he A state of martial law does not suspend the operation of the
necessarily exercises a discretionary power solely vested in Constitution, nor supplant the functioning of the civil courts or
his wisdom.This is clear from the intent of the framers and from legislative assemblies, nor authorize the conferment of
the text of the Constitution itself. The Court, thus, cannot be jurisdiction on military courts and agencies over civilians where
called upon to overrule the Presidents wisdom or substitute its civil courts are able to function, nor automatically suspend the
own.However, this does not prevent an examination of whether privilege of the writ.
such power was exercised within permissible constitutional
limits or whether it was exercised in a manner constituting The suspension of the privilege of the writ shall apply only to
grave abuse of discretion. In view of the constitutional intent to persons judicially charged for rebellion or offenses inherent in
give the President full discretionary power to determine the or directly connected with invasion.
necessity of calling out the armed forces, it is incumbent upon
the petitioner to show that the Presidents decision is totally
bereft of factual basis. The present petition fails to discharge During the suspension of the privilege of the writ, any person
such heavy burden as there is no evidence to support the thus arrested or detained shall be judicially charged within
assertion that there exist no justification for calling out the three days, otherwise he shall be released.
armed forces. There is, likewise, no evidence to support the
proposition that grave abuse was committed because the Under the foregoing provisions, Congress may revoke
power to call was exercised in such a manner as to violate the such proclamation or suspension and the Court may review the
constitutional provision on civilian supremacy over the sufficiency of the factual basis thereof. However, there is no
military. In the performance of this Courts duty of purposeful such equivalent provision dealing with the revocation or review
hesitation[32] before declaring an act of another branch as of the Presidents action to call out the armed forces. The
unconstitutional, only where such grave abuse of discretion is distinction places the calling out power in a different category
clearly shown shall the Court interfere with the Presidents from the power to declare martial law and the power to
judgment. To doubt is to sustain. suspend the privilege of the writ of habeas corpus, otherwise,
the framers of the Constitution would have simply lumped
There is a clear textual commitment under the together the three powers and provided for their revocation and
Constitution to bestow on the President full discretionary power review without any qualification. Expressio unius est exclusio
to call out the armed forces and to determine the necessity for alterius. Where the terms are expressly limited to certain
the exercise of such power. Section 18, Article VII of the matters, it may not, by interpretation or construction, be
Constitution, which embodies the powers of the President as extended to other matters.[33] That the intent of the Constitution
Commander-in-Chief, provides in part: is exactly what its letter says, i.e., that the power to call is fully
discretionary to the President, is extant in the deliberation of
The President shall be the Commander-in-Chief of all armed the Constitutional Commission, to wit:
forces of the Philippines and whenever it becomes necessary,
he may call out such armed forces to prevent or suppress FR. BERNAS. It will not make any difference. I may add that
lawless violence, invasion or rebellion. In case of invasion or there is a graduated power of the President as Commander-in-
rebellion, when the public safety requires it, he may, for a Chief. First, he can call out such Armed Forces as may be
period not exceeding sixty days, suspend the privilege of the necessary to suppress lawless violence; then he can suspend
writ of habeas corpus, or place the Philippines or any part the privilege of the writ of habeas corpus, then he can impose
thereof under martial law. martial law. This is a graduated sequence.

xxx When he judges that it is necessary to impose martial law or


The full discretionary power of the President to determine suspend the privilege of the writ of habeas corpus, his
the factual basis for the exercise of the calling out power is also judgment is subject to review. We are making it subject to
implied and further reinforced in the rest of Section 18, Article review by the Supreme Court and subject to concurrence by
VII which reads, thus: the National Assembly. But when he exercises this lesser
power of calling on the Armed Forces, when he says it is
xxx necessary, it is my opinion that his judgment cannot be
reviewed by anybody.
Within forty-eight hours from the proclamation of martial law or
the suspension of the privilege of the writ of habeas corpus, xxx
FINALS CONSTITUTION I ACJUCO 194

FR. BERNAS. Let me just add that when we only have subjected to unfettered judicial scrutiny could be a veritable
imminent danger, the matter can be handled by the first prescription for disaster, as such power may be unduly
sentence: The President may call out such armed forces to straitjacketed by an injunction or a temporary restraining order
prevent or suppress lawless violence, invasion or rebellion. So every time it is exercised.
we feel that that is sufficient for handling imminent danger.
Thus, it is the unclouded intent of the Constitution to vest
upon the President, as Commander-in-Chief of the Armed
MR. DE LOS REYES. So actually, if a President feels that Forces, full discretion to call forth the military when in his
there is imminent danger, the matter can be handled by the judgment it is necessary to do so in order to prevent or
First Sentence: The President....may call out such Armed suppress lawless violence, invasion or rebellion. Unless the
Forces to prevent or suppress lawless violence, invasion or petitioner can show that the exercise of such discretion was
rebellion. So we feel that that is sufficient for handling imminent gravely abused, the Presidents exercise of judgment deserves
danger, of invasion or rebellion, instead of imposing martial law to be accorded respect from this Court.
or suspending the writ of habeas corpus, he must necessarily
have to call the Armed Forces of the Philippines as their The President has already determined the necessity and
Commander-in-Chief. Is that the idea? factual basis for calling the armed forces. In his Memorandum,
he categorically asserted that, [V]iolent crimes like bank/store
MR. REGALADO. That does not require any concurrence by robberies, holdups, kidnappings and carnappings continue to
the legislature nor is it subject to judicial review. [34] occur in Metro Manila...[35] We do not doubt the veracity of the
Presidents assessment of the situation, especially in the light
of present developments. The Court takes judicial notice of the
The reason for the difference in the treatment of the recent bombings perpetrated by lawless elements in the
aforementioned powers highlights the intent to grant the shopping malls, public utilities, and other public places. These
President the widest leeway and broadest discretion in using are among the areas of deployment described in the LOI
the power to call out because it is considered as the lesser and 2000. Considering all these facts, we hold that the President
more benign power compared to the power to suspend the has sufficient factual basis to call for military aid in law
privilege of the writ of habeas corpus and the power to impose enforcement and in the exercise of this constitutional power.
martial law, both of which involve the curtailment and
suppression of certain basic civil rights and individual
freedoms, and thus necessitating safeguards by Congress and
review by this Court. The deployment of the Marines does not violate the
civilian supremacy clause nor does it infringe the civilian
Moreover, under Section 18, Article VII of the character of the police force.
Constitution, in the exercise of the power to suspend the
privilege of the writ of habeas corpus or to impose martial law,
two conditions must concur: (1) there must be an actual
invasion or rebellion and, (2) public safety must require it. Prescinding from its argument that no emergency
These conditions are not required in the case of the power to situation exists to justify the calling of the Marines, the IBP
call out the armed forces. The only criterion is that whenever it asserts that by the deployment of the Marines, the civilian task
becomes necessary, the President may call the armed forces of law enforcement is militarized in violation of Section 3,
to prevent or suppress lawless violence, invasion or rebellion." Article II[36] of the Constitution.
The implication is that the President is given full discretion and We disagree. The deployment of the Marines does not
wide latitude in the exercise of the power to call as compared constitute a breach of the civilian supremacy clause. The
to the two other powers. calling of the Marines in this case constitutes permissible use
If the petitioner fails, by way of proof, to support the of military assets for civilian law enforcement. The participation
assertion that the President acted without factual basis, then of the Marines in the conduct of joint visibility patrols is
this Court cannot undertake an independent investigation appropriately circumscribed. The limited participation of the
beyond the pleadings. The factual necessity of calling out the Marines is evident in the provisions of the LOI itself, which
armed forces is not easily quantifiable and cannot be sufficiently provides the metes and bounds of the Marines
objectively established since matters considered for satisfying authority. It is noteworthy that the local police forces are the
the same is a combination of several factors which are not ones in charge of the visibility patrols at all times, the real
always accessible to the courts. Besides the absence of textual authority belonging to the PNP. In fact, the Metro Manila Police
standards that the court may use to judge necessity, Chief is the overall leader of the PNP-Philippine Marines joint
information necessary to arrive at such judgment might also visibility patrols.[37] Under the LOI, the police forces are tasked
prove unmanageable for the courts. Certain pertinent to brief or orient the soldiers on police patrol procedures. [38] It
information might be difficult to verify, or wholly unavailable to is their responsibility to direct and manage the deployment of
the courts. In many instances, the evidence upon which the the Marines.[39] It is, likewise, their duty to provide the
President might decide that there is a need to call out the necessary equipment to the Marines and render logistical
armed forces may be of a nature not constituting technical support to these soldiers.[40] In view of the foregoing, it cannot
proof. be properly argued that military authority is supreme over
civilian authority. Moreover, the deployment of the Marines to
On the other hand, the President as Commander-in- assist the PNP does not unmake the civilian character of the
Chief has a vast intelligence network to gather information, police force. Neither does it amount to an insidious incursion of
some of which may be classified as highly confidential or the military in the task of law enforcement in violation of Section
affecting the security of the state. In the exercise of the power 5(4), Article XVI of the Constitution.[41]
to call, on-the-spot decisions may be imperatively necessary in
emergency situations to avert great loss of human lives and In this regard, it is not correct to say that General Angelo
mass destruction of property. Indeed, the decision to call out Reyes, Chief of Staff of the AFP, by his alleged involvement in
the military to prevent or suppress lawless violence must be civilian law enforcement, has been virtually appointed to a
done swiftly and decisively if it were to have any effect at civilian post in derogation of the aforecited provision. The real
all. Such a scenario is not farfetched when we consider the authority in these operations, as stated in the LOI, is lodged
present situation in Mindanao, where the insurgency problem with the head of a civilian institution, the PNP, and not with the
could spill over the other parts of the country. The military. Such being the case, it does not matter whether the
determination of the necessity for the calling out power if AFP Chief actually participates in the Task
Force Tulungan since he does not exercise any authority or
FINALS CONSTITUTION I ACJUCO 195

control over the same.Since none of the Marines was civilian law enforcement is generally prohibited, except in
incorporated or enlisted as members of the PNP, there can be certain allowable circumstances. A provision of the Act states:
no appointment to civilian position to speak of. Hence, the
deployment of the Marines in the joint visibility patrols does not 1385. Use of Army and Air Force as posse comitatus
destroy the civilian character of the PNP.
Considering the above circumstances, the Marines Whoever, except in cases and under circumstances expressly
render nothing more than assistance required in conducting authorized by the Constitution or Act of Congress, willfully uses
the patrols. As such, there can be no insidious incursion of the any part of the Army or the Air Force as posse comitatus or
military in civilian affairs nor can there be a violation of the otherwise to execute the laws shall be fined not more than
civilian supremacy clause in the Constitution. $10,000 or imprisoned not more than two years, or both. [62]
It is worth mentioning that military assistance to civilian
authorities in various forms persists in Philippine To determine whether there is a violation of the Posse
jurisdiction. The Philippine experience reveals that it is not Comitatus Act in the use of military personnel, the US
averse to requesting the assistance of the military in the courts[63] apply the following standards, to wit:
implementation and execution of certain traditionally civil
functions. As correctly pointed out by the Solicitor General, Were Army or Air Force personnel used by the civilian law
some of the multifarious activities wherein military aid has been enforcement officers at Wounded Knee in such a manner that
rendered, exemplifying the activities that bring both the civilian the military personnel subjected the citizens to the exercise of
and the military together in a relationship of cooperation, are: military power which was regulatory, proscriptive, or
compulsory[64] George Washington Law Review, pp. 404-433
1. Elections;[42]
(1986), which discusses the four divergent standards for
2. Administration of the Philippine National Red assessing acceptable involvement of military personnel in civil
Cross;[43] law enforcement. See likewise HONORED IN THE BREECH:
PRESIDENTIAL AUTHORITY TO EXECUTE THE LAWS
3. Relief and rescue operations during calamities WITH MILITARY FORCE, 83 Yale Law Journal, pp. 130-152,
and disasters;[44] 1973. 64 in nature, either presently or prospectively?
4. Amateur sports promotion and development; [45]
xxx
5. Development of the culture and the arts; [46]

6. Conservation of natural resources;[47] When this concept is transplanted into the present legal
context, we take it to mean that military involvement, even
7. Implementation of the agrarian reform when not expressly authorized by the Constitution or a
program;[48] statute, does not violate the Posse Comitatus Act unless it
actually regulates, forbids or compels some conduct on the
8. Enforcement of customs laws;[49] part of those claiming relief. A mere threat of some future injury
would be insufficient.(emphasis supplied)
9. Composite civilian-military law enforcement
activities;[50]
Even if the Court were to apply the above rigid standards
10. Conduct of licensure examinations; [51] to the present case to determine whether there is permissible
use of the military in civilian law enforcement, the conclusion is
11. Conduct of nationwide tests for elementary and
inevitable that no violation of the civilian supremacy clause in
high school students;[52]
the Constitution is committed. On this point, the Court agrees
12. Anti-drug enforcement activities;[53] with the observation of the Solicitor General:

13. Sanitary inspections;[54] 3. The designation of tasks in Annex A[65] does not
14. Conduct of census work; [55] constitute the exercise of regulatory, proscriptive, or
compulsory military power. First, the soldiers do not
15. Administration of the Civil Aeronautics control or direct the operation. This is evident from Nos.
Board;[56] 6,[66] 8(k)[67] and 9(a)[68] of Annex A. These soldiers,
second, also have no power to prohibit or condemn. In
16. Assistance in installation of weather forecasting No. 9(d)[69] of Annex A, all arrested persons are brought
devices;[57] to the nearest police stations for proper disposition.And
last, these soldiers apply no coercive force. The
17. Peace and order policy formulation in local
materials or equipment issued to them, as shown in No.
government units.[58]
8(c)[70] of Annex A, are all low impact and defensive in
This unquestionably constitutes a gloss on executive character. The conclusion is that there being no exercise
power resulting from a systematic, unbroken, executive of regulatory, proscriptive or compulsory military power,
practice, long pursued to the knowledge of Congress and, yet, the deployment of a handful of Philippine Marines
never before questioned.[59] What we have here is mutual constitutes no impermissible use of military power for
support and cooperation between the military and civilian civilian law enforcement.[71]
authorities, not derogation of civilian supremacy.
It appears that the present petition is anchored on fear
In the United States, where a long tradition of suspicion that once the armed forces are deployed, the military will gain
and hostility towards the use of military force for domestic ascendancy, and thus place in peril our cherished
purposes has persisted,[60] and whose Constitution, unlike liberties. Such apprehensions, however, are unfounded. The
ours, does not expressly provide for the power to call, the use power to call the armed forces is just that - calling out the
of military personnel by civilian law enforcement officers is armed forces. Unless, petitioner IBP can show, which it has
allowed under circumstances similar to those surrounding the not, that in the deployment of the Marines, the President has
present deployment of the Philippine Marines. Under violated the fundamental law, exceeded his authority or
the Posse Comitatus Act[61] of the US, the use of the military in jeopardized the civil liberties of the people, this Court is not
FINALS CONSTITUTION I ACJUCO 196

inclined to overrule the Presidents determination of the factual


basis for the calling of the Marines to prevent or suppress
lawless violence.
One last point. Since the institution of the joint visibility
patrol in January, 2000, not a single citizen has complained
that his political or civil rights have been violated as a result of
the deployment of the Marines. It was precisely to safeguard
peace, tranquility and the civil liberties of the people that the
joint visibility patrol was conceived. Freedom and democracy
will be in full bloom only when people feel secure in their homes
and in the streets, not when the shadows of violence and
anarchy constantly lurk in their midst.
WHEREFORE, premises considered, the petition is
hereby DISMISSED.
SO ORDERED.
FINALS CONSTITUTION I ACJUCO 197

[G.R. No. 159085. February 3, 2004] crime of rebellion punishable under Article 134 of the Revised
SANLAKAS, represented by REP. J.V. Bautista, and Penal Code, as amended;
PARTIDO NG MANGGAGAWA, represented by REP.
RENATO MAGTUBO petitioners, vs. EXECUTIVE
WHEREAS, these misguided elements of the Armed Forces of
SECRETARY SECRETARY ANGELO REYES, GENERAL the Philippines are being supported, abetted and aided by
NARCISO ABAYA, DIR. GEN. HERMOGENES known and unknown leaders, conspirators and plotters in the
EBDANE, respondents. government service and outside the government;
[G.R. No. 159103. February 3, 2004]
SOCIAL JUSTICE SOCIETY (SJS) OFFICERS/MEMBERS WHEREAS, under Section 18, Article VII of the present
namely, SAMSON S. ALCANTARA, ED VINCENT S. Constitution, whenever it becomes necessary, the President,
ALBANO, RENE B. GOROSPE, EDWIN R. SANDOVAL and as the Commander-in-Chief of the Armed Forces of the
RODOLFO D. MAPILE, petitioners, vs. HON. EXECUTIVE Philippines, may call out such Armed Forces to suppress the
SECRETARY ALBERTO G. ROMULO, HON. SECRETARY rebellion;
OF JUSTICE SIMEON DATUMANONG, HON. SECRETARY
OF NATIONAL DEFENSE ANGELO REYES, and HON. NOW, THEREFORE, I, GLORIA MACAPAGAL-ARROYO, by
SECRETARY JOSE LINA, JR., respondents. virtue of the powers vested in me by law, hereby confirm the
existence of an actual and on-going rebellion, compelling me
[G.R. No. 159185. February 3, 2004] to declare a state of rebellion.
REP. ROLEX T. SUPLICO, REP. CARLOS M. PADILLA,
REP. CELSO L. LOBREGAT, REP. HUSSIN U. AMIN, REP.
ABRAHAM KAHLIL B. MITRA, REP. EMMYLOU J. TALINO- In view of the foregoing, I am issuing General Order No. 4 in
SANTOS, and REP. GEORGILU R. YUMUL- accordance with Section 18, Article VII of the Constitution,
HERMIDA, petitioners, vs. PRESIDENT GLORIA calling out the Armed Forces of the Philippines and the
MACAPAGAL-ARROYO; and EXECUTIVE SECRETARY Philippine National Police to immediately carry out the
ALBERTO G. ROMULO, respondents. necessary actions and measures to suppress and quell the
rebellion with due regard to constitutional rights.
[G.R. No. 159196. February 3, 2004]
AQUILINO Q. PIMENTEL, JR. as a Member of the General Order No. 4 is similarly worded:
Senate, petitioner, vs. SECRETARY ALBERTO ROMULO,
AS EXECUTIVE SECRETARY; SECRETARY ANGELO
REYES, AS SECRETARY OF NATIONAL DEFENSE; GENERAL ORDER NO. 4
GENERAL NARCISO ABAYA, AS CHIEF OF STAFF OF THE
ARMED FORCES; SECRETARY JOSE LINA, et DIRECTING THE ARMED FORCES OF THE PHILIPPINES
al., respondents. AND THE PHILIPPINE NATIONAL POLICE TO SUPPRESS
REBELLION
DECISION
WHEREAS, certain elements of the Armed Forces of the
TINGA, J.: Philippines, armed with high-powered firearms and explosives,
acting upon the instigation and command and direction of
They came in the middle of the night. Armed with high- known and unknown leaders, have seized a building in Makati
powered ammunitions and explosives, some three hundred City, put bombs in the area, publicly declared withdrawal of
junior officers and enlisted men of the Armed Forces of the support for, and took arms against the duly constituted
Philippines (AFP) stormed into the Oakwood Premiere Government, and continue to rise publicly and show open
apartments in Makati City in the wee hours of July 27, hostility, for the purpose of removing allegiance to the
2003. Bewailing the corruption in the AFP, the soldiers Government certain bodies of the Armed Forces of the
demanded, among other things, the resignation of the Philippines and the Philippine National Police, and depriving
President, the Secretary of Defense and the Chief of the the President of the Republic of the Philippines, wholly or
Philippine National Police (PNP).[1] partially, of her powers and prerogatives which constitute the
crime of rebellion punishable under Article 134 et seq. of the
In the wake of the Oakwood occupation, the President Revised Penal Code, as amended;
issued later in the day Proclamation No. 427 and General
Order No. 4, both declaring a state of rebellion and calling out
WHEREAS, these misguided elements of the Armed Forces of
the Armed Forces to suppress the rebellion. Proclamation No.
the Philippines are being supported, abetted and aided by
427 reads in full:
known and unknown leaders, conspirators and plotters in the
government service and outside the government;
PROCLAMATION NO. 427
WHEREAS, under Section 18, Article VII of the present
DECLARING A STATE OF REBELLION Constitution, whenever it becomes necessary, the President,
as the Commander-in-Chief of all Armed Forces of the
WHEREAS, certain elements of the Armed Forces of the Philippines, may call out such Armed Forces to suppress the
Philippines, armed with high-powered firearms and explosives, rebellion;
acting upon the instigation and command and direction of
known and unknown leaders, have seized a building in Makati NOW, THEREFORE, I, GLORIA MACAPAGAL-ARROYO, by
City, put bombs in the area, publicly declared withdrawal of virtue of the powers vested in me by the Constitution as
support for, and took arms against the duly constituted President of the Republic of the Philippines and Commander-
Government, and continue to rise publicly and show open in-Chief of all the armed forces of the Philippines and pursuant
hostility, for the purpose of removing allegiance to the to Proclamation No. 427 dated July 27, 2003, do hereby call
Government certain bodies of the Armed Forces of the upon the Armed Forces of the Philippines and the Philippine
Philippines and the Philippine National Police, and depriving National Police to suppress and quell the rebellion.
the President of the Republic of the Philippines, wholly or
partially, of her powers and prerogatives which constitute the
FINALS CONSTITUTION I ACJUCO 198

I hereby direct the Chief of the Armed Forces of the Philippines House of Representatives whose rights, powers and functions
and the Chief of the Philippine National Police and the officers were allegedly affected by the declaration of a state of
and men of the Armed Forces of the Philippines and the rebellion.[10] Petitioners do not challenge the power of the
Philippine National Police to immediately carry out the President to call out the Armed Forces.[11] They argue,
necessary and appropriate actions and measures to suppress however, that the declaration of a state of rebellion is a
and quell the rebellion with due regard to constitutional rights. superfluity, and is actually an exercise of emergency
powers.[12] Such exercise, it is contended, amounts to a
By the evening of July 27, 2003, the Oakwood usurpation of the power of Congress granted by Section 23 (2),
occupation had ended. After hours-long negotiations, the Article VI of the Constitution.[13]
soldiers agreed to return to barracks. The President, however, In G.R. No. 159196 (Pimentel v. Romulo, et al.),
did not immediately lift the declaration of a state of rebellion petitioner Senator assails the subject presidential issuances as
and did so only on August 1, 2003, through Proclamation No. an unwarranted, illegal and abusive exercise of a martial law
435: power that has no basis under the Constitution. [14] In the main,
petitioner fears that the declaration of a state of rebellion opens
DECLARING THAT THE STATE OF REBELLION HAS the door to the unconstitutional implementation of warrantless
CEASED TO EXIST arrests for the crime of rebellion.[15]
Required to comment, the Solicitor General argues that
WHEREAS, by virtue of Proclamation No. 427 dated July 27, the petitions have been rendered moot by the lifting of the
2003, a state of rebellion was declared; declaration.[16] In addition, the Solicitor General questions the
standing of the petitioners to bring suit. [17]
WHEREAS, by virtue of General Order No. 4 dated July 27,
The Court agrees with the Solicitor General that the
2003, which was issued on the basis of Proclamation No. 427
issuance of Proclamation No. 435, declaring that the state of
dated July 27, 2003, and pursuant to Article VII, Section 18 of
rebellion has ceased to exist, has rendered the case moot. As
the Constitution, the Armed Forces of the Philippines and the
a rule, courts do not adjudicate moot cases, judicial power
Philippine National Police were directed to suppress and quell
being limited to the determination
the rebellion;
of actual controversies.[18] Nevertheless, courts will decide a
question, otherwise moot, if it is capable of repetition yet
WHEREAS, the Armed Forces of the Philippines and the evading review.[19] The case at bar is one such case.
Philippine National Police have effectively suppressed and
quelled the rebellion. Once before, the President on May 1, 2001 declared a
state of rebellion and called upon the AFP and the PNP to
suppress the rebellion through Proclamation No. 38 and
NOW, THEREFORE, I, GLORIA MACAPAGAL-ARROYO, General Order No. 1. On that occasion, an angry and violent
President of the Philippines, by virtue of the powers vested in mob armed with explosives, firearms, bladed weapons, clubs,
me by law, hereby declare that the state of rebellion has stones and other deadly weapons assaulted and attempted to
ceased to exist. break into Malacaang.[20] Petitions were filed before this Court
assailing the validity of the Presidents declaration. Five days
In the interim, several petitions were filed before this after such declaration, however, the President lifted the
Court challenging the validity of Proclamation No. 427 and same. The mootness of the petitions in Lacson v. Perez and
General Order No. 4. accompanying cases[21] precluded this Court from addressing
the constitutionality of the declaration.
In G.R. No. 159085 (Sanlakas and PM v. Executive
Secretary, et al.),[2] party-list organizations Sanlakas and To prevent similar questions from reemerging, we seize
Partido ng Manggagawa (PM), contend that Section 18, Article this opportunity to finally lay to rest the validity of the
VII of the Constitution does not require the declaration of a declaration of a state of rebellion in the exercise of the
state of rebellion to call out the armed forces. [3] They further Presidents calling out power, the mootness of the petitions
submit that, because of the cessation of the Oakwood notwithstanding.
occupation, there exists no sufficient factual basis for the
proclamation by the President of a state of rebellion for an Only petitioners Rep. Suplico et al. and Sen. Pimentel,
indefinite period.[4] as Members of Congress, have standing to challenge the
subject issuances. In Philippine Constitution Association v.
Petitioners in G.R. No. 159103 (SJS Officers/Members v. Enriquez, [22]this Court recognized that:
Hon. Executive Secretary, et al.) are officers/members of the
Social Justice Society (SJS), Filipino citizens, taxpayers, law To the extent the powers of Congress are impaired, so is the
professors and bar reviewers.[5] Like Sanlakas and PM, they power of each member thereof, since his office confers a right
claim that Section 18, Article VII of the Constitution does not to participate in the exercise of the powers of that institution.
authorize the declaration of a state of rebellion. [6] They contend
that the declaration is a constitutional anomaly that confuses,
confounds and misleads because [o]verzealous public officers, An act of the Executive which injures the institution of
acting pursuant to such proclamation or general order, are Congress causes a derivative but nonetheless substantial
liable to violate the constitutional right of private injury, which can be questioned by a member of Congress. In
citizens.[7] Petitioners also submit that the proclamation is a such a case, any member of Congress can have a resort to the
circumvention of the report requirement under the same courts.
Section 18, Article VII, commanding the President to submit a
report to Congress within 48 hours from the proclamation of Petitioner Members of Congress claim that the declaration of a
martial law.[8] Finally, they contend that the presidential state of rebellion by the President is tantamount to an exercise
issuances cannot be construed as an exercise of emergency of Congress emergency powers, thus impairing the lawmakers
powers as Congress has not delegated any such power to the legislative powers. Petitioners also maintain that the
President.[9] declaration is a subterfuge to avoid congressional scrutiny into
the Presidents exercise of martial law powers.
In G.R. No. 159185 (Rep. Suplico et al. v. President
Macapagal-Arroyo and Executive Secretary Romulo),
petitioners brought suit as citizens and as Members of the
FINALS CONSTITUTION I ACJUCO 199

Petitioners Sanlakas and PM, and SJS economic decision-making shall not be abridged. (Art. XIII, 15-
Officers/Members, have no legal standing or locus standi to 16)
bring suit. Legal standing or locus standi has been defined as
a personal and substantial interest in the case such that the
These provisions have not changed the traditional rule that
party has sustained or will sustain direct injury as a result of only real parties in interest or those with standing, as the case
the governmental act that is being challenged. The gist of the may be, may invoke the judicial power. The jurisdiction of this
question of standing is whether a party alleges such personal Court, even in cases involving constitutional questions, is
stake in the outcome of the controversy as to assure that limited by the case and controversy requirement of Art. VIII,
concrete adverseness which sharpens the presentation of 5. This requirement lies at the very heart of the judicial
issues upon which the court depends for illumination of difficult function. It is what differentiates decisionmaking in the courts
constitutional questions.[23] from decisionmaking in the political departments of the
Petitioners Sanlakas and PM assert that: government and bars the bringing of suits by just any party. [27]

2. As a basic principle of the organizations and as an important That petitioner SJS officers/members are taxpayers and
plank in their programs, petitioners are committed to assert, citizens does not necessarily endow them with standing. A
defend, protect, uphold, and promote the rights, interests, and taxpayer may bring suit where the act complained of directly
welfare of the people, especially the poor and marginalized involves the illegal disbursement of public funds derived from
classes and sectors of Philippine society. Petitioners are taxation.[28] No such illegal disbursement is alleged.
committed to defend and assert human rights, including
On the other hand, a citizen will be allowed to raise a
political and civil rights, of the citizens. constitutional question only when he can show that he has
personally suffered some actual or threatened injury as a result
3. Members of the petitioner organizations resort to mass of the allegedly illegal conduct of the government; the injury is
actions and mobilizations in the exercise of their Constitutional fairly traceable to the challenged action; and the injury is likely
rights to peaceably assemble and their freedom of speech and to be redressed by a favorable action.[29] Again, no such injury
of expression under Section 4, Article III of the 1987 is alleged in this case.
Constitution, as a vehicle to publicly ventilate their grievances
and legitimate demands and to mobilize public opinion to Even granting these petitioners have standing on the
support the same.[24] [Emphasis in the original.] ground that the issues they raise are of transcendental
importance, the petitions must fail.
Petitioner party-list organizations claim no better right It is true that for the purpose of exercising the calling out
than the Laban ng Demokratikong Pilipino, whose standing this power the Constitution does not require the President to make
Court rejected in Lacson v. Perez: a declaration of a state of rebellion. Section 18, Article VII
provides:
petitioner has not demonstrated any injury to itself which would
justify the resort to the Court. Petitioner is a juridical person not Sec. 18. The President shall be the Commander-in-Chief of all
subject to arrest. Thus, it cannot claim to be threatened by a armed forces of the Philippines and whenever it becomes
warrantless arrest.Nor is it alleged that its leaders, members, necessary, he may call out such armed forces to prevent
and supporters are being threatened with warrantless arrest or suppress lawless violence, invasion or rebellion. In
and detention for the crime of rebellion. Every action must be case of invasion or rebellion, when the public safety requires
brought in the name of the party whose legal rights has been it, he may, for a period not exceeding sixty days, suspend the
invaded or infringed, or whose legal right is under imminent privilege of the writ of habeas corpus or place the Philippines
threat of invasion or infringement. or any part thereof under martial law. Within forty-eight hours
from the proclamation of martial law or the suspension of the
At best, the instant petition may be considered as an action for writ of habeas corpus, the President shall submit a report in
declaratory relief, petitioner claiming that it[]s right to freedom person or in writing to the Congress. The Congress, voting
of expression and freedom of assembly is affected by the jointly, by a vote of at least a majority of all its Members in
declaration of a state of rebellion and that said proclamation is regular or special session, may revoke such proclamation or
invalid for being contrary to the Constitution. suspension, which revocation shall not be set aside by the
President. Upon the initiative of the President, the Congress
may, in the same manner, extend such proclamation or
However, to consider the petition as one for declaratory relief suspension for a period to be determined by the Congress, if
affords little comfort to petitioner, this Court not having the invasion or rebellion shall persist and public safety requires
jurisdiction in the first instance over such a petition. Section 5 it.
[1], Article VIII of the Constitution limits the original jurisdiction
of the court to cases affecting ambassadors, other public
ministers and consuls, and over petitions for certiorari, The Congress, if not in session, shall, within twenty-four hours
prohibition, mandamus, quo warranto, and habeas corpus.[25] following such proclamation or suspension, convene in
accordance with its rules without need of a call.

Even assuming that petitioners are peoples


organizations, this status would not vest them with the requisite The Supreme Court may review, in an appropriate proceeding
personality to question the validity of the presidential filed by any citizen, the sufficiency of the factual basis for the
issuances, as this Court made clear in Kilosbayan v. proclamation of martial law or the suspension of the privilege
Morato: [26] of the writ of habeas corpus or the extension thereof, and must
promulgate its decision thereon within thirty days from its filing.

The Constitution provides that the State shall respect the role
of independent peoples organizations to enable the people to A state of martial law does not suspend the operation of the
pursue and protect, within the democratic framework, their Constitution, nor supplant the functioning of the civil courts or
legitimate and collective interests and aspirations through legislative assemblies, nor authorize the conferment of the
peaceful and lawful means, that their right to effective and jurisdiction on military courts and agencies over civilians where
reasonable participation at all levels of social, political, and civil courts are able to function, nor automatically suspend the
privilege of the writ.
FINALS CONSTITUTION I ACJUCO 200

The suspension of the privilege of the writ shall apply only to indissolubility of the Union by defeating South Carolinas
persons judicially charged for rebellion or offenses inherent in nullification effort.[34]
or directly connected with invasion.
The Federal Tariff Acts of 1828 and 1832 that Congress
enacted did not pacify the hotspurs from South Carolina. Its
During the suspension of the privilege of the writ, any person State Legislature ordered an election for a convention, whose
thus arrested or detained shall be judicially charged within members quickly passed an Ordinance of Nullification. The
three days, otherwise he shall be released. [Emphasis Ordinance declared the Tariff Acts unconstitutional, prohibited
supplied.] South Carolina citizens from obeying them after a certain date
in 1833, and threatened secession if the Federal Government
The above provision grants the President, as sought to oppose the tariff laws. The Legislature then
Commander-in-Chief, a sequence of graduated implemented the Ordinance with bristling punitive laws aimed
power[s].[30] From the most to the least benign, these are: the at any who sought to pay or collect customs duties. [35]
calling out power, the power to suspend the privilege of the writ
of habeas corpus, and the power to declare martial law. In the Jackson bided his time. His task of enforcement would
exercise of the latter two powers, the Constitution requires the not be easy. Technically, the President might send troops into
concurrence of two conditions, namely, an actual invasion or a State only if the Governor called for help to suppress an
rebellion, and that public safety requires the exercise of such insurrection, which would not occur in the instance. The
power.[31] However, as we observed in Integrated Bar of the President could also send troops to see to it that the laws
Philippines v. Zamora,[32][t]hese conditions are not required in enacted by Congress were faithfully executed. But these laws
the exercise of the calling out power. The only criterion is that were aimed at individual citizens, and provided no enforcement
whenever it becomes necessary, the President may call the machinery against violation by a State. Jackson prepared to
armed forces to prevent or suppress lawless violence, invasion ask Congress for a force bill.[36]
or rebellion. In a letter to a friend, the President gave the essence of
Nevertheless, it is equally true that Section 18, Article VII his position. He wrote: . . . when a faction in a State attempts
does not expressly prohibit the President from declaring a to nullify a constitutional law of Congress, or to destroy the
state of rebellion. Note that the Constitution vests the Union, the balance of the people composing this Union have a
President not only with Commander-in-Chief powers but, first perfect right to coerce them to obedience. Then in a
and foremost, with Executive powers. Proclamation he issued on December 10, 1832, he called upon
South Carolinians to realize that there could be no peaceable
Section 1, Article VII of the 1987 Philippine Constitution interference with the execution of the laws, and dared them,
states: The executive power shall be vested in the President. disunion by armed force is treason. Are you ready to incur its
As if by exposition, Section 17 of the same Article provides: He guilt? [37]
shall ensure that the laws be faithfully executed. The provisions
trace their history to the Constitution of the United States. The Proclamation frightened nullifiers, non-nullifiers and
tight-rope walkers. Soon, State Legislatures began to adopt
The specific provisions of the U.S. Constitution granting resolutions of agreement, and the President announced that
the U.S. President executive and commander-in-chief powers the national voice from Maine on the north to Louisiana on the
have remained in their original simple form since the south had declared nullification and accession confined to
Philadelphia Constitution of 1776, Article II of which states in contempt and infamy.[38]
part:
No other President entered office faced with problems so
formidable, and enfeebled by personal and political handicaps
Section 1. 1. The Executive Power shall be vested in a so daunting, as Abraham Lincoln.
President of the United States of America . . . .
Lincoln believed the Presidents power broad and that of
Congress explicit and restricted, and sought some source of
....
executive power not failed by misuse or wrecked by
sabotage. He seized upon the Presidents designation by the
Section 2. 1. The President shall be Commander in Chief of Constitution as Commander-in-Chief, coupled it to the
the Army and Navy of the United States. . . . executive power provision and joined them as the war power
which authorized him to do many things beyond the
.... competence of Congress.[39]
Lincoln embraced the Jackson concept of the Presidents
Section 3. he shall take care that the laws be faithfully independent power and duty under his oath directly to
executed. [Article II Executive Power] represent and protect the people. In his Message of July 4,
1861, Lincoln declared that the Executive found the duty of
employing the war power in defense of the government forced
Recalling in historical vignettes the use by the U.S.
upon him. He could not but perform the duty or surrender the
President of the above-quoted provisions, as juxtaposed
existence of the Government . . . . This concept began as a
against the corresponding action of the U.S. Supreme Court,
transition device, to be validated by Congress when it
is instructive. Clad with the prerogatives of the office and
assembled. In less than two-years, it grew into an independent
endowed with sovereign powers, which are drawn chiefly from
power under which he felt authorized to suspend the privilege
the Executive Power and Commander-in-Chief provisions, as
of the writ of habeas corpus, issue the Emancipation
well as the presidential oath of office, the President serves as
Proclamation, and restore reoccupied States. [40]
Chief of State or Chief of Government, Commander-in-Chief,
Chief of Foreign Relations and Chief of Public Opinion. [33] Lincolns Proclamation of April 15, 1861, called for 75,000
troops. Their first service, according to the proclamation, would
First to find definitive new piers for the authority of the
be to recapture forts, places and property, taking care to avoid
Chief of State, as the protector of the people, was President
any devastation, any destruction of or interference with
Andrew Jackson. Coming to office by virtue of a political
property, or any disturbance of peaceful citizens. [41]
revolution, Jackson, as President not only kept faith with the
people by driving the patricians from power. Old Hickory, as he Early in 1863, the U.S. Supreme Court approved
was fondly called, was the first President to champion the President Lincolns report to use the war powers without the
FINALS CONSTITUTION I ACJUCO 201

benefit of Congress. The decision was handed in the The lesson to be learned from the U.S. constitutional
celebrated Prize Cases[42] which involved suits attacking the history is that the Commander-in-Chief powers are broad
Presidents right to legally institute a blockade. Although his enough as it is and become more so when taken together with
Proclamation was subsequently validated by Congress, the the provision on executive power and the presidential oath of
claimants contended that under international law, a blockade office. Thus, the plenitude of the powers of the presidency
could be instituted only as a measure of war under the equips the occupant with the means to address exigencies or
sovereign power of the State. Since under the Constitution threats which undermine the very existence of government or
only Congress is exclusively empowered to declare war, it is the integrity of the State.
only that body that could impose a blockade and all prizes
seized before the legislative declaration were illegal. By a 5 to In The Philippine Presidency A Study of Executive
4 vote, the Supreme Court upheld Lincolns right to act as he Power, the late Mme. Justice Irene R. Cortes, proposed that
had.[43] the Philippine President was vested with residual power and
that this is even greater than that of the U.S. President. She
In the course of time, the U.S. Presidents power to call attributed this distinction to the unitary and highly centralized
out armed forces and suspend the privilege of the writ nature of the Philippine government. She noted that, There is
of habeas corpus without prior legislative approval, in case of no counterpart of the several states of the American union
invasion, insurrection, or rebellion came to be recognized and which have reserved powers under the United States
accepted. The United States introduced the expanded constitution. Elaborating on the constitutional basis for her
presidential powers in the Philippines through the Philippine argument, she wrote:
Bill of 1902.[44] The use of the power was put to judicial test and
this Court held that the case raised a political question and said . The [1935] Philippine [C]onstitution establishes the three
that it is beyond its province to inquire into the exercise of the departments of the government in this manner: The legislative
power.[45] Later, the grant of the power was incorporated in the
power shall be vested in a Congress of the Philippines which
1935 Constitution.[46] shall consist of a Senate and a House of Representatives. The
Elected in 1884, Grover Cleveland took his ascent to the executive power shall be vested in a President of the
presidency to mean that it made him the trustee of all the Philippines. The judicial powers shall be vested in one
people. Guided by the maxim that Public office is a public trust, Supreme Court and in such inferior courts as may be provided
which he practiced during his incumbency, Cleveland sent by law. These provisions not only establish a separation of
federal troops to Illinois to quell striking railway workers who powers by actual division but also confer plenary legislative,
defied a court injunction. The injunction banned all picketing executive, and judicial powers. For as the Supreme Court of
and distribution of handbills. For leading the strikes and the Philippines pointed out in Ocampo v. Cabangis, a grant of
violating the injunction, Debs, who was the union president, legislative power means a grant of all the legislative power; and
was convicted of contempt of court. Brought to the Supreme a grant of the judicial power means a grant of all the judicial
Court, the principal issue was by what authority of the power which may be exercised under the government. If this is
Constitution or statute had the President to send troops without true of the legislative power which is exercised by two
the request of the Governor of the State.[47] chambers with a combined membership [at that time] of more
than 120 and of the judicial power which is vested in a
In In Re: Eugene Debs, et al,[48] the Supreme Court hierarchy of courts, it can equally if not more appropriately
upheld the contempt conviction. It ruled that it is not the apply to the executive power which is vested in one official the
governments province to mix in merely individual present president. He personifies the executive branch. There is a
controversies. Still, so it went on, whenever wrongs unity in the executive branch absent from the two other
complained of are such as affect the public at large, and are in branches of government. The president is not the chief of many
respect of matters which by the Constitution are entrusted to executives. He is the executive. His direction of the executive
the care of the Nation and concerning which the Nation owes branch can be more immediate and direct than the United
the duty to all citizens of securing to them their common rights, States president because he is given by express provision of
then the mere fact that the Government has no pecuniary the constitution control over all executive departments,
interest in the controversy is not sufficient to exclude it from the bureaus and offices.[55]
Courts, or prevent it from taking measures therein to fully
discharge those constitutional duties. [49] Thus, Clevelands The esteemed Justice conducted her study against the
course had the Courts attest. backdrop of the 1935 Constitution, the framers of which, early
Taking off from President Cleveland, President Theodore on, arrived at a general opinion in favor of a strong Executive
Roosevelt launched what political scientists dub the in the Philippines.[56] Since then, reeling from the aftermath of
stewardship theory. Calling himself the steward of the people, martial law, our most recent Charter has restricted the
he felt that the executive power was limited only by the specific Presidents powers as Commander-in-Chief. The same,
restrictions and prohibitions appearing in the Constitution, or however, cannot be said of the Presidents powers as Chief
impleaded by Congress under its constitutional powers.[50] Executive.

The most far-reaching extension of presidential power In her ponencia in Marcos v. Manglapus, Justice Cortes
T.R. ever undertook to employ was his plan to occupy and put her thesis into jurisprudence. There, the Court, by a slim 8-
operate Pennsylvanias coal mines under his authority as 7 margin, upheld the Presidents power to forbid the return of
Commander-in-Chief. In the issue, he found means other than her exiled predecessor. The rationale for the majoritys ruling
force to end the 1902 hard-coal strike, but he had made rested on the Presidents
detailed plans to use his power as Commander-in-Chief to
wrest the mines from the stubborn operators, so that coal unstated residual powers which are implied from the grant of
production would begin again.[51] executive power and which are necessary for her to comply
with her duties under the Constitution. The powers of the
Eventually, the power of the State to intervene in and President are not limited to what are expressly enumerated in
even take over the operation of vital utilities in the public the article on the Executive Department and in scattered
interest was accepted. In the Philippines, this led to the provisions of the Constitution. This is so, notwithstanding the
incorporation of Section 6,[52] Article XIII of the 1935 avowed intent of the members of the Constitutional
Constitution, which was later carried over with modifications in Commission of 1986 to limit the powers of the President as a
Section 7,[53] Article XIV of the 1973 Constitution, and reaction to the abuses under the regime of Mr. Marcos, for the
thereafter in Section 18,[54] Article XII of the 1987 Constitution. result was a limitation of specific powers of the President,
FINALS CONSTITUTION I ACJUCO 202

particularly those relating to the commander-in-chief clause, rights. There is no indication that the President has exercised
but not a diminution of the general grant of executive judicial and legislative powers. In short, there is no illustration
power.[57] [Underscoring supplied. Italics in the original.] that the President has attempted to exercise or has exercised
martial law powers.
Thus, the Presidents authority to declare a state of Nor by any stretch of the imagination can the declaration
rebellion springs in the main from her powers as chief constitute an indirect exercise of emergency powers, which
executive and, at the same time, draws strength from her exercise depends upon a grant of Congress pursuant to
Commander-in-Chief powers. Indeed, as the Solicitor General Section 23 (2), Article VI of the Constitution:
accurately points out, statutory authority for such a declaration
may be found in Section 4, Chapter 2 (Ordinance Power), Book
III (Office of the President) of the Revised Administrative Code Sec. 23. (1) .
of 1987, which states:
(2) In times of war or other national emergency, the Congress
SEC. 4. Proclamations. Acts of the President fixing a date or may, by law, authorize the President, for a limited period and
declaring a status or condition of public moment or subject to such restrictions as it may prescribe, to exercise
interest, upon the existence of which the operation of a powers necessary and proper to carry out a declared national
specific law or regulation is made to depend, shall be policy. Unless sooner withdrawn by resolution of the Congress,
promulgated in proclamations which shall have the force of an such powers shall cease upon the next adjournment thereof.
executive order. [Emphasis supplied.]
The petitions do not cite a specific instance where the
The foregoing discussion notwithstanding, in calling out President has attempted to or has exercised powers beyond
the armed forces, a declaration of a state of rebellion is an utter her powers as Chief Executive or as Commander-in-Chief. The
superfluity.[58] At most, it only gives notice to the nation that President, in declaring a state of rebellion and in calling out the
such a state exists and that the armed forces may be called to armed forces, was merely exercising a wedding of her Chief
prevent or suppress it.[59] Perhaps the declaration may wreak Executive and Commander-in-Chief powers. These
emotional effects upon the perceived enemies of the State, are purely executive powers, vested on the President by
even on the entire nation. But this Courts mandate is to probe Sections 1 and 18, Article VII, as opposed to the delegated
only into the legal consequences of the declaration. This Court legislative powers contemplated by Section 23 (2), Article VI.
finds that such a declaration is devoid of any legal
WHEREFORE, the petitions are hereby DISMISSED.
significance. For all legal intents, the declaration is deemed not
SO ORDERED.
written.
Should there be any confusion generated by the
issuance of Proclamation No. 427 and General Order No. 4,
we clarify that, as the dissenters in Lacson correctly pointed
out, the mere declaration of a state of rebellion cannot diminish
or violate constitutionally protected rights. [60] Indeed, if a state
of martial law does not suspend the operation of the
Constitution or automatically suspend the privilege of the writ
of habeas corpus,[61] then it is with more reason that a simple
declaration of a state of rebellion could not bring about these
conditions.[62] At any rate, the presidential issuances
themselves call for the suppression of the rebellion with due
regard to constitutional rights.
For the same reasons, apprehensions that the military
and police authorities may resort to warrantless arrests are
likewise unfounded. In Lacson vs. Perez, supra, majority of the
Court held that [i]n quelling or suppressing the rebellion, the
authorities may only resort to warrantless arrests of persons
suspected of rebellion, as provided under Section 5, Rule 113
of the Rules of Court,[63] if the circumstances so warrant. The
warrantless arrest feared by petitioners is, thus, not based on
the declaration of a state of rebellion. [64] In other words, a
person may be subjected to a warrantless arrest for the crime
of rebellion whether or not the President has declared a state
of rebellion, so long as the requisites for a valid warrantless
arrest are present.

It is not disputed that the President has full discretionary


power to call out the armed forces and to determine the
necessity for the exercise of such power. While the Court may
examine whether the power was exercised within constitutional
limits or in a manner constituting grave abuse of discretion,
none of the petitioners here have, by way of proof, supported
their assertion that the President acted without factual basis.[65]
The argument that the declaration of a state of rebellion
amounts to a declaration of martial law and, therefore, is a
circumvention of the report requirement, is a leap of
logic. There is no indication that military tribunals have
replaced civil courts in the theater of war or that military
authorities have taken over the functions of civil government.
There is no allegation of curtailment of civil or political
FINALS CONSTITUTION I ACJUCO 203

G.R. No. L-35546 September 17, 1974 IN THE MATTER OF THE PETITION FOR
HABEAS CORPUS OF AMANDO DORONILA JUAN L.
IN THE MATTER OF THE PETITION FOR MERCADO, HERNANDO L. ABAYA, ERNESTO GRANADA,
LUIS D. BELTRAN, TAN CHIN HIAN, BREN GUIAO, RUBEN
HABEAS CORPUS OF BENIGNO S. AQUINO, JR., RAMON
MITRA, JR., FRANCISCO RODRIGO, AND NAPOLEON CUSIPAG, ROBERTO ORDOÑEZ, MANUEL ALMARIO AND
RAMA, petitioners, WILLIE BAUN, petitioners,
vs. vs.
HON JUAN PONCE ENRILE, SECRETARY OF NATIONAL HON. JUAN PONCE ENRILE, SECRETARY OF NATIONAL
DEFENSE; GEN. ROMEO ESPINO, CHIEF OF STAFF, DEFENSE; LIEUT. GEN. ROMEO ESPINO, CHIEF OF
ARMED FORCES OF THE PHILIPPINES; AND GEN. FIDEL STAFF, ARMED FORCES OF THE PHILIPPINES; AND
V. RAMOS, CHIEF, PHILIPPINE BRIG. GEN. FIDEL V. RAMOS, CHIEF, PHILIPPINE
CONSTABULARY, respondents. CONSTABULARY, respondents.

G.R. No. L-35538 September 17, 1974 G.R. No. L-35571 September 17, 1974. *3

IN THE MATTER OF THE PETITION FOR IN THE MATTER OF THE PETITION FOR
HABEAS CORPUS OF JOAQUIN P. ROCES, TEODORO M. HABEAS CORPUS OF BREN Z. GUIAO, TERESITA M.
LOCSIN, SR., ROLANDO FADUL, ROSALINA GALANG, GO GUIAO, petitioner,
ENG GUAN, MAXIMO V. SOLIVEN, RENATO vs.
CONSTANTINO, AND LUIS R. MAURICIO, petitioners, JUAN PONCE ENRILE, THE SECRETARY OF NATIONAL
DEFENSE; LT. GEN. ROMEO ESPINO, CHIEF OF STAFF
vs.
THE SECRETARY OF NATIONAL DEFENSE; THE CHIEF OF THE ARMED FORCES OF THE PHILIPPINES: AND
BRIG. GEN. FIDEL V. RAMOS, CHIEF OF THE PHILIPPINE
OF STAFF, ARMED FORCES OF THE PHILIPPINES; THE
CHIEF, PHILIPPINE CONSTABULARY, et al., respondents. CONSTABULARY, respondents.

G.R. No. L-35539 September 17, 1974 G.R. No. L-35573 September 17, 1974

IN THE MATTER OF THE PETITION FOR ERNESTO RONDON, petitioner,


HABEAS CORPUS OF JOSE W. DIOKNO, CARMEN I. vs.
DIOKNO, *1 petitioner, HON. JUAN PONCE ENRILE, SECRETARY OF NATIONAL
DEFENSE; GEN. FIDEL V. RAMOS, CHIEF, PHILIPPINE
vs.
JUAN PONCE ENRILE, THE SECRETARY OF NATIONAL CONSTABULARY; AND MAJOR RODULFO
MIANA, respondents.
DEFENSE; ROMEO ESPINO, THE CHIEF OF STAFF,
ARMED FORCES OF THE PHILIPPINES. respondents.

G.R. No. L-35540 September 17, 1974


MAKALINTAL, C.J.:p
MAXIMO V. SOLIVEN, NAPOLEON G. RAMA, AND JOSE
MARI VELEZ, petitioners, These cases are all petitions for habeas corpus, the petitioners
vs. having been arrested and detained by the military by virtue of
HON. JUAN PONCE ENRILE, SECRETARY OF NATIONAL the President's Proclamation No. 1081, dated September 21,
DEFENSE; HON. FRANCISCO TATAD, PRESS 1972.
SECRETARY; AND GEN. FIDEL V. RAMOS, CHIEF,
PHILIPPINE CONSTABULARY, respondents. At the outset a word of clarification is in order. This is not the
decision of the Court in the sense that a decision represents a
G.R. No. L-35547 September 17, 1974 *2 consensus of the required majority of its members not only on
the judgment itself but also on the rationalization of the issues
ENRIQUE VOLTAIRE GARCIA II, petitioner, and the conclusions arrived at. On the final result the vote is
practically unanimous; this is a statement of my individual
vs.
BRIG. GEN. FIDEL RAMOS, CHIEF, PHILIPPINE opinion as well as a summary of the voting on the major issues.
Why no particular Justice has been designated to write just one
CONSTABULARY; GEN. ROMEO ESPINO, CHIEF OF
STAFF, ARMED FORCES OF THE PHILIPPINES; AND opinion for the entire Court will presently be explained.
HON. JUAN PONCE ENRILE, SECRETARY OF NATIONAL
DEFENSE, respondents. At one point during our deliberations on these cases it was
suggested that as Chief Justice I should write that opinion. The
G.R. No. L-35556 September 17, 1974 impracticability of the suggestion shortly became apparent for
a number of reasons, only two of which need be mentioned.
First, the discussions, as they began to touch on particular
IN THE MATTER OF THE PETITION FOR issues, revealed a lack of agreement among the Justices as to
HABEAS CORPUS OF VERONICA L. YUYITUNG AND TAN whether some of those issues should be taken up although it
CHIN HIAN, petitioners, was not necessary to do so, they being merely convenient for
vs. the purpose of ventilating vexing questions of public interest,
JUAN PONCE ENRILE, SECRETARY OF NATIONAL or whether the decision should be limited to those issues which
DEFENSE; LIEUT. GEN. ROMEO ESPINO, CHIEF OF are really material and decisive in these cases. Similarly, there
STAFF, ARMED FORCES OF THE PHILIPPINES; AND was no agreement as to the manner the issues should be
BRIG. GEN. FIDEL V. RAMOS, CHIEF OF THE PHILIPPINE treated and developed. The same destination would be
CONSTABULARY, respondents. reached, so to speak, but through different routes and by
means of different vehicles of approach. The writing of
G.R. No. L-35567 September 17, 1974 separate opinions by individual Justices was thus unavoidable,
and understandably so for still another reason, namely, that
FINALS CONSTITUTION I ACJUCO 204

although little overt reference to it was made at the time, the designed to make political capital of his personal situation, as
future verdict of history was very much a factor in the thinking the publicity given to them by some segments of the foreign
of the members, no other case of such transcendental press and by local underground propaganda news sheets
significance to the life of the nation having before confronted subsequently confirmed. It was in fact from that perspective
this Court. Second — and this to me was the insuperable that I deemed it proper to respond in kind, that is, from a non-
obstacle — I was and am of the opinion, which was shared by judicial forum, in an address I delivered on February 19, 1974
six other Justices1 at the time the question was voted upon, before the LAWASIA, the Philippine Bar Association and the
that petitioner Jose W. Diokno's motion of December 28, 1973 Philippine Lawyers' Association. Justice Teehankee, it may be
to withdraw his petition (G.R. No. L-35539) should be granted, stated, is of the opinion that a simple majority of seven votes
and therefore I was in no position to set down the ruling of the out of twelve is legally sufficient to make the withdrawal of
Court on each of the arguments raised by him, except Diokno's petition effective, on the theory that the requirement
indirectly, insofar as they had been raised likewise in the other of a majority of eight votes applies only to a decision on the
cases. merits.

It should be explained at this point that when the Court voted In any event, as it turned out, after petitioner Diokno was
on Diokno's motion to withdraw his petition he was still under released by the President on September 11 all the members
detention without charges, and continued to remain so up to of this Court except Justice Castro were agreed that his petition
the time the separate opinions of the individual Justices were had become moot and therefore should no longer be
put in final form preparatory to their promulgation on considered on the merits. This notwithstanding, some of the
September 12, which was the last day of Justice Zaldivars opinions of the individual members, particularly Justices
tenure in the Court.2 Before they could be promulgated, Castro and Teehankee, should be taken in the time setting in
however, a major development supervened: petitioner Diokno which they were prepared, that is, before the order for the
was released by the President in the morning of September 11, release of Diokno was issued.
1974. In view thereof all the members of this Court except
Justice Castro agreed to dismiss Diokno's petition on the The Cases.
ground that it had become moot, with those who originally
voted to grant the motion for withdrawal citing said motion as
an additional ground for such dismissal. The events which form the background of these nine petitions
are related, either briefly or in great detail, in the separate
opinions filed by the individual Justices. The petitioners were
The petitioners in the other cases, except Benigno Aquino, Jr. arrested and held pursuant to General Order No. 2 of the
(G.R. No. L-35546), either have been permitted to withdraw President (September 22, 1972), "for being participants or for
their petitions or have been released from detention subject to
having given aid and comfort in the conspiracy to seize political
certain restrictions.3 In the case of Aquino, formal charges of and state power in the country and to take over the
murder, subversion and illegal possession of firearms were
Government by force ..."
lodged against him with a Military Commission on August 11,
1973; and on the following August 23 he challenged the
jurisdiction of said Commission as well as his continued General Order No. 2 was issued by the President in the
detention by virtue of those charges in a petition exercise of the powers he assumed by virtue of Proclamation
for certiorari and prohibition filed in this Court (G.R. No. No. 1081 (September 21, 1972) placing the entire country
L-37364). The question came up as to whether or not Aquino's under martial law. The portions of the proclamation
petition for habeas corpus should be dismissed on the ground immediately in point read as follows:
that the case as to him should more appropriately be resolved
in this new petition. Of the twelve Justices, however, eight xxx xxx xxx
voted against such dismissal and chose to consider the case
on the merits.4
NOW, THEREFORE, I, FERDINAND E.
MARCOS, President of the Philippines by
On Diokno's motion to withdraw his petition I voted in favor of virtue of the powers vested upon me by
granting it for two reasons. In the first place such withdrawal Article VII, Section 10, Paragraph (2) of the
would not emasculate the decisive and fundamental issues of Constitution, do hereby place the entire
public interest that demanded to be resolved, for they were Philippines as defined in Article I, Section 1
also raised in the other cases which still remained pending. of the Constitution under martial law and, in
Secondly, since it was this petitioner's personal liberty that was my capacity as their Commander-in-Chief,
at stake, I believed he had the right to renounce the application do hereby command the Armed Forces of
for habeas corpus he initiated. Even if that right were not the Philippines, to maintain law and order
absolute I still would respect his choice to remove the case throughout the Philippines, prevent or
from this Court's cognizance, regardless of the fact that I suppress all forms of lawless violence as
disagreed with many of his reasons for so doing. I could not well as any act of insurrection or rebellion
escape a sense of irony in this Court's turning down the plea and to enforce obedience to all the laws
to withdraw on the ground, so he alleges among others, that and decrees, orders and regulations
this is no longer the Court to which he originally applied for promulgated by me personally or upon my
relief because its members have taken new oaths of office direction.
under the 1973 Constitution, and then ruling adversely to him
on the merits of his petition.
In addition, I do hereby order that all
persons presently detained, as well as all
It is true that some of the statements in the motion are an others who may hereafter be similarly
affront to the dignity of this Court and therefore should not be detained for the crimes of insurrection or
allowed to pass unanswered. Any answer, however, would not rebellion, and all other crimes and offenses
be foreclosed by allowing the withdrawal. For my part, since committed in furtherance or on the
most of those statements are of a subjective character, being occasion thereof, or incident thereto, or in
matters of personal belief and opinion, I see no point in refuting connection therewith, for crimes against
them in these cases. Indeed my impression is that they were national security and the law of nations,
beamed less at this Court than at the world outside and crimes against public order, crimes
FINALS CONSTITUTION I ACJUCO 205

involving usurpation of authority, rank, title Justice Antonio, with whom Justices Makasiar, Fernandez and
and improper use of names, uniforms and Aquino concur, finds that there is no dispute as to the existence
insignia, crimes committed by public of a state of rebellion in the country, and on that premise
officers, and for such other crimes as will emphasizes the factor of necessity for the exercise by the
be enumerated in orders that I shall President of his power under the Constitution to declare martial
subsequently promulgate, as well as law, holding that the decision as to whether or not there is
crimes as a consequence of any violation such necessity is wholly confided to him and therefore is not
of any decree, order or regulation subject to judicial inquiry, his responsibility being directly to the
promulgated by me personally or people.
promulgated upon my direction shall be
kept under detention until otherwise Arrayed on the side of justiciability are Justices Castro,
ordered released by me or by my duly Fernando, Teehankee and Muñoz Palma. They hold that the
designated representative.
constitutional sufficiency of the proclamation may be inquired
into by the Court, and would thus apply the principle laid down
The provision of the 1935 Constitution referred to in the in Lansang although that case refers to the power of the
proclamation reads: "the President shall be commander-in- President to suspend the privilege of the writ of habeas corpus.
chief of all armed forces of the Philippines and, whenever it The recognition of justiciability accorded to the question in
becomes necessary, he may call out such armed forces to Lansang, it should be emphasized, is there expressly
prevent or suppress lawless violence, invasion, insurrection, or distinguished from the power of judicial review in ordinary civil
rebellion. In case of invasion, insurrection, or rebellion, or or criminal cases, and is limited to ascertaining "merely
imminent danger thereof, when the public safety requires it, he whether he (the President) has gone beyond the constitutional
may suspend the privilege of the writ of habeas corpus, or limits of his jurisdiction, not to exercise the power vested in him
place the Philippines or any part thereof under martial law." or to determine the wisdom of his act." The test is not whether
the President's decision is correct but whether, in suspending
1. The first major issue raised by the parties is whether this the writ, he did or did not act arbitrarily. Applying this test, the
Court may inquire into the validity of Proclamation No. 1081. finding by the Justices just mentioned is that there was no
Stated more concretely, is the existence of conditions claimed arbitrariness in the President's proclamation of martial law
pursuant to the 1935 Constitution; and I concur with them in
to justify the exercise of the power to declare martial law
subject to judicial inquiry? Is the question political or justiciable that finding. The factual bases for the suspension of the
privilege of the writ of habeas corpus, particularly in regard to
in character?
the existence of a state of rebellion in the country, had not
disappeared, indeed had been exacerbated, as events shortly
Justices Makasiar, Antonio, Esguerra, Fernandez and Aquino before said proclamation clearly demonstrated. On this Point
hold that the question is political and therefore its determination the Court is practically unanimous; Justice Teehankee merely
is beyond the jurisdiction of this Court. The reasons are given refrained from discussing it.
at length in the separate opinions they have respectively
signed. Justice Fernandez adds that as a member of the
Convention that drafted the 1973 Constitution he believes that Insofar as my own opinion is concerned the cleavage in the
"the Convention put an imprimatur on the proposition that the Court on the issue of justiciability is of not much more than
validity of a martial law proclamation and its continuation is academic interest for purposes of arriving at a judgment. I am
political and non-justiciable in character." not unduly exercised by Americas decisions on the subject
written in another age and political clime, or by theories of
foreign authors in political science. The present state of martial
Justice Barredo, on the other hand, believes that political law in the Philippines is peculiarly Filipino and fits into no
questions are not per se beyond the Court's jurisdiction, the traditional patterns or judicial precedents.
judicial power vested in it by the Constitution being plenary and
all-embracing, but that as a matter of policy implicit in the
Constitution itself the Court should abstain from interfering with In the first place I am convinced (as are the other Justices),
without need of receiving evidence as in an ordinary adversary
the Executive's Proclamation, dealing as it does with national
security, for which the responsibility is vested by the charter in court proceeding, that a state of rebellion existed in the country
when Proclamation No. 1081 was issued. It was a matter of
him alone. But the Court should act, Justice Barredo opines,
when its abstention from acting would result in manifest and contemporary history within the cognizance not only of the
palpable transgression of the Constitution proven by facts of courts but of all observant people residing here at the time.
judicial notice, no reception of evidence being contemplated for Many of the facts and events recited in detail in the different
purposes of such judicial action. "Whereases" of the proclamation are of common knowledge.
The state of rebellion continues up to the present. The
argument that while armed hostilities go on in several
It may be noted that the postulate of non-justiciability as provinces in Mindanao there are none in other regions except
discussed in those opinions involves disparate methods of in isolated pockets in Luzon, and that therefore there is no
approach. Justice Esguerra maintains that the findings of the need to maintain martial law all over the country, ignores the
President on the existence of the grounds for the declaration sophisticated nature and ramifications of rebellion in a modern
of martial law are final and conclusive upon the Courts. He setting. It does not consist simply of armed clashes between
disagrees vehemently with the ruling in Lansang vs. Garcia, 42 organized and identifiable groups on fields of their own
SCRA 448, December 11, 1971, and advocates a return choosing. It includes subversion of the most subtle kind,
to Barcelon vs. Baker, 5 Phil. 87 (1905), and Montenegro vs. necessarily clandestine and operating precisely where there is
Castañeda, 91 Phil. 882 (1952). Justice Barredo, for his part, no actual fighting. Underground propaganda, through printed
holds that Lansang need not be overturned, indeed does not news sheets or rumors disseminated in whispers; recruitment
control in these cases. He draws a distinction between the of armed and ideological adherents, raising of funds,
power of the President to suspend the privilege of the writ procurement of arms and material, fifth-column activities
of habeas corpus, which was the issue in Lansang, and his including sabotage and intelligence — all these are part of the
power to proclaim martial law, calling attention to the fact that rebellion which by their nature are usually conducted far from
while the Bill of Rights prohibits suspension of the privilege the battle fronts. They cannot be counteracted effectively
except in the instances specified therein, it places no such unless recognized and dealt with in that context.
prohibition or qualification with respect to the declaration of
martial law.
FINALS CONSTITUTION I ACJUCO 206

Secondly, my view, which coincides with that of other members is, that the proclamation of martial law automatically suspends
of the Court as stated in their opinions, is that the question of the privilege of the writ as to the persons referred to, the Court
validity of Proclamation No. 1081 has been foreclosed by the is practically unanimous. Justice Fernando, however, says that
transitory provision of the 1973 Constitution [Art. XVII, Sec. to him that is still an open question; and Justice Muñoz Palma
3(2)] that "all proclamations, orders, decrees, instructions, and qualifiedly dissents from the majority in her separate opinion,
acts promulgated, issued, or done by the incumbent President but for the reasons she discusses therein votes for the
shall be part of the law of the land and shall remain valid, legal, dismissal of the petitions.
binding and effective even after ... the ratification of this
Constitution ..." To be sure, there is an attempt in these cases IN VIEW OF ALL THE FOREGOING AND FOR THE
to resuscitate the issue of the effectivity of the new REASONS STATED BY THE MEMBERS OF THE COURT IN
Constitution. All that, however, is behind us now. The question THEIR SEPARATE OPINIONS, JUDGMENT IS HEREBY
has been laid to rest by our decision in Javellana vs. Executive RENDERED DISMISSING ALL THE PETITIONS, EXCEPT
Secretary (L-36142, 50 SCRA 30, March 31, 1973), and of
THOSE WHICH HAVE BEEN PREVIOUSLY WITHDRAWN
course by the existing political realities both in the conduct of BY THE RESPECTIVE PETITIONERS WITH THE
national affairs and in our relations with other countries.
APPROVAL OF THIS COURT, AS HEREINABOVE
MENTIONED. NO COSTS.
On the effect of the transitory provision Justice Muñoz Palma
withholds her assent to any sweeping statement that the same Makasiar, Esguerra, Fernandez, Muñoz Palma and Aquino,
in effect validated, in the constitutional sense, all "such
JJ., concur.
proclamations, decrees, instructions, and acts promulgated,
issued, or done by the incumbent President." All that she
concedes is that the transitory provision merely gives them Prefatory Note
"the imprimatur of a law but not of a constitutional mandate,"
and as such therefore "are subject to judicial review when (written on September 12, 1974)
proper under the Constitution.
My separate opinion below in the nine cases at bar was
Finally, the political-or-justiciable question controversy indeed, handed to Chief Justice Querube C. Makalintal on Monday,
any inquiry by this Court in the present cases into the September 9, 1974, for promulgation (together with the
constitutional sufficiency of the factual bases for the individual opinions of the Chief Justice and the other Justices)
proclamation of martial law — has become moot and on September 12 (today) as agreed upon by the Court.
purposeless as a consequence of the general referendum of
July 27-28, 1973. The question propounded to the voters was:
"Under the (1973) Constitution, the President, if he so desires, On September 11 the petitioner Jose W. Diokno was released
can continue in office beyond 1973. Do you want President from military custody. The implications of this supervening
Marcos to continue beyond 1973 and finish the reforms he event were lengthily discussed by the Court in its deliberations
initiated under Martial Law?" The overwhelming majority of in the afternoon. Eleven members thereafter voted to dismiss
those who cast their ballots, including citizens between 15 and Diokno's petition as being "moot and academic;" I cast the lone
18 years, voted affirmatively on the proposal. The question was dissenting vote. Although perhaps in the strictest technical
thereby removed from the area of presidential power under the sense that accords with conventional legal wisdom, the petition
Constitution and transferred to the seat of sovereignty itself. has become "moot" because Diokno has been freed from
Whatever may be the nature of the exercise of that power by physical confinement, I am nonetheless persuaded that the
the President in the beginning — whether or not purely political grave issues of law he has posed and the highly insulting and
and therefore non-justiciable — this Court is precluded from derogatory imputations made by him against the Court and its
applying its judicial yardstick to the act of the sovereign. members constitute an inescapable residue of questions of
transcendental dimension to the entire nation and its destiny
and to the future of the Court — questions that cannot and
2. With respect to the petitioners who have been released from should not be allowed to remain unresolved and unanswered.
detention but have not withdrawn their petitions because they
are still subject to certain restrictions,5 the ruling of the Court is
that the petitions should be dismissed. The power to detain I have thus not found it needful nor even advisable to recast
persons even without charges for acts related to the situation my separate opinion or change a word of it.
which justifies the proclamation of martial law, such as the
existence of a state of rebellion, necessarily implies the power I invite the reader to assess my 38-page separate opinion
(subject, in the opinion of the Justices who consider Lansang which immediately follows, in the light of the foregoing context
applicable, to the same test of arbitrariness laid down therein), and factual setting.
to impose upon the released detainees conditions or
restrictions which are germane to and necessary to carry out
the purposes of the proclamation. Justice Fernando, however,
"is for easing the restrictions on the right to travel of petitioner
Rodrigo" and others similarly situated and so to this extent
dissents from the ruling of the majority; while Justice
Teehankee believes that those restrictions do not constitute
deprivation of physical liberty within the meaning of the
constitutional provision on the privilege of the writ of habeas
corpus.

It need only be added that, to my mind, implicit in a state of


martial law is the suspension of the said privilege with respect
to persons arrested or detained for acts related to the basic
objective of the proclamation, which is to suppress invasion,
insurrection, or rebellion, or to safeguard public safety against
imminent danger thereof. The preservation of society and
national survival take precedence. On this particular point, that
FINALS CONSTITUTION I ACJUCO 207

Olaquer vs. MC No. 4, 150 SCRA 144 (1987)

In 1979, Olaguer and some others were detained by military


personnel and they were placed inCamp Bagong Diwa.
Logauer and his group are all civilians. They were charged with
(1) unlawfulpossession of explosives and incendiary
devices; (2) conspiracy to assassinate President andMrs.
Marcos; (3) conspiracy to assassinate cabinet members Juan
Ponce Enrile, Francisco Tatadand Vicente Paterno; (4)
conspiracy to assassinate Messrs. Arturo Tangco, Jose Roño
andOnofre Corpus; (5) arson of nine buildings; (6) attempted
murder of Messrs. Leonardo Perez,Teodoro Valencia and
Generals Romeo Espino and Fabian Ver; and (7)
conspiracyand proposal to commit rebellion, and inciting to
rebellion. On August 19, 1980, the petitionerswent to the SC
and filed the instant Petition for prohibition and habeas corpus.
ISSUE:
Whether or not the petition for habeas corpus be granted.
HELD:
The petition for habeas corpus has become moot and
academic because by the time thecase reached the SC
Olaguer and his companions were already released from
military
confinement. “When the release of the persons in whose behalf
the application for a writ of habeas corpus was filed is effected,
the petition for the issuance of the writ becomes moot
andacademic. 18 Inasmuch as the herein petitioners have
been released from their confinement inmilitary detention
centers, the instant Petitions for the issuance of a writ of
habeas corpus
should be dismissed for having become moot and academic.”
But the military court
created totry the case of Olaguer (and the decision it rendered)
still continues to subsist.
ISSUE2:
The issue is then shifted to: Whether or not a military tribunal
has the jurisdiction to trycivilians while the civil courts are open
and functioning.
HELD:
The SC nullified for lack of jurisdiction all decisions rendered
by the military courts ortribunals during the period of martial
law in all cases involving civilian defendants. A
militarycommission or tribunal cannot try and exercise
jurisdiction, even during the period of martiallaw, over civilians
for offenses allegedly committed by them as long as the civil
courts are openand functioning, and that any judgment
rendered by such body relating to a civilian is null andvoid for
lack of jurisdiction on the part of the military tribunal concerned.
FINALS CONSTITUTION I ACJUCO 208

GR No. 162318 network. They claimed that they went to Oakwood to air their
NAVALES VS. ABAYA grievances against the administration of President Gloria
DECISION Macapagal Arroyo. Among those grievances were: the graft
and corruption in the military, the sale of arms and ammunition
CALLEJO, SR, J.: to the enemies of the State, the bombings in Davao City which
were allegedly ordered by Brig. Gen. Victor Corpus, Chief of
Before the Court are two petitions essentially assailing the ISAFP, in order to obtain more military assistance from the
the jurisdiction of the General Court-Martial to conduct the United States government, and the micro-management in the
court-martial proceedings involving several junior officers and AFP by then Department of National Defense (DND) Secretary
enlisted men of the Armed Forces of the Philippines (AFP) Angelo Reyes. They declared their withdrawal of support from
charged with violations of the Articles of War (Commonwealth the chain of command and demanded the resignation of key
Act No. 408, as amended) in connection with their participation civilian and military leaders of the Arroyo administration.
in the take-over of the Oakwood Premier Apartments in Ayala
Around 9:00 a.m., Pres. Arroyo gave the soldiers until
Center, Makati City on July 27, 2003.
5:00 p.m. to give up their positions peacefully and return to
In G.R. No. 162341, Roberto Rafael Pulido, a lawyer, barracks. At about 1:00 p.m., she declared the existence of a
filed with this Court a Petition for Habeas Corpus seeking the state of rebellion and issued an order to use reasonable force
release of his clients, junior officers and enlisted men of the in putting down the rebellion. A few hours later, the soldiers
AFP, who are allegedly being unlawfully detained by virtue of again went on television reiterating their grievances. The
the Commitment Order[1] dated August 2, 2003 issued by deadline was extended twice, initially to 7:00 p.m., and later,
General Narciso L. Abaya, Chief of Staff of the AFP, pursuant indefinitely.
to Article 70 of the Articles of War. Under the said commitment
In the meantime, a series of negotiations ensued
order, all the Major Service Commanders and the Chief of the
between the soldiers and the Government team led by
Intelligence Service of the Armed Forces of the Philippines
Ambassador Roy Cimatu. An agreement was forged between
(ISAFP) were directed to take custodial responsibility of all the
the two groups at 9:30 p.m. Shortly thereafter, Pres. Arroyo
military personnel involved in the 27 July 2003 mutiny
announced that the occupation of Oakwood was over. The
belonging to their respective commands. This included all the
soldiers agreed to return to barracks and were out of the
junior officers and enlisted men (hereinafter referred to as
Oakwood premises by 11:00 p.m.
Capt. Reaso,[2] et al.) who are subject of the instant petition for
habeas corpus. The commitment order, however, expressly
stated that LtSG. Antonio F. Trillanes, LtSG. James A. Layug,
Capt. Garry C. Alejano, Capt. Milo D. Maestrecampo, Capt.
The Filing of Charges
Gerardo O. Gambala, and Capt. Nicanor E. Faeldon would
remain under the custody of the Chief of the ISAFP. [3]
In G.R. No. 162318, the petitioners (hereinafter referred Under the Information[5] dated August 1, 2003 filed with
to as 1Lt. Navales, et al.), seven of the detained junior officers the Regional Trial Court (RTC) of Makati City, the Department
and enlisted men, filed with this Court a Petition for Prohibition of Justice (DOJ) charged 321 of those soldiers who took part
under Rule 65 of the Rules of Court seeking to enjoin the in the Oakwood Incident with violation of Article 134-A (coup
General Court-Martial from proceeding with the trial of the detat) of the Revised Penal Code.[6] Among those charged
petitioners and their co-accused for alleged violations of the were petitioners 1Lt. Navales, et al. (G.R. No. 162318) and
Articles of War. those who are subject of the petition for habeas corpus Capt.
Reaso, et al. (G.R. No. 162341). The case, entitled People v.
Named as respondents in the two petitions are General Capt. Milo Maestrecampo, et al., was docketed as Criminal
Narciso Abaya who, as Chief of Staff of the AFP, exercises Case No. 03-2784 and raffled to Branch 61 presided by Judge
command and control over all the members and agencies of Romeo F. Barza.
the AFP, and Brigadier General Mariano Sarmiento, Jr., the
Judge Advocate General of the AFP and officer in command On September 12, 2003, several (243 in number) of the
of the Judge Advocate General Office (JAGO), the agency of accused in Criminal Case No. 03-2784 filed with the RTC
the AFP tasked to conduct the court-martial proceedings. (Branch 61) an Omnibus Motion praying that the trial court:

1. [A]ssume jurisdiction over all the charges filed before the


[4] military tribunal in accordance with Republic Act No. 7055; and
Background

2. Order the prosecution to present evidence to establish


At past 1:00 a.m. of July 27, 2003, more than three probable cause against 316 of the 321 accused and, should
hundred junior officers and enlisted men, mostly from the elite the prosecution fail to do so, dismiss the case as against the
units of the AFP the Philippine Armys Scout Rangers and the 316 other accused.[7]
Philippine Navys Special Warfare Group (SWAG) quietly
entered the premises of the Ayala Center in Makati City. They While the said motion was pending resolution, the DOJ
disarmed the security guards and took over the Oakwood issued the Resolution dated October 20, 2003 finding probable
Premier Apartments (Oakwood). They planted explosives cause for coup detat[8] against only 31 of the original 321
around the building and in its vicinity. Snipers were posted at accused and dismissing the charges against the other 290 for
the Oakwood roof deck. insufficiency of evidence.
The soldiers, mostly in full battle gear and wearing red Thus, upon the instance of the prosecution, the RTC
arm bands, were led by a small number of junior officers, (Branch 61), in its Order[9] dated November 14, 2003, admitted
widely known as the Magdalo Group. The leaders were later the Amended Information[10] dated October 30, 2003 charging
identified as including Navy LtSG. Antonio Trillanes IV, Army only 31 of the original accused with the crime of coup
Capt. Gerardo Gambala, Army Capt. Milo Maestrecampo, detat defined under Article 134-A of the Revised Penal
Navy LtSG. James Layug, and Marine Capt. Gary Alejano. Code.[11] Only the following were charged under the Amended
Information: CPT. MILO D. MAESTRECAMPO, LTSG.
Between 4:00 to 5:00 a.m., the soldiers were able to ANTONIO F. TRILLANES IV, CPT. GARY C. ALEJANO,
issue a public statement through the ABS-CBN News (ANC)
FINALS CONSTITUTION I ACJUCO 209

LTSG. JAMES A. LAYUG, CPT. LAURENCE LUIS B. The Petitioners Case


SOMERA, CPT. GERARDO O. GAMBALA, CPT. NICANOR
FAELDON, CPT. ALBERT T. BALOLOY, CPT. SEGUNDINO
P. ORFIANO, JR., CPT. JOHN P. ANDRES, CPT. ALVIN H. In support of the petitions for prohibition and for habeas
EBREO, 1LT. FLORENTINO B. SOMERA, 1LT. CLEO B. corpus, the petitioners advance the following arguments:
DUNGGA-AS, 1LT. SONNY S. SARMIENTO, 1LT. AUDIE S.
TOCLOY, 1LT. VON RIO TAYAB, 1LT. REX C. BOLO, 1LT. I. UNDER REPUBLIC ACT NO. 7055, THE
LAURENCE R. SAN JUAN, 1LT. WARREN LEE G. RESPONDENTS AND THE GENERAL
DAGUPON, 1LT. NATHANIEL N. RABONZA, 2LT. COURT-MARTIAL ARE WITHOUT ANY
KRISTOFFER BRYAN M. YASAY, 1LT. JONNEL P. JURISDICTION TO FURTHER CONDUCT
SANGGALANG, 1LT. BILLY S. PASCUA, 1LT. FRANCISCO PROCEEDINGS AGAINST THE
ACEDILLO, LTSG. MANUEL G. CABOCHAN, LTSG. PETITIONERS AND THEIR COLLEAGUES
EUGENE LOUIE GONZALES, LTSG. ANDY G. TORRATO, BECAUSE THE REGIONAL TRIAL COURT
LTJG. ARTURO S. PASCUA, JR., ENS. ARMAND HAS ALREADY DETERMINED THAT THE
PONTEJOS, PO3 JULIUS J. MESA, PO3 CESAR OFFENSES ARE NOT SERVICE-RELATED
GONZALES, and several JOHN DOES and JANE DOES. AND ARE PROPERLY WITHIN THE
Further, the said Order expressly stated that the case against JURISDICTION OF THE CIVILIAN
the other 290 accused, including petitioners 1Lt. Navales, et al. COURTS;[15] and
and those who are subject of the petition for habeas corpus,
Capt. Reaso, et al., was dismissed. In another Order dated II. THE RESPONDENTS HAVE NO AUTHORITY
November 18, 2003, the RTC (Branch 61) issued commitment TO FURTHER DETAIN THE JUNIOR
orders against those 31 accused charged under the Amended OFFICERS AND ENLISTED MEN AS THE
Information and set their arraignment. CHARGES FOR COUP DETAT BEFORE THE
REGIONAL TRIAL COURT HAVE BEEN
Meanwhile, 1Lt. Navales, et al. and Capt. Reaso, et al., DISMISSED FOR LACK OF EVIDENCE
who were earlier dropped as accused in Criminal Case No. 03- UPON MOTION OF THE DEPARTMENT OF
2784, were charged before the General Court-Martial with JUSTICE.[16]
violations of the Articles of War (AW), particularly: AW 67
(Mutiny), AW 97 (Conduct Prejudicial to Good Order and Citing Section 1[17] of Republic Act No. 7055,[18] the
Military Discipline), AW 96 (Conduct Unbecoming an Officer petitioners theorize that since the RTC (Branch 148), in its
and a Gentleman), AW 63 (Disrespect to the President, the Order dated February 11, 2004, already declared that the
Secretary of Defense, etc.) and AW 64 (Disrespect Towards offenses for which all the accused were charged were not
Superior Officer).[12] On the other hand, Capt. Maestrecampo service-connected, but absorbed and in furtherance of the
and the 30 others who remained charged under the Amended crime of coup detat, the General Court-Martial no longer has
Information were not included in the charge sheets for jurisdiction over them. As such, respondents Gen. Abaya and
violations of the Articles of War. the JAGO have no authority to constitute the General Court-
Martial, to charge and prosecute the petitioners and their co-
Thereafter, Criminal Case No. 03-2784 was consolidated accused for violations of the Articles of War in connection with
with Criminal Case No. 03-2678, entitled People v. Ramon the July 27, 2003 Oakwood Incident. The petitioners posit that,
Cardenas, pending before Branch 148 of the RTC of Makati as a corollary, there is no longer any basis for their continued
City, presided by Judge Oscar B. Pimentel. detention under the Commitment Order dated August 2, 2003
issued by Gen. Abaya considering that the charge against
On February 11, 2004, acting on the earlier Omnibus them for coup detat had already been dismissed.
Motion filed by the 243 of the original accused under the
Information dated August 1, 2003, the RTC (Branch 148) In G.R. No. 162318, the petitioners pray that the
issued an Order, the dispositive portion of which reads: respondents be enjoined from constituting the General Court-
Martial and from further proceeding with the court-martial of the
petitioners and their co-accused for violations of the Articles of
WHEREFORE, premises considered, in view of the Orders
War in connection with the Oakwood Incident of July 27, 2003.
dated November 14 and 18, 2003 of Judge Romeo Barza, the
In G.R. No. 162341, the petitioner prays that the respondents
Omnibus Motion to: 1) Assume jurisdiction over all charges
be ordered to explain why the detained junior officers and
filed before the Military Courts in accordance with R.A. 7055;
enlisted men subject of the petition for habeas corpus should
and 2) Implement the August 7, 2003 Order of the Court
not be released without delay.
requiring the prosecution to produce evidence to establish
probable cause are hereby considered MOOT AND
ACADEMIC and, lastly, all charges before the court-martial
against the accused (those included in the Order of November
The Respondents Arguments
18, 2003) as well as those former accused (those included in
the Order of November 14, 2003) are hereby declared not
service-connected, but rather absorbed and in furtherance to
the alleged crime of coup detat.[13] The respondents, through the Office of the Solicitor
General, urge the Court to dismiss the petitions. The
respondents contend that the Order dated February 11, 2004
In the Notice of Hearing dated March 1, 2004, the promulgated by the RTC (Branch 148), insofar as it resolved
General Court-Martial set on March 16, 2004 the the Omnibus Motion and declared that the charges against all
arraignment/trial of those charged with violations of the Articles the accused, including those excluded in the Amended
of War in connection with the July 27, 2003 Oakwood Incident. Information, were not service-connected, is null and void. They
aver that at the time that the said motion was resolved,
The present petitions were then filed with this Court.
petitioners 1Lt. Navales, et al. and Capt. Reaso, et al. (as
Acting on the prayer for the issuance of temporary restraining
movants therein) were no longer parties in Criminal Case No.
order in the petition for prohibition in G.R. No. 162318, this
03-2784 as the charge against them was already dismissed by
Court, in the Resolution dated March 16, 2004, directed the
the RTC (Branch 61) in the Order dated November 14, 2003.
parties to observe the status quo prevailing before the filing of
Thus, 1Lt. Navales, et al. and Capt. Reaso, et al. no longer had
the petition.[14]
any personality to pursue the Omnibus Motion since one who
has no right or interest to protect cannot invoke the jurisdiction
of the court. In other words, the petitioners were not real parties
FINALS CONSTITUTION I ACJUCO 210

in interest at the time that their Omnibus Motion was resolved We agree with the respondents that the sweeping
by the RTC (Branch 148). declaration made by the RTC (Branch 148) in the dispositive
portion of its Order dated February 11, 2004 that all charges
The respondents further claim denial of due process as before the court-martial against the accused were not service-
they were not given an opportunity to oppose or comment on connected, but absorbed and in furtherance of the crime
the Omnibus Motion. Worse, they were not even given a copy of coup detat, cannot be given effect. For reasons which shall
of the Order dated February 11, 2004. As such, the same be discussed shortly, such declaration was made without or in
cannot be enforced against the respondents, especially excess of jurisdiction; hence, a nullity.
because they were not parties to Criminal Case No. 03-2784.
The trial courts declaration was
The respondents, likewise, point out a seeming made when the Omnibus Motion
ambiguity in the February 11, 2004 Order as it declared, on had already been rendered moot
one hand, that the charges filed before the court-martial were and academic with respect to
not service-connected, but on the other hand, it ruled that the 1Lt. Navales, et al. and Capt.
Omnibus Motion was moot and academic. According to the Reaso, et al. by reason of the
respondents, these two pronouncements cannot stand side by dismissal of the charge of coup
side. If the Omnibus Motion was already moot and academic, detat against them
because the accused who filed the same were no longer being
charged with coup detat under the Amended Information, then The Order dated February 11, 2004 was issued
the trial court did not have any authority to further resolve and purportedly to resolve the Omnibus Motion, which prayed for
grant the same Omnibus Motion. the trial court to, inter alia, acquire jurisdiction over all the
charges filed before the military courts in accordance with Rep.
The respondents maintain that since 1Lt. Navales, et al. Act No. 7055. The said Omnibus Motion was filed on
and Capt. Reaso, et al. were not being charged with coup September 12, 2003 by 243 of the original accused under the
detat under the Amended Information, the trial court could not Information dated August 1, 2003. However, this information
make a finding that the charges filed against them before the was subsequently superseded by the Amended Information
General Court-Martial were in furtherance of coup detat. For dated October 20, 2003 under which only 31 were charged with
this reason, the declaration contained in the dispositive portion the crime of coup detat. In the November 14, 2003 Order of the
of the February 11, 2004 Order - that charges filed against the RTC (Branch 61), the Amended Information was admitted and
accused before the court-martial were not service-connected - the case against the 290 accused, including 1Lt. Navales, et
cannot be given effect. al. and Capt. Reaso, et al., was dismissed. The said Order
became final and executory since no motion for
Similarly invoking Section 1 of Rep. Act No. 7055, the
reconsideration thereof had been filed by any of the parties.
respondents vigorously assert that the charges against 1Lt.
Navales, et al. and Capt. Reaso, et al. filed with the General Thus, when the RTC (Branch 148) eventually resolved
Court-Martial, i.e., violations of the Articles of War 63, 64, 67, the Omnibus Motion on February 11, 2004, the said motion had
96 and 97, are, in fact, among those declared to be service- already been rendered moot by the November 14, 2003 Order
connected under the second paragraph of this provision. This of the RTC (Branch 61) admitting the Amended Information
means that the civil court cannot exercise jurisdiction over the under which only 31 of the accused were charged and
said offenses, the same being properly cognizable by the dismissing the case as against the other 290. It had become
General Court-Martial. Thus, the RTC (Branch 148) acted moot with respect to those whose charge against them was
without or in excess of jurisdiction when it declared in its dismissed, including 1Lt. Navales, et al. and Capt. Reaso, et
February 11, 2004 Order that the charges against those al., because they were no longer parties to the case. This was
accused before the General Court-Martial were not service- conceded by the RTC (Branch 148) itself as it stated in the
connected, but absorbed and in furtherance of the crime body of its February 11, 2004 Order that:
of coup detat. Said pronouncement is allegedly null and void.
The respondents denounce the petitioners for their forum Now, after going over the records of the case, the Court is of
shopping. Apparently, a similar petition (petition for habeas the view that the movants first concern in their omnibus motion,
corpus, prohibition with injunction and prayer for issuance of a i.e., assume jurisdiction over all charges filed before military
temporary restraining order) had been filed by the petitioners courts in accordance with R.A. 7055, has been rendered moot
co-accused with the Court of Appeals, docketed as CA-G.R. and academic by virtue of the Order dated November 14, 2003
SP No. 82695. The case was resolved against the petitioners dismissing the case against TSg. Leonel M. Alnas, TSg.
therein. Ramon B. Norico, SSg. Eduardo G. Cedeno, et al. and finding
probable cause in the Order dated November 18, 2003 against
The respondents pray that the petitions be dismissed for accused Cpt. Milo D. Maestrecampo, LtSg. Antonio F. Trillanes
lack of merit. IV, et al., issued by Judge Barza.

In view of the Order of Judge Barza dated November 14, 2003


Issue dismissing the case against aforesaid accused, the Court,
therefore, can no longer assume jurisdiction over all charges
filed before the military courts and this Court cannot undo nor
reverse the Order of November 14, 2003 of Judge Barza, there
The sole issue that needs to be resolved is whether or
being no motion filed by the prosecution to reconsider the order
not the petitioners are entitled to the writs of prohibition
or by any of the accused.[19]
and habeas corpus.

Accordingly, in the dispositive portion of the said Order,


the RTC (Branch 148) held that the Omnibus Motion was
The Courts Ruling considered moot and academic. And yet, in the same
dispositive portion, the RTC (Branch 148) still proceeded to
declare in the last clause thereof that all the charges before the
We rule in the negative. court-martial against the accused (those included in the Order
of November 18, 2003) as well as those former accused (those
included in the Order of November 14, 2003) are hereby
FINALS CONSTITUTION I ACJUCO 211

declared not service-connected, on its perception that the Art. 56. False Muster.
crimes defined in and penalized by the Articles of War were Art. 57. False Returns.
committed in furtherance of coup detat; hence, absorbed by Art. 58. Certain Acts to Constitute Desertion.
the latter crime. Art. 59. Desertion.
Art. 60. Advising or Aiding Another to Desert.
As earlier explained, insofar as those whose case Art. 61. Entertaining a Deserter.
against them was dismissed, there was nothing else left to Art. 62. Absence Without Leave.
resolve after the Omnibus Motion was considered moot and Art. 63. Disrespect Toward the President,
academic. Indeed, as they were no longer parties to the case, Vice-President, Congress of the Philippines,
no further relief could be granted to them. 1Lt. Navales, et al. or Secretary of National Defense.
and Capt. Reaso, et al. could be properly considered as Art. 64. Disrespect Toward Superior Officer.
strangers to the proceedings in Criminal Case No. 03-2784. Art. 65. Assaulting or Willfully Disobeying Superior
And in the same manner that strangers to a case are not bound Officer.
by any judgment rendered by the court,[20] any rulings made by Art. 66. Insubordinate Conduct Toward Non-
the trial court in Criminal Case No. 03-2784 are no longer Commissioned Officer.
binding on 1Lt. Navales, et al. and Capt. Reaso, et al. The RTC Art. 67. Mutiny or Sedition.
(Branch 148) itself recognized this as it made the statement, Art. 68. Failure to Suppress Mutiny or Sedition.
quoted earlier, that in view of the Order of Judge Barza dated Art. 69. Quarrels; Frays; Disorders.
November 14, 2003 dismissing the case against aforesaid Art. 70. Arrest or Confinement.
accused, the Court, therefore, can no longer assume
jurisdiction over all charges filed before the military courts and
this Court cannot undo nor reverse the Order of November 14, Articles 72 to 92
2003 of Judge Barza there being no motion filed by the
prosecution to reconsider the order or by any of the accused. [21] Art. 72. Refusal to Receive and Keep Prisoners.
Art. 73. Report of Prisoners Received.
Thus, 1Lt. Navales, et al. and Capt. Reaso, et al., who Art. 74. Releasing Prisoner Without Authority.
are no longer charged with coup detat, cannot find solace in Art. 75. Delivery of Offenders to Civil Authorities.
the declaration of the RTC (Branch 148) that the charges filed Art. 76. Misbehavior Before the Enemy.
before the General Court-Martial against them were not Art. 77. Subordinates Compelling Commander to
service-connected. The same is a superfluity and cannot be Surrender.
given effect for having been made by the RTC (Branch 148) Art. 78. Improper Use of Countersign.
without or in excess of its jurisdiction. Art. 79. Forcing a Safeguard.
Such declaration was made by the Art. 80. Captured Property to be Secured for Public
RTC (Branch 148) in violation of Service.
Section 1, Republic Act No. 7055 Art. 81. Dealing in Captured or Abandoned
Property.
Section 1 of Rep. Act No. 7055 reads in full: Art. 82. Relieving, Corresponding With, or Aiding
the Enemy.
Art. 83. Spies.
Section 1. Members of the Armed Forces of the Philippines and
Art. 84. Military Property. Willful or Negligent
other persons subject to military law, including members of the
Loss, Damage or Wrongful Disposition.
Citizens Armed Forces Geographical Units, who commit
Art. 85. Waste or Unlawful Disposition of
crimes or offenses penalized under the Revised Penal Code,
Military Property Issued to Soldiers.
other special penal laws, or local government ordinances,
Art. 86. Drunk on Duty.
regardless of whether or not civilians are co-accused, victims,
Art. 87. Misbehavior of Sentinel.
or offended parties which may be natural or juridical persons,
Art. 88. Personal Interest in Sale of
shall be tried by the proper civil court, except when the offense,
Provisions.
as determined before arraignment by the civil court, is service-
Art. 88-A. Unlawfully Influencing Action of
connected, in which case the offense shall be tried by court-
Court.
martial: Provided, That the President of the Philippines may, in
Art. 89. Intimidation of Persons Bringing
the interest of justice, order or direct at any time before
Provisions.
arraignment that any such crimes or offenses be tried by the
Art. 90. Good Order to be Maintained and
proper civil courts.
Wrongs Redressed.
Art. 91. Provoking Speeches or Gestures.
As used in this Section, service-connected crimes or offenses Art. 92. Dueling.
shall be limited to those defined in Articles 54 to 70, Articles 72
to 92, and Articles 95 to 97 of Commonwealth Act No. 408, as Articles 95 to 97:
amended.
Art. 95. Frauds Against the Government.
In imposing the penalty for such crimes or offenses, the court- Art. 96. Conduct Unbecoming an Officer and
martial may take into consideration the penalty prescribed Gentleman.
therefor in the Revised Penal Code, other special laws, or local Art. 97 General Article.
government ordinances.
Further, Section 1 of Rep. Act No. 7055 vests on the
The second paragraph of the above provision explicitly military courts the jurisdiction over the foregoing offenses. The
specifies what are considered service-connected crimes or following deliberations in the Senate on Senate Bill No. 1468,
offenses under Commonwealth Act No. 408 (CA 408), as which, upon consolidation with House Bill No. 31130,
amended, also known as the Articles of War, to wit: subsequently became Rep. Act No. 7055, are instructive:

Articles 54 to 70:
Senator Shahani. I would like to propose an addition to Section
1, but this will have to be on page 2. This will be in line 5, which
Art. 54. Fraudulent Enlistment. should be another paragraph, but still within Section 1. This is
Art. 55. Officer Making Unlawful Enlistment.
FINALS CONSTITUTION I ACJUCO 212

to propose a definition of what service-connected means, service-related or connected under the Articles of War,
because this appears on line 8. My proposal is the following: Commonwealth Act No. 408.[23]

SERVICE-CONNECTED OFFENSES SHALL MEAN THOSE It is clear from the foregoing that Rep. Act No. 7055 did
COMMITTED BY MILITARY PERSONNEL PURSUANT TO not divest the military courts of jurisdiction to try cases involving
THE LAWFUL ORDER OF THEIR SUPERIOR OFFICER OR violations of Articles 54 to 70, Articles 72 to 92 and Articles 95
WITHIN THE CONTEXT OF A VALID MILITARY EXERCISE to 97 of the Articles of War as these are considered service-
OR MISSION. connected crimes or offenses. In fact, it mandates that these
shall be tried by the court-martial.
I believe this amendment seeks to avoid any confusion as to Indeed, jurisdiction is the power and authority of the court
what service-connected offense means. Please note that to hear, try and decide a case.[24] Moreover, jurisdiction over
service-connected offense, under this bill, remains within the the subject matter or nature of the action is conferred only by
jurisdiction of military tribunals. the Constitution or by law.[25] It cannot be (1) granted by the
agreement of the parties; (2) acquired, waived, enlarged or
So, I think that is an important distinction, Mr. President. diminished by any act or omission of the parties; or (3)
conferred by the acquiescence of the courts.[26] Once vested
by law on a particular court or body, the jurisdiction over the
Senator Taada. Yes, Mr. President. I would just want to
subject matter or nature of the action cannot be dislodged by
propose to the Sponsor of this amendment to consider, any body other than by the legislature through the enactment
perhaps, defining what this service-related offenses would be
of a law. The power to change the jurisdiction of the courts is a
under the Articles of War. And so, I would submit for her
matter of legislative enactment which none but the legislature
consideration the following amendment to her amendment
may do. Congress has the sole power to define, prescribe and
which would read as follows: AS USED IN THIS SECTION,
apportion the jurisdiction of the courts. [27]
SERVICE-CONNECTED CRIMES OR OFFENSES SHALL BE
LIMITED TO THOSE DEFINED IN ARTICLES 54 TO 70, In view of the clear mandate of Rep. Act No. 7055, the
ARTICLES 72 TO 75, ARTICLES 76 TO 83 AND ARTICLES RTC (Branch 148) cannot divest the General Court-Martial of
84 TO 92, AND ARTICLES 95 TO 97, COMMONWEALTH its jurisdiction over those charged with violations of Articles 63
ACT NO. 408 AS AMENDED. (Disrespect Toward the President etc.), 64 (Disrespect Toward
Superior Officer), 67 (Mutiny or Sedition), 96 (Conduct
This would identify, I mean, specifically, what these service- Unbecoming an Officer and a Gentleman) and 97 (General
related or connected offenses or crimes would be. Article) of the Articles of War, as these are specifically included
as service-connected offenses or crimes under Section 1
thereof. Pursuant to the same provision of law, the military
The President. What will happen to the definition of service- courts have jurisdiction over these crimes or offenses.
connected offense already put forward by Senator Shahani?
There was no factual and legal basis for the RTC (Branch
Senator Taada. I believe that would be incorporated in the 148) to rule that violations of Articles 63, 64, 67, 96, and 97 of
specification of the Article I have mentioned in the Articles of the Articles of War were committed in furtherance of coup
War. detatand, as such, absorbed by the latter crime. It bears
stressing that, after a reinvestigation, the Panel of Prosecutors
found no probable cause for coup detat against the petitioners
SUSPENSION OF THE SESSION and recommended the dismissal of the case against them. The
trial court approved the recommendation and dismissed the
The President. Will the Gentleman kindly try to work it out case as against the petitioners. There is, as yet, no evidence
between the two of you? I will suspend the session for a on record that the petitioners committed the violations of
minute, if there is no objection. [There was none.] Articles 63, 64, 96, and 97 of the Articles of War in furtherance
of coup detat.
It was 5:02 p.m. In fine, in making the sweeping declaration that these
charges were not service-connected, but rather absorbed and
RESUMPTION OF THE SESSION in furtherance of the crime of coup detat, the RTC (Branch 148)
acted without or in excess of jurisdiction. Such declaration is,
in legal contemplation, necessarily null and void and does not
At 5:06 p.m., the session was resumed. exist.[28]
At this point, a review of its legislative history would put
The President. The session is resumed.
in better perspective the raison detre of Rep. Act No. 7055. As
early as 1938, jurisdiction over offenses punishable under CA
Senator Taada. Mr. President, Senator Shahani has graciously 408, as amended, also known as the Articles of War,
accepted my amendment to her amendment, subject to committed by persons subject to military law was vested on the
refinement and style. military courts. Thereafter, then President Ferdinand E.
Marcos promulgated Presidential Decree (PD) Nos.
The President. Is there any objection? [Silence] There being 1822,[29] 1850[30] and 1852.[31] These presidential decrees
none, the amendment is approved.[22] transferred from the civil courts to the military courts jurisdiction
over all offenses committed by members of the AFP, the former
Philippine Constabulary, the former Integrated National Police,
In the same session, Senator Wigberto E. Taada, the including firemen, jail guards and all persons subject to military
principal sponsor of SB No. 1468, emphasized: law.
In 1991, after a series of failed coup detats, Rep. Act No.
Senator Taada. Section 1, already provides that crimes of
7055 was enacted. In his sponsorship speech, Senator Taada
offenses committed by persons subject to military law ... will be
explained the intendment of the law, thus:
tried by the civil courts, except, those which are service-related
or connected. And we specified which would be considered
FINALS CONSTITUTION I ACJUCO 213

Senator Taada. The long and horrible nightmare of the past of habeas corpus should not be allowed after the party sought
continues to haunt us to this present day. Its vestiges remain to be released had been charged before any court or quasi-
instituted in our legal and judicial system. Draconian decrees judicial body.[34] The term court necessarily includes the
which served to prolong the past dictatorial regime subsist to General Court-Martial. These rules apply to Capt. Reaso, et
rule our new-found lives. Two of these decrees, Presidential al., as they are under detention pursuant to the Commitment
Decree No. 1822 and Presidential Decree No. 1850, as Order dated August 2, 2003 issued by respondent Chief of
amended, remain intact as laws, in spite of the fact that four Staff of the AFP pursuant to Article 70[35] of the Articles of War.
years have passed since we regained our democratic freedom.
On the other hand, the office of the writ of prohibition is
to prevent inferior courts, corporations, boards or persons from
The late Mr. Chief Justice Claudio Teehankee enunciated in usurping or exercising a jurisdiction or power with which they
the case of Olaguer vs. Military Commission No. 34 that the have not been vested by law. [36] As earlier discussed, the
greatest threat to freedom is the shortness of human memory. General Court-Martial has jurisdiction over the charges filed
against petitioners 1Lt. Navales, et al. under Rep. Act No.
PD No. 1822 and PD No. 1850 made all offenses committed 7055. A writ of prohibition cannot be issued to prevent it from
by members of the Armed Forces of the Philippines, the exercising its jurisdiction.
Philippine Constabulary, the Integrated National Police,
including firemen and jail guards, and all persons subject to WHEREFORE, premises considered, the petitions are
military law exclusively triable by military courts though, clearly, hereby DISMISSED.
jurisdiction over common crimes rightly belongs to civil courts. SO ORDERED.

Article II, Section 3 of the 1987 Constitution provides that


civilian authority is, at all times, supreme over the military.
Likewise, Article VIII, Section 1 declares that the judicial power
shall be vested in one Supreme Court and in such lower courts
as may be established by law.

In the case of Anima vs. The Minister of National Defense, (146


Supreme Court Reports Annotated, page 406), the Supreme
Court through Mr. Justice Gutierrez declared:

The jurisdiction given to military tribunals over common crimes


at a time when all civil courts were fully operational and freely
functioning constitutes one of the saddest chapters in the
history of the Philippine Judiciary.

The downgrading of judicial prestige caused by the glorification


of military tribunals ... the many judicial problems spawned by
extended authoritarian rule which effectively eroded judicial
independence and self-respect will require plenty of time and
determined efforts to cure.

The immediate return to civil courts of all cases which properly


belong to them is only a beginning.

...

Thus, as long as the civil courts in the land remain open and
are regularly functioning, military tribunals cannot try and
exercise jurisdiction over military men for criminal offenses
committed by them which are properly cognizable by the civil
courts. ...[32]

Clearly, in enacting Rep. Act No. 7055, the lawmakers


merely intended to return to the civilian courts the jurisdiction
over those offenses that have been traditionally within their
jurisdiction, but did not divest the military courts jurisdiction
over cases mandated by the Articles of War.

Conclusion

The writs of prohibition (G.R. No. 162318) and habeas


corpus (G.R. No. 162341) prayed for by the petitioners must
perforce fail. As a general rule, the writ of habeas corpus will
not issue where the person alleged to be restrained of his
liberty is in the custody of an officer under a process issued by
the court which has jurisdiction to do so. [33] Further, the writ
FINALS CONSTITUTION I ACJUCO 214

G.R. No. L-33964 December 11, 1971 E. Voltaire Garcia II and M. P. Vivo for petitioner Gary Olivar,
IN THE MATTER OF THE PETITION FOR HABEAS etc., et al.
CORPUS OF TEODOSIO LANSANG RODOLFO DEL Jose W. Diokno and Juanito R. Remulla for petitioner Antolin
ROSARIO, and BAYANI ALCALA, petitioners, Oreta, Jr.
vs. Domingo E. de Lara for and in his own behalf.
BRIGADIER-GENERAL EDUARDO M. GARCIA, Chief, Office of the Solicitor General Felix Q. Antonio and Assistant
Philippine Constabulary, respondent. Solicitor General Bernardo P. Pardo for respondents.
G.R. No. L-33965 December 11, 1971
ROGELIO V. ARIENDA, petitioner, CONCEPCION, C.J.:
vs.
SECRETARY OF NATIONAL DEFENSE, and CHIEF, PHIL. In the evening of August 21, 1971, at about 9 p.m., while the
CONSTABULARY, respondents. Liberal Party of the Philippines was holding a public meeting at
G.R. No. L-33973 December 11, 1971
Plaza Miranda, Manila, for the presentation of its candidates in
LUZVIMINDA DAVID, petitioner, the general elections scheduled for November 8, 1971, two (2)
vs.
hand grenades were thrown, one after the other, at the platform
GEN. EDUARDO GARCIA, in his capacity as Chief, where said candidates and other persons were. As a
Philippine Constabulary, COL. N. C. CAMELLO, in his consequence, eight (8) persons were killed and many more
capacity as Chief of Staff, Philippine Constabulary and injured, including practically all of the aforementioned
HON. JUAN PONCE ENRILE in his capacity as Secretary, candidates, some of whom sustained extensive, as well as
Department of National defense, respondents. serious, injuries which could have been fatal had it not been for
G.R. No. L-33982 December 11, 1971 the timely medical assistance given to them.
IN THE MATTER OF THE PETITION FOR HABEAS
CORPUS OF NEMESIO E. PRUDENTE FELICIDAD G.
PRUDENTE, petitioners, On August 23, soon after noontime, the President of the
vs. Philippines announced the issuance of Proclamation No. 889,
GENERAL MANUEL YAN, GEN. EDU dated August 21, 1971, reading as follows:
GARCIA, respondents.
G.R. No. L-34004 December 11, 1971 WHEREAS, on the basis of carefully
IN THE MATTER OF THE APPLICATION FOR HABEAS evaluated information, it is definitely
CORPUSIN BEHALF OF GERARDO TOMAS, ALSO established that lawless elements in the
KNOWN AS "GERRY TOMAS" AND FOR RETURN OF country, which are moved by common or
DOCUMENTS ILLEGALLY SEIZED. DOMINGO E. DE similar ideological conviction, design and
LARA, in his capacity as Chairman, Committee on Legal goal and enjoying the active moral and
Assistance, Philippine Bar Association, petitioner, material support of a foreign power and
vs. being guided and directed by a well trained,
BRIG. GENERAL EDUARDO M. GARCIA, CHIEF, determined and ruthless group of men and
PHILIPPINE CONSTABULARY, respondent. taking advantage of our constitutional
G.R. No. L-34013 December 11, 1971 liberties to promote and attain their ends,
REYNALDO RIMANDO, petitioner, have entered into a conspiracy and have in
vs. fact joined and banded their forces together
BRIG. GEN. EDUARDO M. GARCIA, Chief of the Philippine for the avowed purpose of actually staging,
Constabulary, respondent. undertaking and waging an armed
G.R. No. L-34039 December 11, 1971 insurrection and rebellion in order to
IN THE MATTER OF THE APPLICATION FOR HABEAS forcibly seize political power in this country,
CORPUSIN BEHALF OF SGT. FILOMENO M. DE CASTRO overthrow the duly constituted government,
AND HIS WIFE, MRS. BARCELISA C. DE CASTRO. and supplant our existing political social,
CARLOS C. RABAGO, in his capacity as President of the economic and legal order with an entirely
Conference Delegates Association of the Philippines new one whose form of government,
(CONDA),petitioner, whose system of laws, whose conception
vs. of God and religion, whose notion of
BRIG. GEN. EDUARDO M. GARCIA, Chief, Philippine individual rights and family relations, and
Constabulary, respondent. whose political, social and economic
G.R. No. L-34265 December 11, 1971 precepts are based on the Marxist-Leninist-
IN THE MATTER OF THE PETITION FOR HABEAS Maoist teachings and beliefs;
CORPUS OF ANTOLIN ORETA, JR. ANTOLIN ORETA,
JR., petitioner,
vs. WHEREAS, these lawless elements, acting
GEN. EDUARDO GARCIA and COL. PROSPERO in concert through front organizations that
OLIVAS, respondents. are seemingly innocent and harmless,
G.R. No. L-34339 December 11, 1971 have continuously and systematically
GARY B. OLIVAR, assisted by his father, GEORGE strengthened and broadened their
OLIVAR, petitioner, memberships through sustained and
vs. careful recruiting and enlistment of new
GEN. EDUARDO GARCIA, in his capacity as Chief, adherents from among our peasantry,
Philippine Constabulary, et al., respondents. laborers, professionals, intellectuals,
Ignacio P. Lacsina for petitioners Teodosio Lansang, et al. students, and mass media personnel, and
Ramon A. Gonzales for petitioner Rogelio V. Arienda. through such sustained and careful
E. Voltaire Garcia II for petitioner Luzvimindo David. recruitment and enlistment have
Verzola, Africa and Atencio, Lorenzo M. Tanada, Wigberto E. succeeded in infiltrating almost every
Tañada, Fortunato de Leon, R. G. Suntay and Juan T. David segment of our society in their ceaseless
for petitioner Felicidad G. Prudente. determination to erode and weaken the
Ruben L. Roxas for petitioner Reynaldo Rimando. political, social, economic and moral
Nuñez, Acob, Del Rosario and Ramos for petitioner Carlos foundations of our existing government and
Rabago, etc. to influence many peasant, labor,
FINALS CONSTITUTION I ACJUCO 215

professional, intellectual, student and mass residence, at No. 131-B Kamias Road, Quezon City, and
media organizations to commit acts of detained by the Constabulary;
violence and depredations against our duly
constituted authorities, against the
5. Felicidad G. Prudente, who filed the petition in Case No. L-
members of our law enforcement agencies, 33982 — on August 27, 1971 — upon the ground that her
and worst of all, against the peaceful father, Dr. NEMESIO E. PRUDENTE, had, on August 22,
members of our society; 1971, at about 8 p.m., been apprehended by Constabulary
agents in his house, at St. Ignatius Village, Quezon City, and
WHEREAS, these lawless elements have then detained at the Camp Crame stockade, Quezon City;
created a state of lawlessness and disorder
affecting public safety and the security of 6. ANGELO DE LOS REYES, who was allowed — on August
the State, the latest manifestation of which 30, 1971 — to intervene as one of the petitioners in Cases Nos.
has been the dastardly attack on the Liberal L-33964, L-33965 and L-33973, he having been arrested by
Party rally in Manila on August 21, 1971, members of the Constabulary on August 22, 1971, between
which has resulted in the death and serious 6:30 and 7:30 p.m., in his residence, at 86 Don Manuel Street,
injury of scores of persons; Sta. Mesa Heights, Quezon City, and brought to Camp Crame,
Quezon City, where he is detained and restrained of liberty;
WHEREAS, public safety requires that
immediate and effective action be taken in 7. VICTOR FELIPE, who was similarly allowed to intervene as
order to maintain peace and order, secure one of the petitioners in said three (3) cases, upon the ground
the safety of the people and preserve the
that, on August 23, 1971, at about 8 a.m., he was, likewise,
authority of the State; apprehended at Sta. Rosa, Laguna, by members of the
Philippine Constabulary and brought, first to the Constabulary
NOW, THEREFORE, I, FERDINAND E. headquarters at Canlubang, Laguna, and, then, to Camp
MARCOS, President of the Philippines, by Crame, Quezon City, where he is detained and restrained of
virtue of the powers vested upon me by liberty;
Article VII, Section 10, Paragraph (2) of the
Constitution, do hereby suspend the 8. TERESITO SISON, who was, also, allowed to intervene as
privilege of the writ of habeas corpus, for
one of the petitioners in the same three (3) cases, he having
the persons presently detained, as well as been arrested in his residence, at 318 Lakandula St., Angeles
others who may be hereafter similarly City, on August 22, 1971, between 6 and 7 p.m., and taken to
detained for the crimes of insurrection or the PC offices at Sto. Domingo, Angeles City, then to Camp
rebellion, and all other crimes and offenses Olivas, San Fernando, Pampanga, and eventually to Camp
committed by them in furtherance or on the Crame, Quezon City, where he is restrained and deprived of
occasion thereof, or incident thereto, or in liberty;
connection therewith.

9. GERARDO TOMAS, alias Gerry Tomas, a 17-year old


Presently, petitions for writ of habeas corpus were filed, in the second year college students of St. Louis University, Baguio
above-entitled cases, by the following persons, who, having City, on whose behalf, Domingo E. de Lara — in his capacity
been arrested without a warrant therefor and then detained,
as Chairman, Committee on Legal Assistance, Philippine Bar
upon the authority of said proclamation, assail its validity, as Association — filed on September 3, 1971, the petition in Case
well as that of their detention, namely:
No. L-34004, upon the ground that said Gerardo Tomas had,
on August 23, 1971, at about 6 a.m., been arrested by
1. TEDORO LANSANG, RODOLFO DEL ROSARIO and Constabulary agents, while on his way to school in the City of
BAYANI ALCALA, the petitioners in Case No. L-33964 — filed Baguio, then brought to the Constabulary premises therein at
on August 24, 1971 — who, on August 22, 1971, between 8 Camp Holmes, and, thereafter, taken, on August 24, 1971, to
a.m. and 6 p.m., were "invited" by agents of the Philippine Camp Olivas, Pampanga, and thence, on August 25, 1971, to
Constabulary — which is under the command of respondent the Constabulary headquarters at Camp Crame, Quezon City,
Brig. Gen. Eduardo M. Garcia — to go and did go to the where he is detained;
headquarters of the Philippine Constabulary, at Camp Crame,
Quezon City, for interrogation, and thereafter, detained; 10. REYNALDO RIMANDO, petitioner in Case No. L-34013 —
filed on September 7, 1971 — a 19-year old student of the U.P.
2. ROGELIO V. ARIENDA, the petitioner in Case No. L-33965 College in Baguio city — who, while allegedly on his way home,
— filed, also, on August 24, 1971 — who was picked up in his at Lukban Road, Baguio, on August 23, 1971, at about 1 a.m.,
residence, at No. 55 Road, 3, Urduja Village, Quezon City, by was joined by three (3) men who brought him to the Burnham
members of the Metrocom and then detained; Park, thence, to Camp Olivas at San Fernando, Pampanga,
and, thereafter, to Camp Crame, Quezon City, where he is
3. Soon after the filing of the petition in Case No. L-33965 — detained;
or on August 28, 1971 — the same was amended to include
VICENTE ILAO and JUAN CARANDANG, as petitioners 11. Sgt. FILOMENO M. DE CASTRO and his wife, Mrs.
therein, although, apart from stating that these additional BARCELISA C. DE CASTRO, on whose behalf Carlos C.
petitioners are temporarily residing with the original petitioner, Rabago — as President of the Conference Delegates
Rogelio V. Arienda, the amended petition alleged nothing Association of the Philippines (CONDA) — filed the petition in
whatsoever as regards the circumstances under which said Case No. L-34039 — on September 14, 1971 — against Gen.
Vicente Ilao and Juan Carandang are said to be illegally Eduardo M. Garcia, alleging that, on August 27, 1971, at about
deprived of their liberty; 3 p.m., Mrs. De Castro was arrested, while at Liamzon
Subdivision, Rosario, Pasig, Rizal, by agents of the
4. LUZVIMINDO DAVID, petitioner in Case No. L-33973 — Constabulary, and taken to the PC headquarters at Camp
filed on August 25, 1971 — who was similarly arrested in his Crame, where, later, that same afternoon, her husband was
brought, also, by PC agents and both are detained;
FINALS CONSTITUTION I ACJUCO 216

12. ANTOLIN ORETA, JR., who filed the petition in Case No. 1. A letter of the President to the Secretary of National
L-34265 — on October 26, 1971 — against said Gen. Garcia, Defense, dated August 21, 1971, directing, inter alia, in
as Chief of the Constabulary, and Col. Prospero Olivas, Chief connection with the arrest or detention of suspects pursuant to
of the Central Intelligence Service (CIS), Philippine Proclamation No. 889, that, except when caught inflagrante
Constabulary, alleging that, upon invitation from said CIS, he delicto, no arrest shall be made without warrant authorized in
went, on October 20, 1971, to Camp Aguinaldo, Quezon City, writing by the Secretary of National Defense; that such
to see Gen. Manuel Yan, Chief of Staff of the Armed Forces of authority shall not be granted unless, "on the basis of records
the Philippines, who referred petitioner to Col. Laroya of the and other evidences," it appears satisfactorily, in accordance
CIS; that the latter, in turn, referred him to CIS Investigator Atty. with Rule 113, section 6(b), of the Rules of Court, that the
Berlin Castillo and another CIS against, whose name is person to be arrested is probably guilty of the acts mentioned
unknown to the petitioner; and that, after being interrogated by in the proclamation; that, if such person will be charged with a
the two (2), petitioner was detained illegally; and crime subject to an afflictive penalty under the Anti-Subversion
Act, the authorization for his arrest shall not be issued unless
13. GARY OLIVAR, petitioner in Case No. L-34339 — filed on supported by signed intelligence reports citing at least one
reliable witness to the same overt act; that no unnecessary or
November 10, 1971 — who was apprehended, by agents of
the Constabulary, in the evening of November 8, 1941, in unreasonable force shall be used in effecting arrests; and that
Quezon City, and then detained at Camp Crame, in the same arrested persons shall not be subject to greater restraint than
City. is necessary for their detention;

Upon the filing of the aforementioned cases, the respondents 2. Communications of the Chief of the Constabulary, dated
were forthwith required to answer the petitions therein, which August 23, 27, and 30, 1971, to all units of his command,
they did. The return and answer in L-33964 — which stating that the privilege of the writ is suspended for no other
was, mutatis mutandis, reproduced substantially or by persons than those specified in the proclamation; that the
reference in the other cases, except L-34265 — alleges, inter same does not involve material law; that precautionary
alia, that the petitioners had been apprehended and detained measures should be taken to forestall violence that may be
"on reasonable belief" that they had "participated in the crime precipitated by improper behavior of military personnel; that
of insurrection or rebellion;" that "their continued detention is authority to cause arrest under the proclamation will be
exercised only by the Metrocom, CMA, CIS, and "officers
justified due to the suspension of the privilege of the writ
of habeas corpus pursuant to Proclamation No. 889 of the occupying position in the provinces down to provincial
commanders"; that there shall be no indiscriminate or mass
President of the Philippines;" that there is "a state of
insurrection or rebellion" in this country, and that "public safety arrests; that arrested persons shall not be harmed and shall be
and the security of the State required the suspension of the accorded fair and humane treatment; and that members of the
privilege of the writ of habeas corpus," as "declared by the detainee's immediate family shall be allowed to visit him twice
President of the Philippines in Proclamation No. 889; that in a week;
making said declaration, the "President of the Philippines acted
on relevant facts gathered thru the coordinated efforts of the 3. A memorandum of the Department of National Defense,
various intelligence agents of our government but (of) which dated September 2, 1971, directing the Chief of the
the Chief Executive could not at the moment give a full account Constabulary to establish appropriate Complaints and Action
and disclosure without risking revelation of highly classified Bodies/Groups to prevent and/or check any abuses in
state secrets vital to its safely and security"; that the connection with the suspension of the privilege of the writ; and
determination thus made by the President is "final and
conclusive upon the court and upon all other persons" and 4. Executive Order No. 333, dated August 26, 1971, creating a
"partake(s) of the nature of political question(s) which cannot
Presidential Administrative Assistance Committee to hear
be the subject of judicial inquiry," pursuant to Barcelon v. complaints regarding abuses committed in connection with the
Baker, 5 Phil. 87, and Montenegro v. Castañeda, 91 Phil. 882;
implementation of Proclamation No. 889.
that petitioners "are under detention pending investigation and
evaluation of culpabilities on the reasonable belief" that they
"have committed, and are still committing, individually or in Respondents in L-33965 further alleged that therein petitioners
conspiracy with others, engaged in armed struggle, insurgency Vicente Ilao and Juan Carandang had been released from
and other subversive activities for the overthrow of the custody on August 31, 1971, "after it had been found that the
Government; that petitioners cannot raise, in these evidence against them was insufficient."
proceedings for habeas corpus, "the question of their guilt or
innocence"; that the "Chief of Constabulary had petitioners In L-34265, the "Answer and Return" filed by respondents
taken into custody on the basis of the existence of evidence therein traversed some allegations of fact and conclusions of
sufficient to afford a reasonable ground to believe that law made in the petition therein and averred that Antolin Oreta,
petitioners come within the coverage of persons to whom the Jr., the petitioner therein, had been and is detained "on the
privilege of the writ of habeas corpus has been suspended"; basis of a reasonable ground to believe that he has committed
that the "continuing detention of the petitioners as an urgent overt acts in furtherance of rebellion or insurrection against the
bona fide precautionary and preventive measure demanded by government" and, accordingly, "comes within the class of
the necessities of public safety, public welfare and public persons as to whom the privilege of the writ of habeas
interest"; that the President of the Philippines has "undertaken corpus has been suspended by Proclamation No. 889, as
concrete and abundant steps to insure that the constitutional amended," the validity of which is not contested by him.
rights and privileges of the petitioners as well as of the other
persons in current confinement pursuant to Proclamation 889
remain unimpaired and unhampered"; and that "opportunities On August 30, 1971, the President issued Proclamation No.
or occasions for abuses by peace officers in the 889-A, amending Proclamation No. 889, so as to read as
implementation of the proclamation have been greatly follows:
minimized, if not completely curtailed, by various safeguards
contained in directives issued by proper authority." WHEREAS, on the basis of carefully
evaluated information, it is definitely
These safeguards are set forth in: established that lawless elements in the
country, which are moved by common or
similar ideological conviction, design and
FINALS CONSTITUTION I ACJUCO 217

goal and enjoying the active moral and privilege of the writ of habeas corpus for
material support of a foreign power and the persons presently detained, as well as
being guided and directed by a well- all others who may be hereafter similarly
trained, determined and ruthless group of detained for the crimes of insurrection or
men and taking advantage of our rebellion [,] and [all] other [crimes and
constitutional liberties to promote and offenses] overt acts committed by them in
attain their ends, have entered into a furtherance [or on the occasion] thereof[,].
conspiracy and have in fact joined and [or incident thereto, or in connection
banded their forces together for the therewith.]1
avowed purpose of [actually] staging,
undertaking, [and] wagging and On September 1, 1971, Cases Nos. L-33964, L-33965, L-
are actually engaged in an armed 33973 and L-33982 were jointly heard and then the parties
insurrection and rebellion in order to
therein were allowed to file memoranda, which were submitted
forcibly seize political power in this country, from September 3 to September 9, 1971.
overthrow the duly constituted government,
and supplant our existing political, social,
economic and legal order with an entirely Soon thereafter, or on September 18, 1971, Proclamation No.
new one whose form of government, 889 was further amended by Proclamation No. 889-B, lifting
whose system of laws, whose conception the suspension of the privilege of the writ of habeas corpus in
of God and religion, whose notion of the following provinces, sub-provinces and cities of the
individual rights and family relations, and Philippine, namely:
whose political, social and economic
precepts are based on the Marxist-Leninist- A. PROVINCES:
Maoist teaching and beliefs;
1. Batanes 15. Negros
WHEREAS, these lawless elements, acting Occ.
in concert through front organizations that 2. Ilocos Norte 16.
are seemingly innocent and harmless, Negros Or.
have continuously and systematically 3. Ilocos Sur 17. Cebu
strengthened and broadened their 4. Abra 18. Bohol
memberships through sustained and 5. Abra 19. Capiz
careful recruiting and enlistment of new 6. Pangasinan 20.
adherents from among our peasantly, Aklan
laborers, professionals, intellectuals, 7. Batangas 21.
students, and mass media personnel, and Antique
through such sustained and careful 8. Catanduanes 22.
recruitment and enlistment have Iloilo
succeeded in infiltrating almost every 9. Masbate 23. Leyte
segment of our society in their ceaseless 10. Romblon 24. Leyte
determination to erode and weaken the del Sur
political, social, economic and moral 11. Marinduque 25.
foundations of our existing government and Northern Samar
influence many peasant, labor, 12. Or. Mindoro 26.
professional, intellectual, student and mass Eastern Samar
media organizations to commit acts of 13. Occ. Mindoro 27.
violence and depredations against our duly Western Samar
constituted authorities, against the 14. Palawan.
members of our law enforcement agencies,
and worst of all, against the peaceful
members of our society; B. SUB-PROVINCES:

WHEREAS, these lawless elements, by 1. Guimaras 3. Siquior


their acts of rebellion and insurrection, 2. Biliran
have created a state of lawlessness and
disorder affecting public safety and security C. CITIES:
of the State, the latest manifestation of
which has been the dastardly attack on the
Liberal Party rally in Manila on August 21, 1. Laog 10. Bacolod
1971, which has resulted in the death and 2. Dagupan 11. Bago
serious injury of scores of persons; 3. San Carlos 12.
Canlaon
4. Batangas 13. La
WHEREAS, public safety requires that Carlota
immediate and effective action be taken in 5. Lipa 14. Bais
order to maintain peace and order, secure 6. Puerto Princesa 15.
the safety of the people and preserve the Dumaguete
authority of the State; 7. San Carlos (Negros
16. Iloilo
NOW THEREFORE, I, FERDINAND E. Occ.) 17. Roxas
MARCOS, President of the Philippines, by 8. Cadiz 18. Tagbilaran
virtue of the powers vested upon me by 9. Silay 19. Lapu-lapu
Article VII, Section 10, Paragraph (2) of the
Constitution, do hereby suspend the 20. Cebu 24. Tacloban
FINALS CONSTITUTION I ACJUCO 218

21. Mandaue 25. 3. Bulacan 13.


Ormoc Pampanga
22. Danao 26. 4. Camarines Sur 14.
Calbayog Quezon
23. Toledo 5. Ifugao 15. Rizal
6. Isabela 16. South
On September 25, 1971, the President issued Proclamation Cotabato
No. 889-C, restoring the privilege of the writ in the following 7. Laguna 17. Tarlac
provinces and cities: 8. Lanao del Norte 18.
Zambales
9. Lanao del Norte
A. PROVINCES:
B. SUB-PROVINCES:
1. Surigao del Norte 8.
Agusan del Sur
2. Surigao del Sur 9. 1. Aurora 2. Quirino
Misamis Or.
3. Davao del Norte 10. C. CITIES:
Misamis Occ.
4. Davao del Sur 11. 1. Angeles 10. Manila
Zamboanga del Norte 2. Baguio 11. Marawi
5. Davao Oriental 12. 3. Cabanatuan 12.
Basilan Naga
6. Bukidnon 13. 4. Caloocan 13.
Pagadian Olongapo
7. Agusan del Norte
5. Cotabato 14.
Palayan
B. CITIES: 6. General Santos 15.
Pasay
1. Surigao 8. Tangub 7. Iligan 16. Quezon
2. Davao 9. Dapitan 8 Iriga 17. San Jose
9 Lucena 18. San Pablo
3. Butuan 10. Dipolog
4. Cagayan 11.
Zamboanga The first major question that the Court had to consider was
5. Gingoong 12. whether it would adhere to the view taken in Barcelon v.
Basilan Baker,2 and reiterated in Montenegro v. Castañeda,3 pursuant
6. Ozamiz 13. to which, "the authority to decide whether the exigency has
Pagadian. arisen requiring suspension (of the privilege of the writ
7. Oroquieta of habeas corpus) belongs to the President and his 'decision is
final and conclusive' upon the courts and upon all other
On October 4, 1971, the suspension of the privilege was further persons." Indeed, had said question been decided in the
lifted by Proclamation No. 889-D, in the following places: affirmative the main issue in all of these cases, except
L-34339, would have been settled, and, since the other issues
were relatively of minor importance, said cases could have
A. PROVINCES: been readily disposed of. Upon mature deliberation, a majority
of the Members of the Court had, however, reached, although
1. Cagayan 5. tentatively, a consensus to the contrary, and decided that the
Camarines Court had authority to and should inquire into the existence of
2. Cavite 6. Albay the factual bases required by the Constitution for the
3. Mountain Province 7. suspension of the privilege of the writ; but before proceeding
Sorsogon to do so, the Court deemed it necessary to hear the parties on
4. Kalinga-Apayao the nature and extent of the inquiry to be undertaken, none of
them having previously expressed their views thereof.
Accordingly, on October 5, 1971, the Court issued, in L-33964,
B. CITIES: L-33965, L-33973 and L-33982, a resolution stating in part that

1. Cavite City 3. Trece
Martires ... a majority of the Court having tentatively
2. Tagaytay 4. Legaspi arrived at a consensus that it may inquire in
order to satisfy itself of the existence of the
As a consequences, the privilege of the writ of habeas factual bases for the issuance of
corpus is still suspended in the following eighteen (18) Presidential Proclamations Nos. 889 and
provinces, two (2) sub-provinces and eighteen (18) cities, to 889-A (suspending the privilege of the writ
wit: of habeas corpus for all persons detained
or to be detained for the crimes of rebellion
or insurrection throughout the Philippines,
A. PROVINCE: which area has lately been reduced to
some eighteen provinces, two
1. Bataan 10. North subprovinces and eighteen cities with the
Cotabato partial lifting of the suspension of the
2. Benguet 11. Nueva privilege effected by Presidential
Ecija Proclamations Nos. 889-B, 889-C and 889-
FINALS CONSTITUTION I ACJUCO 219

D) and thus determine the constitutional 1700 (Anti-Subversion Act), in the City Fiscal's Office of
sufficiency of such bases in the light of the Quezon City:
requirements of Article III, sec. 1, par. 14,
and Article VII, sec. 10, par. 2, of the
(1) Angelo de los Reyes -- G.R. No. L-
Philippine Constitution; and considering 22982 *
that the members of the Court are not (2) Teresito Sison -- " " L-33982 *
agreed on the precise scope and nature of
the inquiry to be made in the premises,
even as all of them are agreed that the (c) accused, together with many others named in the criminal
Presidential findings are entitled to great complaint filed therefor, of a violation of section 4 of Republic
respect, the Court RESOLVED that these Act No. 1700 (Anti-Subversion Act), in the Court of First
cases be set for rehearing on October 8, Instance of Rizal:
1971 at 9:30 A.M.
(1) Rodolfo del Rosario -- G.R. No. L-
xxx xxx xxx 33969 **
(2) Luzvimindo David -- " " L-33973
(3) Victor Felipe -- " " L-33982 *
On October 8, 1971, said four cases were, therefore, heard,
once again, but, this time jointly with cases Nos. L-34004, L-
34013, and L-34039, and the parties were then granted a and continue under detention pursuant to Proclamation No.
period to file memoranda, in amplification of their respective 889, as amended, and praying that the petitions in G.R. Nos.
oral arguments, which memoranda were submitted from L-33964, L-33965, L-33982, L-34004, L-34013 and L-34039 be
October 12 to October 21, 1971. dismissed, without prejudice to the resolution of the remaining
cases. Copy of the criminal complaint filed, as above stated,
with the Court of First Instance of Rizal and docketed therein
Respondents having expressed, during the oral arguments, on as Criminal Case No. Q-1623 of said court — which was
September 1 and October 8, 1971, their willingness to impart
appended to said manifestations-motions of the respondent as
to the Court classified information relevant to these cases, Annex 2 thereof — shows that Gary Olivar, the petitioner in L-
subject to appropriate security measures, the Court met at
34339, is one of the defendants in said case.
closed doors, on October 28 and 29, 1971, and, in the
presence of three (3) attorneys for the petitioners, chosen by
the latter, namely, Senator Jose W. Diokno, Senator Salvador Required to comment on said manifestations-motions,
H. Laurel, and Atty. Leopoldo Africa, as well as of the Solicitor Luzvimindo David, petitioner in L-33973, in his comment dated
General and two (2) members of his staff, was briefed, by Gen. November 23, 1971, urged the Court to rule on the merits of
Manuel Yan, Chief of Staff of the Armed Forces of the the petitions in all of these cases, particularly on the
Philippines, Gen. Fidel Ramos, Deputy Chief of Staff, Gen. constitutionality of Presidential Proclamation No. 889, as
Felizardo Tanabe, Col. Tagumpay Nanadiego, Judge amended, upon the ground that he is still detained and that the
Advocate General, JAGS (GSC), and other ranking officers of main issue is one of public interest involving as it does the civil
said Armed Forces, on said classified information, most of liberties of the people. Angelo de los Reyes, one of the
which was contained in reports and other documents already petitioners in L-33964, L-33965 and L-33973, Nemesio E.
attached to the records. During the proceedings, the members Prudente and Gerardo Tomas, for whose respective benefit
of the Court, and, occassionally, counsel for the petitioners, the petitions in L-33982 and L-34004 have been filed,
propounded pertinent questions to said officers of the Armed maintained that the issue in these cases is not moot, not even
Forces. Both parties were then granted a period of time within for the detainees who have been released, for, as long as the
which to submit their respective observations, which were filed privilege of the writ remains suspended, they are in danger of
on November 3, 1971, and complemented by some documents being arrested and detained again without just cause or valid
attached to the records on November 6, 1971, and a summary, reason. In his reply, dated and filed on November 29, 1971, the
submitted on November 15, 1971, of the aforesaid classified Solicitor General insisted that the release of the above-named
information. petitioners rendered their respective petitions moot and
academic.
In the meantime, cases Nos. L-34265 (Oreta) and L-34339
(Olivar) had been filed and the parties therein were heard in I
oral argument on November 4, and 16, 1971, respectively.
Petitioners herein, except Antolin Oreta, Jr. in L-34265,
On November 15, 1971, the Solicitor General filed question the formal validity of the proclamation suspending the
manifestations — motions stating that on November 13, 1971, privilege of the writ of habeas corpus. In this connection, it
the following petitioners were: should be noted that, as originally formulated, Proclamation
No. 889 was contested upon the ground that it did not comply
with the pertinent constitutional provisions, namely, paragraph
(a) released from custody: (14) of section 1, Article III of our Constitution, reading:

(1) Teodosio Lansang -- G.R. No. L-33964 The privilege of the writ of habeas
(2) Bayani Alcala -- " " L-33964 corpus shall not be suspended except in
(3) Rogelio Arienda -- " " L-33965 cases of invasion, insurrection, or rebellion,
(4) Nemesio Prudente -- " " L-33982 when the public safety requires it, in any
(5) Gerardo Tomas -- " " L-34004
way of which events the same may be
(6) Reynaldo Rimando -- " " L-34013 suspended wherever during such period
(7) Filomeno M. de Castro -- " " L-34039
the necessity for such suspension shall
(8) Barcelisa de Castro -- " " L-34039 exist.
(9) Antolin Oreta, Jr. -- " " L-34264.

and paragraph (2), section 10, Article VII of the same


(b) charged, together with other persons named in the criminal
instrument, which provides that:
complaint filed therefor, with a violation of Republic Act No.
FINALS CONSTITUTION I ACJUCO 220

The President shall be commander-in-chief II


of all armed forces of the Philippines, and
whenever it becomes necessary, he may Let us now consider the substantive validity of the
call out such armed forces to prevent or
proclamation, as amended. Pursuant to the above-quoted
suppress lawless violence, invasion, provisions of the Constitution, two (2) conditions must concur
insurrection, or rebellion. In case of for the valid exercise of the authority to suspend the privilege
invasion, insurrection, or rebellion, or to the writ, to wit: (a) there must be "invasion, insurrection, or
imminent danger thereof when the public rebellion" or — pursuant to paragraph (2), section 10 of Art. VII
safety requires it, he may suspend the of the Constitution — "imminent danger thereof," and (b)
privileges of the writ of habeas corpus, or "public safety" must require the suspension of the privilege.
place the Philippines or any part thereof The Presidential Proclamation under consideration declares
under martial law. that there has been and there is actually a state of rebellion
and
Regardless of whether or not the President may suspend the that4 "public safety requires that immediate and effective action
privilege of the writ of habeas corpus in case of "imminent be taken in order to maintain peace and order, secure the
danger" of invasion, insurrection or rebellion — which is one of safety of the people and preserve the authority of the State."
the grounds stated in said paragraph (2), section 10 of Art. VII
of the Constitution, but not mentioned in paragraph (14), Are these findings conclusive upon the Court? Respondents
section 1 of its Bill of Rights — petitioners maintained that
maintain that they are, upon the authority of Barcelon v.
Proclamation No. 889 did not declare the existence Baker5 and Montenegro v. Castañeda.6 Upon the other hand,
of actual "invasion insurrection or rebellion or imminent danger petitioners press the negative view and urge a reexamination
thereof," and that, consequently, said Proclamation was of the position taken in said two (2) cases, as well as a reversal
invalid. This contention was predicated upon the fact that, thereof.
although the first "whereas" in Proclamation No. 889 stated
that "lawless elements" had "entered into a conspiracy and
have in fact joined and banded their forces together for The weight of Barcelon v. Baker, as a precedent, is diluted by
the avowed purpose of actually staging, undertaking and two (2) factors, namely: (a) it relied heavily upon Martin v.
waging an armed insurrection and rebellion," the actuality so Mott7 involving the U.S. President's power to call out the
alleged refers to the existence, not of an uprising that militia, which — he being the commander-in-chief of all the
constitutes the essence of a rebellion or insurrection, but of armed forces — may be exercised to suppress or prevent any
the conspiracy and the intent to rise in arms. lawless violence, even without invasion, insurrection or
rebellion, or imminent danger thereof, and is, accordingly,
much broader than his authority to suspend the privilege of the
Whatever may be the merit of this claim, the same has been writ of habeas corpus, jeopardizing as the latter does individual
rendered moot and academic by Proclamation No. 889-A,
liberty; and (b) the privilege had been suspended by the
issued nine (9) days after the promulgation of the original American Governor-General, whose act, as representative of
proclamation, or on August 30, 1971. Indeed, said the Sovereign, affecting the freedom of its subjects, can hardly
Proclamation No. 889-A amended, inter alia, the first be equated with that of the President of the Philippines dealing
"whereas" of the original proclamation by postulating the said with the freedom of the Filipino people, in whom sovereignty
lawless elements "have entered into a conspiracy and have in resides, and from whom all government authority emanates.
fact joined and banded their forces together for the avowed The pertinent ruling in the Montenegro case was based mainly
purpose of staging, undertaking, waging and are actually
upon the Barcelon case, and hence, cannot have more weight
engaged in an armed insurrection and rebellion in order to than the same. Moreover, in the Barcelon case, the Court held
forcibly seize political power in this country, overthrow the duly
that it could go into the question: "Did the Governor-General"
constituted government, and supplant our existing political, — acting under the authority vested in him by the Congress of
social, economic and legal order with an entirely new one ...."
the United States, to suspend the privilege of the writ of habeas
Moreover, the third "whereas" in the original proclamation was, corpus under certain conditions — "act in conformance with
likewise, amended by alleging therein that said lawless such authority?" In other words, it did determine whether or not
elements, "by their acts of rebellion and insurrection," have the Chief Executive had acted in accordance with law.
created a state of lawlessness and disorder affecting public Similarly, in the Montenegro case, the Court held that petitioner
safety and the security of the State. In other words, apart from therein had "failed to overcome the presumption of correctness
adverting to the existence of actual conspiracy and of which the judiciary accords to acts of the Executive ...." In
the intent to rise in arms to overthrow the government,
short, the Court considered the question whether or not there
Proclamation No. 889-A asserts that the lawless elements really was are rebellion, as stated in the proclamation therein
"are actually engaged in an armed insurrection and rebellion"
contested.
to accomplish their purpose.

Incidentally, even the American jurisprudence is neither explicit


It may not be amiss to note, at this juncture, that the very tenor nor clear on the point under consideration. Although some
of the original proclamation and particularly, the circumstances
cases8 purport to deny the judicial power to "review" the
under which it had been issued, clearly suggest the intent to findings made in the proclamations assailed in said cases, the
aver that there was and is, actually, a state of rebellion in the tenor of the opinions therein given, considered as a whole,
Philippines, although the language of said proclamation was strongly suggests the court's conviction that the conditions
hardly a felicitous one, it having in effect, stressed the actuality essential for the validity of said proclamations or orders were,
of the intent to rise in arms, rather than of the factual existence in fact, present therein, just as the opposite view taken in other
of the rebellion itself. The pleadings, the oral arguments and cases9 had a backdrop permeated or characterized by the
the memoranda of respondents herein have consistently and belief that said conditions were absent. Hence, the dictum of
abundantly emphasized — to justify the suspension of the Chief Justice Taney to the effect that "(e)very case must
privilege of the writ of habeas corpus — the acts of violence
depend on its own circumstances." 10 One of the important, if
and subversion committed prior to August 21, 1971, by the not dominant, factors, in connection therewith, was intimated
lawless elements above referred to, and the conditions
in Sterling v. Constantin, 11 in which the Supreme Court of the
obtaining at the time of the issuance of the original United States, speaking through Chief Justice Hughes,
proclamation. In short, We hold that Proclamation No. 889-A declared that:
has superseded the original proclamation and that the flaws
attributed thereto are purely formal in nature.
FINALS CONSTITUTION I ACJUCO 221

.... When there is a substantial showing Manifestly, however, the liberty guaranteed and protected by
that the exertion of state power has our Basic Law is one enjoyed and exercised, not in derogation
overridden private rights secured by that thereof, but consistently therewith, and, hence, within the
Constitution, the subject is necessarily one framework of the social order established by the Constitution
for judicial inquiry in an appropriate and the context of the Rule of Law. Accordingly, when
proceeding directed against the individuals individual freedom is used to destroy that social order, by
charged with the transgression. To such a means of force and violence, in defiance of the Rule of Law —
case the Federal judicial power extends such as by rising publicly and taking arms against the
(Art. 3, sec. 2) and, so extending, the court government to overthrow the same, thereby committing the
has all the authority appropriate to its crime of rebellion — there emerges a circumstance that may
exercise. .... 12 warrant a limited withdrawal of the aforementioned guarantee
or protection, by suspending the privilege of the writ of habeas
corpus, when public safety requires it. Although we must be
In our resolution of October 5, 1971, We stated that "a majority
of the Court" had "tentatively arrived at a consensus that it may forewarned against mistaking mere dissent — no matter how
emphatic or intemperate it may be — for dissidence amounting
inquire in order to satisfy itself of the existence of the factual
bases for the issuance of Presidential Proclamations Nos. 889 to rebellion or insurrection, the Court cannot hesitate, much
and 889-A ... and thus determine the constitutional sufficiency less refuse — when the existence of such rebellion or
of such bases in the light of the requirements of Article III, sec. insurrection has been fairly established or cannot reasonably
1, par. 14, and Article VII, sec. 10, par 2, of the Philippine be denied — to uphold the finding of the Executive thereon,
Constitution...." Upon further deliberation, the members of the without, in effect, encroaching upon a power vested in him by
Court are now unanimous in the conviction that it has the the Supreme Law of the land and depriving him, to this extent,
of such power, and, therefore, without violating the Constitution
authority to inquire into the existence of said factual bases in
order to determine the constitutional sufficiency thereof. and jeopardizing the very Rule of Law the Court is called upon
to epitomize.

Indeed, the grant of power to suspend the privilege is neither


absolute nor unqualified. The authority conferred by the As heretofore adverted to, for the valid suspension of the
Constitution, both under the Bill of Rights and under the privilege of the writ: (a) there must be "invasion, insurrection or
rebellion" or — pursuant to paragraph (2), section 10 of Art. VII
Executive Department, is limited and conditional. The precept
in the Bill of Rights establishes a general rule, as well as an of the Constitution — "imminent danger thereof"; and (b) public
safety must require the aforementioned suspension. The
exception thereto. What is more, it postulates the former in
the negative, evidently to stress its importance, by providing President declared in Proclamation No. 889, as amended, that
that "(t)he privilege of the writ of habeas corpus shall not be both conditions are present.
suspended ...." It is only by way of exception that it permits the
suspension of the privilege "in cases of invasion, insurrection, As regards the first condition, our jurisprudence 14 attests
or rebellion" — or, under Art VII of the Constitution, "imminent abundantly to the Communist activities in the Philippines,
danger thereof" — "when the public safety requires it, in any of especially in Manila, from the late twenties to the early thirties,
which events the same may be suspended wherever during then aimed principally at incitement to sedition or rebellion, as
such period the necessity for such suspension shall the immediate objective. Upon the establishment of the
exist." 13 For from being full and plenary, the authority to Commonwealth of the Philippines, the movement seemed to
suspend the privilege of the writ is thus circumscribed, confined have waned notably; but, the outbreak of World War II in the
and restricted, not only by the prescribed setting or the Pacific and the miseries, the devastation and havoc, and the
conditions essential to its existence, but, also, as regards the proliferation of unlicensed firearms concomitant with the
time when and the place where it may be exercised. These military occupation of the Philippines and its subsequent
factors and the aforementioned setting or conditions mark, liberation, brought about, in the late forties, a resurgence of the
establish and define the extent, the confines and the limits of Communist threat, with such vigor as to be able to organize
said power, beyond which it does not exist. And, like the and operate in Central Luzon an army — called
limitations and restrictions imposed by the Fundamental Law HUKBALAHAP, during the occupation, and renamed Hukbong
upon the legislative department, adherence thereto and Mapagpalaya ng Bayan (HMP) after liberation — which
compliance therewith may, within proper bounds, be inquired clashed several times with the armed forces of the Republic.
into by courts of justice. Otherwise, the explicit constitutional This prompted then President Quirino to issue Proclamation
provisions thereon would be meaningless. Surely, the framers No. 210, dated October 22, 1950, suspending the privilege of
of our Constitution could not have intended to engage in such the writ of habeas corpus, the validity of which was upheld
a wasteful exercise in futility. in Montenegro v. Castañeda. 15 Days before the promulgation
of said Proclamation, or on October 18, 1950, members of the
Much less may the assumption be indulged in when we bear Communist Politburo in the Philippines were apprehended in
in mind that our political system is essentially democratic and Manila. Subsequently accused and convicted of the crime of
republican in character and that the suspension of the privilege rebellion, they served their respective sentences. 16
affects the most fundamental element of that system, namely,
individual freedom. Indeed, such freedom includes and The fifties saw a comparative lull in Communist activities,
connotes, as well as demands, the right of every single insofar as peace and order were concerned. Still, on June 20,
member of our citizenry to freely discuss and dissent from, as 1957, Rep. Act No. 1700, otherwise known as the Anti-
well as criticize and denounce, the views, the policies and the Subversion Act, was approved, upon the ground — stated in
practices of the government and the party in power that he the very preamble of said statute — that.
deems unwise, improper or inimical to the commonwealth,
regardless of whether his own opinion is objectively correct or ... the Communist Party of the Philippines,
not. The untrammelled enjoyment and exercise of such right — although purportedly a political party, is in
which, under certain conditions, may be a civic duty of the
fact an organized conspiracy to overthrow
highest order — is vital to the democratic system and essential the Government of the Republic of the
to its successful operation and wholesome growth and
Philippines, not only by force and violence
development. but also by deceit, subversion and other
illegal means, for the purpose of
establishing in the Philippines a totalitarian
FINALS CONSTITUTION I ACJUCO 222

regime subject to alien domination and In the year 1969, the NPA had — according to the records of
control; the Department of National Defense — conducted raids,
resorted to kidnappings and taken part in other violent
incidents numbering over 230, in which it inflicted 404
... the continued existence and activities of
the Communist Party of the Philippines casualties, and, in turn, suffered 243 losses. In 1970, its
constitutes a clear, records of violent incidents was about the same, but the NPA
present and grave danger to the security of casualties more than doubled.
the Philippines; 17 and
At any rate, two (2) facts are undeniable: (a) all Communists,
... in the face of the organized, systematic whether they belong to the traditional group or to the Maoist
and persistent subversion, national in faction, believe that force and violence are indispensable to the
scope but international in direction, posed attainment of their main and ultimate objective, and act in
by the Communist Party of the Philippines accordance with such belief, although they may disagree on
and its activities, there is urgent need for the means to be used at a given time and in a particular place;
special legislation to cope with this and (b) there is a New People's Army, other, of course, that the
continuing menace to the freedom and arm forces of the Republic and antagonistic thereto. Such New
People's Army is per se proof of the existence of a rebellion,
security of the country....
especially considering that its establishment was announced
publicly by the reorganized CPP. Such announcement is in the
In the language of the Report on Central Luzon, submitted, on nature of a public challenge to the duly constituted authorities
September 4, 1971, by the Senate Ad Hoc Committee of and may be likened to a declaration of war, sufficient to
Seven — copy of which Report was filed in these cases by the establish a war status or a condition of belligerency, even
petitioners herein — before the actual commencement of hostilities.

The years following 1963 saw the We entertain, therefore, no doubts about the existence of a
successive emergence in the country of sizeable group of men who have publicly risen in arms to
several mass organizations, notably the overthrow the government and have thus been and still are
Lapiang Manggagawa (now the Socialist engaged in rebellion against the Government of the
Party of the Philippines) among the Philippines.
workers; the Malayang Samahan ng mga
Magsasaka (MASAKA) among the
peasantry; the Kabataang Makabayan In fact, the thrust of petitioners' argument is that the New
(KM) among the youth/students; and the People's Army proper is too small, compared with the size of
Movement for the Advancement of the armed forces of the Government, that the Communist
Nationalism (MAN) among the rebellion or insurrection cannot so endanger public safety as to
intellectuals/professionals. The PKP has require the suspension of the privilege of the writ of habeas
corpus. This argument does not negate, however, the
exerted all-out effort to infiltrate, influence
and utilize these organizations in promoting existence of a rebellion, which, from the constitutional and
statutory viewpoint, need not be widespread or attain the
its radical brand of
nationalism. 18 magnitude of a civil war. This is apparent from the very
provision of the Revised Penal Code defining the crime of
rebellion, 20 which may be limited in its scope to "any part" of
Meanwhile, the Communist leaders in the Philippines had been the Philippines, and, also, from paragraph (14) of section 1,
split into two (2) groups, one of which — composed mainly of Article III of the Constitution, authorizing the suspension of the
young radicals, constituting the Maoist faction — reorganized privilege of the writ "wherever" — in case of rebellion — "the
the Communist Party of the Philippines early in 1969 and necessity for such suspension shall exist." In fact, the case of
established a New People's Army. This faction adheres to the Barcelon v. Baker referred to a proclamation suspending the
Maoist concept of the "Protracted People's War" or "War of privilege in the provinces of Cavite and Batangas only. The
National Liberation." Its "Programme for a People's Democratic case of In re Boyle 21involved a valid proclamation suspending
Revolution" states, inter alia: the privilege in a smaller area — a country of the state of Idaho.

The Communist Party of the Philippines is The magnitude of the rebellion has a bearing on the second
determined to implement its general condition essential to the validity of the suspension of the
programme for a people's democratic privilege — namely, that the suspension be required by public
revolution. All Filipino communists are safety. Before delving, however, into the factual bases of the
ready to sacrifice their lives for the worthy presidential findings thereon, let us consider the precise nature
cause of achieving the new type of of the Court's function in passing upon the validity of
democracy, of building a new Philippines Proclamation No. 889, as amended.
that is genuinely and completely
independent, democratic, united, just and
prosperous ... Article VII of the Constitution vests in the Executive the power
to suspend the privilege of the writ of habeas corpus under
specified conditions. Pursuant to the principle of separation of
xxx xxx xxx powers underlying our system of government, the Executive is
supreme within his own sphere. However, the separation of
The central task of any revolutionary powers, under the Constitution, is not absolute. What is more,
movement is to seize political power. The it goes hand in hand with the system of checks and balances,
Communist Party of the Philippines under which the Executive is supreme, as regards the
assumes this task at a time that both the suspension of the privilege, but only if and when he
international and national situations are acts within the sphere allotted to him by the Basic Law, and the
favorable of asking the road of armed authority to determine whether or not he has so acted is vested
revolution ... 19 in the Judicial Department, which, in this respect, is, in turn,
constitutionally supreme.
FINALS CONSTITUTION I ACJUCO 223

In the exercise of such authority, the function of the Court is demand that the test of the validity of acts of Congress and of
merely to check — not to supplant 22 — the Executive, or to those of the Executive be, mutatis mutandis, fundamentally the
ascertain merely whether he had gone beyond the same. Hence, counsel for petitioner Rogelio Arienda admits
constitutional limits of his jurisdiction, not to exercise the power that the proper standard is not correctness, but arbitrariness.
vested in him or to determine the wisdom of his act. To be sure,
the power of the Court to determine the validity of the contested Did public safety require the suspension of the privilege of the
proclamation is far from being identical to, or even comparable writ of habeas corpus decreed in Proclamation No. 889, as
with, its power over ordinary civil or criminal cases elevated amended? Petitioners submit a negative answer upon the
thereto by ordinary appeal from inferior courts, in which cases ground: (a) that there is no rebellion; (b) that, prior to and at the
the appellate court has all of the powers of the court of origin. time of the suspension of the privilege, the Government was
functioning normally, as were the courts; (c) that no untoward
Under the principle of separation of powers and the system of incident, confirmatory of an alleged July-August Plan, has
checks and balances, the judicial authority to review decisions actually taken place after August 21, 1971; (d) that the
of administrative bodies or agencies is much more limited, as President's alleged apprehension, because of said plan, is
regards findings of fact made in said decisions. Under the non-existent and unjustified; and (e) that the Communist forces
English law, the reviewing court determines only whether there in the Philippines are too small and weak to jeopardize public
is some evidentiary basis for the contested administrative safety to such extent as to require the suspension of the
findings; no quantitative examination of the supporting privilege of the writ of habeas corpus.
evidence is undertaken. The administrative findings can be
interfered with only if there is no evidence whatsoever in As above indicated, however, the existence of a rebellion is
support thereof, and said finding is, accordingly, arbitrary, obvious, so much so that counsel for several petitioners herein
capricious and obviously unauthorized. This view has been have admitted it.
adopted by some American courts. It has, likewise, been
adhered to in a number of Philippine cases. Other cases,
in bothjurisdictions, have applied the "substantial evidence" With respect to the normal operation of government, including
rule, which has been construed to mean "more than a mere courts, prior to and at the time of the suspension of the
scintilla" or "relevant evidence as a reasonable mind might privilege, suffice it to say that, if the conditions were such that
accept as adequate to support a conclusion," 23 even if other courts of justice no longer functioned, a suspension of the
minds equally reasonable might conceivably opine otherwise. privilege would have been unnecessary, there being no courts
to issue the writ of habeas corpus. Indeed, petitioners'
reference to the normal operation of courts as a factor
Manifestly, however, this approach refers to the review of indicative of the illegality of the contested act of the Executive
administrative determinations involving the exercise of quasi-
stems, perhaps, from the fact that this circumstance was
judicial functions calling for or entailing the reception of adverted to in some American cases to justify the invalidation
evidence. It does not and cannot be applied, in its aforesaid
therein decreed of said act of the Executive. Said cases
form, in testing the validity of an act of Congress or of the involved, however, the conviction by military courts of
Executive, such as the suspension of the privilege of the writ members of the civilian population charged
of habeas corpus, for, as a general rule, neither body takes with common crimes. It was manifestly, illegal for military
evidence — in the sense in which the term is used in judicial courts to assume jurisdiction over civilians so charged, when
proceedings — before enacting a legislation or suspending the civil courts were functioning normally.
writ. Referring to the test of the validity of a statute, the
Supreme Court of the United States, speaking through Mr.
Justice Roberts, expressed, in the leading case of Nebbia v. Then, too, the alleged absence of any untoward incident after
New York, 24 the view that: August 21, 1971, does not necessarily bear out petitioners'
view. What is more, it may have been due precisely to the
suspension of the privilege. To be sure, one of its logical effects
... If the laws passed are seen to have is to compel those connected with the insurrection or rebellion
a reasonable relation to a proper legislative to go into hiding. In fact, most of them could not be located by
purpose, and are neither arbitrary nor
the authorities, after August 21, 1971.
discriminatory, the requirements of due
process are satisfied, and judicial
determination to that effect renders a court The alleged July-August Plan to terrorize Manila is branded as
functus officio ... With the wisdom of the incredible, upon the theory that, according to Professor Egbal
policy adopted, with the adequacy or Ahman of Cornell University, "guerrilla use of terror ... is
practically of the law enacted to forward it, sociological and psychologically selective," and that the
the courts are indiscriminate resort to terrorism is bound to boomerang, for it
both incompetent and unauthorized to deal tends to alienate the people's symphaty and to deprive the
... dissidents of much needed mass support. The fact, however,
is that the violence used is some demonstrations held in Manila
in 1970 and 1971 tended to terrorize the bulk of its inhabitants.
Relying upon this view, it is urged by the Solicitor General — It would have been highly imprudent, therefore, for the
Executive to discard the possibility of a resort to terrorism, on
... that judicial inquiry into the basis of the a much bigger scale, under the July-August Plan.
questioned proclamation can go no
further than to satisfy the Court not that the We will now address our attention to petitioners' theory to the
President's decision is correct and that effect that the New People's Army of the Communist Party of
public safety was endanger by the rebellion the Philippines is too small to pose a danger to public safety of
and justified the suspension of the writ, but such magnitude as to require the suspension of the privilege
that in suspending the writ, the President
of the writ of habeas corpus. The flaw in petitioners' stand
did not act arbitrarily. becomes apparent when we consider that it assumes that the
Armed Forces of the Philippines have no other task than to
No cogent reason has been submitted to warrant the rejection fight the New People's Army, and that the latter is the only
of such test. Indeed, the co-equality of coordinate branches of threat — and a minor one — to our security. Such assumption
the Government, under our constitutional system, seems to is manifestly erroneous.
FINALS CONSTITUTION I ACJUCO 224

The records before Us show that, on or before August 21, (1) plane, and wounding one (1) soldier; that the NPA had in
1971, the Executive had information and reports — Central Luzon a total of four (4) encounters, with two (2) killed
subsequently confirmed, in many respects, by the and three (3) wounded on the side of the Government, one (1)
abovementioned Report of the Senate Ad-Hoc Committee of BSDU killed and three (3) NPA casualties; that in an encounter
Seven 25 — to the effect that the Communist Party of the at Botolan, Zambales, one (1) KM-SDK leader, an unidentified
Philippines does not merely adhere to Lenin's idea of a swift dissident, and Commander Panchito, leader of the dissident
armed uprising; that it has, also, adopted Ho Chi Minh's group were killed; that on August 26, 1971, there was an
terrorist tactics and resorted to the assassination of encounter in the barrio of San Pedro. Iriga City, Camarines
uncooperative local official; that, in line with this policy, the Sur, between the PC and the NPA, in which a PC and two (2)
insurgents have killed 5 mayors, 20 barrio captains and 3 KM members were killed; that the current disturbances in
chiefs of police; that there were fourteen (14) meaningful Cotabato and the Lanao provinces have been rendered more
bombing incidents in the Greater Manila Area in 1970; that the complex by the involvement of the CPP/NPA, for, in mid-1971,
Constitutional Convention Hall was bombed on June 12, 1971; a KM group, headed by Jovencio Esparagoza, contacted the
that, soon after the Plaza Miranda incident, the NAWASA main Higa-onan tribes, in their settlement in Magsaysay, Misamis
pipe, at the Quezon City-San Juan boundary, was bombed; Oriental, and offered them books, pamphlets and brochures of
that this was followed closely by the bombing of the Manila City Mao Tse Tung, as well as conducted teach-ins in the
Hall, the COMELEC building, the Congress Building and the reservation; that Esparagoza an operation of the PC in said
MERALCO substation at Cubao, Quezon City; and that the reservation; and that there are now two (2) NPA cadres in
respective residences of Senator Jose J. Roy and Mindanao.
Congressman Eduardo Cojuangco were, likewise, bombed, as
were the MERALCO main office premises, along Ortigas It should, also, be noted that adherents of the CPP and its front
Avenue, and the Doctor's Pharmaceuticals, Inc. Building, in
organizations are, according to intelligence findings, definitely
Caloocan City. capable of preparing powerful explosives out of locally
available materials; that the bomb used in the Constitutional
Petitioners, similarly, fail to take into account that — as per said Convention Hall was a "clay-more" mine, a powerful explosive
information and reports — the reorganized Communist Party device used by the U.S. Army, believed to have been one of
of the Philippines has, moreover, adopted Mao's concept of many pilfered from the Subic Naval Base a few days before;
protracted people's war, aimed at the paralyzation of the will to that the President had received intelligence information to the
resist of the government, of the political, economic and effect that there was a July-August Plan involving a wave of
intellectual leadership, and of the people themselves; that assassinations, kidnappings, terrorism and mass destruction
conformably to such concept, the Party has placed special of property and that an extraordinary occurence would signal
emphasis upon a most extensive and intensive program of the beginning of said event; that the rather serious condition of
subversion by the establishment of front organizations in urban peace and order in Mindanao, particularly in Cotabato and
centers, the organization of armed city partisans and the Lanao, demanded the presence therein of forces sufficient to
infiltration in student groups, labor unions, and farmer and cope with the situation; that a sizeable part of our armed forces
professional groups; that the CPP has managed to infiltrate or discharge other functions; and that the expansion of the CPP
establish and control nine (9) major labor organizations; that it activities from Central Luzon to other parts of the country,
has exploited the youth movement and succeeded in making particularly Manila and its suburbs, the Cagayan Valley,
Communist fronts of eleven (11) major student or youth Ifugao, Zambales, Laguna, Quezon and Bicol Region, required
organizations; that there are, accordingly, about thirty (30) that the rest of our armed forces be spread thin over a wide
mass organizations actively advancing the CPP interests, area.
among which are the Malayang Samahan ng Magsasaka
(MASAKA), the Kabataang Makabayan (KM), the Movement Considering that the President was in possession of the above
for the Advancement of Nationalism (MAN), the Samahang data — except those related to events that happened after
Demokratiko ng Kabataan (SDK), the Samahang Molave (SM) August 21, 1971 — when the Plaza Miranda bombing took
and the Malayang Pagkakaisa ng Kabataang Pilipino(MPKP); place, the Court is not prepared to hold that the Executive had
that, as of August, 1971, the KM had two hundred forty-five acted arbitrarily or gravely abused his discretion when he then
(245) operational chapters throughout the Philippines, of which
concluded that public safety and national security required the
seventy-three (73) were in the Greater Manila Area, sixty (60) suspension of the privilege of the writ, particularly if the NPA
in Northern Luzon, forty-nine (49) in Central Luzon, forty-two
were to strike simultaneously with violent demonstrations
(42) in the Visayas and twenty-one (21) in Mindanao and Sulu; staged by the two hundred forty-five (245) KM chapters, all
that in 1970, the Party had recorded two hundred fifty-eight over the Philippines, with the assistance and cooperation of the
(258) major demonstrations, of which about thirty-three (33) dozens of CPP front organizations, and the bombing or water
ended in violence, resulting in fifteen (15) killed and over five mains and conduits, as well as electric power plants and
hundred (500) injured; that most of these actions were installations — a possibility which, no matter how remote, he
organized, coordinated or led by the aforementioned front was bound to forestall, and a danger he was under obligation
organizations; that the violent demonstrations were generally
to anticipate and arrest.
instigated by a small, but well-trained group of armed agitators;
that the number of demonstrations heretofore staged in 1971
has already exceeded those of 1970; and that twenty-four (24) He had consulted his advisers and sought their views. He had
of these demonstrations were violent, and resulted in the death reason to feel that the situation was critical — as, indeed, it
of fifteen (15) persons and the injury of many more. was — and demanded immediate action. This he took
believing in good faith that public safety required it. And, in the
light of the circumstances adverted to above, he had
Subsequent events — as reported — have also proven that substantial grounds to entertain such belief.
petitioners' counsel have underestimated the threat to public
safety posed by the New People's Army. Indeed, it appears
that, since August 21, 1971, it had in Northern Luzon six (6) Petitioners insist that, nevertheless, the President had no
encounters and staged one (1) raid, in consequence of which authority to suspend the privilege in the entire Philippines, even
seven (7) soldiers lost their lives and two (2)others were if he may have been justified in doing so in some provinces or
wounded, whereas the insurgents suffered five (5) casualties; cities thereof. At the time of the issuance of Proclamation No.
that on August 26, 1971, a well-armed group of NPA, trained 889, he could not be reasonably certain, however, about the
by defector Lt. Victor Corpus, attacked the very command port placed to be excluded from the operation of the proclamation.
of TF LAWIN in Isabela, destroying two (2) helicopters and one He needed some time to find out how it worked, and as he did
FINALS CONSTITUTION I ACJUCO 225

so, he caused the suspension to be gradually lifted, first, on — together with over fifteen (15) other persons, who are, also,
September 18, 1971, in twenty-seven (27) provinces, three (3) at large — with another violation of said Act, in a criminal
sub-provinces and twenty six (26) cities; then, on September complaint filed with the City Fiscal's Office of Quezon City.
25, 1971, in order fourteen (14) provinces and thirteen (13)
cities; and, still later, on October 4, 1971, in seven (7) With respect to Vicente Ilao and Juan Carandang — petitioners
additional provinces and four (4) cities, or a total of forty-eight in L-33965 — who were released as early as August 31, 1971,
(48) provinces, three (3) sub-provinces and forth-three (43) as well as to petitioners Nemesio Prudente, Teodosio
cities, within a period of forty-five (45) days from August 21, Lansang, Rogelio Arienda, Antolin Oreta, Jr., Filomeno de
1971. Castro, Barcelisa C. de Castro, Reynaldo Rimando, Gerardo
Tomas and Bayani Alcala, who were released on November
Neither should We overlook the significance of another fact. 13, 1971, and are no longer deprived of their liberty, their
The President could have declared a general suspension of respective petitions have, thereby, become moot and
the privilege. Instead, Proclamation No. 889 limited the academic, as far as their prayer for release is concerned, and
suspension to persons detained "for crimes of insurrection or should, accordingly, be dismissed, despite the opposition
rebellion, and all other crimes and offenses committed by thereto of counsel for Nemesio Prudente and Gerardo Tomas
them in furtherance or on the occasion thereof, or incident who maintain that, as long as the privilege of the writ remains
thereto, or in connection therewith." Even this was further suspended, these petitioners might be arrested and detained
limited by Proclamation No. 889-A, which withdrew from the again, without just cause, and that, accordingly, the issue
coverage of the suspension persons detained for other crimes raised in their respective petitions is not moot. In any event, the
and offenses committed "on the occasion" of the insurrection common constitutional and legal issues raised in these cases
or rebellion, or "incident thereto, in or connection therewith." In have, in fact, been decided in this joint decision.
fact, the petitioners in L-33964, L-33982 and L-34004 concede
that the President had acted in good faith. Must we order the release of Rodolfo del Rosario, one of the
petitioners in
In case of invasion, insurrection or rebellion or imminent L-33964, Angelo de los Reyes, Victor Felipe and Teresito
danger thereof, the President has, under the Constitution, Sison, intervenors in L-33964, L-33965 and L-33973,
three (3) courses of action open to him, namely: (a) to call out Luzvimindo David, petitioner in L-33973, and Gary Olivar,
the armed forces; (b) to suspend the privilege of the writ petitioner in L-34339, who are still detained? The suspension
of habeas corpus; and (c) to place the Philippines or any part of the privilege of the writ was decreed by Proclamation No.
thereof under martial law. He had, already, called out the 889, as amended, for persons detained "for the crimes of
armed forces, which measure, however, proved inadequate to insurrection or rebellion and other overt acts committed by
attain the desired result. Of the two (2)other alternatives, the them in furtherance thereof."
suspension of the privilege is the least harsh.
The records shows that petitioners Luzvimindo David, Rodolfo
In view of the foregoing, it does not appear that the President del Rosario, Victor Felipe, Angelo de los Reyes, Teresito Sison
has acted arbitrary in issuing Proclamation No. 889, as and Gary Olivar are accused in Criminal Case No. Q-1623 of
amended, nor that the same is unconstitutional. the Court of First Instance of Rizal with a violation of the Anti-
Subversion Act and that the similar charge against petitioners
III Angelo de los Reyes and Teresito Sison in a criminal
complaint, originally filed with the City Fiscal of Quezon City,
has, also, been filed with said court. Do the offenses so
The next question for determination is whether petitioners charged constitute one of the crimes or overt acts mentioned
herein are covered by said Proclamation, as amended. In other in Proclamation No. 889, as amended?
words, do petitioners herein belong to the class of persons as
to whom privilege of the writ of habeas corpus has been
suspended? In the complaint in said Criminal Case No. 1623, it is alleged:

In this connection, it appears that Bayani Alcala, one of the That in or about the year 1968 and for
sometime prior thereto and thereafter up to
petitioners in L-33964, Gerardo Tomas, petitioner in L-34004,
and Reynaldo Rimando, petitioner in L-34013, were, on and including August 21, 1971, in the city
November 13, 1971, released "permanently" — meaning, of Quezon, Philippines, and elsewhere in
perhaps, without any intention to prosecute them — upon the the Philippines, within the jurisdiction of this
ground that, although there was reasonable ground to believe Honorable Court, the above-named
that they had committed an offense related to subversion, the accused knowingly, wilfully and by overt
evidence against them is insufficient to warrant their acts became officers and/or ranking
leaders of the Communist Party of the
prosecution; that Teodosio Lansang, one of the petitioners in
L-33964, Rogelio Arienda, petitioner in L-33965, Nemesio Philippines, a subversive association as
defined by Republic Act No. 1700, which
Prudente, petitioner in L-33982, Filomeno de Castro and
Barcelisa C. de Castro, for whose benefit the petition in L- is an organized conspiracy to overthrow
34039 was filed, and Antolin Oreta, Jr., petitioner in L-34265, the government of the Republic of the
were, on said date, "temporarily released"; that Rodolfo del Philippines by force, violence, deceit,
Rosario, one of the petitioners in subversion and other illegal means, for the
L-33964, Victor Felipe, an intervenor in L-33964, L-33965 and purpose of establishing in the Philippines a
L-33973, as well as Luzvimindo David, petitioner in L-33973, communist totalitarian regime subject to
and Gary Olivar, petitioner in L-34339, are still under detention alien domination and control;
and, hence, deprived of their liberty, they — together with over
forty (40) other persons, who are at large — having been That all the above-named accused, as
accused, in the Court of First Instance of Rizal, of a violation of such officers and/or ranking leaders of the
section 4 of Republic Act No. 1700 (Anti-Subversion Act); and Communist Party of the Philippines
that Angelo delos Reyes and Teresito Sison, intervenors in conspiring, confederating and mutual
said L-33964, L-33965 and helping one another, did then and there
L-33973, are, likewise, still detained and have been charged knowingly, wilfully, and feloniously and by
FINALS CONSTITUTION I ACJUCO 226

overt acts committed subversive acts all 3. Thru urban guerilla


intended to overthrow the government of warfare characterized
the Republic of the Philippines, as follows: by assassinations,
bombings, sabotage,
1. By rising publicly and kidnapping and arson,
taking arms against the intended to advertise
forces of the the movement, build up
government, engaging its morale and prestige,
in war against the discredit and
forces of the demoralize the
government, destroying authorities to use harsh
property or committing and repressive
measures, demoralize
serious violence,
exacting contributions the people and weaken
their confidence in the
or diverting public lands
or property from the law government and to
purposes for which they weaken the will of the
have been government to resist.
appropriated;
That the following aggravating
2. By engaging by circumstances attended the commission of
subversion thru the offense:
expansion and
requirement activities a. That the offense was committed in
not only of the contempt of and with insult to the public
Communist Party of the authorities;
Philippines but also of
the united front b. That some of the overt acts were
organizations of the committed in the Palace of the Chief
Communist Party of the Executive;
Philippines as the
Kabataang Makabayan
(KM), Movement for the c. That craft, fraud, or disguise was
Democratic Philippines employed;
(MDP), Samahang
Demokratikong d. That the offense was committed with the
Kabataan (SDK), aid of armed men;
Students' Alliance for
National Democracy
(STAND), MASAKA e. That the offense was committed with the
Olalia-faction, Student aid of persons under fifteen(15) years old.
Cultural Association of
the University of the Identical allegations are made in the complaint filed with the
Philippines (SCAUP), City Fiscal of Quezon City, except that the second paragraph
KASAMA, Pagkakaisa thereof is slightly more elaborate than that of the complaint filed
ng Magbubukid ng with the CFI, although substantially the same. 26
Pilipinas (PMP) and
many others; thru
agitation promoted by In both complaints, the acts imputed to the defendants herein
rallies, demonstration constitute rebellion and subversion, of — in the language of the
and strikes some of proclamation — "other overt acts committed ... in furtherance"
them violent in nature, of said rebellion, both of which are covered by the proclamation
intended to create suspending the privilege of the writ. It is clear, therefore, that
social discontent, the crime for which the detained petitioners are held and
discredit those in power deprived of their liberty are among those for which the privilege
and weaken the of the writ of habeas corpus has been suspended.
people's confidence in
the government; thru Up to this point, the Members of the Court are unanimous on
consistent propaganda the legal principles enunciated.
by publications, writing,
posters, leaflets of
After finding that Proclamation No. 889, as amended, is not
similar means;
invalid and that petitioners Luzvimindo David, Victor Felipe,
speeches, teach-ins,
Gary Olivar, Angelo de los Reyes, Rodolfo del Rosario and
messages, lectures or
Teresito Sison are detained for and actually accused of an
other similar means; or
offense for which the privilege of the writ has been suspended
thru the media as the
by said proclamation, our next step would have been the
TV, radio or
following: The Court, or a commissioner designated by it,
newspapers, all
would have received evidence on whether — as stated in
intended to promote the
respondents' "Answer and Return" — said petitioners had
Communist pattern of
been apprehended and detained "on reasonable belief" that
subversion;
they had "participated in the crime of insurrection or rebellion."
FINALS CONSTITUTION I ACJUCO 227

It is so happened, however, that on November 13, 1971 — or suspension of the privilege, it would be more reasonable to
two (2) days before the proceedings relative to the briefing held construe the filing of said formal charges with the court of first
on October 28 and 29, 1971, had been completed by the instance as an expression of the President's belief that there
filing 27 of the summary of the matters then taken up — the are sufficient evidence to convict the petitioners so charged
aforementioned criminal complaints were filed against said and that hey should not be released, therefore, unless and
petitioners. What is more, the preliminary examination and/or until said court — after conducting the corresponding
investigation of the charges contained in said complaints has preliminary examination and/or investigation — shall find that
already begun. The next question, therefore, is: Shall We now the prosecution has not established the existence of a probable
order, in the cases at hand, the release of said petitioners cause. Otherwise, the Executive would have released said
herein, despite the formal and substantial validity of the accused, as were the other petitioners herein;
proclamation suspending the privilege, despite the fact that
they are actually charged with offenses covered by said (c) From a long-range viewpoint, this interpretation — of the
proclamation and despite the aforementioned criminal
act of the President in having said formal charges filed — is,
complaints against them and the preliminary examination We believe, more beneficial to the detainees than that favored
and/or investigations being conducted therein?
by Mr. Justice Fernando. His view — particularly the theory that
the detainees should be released immediately, without bail,
The Members of the Court, with the exception of Mr. Justice even before the completion of said preliminary examination
Fernando, are of the opinion, and, so hold, that, instead of this and/or investigation — would tend to induce the Executive to
Court or its Commissioner taking the evidence adverted to refrain from filing formal charges as long as it may be possible.
above, it is best to let said preliminary examination and/or Manifestly, We should encourage the early filing of said
investigation to be completed, so that petitioners' released charges, so that courts of justice could assume jurisdiction over
could be ordered by the court of first instance, should it find the detainees and extend to them effective protection.
that there is no probable cause against them, or a warrant for
their arrest could be issued, should a probable cause be Although some of the petitioners in these cases pray that the
established against them. Such course of action is more Court decide whether the constitutional right to bail is affected
favorable to the petitioners, inasmuch as the preliminary by the suspension of the privilege of the writ of habeas corpus,
examination or investigation requires a greater quantum of We do not deem it proper to pass upon such question, the
proof than that needed to establish that the Executive had not
same not having been sufficiently discussed by the parties
acted arbitrary in causing the petitioners to be apprehended herein. Besides, there is no point in settling said question with
and detained upon the ground that they had participated in the
respect to petitioners herein who have been released. Neither
commission of the crime of insurrection or rebellion. And, it is is necessary to express our view thereon, as regards those still
mainly for the reason that the Court has opted to allow the detained, inasmuch as their release without bail might still be
Court of First Instance of Rizal to proceed with the decreed by the court of first instance, should it hold that there
determination of the existence of probable cause, although is no probable cause against them. At any rate, should an
ordinarily the Court would have merely determined the actual issue on the right to bail arise later, the same may be
existence of the substantial evidence of petitioners' connection brought up in appropriate proceedings.
with the crime of rebellion. Besides, the latter alternative would
require the reception of evidence by this Court and thus
duplicate the proceedings now taking place in the court of first WHEREFORE, judgment is hereby rendered:
instance. What is more, since the evidence involved in the
same proceedings would be substantially the same and the 1. Declaring that the President did not act arbitrarily in issuing
presentation of such evidence cannot be made Proclamation No. 889, as amended, and that, accordingly, the
simultaneously, each proceeding would tend to delay the same is not unconstitutional;
other.
2. Dismissing the petitions in L-33964, L-33965, L-33982, L-
Mr. Justice Fernando is of the opinion — in line with the view 34004, L-34013, L-34039 and L-34265, insofar as petitioners
of Mr. Justice Tuason, in Nava v. Gatmaitan, 28 to the effect Teodosio Lansang, Bayani Alcala, Rogelio Arienda,
that "... if and when formal complaint is presented, the court Vicentellao, Juan Carandang, Nemesio E. Prudente, Gerardo
steps in and the executive steps out. The detention ceases to Tomas, Reynaldo Rimando, Filomeno M. de Castro, Barcelisa
be an executive and becomes a judicial concern ..." — that the C. de Castro and Antolin Oreta, Jr. are concerned;
filing of the above-mentioned complaint against the six (6)
detained petitioners herein, has the effect of the Executive
giving up his authority to continue holding them pursuant to 3. The Court of First Instance of Rizal is hereby directed to act
Proclamation No. 889, as amended, even if he did not so with utmost dispatch in conducting the preliminary examination
intend, and to place them fully under the authority of courts of and/or investigation of the charges for violation of the Anti-
justice, just like any other person, who, as such, cannot be Subversion Act filed against herein petitioners Luzvimindo
deprived of his liberty without lawful warrant, which has not, as David, Victor Felipe, Gary Olivar, Angelo de los Reyes, Rodolfo
yet, been issued against anyone of them, and that, accordingly, del Rosario and Teresito Sison, and to issue the corresponding
We should order their immediate release. Despite the warrants of arrest, if probable cause is found to exist against
humanitarian and libertarian spirit with which this view had them, or, otherwise, to order their release; and
been espoused, the other Members of the Court are unable to
accept it because: 4. Should there be undue delay, for any reason whatsoever,
either in the completion of the aforementioned preliminary
(a) If the proclamation suspending the privilege of the writ examination and/or investigation, or in the issuance of the
of habeas corpus is valid — and We so hold it to be — and the proper orders or resolution in connection therewith, the parties
detainee is covered by the proclamation, the filing of a may by motion seek in these proceedings the proper relief.
complaint or information against him does not affect the
suspension of said privilege, and, consequently, his release 5. Without special pronouncement as to costs. It is so ordered.
may not be ordered by Us;

(b) Inasmuch as the filing of a formal complaint or information


does not detract from the validity and efficacy of the
FINALS CONSTITUTION I ACJUCO 228

[G.R. No. 158802. November 17, 2004] The trial court found petitioner guilty beyond reasonable
doubt of the crime of qualified rape, and sentenced him to
death, to indemnify the victim in the amount of P50,000.00, to
pay the costs of the suit and to support the child, Leahlyn
Mendoza.[7]
IN RE: THE WRIT OF HABEAS CORPUS FOR REYNALDO
DE VILLA (detained at the New Bilibid Prisons, On automatic review,[8] we found that the date of birth of
Muntinlupa City) REYNALDO DE Aileens child was medically consistent with the time of the
VILLA, petitioner, JUNE DE VILLA, petitioner- rape. Since it was never alleged that Aileen gave birth to a full-
relator, vs. THE DIRECTOR, NEW BILIBID term nine-month old baby, we gave credence to the
PRISONS, respondent. prosecutions contention that she prematurely gave birth to an
eight-month old baby by normal delivery.[9] Thus, we affirmed
DECISION petitioners conviction for rape, in a Decision the dispositive
portion of which reads:
YNARES-SANTIAGO, J.:
WHEREFORE, the judgment of the Regional Trial Court,
This is a petition for the issuance of a writ of habeas finding accused-appellant guilty beyond reasonable doubt of
corpus under Rule 102 of the Rules of Court. Petitioner the crime of rape, is AFFIRMED with the MODIFICATIONS
Reynaldo de Villa, joined by his son, petitioner-relator June de that he is sentenced to suffer the penalty of reclusin
Villa, seeks a two-fold relief: First, that respondent Director of perpetua and ordered to pay the offended party P50,000.00 as
Prisons justify the basis for the imprisonment of petitioner civil indemnity; P50,000.00 as moral damages; costs of the suit
Reynaldo de Villa; and second, that petitioner be granted a and to provide support for the child Leahlyn Corales Mendoza.
new trial.[1] These reliefs are sought on the basis of purportedly
exculpatory evidence, gathered after performing SO ORDERED.[10]
deoxyribonucleic acid (DNA) testing on samples allegedly
collected from the petitioner and a child born to the victim of
the rape. Three years after the promulgation of our Decision, we
are once more faced with the question of Reynaldo de Villas
By final judgment dated February 1, 2001, in People of guilt or innocence.
the Philippines v. Reynaldo de Villa,[2] we found petitioner
guilty of the rape of Aileen Mendoza, his niece by affinity; Petitioner-relator in this case, June de Villa, is the son of
sentenced him to suffer the penalty of reclusin perpetua; and Reynaldo. He alleges that during the trial of the case, he was
ordered him to pay the offended party civil indemnity, moral unaware that there was a scientific test that could determine
damages, costs of the suit, and support for Leahlyn Corales once and for all if Reynaldo was the father of the victims child,
Mendoza, the putative child born of the rape. Petitioner is Leahlyn. Petitioner-relator was only informed during the
currently serving his sentence at the New Bilibid Prison, pendency of the automatic review of petitioners case that DNA
Muntinlupa City. testing could resolve the issue of paternity. [11] This information
was apparently furnished by the Free Legal Assistance Group
As summarized in our Decision dated February 1, 2001, (FLAG) Anti-Death Penalty Task Force, which took over as
Aileen Mendoza charged petitioner Reynaldo de Villa with rape counsel for petitioner.
in an information dated January 9, 1995, filed with the Regional
Trial Court of Pasig City. When arraigned on January 26, 1995, Thus, petitioners brief in People v. de Villa sought the
petitioner entered a plea of not guilty.[3] conduct of a blood type test and DNA test in order to determine
the paternity of the child allegedly conceived as a result of the
During the trial, the prosecution established that rape.[12] This relief was implicitly denied in our Decision of
sometime in the third week of April 1994, at about 10:00 in the February 21, 2001.
morning, Aileen Mendoza woke up in her familys rented room
in Sagad, Pasig, Metro Manila, to find petitioner on top of her. On March 16, 2001, Reynaldo de Villa filed a Motion for
Aileen was then aged 12 years and ten months. She was Partial Reconsideration of the Decision, wherein he once more
unable to shout for help because petitioner covered her mouth prayed that DNA tests be conducted.[13] The Motion was
with a pillow and threatened to kill her. Aileen could not do denied with finality in a Resolution dated November 20,
anything but cry. Petitioner succeeded in inserting his penis 2001.[14] Hence, the Decision became final and executory on
inside her vagina. After making thrusting motions with his body, January 16, 2002.[15]
petitioner ejaculated. This encounter allegedly resulted in Petitioner-relator was undaunted by these challenges.
Aileens pregnancy, which was noticed by her mother, Leonila Having been informed that DNA tests required a sample that
Mendoza, sometime in November 1994. When confronted by could be extracted from saliva, petitioner-relator asked Billy
her mother, Aileen revealed that petitioner raped her. Aileens Joe de Villa, a grandson of Reynaldo de Villa and a classmate
parents then brought her to the Pasig Police Station, where of Leahlyn Mendoza, to ask Leahlyn to spit into a new, sterile
they lodged a criminal complaint against petitioner. [4] cup.[16] Leahlyn readily agreed and did so. Billy Joe took the
Dr. Rosaline Cosidon, who examined Aileen, confirmed sample home and gave it to the petitioner-relator, who
that she was eight months pregnant and found in her hymen immediately labeled the cup as Container A.
healed lacerations at the 5:00 and 8:00 positions. On Petitioner-relator then gathered samples from four
December 19, 1994, Aileen gave birth to a baby girl whom she grandchildren of Reynaldo de Villa. These samples were
named Leahlyn Mendoza.[5] placed in separate containers with distinguishing labels and
In his defense, petitioner alleged that, at the time of the temporarily stored in a refrigerator prior to transport to the DNA
alleged rape, he was already 67 years old. Old age and Analysis Laboratory at the National Science Research Institute
sickness had rendered him incapable of having an erection. He (NSRI).[17] During transport, the containers containing the
further averred that Aileens family had been holding a grudge saliva samples were kept on ice.
against him, which accounted for the criminal charges. Finally, Petitioner-relator requested the NSRI to conduct DNA
he interposed the defense of alibi, claiming that at the time of testing on the sample given by Leahlyn Mendoza, those given
the incident, he was in his hometown of San Luis, Batangas. [6] by the grandchildren of Reynaldo de Villa, and that given by
Reynaldo de Villa himself. The identities of the donors of the
FINALS CONSTITUTION I ACJUCO 229

samples, save for the sample given by Reynaldo de Villa, were availability as a post-conviction remedy. In the recent case
not made known to the DNA Analysis Laboratory. [18] of Feria v. Court of Appeals,[25] we ruled that review of a
judgment of conviction is allowed in a petition for the issuance
After testing, the DNA Laboratory rendered a preliminary of the writ of habeas corpus only in very specific instances,
report on March 21, 2003, which showed that Reynaldo de Villa such as when, as a consequence of a judicial proceeding, (a)
could not have sired any of the children whose samples were there has been a deprivation of a constitutional right resulting
tested, due to the absence of a match between the pertinent in the restraint of a person; (b) the court had no jurisdiction to
genetic markers in petitioners sample and those of any of the impose the sentence; or (c) an excessive penalty has been
other samples, including Leahlyns.[19] imposed, as such sentence is void as to such excess. [26]
Hence, in the instant petition for habeas corpus, In this instance, petitioner invokes the writ of habeas
petitioner argues as follows: corpus to assail a final judgment of conviction, without,
however, providing a legal ground on which to anchor his
DNA ANALYSIS ON PATERNITY SHOWS CONCLUSIVELY petition. In fine, petitioner alleges neither the deprivation of a
THAT PETITIONER DE VILLA IS NOT THE FATHER OF constitutional right, the absence of jurisdiction of the court
LEAHLYN MENDOZA; HIS CONVICTION FOR RAPE, imposing the sentence, or that an excessive penalty has been
BASED ON THE FACT THAT LEAHLYN WAS SIRED AS A imposed upon him.
RESULT OF THE ALLEGED RAPE, CANNOT STAND AND
MUST BE SET ASIDE.[20] In fine, petitioner invokes the remedy of habeas
corpus in order to seek the review of findings of fact long
passed upon with finality. This relief is far outside the scope
xxxxxxxxx of habeas corpusproceedings. In the early case of Abriol v.
Homeres,[27] for example, this Court stated the general rule that
A NEW TRIAL TO CONSIDER NEWLY DISCOVERED the writ of habeas corpus is not a writ of error, and should not
EVIDENCE IS PROPER AND MAY BE ORDERED BY THIS be thus used. The writ of habeas corpus, whereas permitting a
COURT IN VIEW OF THE RESULTS OF THE DNA TESTS collateral challenge of the jurisdiction of the court or tribunal
CONDUCTED.[21] issuing the process or judgment by which an individual is
deprived of his liberty, cannot be distorted by extending the
inquiry to mere errors of trial courts acting squarely within their
Considering that the issues are inter-twined, they shall be jurisdiction.[28] The reason for this is explained very simply in
discussed together. the case of Velasco v. Court of Appeals:[29] a habeas
corpus petition reaches the body, but not the record of the
In brief, petitioner relies upon the DNA evidence
case. [30] A record must be allowed to remain extant, and
gathered subsequent to the trial in order to re-litigate the
cannot be revised, modified, altered or amended by the simple
factual issue of the paternity of the child Leahlyn Mendoza.
expedient of resort to habeas corpus proceedings.
Petitioner alleges that this issue is crucial, considering that his
conviction in 2001 was based on the factual finding that he Clearly, mere errors of fact or law, which did not have the
sired the said child. Since this paternity is now conclusively effect of depriving the trial court of its jurisdiction over the case
disproved, he argues that the 2001 conviction must be and the person of the defendant, are not correctible in a petition
overturned. for the issuance of the writ of habeas corpus; if at all, these
errors must be corrected on certiorari or on appeal, in the form
In essence, petitioner invokes the remedy of the writ
and manner prescribed by law.[31] In the past, this Court has
of habeas corpus to collaterally attack the 2001 Decision. The
disallowed the review of a courts appreciation of the evidence
ancillary remedy of a motion for new trial is resorted to solely
in a petition for the issuance of a writ of habeas corpus, as this
to allow the presentation of what is alleged to be newly-
is not the function of said writ.[32] A survey of our decisions
discovered evidence. This Court is thus tasked to
in habeas corpus cases demonstrates that, in general, the writ
determine, first, the propriety of the issuance of a writ
of habeas corpus is a high prerogative writ which furnishes an
of habeas corpus to release an individual already convicted
extraordinary remedy; it may thus be invoked only under
and serving sentence by virtue of a final and executory
extraordinary circumstances.[33] We have been categorical in
judgment; and second, the propriety of granting a new trial
our pronouncements that the writ of habeas corpus is not to be
under the same factual scenario.
used as a substitute for another, more proper remedy. Resort
The extraordinary writ of habeas corpus has long been a to the writ of habeas corpus is available only in the limited
haven of relief for those seeking liberty from any unwarranted instances when a judgment is rendered by a court or tribunal
denial of freedom of movement. Very broadly, the writ applies devoid of jurisdiction. If, for instance, it can be demonstrated
to all cases of illegal confinement or detention by which a that there was a deprivation of a constitutional right, the writ
person has been deprived of his liberty, or by which the rightful can be granted even after an individual has been meted a
custody of any person has been withheld from the person sentence by final judgment.
entitled thereto.[22] Issuance of the writ necessitates that a
Thus, in the case of Chavez v. Court of Appeals,[34] the
person be illegally deprived of his liberty. In the celebrated
writ of habeas corpus was held to be available where an
case of Villavicencio v. Lukban,[23] we stated that [a]ny restraint
accused was deprived of the constitutional right against self-
which will preclude freedom of action is sufficient. [24]
incrimination. A defect so pronounced as the denial of an
The most basic criterion for the issuance of the writ, accuseds constitutional rights results in the absence or loss of
therefore, is that the individual seeking such relief be illegally jurisdiction, and therefore invalidates the trial and the
deprived of his freedom of movement or placed under some consequent conviction of the accused. That void judgment of
form of illegal restraint. If an individuals liberty is restrained via conviction may be challenged by collateral attack, which
some legal process, the writ of habeas corpus is unavailing. precisely is the function of habeas corpus.[35] Later,
Concomitant to this principle, the writ of habeas corpus cannot in Gumabon v. Director of the Bureau of Prisons,[36] this Court
be used to directly assail a judgment rendered by a competent ruled that, once a deprivation of a constitutional right is shown
court or tribunal which, having duly acquired jurisdiction, was to exist, the court that rendered the judgment is deemed ousted
not deprived or ousted of this jurisdiction through some of jurisdiction and habeas corpus is the appropriate remedy to
anomaly in the conduct of the proceedings. assail the legality of the detention.[37] Although in Feria v. Court
of Appeals[38] this Court was inclined to allow the presentation
Thus, notwithstanding its historic function as the great of new evidence in a petition for the issuance of a writ
writ of liberty, the writ of habeas corpus has very limited of habeas corpus, this was an exceptional situation. In that
FINALS CONSTITUTION I ACJUCO 230

case, we laid down the general rule, which states that the Court on September 14, 1999.[49] Petitioner alleged that his
burden of proving illegal restraint by the respondent rests on counsels withdrawal is an untimely and heartbreaking event,
the petitioner who attacks such restraint. Where the return is considering that he had placed all [his] trust and confidence on
not subject to exception, that is, where it sets forth a process [his counsels] unquestionable integrity and dignity. [50]
which, on its face, shows good ground for the detention of the
prisoner, it is incumbent on petitioner to allege and prove new While we are sympathetic to petitioners plight, we do not,
matter that tends to invalidate the apparent effect of such however, find that there was such negligence committed by his
process.[39] earlier counsel so as to amount to a denial of a constitutional
right. There is likewise no showing that the proceedings were
In the recent case of Calvan v. Court of Appeals,[40] we tainted with any other jurisdictional defect.
summarized the scope of review allowable in a petition for the
issuance of the writ of habeas corpus. We ruled that the writ In fine, we find that petitioner invokes the remedy of the
of habeas corpus, although not designed to interrupt the petition for a writ of habeas corpus to seek a re-examination of
orderly administration of justice, can be invoked by the the records of People v. de Villa, without asserting any legal
attendance of a special circumstance that requires immediate grounds therefor. For all intents and purposes, petitioner seeks
action. In such situations, the inquiry on a writ of habeas a reevaluation of the evidentiary basis for his conviction. We
corpus would be addressed, not to errors committed by a court are being asked to reexamine the weight and sufficiency of the
within its jurisdiction, but to the question of whether the evidence in this case, not on its own, but in light of the new
proceeding or judgment under which a person has been DNA evidence that the petitioner seeks to present to this
restrained is a complete nullity. The probe may thus proceed Court. This relief is outside the scope of a habeas
to check on the power and authority, itself an equivalent test of corpus petition. The petition for habeas corpus must,
jurisdiction, of the court or the judge to render the order that so therefore, fail.
serves as the basis of imprisonment or detention.[41] It is
Coupled with the prayer for the issuance of a writ
the nullity of an assailed judgment of conviction which makes
of habeas corpus, petitioner seeks a new trial to re-litigate the
it susceptible to collateral attack through the filing of a petition
issue of the paternity of the child Leahlyn Mendoza.
for the issuance of the writ of habeas corpus.
It must be stressed that the issue of Leahlyn Mendozas
Upon a perusal of the records not merely of this case but
paternity is not central to the issue of petitioners guilt or
of People v. de Villa, we find that the remedy of the writ
innocence. The rape of the victim Aileen Mendoza is an entirely
of habeas corpus is unavailing.
different question, separate and distinct from the question of
First, the denial of a constitutional right has not been the father of her child. Recently, in the case of People v.
alleged by petitioner. As such, this Court is hard-pressed to Alberio,[51] we ruled that the fact or not of the victims pregnancy
find legal basis on which to anchor the grant of a writ of habeas and resultant childbirth are irrelevant in determining whether or
corpus. Much as this Court sympathizes with petitioners plea, not she was raped. Pregnancy is not an essential element of
a careful scrutiny of the records does not reveal any the crime of rape. Whether the child which the victim bore was
constitutional right of which the petitioner was unduly deprived. fathered by the purported rapist, or by some unknown
individual, is of no moment in determining an individuals guilt.
We are aware that other jurisdictions have seen fit to
grant the writ of habeas corpus in order to test claims that a In the instant case, however, we note that the grant of
defendant was denied effective aid of counsel. [42] In this child support to Leahlyn Mendoza indicates that our Decision
instance, we note that the record is replete with errors was based, at least in small measure, on the victims claim that
committed by counsel, and it can be alleged that the petitioner the petitioner fathered her child. This claim was given credence
was, at trial, denied the effective aid of counsel. The United by the trial court, and, as a finding of fact, was affirmed by this
States Supreme Court requires a defendant alleging Court on automatic review.
incompetent counsel to show that the attorneys performance
The fact of the childs paternity is now in issue, centrally
was deficient under a reasonable standard, and additionally to
relevant to the civil award of child support. It is only tangentially
show that the outcome of the trial would have been different
related to the issue of petitioners guilt. However, if it can be
with competent counsel.[43] The purpose of the right to effective
conclusively determined that the petitioner did not sire Leahlyn
assistance of counsel is to ensure that the defendant receives
Mendoza, this may cast the shadow of reasonable doubt, and
a fair trial.[44]
allow the acquittal of the petitioner on this basis.
The U.S. Supreme Court asserts that in judging any
Be that as it may, it appears that the petitioner once more
claim of ineffective assistance of counsel, one must examine
relies upon erroneous legal grounds in resorting to the remedy
whether counsels conduct undermined the proper functioning
of a motion for new trial. A motion for new trial, under the
of the adversarial process to such an extent that the trial did
Revised Rules of Criminal Procedure, is available only for a
not produce a fair and just result.[45] The proper measure of
limited period of time, and for very limited grounds. Under
attorney performance is reasonable under the prevailing
Section 1, Rule 121, of the Revised Rules of Criminal
professional norms, and the defendant must show that the
Procedure, a motion for new trial may be filed at any time
representation received fell below the objective standard of
before a judgment of conviction becomes final, that is, within
reasonableness.[46] For the petition to succeed, the strong
fifteen (15) days from its promulgation or notice. Upon finality
presumption that the counsels conduct falls within the wide
of the judgment, therefore, a motion for new trial is no longer
range or reasonable professional assistance must be
an available remedy. Section 2 of Rule 121 enumerates the
overcome.[47]
grounds for a new trial:
In the case at bar, it appears that in the middle of the
appeal, the petitioners counsel of record, a certain Atty. SEC. 2. Grounds for a new trial.The court shall grant a new
Alfonso G. Salvador, suddenly and inexplicably withdrew his trial on any of the following grounds:
appearance as counsel, giving the sole explanation that he
was leaving for the United States for an indefinite period of time
by virtue of a petition filed in his favor. [48] In the face of this (a) That errors of law or irregularities prejudicial to the
abandonment, petitioner made an impassioned plea that his substantial rights of the accused have been committed during
lawyer be prevented from this withdrawal in a handwritten the trial;
Urgent Motion for Reconsideration and Opposition of Counsels
Withdrawal of Appearance with Leave of Court received by this
FINALS CONSTITUTION I ACJUCO 231

(b) That new and material evidence has been discovered which
the accused could not with reasonable diligence have
discovered and produced at the trial and which if introduced
and admitted would probably change the judgment.

In the case at bar, petitioner anchors his plea on the


basis of purportedly newly-discovered evidence, i.e., the DNA
test subsequently conducted, allegedly excluding petitioner
from the child purportedly fathered as a result of the rape.
The decision sought to be reviewed in this petition for the
issuance of a writ of habeas corpus has long attained finality,
and entry of judgment was made as far back as January 16,
2002. Moreover, upon an examination of the evidence
presented by the petitioner, we do not find that the DNA
evidence falls within the statutory or jurisprudential definition of
newly- discovered evidence.
A motion for new trial based on newly-discovered
evidence may be granted only if the following requisites are
met: (a) that the evidence was discovered after trial; (b) that
said evidence could not have been discovered and produced
at the trial even with the exercise of reasonable diligence; (c)
that it is material, not merely cumulative, corroborative or
impeaching; and (d) that the evidence is of such weight that
that, if admitted, it would probably change the judgment.[52] It is
essential that the offering party exercised reasonable diligence
in seeking to locate the evidence before or during trial but
nonetheless failed to secure it.[53]
In this instance, although the DNA evidence was
undoubtedly discovered after the trial, we nonetheless find that
it does not meet the criteria for newly-discovered evidence that
would merit a new trial. Such evidence disproving paternity
could have been discovered and produced at trial with the
exercise of reasonable diligence.
Petitioner-relators claim that he was unaware of the
existence of DNA testing until the trial was concluded carries
no weight with this Court. Lack of knowledge of the existence
of DNA testing speaks of negligence, either on the part of
petitioner, or on the part of petitioners counsel. In either
instance, however, this negligence is binding upon petitioner.
It is a settled rule that a party cannot blame his counsel for
negligence when he himself was guilty of neglect.[54] A client is
bound by the acts of his counsel, including the latters mistakes
and negligence.[55] It is likewise settled that relief will not be
granted to a party who seeks to be relieved from the effects of
the judgment when the loss of the remedy at law was due to
his own negligence, or to a mistaken mode of procedure.[56]
Even with all of the compelling and persuasive scientific
evidence presented by petitioner and his counsel, we are not
convinced that Reynaldo de Villa is entitled to outright acquittal.
As correctly pointed out by the Solicitor General, even if it is
conclusively proven that Reynaldo de Villa is not the father of
Leahlyn Mendoza, his conviction could, in theory, still stand,
with Aileen Mendozas testimony and positive identification as
its bases.[57] The Solicitor General reiterates, and correctly so,
that the pregnancy of the victim has never been an element of
the crime of rape.[58] Therefore, the DNA evidence has failed to
conclusively prove to this Court that Reynaldo de Villa should
be discharged. Although petitioner claims that conviction was
based solely on a finding of paternity of the child Leahlyn, this
is not the case. Our conviction was based on the clear and
convincing testimonial evidence of the victim, which, given
credence by the trial court, was affirmed on appeal.
WHEREFORE, in view of the foregoing, the instant
petition for habeas corpus and new trial is DISMISSED for lack
of merit.
No costs.
SO ORDERED.
FINALS CONSTITUTION I ACJUCO 232

GR No 171390
DAVID vs. ARROYO WHEREAS, over these past
DECISION months, elements in the political
opposition have conspired with
authoritarians of the extreme Left
SANDOVAL-GUTIERREZ, J.: represented by the NDF-CPP-NPA and
the extreme Right, represented by
military adventurists Ð the historical
All powers need some restraint; practical enemies of the democratic Philippine
adjustments rather than rigid formula are State Ð who are now in a tactical alliance
necessary.[1] Superior strength Ð the use of force Ð cannot and engaged in a concerted and
make wrongs into rights. In this regard, the courts should be systematic conspiracy, over a broad front,
vigilant in safeguarding the constitutional rights of the citizens, to bring down the duly constituted
specifically their liberty. Government elected in May 2004;

Chief Justice Artemio V. PanganibanÕs philosophy WHEREAS, these conspirators


of liberty is thus most relevant. He said: ÒIn cases involving have repeatedly tried to bring down the
liberty, the scales of justice should weigh heavily against President;
government and in favor of the poor, the oppressed, the
marginalized, the dispossessed and the weak.Ó Laws and WHEREAS, the claims of these
actions that restrict fundamental rights come to the courts elements have been recklessly
Òwith a heavy presumption against their constitutional magnified by certain segments of the
validity.Ó[2] national media;

WHEREAS, this series of actions


These seven (7) consolidated petitions for certiorari and is hurting the Philippine State Ð by
prohibition allege that in issuing Presidential Proclamation No. obstructing governance
1017 (PP 1017) and General Order No. 5 (G.O. No. 5), including hindering the growth of the
President Gloria Macapagal-Arroyo committed grave abuse of economy and sabotaging the peopleÕs
discretion. Petitioners contend that respondent officials of the confidence in government and their
Government, in their professed efforts to defend and preserve faith in the future of this country;
democratic institutions, are actually trampling upon the very
freedom guaranteed and protected by the WHEREAS, these actions are
Constitution. Hence, such issuances are void for being adversely affecting the economy;
unconstitutional.
WHEREAS, these activities
give totalitarian forces of both the
Once again, the Court is faced with an age-old but extreme Left and extreme Right the
persistently modern problem. How does the Constitution of a opening to intensify their avowed aims
free people combine the degree of liberty, without which, law to bring down the democratic Philippine
becomes tyranny, with the degree of law, without which, liberty State;
becomes license?[3]
WHEREAS, Article 2, Section 4
of the our Constitution makes the defense
On February 24, 2006, as the nation celebrated the and preservation of the democratic
20th Anniversary of the Edsa People Power I, President Arroyo institutions and the State the primary duty
issued PP 1017 declaring a state of national emergency, thus: of Government;
NOW, THEREFORE, I, Gloria WHEREAS, the activities above-
Macapagal-Arroyo, President of the described, their consequences,
Republic of the Philippines and ramifications and collateral effects
Commander-in-Chief of the Armed Forces constitute a clear and present danger to
of the Philippines, by virtue of the powers the safety and the integrity of the Philippine
vested upon me by Section 18, Article 7 of State and of the Filipino people;
the Philippine Constitution which states
that: ÒThe President. . . whenever it
becomes necessary, . . . may call out (the) On the same day, the President issued G. O. No. 5
armed forces to prevent or suppress. . implementing PP 1017, thus:
.rebellion. . .,Ó and in my capacity as their
Commander-in-Chief, do hereby WHEREAS, over these past months,
command the Armed Forces of the elements in the political opposition have
Philippines, to maintain law and order conspired with authoritarians of the
throughout the Philippines, prevent or extreme Left, represented by the NDF-
suppress all forms of lawless violence CPP-NPA and the extreme Right,
as well as any act of insurrection or represented by military adventurists - the
rebellion and to enforce obedience to all historical enemies of the democratic
the laws and to all decrees, orders and Philippine State Ð and who are now in a
regulations promulgated by me tactical alliance and engaged in a
personally or upon my direction; and as concerted and systematic conspiracy, over
provided in Section 17, Article 12 of the a broad front, to bring down the duly-
Constitution do hereby declare a State constituted Government elected in May
of National Emergency. 2004;
She cited the following facts as bases:
FINALS CONSTITUTION I ACJUCO 233

WHEREAS, these conspirators 24, 2006, was issued declaring a state of


have repeatedly tried to bring down our national emergency;
republican government;
WHEREAS, by virtue of General
WHEREAS, the claims of these Order No.5 and No.6 dated February 24,
elements have been recklessly magnified 2006, which were issued on the basis of
by certain segments of the national media; Proclamation No. 1017, the Armed
Forces of the Philippines (AFP) and the
WHEREAS, these series of Philippine National Police (PNP), were
actions is hurting the Philippine State by directed to maintain law and order
obstructing governance, including throughout the Philippines, prevent and
hindering the growth of the economy and suppress all form of lawless violence as
sabotaging the peopleÕs confidence in the well as any act of rebellion and to
government and their faith in the future of undertake such action as may be
this country; necessary;

WHEREAS, these actions are WHEREAS, the AFP and PNP


adversely affecting the economy; have effectively prevented, suppressed
and quelled the acts lawless violence
WHEREAS, these activities give and rebellion;
totalitarian forces; of both the extreme Left
and extreme Right the opening to intensify NOW, THEREFORE, I, GLORIA
their avowed aims to bring down the MACAPAGAL-ARROYO, President of
democratic Philippine State; the Republic of the Philippines, by virtue
of the powers vested in me by law,
WHEREAS, Article 2, Section 4 hereby declare that the state of
of our Constitution makes the defense and national emergency has ceased to
preservation of the democratic institutions exist.
and the State the primary duty of
Government;
In their presentation of the factual bases of PP 1017
WHEREAS, the activities above- and G.O. No. 5, respondents stated that the proximate cause
described, their consequences, behind the executive issuances was the conspiracy among
ramifications and collateral effects some military officers, leftist insurgents of the New PeopleÕs
constitute a clear and present danger to the Army (NPA), and some members of the political opposition in
safety and the integrity of the Philippine a plot to unseat or assassinate President Arroyo. [4] They
State and of the Filipino people; considered the aim to oust or assassinate the President and
take-over the reigns of government as a clear and present
WHEREAS, Proclamation 1017 danger.
date February 24, 2006 has been issued
declaring a State of National Emergency;
During the oral arguments held on March 7, 2006, the
NOW, THEREFORE, I GLORIA Solicitor General specified the facts leading to the issuance of
MACAPAGAL-ARROYO, by virtue of the PP 1017 and G.O. No. 5. Significantly, there was no
powers vested in me under the Constitution refutation from petitionersÕ counsels.
as President of the Republic of the
Philippines, and Commander-in-Chief of
the Republic of the Philippines, and The Solicitor General argued that the intent of the
pursuant to Proclamation No. 1017 dated Constitution is to give full discretionary powers to the
February 24, 2006, do hereby call upon the President in determining the necessity of calling out the armed
Armed Forces of the Philippines (AFP) and forces. He emphasized that none of the petitioners has
the Philippine National Police (PNP), to shown that PP 1017 was without factual bases. While he
prevent and suppress acts of terrorism and explained that it is not respondentsÕ task to state the facts
lawless violence in the country; behind the questioned Proclamation, however, they are
presenting the same, narrated hereunder, for the elucidation of
I hereby direct the Chief of Staff of the issues.
the AFP and the Chief of the PNP, as well On January 17, 2006, Captain Nathaniel Rabonza
as the officers and men of the AFP and and First Lieutenants Sonny Sarmiento, Lawrence
PNP, to immediately carry out the San Juan and Patricio Bumidang, members of the Magdalo
necessary and appropriate actions and Group indicted in the Oakwood mutiny, escaped their detention
measures to suppress and prevent acts cell in Fort Bonifacio, Taguig City. In a public statement, they
of terrorism and lawless violence. vowed to remain defiant and to elude arrest at all costs. They
called upon the people to Òshow and proclaim our displeasure
at the sham regime. Let us demonstrate our disgust, not only
On March 3, 2006, exactly one week after the by going to the streets in protest, but also by wearing red bands
declaration of a state of national emergency and after all these on our left arms.Ó [5]
petitions had been filed, the President lifted PP 1017. She
issued Proclamation No. 1021 which reads:
On February 17, 2006, the authorities got hold of a
WHEREAS, pursuant to document entitled ÒOplan Hackle I Ó which detailed plans for
Section 18, Article VII and Section 17, bombings and attacks during the Philippine Military Academy
Article XII of the Constitution, Alumni Homecoming in Baguio City. The plot was to
Proclamation No. 1017 dated February assassinate selected targets including some cabinet members
FINALS CONSTITUTION I ACJUCO 234

and President Arroyo herself.[6] Upon the advice of her political parties, coalitions, plus the groups that have been
security, President Arroyo decided not to attend the Alumni reinforcing since June 2005, it is probable that the PresidentÕs
Homecoming. The next day, at the height of the celebration, a ouster is nearing its concluding stage in the first half of 2006.
bomb was found and detonated at the PMA parade ground.

Respondents further claimed that the bombing of


On February 21, 2006, Lt. San Juan was recaptured telecommunication towers and cell sites in Bulacan and
in a communist safehouse in Batangas province. Found in his Bataan was also considered as additional factual basis for the
possession were two (2) flash disks containing minutes of the issuance of PP 1017 and G.O. No. 5. So is the raid of an army
meetings between members of the Magdalo Group and the outpost in Benguet resulting in the death of three (3)
National PeopleÕs Army (NPA), a tape recorder, audio soldiers. And also the directive of the Communist Party of the
cassette cartridges, diskettes, and copies of subversive Philippines ordering its front organizations to join 5,000 Metro
documents.[7] Prior to his arrest, Lt. San Juan announced Manila radicals and 25,000 more from the provinces in mass
through DZRH that the ÒMagdaloÕs D-Day would be on protests.[10]
February 24, 2006, the 20th Anniversary of Edsa I.Ó

By midnight of February 23, 2006, the President


On February 23, 2006, PNP Chief Arturo Lomibao convened her security advisers and several cabinet members
intercepted information that members of the PNP- Special to assess the gravity of the fermenting peace and order
Action Force were planning to defect. Thus, he immediately situation. She directed both the AFP and the PNP to account
ordered SAF Commanding General Marcelino Franco, Jr. for all their men and ensure that the chain of command remains
to ÒdisavowÓ any defection. The latter promptly obeyed and solid and undivided. To protect the young students from any
issued a public statement: ÒAll SAF units are under the possible trouble that might break loose on the streets, the
effective control of responsible and trustworthy officers with President suspended classes in all levels in the entire National
proven integrity and unquestionable loyalty.Ó Capital Region.

On the same day, at the house of former For their part, petitioners cited the events that
Congressman Peping Cojuangco, President Cory AquinoÕs followed after the issuance of PP 1017 and G.O. No. 5.
brother, businessmen and mid-level government officials
plotted moves to bring down the Arroyo administration. Nelly
Sindayen of TIME Magazine reported that Pastor Saycon, Immediately, the Office of the President announced
longtime Arroyo critic, called a U.S. government official about the cancellation of all programs and activities related to the
his groupÕs plans if President Arroyo is ousted. Saycon also 20th anniversary celebration of Edsa People Power I; and
phoned a man code-named Delta. Saycon identified him as revoked the permits to hold rallies issued earlier by the local
B/Gen. Danilo Lim, Commander of the ArmyÕs elite Scout governments. Justice Secretary Raul Gonzales stated that
Ranger. Lim said Òit was all systems go for the planned political rallies, which to the PresidentÕs mind were organized
movement against Arroyo.Ó[8] for purposes of destabilization, are cancelled. Presidential
Chief of Staff Michael Defensor announced that Òwarrantless
arrests and take-over of facilities, including media, can already
B/Gen. Danilo Lim and Brigade Commander Col. be implemented.Ó[11]
Ariel Querubin confided to Gen. Generoso Senga, Chief of
Staff of the Armed Forces of the Philippines (AFP), that a huge
number of soldiers would join the rallies to provide a critical Undeterred by the announcements that rallies and
mass and armed component to the Anti-Arroyo protests to be public assemblies would not be allowed, groups of protesters
held on February 24, 2005. According to these two (2) (members of Kilusang Mayo Uno [KMU] and National
officers, there was no way they could possibly stop the soldiers Federation of Labor Unions-Kilusang Mayo Uno [NAFLU-
because they too, were breaking the chain of command to join KMU]), marched from various parts of Metro Manila with the
the forces foist to unseat the President. However, Gen. intention of converging at the EDSA shrine. Those who were
Senga has remained faithful to his Commander-in-Chief and to already near the EDSA site were violently dispersed by huge
the chain of command. He immediately took custody of clusters of anti-riot police. The well-trained policemen used
B/Gen. Lim and directed Col. Querubin to return to the truncheons, big fiber glass shields, water cannons, and tear
Philippine Marines Headquarters in Fort Bonifacio. gas to stop and break up the marching groups, and scatter the
massed participants. The same police action was used against
the protesters marching forward to Cubao, Quezon City and to
Earlier, the CPP-NPA called for intensification of the corner of Santolan Street and EDSA. That same evening,
political and revolutionary work within the military and the hundreds of riot policemen broke up an EDSA celebration rally
police establishments in order to forge alliances with its held along Ayala Avenue and Paseo de Roxas Street in Makati
members and key officials. NPA spokesman Gregorio ÒKa City.[12]
RogerÓ Rosal declared: ÒThe Communist Party and
revolutionary movement and the entire people look forward to
the possibility in the coming year of accomplishing its According to petitioner Kilusang Mayo Uno, the
immediate task of bringing down the Arroyo regime; of police cited PP 1017 as the ground for the dispersal of their
rendering it to weaken and unable to rule that it will not take assemblies.
much longer to end it.Ó[9]
During the dispersal of the rallyists along EDSA,
On the other hand, Cesar Renerio, spokesman for police arrested (without warrant) petitioner Randolf S. David, a
the National Democratic Front (NDF) at North Central professor at the University of the Philippines and newspaper
Mindanao, publicly announced: ÒAnti-Arroyo groups within the columnist. Also arrested was his companion, Ronald Llamas,
military and police are growing rapidly, hastened by the president of party-list Akbayan.
economic difficulties suffered by the families of AFP officers
and enlisted personnel who undertake counter-insurgency
operations in the field.Ó He claimed that with the forces of the At around 12:20 in the early morning of February 25,
national democratic movement, the anti-Arroyo conservative 2006, operatives of the Criminal Investigation and Detection
FINALS CONSTITUTION I ACJUCO 235

Group (CIDG) of the PNP, on the basis of PP 1017 and G.O. turned over to the custody of the House of Representatives
No. 5, raided the Daily Tribune offices in Manila. The raiding where the ÒBatasan 5Ó decided to stay indefinitely.
team confiscated news stories by reporters, documents,
pictures, and mock-ups of the Saturday issue. Policemen
from Camp Crame in Quezon City were stationed inside the Let it be stressed at this point that the alleged
editorial and business offices of the newspaper; while violations of the rights of Representatives Beltran, Satur
policemen from the Manila Police District were stationed Ocampo, et al., are not being raised in these petitions.
outside the building.[13]
On March 3, 2006, President Arroyo issued PP 1021
A few minutes after the search and seizure at declaring that the state of national emergency has ceased to
the Daily Tribune offices, the police surrounded the premises exist.
of another pro-opposition paper, Malaya, and its sister
publication, the tabloid Abante.
In the interim, these seven (7) petitions challenging
the constitutionality of PP 1017 and G.O. No. 5 were filed with
The raid, according to Presidential Chief of Staff this Court against the above-named respondents. Three (3) of
Michael Defensor, is Òmeant to show a Ôstrong presence,Õ these petitions impleaded President Arroyo as respondent.
to tell media outlets not to connive or do anything that would
help the rebels in bringing down this government.Ó The PNP
warned that it would take over any media organization that In G.R. No. 171396, petitioners Randolf S. David, et al.
would not follow Òstandards set by the government during the assailed PP 1017 on the grounds that (1) it encroaches on the
state of national emergency.Ó Director General Lomibao emergency powers of Congress; (2) it is a subterfuge to avoid
stated that Òif they do not follow the standards Ð and the the constitutional requirements for the imposition of martial
standards are - if they would contribute to instability in the law; and (3) it violates the constitutional guarantees of freedom
government, or if they do not subscribe to what is in General of the press, of speech and of assembly.
Order No. 5 and Proc. No. 1017 Ð we will recommend a
Ôtakeover.ÕÓ National TelecommunicationsÕ
In G.R. No. 171409, petitioners Ninez Cacho-Olivares
Commissioner Ronald Solis urged television and radio
and Tribune Publishing Co., Inc. challenged the CIDGÕs act of
networks to ÒcooperateÓ with the government for the duration
raiding the Daily Tribune offices as a clear case of
of the state of national emergency. He asked for Òbalanced
ÒcensorshipÓ or Òprior restraint.Ó They also claimed that
reportingÓ from broadcasters when covering the events
the term ÒemergencyÓ refers only to tsunami, typhoon,
surrounding the coup attempt foiled by the government. He
hurricane and similar occurrences, hence, there is Òabsolutely
warned that his agency will not hesitate to recommend the
no emergencyÓ that warrants the issuance of PP 1017.
closure of any broadcast outfit that violates rules set out for
media coverage when the national security is threatened. [14]
In G.R. No. 171485, petitioners herein are
Representative Francis Joseph G. Escudero, and twenty one
Also, on February 25, 2006, the police arrested
(21) other members of the House of Representatives,
Congressman Crispin Beltran, representing
including Representatives Satur Ocampo, Rafael Mariano,
the Anakpawis Party and Chairman of Kilusang Mayo
Teodoro Casi–o, Liza Maza, and Josel Virador. They
Uno (KMU), while leaving his farmhouse in Bulacan. The
asserted that PP 1017 and G.O. No. 5 constitute Òusurpation
police showed a warrant for his arrest dated 1985. BeltranÕs
of legislative powersÓ; Òviolation of freedom of expressionÓ
lawyer explained that the warrant, which stemmed from a case
and Òa declaration of martial law.Ó They alleged that
of inciting to rebellion filed during the Marcos regime, had long
President Arroyo Ògravely abused her discretion in calling out
been quashed. Beltran, however, is not a party in any of these
the armed forces without clear and verifiable factual basis of
petitions.
the possibility of lawless violence and a showing that there is
necessity to do so.Ó
When members of petitioner KMU went to Camp
Crame to visit Beltran, they were told they could not be
In G.R. No. 171483, petitioners KMU, NAFLU-KMU,
admitted because of PP 1017 and G.O. No. 5. Two members
and their members averred that PP 1017 and G.O. No. 5 are
were arrested and detained, while the rest were dispersed by
unconstitutional because (1) they arrogate unto President
the police.
Arroyo the power to enact laws and decrees; (2) their issuance
was without factual basis; and (3) they violate freedom of
Bayan Muna Representative Satur Ocampo eluded expression and the right of the people to peaceably assemble
arrest when the police went after him during a public forum at to redress their grievances.
the Sulo Hotel in Quezon City. But his two drivers, identified
as Roel and Art, were taken into custody.
In G.R. No. 171400, petitioner Alternative Law
Groups, Inc. (ALGI) alleged that PP 1017 and G.O. No. 5 are
Retired Major General Ramon Monta–o, former head unconstitutional because they violate (a) Section 4[15] of
of the Philippine Constabulary, was arrested while with his wife Article II, (b) Sections 1,[16] 2,[17] and 4[18] of Article
and golfmates at the Orchard Golf and Country Club in III, (c) Section 23[19] of Article VI, and (d) Section 17[20] of
Dasmari–as, Cavite. Article XII of the Constitution.

Attempts were made to In G.R. No. 171489, petitioners Jose Anselmo I.


arrest Anakpawis Representative Satur Ocampo, Cadiz et al., alleged that PP 1017 is an Òarbitrary and unlawful
Representative Rafael Mariano, Bayan Muna Representative exercise by the President of her Martial Law powers.Ó And
Teodoro Casi–o and Gabriela Representative Liza assuming that PP 1017 is not really a declaration of Martial
Maza. Bayan Muna Representative Josel Virador was Law, petitioners argued that Òit amounts to an exercise by the
arrested at the PAL Ticket Office in Davao City. Later, he was President of emergency powers without congressional
approval.Ó In addition, petitioners asserted that PP 1017
FINALS CONSTITUTION I ACJUCO 236

Ògoes beyond the nature and function of a proclamation as inviolate the will of the people as
defined under the Revised Administrative Code.Ó expressed in the Constitution. This
power the courts exercise. This is the
beginning and the end of the theory of
And lastly, in G.R. No. 171424, petitioner Loren B. judicial review.[22]
Legarda maintained that PP 1017 and G.O. No. 5 are
Òunconstitutional for being violative of the freedom of But the power of judicial review does not repose upon the
expression, including its cognate rights such as freedom of the courts a Òself-starting capacity.Ó[23] Courts may exercise
press and the right to access to information on matters of public such power only when the following requisites are
concern, all guaranteed under Article III, Section 4 of the 1987 present: first, there must be an actual case or
Constitution.Ó In this regard, she stated that these issuances controversy; second, petitioners have to raise a question of
prevented her from fully prosecuting her election protest constitutionality; third, the constitutional question must be
pending before the Presidential Electoral Tribunal. raised at the earliest opportunity; and fourth, the decision of
the constitutional question must be necessary to the
determination of the case itself.[24]
In respondentsÕ Consolidated Comment, the Solicitor
General countered that: first, the petitions should be dismissed
for being moot; second, petitioners in G.R. Nos. Respondents maintain that the first and second
171400 (ALGI), 171424 (Legarda), 171483 (KMU et al.), requisites are absent, hence, we shall limit our discussion
171485 (Escudero et al.) and 171489 (Cadiz et al.) have no thereon.
legal standing; third, it is not necessary for petitioners to
implead President Arroyo as respondent; fourth, PP 1017 has
constitutional and legal basis; and fifth, PP 1017 does not An actual case or controversy involves a conflict of
violate the peopleÕs right to free expression and redress of legal right, an opposite legal claims susceptible of judicial
grievances. resolution. It is Òdefinite and concrete, touching the legal
relations of parties having adverse legal interest;Ó a real and
substantial controversy admitting of specific relief. [25] The
On March 7, 2006, the Court conducted oral arguments Solicitor General refutes the existence of such actual case or
and heard the parties on the above interlocking issues which controversy, contending that the present petitions were
may be summarized as follows: rendered Òmoot and academicÓ by President ArroyoÕs
issuance of PP 1021.
A. PROCEDURAL:
1) Whether the issuance of PP Such contention lacks merit.
1021 renders the petitions moot and
academic.
2) Whether petitioners A moot and academic case is one that ceases to
in 171485 (Escudero et al.), G.R. Nos. present a justiciable controversy by virtue of supervening
171400 (ALGI), 171483 (KMU et events,[26] so that a declaration thereon would be of no
al.), 171489 (Cadiz et al.), practical use or value.[27] Generally, courts decline jurisdiction
and 171424 (Legarda) have legal standing. over such case[28] or dismiss it on ground of mootness.[29]
B. SUBSTANTIVE:
1) Whether the Supreme Court
can review the factual bases of PP 1017. The Court holds that President ArroyoÕs issuance of
2) Whether PP 1017 and G.O. PP 1021 did not render the present petitions moot and
No. 5 are unconstitutional. academic. During the eight (8) days that PP 1017 was
a. Facial Challenge operative, the police officers, according to petitioners,
b. Constitutional Basis committed illegal acts in implementing it. Are PP 1017 and
c. As Applied Challenge G.O. No. 5 constitutional or valid? Do they justify these
alleged illegal acts? These are the vital issues that must be
A. PROCEDURAL resolved in the present petitions. It must be stressed that Òan
unconstitutional act is not a law, it confers no rights, it
First, we must resolve the procedural imposes no duties, it affords no protection; it is in legal
roadblocks. contemplation, inoperative.Ó[30]

I- Moot and Academic Principle The Òmoot and academicÓ principle is not a magical
formula that can automatically dissuade the courts in resolving
a case. Courts will decide cases, otherwise moot and
One of the greatest contributions of the American system academic, if: first, there is a grave violation of the
to this country is the concept of judicial review enunciated Constitution;[31] second, the exceptional character of the
in Marbury v. Madison.[21] This concept rests on the situation and the paramount public interest is
extraordinary simple foundation -- involved;[32] third, when constitutional issue raised requires
formulation of controlling principles to guide the bench, the bar,
and the public;[33] and fourth, the case is capable of repetition
The Constitution is the supreme law. yet evading review.[34]
It was ordained by the people, the ultimate
source of all political authority. It confers
limited powers on the national government. All the foregoing exceptions are present here and
x x x If the government consciously or justify this CourtÕs assumption of jurisdiction over the instant
unconsciously oversteps these petitions. Petitioners alleged that the issuance of PP 1017 and
limitations there must be some G.O. No. 5 violates the Constitution. There is no question that
authority competent to hold it in control, the issues being raised affect the publicÕs interest, involving
to thwart its unconstitutional attempt, as they do the peopleÕs basic rights to freedom of expression,
and thus to vindicate and preserve of assembly and of the press. Moreover, the Court has the
FINALS CONSTITUTION I ACJUCO 237

duty to formulate guiding and controlling constitutional However, to prevent just about any person from seeking
precepts, doctrines or rules. It has the symbolic function of judicial interference in any official policy or act with which he
educating the bench and the bar, and in the present disagreed with, and thus hinders the activities of governmental
petitions, the military and the police, on the extent of the agencies engaged in public service, the United State Supreme
protection given by constitutional guarantees. [35] And lastly, Court laid down the more stringent Òdirect injuryÓ test in Ex
respondentsÕ contested actions are capable of Parte Levitt,[42] later reaffirmed in Tileston v. Ullman.[43] The
repetition. Certainly, the petitions are subject to same Court ruled that for a private individual to invoke the
judicial review. judicial power to determine the validity of an executive or
legislative action, he must show that he has sustained a
direct injury as a result of that action, and it is not
In their attempt to prove the alleged mootness of this sufficient that he has a general interest common to all
case, respondents cited Chief Justice Artemio V. members of the public.
PanganibanÕs Separate Opinion in Sanlakas v. Executive
Secretary.[36] However, they failed to take into account the
Chief JusticeÕs very statement that an otherwise ÒmootÓ This Court adopted the Òdirect injuryÓ test in our
case may still be decided Òprovided the party raising it in a jurisdiction. In People v. Vera,[44] it held that the person who
proper case has been and/or continues to be prejudiced or impugns the validity of a statute must have Òa personal and
damaged as a direct result of its issuance.Ó The present case substantial interest in the case such that he has sustained,
falls right within this exception to the mootness rule pointed out or will sustain direct injury as a result.Ó The Vera doctrine
by the Chief Justice. was upheld in a litany of cases, such as, Custodio v. President
of the Senate,[45] Manila Race Horse TrainersÕ Association v.
De la Fuente,[46] Pascual v. Secretary of Public
II- Legal Standing Works[47] and Anti-Chinese League of the Philippines v.
Felix.[48]
In view of the number of petitioners suing in various
personalities, the Court deems it imperative to have a more However, being a mere procedural technicality, the
than passing discussion on legal standing or locus standi. requirement of locus standi may be waived by the Court in the
exercise of its discretion. This was done in the 1949
Locus standi is defined as Òa right of appearance in a Emergency Powers Cases, Araneta v. Dinglasan,[49] where
court of justice on a given question.Ó [37] In private suits, the Òtranscendental importanceÓ of the cases prompted the
standing is governed by the Òreal-parties-in interestÓ rule as Court to act liberally. Such liberality was neither a rarity nor
contained in Section 2, Rule 3 of the 1997 Rules of Civil accidental. In Aquino v. Comelec,[50] this Court resolved to
Procedure, as amended. It provides that Òevery action must pass upon the issues raised due to the Òfar-reaching
be prosecuted or defended in the name of the real party in implicationsÓ of the petition notwithstanding its categorical
interest.Ó Accordingly, the Òreal-party-in interestÓ is Òthe statement that petitioner therein had no personality to file the
party who stands to be benefited or injured by the suit. Indeed, there is a chain of cases where this liberal policy
judgment in the suit or the party entitled to the avails of has been observed, allowing ordinary citizens, members of
the suit.Ó[38] Succinctly put, the plaintiffÕs standing is based Congress, and civic organizations to prosecute actions
on his own right to the relief sought. involving the constitutionality or validity of laws, regulations
and rulings.[51]

The difficulty of determining locus standi arises in public


suits. Here, the plaintiff who asserts a Òpublic rightÓ in Thus, the Court has adopted a rule that even where the
assailing an allegedly illegal official action, does so as a petitioners have failed to show direct injury, they have been
representative of the general public. He may be a person who allowed to sue under the principle of Òtranscendental
is affected no differently from any other person. He could be importance.Ó Pertinent are the following cases:
suing as a Òstranger,Ó or in the category of a Òcitizen,Ó or
Ôtaxpayer.Ó In either case, he has to adequately show that (1) Chavez v. Public Estates
he is entitled to seek judicial protection. In other words, he Authority,[52] where the Court ruled that the
has to make out a sufficient interest in the vindication of the enforcement of the constitutional right
public order and the securing of relief as a ÒcitizenÓ or to information and the equitable
Òtaxpayer. diffusion of natural resources are
matters of transcendental importance
which clothe the petitioner with locus
Case law in most jurisdictions now allows both ÒcitizenÓ standi;
and ÒtaxpayerÓ standing in public actions. The distinction
was first laid down in Beauchamp v. Silk,[39] where it was held (2) Bagong Alyansang
that the plaintiff in a taxpayerÕs suit is in a different category Makabayan v. Zamora,[53] wherein the
from the plaintiff in a citizenÕs suit. In the former, the Court held that Ògiven the
plaintiff is affected by the expenditure of public funds, transcendental importance of the issues
while in the latter, he is but the mere instrument of the involved, the Court may relax the
public concern. As held by the New York Supreme Court standing requirements and allow the
in People ex rel Case v. Collins:[40] ÒIn matter of mere suit to prosper despite the lack of direct
public right, howeverÉthe people are the real partiesÉIt is injury to the parties seeking judicial
at least the right, if not the duty, of every citizen to interfere reviewÓ of the Visiting Forces Agreement;
and see that a public offence be properly pursued and
punished, and that a public grievance be
remedied.Ó With respect to taxpayerÕs suits, Terr v. (3) Lim v. Executive
Jordan[41] held that Òthe right of a citizen and a taxpayer to Secretary,[54] while the Court noted that the
maintain an action in courts to restrain the unlawful use of petitioners may not file suit in their capacity
public funds to his injury cannot be denied.Ó as taxpayers absent a showing that
ÒBalikatan 02-01Ó involves the exercise of
CongressÕ taxing or spending powers,
FINALS CONSTITUTION I ACJUCO 238

it reiterated its ruling in Bagong Now, the application of the above principles to the
Alyansang Makabayan v. Zamora,[55] that present petitions.
in cases of transcendental importance,
the cases must be settled promptly and
definitely and standing requirements The locus standi of petitioners in G.R. No. 171396,
may be relaxed. particularly David and Llamas, is beyond doubt. The same
holds true with petitioners in G.R. No. 171409, Cacho-Olivares
and Tribune Publishing Co. Inc. They alleged Òdirect injuryÓ
resulting from Òillegal arrestÓ and Òunlawful searchÓ
By way of summary, the following rules may be culled committed by police operatives pursuant to PP 1017. Rightly
from the cases decided by this Court. Taxpayers, voters, so, the Solicitor General does not question their legal standing.
concerned citizens, and legislators may be accorded standing
to sue, provided that the following requirements are met:
In G.R. No. 171485, the opposition Congressmen
alleged there was usurpation of legislative powers. They also
(1) the cases involve constitutional raised the issue of whether or not the concurrence of Congress
issues; is necessary whenever the alarming powers incident to Martial
Law are used. Moreover, it is in the interest of justice that
(2) for taxpayers, there must be a claim those affected by PP 1017 can be represented by their
of illegal disbursement of public funds or that Congressmen in bringing to the attention of the Court the
the tax measure is unconstitutional; alleged violations of their basic rights.

(3) for voters, there must be a showing of In G.R. No. 171400, (ALGI), this Court applied the
obvious interest in the validity of the election liberality rule in Philconsa v. Enriquez,[60] Kapatiran Ng Mga
law in question; Naglilingkod sa Pamahalaan ng Pilipinas, Inc. v.
Tan,[61] Association of Small Landowners in the Philippines,
(4) for concerned citizens, there must Inc. v. Secretary of Agrarian Reform,[62] Basco v. Philippine
be a showing that the issues raised are of Amusement and Gaming Corporation,[63] and Ta–ada v.
transcendental importance which must be Tuvera,[64] that when the issue concerns a public right, it is
settled early; and sufficient that the petitioner is a citizen and has an interest in
the execution of the laws.
(5) for legislators, there must be a claim
that the official action complained of infringes
upon their prerogatives as legislators. In G.R. No. 171483, KMUÕs assertion that PP 1017
and G.O. No. 5 violated its right to peaceful assembly may be
deemed sufficient to give it legal standing. Organizations
Significantly, recent decisions show a certain may be granted standing to assert the rights of their
toughening in the CourtÕs attitude toward legal standing. members.[65] We take judicial notice of the announcement by
the Office of the President banning all rallies and canceling all
permits for public assemblies following the issuance of PP
In Kilosbayan, Inc. v. Morato,[56] the Court ruled that the 1017 and G.O. No. 5.
status of Kilosbayan as a peopleÕs organization does not give
it the requisite personality to question the validity of the on-line
lottery contract, more so where it does not raise any issue of In G.R. No. 171489, petitioners, Cadiz et al., who are
constitutionality. Moreover, it cannot sue as a taxpayer absent national officers of the Integrated Bar of the Philippines (IBP)
any allegation that public funds are being misused. Nor can it have no legal standing, having failed to allege any direct or
sue as a concerned citizen as it does not allege any specific potential injury which the IBP as an institution or its members
injury it has suffered. may suffer as a consequence of the issuance of PP No. 1017
and G.O. No. 5. In Integrated Bar of the Philippines v.
Zamora,[66] the Court held that the mere invocation by the IBP
In Telecommunications and Broadcast Attorneys of the of its duty to preserve the rule of law and nothing more, while
Philippines, Inc. v. Comelec,[57] the Court reiterated the undoubtedly true, is not sufficient to clothe it with standing in
Òdirect injuryÓ test with respect to concerned citizensÕ cases this case. This is too general an interest which is shared by
involving constitutional issues. It held that Òthere must be a other groups and the whole citizenry. However, in view of the
showing that the citizen personally suffered some actual or transcendental importance of the issue, this Court declares
threatened injury arising from the alleged illegal official act.Ó that petitioner have locus standi.

In Lacson v. Perez,[58] the Court ruled that one of the In G.R. No. 171424, Loren Legarda has no
petitioners, Laban ng Demokratikong Pilipino (LDP), is not a personality as a taxpayer to file the instant petition as there are
real party-in-interest as it had not demonstrated any injury to no allegations of illegal disbursement of public funds. The fact
itself or to its leaders, members or supporters. that she is a former Senator is of no consequence. She can
no longer sue as a legislator on the allegation that her
prerogatives as a lawmaker have been impaired by PP 1017
In Sanlakas v. Executive Secretary,[59] the Court and G.O. No. 5. Her claim that she is a media personality will
ruled that only the petitioners who are members of Congress not likewise aid her because there was no showing that the
have standing to sue, as they claim that the PresidentÕs enforcement of these issuances prevented her from pursuing
declaration of a state of rebellion is a usurpation of the her occupation. Her submission that she has pending
emergency powers of Congress, thus impairing their electoral protest before the Presidential Electoral Tribunal is
legislative powers. As to petitioners Sanlakas, Partido likewise of no relevance. She has not sufficiently shown that
Manggagawa, and Social Justice Society, the Court declared PP 1017 will affect the proceedings or result of her case. But
them to be devoid of standing, equating them with the LDP considering once more the transcendental importance of the
in Lacson. issue involved, this Court may relax the standing rules.
FINALS CONSTITUTION I ACJUCO 239

It must always be borne in mind that the question constitutionally supreme.Ó[76] In 1973, the unanimous
of locus standi is but corollary to the bigger question of proper Court of Lansang was divided in Aquino v. Enrile.[77] There,
exercise of judicial power. This is the underlying legal tenet of the Court was almost evenly divided on the issue
the Òliberality doctrineÓ on legal standing. It cannot be of whether the validity of the imposition of
doubted that the validity of PP No. 1017 and G.O. No. 5 is a Martial Law is a political or justiciable question. [78] Then
judicial question which is of paramount importance to the came Garcia-Padilla v. Enrile which greatly
Filipino people. To paraphrase Justice Laurel, the whole of diluted Lansang. It declared that there is a need to re-examine
Philippine society now waits with bated breath the ruling of this the latter case, ratiocinating that Òin times of war or national
Court on this very critical matter. The petitions thus call for the emergency, the President must be given absolute control
application of the Òtranscendental importanceÓ doctrine, a for the very life of the nation and the government is in
relaxation of the standing requirements for the petitioners in great peril. The President, it intoned, is answerable only
the ÒPP 1017 cases.Ó to his conscience, the People, and God.Ó[79]

This Court holds that all the petitioners herein


have locus standi. The Integrated Bar of the Philippines v. Zamora[80] -
- a recent case most pertinent to these cases at bar -- echoed
Incidentally, it is not proper to implead President a principle similar to Lansang. While the Court considered the
Arroyo as respondent. Settled is the doctrine that the PresidentÕs Òcalling-outÓ power as a discretionary power
President, during his tenure of office or actual solely vested in his wisdom, it stressed that Òthis does not
incumbency,[67] may not be sued in any civil or criminal case, prevent an examination of whether such power was
and there is no need to provide for it in the Constitution or exercised within permissible constitutional limits or
law. It will degrade the dignity of the high office of the whether it was exercised in a manner constituting grave
President, the Head of State, if he can be dragged into court abuse of discretion.Ó This ruling is mainly a result of the
litigations while serving as such. Furthermore, it is important CourtÕs reliance on Section 1, Article VIII of 1987 Constitution
that he be freed from any form of harassment, hindrance or which fortifies the authority of the courts to determine in an
distraction to enable him to fully attend to the performance of appropriate action the validity of the acts of the political
his official duties and functions. Unlike the legislative and departments. Under the new definition of judicial power, the
judicial branch, only one constitutes the executive branch and courts are authorized not only Òto settle actual controversies
anything which impairs his usefulness in the discharge of the involving rights which are legally demandable and
many great and important duties imposed upon him by the enforceable,Ó but also Òto determine whether or not there
Constitution necessarily impairs the operation of the has been a grave abuse of discretion amounting to lack or
Government. However, this does not mean that the President excess of jurisdiction on the part of any branch or
is not accountable to anyone. Like any other official, he instrumentality of the government.Ó The latter part of the
remains accountable to the people[68] but he may be removed authority represents a broadening of judicial power to enable
from office only in the mode provided by law and that is by the courts of justice to review what was before a forbidden
impeachment.[69] territory, to wit, the discretion of the political departments of
the government.[81] It speaks of judicial prerogative not only in
terms of power but also of duty.[82]

B. SUBSTANTIVE As to how the Court may inquire into the PresidentÕs


exercise of power, Lansang adopted the test that Òjudicial
I. Review of Factual Bases inquiry can go no further than to satisfy the Court not that the
PresidentÕs decision is correct,Ó but that Òthe President did
not act arbitrarily.Ó Thus, the standard laid down is not
correctness, but arbitrariness.[83] In Integrated Bar of the
Petitioners maintain that PP 1017 has no factual
Philippines, this Court further ruled that Òit is incumbent
basis. Hence, it was not ÒnecessaryÓ for President Arroyo to
upon the petitioner to show that the PresidentÕs decision
issue such Proclamation.
is totally bereft of factual basisÓ and that if he fails, by way
of proof, to support his assertion, then Òthis Court cannot
The issue of whether the Court may review the undertake an independent investigation beyond the
factual bases of the PresidentÕs exercise of his Commander- pleadings.Ó
in-Chief power has reached its distilled point - from the
indulgent days of Barcelon v.
Petitioners failed to show that President ArroyoÕs
Baker[70] and Montenegro v. Castaneda[71] to
exercise of the calling-out power, by issuing PP 1017, is totally
the volatile era of Lansang v. Garcia,[72] Aquino, Jr. v. bereft of factual basis. A reading of the Solicitor GeneralÕs
Enrile, and Garcia-Padilla v. Enrile.[74] The tug-of-war
[73]
Consolidated Comment and Memorandum shows a detailed
always cuts across the line defining Òpolitical questions,Ó narration of the events leading to the issuance of PP 1017, with
particularly those questions Òin regard to which full
supporting reports forming part of the records. Mentioned are
discretionary authority has been delegated to the legislative or
the escape of the Magdalo Group, their audacious threat of
executive branch of the government.Ó[75] Barcelon and
the Magdalo D-Day, the defections in the military, particularly
Montenegro were in unison in declaring that the authority to
in the Philippine Marines, and the reproving statements from
decide whether an exigency has arisen belongs to the
the communist leaders. There was also the Minutes of the
President and his decision is final and conclusive on the
Intelligence Report and Security Group of the Philippine Army
courts. Lansang took the opposite view. There, the members
showing the growing alliance between the NPA and the
of the Court were unanimous in the conviction that the Court
military. Petitioners presented nothing to refute such
has the authority to inquire into the existence of factual bases events. Thus, absent any contrary allegations, the Court is
in order to determine their constitutional sufficiency. From the
convinced that the President was justified in issuing PP 1017
principle of separation of powers, it shifted the focus to calling for military aid.
the system of checks and balances, Òunder which the
President is supreme, x x x only if and when he acts within
the sphere allotted to him by the Basic Law, and the
authority to determine whether or not he has so acted is
vested in the Judicial Department, which in this Indeed, judging the seriousness of the incidents,
respect, is, in turn, President Arroyo was not expected to simply fold her arms and
FINALS CONSTITUTION I ACJUCO 240

do nothing to prevent or suppress what she believed was cases of extreme necessity, the assumption of absolute
lawless violence, invasion or rebellion. However, the exercise power in the form of a temporary dictatorship.Ó[88]
of such power or duty must not stifle liberty.

II. Constitutionality of PP 1017 and G.O. No. 5


Nicollo MachiavelliÕs view of emergency powers, as
Doctrines of Several Political Theorists one element in the whole scheme of limited government,
on the Power of the President furnished an ironic contrast to the Lockean theory of
in Times of Emergency prerogative. He recognized and attempted to bridge this
chasm in democratic political theory, thus:

This case brings to fore a contentious subject -- the


power of the President in times of emergency. A glimpse at Now, in a well-ordered society, it
the various political theories relating to this subject provides an should never be necessary to resort to
adequate backdrop for our ensuing discussion. extra Ðconstitutional measures; for
although they may for a time be beneficial,
yet the precedent is pernicious, for if the
John Locke, describing the architecture of civil practice is once established for good
government, called upon the English doctrine of prerogative to objects, they will in a little while be
cope with the problem of emergency. In times of danger to disregarded under that pretext but for evil
the nation, positive law enacted by the legislature might be purposes. Thus, no republic will ever be
inadequate or even a fatal obstacle to the promptness of action perfect if she has not by law provided for
necessary to avert catastrophe. In these situations, the Crown everything, having a remedy for every
retained a prerogative Òpower to act according to emergency and fixed rules for applying
discretion for the public good, without the proscription of it.[89]
the law and sometimes even against it.Ó[84] But Locke
recognized that this moral restraint might not suffice to avoid
abuse of prerogative powers. Who shall judge the need for Machiavelli Ð in contrast to Locke, Rosseau and Mill Ð
resorting to the prerogative and how may its abuse be sought to incorporate into the constitution a regularized system
avoided? Here, Locke readily admitted defeat, suggesting of standby emergency powers to be invoked with suitable
that Òthe people have no other remedy in this, as in all checks and controls in time of national danger. He attempted
other cases where they have no judge on earth, but to forthrightly to meet the problem of combining a capacious
appeal to Heaven.Ó[85] reserve of power and speed and vigor in its application in time
of emergency, with effective constitutional restraints. [90]

Jean-Jacques Rousseau also assumed the need for


temporary suspension of democratic processes of government
Contemporary political theorists, addressing
in time of emergency. According to him:
themselves to the problem of response to emergency by
constitutional democracies, have employed the doctrine of
The inflexibility of the laws, which constitutional dictatorship.[91] Frederick M. Watkins saw Òno
prevents them from adopting themselves to reason why absolutism should not be used as a means for
circumstances, may, in certain cases, the defense of liberal institutions,Ó provided it Òserves to
render them disastrous and make them protect established institutions from the danger of
bring about, at a time of crisis, the ruin of permanent injury in a period of temporary emergency and
the StateÉ is followed by a prompt return to the previous forms of
political life.Ó[92] He recognized the two (2) key elements of
It is wrong therefore to wish to the problem of emergency governance, as well as all
make political institutions as strong as to constitutional governance: increasing administrative
render it impossible to suspend their powers of the executive, while at the same
operation. Even Sparta allowed its law to time Òimposing limitation upon that power.Ó[93] Watkins
lapse... placed his real faith in a scheme of constitutional
dictatorship. These are the conditions of success of such a
If the peril is of such a kind that dictatorship: ÒThe period of dictatorship must be relatively
the paraphernalia of the laws are an shortÉDictatorship should always be strictly legitimate in
obstacle to their preservation, the method characterÉFinal authority to determine the need for
is to nominate a supreme lawyer, who shall dictatorship in any given case must never rest with the
silence all the laws and suspend for a dictator himselfÉÓ[94] and the objective of such an
moment the sovereign authority. In such a emergency dictatorship should be Òstrict political
case, there is no doubt about the general conservatism.Ó
will, and it clear that the peopleÕs first
intention is that the State shall not perish. [86]
Carl J. Friedrich cast his analysis in terms similar to
those of Watkins.[95] ÒIt is a problem of concentrating power
Rosseau did not fear the abuse of the emergency
Ð in a government where power has consciously been divided
dictatorship or Òsupreme magistracyÓ as he termed it. For
Ð to cope withÉ situations of unprecedented magnitude and
him, it would more likely be cheapened by Òindiscreet
gravity. There must be a broad grant of powers, subject to
use.Ó He was unwilling to rely upon an Òappeal to
equally strong limitations as to who shall exercise such powers,
heaven.Ó Instead, he relied upon a tenure of office of
when, for how long, and to what end.Ó[96] Friedrich, too,
prescribed duration to avoid perpetuation of the dictatorship.[87]
offered criteria for judging the adequacy of any of scheme of
emergency powers, to wit: ÒThe emergency executive must
be appointed by constitutional means Ð i.e., he must be
John Stuart Mill concluded his ardent defense of
legitimate; he should not enjoy power to determine the
representative government: ÒI am far from condemning, in
FINALS CONSTITUTION I ACJUCO 241

existence of an emergency; emergency powers should be dictatorship, like the decision to


exercised under a strict time limitation; and last, the institute one should never be in
objective of emergency action must be the defense of the the hands of the man or men who
constitutional order.Ó[97] constitute the dictator. . .

10) No constitutional
dictatorship should extend
Clinton L. Rossiter, after surveying the history of the beyond the termination of the
employment of emergency powers in Great Britain, France, crisis for which it was institutedÉ
Weimar, Germany and the United States, reverted to a
description of a scheme of Òconstitutional dictatorshipÓ as 11) Éthe termination of
solution to the vexing problems presented by the crisis must be followed by a
emergency.[98] Like Watkins and Friedrich, he stated a complete return as possible to the
priori the conditions of success of the Òconstitutional political and governmental
dictatorship,Ó thus: conditions existing prior to the
initiation of the constitutional
dictatorshipÉ[99]
1) No general regime or
particular institution of
constitutional dictatorship should Rossiter accorded to legislature a far greater role in the
be initiated unless it is necessary oversight exercise of emergency powers than did
or even indispensable to the Watkins. He would secure to Congress final responsibility for
preservation of the State and its declaring the existence or termination of an emergency, and
constitutional orderÉ he places great faith in the effectiveness of congressional
investigating committees.[100]
2) Éthe decision to Scott and Cotter, in analyzing the above contemporary
institute a constitutional theories in light of recent experience, were one in saying that,
dictatorship should never be in Òthe suggestion that democracies surrender the control
the hands of the man or men who of government to an authoritarian ruler in time of grave
will constitute the dictatorÉ danger to the nation is not based upon sound
constitutional theory.Ó To appraise emergency power in
3) No government terms of constitutional dictatorship serves merely to distort the
should initiate a constitutional problem and hinder realistic analysis. It matters not whether
dictatorship without making the term ÒdictatorÓ is used in its normal sense (as applied to
specific provisions for its authoritarian rulers) or is employed to embrace all chief
terminationÉ executives administering emergency powers. However used,
Òconstitutional dictatorshipÓ cannot be divorced from the
4) Éall uses of implication of suspension of the processes of
emergency powers and all constitutionalism. Thus, they favored instead the Òconcept of
readjustments in the organization constitutionalismÓ articulated by Charles H. McIlwain:
of the government should be
effected in pursuit of A concept of constitutionalism
constitutional or legal which is less misleading in the analysis of
requirementsÉ problems of emergency powers, and which
is consistent with the findings of this study,
5) É no dictatorial is that formulated by Charles H. McIlwain.
institution should be adopted, no While it does not by any means necessarily
right invaded, no regular exclude some indeterminate limitations
procedure altered any more than upon the substantive powers of
is absolutely necessary for the government, full emphasis is placed
conquest of the particular crisis . . upon procedural limitations,
. and political responsibility. McIlwain
clearly recognized the need to repose
6) The measures adequate power in government. And in
adopted in the prosecution of the discussing the meaning of
a constitutional dictatorship constitutionalism, he insisted that
should never be permanent in the historical and proper test of
character or effectÉ constitutionalism was the existence of
adequate processes for keeping
7) The dictatorship government responsible. He refused to
should be carried on by persons equate constitutionalism with the
representative of every part of the enfeebling of government by an
citizenry interested in the defense exaggerated emphasis upon separation of
of the existing constitutional powers and substantive limitations on
order. . . governmental power. He found that the
really effective checks on despotism have
8) Ultimate consisted not in the weakening of
responsibility should be government but, but rather in the limiting
maintained for every action taken of it; between which there is a great and
under a constitutional very significant difference. In associating
dictatorship. . . constitutionalism with ÒlimitedÓ as
distinguished from ÒweakÓ
9) The decision to government, McIlwain meant
terminate a constitutional government limited to the orderly
FINALS CONSTITUTION I ACJUCO 242

procedure of law as opposed to the and Òconstitutionally unprotected conduct.Ó In Broadrick v.


processes of force. The two Oklahoma,[105] it was held:
fundamental correlative elements of
constitutionalism for which all lovers of It remains a Ômatter of no little
liberty must yet fight are the legal limits difficultyÕ to determine when a law may
to arbitrary power and a complete properly be held void on its face and when
political responsibility of government to Ôsuch summary actionÕ is
the governed.[101] inappropriate. But the plain import of
our cases is, at the very least, that facial
overbreadth adjudication is an
exception to our traditional rules of
In the final analysis, the various approaches to practice and that its function, a limited
emergency of the above political theorists Ð- from LockÕs one at the outset, attenuates as the
Òtheory of prerogative,Ó to WatkinsÕ doctrine of otherwise unprotected behavior that it
Òconstitutional dictatorshipÓ and, eventually, to McIlwainÕs forbids the State to sanction moves
Òprinciple of constitutionalismÓ --- ultimately aim to solve one from Ôpure speechÕ toward
real problem in emergency governance, i.e., that of allotting conduct and that conduct Ðeven if
increasing areas of discretionary power to the Chief expressive Ð falls within the scope of
Executive, while insuring that such powers will be otherwise valid criminal laws that reflect
exercised with a sense of political responsibility and legitimate state interests in maintaining
under effective limitations and checks. comprehensive controls over harmful,
constitutionally unprotected conduct.
Our Constitution has fairly coped with this
problem. Fresh from the fetters of a repressive regime, the
1986 Constitutional Commission, in drafting the 1987 Thus, claims of facial overbreadth are entertained in
Constitution, endeavored to create a government in the cases involving statutes which, by their terms, seek to
concept of Justice JacksonÕs Òbalanced power regulate only Òspoken wordsÓ and again, that
structure.Ó[102] Executive, legislative, and judicial powers are Òoverbreadth claims, if entertained at all, have been
dispersed to the President, the Congress, and the Supreme curtailed when invoked against ordinary criminal laws that
Court, respectively. Each is supreme within its own are sought to be applied to protected conduct.Ó[106] Here,
sphere. But none has the monopoly of power in times of the incontrovertible fact remains that PP 1017 pertains to a
emergency. Each branch is given a role to serve as spectrum of conduct, not free speech, which is manifestly
limitation or check upon the subject to state regulation.
other. This system does not weaken the
President, it just limits his power, using the language of
McIlwain. In other words, in times of emergency, our Second, facial invalidation of laws is considered as
Constitution reasonably demands that we repose a certain Òmanifestly strong medicine,Ó to be used Òsparingly and
amount of faith in the basic integrity and wisdom of the Chief only as a last resort,Ó and is Ògenerally
Executive but, at the same time, it obliges him to operate disfavored;Ó[107] The reason for this is obvious. Embedded in
within carefully prescribed procedural limitations. the traditional rules governing constitutional adjudication is the
principle that a person to whom a law may be applied will not
a. ÒFacial ChallengeÓ be heard to challenge a law on the ground that it may
conceivably be applied unconstitutionally to others, i.e., in
other situations not before the Court.[108] A writer and
Petitioners contend that PP 1017 is void on its face scholar in Constitutional Law explains further:
because of its Òoverbreadth.Ó They claim that its enforcement
encroached on both unprotected and protected rights under The most distinctive feature of
Section 4, Article III of the Constitution and sent a Òchilling the overbreadth technique is that it
effectÓ to the citizens. marks an exception to some of the usual
rules of constitutional
A facial review of PP 1017, using the overbreadth litigation. Ordinarily, a particular
doctrine, is uncalled for. litigant claims that a statute is
unconstitutional as applied to him or
her; if the litigant prevails, the courts
First and foremost, the overbreadth doctrine is an carve away the unconstitutional aspects
analytical tool developed for testing Òon their facesÓ statutes of the law by invalidating its improper
in free speech cases, also known under the American Law as applications on a case to case
First Amendment cases.[103] basis. Moreover, challengers to a law
are not permitted to raise the rights of
third parties and can only assert their
A plain reading of PP 1017 shows that it is not own interests. In overbreadth analysis,
primarily directed to speech or even speech-related those rules give way; challenges are
conduct. It is actually a call upon the AFP to prevent or permitted to raise the rights of third
suppress all forms of lawless violence. In United States v. parties; and the court invalidates the entire
Salerno,[104] the US Supreme Court held that Òwe have not statute Òon its face,Ó not merely Òas
recognized an ÔoverbreadthÕ doctrine outside the limited applied forÓ so that the overbroad law
context of the First AmendmentÓ (freedom of speech). becomes unenforceable until a properly
authorized court construes it more
narrowly. The factor that motivates courts
Moreover, the overbreadth doctrine is not intended to depart from the normal adjudicatory
for testing the validity of a law that Òreflects legitimate state rules is the concern with the Òchilling;Ó
interest in maintaining comprehensive control over harmful, deterrent effect of the overbroad statute on
constitutionally unprotected conduct.Ó Undoubtedly, lawless third parties not courageous enough to
violence, insurrection and rebellion are considered ÒharmfulÓ bring suit. The Court assumes that an
FINALS CONSTITUTION I ACJUCO 243

overbroad lawÕs Òvery existence may suppress all forms of lawless violence as
cause others not before the court to refrain well any act of insurrection or rebellionÓ
from constitutionally protected speech or
expression.Ó An overbreadth ruling is Second provision:
designed to remove that deterrent effect on
the speech of those third parties.

In other words, a facial challenge using the Òand to enforce obedience to all
overbreadth doctrine will require the Court to examine PP 1017 the laws and to all decrees, orders and
and pinpoint its flaws and defects, not on the basis of its actual regulations promulgated by me personally
operation to petitioners, but on the assumption or prediction or upon my direction;Ó
that its very existence may cause others not before the
Court to refrain from constitutionally protected speech or
expression. In Younger v. Harris,[109] it was held that:

[T]he task of analyzing a Third provision:


proposed statute, pinpointing its
deficiencies, and requiring correction of
these deficiencies before the statute is put
into effect, is rarely if ever an appropriate
task for the judiciary. The combination of Òas provided in Section 17,
the relative remoteness of the Article XII of the Constitution do hereby
controversy, the impact on the declare a State of National Emergency.Ó
legislative process of the relief sought,
and above all the speculative and
amorphous nature of the required line-
by-line analysis of detailed First Provision: Calling-out Power
statutes,...ordinarily results in a kind of
case that is wholly unsatisfactory for
deciding constitutional questions, The first provision pertains to the PresidentÕs
whichever way they might be decided. calling-out power. In
Sanlakas v. Executive Secretary,[111] this Court, through Mr.
And third, a facial challenge on the ground of Justice Dante O. Tinga, held that Section 18, Article VII of the
overbreadth is the most difficult challenge to mount Constitution reproduced as follows:
successfully, since the challenger must establish that there
can be no instance when the assailed law may be Sec. 18. The President shall be the
valid. Here, petitioners did not even attempt to show whether Commander-in-Chief of all armed forces of
this situation exists. the Philippines and whenever it becomes
necessary, he may call out such armed
Petitioners likewise seek a facial review of PP 1017 forces to prevent or suppress lawless
on the ground of vagueness. This, too, is unwarranted. violence, invasion or rebellion. In case of
invasion or rebellion, when the public
Related to the ÒoverbreadthÓ doctrine is the Òvoid safety requires it, he may, for a period not
for vagueness doctrineÓ which holds that Òa law is facially exceeding sixty days, suspend the privilege
invalid if men of common intelligence must necessarily of the writ of habeas corpus or place the
guess at its meaning and differ as to its application.Ó[110] It Philippines or any part thereof under
is subject to the same principles governing overbreadth martial law. Within forty-eight hours from
doctrine. For one, it is also an analytical tool for testing Òon the proclamation of martial law or the
their facesÓ statutes in free speech cases. And like suspension of the privilege of the writ
overbreadth, it is said that a litigant may challenge a statute on of habeas corpus, the President shall
its face only if it is vague in all its possible applications. submit a report in person or in writing to the
Again, petitioners did not even attempt to show that PP Congress. The Congress, voting jointly, by
1017 is vague in all its application. They also failed to a vote of at least a majority of all its
establish that men of common intelligence cannot understand Members in regular or special session, may
the meaning and application of PP 1017. revoke such proclamation or suspension,
which revocation shall not be set aside by
the President. Upon the initiative of the
b. Constitutional Basis of PP 1017 President, the Congress may, in the same
manner, extend such proclamation or
suspension for a period to be determined
Now on the constitutional foundation of PP 1017. by the Congress, if the invasion or rebellion
shall persist and public safety requires it.
The operative portion of PP 1017 may be divided into
three important provisions, thus: The Congress, if not in session, shall
within twenty-four hours following such
First provision: proclamation or suspension, convene in
accordance with its rules without need of a
call.
Òby virtue of the power vested
upon me by Section 18, Artilce VII É do The Supreme Court may review, in
hereby command the Armed Forces of the an appropriate proceeding filed by any
Philippines, to maintain law and order citizen, the sufficiency of the factual bases
throughout the Philippines, prevent or of the proclamation of martial law or the
FINALS CONSTITUTION I ACJUCO 244

suspension of the privilege of the writ or the President ArroyoÕs declaration of a Òstate of rebellionÓ
extension thereof, and must promulgate its was merely an act declaring a status or condition of public
decision thereon within thirty days from its moment or interest, a declaration allowed under Section 4 cited
filing. above. Such declaration, in the words of Sanlakas, is
harmless, without legal significance, and deemed not
A state of martial law does not written. In these cases, PP 1017 is more than that. In
suspend the operation of the Constitution, declaring a state of national emergency, President Arroyo did
nor supplant the functioning of the civil not only rely on Section 18, Article VII of the Constitution, a
courts or legislative assemblies, nor provision calling on the AFP to prevent or suppress lawless
authorize the conferment of jurisdiction on violence, invasion or rebellion. She also relied on Section 17,
military courts and agencies over civilians Article XII, a provision on the StateÕs extraordinary power to
where civil courts are able to function, nor take over privately-owned public utility and business affected
automatically suspend the privilege of the with public interest. Indeed, PP 1017 calls for the exercise of
writ. an awesome power. Obviously, such Proclamation cannot
be deemed harmless, without legal significance, or not written,
The suspension of the privilege of the as in the case of Sanlakas.
writ shall apply only to persons judicially
charged for rebellion or offenses inherent in Some of the petitioners vehemently maintain that PP
or directly connected with invasion. 1017 is actually a declaration of Martial Law. It is no so. What
defines the character of PP 1017 are its wordings. It is plain
During the suspension of the privilege therein that what the President invoked was her calling-out
of the writ, any person thus arrested or power.
detained shall be judicially charged within
three days, otherwise he shall be released. The declaration of Martial Law is a Òwarn[ing] to citizens
that the military power has been called upon by the executive
to assist in the maintenance of law and order, and that, while
grants the President, as Commander-in-Chief, a ÒsequenceÓ the emergency lasts, they must, upon pain of arrest and
of graduated powers. From the most to the least benign, these punishment, not commit any acts which will in any way render
are: the calling-out power, the power to suspend the privilege more difficult the restoration of order and the enforcement of
of the writ of habeas corpus, and the power to declare Martial law.Ó[113]
Law. Citing Integrated Bar of the Philippines v.
Zamora,[112] the Court ruled that the only criterion for the In his ÒStatement before the Senate Committee on
exercise of the calling-out power is that Òwhenever it JusticeÓ on March 13, 2006, Mr. Justice Vicente V.
becomes necessary,Ó the President may call the armed Mendoza,[114] an authority in constitutional law, said that of the
forces Òto prevent or suppress lawless violence, invasion three powers of the President as Commander-in-Chief, the
or rebellion.Ó Are these conditions present in the instant power to declare Martial Law poses the most severe threat to
cases? As stated earlier, considering the circumstances then civil liberties. It is a strong medicine which should not be
prevailing, President Arroyo found it necessary to issue PP resorted to lightly. It cannot be used to stifle or persecute
1017. Owing to her OfficeÕs vast intelligence network, she is critics of the government. It is placed in the keeping of the
in the best position to determine the actual condition of the President for the purpose of enabling him to secure the people
country. from harm and to restore order so that they can enjoy their
individual freedoms. In fact, Section 18, Art. VII, provides:
Under the calling-out power, the President may summon
the armed forces to aid him in suppressing lawless violence, A state of martial law does not
invasion and rebellion. This involves ordinary police suspend the operation of the Constitution,
action. But every act that goes beyond the PresidentÕs nor supplant the functioning of the civil
calling-out power is considered illegal or ultra vires. For this courts or legislative assemblies, nor
reason, a President must be careful in the exercise of his authorize the conferment of jurisdiction on
powers. He cannot invoke a greater power when he wishes military courts and agencies over civilians
to act under a lesser power. There lies the wisdom of our where civil courts are able to function, nor
Constitution, the greater the power, the greater are the automatically suspend the privilege of the
limitations. writ.

It is pertinent to state, however, that there is a distinction


between the PresidentÕs authority to declare a Òstate of Justice Mendoza also stated that PP 1017 is not a
rebellionÓ (in Sanlakas) and the authority to proclaim a state declaration of Martial Law. It is no more than a call by the
of national emergency. While President ArroyoÕs authority to President to the armed forces to prevent or suppress lawless
declare a Òstate of rebellionÓ emanates from her powers as violence. As such, it cannot be used to justify acts that only
Chief Executive, the statutory authority cited in Sanlakas was under a valid declaration of Martial Law can be done. Its use
Section 4, Chapter 2, Book II of the Revised Administrative for any other purpose is a perversion of its nature and scope,
Code of 1987, which provides: and any act done contrary to its command is ultra vires.

SEC. 4. Ð Proclamations. Ð Justice Mendoza further stated that specifically, (a)


Acts of the President fixing a date arrests and seizures without judicial warrants; (b) ban on public
or declaring a status or condition assemblies; (c) take-over of news media and agencies and
of public moment or interest, press censorship; and (d) issuance of Presidential Decrees,
upon the existence of which the are powers which can be exercised by the President as
operation of a specific law or Commander-in-Chief only where there is a valid declaration of
regulation is made to depend, Martial Law or suspension of the writ of habeas corpus.
shall be promulgated in
proclamations which shall have Based on the above disquisition, it is clear that PP
the force of an executive order. 1017 is not a declaration of Martial Law. It is merely an
exercise of President ArroyoÕs calling-out power for the
FINALS CONSTITUTION I ACJUCO 245

armed forces to assist her in preventing or suppressing lawless


violence.
We all know that it was PP 1081 which granted
President Marcos legislative power. Its enabling clause
states: Òto enforce obedience to all the laws and decrees,
orders and regulations promulgated by me personally or
Second Provision: ÒTake CareÓ Power upon my direction.Ó Upon the other hand, the enabling
clause of PP 1017 issued by President Arroyo is: to enforce
The second provision pertains to the power of the obedience to all the laws and to all decrees, orders and
President to ensure that the laws be faithfully executed. This regulations promulgated by me personally or upon my
is based on Section 17, Article VII which reads: direction.Ó

Is it within the domain of President Arroyo to


SEC. 17. The President shall promulgate ÒdecreesÓ?
have control of all the executive
departments, bureaus, and offices. He PP 1017 states in
shall ensure that the laws be faithfully part: Òto enforce obedience to all the laws and decrees
executed. x x x promulgated by me personally or upon my
direction.Ó

The President is granted an Ordinance Power under


As the Executive in whom the executive power is Chapter 2, Book III of Executive Order No. 292 (Administrative
[115]
vested, the primary function of the President is to enforce Code of 1987). She may issue any of the following:
the laws as well as to formulate policies to be embodied in
existing laws. He sees to it that all laws are enforced by the
officials and employees of his department. Before assuming
Sec. 2. Executive Orders. Ñ Acts
office, he is required to take an oath or affirmation to the effect
of the President providing for rules of a
that as President of the Philippines, he will, among others,
general or permanent character in
Òexecute its laws.Ó[116] In the exercise of such function, the
implementation or execution of
President, if needed, may employ the powers attached to his
constitutional or statutory powers shall be
office as the Commander-in-Chief of all the armed forces of the
promulgated in executive orders.
country,[117] including the Philippine National Police[118] under
the Department of Interior and Local Government. [119] Sec. 3. Administrative Orders. Ñ
Acts of the President which relate to
particular aspect of governmental
Petitioners, especially Representatives Francis operations in pursuance of his duties as
Joseph G. Escudero, Satur Ocampo, Rafael Mariano, Teodoro administrative head shall be promulgated
Casi–o, Liza Maza, and Josel Virador argue that PP 1017 is in administrative orders.
unconstitutional as it arrogated upon President Arroyo the
power to enact laws and decrees in violation of Section 1, Sec. 4. Proclamations. Ñ Acts of
Article VI of the Constitution, which vests the power to enact the President fixing a date or declaring a
laws in Congress. They assail the clause Òto enforce status or condition of public moment or
obedience to all the laws and to all decrees, orders and interest, upon the existence of which the
regulations promulgated by me personally or upon my operation of a specific law or regulation is
direction.Ó made to depend, shall be promulgated in
proclamations which shall have the force of
\ an executive order.
Sec. 5. Memorandum Orders. Ñ
PetitionersÕ contention is understandable. A Acts of the President on matters of
reading of PP 1017 operative clause shows that it was administrative detail or of subordinate or
lifted[120] from Former President MarcosÕ Proclamation No. temporary interest which only concern a
1081, which partly reads: particular officer or office of the
Government shall be embodied in
memorandum orders.
NOW, THEREFORE, I,
FERDINAND E. MARCOS, President of Sec. 6. Memorandum Circulars.
the Philippines by virtue of the powers Ñ Acts of the President on matters relating
vested upon me by Article VII, Section 10, to internal administration, which the
Paragraph (2) of the Constitution, do President desires to bring to the attention
hereby place the entire Philippines as of all or some of the departments,
defined in Article 1, Section 1 of the agencies, bureaus or offices of the
Constitution under martial law and, in my Government, for information or
capacity as their Commander-in-Chief, do compliance, shall be embodied in
hereby command the Armed Forces of memorandum circulars.
the Philippines, to maintain law and
Sec. 7. General or Special
order throughout the Philippines,
Orders. Ñ Acts and commands of the
prevent or suppress all forms of lawless
President in his capacity as Commander-
violence as well as any act of
in-Chief of the Armed Forces of the
insurrection or rebellion and to enforce
Philippines shall be issued as general or
obedience to all the laws and decrees,
special orders.
orders and regulations promulgated by
me personally or upon my direction.
FINALS CONSTITUTION I ACJUCO 246

President ArroyoÕs ordinance power is limited to the What could be the reason of President Arroyo in
foregoing issuances. She cannot issue decrees similar to invoking the above provision when she issued PP 1017?
those issued by Former President Marcos under PP
1081. Presidential Decrees are laws which are of the same The answer is simple. During the existence of the
category and binding force as statutes because they were state of national emergency, PP 1017 purports to grant the
issued by the President in the exercise of his legislative power President, without any authority or delegation from Congress,
during the period of Martial Law under the 1973 to take over or direct the operation of any privately-owned
Constitution.[121] public utility or business affected with public interest.

This Court rules that the assailed PP 1017 is This provision was first introduced in the 1973
unconstitutional insofar as it grants President Arroyo the Constitution, as a product of the Òmartial lawÓ thinking of the
authority to promulgate Òdecrees.Ó Legislative power is 1971 Constitutional Convention.[122] In effect at the time of its
peculiarly within the province of the Legislature. Section 1, approval was President MarcosÕ Letter of Instruction No. 2
Article VI categorically states that Ò[t]he legislative power dated September 22, 1972 instructing the Secretary of
shall be vested in the Congress of the Philippines which National Defense to take over Òthe management, control and
shall consist of a Senate and a House of operation of the Manila Electric Company, the
Representatives.Ó To be sure, neither Martial Law nor a Philippine Long Distance Telephone Company, the National
state of rebellion nor a state of emergency can justify President Waterworks and Sewerage Authority, the Philippine National
ArroyoÕs exercise of legislative power by issuing decrees. Railways, the Philippine Air Lines, Air Manila (and) Filipinas
Orient Airways . . . for the successful prosecution by the
Can President Arroyo enforce obedience to all decrees Government of its effort to contain, solve and end the present
and laws through the military? national emergency.Ó

Petitioners, particularly the members of the House of


As this Court stated earlier, President Arroyo has no Representatives, claim that President ArroyoÕs inclusion of
authority to enact decrees. It follows that these decrees are Section 17, Article XII in PP 1017 is an encroachment on the
void and, therefore, cannot be enforced. With respect to legislatureÕs emergency powers.
Òlaws,Ó she cannot call the military to enforce or implement
certain laws, such as customs laws, laws governing family and This is an area that needs delineation.
property relations, laws on obligations and contracts and the
like. She can only order the military, under PP 1017, to enforce A distinction must be drawn between the PresidentÕs
laws pertinent to its duty to suppress lawless violence. authority to declare Òa state of national emergencyÓ
and to exercise emergency powers. To the first, as
elucidated by the Court, Section 18, Article VII grants the
President such power, hence, no legitimate constitutional
objection can be raised. But to the second, manifold
Third Provision: Power to Take Over constitutional issues arise.

The pertinent provision of PP 1017 states:


Section 23, Article VI of the Constitution reads:
x x x and to
enforce obedience to all SEC. 23. (1) The Congress, by a
the laws and to all vote of two-thirds of both Houses in joint
decrees, orders, and session assembled, voting separately,
regulations shall have the sole power to declare the
promulgated by me existence of a state of war.
personally or upon my
direction; and as (2) In times of war or other
provided in Section
national emergency, the Congress may,
17, Article XII of the by law, authorize the President, for a limited
Constitution do
period and subject to such restrictions as it
hereby declare a state
may prescribe, to exercise powers
of national
necessary and proper to carry out a
emergency.
declared national policy. Unless sooner
withdrawn by resolution of the Congress,
such powers shall cease upon the next
The import of this provision is that President Arroyo, adjournment thereof.
during the state of national emergency under PP 1017, can call
the military not only to enforce obedience Òto all the laws and
to all decrees x x xÓ but also to act pursuant to the provision
of Section 17, Article XII which reads: It may be pointed out that the second paragraph of the
above provision refers not only to war but also to Òother
Sec. 17. In times of national national emergency.Ó If the intention of the Framers of our
emergency, when the public interest so Constitution was to withhold from the President the authority to
requires, the State may, during the declare a Òstate of national emergencyÓ pursuant to Section
emergency and under reasonable terms 18, Article VII (calling-out power) and grant it to Congress (like
prescribed by it, temporarily take over or the declaration of the existence of a state of war), then the
direct the operation of any privately-owned Framers could have provided so. Clearly, they did not intend
public utility or business affected with that Congress should first authorize the President before he
public interest. can declare a Òstate of national emergency.Ó The logical
conclusion then is that President Arroyo could validly declare
the existence of a state of national emergency even in the
absence of a Congressional enactment.
FINALS CONSTITUTION I ACJUCO 247

that the Laws be faithfully executed;Ó and


But the exercise of emergency powers, such as the that he Òshall be Commander-in-Chief of
taking over of privately owned public utility or business affected the Army and Navy of the United States.
with public interest, is a
different matter. This requires a delegation from Congress. The order cannot properly be
sustained as an exercise of the
Courts have often said that constitutional provisions PresidentÕs military power as
in pari materia are to be construed together. Otherwise stated, Commander-in-Chief of the Armed
different clauses, sections, and provisions of a constitution Forces. The Government attempts to do
which relate to the same subject matter will be construed so by citing a number of cases upholding
together and considered in the light of each broad powers in military commanders
other.[123] Considering that Section 17 of Article XII and engaged in day-to-day fighting in a theater
Section 23 of Article VI, previously quoted, relate to national of war. Such cases need not concern us
emergencies, they must be read together to determine the here. Even though Òtheater of warÓ be
limitation of the exercise of emergency powers. an expanding concept, we cannot with
faithfulness to our constitutional
Generally, Congress is the repository of system hold that the Commander-in-
emergency powers. This is evident in the tenor of Section 23 Chief of the Armed Forces has the
(2), Article VI authorizing it to delegate such powers to the ultimate power as such to take
President. Certainly, a body cannot delegate a power not possession of private property in order
reposed upon it. However, knowing that during grave to keep labor disputes from stopping
emergencies, it may not be possible or practicable for production. This is a job for the
Congress to meet and exercise its powers, the Framers of our nationÕs lawmakers, not for its military
Constitution deemed it wise to allow Congress to grant authorities.
emergency powers to the President, subject to certain
conditions, thus: Nor can the seizure order be
sustained because of the several
constitutional provisions that grant
(1) There must be executive power to the President. In the
a war or other emergency. framework of our Constitution, the
PresidentÕs power to see that the laws
(2) The delegation must be for are faithfully executed refutes the idea
a limited period only. that he is to be a lawmaker. The
Constitution limits his functions in the
(3) The delegation must lawmaking process to the
be subject to such recommending of laws he thinks wise
restrictions as the and the vetoing of laws he thinks
Congress may bad. And the Constitution is neither
prescribe. silent nor equivocal about who shall
(4) The emergency powers must make laws which the President is to
be exercised to carry out execute. The first section of the first
a national article says that ÒAll legislative Powers
policy declared by herein granted shall be vested in a
Congress.[124] Congress of the United States. . .Ó[126]

Section 17, Article XII must be understood as an aspect Petitioner Cacho-Olivares, et al. contends that the
of the emergency powers clause. The taking over of private term ÒemergencyÓ under Section 17, Article XII refers to
business affected with public interest is just another facet of Òtsunami,Ó Òtyphoon,Ó ÒhurricaneÓ and Òsimilar
the emergency powers generally reposed upon occurrences.Ó This is a limited view of Òemergency.Ó
Congress. Thus, when Section 17 states that the Òthe State
may, during the emergency and under reasonable terms
prescribed by it, temporarily take over or direct the
operation of any privately owned public utility or business Emergency, as a generic term, connotes the
affected with public interest,Ó it refers to Congress, not the existence of conditions suddenly intensifying the degree of
President. Now, whether or not the President may exercise existing danger to life or well-being beyond that which is
such power is dependent on whether Congress may delegate accepted as normal. Implicit in this definitions are the
it to him pursuant to a law prescribing the reasonable terms elements of intensity, variety, and
thereof. Youngstown Sheet & Tube Co. et al. v. perception.[127] Emergencies, as perceived by legislature or
Sawyer,[125] held: executive in the United Sates since 1933, have been
occasioned by a wide range of situations, classifiable under
It is clear that if the President had three (3) principal heads: a) economic,[128] b) natural
authority to issue the order he did, it must disaster,[129] and c) national security.[130]
be found in some provision of the
Constitution. And it is not claimed that
express constitutional language grants this
power to the President. The contention is
that presidential power should be implied
from the aggregate of his powers under the ÒEmergency,Ó as contemplated in our Constitution,
Constitution. Particular reliance is placed is of the same breadth. It may include rebellion, economic
on provisions in Article II which say that crisis, pestilence or epidemic, typhoon, flood, or other similar
ÒThe executive Power shall be vested in a catastrophe of nationwide proportions or effect.[131] This is
President . . . .;Ó that Òhe shall take Care evident in the Records of the Constitutional Commission, thus:
FINALS CONSTITUTION I ACJUCO 248

government, legislation is
MR. GASCON. Yes. What is the preserved for Congress all the
CommitteeÕs definition of Ònational time, not excepting periods of
emergencyÓ which appears in Section 13, crisis no matter how
page 5? It reads: serious. Never in the history of
the United States, the basic
When the common good so requires, features of whose Constitution
the State may temporarily take over or have been copied in ours, have
direct the operation of any privately owned specific functions of the
public utility or business affected with legislative branch of enacting
public interest. laws been surrendered to another
department Ð unless we regard
MR. VILLEGAS. What I mean is as legislating the carrying out of a
threat from external aggression, for legislative policy according to
example, calamities or natural disasters. prescribed standards; no, not
even when that Republic was
fighting a total war, or when it was
MR. GASCON. There is a question
engaged in a life-and-death
by Commissioner de los Reyes. What
struggle to preserve the
about strikes and riots?
Union. The truth is that under our
concept of constitutional
MR. VILLEGAS. Strikes, no; those government, in times of extreme
would not be covered by the term Ònational perils more than in normal
emergency.Ó circumstances Ôthe various
branches, executive, legislative,
MR. BENGZON. Unless they are of and judicial,Õ given the ability to
such proportions such that they would act, are called upon Ôto perform
paralyze government service.[132] the duties and discharge the
responsibilities committed to
x x x them respectively.Ó
x x x

MR. TINGSON. May I ask the Following our interpretation of Section 17, Article XII,
committee if Ònational emergencyÓ refers invoked by President Arroyo in issuing PP 1017, this Court
to military national emergency or could rules that such Proclamation does not authorize her during the
this be economic emergency?Ó emergency to temporarily take over or direct the operation of
any privately owned public utility or business affected with
public interest without authority from Congress.
MR. VILLEGAS. Yes, it could refer
to both military or economic Let it be emphasized that while the President alone
dislocations. can declare a state of national emergency, however, without
legislation, he has no power to take over privately-owned
MR. TINGSON. Thank you very public utility or business affected with public interest. The
much.[133] President cannot decide whether
exceptional circumstances exist warranting the take over of
privately-owned public utility or business affected with
It may be argued that when there is national emergency, public interest. Nor can he determine when such exceptional
Congress may not be able to convene and, therefore, unable circumstances have ceased. Likewise, without
to delegate to the President the power to take over privately- legislation, the President has no power to point out the types
owned public utility or business affected with public interest. of businesses affected with public interest that should be taken
over. In short, the President has no absolute authority to
In Araneta v. Dinglasan,[134] this Court emphasized that exercise all the powers of the State under Section 17, Article
legislative power, through which extraordinary measures are VII in the absence of an emergency powers act passed by
exercised, remains in Congress even in times of crisis. Congress.

Òx x x
c. ÒAS APPLIED CHALLENGEÓ
After all the criticisms that
have been made against the One of the misfortunes of an emergency, particularly,
efficiency of the system of the that which pertains to security, is that military necessity and the
separation of powers, the fact guaranteed rights of the individual are often not
remains that the Constitution has compatible. Our history reveals that in the crucible of conflict,
set up this form of government, many rights are curtailed and trampled upon. Here, the right
with all its defects and against unreasonable search and seizure; the right
shortcomings, in preference to against warrantless arrest; and the freedom of speech, of
the commingling of powers in one expression, of the press, and of assembly under the Bill of
man or group of men. The Rights suffered the greatest blow.
Filipino people by adopting
parliamentary government have Of the seven (7) petitions, three (3) indicate Òdirect
given notice that they share the injury.Ó
faith of other democracy-loving
peoples in this system, with all its In G.R. No. 171396, petitioners David and Llamas
faults, as the ideal. The point is, alleged that, on February 24, 2006, they were arrested without
under this framework of warrants on their way to EDSA to celebrate the 20thAnniversary
FINALS CONSTITUTION I ACJUCO 249

of People Power I. The arresting officers cited PP 1017 as actions and measures to suppress and prevent acts of
basis of the arrest. terrorism and lawless violence.Ó

In G.R. No. 171409, petitioners Cacho-Olivares Unlike the term Òlawless violenceÓ which is
and Tribune Publishing Co., Inc. claimed that on February 25, unarguably extant in our statutes and the Constitution, and
2006, the CIDG operatives Òraided and ransacked without which is invariably associated with Òinvasion, insurrection or
warrantÓ their office. Three policemen were assigned to rebellion,Ó the phrase Òacts of terrorismÓ is still an
guard their office as a possible Òsource of amorphous and vague concept. Congress has yet to enact a
destabilization.Ó Again, the basis was PP 1017. law defining and punishing acts of terrorism.

And in G.R. No. 171483, petitioners KMU and


NAFLU-KMU et al. alleged that their members were Òturned
away and dispersedÓ when they went to EDSA and later, to In fact, this Òdefinitional predicamentÓ or the
Ayala Avenue, to celebrate the 20th Anniversary of People Òabsence of an agreed definition of terrorismÓ confronts not
Power I. only our country, but the international
community as well. The following observations are quite
A perusal of the Òdirect injuriesÓ allegedly suffered by apropos:
the said petitioners shows that they resulted from
the implementation, pursuant to G.O. No. 5, of PP 1017. In the actual unipolar context of
international relations, the Òfight against
Can this Court adjudge as unconstitutional PP 1017 terrorismÓ has become one of the basic
and G.O. No 5 on the basis of these illegal acts? In slogans when it comes to the justification of
general, does the illegal implementation of a law render it the use of force against certain states and
unconstitutional? against groups operating
internationally. Lists of states Òsponsoring
Settled is the rule that courts are not at liberty to terrorismÓ and of terrorist organizations
declare statutes invalid although they may be abused and are set up and constantly being updated
misabused[135] and may afford an opportunity for abuse in according to criteria that are not always
the manner of application.[136] The validity of a statute or known to the public, but are clearly
ordinance is to be determined from its general purpose and its determined by strategic interests.
efficiency to accomplish the end desired, not from its effects
in a particular case.[137] PP 1017 is merely an invocation of The basic problem underlying all
the PresidentÕs calling-out power. Its general purpose is to these military actions Ð or threats of the
command the AFP to suppress all forms of lawless violence, use of force as the most recent by the
invasion or rebellion. It had accomplished the end desired United States against Iraq Ð consists in the
which prompted President Arroyo to issue PP 1021. But there absence of an agreed definition of
is nothing in PP 1017 allowing the police, expressly or terrorism.
impliedly, to conduct illegal arrest, search or violate the
citizensÕ constitutional rights. Remarkable confusion persists in
regard to the legal categorization of acts of
violence either by states, by armed groups
such as liberation movements, or by
individuals.
Now, may this Court adjudge a law or ordinance
unconstitutional on the ground that its implementor committed The dilemma can by summarized
illegal acts? The answer is no. The criterion by which the in the saying ÒOne countryÕs terrorist is
validity of the statute or ordinance is to be measured is the another countryÕs freedom fighter.Ó The
essential basis for the exercise of power, and not a mere apparent contradiction or lack of
incidental result arising from its exertion.[138] This is consistency in the use of the term
logical. Just imagine the absurdity of situations when laws ÒterrorismÓ may further be demonstrated
maybe declared unconstitutional just because the officers by the historical fact that leaders of national
implementing them have acted arbitrarily. If this were so, liberation movements such as Nelson
judging from the blunders committed by policemen in the cases Mandela in South Africa, Habib Bourgouiba
passed upon by the Court, majority of the provisions of the in Tunisia, or Ahmed Ben Bella in Algeria,
Revised Penal Code would have been declared to mention only a few, were originally
unconstitutional a long time ago. labeled as terrorists by those who
controlled the territory at the time, but later
became internationally respected
President Arroyo issued G.O. No. 5 to carry into statesmen.
effect the provisions of PP 1017. General orders are Òacts
and commands of the President in his capacity as What, then, is the defining
Commander-in-Chief of the Armed Forces of the Philippines.Ó criterion for terrorist acts Ð the differentia
They are internal rules issued by the executive officer to his specifica distinguishing those acts from
subordinates precisely for eventually legitimate acts of national
the proper and efficientadministration of law. Such rules resistance or self-defense?
and regulations create no relation except between the official
who issues them and the official who receives them.[139] They Since the times of the Cold War
are based on and are the product of, a relationship in which the United Nations Organization has been
power is their source, and obedience, their object.[140] For trying in vain to reach a consensus on the
these reasons, one requirement for these rules to be valid is basic issue of definition. The organization
that they must be reasonable, not arbitrary or capricious. has intensified its efforts recently, but has
been unable to bridge the gap between
G.O. No. 5 mandates the AFP and the PNP to those who associate ÒterrorismÓ with any
immediately carry out the Ònecessary and appropriate violent act by non-state groups against
FINALS CONSTITUTION I ACJUCO 250

civilians, state functionaries or the decisive role in the Security Council,


infrastructure or military installations, and former great powers of the Cold War era as
those who believe in the concept of the well as medium powers are increasingly
legitimate use of force when resistance being marginalized; and the problem has
against foreign occupation or against become even more acute since the terrorist
systematic oppression of ethnic and/or attacks of 11 September 2001 I the United
religious groups within a state is States.[141]
concerned.
The absence of a law defining Òacts of terrorismÓ
The dilemma facing the may result in abuse and oppression on the part of the police or
international community can best be military. An illustration is when a group of persons are merely
illustrated by reference to the contradicting engaged in a drinking spree. Yet the military or the police may
categorization of organizations and consider the act as an act of terrorism and immediately arrest
movements such as Palestine Liberation them pursuant to G.O. No. 5. Obviously, this is abuse and
Organization (PLO) Ð which is a terrorist oppression on their part. It must be remembered that an act
group for Israel and a liberation movement can only be considered a crime if there is a law defining the
for Arabs and Muslims Ð the Kashmiri same as such and imposing the corresponding penalty
resistance groups Ð who are terrorists in thereon.
the perception of India, liberation fighters in
that of Pakistan Ð the earlier Contras in
Nicaragua Ð freedom fighters for the So far, the word ÒterrorismÓ appears only once in
United States, terrorists for the Socialist our criminal laws, i.e., in P.D. No. 1835 dated January 16, 1981
camp Ð or, most drastically, the Afghani enacted by President Marcos during the Martial Law
Mujahedeen (later to become the Taliban regime. This decree is entitled ÒCodifying The Various Laws
movement): during the Cold War period on Anti-Subversion and Increasing The Penalties for
they were a group of freedom fighters for Membership in Subversive Organizations.Ó The word
the West, nurtured by the United States, ÒterrorismÓ is mentioned in the following provision: ÒThat
and a terrorist gang for the Soviet one who conspires with any other person for the purpose of
Union. One could go on and on in overthrowing the Government of the Philippines x x x by force,
enumerating examples of conflicting violence, terrorism, x x x shall be punished by reclusion
categorizations that cannot be reconciled in temporal x x x.Ó
any way Ð because of opposing political
interests that are at the roots of those
perceptions.
P.D. No. 1835 was repealed by E.O. No. 167 (which
How, then, can those outlaws the Communist Party of the Philippines) enacted by
contradicting definitions and conflicting President Corazon Aquino on May 5, 1985. These two (2)
perceptions and evaluations of one and the laws, however, do not define Òacts of terrorism.Ó Since there
same group and its actions be is no law defining Òacts of terrorism,Ó it is President Arroyo
explained? In our analysis, the basic alone, under G.O. No. 5, who has the discretion to determine
reason for these striking inconsistencies what acts constitute terrorism. Her judgment on this aspect is
lies in the divergent interest of absolute, without restrictions. Consequently, there can be
states. Depending on whether a state is in indiscriminate arrest without warrants, breaking into offices
the position of an occupying power or in and residences, taking over the media enterprises, prohibition
that of a rival, or adversary, of an occupying and dispersal of all assemblies and gatherings unfriendly to the
power in a given territory, the definition of administration. All these can be effected in the name of G.O.
terrorism will ÒfluctuateÓ accordingly. A No. 5. These acts go far beyond the calling-out power of the
state may eventually see itself as protector President. Certainly, they violate the due process clause of
of the rights of a certain ethnic group the Constitution. Thus, this Court declares that the Òacts of
outside its territory and will therefore speak terrorismÓ portion of G.O. No. 5 is unconstitutional.
of a Òliberation struggle,Ó not of
ÒterrorismÓ when acts of violence by this
group are concerned, and vice-versa.

The United Nations Organization Significantly, there is nothing in G.O. No. 5


has been unable to reach a decision on the authorizing the military or police to commit acts beyond what
definition of terrorism exactly because of are necessary and appropriate to suppress and prevent
these conflicting interests of sovereign lawless violence, the limitation of their authority in pursuing
states that determine in each and every the Order. Otherwise, such acts are considered illegal.
instance how a particular armed movement
(i.e. a non-state actor) is labeled in regard
to the terrorists-freedom fighter
dichotomy. A Òpolicy of double
standardsÓ on this vital issue of
international affairs has been the We first examine G.R. No. 171396 (David et al.)
unavoidable consequence.

This Òdefinitional predicamentÓ


of an organization consisting of sovereign The Constitution provides that Òthe right of the
states Ð and not of peoples, in spite of the people to be secured in their persons, houses, papers and
emphasis in the Preamble to the United effects against unreasonable search and seizure of whatever
Nations Charter! Ð has become even more nature and for any purpose shall be inviolable, and no search
serious in the present global power warrant or warrant of arrest shall issue except upon probable
constellation: one superpower exercises cause to be determined personally by the judge after
FINALS CONSTITUTION I ACJUCO 251

examination under oath or affirmation of the complainant and


the witnesses he may produce, and particularly describing the
place to be searched and the persons or things to be But what made it doubly worse for petitioners
seized.Ó[142] The plain import of the language of the David et al. is that not only was their right against warrantless
Constitution is that searches, seizures and arrests arrest violated, but also their right to peaceably assemble.
are normally unreasonable unless authorized by a validly
issued search warrant or warrant of arrest. Thus, the
fundamental protection given by this provision is that between
person and police must stand the protective authority of a
magistrate clothed with power to issue or refuse to issue
search warrants or warrants of arrest.[143]
Section 4 of Article III guarantees:

In the Brief Account[144] submitted by petitioner David, No law shall be passed abridging
certain facts are established: first, he was arrested without the freedom of speech, of expression, or of
warrant; second, the PNP operatives arrested him on the press, or the right of the people
the basis of PP 1017; third, he was brought at Camp peaceably to assemble and petition the
Karingal, Quezon City where he was fingerprinted, government for redress of grievances.
photographed and booked like a criminal suspect; fourth, he
was treated brusquely by policemen who Òheld his head and
tried to push himÓ inside an unmarked car; fifth, he was ÒAssemblyÓ means a right on the part of the citizens
charged with Violation of Batas Pambansa Bilang No. to meet peaceably for consultation in respect to public
880[145]and Inciting to Sedition; sixth, he was detained for affairs. It is a necessary consequence of our republican
seven (7) hours; and seventh, he was eventually released for institution and complements the right of speech. As in the case
insufficiency of evidence. of freedom of expression, this right is not to be limited, much
less denied, except on a showing of a clear and present
danger of a substantive evil that Congress has a right to
prevent. In other words, like other rights embraced in the
freedom of expression, the right to assemble is not subject to
previous restraint or censorship. It may not be conditioned
Section 5, Rule 113 of the Revised Rules on upon the prior issuance of a permit or authorization from the
Criminal Procedure provides: government authorities except, of course, if the assembly is
intended to be held in a public place, a permit for the use of
Sec. 5. Arrest without such place, and not for the assembly itself, may be validly
warrant; when lawful. - A peace required.
officer or a private person may,
without a warrant, arrest a person:

(a) When, in his presence, The ringing truth here is that petitioner David, et al.
the person to be arrested has were arrested while they were exercising their right to peaceful
committed, is actually committing, or assembly. They were not committing any crime, neither was
is attempting to commit an offense. there a showing of a clear and present danger that warranted
the limitation of that right. As can be gleaned from
(b) When an offense has circumstances, the charges of inciting to
just been committed and he has seditionand violation of BP 880 were mere
probable cause to believe based on afterthought. Even the Solicitor General, during the oral
personal knowledge of facts or argument, failed to justify the arresting officersÕ
circumstances that the person to be conduct. In De Jonge v. Oregon,[148] it was held that
arrested has committed it; and peaceable assembly cannot be made a crime, thus:

x x x. Peaceable assembly for lawful


discussion cannot be made a crime. The
holding of meetings for peaceable political
action cannot be proscribed. Those who
assist in the conduct of such meetings
cannot be branded as criminals on that
Neither of the two (2) exceptions mentioned above score. The question, if the rights of free
justifies petitioner DavidÕs warrantless arrest. During the speech and peaceful assembly are not to
inquest for the charges of inciting to sedition andviolation be preserved, is not as to the auspices
of BP 880, all that the arresting officers could invoke under which the meeting was held but as to
was their observation that some rallyists were wearing t- its purpose; not as to the relations of the
shirts with the invective ÒOust Gloria speakers, but whether their utterances
NowÓand their erroneous assumption that petitioner David transcend the bounds of the freedom of
was the leader of the rally.[146] Consequently, the Inquest speech which the Constitution protects. If
Prosecutor ordered his immediate release on the ground of the persons assembling have committed
insufficiency of evidence. He noted that petitioner David was crimes elsewhere, if they have formed or
not wearing the subject t-shirt and even if he was wearing it, are engaged in a conspiracy against the
such fact is insufficient to charge him with inciting to public peace and order, they may be
sedition. Further, he also stated that there is insufficient prosecuted for their conspiracy or other
evidence for the charge of violation of BP 880 as it was not violations of valid laws. But it is a different
even known whether petitioner David was the leader of the matter when the State, instead of
rally.[147] prosecuting them for such offenses,
seizes upon mere participation in a
peaceable assembly and a lawful public
FINALS CONSTITUTION I ACJUCO 252

discussion as the basis for a criminal


charge.
The search is illegal. Rule 126 of The Revised Rules
on Criminal Procedure lays down the steps in the conduct of
search and seizure. Section 4 requires that a search
On the basis of the above principles, the Court warrant be issued upon probable cause in connection with
likewise considers the dispersal and arrest of the members of one specific offence to be determined personally by the judge
KMU et al. (G.R. No. 171483) unwarranted. Apparently, their after examination under oath or affirmation of the complainant
dispersal was done merely on the basis of Malaca–angÕs and the witnesses he may produce. Section 8 mandates that
directive canceling all permits previously issued by local the search of a house, room, or any other premise be made in
government units. This is arbitrary. The wholesale the presence of the lawful occupantthereof or any member
cancellation of all permits to rally is a blatant disregard of the of his family or in the absence of the latter, in the presence of
principle that Òfreedom of assembly is not to be limited, two (2) witnesses of sufficient age and discretion residing in
much less denied, except on a showing of a clear and the same locality. And Section 9 states that the warrant
present danger of a substantive evil that the State has a must direct that it be served in the daytime, unless the
right to prevent.Ó[149] Tolerance is the rule and limitation is property is on the person or in the place ordered to be
the exception. Only upon a showing that an assembly searched, in which case a direction may be inserted that it be
presents a clear and present danger that the State may deny served at any time of the day or night. All these rules were
the citizensÕ right to exercise it. Indeed, respondents failed to violated by the CIDG operatives.
show or convince the Court that the rallyists committed acts
amounting to lawless violence, invasion or rebellion. With the
blanket revocation of permits, the distinction between
protected and unprotected assemblies was eliminated.
Not only that, the search violated petitionersÕ
freedom of the press. The best gauge of a free and
Moreover, under BP 880, the authority to regulate democratic society rests in the degree of freedom enjoyed by
assemblies and rallies is lodged with the local government its media. In the Burgos v. Chief of Staff[152] this Court held
units. They have the power to issue permits and to revoke that --
such permits after due notice and hearing on the
determination of the presence of clear and present danger.
Here, petitioners were not even notified and heard on the As heretofore stated, the
revocation of their permits.[150] The first time they learned of it premises searched were the business and
was at the time of the dispersal. Such absence of notice is a printing offices of the "Metropolitan Mail"
fatal defect. When a personÕs right is restricted by and the "We ForumÓ newspapers. As a
government action, it behooves a democratic government to consequence of the search and
see to it that the restriction is fair, reasonable, and according seizure, these premises were padlocked
to procedure. and sealed, with the further result that
the printing and publication of said
newspapers were discontinued.

G.R. No. 171409, (Cacho-Olivares, et al.) presents Such closure is in the nature of
another facet of freedom of speech i.e., the freedom of the previous restraint or censorship
press. PetitionersÕ narration of facts, which the Solicitor abhorrent to the freedom of the press
General failed to refute, established the guaranteed under the fundamental law,
following: first, the Daily TribuneÕs offices were searched and constitutes a virtual denial of
without warrant; second, the police operatives seized several petitioners' freedom to express
materials for publication; third, the search was conducted at themselves in print. This state of being
about 1:00 oÕ clock in the morning of February 25, is patently anathematic to a democratic
2006; fourth, the search was conducted in the absence of any framework where a free, alert and even
official of the Daily Tribune except the security guard of the militant press is essential for the
building; and fifth, policemen stationed themselves at the political enlightenment and growth of
vicinity of the Daily Tribune offices. the citizenry.

Thereafter, a wave of warning came from government While admittedly, the Daily Tribune was not
officials. Presidential Chief of Staff Michael Defensor was padlocked and sealed like the ÒMetropolitan MailÓ and ÒWe
quoted as saying that such raid was Òmeant to show a ForumÓ newspapers in the above case, yet it cannot be denied
Ôstrong presence,Õ to tell media outlets not to connive or that the CIDG operatives exceeded their enforcement
do anything that would help the rebels in bringing down duties. The search and seizure of materials for publication, the
this government.Ó Director General Lomibao further stated stationing of policemen in the vicinity of the The Daily
that Òif they do not follow the standards Ðand the Tribune offices, and the arrogant warning of government
standards are if they would contribute to instability in the officials to media, are plain censorship. It is that officious
government, or if they do not subscribe to what is in functionary of the repressive government who tells the citizen
General Order No. 5 and Proc. No. 1017 Ð we will that he may speak only if allowed to do so, and no more and
recommend a Ôtakeover.ÕÓ National Telecommunications no less than what he is permitted to say on pain of punishment
Commissioner Ronald Solis urged television and radio should he be so rash as to disobey.[153] Undoubtedly, the The
networks to ÒcooperateÓ with the government for the Daily Tribune was subjected to these arbitrary intrusions
duration of the state of national emergency. He warned that because of its anti-government sentiments. This Court cannot
his agency will not hesitate to recommend the closure of tolerate the blatant disregard of a constitutional right even if it
any broadcast outfit that violates rules set out for media involves the most defiant of our citizens. Freedom to
coverage during times when the national security is comment on public affairs is essential to the vitality of a
threatened.[151] representative democracy. It is the duty of the courts to be
watchful for the constitutional rights of the citizen, and against
FINALS CONSTITUTION I ACJUCO 253

any stealthy encroachments thereon. The motto should issues. So


always be obsta principiis.[154] why do you
have to go
there at 1
oÕclock in the
morning and
Incidentally, during the oral arguments, the Solicitor without any
General admitted that the search of the TribuneÕs offices and search
the seizure of its materials for publication and other papers are warrant? Did
illegal; and that the same are inadmissible Òfor any purpose,Ó they become
thus: suddenly part
of the
JUSTICE CALLEJO: evidence of
rebellion or
You made quite inciting to
a mouthful of sedition or
admission what?
when you
said that the SOLGEN BENIPAYO:
policemen,
when Well, it was the
inspected the police that did
Tribune for that, Your
the purpose Honor. Not
of gathering upon my
evidence and instructions.
you admitted
that the SR. ASSO. JUSTICE
policemen PUNO:
were able to
get the Are you saying
clippings. Is that the act of
that not in the policeman
admission of is illegal, it is
the not based on
admissibility any law, and it
of these is not based
clippings that on
were taken Proclamation
from the 1017.
Tribune?
SOLGEN BENIPAYO:
SOLICITOR GENERAL
BENIPAYO: It is not based
on
Under the law Proclamation
they would 1017, Your
seem to be, if Honor,
they were because
illegally there is
seized, I think nothing in
and I know, 1017 which
Your Honor, says that the
and these are police could
inadmissible go and
for any inspect and
purpose.[155] gather
clippings from
x x x x x Daily Tribune
x x or any other
xx newspaper.

SR. ASSO. JUSTICE SR. ASSO. JUSTICE


PUNO: PUNO:

These have Is it based on


been any law?
published in
the past SOLGEN BENIPAYO:
issues of the
Daily Tribune; As far as I
all you have know, no,
to do is to get Your Honor,
those past
FINALS CONSTITUTION I ACJUCO 254

from the result in no constitutional or statutory breaches if applied


facts, no. according to their letter.Ó

SR. ASSO. JUSTICE The Court has passed upon the constitutionality of
PUNO: these issuances. Its ratiocination has been exhaustively
presented. At this point, suffice it to reiterate that PP 1017 is
So, it has no limited to the calling out by the President of the military to
basis, no prevent or suppress lawless violence, invasion or
legal basis rebellion. When in implementing its provisions, pursuant to
whatsoever? G.O. No. 5, the military and the police committed acts which
violate the citizensÕ rights under the Constitution, this Court
has to declare such acts unconstitutional and illegal.

In this connection, Chief Justice Artemio V.


SOLGEN BENIPAYO: PanganibanÕs concurring opinion, attached hereto, is
considered an integral part of this ponencia.
Maybe so, Your
Honor. Mayb
e so, that is SUMMATION
why I said, I
donÕt know if In sum, the lifting of PP 1017 through the issuance of
it is premature PP 1021 Ð a supervening event Ð would have normally
to say rendered this case moot and academic. However, while PP
this, we do 1017 was still operative, illegal acts were committed allegedly
not condone in pursuance thereof. Besides, there is no guarantee that PP
this. If the 1017, or one similar to it, may not again be issued. Already,
people who there have been media reports on April 30, 2006 that allegedly
have been PP 1017 would be reimposed Òif the May 1 ralliesÓ become
injured by Òunruly and violent.Ó Consequently, the transcendental
this would issues raised by the parties should not be Òevaded;Ó they
want to sue must now be resolved to prevent future constitutional
them, they aberration.
can sue and
there are The Court finds and so holds that PP 1017 is
remedies for constitutional insofar as it constitutes a call by the President for
this.[156] the AFP to prevent or suppress lawless violence. The
proclamation is sustained by Section 18, Article VII of the
Constitution and the relevant jurisprudence discussed
Likewise, the warrantless arrests and seizures earlier. However, PP 1017Õs extraneous provisions giving the
executed by the police were, according to the Solicitor General, President express or implied power (1) to issue decrees; (2) to
illegal and cannot be condoned, thus: direct the AFP to enforce obedience to all laws even those not
related to lawless violence as well as decrees promulgated by
CHIEF JUSTICE the President; and (3) to impose standards on media or any
PANGANIBAN: form of prior restraint on the press, are ultra
vires and unconstitutional. The Court also rules that under
There seems Section 17, Article XII of the Constitution, the President, in the
to be some confusions absence of a legislation, cannot take over privately-owned
if not contradiction in public utility and private business affected with public interest.
your theory.

SOLICITOR GENERAL
BENIPAYO: In the same vein, the Court finds G.O. No. 5 valid. It
is an Order issued by the President Ð acting as Commander-
I donÕt know in-Chief Ð addressed to subalterns in the AFP to carry out the
whether this will provisions of PP 1017. Significantly, it also provides a valid
clarify. The acts, the standard Ð that the military and the police should take only the
supposed illegal or Ònecessary and appropriate actions and measures to
unlawful acts suppress and prevent acts of lawless violence.Ó But the
committed on the words Òacts of terrorismÓ found in G.O. No. 5 have not been
occasion of 1017, as I legally defined and made punishable by Congress and should
said, it cannot be thus be deemed deleted from the said G.O. While
condoned. You ÒterrorismÓ has been denounced generally in media, no law
cannot blame the has been enacted to guide the military, and eventually the
President for, as you courts, to determine the limits of the AFPÕs authority in
said, a misapplication carrying out this portion of G.O. No. 5.
of the law. These are
acts of the police
officers, that is their
responsibility.[157] On the basis of the relevant and uncontested facts
narrated earlier, it is also pristine clear that (1) the warrantless
arrest of petitioners Randolf S. David and Ronald Llamas; (2)
the dispersal of the rallies and warrantless arrest of the KMU
The Dissenting Opinion states that PP 1017 and and NAFLU-KMU members; (3) the imposition of standards on
G.O. No. 5 are constitutional in every aspect and Òshould media or any prior restraint on the press; and (4) the
FINALS CONSTITUTION I ACJUCO 255

warrantless search of the Tribune offices and the whimsical No costs.


seizures of some articles for publication and other materials, SO ORDERED.
are not authorized by the Constitution, the law and
jurisprudence. Not even by the valid provisions of PP 1017
and G.O. No. 5.

Other than this declaration of invalidity, this Court


cannot impose any civil, criminal or administrative sanctions on
the individual police officers concerned. They have not been
individually identified and given their day in court. The civil
complaints or causes of action and/or relevant criminal
Informations have not been presented before this
Court. Elementary due process bars this Court from making
any specific pronouncement of civil, criminal or administrative
liabilities.

It is well to remember that military power is a


means to an end and substantive civil rights are ends in
themselves. How to give the military the power it needs to
protect the Republic without unnecessarily trampling
individual rights is one of the eternal balancing tasks of a
democratic state. During emergency, governmental action
may vary in breadth and intensity from normal times, yet they
should not be arbitrary as to unduly restrain our peopleÕs
liberty.

Perhaps, the vital lesson that we must learn from the


theorists who studied the various competing political
philosophies is that, it is possible to grant government the
authority to cope with crises without surrendering the two vital
principles of constitutionalism: the maintenance of legal
limits to arbitrary power, and political responsibility of the
government to the governed.[158]

WHEREFORE, the Petitions are partly granted. The


Court rules that PP 1017 is CONSTITUTIONAL insofar as it
constitutes a call by President Gloria Macapagal-Arroyo on the
AFP to prevent or suppress lawless violence. However,
the provisions of PP 1017 commanding the AFP to enforce
laws not related to lawless violence, as well as decrees
promulgated by the President, are
declared UNCONSTITUTIONAL. In addition, the provision in
PP 1017 declaring national emergency under Section
17, Article VII of the Constitution is CONSTITUTIONAL, but
such declaration does not authorize the President to take over
privately-owned public utility or business affected with public
interest without prior legislation.

G.O. No. 5 is CONSTITUTIONAL since it provides a


standard by which the AFP and the PNP should implement PP
1017, i.e. whatever is Ònecessary and appropriate actions
and measures to suppress and prevent acts of lawless
violence.Ó Considering that Òacts of terrorismÓ have not yet
been defined and made punishable by the Legislature, such
portion of G.O. No. 5 is declared UNCONSTITUTIONAL.
The warrantless arrest of Randolf S. David and
Ronald Llamas; the dispersal and warrantless arrest of the
KMU and NAFLU-KMU members during their rallies, in the
absence of proof that these petitioners were committing acts
constituting lawless violence, invasion or rebellion and
violating BP 880; the imposition of standards on media or any
form of prior restraint on the press, as well as the warrantless
search of the Tribune offices and whimsical seizure of its
articles for publication and other materials, are
declared UNCONSTITUTIONAL.
FINALS CONSTITUTION I ACJUCO 256

GR 190293 constitutionality, said the Court in Biraogo v. Philippine Truth


Fortun vs. Arroyo Commission of 2010,[1] must be the very issue of the case, that
DECISION the resolution of such issue is unavoidable.
The issue of the constitutionality of Proclamation 1959 is not
ABAD, J.: unavoidable for two reasons:

One. President Arroyo withdrew her proclamation of martial


These cases concern the constitutionality of a presidential law and suspension of the privilege of the writ of habeas
proclamation of martial law and suspension of the privilege corpus before the joint houses of Congress could fulfill their
of habeas corpus in 2009 in a province in Mindanao which automatic duty to review and validate or invalidate the
were withdrawn after just eight days. same. The pertinent provisions of Section 18, Article VII of the
1987 Constitution state:
The Facts and the Case
Sec. 18. The President shall be the
The essential background facts are not in dispute. On Commander-in-Chief of all armed forces of
November 23, 2009 heavily armed men, believed led by the the Philippines and whenever it becomes
ruling Ampatuan family, gunned down and buried under necessary, he may call out such armed
shoveled dirt 57 innocent civilians on a highway in forces to prevent or suppress lawless
Maguindanao. In response to this carnage, on November 24 violence, invasion or rebellion. In case of
President Arroyo issued Presidential Proclamation 1946, invasion or rebellion, when the public
declaring a state of emergency in Maguindanao, Sultan safety requires it, he may, for a period not
Kudarat, and Cotabato City to prevent and suppress similar exceeding sixty days, suspend the privilege
lawless violence in Central Mindanao. of the writ of habeas corpus or place
the Philippines or any part thereof under
Believing that she needed greater authority to put martial law. Within forty-eight hours from
order in Maguindanao and secure it from large groups of the proclamation of martial law or the
persons that have taken up arms against the constituted suspension of the privilege of writ
authorities in the province, on December 4, 2009 President of habeas corpus, the President shall
Arroyo issued Presidential Proclamation 1959 declaring submit a report in person or in writing to the
martial law and suspending the privilege of the writ of habeas Congress. The Congress, voting jointly, by
corpus in that province except for identified areas of the Moro a vote of at least a majority of all its
Islamic Liberation Front. Members in regular or special session, may
revoke such proclamation or suspension,
Two days later or on December 6, 2009 President which revocation shall not be set aside by
Arroyo submitted her report to Congress in accordance with the President. Upon the initiative of the
Section 18, Article VII of the 1987 Constitution which required President, the Congress may, in the same
her, within 48 hours from the proclamation of martial law or the manner, extend such proclamation or
suspension of the privilege of the writ of habeas corpus, to suspension for a period to be determined
submit to that body a report in person or in writing of her action. by the Congress, if the invasion or rebellion
shall persist and public safety requires it.
In her report, President Arroyo said that she acted
based on her finding that lawless men have taken up arms in The Congress, if not in session, shall,
Maguindanao and risen against the government. The within twenty-four hours following such
President described the scope of the uprising, the nature, proclamation or suspension, convene in
quantity, and quality of the rebels weaponry, the movement of accordance with its rules without any need
their heavily armed units in strategic positions, the closure of of a call.
the Maguindanao Provincial Capitol, Ampatuan Municipal Hall,
Datu Unsay Municipal Hall, and 14 other municipal halls, and xxxx
the use of armored vehicles, tanks, and patrol cars with Although the above vests in the President the power to
unauthorized PNP/Police markings. proclaim martial law or suspend the privilege of the writ
of habeas corpus, he shares such power with the
On December 9, 2009 Congress, in joint session, Congress. Thus:
convened pursuant to Section 18, Article VII of the 1987
Constitution to review the validity of the Presidents action. But, 1. The Presidents proclamation or
two days later or on December 12 before Congress could act, suspension is temporary, good for only 60
the President issued Presidential Proclamation 1963, lifting days;
martial law and restoring the privilege of the writ of habeas
corpus in Maguindanao. 2. He must, within 48 hours of the
proclamation or suspension, report his
Petitioners Philip Sigfrid A. Fortun and the other petitioners in action in person or in writing to Congress;
G.R. 190293, 190294, 190301,190302, 190307, 190356, and
190380 brought the present actions to challenge the 3. Both houses of Congress, if not in
constitutionality of President Arroyos Proclamation 1959 session must jointly convene within 24
affecting Maguindanao. But, given the prompt lifting of that hours of the proclamation or suspension for
proclamation before Congress could review it and before any the purpose of reviewing its validity; and
serious question affecting the rights and liberties of
Maguindanaos inhabitants could arise, the Court deems any 4. The Congress, voting jointly, may revoke
review of its constitutionality the equivalent of beating a dead or affirm the Presidents proclamation or
horse. suspension, allow their limited effectivity to
lapse, or extend the same if Congress
Prudence and respect for the co-equal departments of the deems warranted.
government dictate that the Court should be cautious in
entertaining actions that assail the constitutionality of the acts It is evident that under the 1987 Constitution the President and
of the Executive or the Legislative department. The issue of the Congress act in tandem in exercising the power to proclaim
FINALS CONSTITUTION I ACJUCO 257

martial law or suspend the privilege of the writ of habeas


corpus. They exercise the power, not only sequentially, but in The Court does not resolve purely academic
a sense jointly since, after the President has initiated the questions to satisfy scholarly interest, however intellectually
proclamation or the suspension, only the Congress can challenging these are.[5] This is especially true, said the Court
maintain the same based on its own evaluation of the situation in Philippine Association of Colleges and
on the ground, a power that the President does not have. Universities v. Secretary of
Education,[6] where the issues reach constitutional
Consequently, although the Constitution reserves to the dimensions, for then there comes into play regard for the courts
Supreme Court the power to review the sufficiency of the duty to avoid decision of constitutional issues unless
factual basis of the proclamation or suspension in a proper suit, avoidance becomes evasion. The Courts duty is to steer clear
it is implicit that the Court must allow Congress to exercise its of declaring unconstitutional the acts of the Executive or the
own review powers, which is automatic rather than Legislative department, given the assumption that it carefully
initiated. Only when Congress defaults in its express duty to studied those acts and found them consistent with the
defend the Constitution through such review should the fundamental law before taking them. To doubt is to sustain. [7]
Supreme Court step in as its final rampart. The constitutional
validity of the Presidents proclamation of martial law or Notably, under Section 18, Article VII of the 1987
suspension of the writ of habeas corpus is first a political Constitution, the Court has only 30 days from the filing of an
question in the hands of Congress before it becomes a appropriate proceeding to review the sufficiency of the factual
justiciable one in the hands of the Court. basis of the proclamation of martial law or the suspension of
the privilege of the writ of habeas corpus. Thus
Here, President Arroyo withdrew Proclamation 1959
before the joint houses of Congress, which had in fact The Supreme Court may review, in an
convened, could act on the same. Consequently, the petitions appropriate proceeding filed by any citizen,
in these cases have become moot and the Court has nothing the sufficiency of the factual basis of the
to review. The lifting of martial law and restoration of the proclamation of martial law or the
privilege of the writ of habeas corpus in Maguindanao was a suspension of the privilege of the writ
supervening event that obliterated any justiciable of habeas corpus or the extension thereof,
controversy.[2] and must promulgate its decision
Two. Since President Arroyo withdrew her proclamation of thereon within thirty days from its filing.
martial law and suspension of the privilege of the writ (Emphasis supplied)
of habeas corpus in just eight days, they have not been
meaningfully implemented. The military did not take over the More than two years have passed since petitioners
operation and control of local government units in filed the present actions to annul Proclamation 1959. When the
Maguindanao. The President did not issue any law or decree Court did not decide it then, it actually opted for a default as
affecting Maguindanao that should ordinarily be enacted by was its duty, the question having become moot and academic.
Congress. No indiscriminate mass arrest had been
reported. Those who were arrested during the period were Justice Carpio of course points out that should the
either released or promptly charged in court. Indeed, no Court regard the powers of the President and Congress
petition for habeas corpus had been filed with the Court respecting the proclamation of martial law or the suspension of
respecting arrests made in those eight days. The point is that the privilege of the writ of habeas corpus as sequential or joint,
the President intended by her action to address an uprising in it would be impossible for the Court to exercise its power of
a relatively small and sparsely populated province. In her review within the 30 days given it.
judgment, the rebellion was localized and swiftly disintegrated
in the face of a determined and amply armed government But those 30 days, fixed by the Constitution, should
presence. be enough for the Court to fulfill its duty without pre-empting
congressional action. Section 18, Article VII, requires the
In Lansang v. Garcia,[3] the Court received evidence in President to report his actions to Congress, in person or in
executive session to determine if President Marcos writing, within 48 hours of such proclamation or suspension. In
suspension of the privilege of the writ of habeas corpus in 1971 turn, the Congress is required to convene without need of a call
had sufficient factual basis. In Aquino, Jr. v. Enrile,[4] while the within 24 hours following the Presidents proclamation or
Court took judicial notice of the factual bases for President suspension. Clearly, the Constitution calls for quick action on
Marcos proclamation of martial law in 1972, it still held hearings the part of the Congress. Whatever form that action takes,
on the petitions for habeas corpus to determine the therefore, should give the Court sufficient time to fulfill its own
constitutionality of the arrest and detention of the mandate to review the factual basis of the proclamation or
petitioners. Here, however, the Court has not bothered to suspension within 30 days of its issuance.
examine the evidence upon which President Arroyo acted in
issuing Proclamation 1959, precisely because it felt no need If the Congress procrastinates or altogether fails to
to, the proclamation having been withdrawn within a few days fulfill its duty respecting the proclamation or suspension within
of its issuance. the short time expected of it, then the Court can step in, hear
the petitions challenging the Presidents action, and ascertain
Justice Antonio T. Carpio points out in his dissenting opinion if it has a factual basis. If the Court finds none, then it can annul
the finding of the Regional Trial Court (RTC) of Quezon City the proclamation or the suspension. But what if the 30 days
that no probable cause exist that the accused before it given it by the Constitution proves inadequate? Justice Carpio
committed rebellion in Maguindanao since the prosecution himself offers the answer in his dissent: that 30-day period
failed to establish the elements of the crime. But the Court does not operate to divest this Court of its jurisdiction over the
cannot use such finding as basis for striking down the case. The settled rule is that jurisdiction once acquired is not
Presidents proclamation and suspension. For, firstly, the Court lost until the case has been terminated.
did not delegate and could not delegate to the RTC of Quezon
City its power to determine the factual basis for the presidential The problem in this case is that the President aborted
proclamation and suspension. Secondly, there is no showing the proclamation of martial law and the suspension of the
that the RTC of Quezon City passed upon the same evidence privilege of the writ of habeas corpus in Maguindanao in just
that the President, as Commander-in-Chief of the Armed eight days. In a real sense, the proclamation and the
Forces, had in her possession when she issued the suspension never took off. The Congress itself adjourned
proclamation and suspension.
FINALS CONSTITUTION I ACJUCO 258

without touching the matter, it having become moot and


academic.

Of course, the Court has in exceptional cases passed


upon issues that ordinarily would have been regarded as
moot. But the present cases do not present sufficient basis for
the exercise of the power of judicial review. The proclamation
of martial law and the suspension of the privilege of the writ
of habeas corpus in this case, unlike similar Presidential acts
in the late 60s and early 70s, appear more like saber-rattling
than an actual deployment and arbitrary use of political power.

WHEREFORE, the Court DISMISSES the


consolidated petitions on the ground that the same have
become moot and academic.

SO ORDERED.
FINALS CONSTITUTION I ACJUCO 259

SPOUSES RENATO G.R. No. 106064 government organization that advocates a pro-people and just
CONSTANTINO, JR. and Philippine debt policy.[2] Named respondents were the then
LOURDES CONSTANTINO Present: Governor of the Bangko Sentral ng Pilipinas, the Secretary of
and their minor children Finance, the National Treasurer, and the Philippine Debt
RENATO REDENTOR, DAVIDE, JR., CJ., Negotiation Chairman Emmanuel V. Pelaez.[3] All respondents
ANNA MARIKA LISSA, PUNO, were members of the Philippine panel tasked to negotiate with
NINA ELISSA, and PANGANIBAN, the countrys foreign creditors pursuant to the Financing
ANNA KARMINA, QUISUMBING, Program.
FREEDOM FROM DEBT YNARES-SANTIAGO,
COALITION, and FILOMENO SANDOVAL-GUTIERREZ,
STA. ANA III, CARPIO,
Petitioners , AUSTRIA-MARTINEZ, The operative facts are sparse and there is little need
CORONA, to elaborate on them.
CARPIO-MORALES,
CALLEJO, SR., The Financing Program was the culmination of
- versus - AZCUNA, efforts that began during the term of former President Corazon
TINGA, Aquino to manage the countrys external debt problem through
CHICO-NAZARIO, and a negotiation-oriented debt strategy involving cooperation and
GARCIA, JJ. negotiation with foreign creditors.[4]Pursuant to this strategy,
HON. JOSE B. CUISIA, the Aquino government entered into three restructuring
in his capacity as Governor agreements with representatives of foreign creditor
of the Central Bank, governments during the period of 1986 to 1991. [5] During the
HON. RAMON DEL ROSARIO, same period, three similarly-oriented restructuring agreements
in his capacity as Secretary were executed with commercial bank creditors.[6]
of Finance, HON. EMMANUEL V.
PELAEZ, in his capacity as On 28 February 1992, the Philippine Debt
Philippine Debt Negotiating Negotiating Team, chaired by respondent Pelaez, negotiated
Chairman, and the NATIONAL Promulgated: an agreement with the countrys Bank Advisory Committee,
TREASURER, representing all foreign commercial bank creditors, on the
Respondents. October 13, 2005 Financing Program which respondents characterized as a
x-------------------------------------------------------------------x multi-option financing

DECISION

TINGA, J.: package.[7] The Program was scheduled to be executed on 24


July 1992 by respondents in behalf of the Republic.
The quagmire that is the foreign debt problem has Nonetheless, petitioners alleged that even prior to the
especially confounded developing nations around the world for execution of the Program respondents had already
decades. It has defied easy solutions acceptable both to debtor implemented its buyback component when on 15 May 1992,
countries and their creditors. It has also emerged as cause the Philippines bought back P1.26 billion of external debts
celebre for various political movements and grassroots pursuant to the Program.[8]
activists and the wellspring of much scholarly thought and
debate. The petition sought to enjoin the ratification of the
Program, but the Court did not issue any injunctive relief.
The present petition illustrates some of the Hence, it came to pass that the Program was signed
ideological and functional differences between experts on how in London as scheduled. The petition still has to be resolved
to achieve debt relief. However, this being a court of law, not though as petitioners seek the annulment of
an academic forum or a convention on development any and all acts done by respondents, their subordinates and
economics, our resolution has to hinge on the presented legal any other public officer pursuant to the agreement and program
issues which center on the appreciation of the constitutional in question.[9]Even after the signing of the
provision that empowers the President to contract and Program, respondents themselves acknowledged that the
guarantee foreign loans. The ultimate choice is between a remaining principal objective of the petition is to set aside
restrictive reading of the constitutional provision and an respondents actions.[10]
alimentative application thereof consistent with time-honored
principles on executive power and the alter ego doctrine. Petitioners characterize the Financing Program as a
package offered to the countrys foreign creditors consisting of
This Petition for Certiorari, Prohibition and two debt-relief options.[11] The first option was a cash buyback
Mandamus assails said contracts which were entered into of portions of the Philippine foreign debt at a discount. [12] The
pursuant to the Philippine Comprehensive Financing Program second option allowed creditors to convert existing Philippine
for 1992 (Financing Program or Program). It seeks to enjoin debt instruments into any of three kinds of bonds/securities: (1)
respondents from executing additional debt-relief contracts new money bonds with a five-year grace period and 17 years
pursuant thereto. It also urges the Court to issue an order final maturity, the purchase of which would allow the creditors
compelling the Secretary of Justice to institute criminal and to convert their eligible debt papers into bearer bonds with the
administrative cases against respondents for acts which same terms; (2) interest-reduction bonds with a maturity of 25
circumvent or negate the provisions Art. XII of the years; and (3) principal-collateralized interest-reduction bonds
Constitution.[1] with a maturity of 25 years.[13]

Parties and Facts On the other hand, according to respondents the Financing
Program would cover about U.S. $5.3 billion of foreign
The petition was filed on 17 July 1992 by petitioners commercial debts and it was expected to deal
spouses Renato Constantino, Jr. and Lourdes Constantino comprehensively with the commercial bank debt problem of
and their minor children, Renato Redentor, Anna Marika Lissa, the country and pave the way for the countrys access to capital
Nina Elissa, and Anna Karmina, Filomeno Sta. Ana III, and the markets.[14] They add that the Program carried three basic
Freedom from Debt Coalition, a non-stock, non-profit, non- options from which foreign bank lenders could choose, namely:
FINALS CONSTITUTION I ACJUCO 260

to lend money, to exchange existing restructured Philippine Jr.,[20] this Court reiterated that the prevailing doctrines in
debts with an interest reduction bond; or to exchange the same taxpayers suits are to allow taxpayers to question contracts
Philippine debts with a principal collateralized interest entered into by the national government or government owned
reduction bond.[15] and controlled corporations allegedly in contravention of
law.[21] A taxpayer is allowed to sue where there is a claim that
Issues for Resolution public funds are illegally disbursed, or that public money is
being deflected to any improper purpose, or that there is a
Petitioners raise several issues before this Court. wastage of public funds through the enforcement of an invalid
or unconstitutional law.[22]
First, they object to the debt-relief contracts entered
into pursuant to the Financing Program as beyond the powers Moreover, a ruling on the issues of this case will not
granted to the President under Section 20, only determine the validity or invalidity of the subject pre-
Article VII of the Constitution.[16] The provision states that the termination and bond-conversion of foreign debts but also
President may contract or guarantee foreign loans in behalf of create a precedent for other debts or debt-related contracts
the Republic. It is claimed that the buyback and executed or to be executed in behalf of the President of the
securitization/bond conversion schemes are neither loans nor Philippines by the Secretary of Finance. Considering
guarantees, and hence beyond the power of the President to the reported Philippine debt of P3.80 trillion as of November
execute. 2004, the foreign public borrowing component of which
reached P1.81 trillion in November, equivalent to 47.6% of
Second, according to petitioners even assuming that total government borrowings,[23] the importance of the issues
the contracts under the Financing Program are constitutionally raised and the magnitude of the public interest involved are
permissible, yet it is only the President who may exercise the indubitable.
power to enter into these contracts and such power may not
be delegated to respondents. Thus, the Courts cognizance of this petition is also
based on the consideration that the determination of the issues
Third, petitioners argue that the Financing Program presented will have a bearing on the state of the countrys
violates several constitutional policies and that contracts economy, its international financial ratings, and perhaps even
executed or to be executed pursuant thereto were or will be the Filipinos way of life. Seen in this light, the transcendental
done by respondents with grave abuse of discretion amounting importance of the issues herein presented cannot be doubted.
to lack or excess of jurisdiction.
Where constitutional issues are properly raised in the
Petitioners contend that the Financing Program was context of alleged facts, procedural questions acquire a
made available for debts that were either fraudulently relatively minor significance.[24] We thus hold that by the very
contracted or void. In this regard, petitioners rely on a 1992 nature of the power wielded by the President, the effect of
Commission on Audit (COA) report which identified several using this power on the economy, and the well-being in general
behest loans as either contracted or guaranteed fraudulently of the Filipino nation, the Court must set aside the procedural
during the Marcos regime.[17] They posit that since these and barrier of standing and rule on the justiciable issues presented
other similar debts, such as the ones pertaining to the Bataan by the parties.
Nuclear Power Plant,[18] were eligible for buyback or
conversion under the Program, the resultant relief agreements Ripeness/Actual Case Dimension
pertaining thereto would be void for being waivers of the
Republics right to repudiate the void or fraudulently contracted Even as respondents concede the transcendental
loans. importance of the issues at bar, in their Rejoinder they ask this
Court to dismiss the Petition. Allegedly, petitioners arguments
For their part, respondents dispute the points raised by are mere attempts at abstraction.[25] Respondents are correct
petitioners. They also question the standing of petitioners to to some degree. Several issues, as shall be discussed in due
institute the present petition and the justiciability of the issues course, are not ripe for adjudication.
presented.
The allegation that respondents waived the
The Court shall tackle the procedural questions ahead of the Philippines right to repudiate void and fraudulently contracted
substantive issues. loans by executing the debt-relief agreements is, on many
levels, not justiciable.

The Courts Rulings In the first place, records do not show whether the
so-called behest loansor other allegedly void or fraudulently
Standing of Petitioners contracted loans for that matterwere subject of the debt-relief
contracts entered into under the Financing Program.
The individual petitioners are suing as citizens of
the Philippines; those among them who are of age are suing in Moreover, asserting a right to repudiate void or
their additional capacity as taxpayers. [19] It is not indicated in fraudulently contracted loans begs the question of whether
what capacity the Freedom from Debt Coalition is suing. indeed particular loans are void or fraudulently contracted.
Fraudulently contracted loans are voidable and, as such, valid
Respondents point out that petitioners have no and enforceable until annulled by the courts. On the other
standing to file the present suit since the rule allowing hand, void contracts that have already been fulfilled must be
taxpayers to assail executive or legislative acts has been declared void in view of the maxim that no one is allowed to
applied only to cases where the constitutionality of a statute is take the law in his own hands.[26] Petitioners theory depends
involved. At the same time, however, they urge this Court to on a prior annulment or declaration of nullity of the pre-existing
exercise its wide discretion and waive petitioners lack of loans, which thus far have not been submitted to this Court.
standing. They invoke the transcendental importance of Additionally, void contracts are unratifiable by their very nature;
resolving the validity of the questioned debt-relief contracts they are null and void ab initio. Consequently, from the
and others of similar import. viewpoint of civil law, what petitioners present as the Republics
right to repudiate is yet a contingent right, one which cannot be
The recent trend on locus standi has veered towards allowed as an anticipatory basis for annulling the debt-relief
a liberal treatment in taxpayers suits. In Tatad v. Garcia contracts. Petitioners contention that the debt-relief
FINALS CONSTITUTION I ACJUCO 261

agreements are tantamount to waivers of the Republics right Republic did not waive any right to repudiate void or
to repudiate so-called behest loans is without legal foundation. fraudulently contracted loans, it having incorporated a no-
waiver clause in the agreements.[33]
It may not be amiss to recognize that there are many
advocates of the position that the Republic should renege on Substantive Issues
obligations that are considered as illegitimate. However,
should the executive branch unilaterally, and possibly even It is helpful to put the matter in perspective before moving on
without prior court determination of the validity or invalidity of to the merits. The Financing Program extinguished portions of
these contracts, repudiate or otherwise declare to the the countrys pre-existing loans
international community its resolve not to recognize a certain
set of illegitimate loans, adverse repercussions [27] would come
into play. Dr. Felipe Medalla, former Director General of the through either debt buyback or bond-conversion. The buyback
National Economic Development Authority, has warned, thus: approach essentially pre-terminated portions of public debts
while the bond-conversion scheme extinguished public debts
One way to reduce debt service is through the obtention of a new loan by virtue of a sovereign
to repudiate debts, totally or selectively. bond issuance, the proceeds of which in turn were used for
Taken to its limit, however, such a strategy terminating the original loan.
would put the Philippines at such odds with
too many enemies. Foreign commercial First Issue: The Scope of Section 20, Article VII
banks by themselves and without the
cooperation of creditor governments, For their first constitutional argument, petitioners
especially the United States, may not be in submit that the buyback and bond-conversion schemes do not
a position to inflict much damage, but constitute the loan contract or guarantee contemplated in the
concerted sanctions from commercial Constitution and are consequently prohibited. Sec. 20, Art. VII
banks, multilateral financial institutions and of the Constitution provides, viz:
creditor governments would affect not only
our sources of credit but also our access to
markets for our exports and the level of The President may contract or
development assistance. . . . [T]he country guarantee foreign loans in behalf of the
might face concerted sanctions even if Republic of the Philippines with the prior
debts were repudiated only selectively. concurrence of the Monetary Board and
subject to such limitations as may be
The point that must be stressed is provided under law. The Monetary Board
that repudiation is not an attractive shall, within thirty days from the end of
alternative if net payments to creditors in every quarter of the calendar year, submit
the short and medium-run can be reduced to the Congress a complete report of its
through an agreement (as opposed to a decisions on applications for loans to be
unilaterally set ceiling on debt service contracted or guaranteed by the
payments) which provides for both government or government-owned and
rescheduling of principal and capitalization controlled corporations which would have
of interest, or its equivalent in new loans, the effect of increasing the foreign debt,
which would make it easier for the country and containing other matters as may be
to pay interest.[28] provided by law.

Sovereign default is not new to the Philippine setting.


In October 1983, the Philippines declared a moratorium on
principal payments on its external debts that eventually
On Bond-conversion

Loans are transactions wherein the owner of a


lasted four years,[29] that virtually closed the countrys access to property allows another party to use the property and where
new foreign money[30] and drove investors to leave the customarily, the latter promises to return the property after a
Philippine market, resulting in some devastating specified period with payment for its use, called interest. [34] On
consequences.[31] It would appear then that this beguilingly the other hand, bonds are interest-bearing or discounted
attractive and dangerously simplistic solution deserves the government or corporate securities that obligate the issuer to
utmost circumspect cogitation before it is resorted to. pay the bondholder a specified sum of money, usually at
specific intervals, and to repay the principal amount of the loan
In any event, the discretion on the matter lies not with at maturity.[35] The word bond means contract, agreement, or
the courts but with the executive. Thus, the Program was guarantee. All of these terms are applicable to the securities
conceptualized as an offshoot of the decision made by then known as bonds. An investor who purchases a bond is lending
money to the issuer, and the bond represents the issuers
contractual promise to pay interest and repay principal
according to specific terms. A short-term bond is often called a
President Aquino that the Philippines should recognize its note.[36]
sovereign debts[32] despite the controversy that engulfed many
debts incurred during the Marcos era. It is a scheme whereby The language of the Constitution is simple and
the Philippines restructured its debts following a negotiated clear as it is broad. It allows the President to contract and
approach instead of a default approach to manage the bleak guarantee foreign loans. It makes no prohibition on the
Philippine debt situation. issuance of certain kinds of loans or distinctions as to
which kinds of debt instruments are more onerous than
As a final point, petitioners have no real basis to fret others. This Court may not ascribe to the Constitution
over a possible waiver of the right to repudiate void contracts. meanings and restrictions that would unduly burden the
Even assuming that spurious loans had become the subject of powers of the President. The plain, clear and
debt-relief contracts, respondents unequivocally assert that the unambiguous language of the Constitution should be
FINALS CONSTITUTION I ACJUCO 262

construed in a sense that will allow the full exercise of the


power provided therein. It would be the worst kind of inability and/or unwillingness to pay the
judicial legislation if the courts were to misconstrue and indebtedness.[44] Petitioners have not presented a plausible
change the meaning of the organic act. reason that would preclude the Philippines from acting in a
The only restriction that the Constitution provides, similar fashion, should it so opt.
aside from the prior concurrence of the Monetary Board, is that
the loans must be subject to limitations provided by law. In this
regard, we note that Republic Act (R.A.) No. 245 as amended This theory may even be dismissed in a
by Pres. Decree (P.D.) No. 142, s. 1973, entitled An Act perfunctory manner since petitioners are merely
Authorizing the Secretary of Finance to Borrow to Meet Public expecting that the Philippines would opt to restructure the
Expenditures Authorized by Law, and for Other bonds but with the negotiable character of the bonds,
Purposes, allows foreign loans to be contracted in the form would be prevented from so doing. This is a contingency
of, inter alia, bonds. Thus: which petitioners do not assert as having come to pass or
even imminent. Consummated acts of the executive
Sec. 1. In order to meet public expenditures cannot be struck down by this Court merely on the basis
authorized by law or to provide for the of petitioners anticipatory cavils.
purchase, redemption, or refunding of any
obligations, either direct or guaranteed of
the Philippine Government, the Secretary On the Buyback Scheme
of Finance, with the approval of the
President of the Philippines, after In their Comment, petitioners assert that the power to
consultation with the Monetary Board, pay public debts lies with Congress and was deliberately
is authorized to borrow from time to
time on the credit of the Republic of the
Philippines such sum or sums as in his
judgment may be necessary, and to withheld by the Constitution from the President. [45] It is true that
issue therefor evidences of in the balance of power between the three branches of
indebtedness of the Philippine government, it is Congress that manages the countrys coffers
Government." by virtue of its taxing and spending powers. However, the law-
Such evidences of indebtedness may be making authority has promulgated a law ordaining an
of the following types: automatic appropriations provision for debt servicing[46] by
virtue of which the President is empowered to execute debt
.... payments without the need for further appropriations.
Regarding these legislative enactments, this Court has
c. Treasury bonds, notes, securities or held, viz:
other evidences of indebtedness having
maturities of one year or more but not Congress deliberates or acts on the budget
exceeding twenty-five years from the proposals of the President, and Congress
date of issue. (Emphasis supplied.) in the exercise of its own judgment and
wisdom formulates an appropriation act
precisely following the process established
Under the foregoing provisions, sovereign bonds by the Constitution, which specifies that no
may be issued not only to supplement government money may be paid from the Treasury
expenditures but also to provide for the except in accordance with an appropriation
purchase,[37] redemption,[38] or refunding[39] of any obligation, made by law.
either direct or guaranteed, of the Philippine Government.
Debt service is not included in the General
Appropriation Act, since authorization
Petitioners, however, point out that a supposed therefor already exists under RA Nos. 4860
difference between contracting a loan and issuing bonds is that and 245, as amended, and PD 1967.
the former creates a definite creditor-debtor relationship Precisely in the light of this subsisting
between the parties while the latter does not.[40] They explain authorization as embodied in said Republic
that a contract of loan enables the debtor to restructure or Acts and PD for debt service, Congress
novate the loan, which benefit is lost upon the conversion of does not concern itself with details for
the debts to bearer bonds such that the Philippines surrenders implementation by the Executive, but
the novatable character of a loan contract for the irrevocable largely with annual levels and approval
and unpostponable demandability of a bearer thereof upon due deliberations as part of
bond.[41] Allegedly, the Constitution prohibits the President the whole obligation program for the year.
from issuing bonds which are far more onerous than loans. [42] Upon such approval, Congress has spoken
and cannot be said to have delegated its
This line of thinking is flawed to say the least. The wisdom to the Executive, on whose part lies
negotiable character of the subject bonds is not mutually the implementation or execution of the
exclusive with the Republics freedom to negotiate with legislative wisdom.[47]
bondholders for the revision of the terms of the debt. Moreover,
the securities market provides some flexibilityif the Philippines
wants to pay in advance, it can buy out its bonds in the market; Specific legal authority for the buyback of loans is established
if interest rates go down but the Philippines does not have under Section 2 of Republic Act (R.A.) No. 240, viz:
money to retire the bonds, it can replace the old bonds with
new ones; if it defaults on the bonds, the bondholders shall Sec. 2. The Secretary of
organize and bring about a re-negotiation or settlement.[43] In Finance shall cause to be paid
fact, several countries have restructured their sovereign bonds out of any moneys in the
in view either of National Treasury not
otherwise appropriated, or
from any sinking funds
FINALS CONSTITUTION I ACJUCO 263

provided for the purpose by payments or to make payments unavailing by either


law, any interest falling due, or restructuring the loans or even refusing to make any payment
accruing, on any portion of the altogether.
public debt authorized by law.
He shall also cause to be paid More fundamentally, when taken in the context of
out of any such money, or from sovereign debts, a buyback is simply the purchase by the
any such sinking funds the sovereign issuer of its own debts at a discount. Clearly then,
principal amount of any the objection to the validity of the buyback scheme is without
obligations which have basis.
matured, or which have been
called for redemption or for which Second Issue: Delegation of Power
redemption has been demanded
in accordance with terms Petitioners stress that unlike other powers which may
prescribed by him prior to date of be validly delegated by the President, the power to incur
issue: Provided, however, That foreign debts is expressly reserved by the Constitution in the
he may, if he so chooses and if person of the President. They argue that the gravity by which
the holder is willing, exchange the exercise of the power will affect the Filipino nation requires
any such obligation with any that the President alone must exercise this power. They submit
other direct or guaranteed that the requirement of prior concurrence of an entity
obligation or obligations of the specifically named by the Constitutionthe Monetary
Philippine Government of Boardreinforces the submission that not respondents but the
equivalent value. In the case of President alone and personally can validly bind the country.
interest-bearing obligations, he
shall pay not less than their face Petitioners position is negated both by explicit
value; in the case of obligations constitutional[52] and legal[53] imprimaturs, as well as the
issued at a discount he shall pay doctrine of qualified political agency.
the face value at maturity; or, if
redeemed prior to maturity, The evident exigency of having the Secretary of
such portion of the face value Finance implement the decision of the President to execute the
as is prescribed by the terms debt-relief contracts is made manifest by the fact that the
and conditions under which process of establishing and executing a strategy for managing
such obligations were the governments debt is deep within the realm of the expertise
originally issued. (Emphasis of the Department of Finance, primed as it is to raise the
supplied.) required amount of funding, achieve its risk and cost
objectives, and meet any other sovereign debt management
The afore-quoted provisions of law specifically allow the goals.[54]
President to pre-terminate debts without further action from
Congress. If, as petitioners would have it, the President
were to personally exercise every aspect of the foreign
borrowing power, he/she would have to pause from
running the country long enough to focus on a welter of
Petitioners claim that the buyback scheme is neither time-consuming detailed activitiesthe propriety of
a guarantee nor a loan since its underlying intent is to incurring/guaranteeing loans, studying and choosing
extinguish debts that are not yet due and among the many methods that may be taken toward this
demandable.[48] Thus, they suggest that contracts entered end, meeting countless times with creditor
pursuant to the buyback scheme are unconstitutional for not representatives to negotiate, obtaining the concurrence of
being among those contemplated in Sec. 20, Art. VII of the the Monetary Board, explaining and defending the
Constitution. negotiated deal to the public, and more often than not,
flying to the agreed place of execution to sign the
Buyback is a necessary power which springs from documents. This sort of constitutional interpretation would
the grant of the foreign borrowing power. Every statute is negate the very existence of cabinet positions and the
understood, by implication, to contain all such provisions as respective expertise which the holders thereof are
may be necessary to effectuate its object and purpose, or to accorded and would unduly hamper the Presidents
make effective rights, powers, privileges or jurisdiction which it effectivity in running the government.
grants, including all such collateral and subsidiary
consequences as may be fairly and logically inferred from its Necessity thus gave birth to the doctrine of qualified
terms.[49] The President is not empowered to borrow money political agency, later adopted in Villena v. Secretary of
from foreign banks and governments on the credit of the the Interior[55] from American jurisprudence, viz:
Republic only to be left bereft of authority to implement the
payment despite appropriations therefor. With reference to the Executive
Department of the government, there is
Even petitioners concede that [t]he Constitution, as a one purpose which is crystal-clear and is
rule, does not enumeratelet alone enumerate allthe acts which readily visible without the projection of
the President (or any other public officer) may not judicial searchlight, and that is the
establishment of a single, not plural,
Executive. The first section of Article VII of
the Constitution, dealing with the Executive
do,[50] and [t]he fact that the Constitution does not explicitly bar Department, begins with the enunciation of
the President from exercising a power does not mean that he the principle that "The executive power
or she does not have that power.[51] It is inescapable from the shall be vested in a President of
standpoint of reason and necessity that the authority to the Philippines." This means that the
contract foreign loans and guarantees without restrictions on President of the Philippines is the
payment or manner thereof coupled with the availability of the Executive of the Government of
corresponding appropriations, must include the power to effect the Philippines, and no other. The heads of
FINALS CONSTITUTION I ACJUCO 264

the executive departments occupy political


positions and hold office in an advisory
capacity, and, in the language of Thomas
Jefferson, "should be of the President's
bosom confidence" (7 Writings, Ford ed., akin to any contractual obligation undertaken by the sovereign,
498), and, in the language of Attorney- which arises not from any extraordinary incident, but from the
General Cushing (7 Op., Attorney-General, established functions of governance.
453), "are subject to the direction of the
President." Without minimizing the Another important qualification must be made. The
importance of the heads of the various Secretary of Finance or any designated alter ego of the
departments, their personality is in reality President is bound to secure the latters prior consent to or
but the projection of that of the President. subsequent ratification of his acts. In the matter of contracting
Stated otherwise, and as forcibly or guaranteeing foreign loans, the repudiation by the President
characterized by Chief Justice Taft of the of the very acts performed in this regard by the alter ego will
Supreme Court of the United States, "each definitely have binding effect. Had petitioners herein
head of a department is, and must be, the succeeded in demonstrating that the President actually
President's alter ego in the matters of that withheld approval and/or repudiated the Financing Program,
department where the President is required there could be a cause of action to nullify the acts of
by law to exercise authority" (Myers vs. respondents. Notably though, petitioners do not assert that
United States, 47 Sup. Ct. Rep., 21 at 30; respondents pursued the Program without prior authorization
272 U. S., 52 at 133; 71 Law. ed., 160).[56] of the President or that the terms of the contract were agreed
upon without the Presidents authorization. Congruent with the
As it was, the backdrop consisted of a major policy avowed preference of then President Aquino to honor and
determination made by then President Aquino that sovereign restructure existing foreign debts, the lack of showing that she
debts have to be respected and the concomitant reality that countermanded the acts of respondents leads us to conclude
the Philippines did not have enough funds to pay the debts. that said acts carried presidential approval.
Inevitably, it fell upon the Secretary of Finance, as the alter
ego of the President regarding the sound and efficient
management of the financial resources of the
Government,[57] to formulate a scheme for the implementation
of the policy publicly expressed by the President herself.
With constitutional parameters already established,
Nevertheless, there are powers vested in the we may also note, as a source of suppletory guidance, the
President by the Constitution which may not be delegated to or provisions of R.A. No. 245. The afore-quoted Section 1 thereof
exercised by an agent or alter ego of the President. Justice empowers the Secretary of Finance with the approval of the
Laurel, in his ponencia in Villena, makes this clear: President and after consultation[59] of the Monetary Board, to
borrow from time to time on the credit of the Republic of the
Withal, at first blush, the argument of Philippines such sum or sums as in his judgment may be
ratification may seem plausible under the necessary, and to issue therefor evidences of indebtedness of
circumstances, it should be observed that the Philippine Government. Ineluctably then, while the
there are certain acts which, by their very President wields the borrowing power it is the Secretary of
nature, cannot be validated by subsequent Finance who normally carries out its thrusts.
approval or ratification by the President.
There are certain constitutional powers and
prerogatives of the Chief Executive of the In our recent rulings in Southern Cross Cement
Nation which must be exercised by him in Corporation v. The Philippine Cement Manufacturers
person and no amount of approval or Corp.,[60] this Court had occasion to examine the authority
ratification will validate the exercise of any granted by Congress to the Department of Trade and Industry
of those powers by any other person. Such, (DTI) Secretary to impose safeguard measures pursuant to the
for instance, in his power to suspend the Safeguard Measures Act. In doing so, the Court was impelled
writ of habeas corpus and proclaim martial to construe Section 28(2), Article VI of the Constitution, which
law (PAR. 3, SEC. 11, Art. VII) and the allowed Congress, by law, to authorize the President to fix
exercise by him of the benign prerogative within specified limits, and subject to such limitations and
of mercy (par. 6, sec. 11, idem).[58] restrictions as it may impose, tariff rates, import and export
quotas, tonnage and wharfage dues, and other duties or
imposts within the framework of the national development
These distinctions hold true to this day. There are certain program of the Government.[61]
presidential powers which arise out of exceptional
circumstances, and if exercised, would involve the suspension While the Court refused to uphold the broad
of fundamental freedoms, or at least call for the supersedence construction of the grant of power as preferred by the DTI
of executive prerogatives over those exercised by co-equal Secretary, it nonetheless tacitly acknowledged that Congress
branches of government. The declaration of martial law, the could designate the DTI Secretary, in his capacity as alter
suspension of the writ of habeas corpus, and the exercise of ego of the President, to exercise the authority vested on the
the pardoning power notwithstanding the judicial determination chief executive under Section 28(2), Article VI.[62] At the same
of guilt of the accused, all fall within this special class that time, the Court emphasized that since Section 28(2), Article VI
demands the exclusive exercise by the President of the authorized Congress to impose limitations and restrictions on
constitutionally vested power. The list is by no means the authority of the President to impose tariffs and imposts, the
exclusive, but there must be a showing that the executive DTI Secretary was necessarily subjected to the same
power in question is of similar gravitas and exceptional import. restrictions that Congress could impose on the President in the
exercise of this taxing power.
We cannot conclude that the power of the President
to contract or guarantee foreign debts falls within the same Similarly, in the instant case, the Constitution
exceptional class. Indubitably, the decision to contract or allocates to the President the exercise of the foreign borrowing
guarantee foreign debts is of vital public interest, but only power subject to such limitations as may be provided under
FINALS CONSTITUTION I ACJUCO 265

law. Following Southern Cross, but in line with the limitations Debt Councils endorsement of the approval of the financing
as defined in Villena, the presidential prerogative may be package containing the debt-
exercised by the Presidents alter ego, who in this case is the
Secretary of Finance.

It bears emphasis that apart from the Constitution, relief agreements and issuance of a Motion to Urge the
there is also a relevant statute, R.A. No. 245, that establishes Philippine Debt Negotiating Panel to continue with the
the parameters by which the alter ego may act in behalf of the negotiation on the aforesaid package. [70]
President with respect to the borrowing power. This law
expressly provides that the Secretary of Finance may enter into
foreign borrowing contracts. This law neither amends nor goes Even with these justifications, respondents aver that their acts
contrary to the Constitution but merely implements the subject are within the arena of political questions which, based on the
provision in a manner consistent with the structure of the doctrine of separation of powers,[71] the judiciary must leave
Executive Department and the alter ego doctine. In this regard, without interference lest the courts substitute their judgment for
respondents have declared that they have followed the that of the official concerned and decide a matter which by its
restrictions provided under R.A. No. 245, [63] which include the nature or law is for the latter alone to decide. [72]
requisite presidential authorization and which, in the absence
of proof and even allegation to the contrary, should be On the other hand, in furtherance of their argument on
regarded in a fashion congruent with the presumption of respondents violation of constitutional policies, petitioners cite
regularity bestowed on acts done by public officials. an article of Jude Esguerra, The 1992 Buyback and
Securitization Agreement with Philippine Commercial Bank
Moreover, in praying that the acts of the Creditors,[73] in illustrating a best-case scenario in entering the
respondents, especially that of the Secretary of Finance, subject debt-relief agreements. The computation results in a
be nullified as being in violation of a restrictive yield of $218.99 million, rather
constitutional interpretation, petitioners in effect would
have this Court declare R.A. No. 245 unconstitutional. We
will not strike
than the $2,041.00 million claimed by the debt
negotiators.[74] On the other hand, the worst-case scenario
allegedly is that a net amount of $1.638 million will flow out of
down a law or provisions thereof without so much as a the country as a result of the debt package. [75]
direct attack thereon when simple and logical statutory
construction would suffice. Assuming the accuracy of the foregoing for the nonce, despite
the watered-down parameters of petitioners computations, we
Petitioners also submit that the unrestricted character of the can make no conclusion other than that respondents efforts
Financing Program violates the framers intent behind Section were geared towards debt-relief with marked positive results
20, Article VII to restrict the power of the President. This intent, and towards achieving the constitutional policies which
petitioners note, is embodied in the proviso in Sec. 20, Art. VII, petitioners so hastily declare as having been violated by
which states that said power is subject to such limitations as respondents. We recognize that as with other schemes
may be provided under law. However, as previously discussed, dependent on volatile market and economic structures, the
the debt-relief contracts are governed by the terms of R.A. No. contracts entered into by respondents may possibly have a net
245, as amended by P.D. No. 142 s. 1973, and therefore were outflow and therefore negative result. However, even
not developed in an unrestricted setting. petitioners call this latter event the worst-case scenario. Plans
are seldom foolproof. To ask the Court to strike down debt-
relief contracts, which, according to independent third party
Third Issue: Grave Abuse of Discretion and evaluations using historically-suggested rates would result in
Violation of Constitutional Policies substantial debt-relief,[76] based merely on the possibility of
petitioners worst-case scenario projection, hardly seems
reasonable.
We treat the remaining issues jointly, for in view of the
foregoing determination, the general allegation of grave abuse
of discretion on the part of respondents would arise from the Moreover, the policies set by the Constitution as litanized by
purported violation of various state policies as expressed in the petitioners are not a panacea that can annul every
Constitution. governmental act sought to be struck down. The gist of
petitioners arguments on violation of constitutional policies and
Petitioners allege that the Financing Program violates the grave abuse of discretion boils down to their allegation that the
constitutional state policies to promote a social order that will debt-relief agreements entered into by respondents do not
ensure the prosperity and independence of the nation and free deliver the kind of debt-relief that petitioners would want.
the people from poverty,[64] foster social justice in all phases of Petitioners cite the aforementioned article in stating that that
national development,[65] and develop a self-reliant and the agreement achieves little that cannot be gained through
independent national economy effectively controlled by less complicated means like postponing (rescheduling)
Filipinos;[66] thus, the contracts executed or to be executed principal payments,[77] thus:
pursuant thereto were or would be tainted by a grave abuse of
discretion amounting to lack or excess of jurisdiction. [T]he price of success in putting together
this debt-relief package (indicates) the
Respondents cite the following in support of the propriety of possibility that a simple rescheduling
their acts:[67] (1) a Department of Finance study showing that agreement may well turn out to be less
as a result of the implementation of voluntary debt reductions expensive than this comprehensive debt-
schemes, the countrys debt stock was reduced by U.S. $4.4 relief package. This means that in the next
billion as of December 1991; [68] (2) revelations made by six years the humble and simple
independent individuals made in a hearing before the Senate rescheduling process may well be the
Committee on Economic Affairs indicating that the assailed lesser evil because there is that distinct
agreements would bring about substantial benefits to the possibility that less money will flow out of
country;[69] and (3) the Joint Legislative-Executive Foreign the country as a result.
FINALS CONSTITUTION I ACJUCO 266

REPRESENTATIVES LUZ ILAGAN AND EMERENCIANA


DE JESUS, ACT TEACHERS PARTY-LIST
REPRESENTATIVE ANTONIO L. TINIO, ANAKPAWIS
PARTY-LIST REPRESENTATIVE FERNANDO HICAP,
Note must be taken that from these citations, petitioners submit KABATAAN PARTY-LIST REPRESENTATIVE TERRY
that there is possibly a better way to go about debt RIDON, MAKABAYANG KOALISYON NG MAMAMAYAN
rescheduling and, on that basis, insist that the acts of (MAKABAYAN), REPRESENTED BY SATURNINO
respondents must be struck down. These are rather tenuous OCAMPO AND LIZA MAZA, BIENVENIDO LUMBERA,
grounds to condemn the subject agreements as violative of JOEL C. LAMANGAN, RAFAEL MARIANO, SALVADOR
constitutional principles. FRANCE, ROGELIO M. SOLUTA, AND CLEMENTE G.
BAUTISTA, Petitioners,
Conclusion vs.
DEPARTMENT OF NATIONAL DEFENSE (DND)
The raison d etre of the Financing Program is to manage debts SECRETARY VOLTAIRE GAZMIN, DEPARTMENT OF
incurred by the Philippines in a manner that will lessen the FOREIGN AFFAIRS SECRETARY ALBERT DEL ROSARIO,
burden on the Filipino taxpayersthus the term debt-relief EXECUTIVE SECRETARY PAQUITO N. OCHOA, JR.,
agreements. The measures objected to by petitioners were not ARMED FORCES OF THE PHILIPPINES CHIEF OF STAFF
aimed at incurring more debts but at terminating pre-existing GENERAL EMMANUEL T. BAUTISTA, DEFENSE
debts and were backed by the know-how of the countrys UNDERSECRETARY PIO LORENZO BATINO,
economic managers as affirmed by third party empirical AMBASSADOR LOURDES YPARRAGUIRRE,
analysis. AMBASSADOR J. EDUARDO MALAYA, DEPARTMENT OF
JUSTICE UNDERSECRETARY FRANCISCO BARAAN III,
That the means employed to achieve the goal of AND DND ASSISTANT SECRETARY FOR STRATEGIC
debt-relief do not sit well with petitioners is beyond the power ASSESSMENTS RAYMUND JOSE QUILOP AS
of this Court to remedy. The exercise of the power of judicial CHAIRPERSON AND MEMBERS, RESPECTIVELY, OF THE
review is merely to checknot supplantthe Executive, or to NEGOTIATING PANEL FOR THE PHILIPPINES ON
simply ascertain whether he has gone beyond the EDCA, Respondents.
constitutional limits of his jurisdiction but not to exercise the
power vested in him or to determine the wisdom of his act.[78] In x-----------------------x
cases where the main purpose is to nullify governmental acts
whether as unconstitutional or done with grave abuse of
discretion, there is a strong presumption in favor of the validity KILUSANG MAYO UNO, REPRESENTED BY ITS
of the assailed acts. The heavy onus is in on petitioners to CHAIRPERSON, ELMER LABOG, CONFEDERATION FOR
overcome the presumption of regularity. UNITY, RECOGNITION AND ADVANCEMENT OF
GOVERNMENT EMPLOYEES (COURAGE),
We find that petitioners have not sufficiently REPRESENTED BY ITS NATIONAL PRESIDENT
established any basis for the Court to declare the acts of FERDINAND GAITE, NATIONAL FEDERATION OF LABOR
respondents as unconstitutional. UNIONS-KILUSANG MAYO UNO, REPRESENTED BY ITS
NATIONAL PRESIDENT JOSELITO USTAREZ, NENITA
GONZAGA, VIOLETA ESPIRITU, VIRGINIA FLORES, AND
WHEREFORE the petition is hereby DISMISSED. No ARMANDO TEODORO, JR., Petitioners-in-Intervention,
costs.SO ORDERED. RENE A.Q. SAGUISAG, JR., Petitioner-in-Intervention.

DECISION
G.R. No. 212426
SERENO, J.:
RENE A.V. SAGUISAG, WIGBERTO E. TAÑADA,
FRANCISCO "DODONG" NEMENZO, JR., SR. MARY JOHN The petitions1 before this Court question the constitutionality of
MANANZAN, PACIFICO A. AGABIN, ESTEBAN "STEVE" the Enhanced Defense Cooperation Agreement (EDCA)
SALONGA, H. HARRY L. ROQUE, JR., EVALYN G. URSUA, between the Republic of the Philippines and the United States
EDRE U. OLALIA, DR. CAROL PAGADUAN-ARAULLO, DR. of America (U.S.). Petitioners allege that respondents
ROLAND SIMBULAN, AND TEDDY CASIÑO, Petitioners, committed grave abuse of discretion amounting to lack or
vs. excess of jurisdiction when they entered into EDCA with the
EXECUTIVE SECRETARY PAQUITO N. OCHOA, JR., U.S.,2 claiming that the instrument violated multiple
DEPARTMENT OF NATIONAL DEFENSE SECRETARY constitutional provisions.3 In reply, respondents argue that
VOLTAIRE GAZMIN, DEPARTMENT OF FOREIGN petitioners lack standing to bring the suit. To support the
AFFAIRS SECRETARY ALBERT DEL ROSARIO, JR., legality of their actions, respondents invoke the 1987
DEPARTMENT OF BUDGET AND MANAGEMENT Constitution, treaties, and judicial precedents. 4
SECRETARY FLORENCIO ABAD, AND ARMED FORCES
OF THE PHILIPPINES CHIEF OF STAFF GENERAL
A proper analysis of the issues requires this Court to lay down
EMMANUEL T. BAUTISTA, Respondents.
at the outset the basic parameters of the constitutional powers
and roles of the President and the Senate in respect of the
x-----------------------x above issues. A more detailed discussion of these powers and
roles will be made in the latter portions.
G.R. No. 212444
I. BROAD CONSTITUTIONAL CONTEXT OF THE POWERS
BAGONG ALYANSANG MAKABAYAN (BAYAN), OF THE PRESIDENT: DEFENSE, FOREIGN RELATIONS,
REPRESENTED BY ITS SECRETARY GENERAL RENATO AND EDCA
M. REYES, JR., BAYAN MUNA PARTY-LIST
REPRESENTATIVES NERI J. COLMENARES AND A. The Prime Duty of the State and the Consolidation of
CARLOS ZARATE, GABRIELA WOMEN'S PARTY-LIST Executive Power in the President
FINALS CONSTITUTION I ACJUCO 267

Mataimtim kong pinanunumpaan (o pinatotohanan) na suspension. The same provision provides for the Supreme
tutuparin ko nang buong katapatan at sigasig ang aking mga Court's review of the factual basis for the proclamation or
tungkulin bilang Pangulo (o Pangalawang Pangulo o suspension, as well as the promulgation of the decision within
Nanunungkulang Pangulo) ng Pilipinas, pangangalagaan at 30 days from filing.
ipagtatanggol ang kanyang Konstitusyon, ipatutupad ang mga
batas nito, magiging makatarungan sa bawat tao, at itatalaga C. The power and duty to conduct foreign relations
ang aking sarili sa paglilingkod sa Bansa. Kasihan nawa aka
ng Diyos.
The President also carries the mandate of being the sole organ
in the conduct of foreign relations.15 Since every state has the
- Panunumpa sa Katungkulan ng Pangulo capacity to interact with and engage in relations with other
ng Pilipinas ayon sa Saligang Batas 5 sovereign states,16 it is but logical that every state must vest in
an agent the authority to represent its interests to those other
The 1987 Constitution has "vested the executive power in the sovereign states.
President of the Republic of the Philippines." 6 While the
vastness of the executive power that has been consolidated in The conduct of foreign relations is full of complexities and
the person of the President cannot be expressed fully in one consequences, sometimes with life and death significance to
provision, the Constitution has stated the prime duty of the the nation especially in times of war. It can only be entrusted
government, of which the President is the head: to that department of government which can act on the basis
of the best available information and can decide with
The prime duty of the Government is to serve and protect decisiveness. x x x It is also the President who possesses the
the people. The Government may call upon the people to most comprehensive and the most confidential information
defend the State and, in the fulfillment thereof, all citizens may about foreign countries for our diplomatic and consular officials
be required, under conditions provided by law, to render regularly brief him on meaningful events all over the world. He
personal military or civil service.7 (Emphases supplied) has also unlimited access to ultra-sensitive military intelligence
data. In fine, the presidential role in foreign affairs is dominant
B. The duty to protect the territory and the citizens of the and the President is traditionally accorded a wider degree of
discretion in the conduct of foreign affairs. The regularity, nay,
Philippines, the power to call upon the people to defend
the State, and the President as Commander-in-Chief validity of his actions are adjudged under less stringent
standards, lest their judicial repudiation lead to breach of an
international obligation, rupture of state relations, forfeiture of
The duty to protect the State and its people must be carried out confidence, national embarrassment and a plethora of other
earnestly and effectively throughout the whole territory of the problems with equally undesirable consequences. 17
Philippines in accordance with the constitutional provision on
national territory. Hence, the President of the Philippines, as
the sole repository of executive power, is the guardian of the The role of the President in foreign affairs is qualified by the
Philippine archipelago, including all the islands and waters Constitution in that the Chief Executive must give paramount
embraced therein and all other territories over which it has importance to the sovereignty of the nation, the integrity of its
sovereignty or jurisdiction. These territories consist of its territory, its interest, and the right of the sovereign Filipino
terrestrial, fluvial, and aerial domains; including its territorial people to self-determination.18 In specific provisions, the
sea, the seabed, the subsoil, the insular shelves, and other President's power is also limited, or at least shared, as in
Section 2 of Article II on the conduct of war; Sections 20 and
submarine areas; and the waters around, between, and
connecting the islands of the archipelago, regardless of their 21 of Article VII on foreign loans, treaties, and international
agreements; Sections 4(2) and 5(2)(a) of Article VIII on the
breadth and dimensions.8
judicial review of executive acts; Sections 4 and 25 of Article
XVIII on treaties and international agreements entered into
To carry out this important duty, the President is equipped with prior to the Constitution and on the presence of foreign military
authority over the Armed Forces of the Philippines troops, bases, or facilities.
(AFP),9 which is the protector of the people and the state. The
AFP's role is to secure the sovereignty of the State and the
D. The relationship between the two major presidential
integrity of the national territory.10 In addition, the Executive is
constitutionally empowered to maintain peace and order; functions and the role of the Senate
protect life, liberty, and property; and promote the general
welfare.11 Clearly, the power to defend the State and to act as its
representative in the international sphere inheres in the person
In recognition of these powers, Congress has specified that the of the President. This power, however, does not crystallize into
President must oversee, ensure, and reinforce our defensive absolute discretion to craft whatever instrument the Chief
capabilities against external and internal threats 12 and, in the Executive so desires. As previously mentioned, the Senate has
same vein, ensure that the country is adequately prepared for a role in ensuring that treaties or international agreements the
all national and local emergencies arising from natural and President enters into, as contemplated in Section 21 of Article
man-made disasters.13 VII of the Constitution, obtain the approval of two-thirds of its
members.

To be sure, this power is limited by the Constitution itself. To


illustrate, the President may call out the AFP to prevent or Previously, treaties under the 1973 Constitution required
suppress instances of lawless violence, invasion or ratification by a majority of the Batasang Pambansa,19except
in instances wherein the President "may enter into international
rebellion,14 but not suspend the privilege of the writ of habeas
corpus for a period exceeding 60 days, or place the Philippines treaties or agreements as the national welfare and interest may
require."20 This left a large margin of discretion that the
or any part thereof under martial law exceeding that same
span. In the exercise of these powers, the President is also President could use to bypass the Legislature altogether. This
was a departure from the 1935 Constitution, which explicitly
duty-bound to submit a report to Congress, in person or in
writing, within 48 hours from the proclamation of martial law or gave the President the power to enter into treaties only with the
the suspension of the privilege of the writ of habeas corpus; concurrence of two-thirds of all the Members of the
and Congress may in turn revoke the proclamation or Senate.21 The 1987 Constitution returned the Senate's
FINALS CONSTITUTION I ACJUCO 268

power22 and, with it, the legislative's traditional role in foreign areas that would be covered by the American military bases in
affairs.23 the country.39 This treaty eventually led to the creation of the
post-colonial legal regime on which would hinge the continued
presence of U.S. military forces until 1991: the Military Bases
The responsibility of the President when it comes to treaties
and international agreements under the present Constitution is Agreement (MBA) of 1947, the Military Assistance Agreement
therefore shared with the Senate. This shared role, petitioners of 1947, and the Mutual Defense Treaty (MDT) of 1951. 40
claim, is bypassed by EDCA.
B. Former legal regime on the presence of U.S. armed
II. HISTORICAL ANTECEDENTS OF EDCA forces in the territory of an independent Philippines (1946-
1991)

A. U.S. takeover of Spanish colonization and its military


Soon after the Philippines was granted independence, the two
bases, and the transition to Philippine independence
countries entered into their first military arrangement pursuant
to the Treaty of General Relations - the 1947 MBA.41 The
The presence of the U.S. military forces in the country can be Senate concurred on the premise of "mutuality of security
traced to their pivotal victory in the 1898 Battle of Manila Bay interest,"42 which provided for the presence and operation of
during the Spanish-American War.24 Spain relinquished its 23 U.S. military bases in the Philippines for 99 years or until
sovereignty over the Philippine Islands in favor of the U.S. the year 2046.43 The treaty also obliged the Philippines to
upon its formal surrender a few months later. 25 By 1899, the negotiate with the U.S. to allow the latter to expand the existing
Americans had consolidated a military administration in the bases or to acquire new ones as military necessity might
archipelago.26 require.44

When it became clear that the American forces intended to A number of significant amendments to the 1947 MBA were
impose colonial control over the Philippine Islands, General made.45 With respect to its duration, the parties entered into
Emilio Aguinaldo immediately led the Filipinos into an all-out the Ramos-Rusk Agreement of 1966, which reduced the term
war against the U.S.27 The Filipinos were ultimately defeated of the treaty from 99 years to a total of 44 years or until
in the Philippine-American War, which lasted until 1902 and led 1991.46 Concerning the number of U.S. military bases in the
to the downfall of the first Philippine Republic. 28 The Americans country, the Bohlen-Serrano Memorandum of Agreement
henceforth began to strengthen their foothold in the provided for the return to the Philippines of 17 U.S. military
country.29 They took over and expanded the former Spanish bases covering a total area of 117,075 hectares.47 Twelve
Naval Base in Subic Bay, Zambales, and put up a cavalry post years later, the U.S. returned Sangley Point in Cavite City
called Fort Stotsenberg in Pampanga, now known as Clark Air through an exchange of notes.48 Then, through the Romulo-
Base.30 Murphy Exchange of Notes of 1979, the parties agreed to the
recognition of Philippine sovereignty over Clark and Subic
When talks of the eventual independence of the Philippine Bases and the reduction of the areas that could be used by the
Islands gained ground, the U.S. manifested the desire to U.S. military.49 The agreement also provided for the mandatory
maintain military bases and armed forces in the country. 31 The review of the treaty every five years. 50 In 1983, the parties
U.S. Congress later enacted the Hare-Hawes-Cutting Act of revised the 1947 MBA through the Romualdez-Armacost
1933, which required that the proposed constitution of an Agreement.51 The revision pertained to the operational use of
independent Philippines recognize the right of the U.S. to the military bases by the U.S. government within the context of
maintain the latter's armed forces and military bases. 32 The Philippine sovereignty,52 including the need for prior
Philippine Legislature rejected that law, as it also gave the U.S. consultation with the Philippine government on the former' s
the power to unilaterally designate any part of Philippine use of the bases for military combat operations or the
territory as a permanent military or naval base of the U.S. establishment of long-range missiles.53
within two years from complete independence. 33
Pursuant to the legislative authorization granted under
The U.S. Legislature subsequently crafted another law called Republic Act No. 9,54 the President also entered into the 1947
the Tydings-McDuffie Act or the Philippine Independence Act Military Assistance Agreement55 with the U.S. This executive
of 1934. Compared to the old Hare-Hawes-Cutting Act, the agreement established the conditions under which U.S.
new law provided for the surrender to the Commonwealth military assistance would be granted to the
Government of "all military and other reservations" of the U.S. Philippines,56 particularly the provision of military arms,
government in the Philippines, except "naval reservations and ammunitions, supplies, equipment, vessels, services, and
refueling stations."34 Furthermore, the law authorized the U.S. training for the latter's defense forces.57 An exchange of notes
President to enter into negotiations for the adjustment and in 1953 made it clear that the agreement would remain in force
settlement of all questions relating to naval reservations and until terminated by any of the parties.58
fueling stations within two years after the Philippines would
have gained independence.35 Under the Tydings-McDuffie Act, To further strengthen their defense and security
the U.S. President would proclaim the American withdrawal relationship,59 the Philippines and the U.S. next entered into
and surrender of sovereignty over the islands 10 years after the MDT in 1951. Concurred in by both the Philippine60 and the
the inauguration of the new government in the U.S.61 Senates, the treaty has two main features: first, it
Philippines.36 This law eventually led to the promulgation of the allowed for mutual assistance in maintaining and developing
1935 Philippine Constitution. their individual and collective capacities to resist an armed
attack;62 and second, it provided for their mutual self-defense
The original plan to surrender the military bases changed. 37 At in the event of an armed attack against the territory of either
the height of the Second World War, the Philippine and the party.63 The treaty was premised on their recognition that an
U.S. Legislatures each passed resolutions authorizing their armed attack on either of them would equally be a threat to the
respective Presidents to negotiate the matter of retaining security of the other.64
military bases in the country after the planned withdrawal of the
U.S.38 Subsequently, in 1946, the countries entered into the C. Current legal regime on the presence of U.S. armed
Treaty of General Relations, in which the U.S. relinquished all forces in the country
control and sovereignty over the Philippine Islands, except the
FINALS CONSTITUTION I ACJUCO 269

In view of the impending expiration of the 1947 MBA in 1991, According to the Philippine government, the conclusion of
the Philippines and the U.S. negotiated for a possible renewal EDCA was the result of intensive and comprehensive
of their defense and security relationship. 65 Termed as the negotiations in the course of almost two years.87 After eight
Treaty of Friendship, Cooperation and Security, the countries rounds of negotiations, the Secretary of National Defense and
sought to recast their military ties by providing a new the U.S. Ambassador to the Philippines signed the agreement
framework for their defense cooperation and the use of on 28 April 2014.88 President Benigno S. Aquino III ratified
Philippine installations.66 One of the proposed provisions EDCA on 6 June 2014.89 The OSG clarified during the oral
included an arrangement in which U.S. forces would be arguments90 that the Philippine and the U.S. governments had
granted the use of certain installations within the Philippine yet to agree formally on the specific sites of the Agreed
naval base in Subic.67 On 16 September 1991, the Senate Locations mentioned in the agreement.
rejected the proposed treaty.68
Two petitions for certiorari were thereafter filed before us
The consequent expiration of the 1947 MBA and the resulting assailing the constitutionality of EDCA. They primarily argue
paucity of any formal agreement dealing with the treatment of that it should have been in the form of a treaty concurred in by
U.S. personnel in the Philippines led to the suspension in 1995 the Senate, not an executive agreement.
of large-scale joint military exercises.69In the meantime, the
respective governments of the two countries agreed 70 to hold
On 10 November 2015, months after the oral arguments were
joint exercises at a substantially reduced level. 71 The military concluded and the parties ordered to file their respective
arrangements between them were revived in 1999 when they
memoranda, the Senators adopted Senate Resolution No.
concluded the first Visiting Forces Agreement (VFA).72 (SR) 105.91 The resolution expresses the "strong sense"92 of
the Senators that for EDCA to become valid and effective, it
As a "reaffirm[ation] [of the] obligations under the MDT," 73 the must first be transmitted to the Senate for deliberation and
VFA has laid down the regulatory mechanism for the treatment concurrence.
of U.S. military and civilian personnel visiting the country. 74 It
contains provisions on the entry and departure of U.S. III. ISSUES
personnel; the purpose, extent, and limitations of their
activities; criminal and disciplinary jurisdiction; the waiver of
certain claims; the importation and exportation of equipment, Petitioners mainly seek a declaration that the Executive
materials, supplies, and other pieces of property owned by the Department committed grave abuse of discretion in entering
U.S. government; and the movement of U.S. military vehicles, into EDCA in the form of an executive agreement. For this
vessels, and aircraft into and within the country. 75 The reason, we cull the issues before us:
Philippines and the U.S. also entered into a second counterpart
agreement (VFA II), which in turn regulated the treatment of A. Whether the essential requisites for judicial
Philippine military and civilian personnel visiting the U.S. 76 The review are present
Philippine Senate concurred in the first VFA on 27 May 1999. 77
B. Whether the President may enter into an
Beginning in January 2002, U.S. military and civilian personnel executive agreement on foreign military bases,
started arriving in Mindanao to take part in joint military troops, or facilities
exercises with their Filipino
counterparts.78 Called Balikatan, these exercises involved
trainings aimed at simulating joint military maneuvers pursuant C. Whether the provisions under EDCA are
to the MDT.79 consistent with the Constitution, as well as with
existing laws and treaties
In the same year, the Philippines and the U.S. entered into the
Mutual Logistics Support Agreement to "further the IV. DISCUSSION
interoperability, readiness, and effectiveness of their
respective military forces"80 in accordance with the MDT, the A. Whether the essential requisites for judicial review have
Military Assistance Agreement of 1953, and the VFA.81 The been satisfied
new agreement outlined the basic terms, conditions, and
procedures for facilitating the reciprocal provision of logistics
support, supplies, and services between the military forces of Petitioners are hailing this Court's power of judicial review in
the two countries.82 The phrase "logistics support and order to strike down EDCA for violating the Constitution. They
services" includes billeting, operations support, construction stress that our fundamental law is explicit in prohibiting the
and use of temporary structures, and storage services during presence of foreign military forces in the country, except under
an approved activity under the existing military a treaty concurred in by the Senate. Before this Court may
arrangements.83 Already extended twice, the agreement will begin to analyze the constitutionality or validity of an official act
last until 2017.84 of a coequal branch of government, however, petitioners must
show that they have satisfied all the essential requisites for
judicial review.93
D. The Enhanced Defense Cooperation Agreement
Distinguished from the general notion of judicial power, the
EDCA authorizes the U.S. military forces to have access to and power of judicial review specially refers to both the authority
conduct activities within certain "Agreed Locations" in the and the duty of this Court to determine whether a branch or an
country. It was not transmitted to the Senate on the executive's instrumentality of government has acted beyond the scope of
understanding that to do so was no longer the latter's constitutional powers.94 As articulated in Section 1,
necessary.85 Accordingly, in June 2014, the Department of Article VIII of the Constitution, the power of judicial review
Foreign Affairs (DFA) and the U.S. Embassy exchanged involves the power to resolve cases in which the questions
diplomatic notes confirming the completion of all necessary concern the constitutionality or validity of any treaty,
internal requirements for the agreement to enter into force in international or executive agreement, law, presidential decree,
the two countries.86 proclamation, order, instruction, ordinance, or
regulation.95 In Angara v. Electoral Commission, this Court
exhaustively discussed this "moderating power" as part of the
FINALS CONSTITUTION I ACJUCO 270

system of checks and balances under the Constitution. In our insurmountable obstacle to the exercise of judicial power or the
fundamental law, the role of the Court is to determine whether impenetrable shield that protects executive and legislative
a branch of government has adhered to the specific restrictions actions from judicial inquiry or review."98
and limitations of the latter's power:96
This moderating power, however, must be exercised carefully
The separation of powers is a fundamental principle in our and only if it cannot be completely avoided. We stress that our
system of government. It obtains not through express provision Constitution is so incisively designed that it identifies the
but by actual division in our Constitution. Each department of spheres of expertise within which the different branches of
the government has exclusive cognizance of matters government shall function and the questions of policy that they
within its jurisdiction, and is supreme within its own shall resolve.99 Since the power of judicial review involves the
sphere. But it does not follow from the fact that the three delicate exercise of examining the validity or constitutionality of
powers are to be kept separate and distinct that the an act of a coequal branch of government, this Court must
Constitution intended them to be absolutely unrestrained and continually exercise restraint to avoid the risk of supplanting
independent of each other. The Constitution has provided the wisdom of the constitutionally appointed actor with that of
for an elaborate system of checks and balances to secure its own.100
coordination in the workings of the various departments of the
government. x x x. And the judiciary in turn, with the
Even as we are left with no recourse but to bare our power to
Supreme Court as the final arbiter, effectively checks the check an act of a coequal branch of government - in this case
other departments in the exercise of its power to
the executive - we must abide by the stringent requirements for
determine the law, and hence to declare executive and the exercise of that power under the Constitution. Demetria v.
legislative acts void if violative of the Constitution. Alba101 and Francisco v. House of Representatives 102 cite the
"pillars" of the limitations on the power of judicial review as
xxxx enunciated in the concurring opinion of U.S. Supreme Court
Justice Brandeis in Ashwander v. Tennessee Valley
As any human production, our Constitution is of course Authority.103 Francisco104 redressed these "pillars" under the
lacking perfection and perfectibility, but as much as it was following categories:
within the power of our people, acting through their delegates
to so provide, that instrument which is the expression of 1. That there be absolute necessity of deciding a
their sovereignty however limited, has established a case
republican government intended to operate and function
as a harmonious whole, under a system of checks and 2. That rules of constitutional law shall
balances, and subject to specific limitations and be formulated only as required by the facts of the
restrictions provided in the said instrument. The case
Constitution sets forth in no uncertain language the
restrictions and limitations upon governmental powers
and agencies. If these restrictions and limitations are 3. That judgment may not be sustained on some
transcended it would be inconceivable if the Constitution other ground
had not provided for a mechanism by which to direct the
course of government along constitutional channels, for 4. That there be actual injury sustained by the
then the distribution of powers would be mere verbiage, party by reason of the operation of the statute
the bill of rights mere expressions of sentiment, and the
principles of good government mere political apothegms.
Certainly, the limitations and restrictions embodied in our 5. That the parties are not in estoppel
Constitution are real as they should be in any living
constitution. x x x. In our case, this moderating power is 6. That the Court upholds the presumption of
granted, if not expressly, by clear implication from section 2 of constitutionality
article VIII of [the 1935] Constitution.
(Emphases supplied)
The Constitution is a definition of the powers of government.
Who is to determine the nature, scope and extent of such
powers? The Constitution itself has provided for the These are the specific safeguards laid down by the Court when
instrumentality of the judiciary as the rational way. And when it exercises its power of judicial review. 105 Guided by these
the judiciary mediates to allocate constitutional pillars, it may invoke the power only when the following four
boundaries, it does not assert any superiority over the stringent requirements are satisfied: (a) there is an actual case
other departments; it does not in reality nullify or or controversy; (b) petitioners possess locus standi; (c) the
invalidate an act of the legislature, but only asserts the question of constitutionality is raised at the earliest opportunity;
solemn and sacred obligation assigned to it by the and (d) the issue of constitutionality is the lis mota of the
Constitution to determine conflicting claims of authority case.106 Of these four, the first two conditions will be the focus
under the Constitution and to establish for the parties in of our discussion.
an actual controversy the rights which that instrument
secures and guarantees to them. This is in truth all that is 1. Petitioners have shown the presence of an actual case
involved in what is termed "judicial supremacy" which or controversy.
properly is the power of judicial review under the
Constitution. x x x x. (Emphases supplied)
The OSG maintains107 that there is no actual case or
controversy that exists, since the Senators have not been
The power of judicial review has since been strengthened in deprived of the opportunity to invoke the privileges of the
the 1987 Constitution. The scope of that power has been institution they are representing. It contends that the
extended to the determination of whether in matters nonparticipation of the Senators in the present petitions only
traditionally considered to be within the sphere of appreciation confirms that even they believe that EDCA is a binding
of another branch of government, an exercise of discretion has executive agreement that does not require their concurrence.
been attended with grave abuse.97 The expansion of this
power has made the political question doctrine "no longer the
FINALS CONSTITUTION I ACJUCO 271

It must be emphasized that the Senate has already expressed or privilege to which they are lawfully entitled, or that they are
its position through SR 105.108 Through the Resolution, the about to be subjected to some burden or penalty by reason of
Senate has taken a position contrary to that of the OSG. As the the act complained of.118 The reason why those who challenge
body tasked to participate in foreign affairs by ratifying treaties, the validity of a law or an international agreement are required
its belief that EDCA infringes upon its constitutional role to allege the existence of a personal stake in the outcome of
indicates that an actual controversy - albeit brought to the the controversy is "to assure the concrete adverseness which
Court by non-Senators, exists. sharpens the presentation of issues upon which the court so
largely depends for illumination of difficult constitutional
Moreover, we cannot consider the sheer abstention of the questions."119
Senators from the present proceedings as basis for finding that
there is no actual case or controversy before us. We point out The present petitions cannot qualify as citizens', taxpayers', or
that the focus of this requirement is the ripeness for legislators' suits; the Senate as a body has the requisite
adjudication of the matter at hand, as opposed to its being standing, but considering that it has not formally filed a
merely conjectural or anticipatory.109 The case must involve a pleading to join the suit, as it merely conveyed to the Supreme
definite and concrete issue involving real parties with Court its sense that EDCA needs the Senate's concurrence to
conflicting legal rights and legal claims admitting of specific be valid, petitioners continue to suffer from lack of standing.
relief through a decree conclusive in nature. 110 It should not
equate with a mere request for an opinion or advice on what In assailing the constitutionality of a governmental act,
the law would be upon an abstract, hypothetical, or contingent
petitioners suing as citizens may dodge the requirement of
state of facts.111 As explained in Angara v. Electoral having to establish a direct and personal interest if they show
Commission:112 that the act affects a public right. 120 In arguing that they have
legal standing, they claim121 that the case they have filed is a
[The] power of judicial review is limited to actual cases and concerned citizen's suit. But aside from general statements
controversies to be exercised after full opportunity of that the petitions involve the protection of a public right, and
argument by the parties, and limited further to the that their constitutional rights as citizens would be violated,
constitutional question raised or the very lis they fail to make any specific assertion of a particular public
mota presented. Any attempt at abstraction could only lead right that would be violated by the enforcement of EDCA. For
to dialectics and barren legal questions and to sterile their failure to do so, the present petitions cannot be
conclusions of wisdom, justice or expediency of considered by the Court as citizens' suits that would
legislation. More than that, courts accord the presumption of justify a disregard of the aforementioned requirements.
constitutionality to legislative enactments, not only because the
legislature is presumed to abide by the Constitution but also
In claiming that they have legal standing as taxpayers,
because the judiciary in the determination of actual cases petitioners122 aver that the implementation of EDCA would
and controversies must reflect the wisdom and justice of
result in the unlawful use of public funds. They emphasize that
the people as expressed through their representatives in Article X(1) refers to an appropriation of funds; and that the
the executive and legislative departments of the agreement entails a waiver of the payment of taxes, fees, and
government. (Emphases supplied) rentals. During the oral arguments, however, they admitted
that the government had not yet appropriated or actually
We find that the matter before us involves an actual case or disbursed public funds for the purpose of implementing the
controversy that is already ripe for adjudication. The Executive agreement.123 The OSG, on the other hand, maintains that
Department has already sent an official confirmation to the petitioners cannot sue as taxpayers. 124Respondent explains
U.S. Embassy that "all internal requirements of the Philippines that EDCA is neither meant to be a tax measure, nor is it
x x x have already been complied with."113 By this exchange of directed at the disbursement of public funds.
diplomatic notes, the Executive Department effectively
performed the last act required under Article XII(l) of EDCA A taxpayer's suit concerns a case in which the official act
before the agreement entered into force. Section 25, Article complained of directly involves the illegal disbursement of
XVIII of the Constitution, is clear that the presence of foreign
public funds derived from taxation.125 Here, those challenging
military forces in the country shall only be allowed by virtue of the act must specifically show that they have sufficient interest
a treaty concurred in by the Senate. Hence, the performance
in preventing the illegal expenditure of public money, and that
of an official act by the Executive Department that led to the they will sustain a direct injury as a result of the enforcement
entry into force of an executive agreement was sufficient to of the assailed act.126 Applying that principle to this case, they
satisfy the actual case or controversy requirement. must establish that EDCA involves the exercise by
Congress of its taxing or spending powers.127
2. While petitioners Saguisag et. al., do not have legal
standing, they nonetheless raise issues involving matters We agree with the OSG that the petitions cannot qualify as
of transcendental importance. taxpayers' suits. We emphasize that a taxpayers' suit
contemplates a situation in which there is already an
The question of locus standi or legal standing focuses on the appropriation or a disbursement of public funds. 128 A reading
determination of whether those assailing the governmental act of Article X(l) of EDCA would show that there has been neither
have the right of appearance to bring the matter to the court for an appropriation nor an authorization of disbursement of funds.
adjudication.114 They must show that they have a personal and The cited provision reads:
substantial interest in the case, such that they have sustained
or are in immediate danger of sustaining, some direct injury as All obligations under this Agreement are subject to the
a consequence of the enforcement of the challenged availability of appropriated funds authorized for these
governmental act.115 Here, "interest" in the question involved purposes. (Emphases supplied)
must be material - an interest that is in issue and will be
affected by the official act - as distinguished from being merely
incidental or general.116 Clearly, it would be insufficient to show This provision means that if the implementation of EDCA would
that the law or any governmental act is invalid, and that require the disbursement of public funds, the money must
petitioners stand to suffer in some indefinite way. 117 They must come from appropriated funds that are
show that they have a particular interest in bringing the suit, specifically authorized for this purpose. Under the agreement,
and that they have been or are about to be denied some right before there can even be a disbursement of public funds, there
FINALS CONSTITUTION I ACJUCO 272

must first be a legislative action. Until and unless the prerogative granted by the Constitution to the Senate, not to
Legislature appropriates funds for EDCA, or unless the entire Legislature. In Pimentel v. Office of the Executive
petitioners can pinpoint a specific item in the current Secretary, this Court did not recognize the standing of one of
budget that allows expenditure under the agreement, we the petitioners therein who was a member of the House of
cannot at this time rule that there is in fact an Representatives. The petition in that case sought to compel the
appropriation or a disbursement of funds that would transmission to the Senate for concurrence of the signed text
justify the filing of a taxpayers' suit. of the Statute of the International Criminal Court. Since that
petition invoked the power of the Senate to grant or withhold
Petitioners Bayan et al. also claim129 that their co-petitioners its concurrence in a treaty entered into by the Executive
who are party-list representatives have the standing to Department, only then incumbent Senator Pimentel was
challenge the act of the Executive Department, especially if it allowed to assert that authority of the Senate of which he was
impairs the constitutional prerogatives, powers, and privileges a member.
of their office. While they admit that there is no incumbent
Senator who has taken part in the present petition, they Therefore, none of the initial petitioners in the present
nonetheless assert that they also stand to sustain a derivative controversy has the standing to maintain the suits as
but substantial injury as legislators. They argue that under the legislators.
Constitution, legislative power is vested in both the Senate and
the House of Representatives; consequently, it is the entire Nevertheless, this Court finds that there is basis for it to review
Legislative Department that has a voice in determining whether
the act of the Executive for the following reasons.
or not the presence of foreign military should be allowed. They
maintain that as members of the Legislature, they have the
requisite personality to bring a suit, especially when a In any case, petitioners raise issues involving matters of
constitutional issue is raised. transcendental importance.

The OSG counters130 that petitioners do not have any legal Petitioners138 argue that the Court may set aside procedural
standing to file the suits concerning the lack of Senate technicalities, as the present petition tackles issues that are of
concurrence in EDCA. Respondent emphasizes that the power transcendental importance. They point out that the matter
to concur in treaties and international agreements is an before us is about the proper exercise of the Executive
"institutional prerogative" granted by the Constitution to the Department's power to enter into international agreements in
Senate. Accordingly, the OSG argues that in case of an relation to that of the Senate to concur in those agreements.
allegation of impairment of that power, the injured party would They also assert that EDCA would cause grave injustice, as
be the Senate as an institution or any of its incumbent well as irreparable violation of the Constitution and of the
members, as it is the Senate's constitutional function that is Filipino people's rights.
allegedly being violated.
The OSG, on the other hand, insists139 that petitioners cannot
The legal standing of an institution of the Legislature or of any raise the mere fact that the present petitions involve matters of
of its Members has already been recognized by this Court in a transcendental importance in order to cure their inability to
number of cases.131 What is in question here is the alleged comply with the constitutional requirement of standing.
impairment of the constitutional duties and powers granted to, Respondent bewails the overuse of "transcendental
or the impermissible intrusion upon the domain of, the importance" as an exception to the traditional requirements of
Legislature or an institution thereof. 132 In the case of suits constitutional litigation. It stresses that one of the purposes of
initiated by the legislators themselves, this Court has these requirements is to protect the Supreme Court from
recognized their standing to question the validity of any official unnecessary litigation of constitutional questions.
action that they claim infringes the prerogatives, powers, and
privileges vested by the Constitution in their office. 133 As aptly In a number of cases,140 this Court has indeed taken a liberal
explained by Justice Perfecto in Mabanag v. Lopez Vito:134 stance towards the requirement of legal standing, especially
when paramount interest is involved. Indeed, when those who
Being members of Congress, they are even duty bound to challenge the official act are able to craft an issue of
see that the latter act within the bounds of the transcendental significance to the people, the Court may
Constitution which, as representatives of the people, they exercise its sound discretion and take cognizance of the suit.
should uphold, unless they are to commit a flagrant betrayal of It may do so in spite of the inability of the petitioners to show
public trust. They are representatives of the sovereign people that they have been personally injured by the operation of a
and it is their sacred duty to see to it that the fundamental law or any other government act.
law embodying the will of the sovereign people is not
trampled upon. (Emphases supplied) While this Court has yet to thoroughly delineate the outer limits
of this doctrine, we emphasize that not every other case,
We emphasize that in a legislators' suit, those Members of however strong public interest may be, can qualify as an issue
Congress who are challenging the official act have standing of transcendental importance. Before it can be impelled to
only to the extent that the alleged violation impinges on their brush aside the essential requisites for exercising its power of
right to participate in the exercise of the powers of the judicial review, it must at the very least consider a number of
institution of which they are members.135 Legislators have the factors: (1) the character of the funds or other assets involved
standing "to maintain inviolate the prerogatives, powers, and in the case; (2) the presence of a clear case of disregard of a
privileges vested by the Constitution in their office and are constitutional or statutory prohibition by the public respondent
allowed to sue to question the validity of any official action, agency or instrumentality of the government; and (3) the lack
which they claim infringes their prerogatives as of any other party that has a more direct and specific interest
legislators."136 As legislators, they must clearly show that there in raising the present questions.141
was a direct injury to their persons or the institution to which
they belong.137 An exhaustive evaluation of the memoranda of the parties,
together with the oral arguments, shows that petitioners have
As correctly argued by respondent, the power to concur in a presented serious constitutional issues that provide ample
treaty or an international agreement is an institutional justification for the Court to set aside the rule on standing. The
FINALS CONSTITUTION I ACJUCO 273

transcendental importance of the issues presented here is 1973 Constitution, for instance, the provision simply gives the
rooted in the Constitution itself. Section 25, Article XVIII President control over the ministries.149 A similar language, not
thereof, cannot be any clearer: there is a much stricter in the form of the President's oath, was present in the 1935
mechanism required before foreign military troops, facilities, or Constitution, particularly in the enumeration of executive
bases may be allowed in the country. The DFA has already functions.150 By 1987, executive power was codified not only in
confirmed to the U.S. Embassy that "all internal requirements the Constitution, but also in the Administrative Code:151
of the Philippines x x x have already been complied with." 142 It
behooves the Court in this instance to take a liberal stance SECTION 1. Power of Control. - The President shall have
towards the rule on standing and to determine forthwith control of all the executive departments, bureaus, and
whether there was grave abuse of discretion on the part of the offices. He shall ensure that the laws be faithfully
Executive Department. executed. (Emphasis supplied)

We therefore rule that this case is a proper Hence, the duty to faithfully execute the laws of the land is
subject for judicial review. inherent in executive power and is intimately related to the
other executive functions. These functions include the faithful
B. Whether the President may enter into an execution of the law in autonomous regions;152 the right to
executive agreement on foreign military bases, prosecute crimes;153 the implementation of transportation
troops, or facilities projects;154 the duty to ensure compliance with treaties,
executive agreements and executive orders;155 the authority to
C. Whether the provisions under EDCA are deport undesirable aliens;156 the conferment of national
awards under the President's jurisdiction;157 and the overall
consistent with the Constitution, as well as with
existing laws and treaties administration and control of the executive department. 158

Issues B and C shall be discussed together infra. These obligations are as broad as they sound, for a President
cannot function with crippled hands, but must be capable of
securing the rule of law within all territories of the Philippine
1. The role of the President as the executor of the law Islands and be empowered to do so within constitutional limits.
includes the duty to defend the State, for which purpose Congress cannot, for instance, limit or take over the
he may use that power in the conduct of foreign relations President's power to adopt implementing rules and regulations
for a law it has enacted.159
Historically, the Philippines has mirrored the division of powers
in the U.S. government. When the Philippine government was More important, this mandate is self-executory by virtue of its
still an agency of the Congress of the U.S., it was as an agent being inherently executive in nature. 160 As Justice Antonio T.
entrusted with powers categorized as executive, legislative, Carpio previously wrote,161
and judicial, and divided among these three great
branches.143 By this division, the law implied that the divided
powers cannot be exercised except by the department given [i]f the rules are issued by the President in implementation or
execution of self-executory constitutional powers vested in the
the power.144
President, the rule-making power of the President is not a
delegated legislative power. The most important self-executory
This divide continued throughout the different versions of the constitutional power of the President is the President's
Philippine Constitution and specifically vested the supreme constitutional duty and mandate to "ensure that the laws be
executive power in the Governor-General of the faithfully executed." The rule is that the President can execute
Philippines,145 a position inherited by the President of the the law without any delegation of power from the legislature.
Philippines when the country attained independence. One of
the principal functions of the supreme executive is the
responsibility for the faithful execution of the laws as embodied The import of this characteristic is that the manner of the
by the oath of office.146 The oath of the President prescribed President's execution of the law, even if not expressly
by the 1987 Constitution reads thus: granted by the law, is justified by necessity and limited
only by law, since the President must "take necessary and
proper steps to carry into execution the law." 162 Justice
I do solemnly swear (or affirm) that I will faithfully and George Malcolm states this principle in a grand manner:163
conscientiously fulfill my duties as President (or Vice-
President or Acting President) of the Philippines, preserve and
defend its Constitution, execute its laws, do justice to every The executive should be clothed with sufficient power to
man, and consecrate myself to the service of the Nation. So administer efficiently the affairs of state. He should have
complete control of the instrumentalities through whom his
help me God. (In case of affirmation, last sentence will be
omitted.)147 (Emphases supplied) responsibility is discharged. It is still true, as said by Hamilton,
that "A feeble executive implies a feeble execution of the
government. A feeble execution is but another phrase for a bad
This Court has interpreted the faithful execution clause as an execution; and a government ill executed, whatever it may be
obligation imposed on the President, and not a separate grant in theory, must be in practice a bad government." The mistakes
of power.148 Section 1 7, Article VII of the Constitution, of State governments need not be repeated here.
expresses this duty in no uncertain terms and includes it in the
provision regarding the President's power of control over the
executive department, viz: xxxx

Every other consideration to one side, this remains certain -


The President shall have control of all the executive
departments, bureaus, and offices. He shall ensure that the The Congress of the United States clearly intended that the
laws be faithfully executed. Governor-General's power should be commensurate with his
responsibility. The Congress never intended that the
Governor-General should be saddled with the responsibility of
The equivalent provisions in the next preceding Constitution administering the government and of executing the laws but
did not explicitly require this oath from the President. In the shorn of the power to do so. The interests of the Philippines
FINALS CONSTITUTION I ACJUCO 274

will be best served by strict adherence to the basic principles This approach of giving utmost deference to presidential
of constitutional government. initiatives in respect of foreign affairs is not novel to the Court.
The President's act of treating EDCA as an executive
agreement is not the principal power being analyzed as the
In light of this constitutional duty, it is the President's
prerogative to do whatever is legal and necessary for Dissenting Opinion seems to suggest. Rather, the preliminary
Philippine defense interests. It is no coincidence that the analysis is in reference to the expansive power of foreign
constitutional provision on the faithful execution clause was affairs. We have long treated this power as something the
followed by that on the President's commander-in-chief Courts must not unduly restrict. As we stated recently in Vinuya
powers,164 which are specifically granted during extraordinary v. Romulo:
events of lawless violence, invasion, or rebellion. And this duty
of defending the country is unceasing, even in times when To be sure, not all cases implicating foreign relations present
there is no state of lawlesss violence, invasion, or rebellion. At political questions, and courts certainly possess the authority
such times, the President has full powers to ensure the faithful to construe or invalidate treaties and executive agreements.
execution of the laws. However, the question whether the Philippine government
should espouse claims of its nationals against a foreign
It would therefore be remiss for the President and repugnant government is a foreign relations matter, the authority for which
is demonstrably committed by our Constitution not to the courts
to the faithful-execution clause of the Constitution to do nothing
when the call of the moment requires increasing the military's but to the political branches. In this case, the Executive
Department has already decided that it is to the best interest
defensive capabilities, which could include forging alliances
with states that hold a common interest with the Philippines or of the country to waive all claims of its nationals for reparations
bringing an international suit against an offending state. against Japan in the Treaty of Peace of 1951. The wisdom of
such decision is not for the courts to question. Neither could
petitioners herein assail the said determination by the
The context drawn in the analysis above has been termed by Executive Department via the instant petition for certiorari.
Justice Arturo D. Brion's Dissenting Opinion as the beginning
of a "patent misconception."165 His dissent argues that this
approach taken in analyzing the President's role as executor In the seminal case of US v. Curtiss-Wright Export Corp., the
of the laws is preceded by the duty to preserve and defend the US Supreme Court held that "[t]he President is the sole organ
Constitution, which was allegedly overlooked.166 of the nation in its external relations, and its sole representative
with foreign relations."

In arguing against the approach, however, the dissent grossly


failed to appreciate the nuances of the analysis, if read It is quite apparent that if, in the
holistically and in context. The concept that the President maintenance of our international relations,
cannot function with crippled hands and therefore can embarrassment - perhaps serious
disregard the need for Senate concurrence in treaties 167 was embarrassment - is to be avoided and
never expressed or implied. Rather, the appropriate reading of success for our aims achieved,
congressional legislation which is to be
the preceding analysis shows that the point being elucidated is
the reality that the President's duty to execute the laws and made effective through negotiation and
inquiry within the international field must
protect the Philippines is inextricably interwoven with his
foreign affairs powers, such that he must resolve issues often accord to the President a degree
imbued with both concerns to the full extent of his powers, of discretion and freedom from
subject only to the limits supplied by law. In other words, apart statutory restriction which would not be
from an expressly mandated limit, or an implied limit by virtue admissible where domestic affairs alone
of incompatibility, the manner of execution by the President involved. Moreover, he, not Congress, has
must be given utmost deference. This approach is not different the better opportunity of knowing the
from that taken by the Court in situations with fairly similar conditions which prevail in foreign
contexts. countries, and especially is this true in time
of war. He has his confidential sources of
information. He has his agents in the form
Thus, the analysis portrayed by the dissent does not give the of diplomatic, consular and other officials
President authority to bypass constitutional safeguards and ....
limits. In fact, it specifies what these limitations are, how these
limitations are triggered, how these limitations function, and
what can be done within the sphere of constitutional duties and This ruling has been incorporated in our jurisprudence
through Bavan v. Executive Secretary and Pimentel v.
limitations of the President.
Executive Secretary; its overreaching principle was, perhaps,
best articulated in (now Chief) Justice Puno's dissent in
Justice Brion's dissent likewise misinterprets the analysis Secretary of Justice v. Lantion:
proffered when it claims that the foreign relations power of the
President should not be interpreted in isolation. 168 The analysis
itself demonstrates how the foreign affairs function, while . . . The conduct of foreign relations is full
mostly the President's, is shared in several instances, namely of complexities and consequences,
in Section 2 of Article II on the conduct of war; Sections 20 and sometimes with life and death significance
21 of Article VII on foreign loans, treaties, and international to the nation especially in times of war. It
agreements; Sections 4(2) and 5(2)(a) of Article VIII on the can only be entrusted to that department of
judicial review of executive acts; Sections 4 and 25 of Article government which can act on the basis of
XVIII on treaties and international agreements entered into the best available information and can
prior to the Constitution and on the presence of foreign military decide with decisiveness .... It is also the
President who possesses the most
troops, bases, or facilities.
comprehensive and the most confidential
information about foreign countries for our
In fact, the analysis devotes a whole subheading to the diplomatic and consular officials regularly
relationship between the two major presidential functions and brief him on meaningful events all over the
the role of the Senate in it. world. He has also unlimited access to
ultra-sensitive military intelligence data. In
FINALS CONSTITUTION I ACJUCO 275

fine, the presidential role in foreign referendum held for that purpose, and recognized as a treaty
affairs is dominant and the President is by the other contracting State. (Emphases supplied)
traditionally accorded a wider degree of
discretion in the conduct of foreign
In view of this provision, petitioners argue170 that EDCA must
affairs. The regularity, nay, validity of be in the form of a "treaty" duly concurred in by the Senate.
his actions are adjudged under less They stress that the Constitution is unambigous in mandating
stringent standards, lest their judicial the transmission to the Senate of all international agreements
repudiation lead to breach of an concluded after the expiration of the MBA in 1991 - agreements
international obligation, rupture of state that concern the presence of foreign military bases, troops, or
relations, forfeiture of confidence, facilities in the country. Accordingly, petitioners maintain that
national embarrassment and a plethora the Executive Department is not given the choice to conclude
of other problems with equally agreements like EDCA in the form of an executive agreement.
undesirable
consequences.169 (Emphases supplied)
This is also the view of the Senate, which, through a majority
vote of 15 of its members - with 1 against and 2 abstaining -
Understandably, this Court must view the instant case with the says in SR 105171 that EDCA must be submitted to the Senate
same perspective and understanding, knowing full well the
in the form of a treaty for concurrence by at least two-thirds of
constitutional and legal repercussions of any judicial all its members.
overreach.

The Senate cites two constitutional provisions (Article VI,


2. The plain meaning of the Constitution prohibits the
Section 21 and Article XVIII, Section 25) to support its position.
entry of foreign military bases, troops or facilities, except Compared with the lone constitutional provision that the Office
by way of a treaty concurred in by the Senate - a clear
of the Solicitor General (OSG) cites, which is Article XVIII,
limitation on the President's dual role as defender of the Section 4(2), which includes the constitutionality of "executive
State and as sole authority in foreign relations. agreement(s)" among the cases subject to the Supreme
Court's power of judicial review, the Constitution clearly
Despite the President's roles as defender of the State and sole requires submission of EDCA to the Senate. Two specific
authority in foreign relations, the 1987 Constitution expressly provisions versus one general provision means that the
limits his ability in instances when it involves the entry of specific provisions prevail. The term "executive agreement" is
foreign military bases, troops or facilities. The initial limitation "a term wandering alone in the Constitution, bereft of
is found in Section 21 of the provisions on the Executive provenance and an unidentified constitutional mystery."
Department: "No treaty or international agreement shall be
valid and effective unless concurred in by at least two-thirds of The author of SR 105, Senator Miriam Defensor Santiago,
all the Members of the Senate." The specific limitation is given upon interpellation even added that the MDT, which the
by Section 25 of the Transitory Provisions, the full text of which Executive claims to be partly implemented through EDCA, is
reads as follows:
already obsolete.

SECTION 25. After the expiration in 1991 of the Agreement There are two insurmountable obstacles to this Court's
between the Republic of the Philippines and the United States agreement with SR 105, as well as with the comment on
of America concerning Military Bases, foreign military bases,
interpellation made by Senator Santiago.
troops, or facilities shall not be allowed in the Philippines
except under a treaty duly concurred in by the Senate and,
when the Congress so requires, ratified by a majority of the First, the concept of "executive agreement" is so well-
votes cast by the people in a national referendum held for that entrenched in this Court's pronouncements on the powers of
purpose, and recognized as a treaty by the other contracting the President. When the Court validated the concept of
State. "executive agreement," it did so with full knowledge of the
Senate's role in concurring in treaties. It was aware of the
problematique of distinguishing when an international
It is quite plain that the Transitory Provisions of the 1987
agreement needed Senate concurrence for validity, and when
Constitution intended to add to the basic requirements of a it did not; and the Court continued to validate the existence of
treaty under Section 21 of Article VII. This means that both "executive agreements" even after the 1987
provisions must be read as additional limitations to the Constitution.172 This follows a long line of similar decisions
President's overarching executive function in matters of upholding the power of the President to enter into an executive
defense and foreign relations. agreement.173

3. The President, however, may enter into an executive Second, the MDT has not been rendered obsolescent,
agreement on foreign military bases, troops, or facilities, considering that as late as 2009,174 this Court continued to
if (a) it is not the instrument that allows the presence of recognize its validity.
foreign military bases, troops, or facilities; or (b) it merely
aims to implement an existing law or treaty.
Third, to this Court, a plain textual reading of Article XIII,
Section 25, inevitably leads to the conclusion that it applies
Again we refer to Section 25, Article XVIII of the Constitution: only to a proposed agreement between our government and a
foreign government, whereby military bases, troops, or
SECTION 25. After the expiration in 1991 of the Agreement facilities of such foreign government would be "allowed" or
between the Republic of the Philippines and the United States would "gain entry" Philippine territory.
of America concerning Military Bases, foreign military bases,
troops, or facilities shall not be allowed in the Note that the provision "shall not be allowed" is a negative
Philippines except under a treaty duly concurred in by the injunction. This wording signifies that the President is not
Senate and, when the Congress so requires, ratified by a authorized by law to allow foreign military bases, troops, or
majority of the votes cast by the people in a national
facilities to enter the Philippines, except under a treaty
concurred in by the Senate. Hence, the constitutionally
FINALS CONSTITUTION I ACJUCO 276

restricted authority pertains to the entry of the bases, troops, The VFA has already allowed the entry of troops in the
or facilities, and not to the activities to be done after entry. Philippines. This Court stated in Lim v. Executive Secretary:

Under the principles of constitutional construction, of After studied reflection, it appeared farfetched that the
paramount consideration is the plain meaning of the language ambiguity surrounding the meaning of the word "activities"
expressed in the Constitution, or the verba legis rule.175 It is arose from accident. In our view, it was deliberately made that
presumed that the provisions have been carefully crafted in way to give both parties a certain leeway in negotiation. In this
order to express the objective it seeks to attain. 176 It is manner, visiting US forces may sojourn in Philippine
incumbent upon the Court to refrain from going beyond the territory for purposes other than military. As conceived, the
plain meaning of the words used in the Constitution. It is joint exercises may include training on new techniques of patrol
presumed that the framers and the people meant what they and surveillance to protect the nation's marine resources, sea
said when they said it, and that this understanding was search-and-rescue operations to assist vessels in distress,
reflected in the Constitution and understood by the people in disaster relief operations, civic action projects such as the
the way it was meant to be understood when the fundamental building of school houses, medical and humanitarian missions,
law was ordained and promulgated.177 As this Court has often and the like.
said:
Under these auspices, the VFA gives legitimacy to the current
We look to the language of the document itself in our search Balikatan exercises. It is only logical to assume that "Balikatan
for its meaning. We do not of course stop there, but that is 02-1," a "mutual anti- terrorism advising, assisting and training
where we begin. It is to be assumed that the words in which exercise," falls under the umbrella of sanctioned or allowable
constitutional provisions are couched express the objective activities in the context of the agreement. Both the history and
sought to be attained. They are to be given their ordinary intent of the Mutual Defense Treaty and the VFA support the
meaning except where technical terms are employed in conclusion that combat-related activities -as opposed to
which case the significance thus attached to them prevails. combat itself-such as the one subject of the instant petition, are
As the Constitution is not primarily a lawyer's document, indeed authorized.184 (Emphasis supplied)
it being essential for the rule of law to obtain that it should ever
be present in the people's consciousness, its language as Moreover, the Court indicated that the Constitution continues
much as possible should be understood in the sense they to govern the conduct of foreign military troops in the
have in common use. What it says according to the text of the Philippines,185 readily implying the legality of their initial entry
provision to be construed compels acceptance and negates into the country.
the power of the courts to alter it, based on the postulate that
the framers and the people mean what they say. Thus,
these are the cases where the need for construction is The OSG emphasizes that EDCA can be in the form of an
reduced to a minimum.178(Emphases supplied) executive agreement, since it merely involves "adjustments in
detail" in the implementation of the MDT and the VFA. 186 It
points out that there are existing treaties between the
It is only in those instances in which the constitutional provision
Philippines and the U.S. that have already been concurred in
is unclear, ambiguous, or silent that further construction must by the Philippine Senate and have thereby met the
be done to elicit its meaning.179 In Ang Bagong Bayani-OFW v.
requirements of the Constitution under Section 25. Because of
Commission on Elections,180 we reiterated this guiding the status of these prior agreements, respondent emphasizes
principle: that EDCA need not be transmitted to the Senate.

it [is] safer to construe the Constitution from what appears


The aforecited Dissenting Opinion of Justice Brion disagrees
upon its face. The proper interpretation therefore depends with the ponencia's application of verba legis construction to
more on how it was understood by the people adopting it the words of Article XVIII, Section 25. 187 It claims that the
than in the framers' understanding thereof. (Emphases provision is "neither plain, nor that simple." 188 To buttress its
supplied) disagreement, the dissent states that the provision refers to a
historical incident, which is the expiration of the 1947
The effect of this statement is surprisingly profound, for, if MBA.189 Accordingly, this position requires questioning the
taken literally, the phrase "shall not be allowed in the circumstances that led to the historical event, and the meaning
Philippines" plainly refers to the entry of bases, troops, or of the terms under Article XVIII, Section 25.
facilities in the country. The Oxford English Dictionary defines
the word "allow" as a transitive verb that means "to permit, This objection is quite strange. The construction technique
enable"; "to give consent to the occurrence of or relax restraint of verba legis is not inapplicable just because a provision has
on (an action, event, or activity)"; "to consent to the presence a specific historical context. In fact, every provision of the
or attendance of (a person)"; and, when with an adverbial of
Constitution has a specific historical context. The purpose of
place, "to permit (a person or animal) to go, come, or be in, out, constitutional and statutory construction is to set tiers of
near, etc."181 Black's Law Dictionary defines the term as one
interpretation to guide the Court as to how a particular
that means "[t]o grant, approve, or permit."182 provision functions. Verba legis is of paramount consideration,
but it is not the only consideration. As this Court has often said:
The verb "allow" is followed by the word "in," which is a
preposition used to indicate "place or position in space or
We look to the language of the document itself in our search
anything having material extension: Within the limits or bounds for its meaning. We do not of course stop there, but that is
of, within (any place or thing)."183 That something is the where we begin. It is to be assumed that the words in which
Philippines, which is the noun that follows. constitutional provisions are couched express the objective
sought to be attained. They are to be given their ordinary
It is evident that the constitutional restriction refers solely to the meaning except where technical terms are employed in
initial entry of the foreign military bases, troops, or facilities. which case the significance thus attached to them prevails. As
Once entry is authorized, the subsequent acts are thereafter the Constitution is not primarily a lawyer's document, it being
subject only to the limitations provided by the rest of the essential for the rule of law to obtain that it should ever be
Constitution and Philippine law, and not to the Section 25 present in the people's consciousness, its language as much
requirement of validity through a treaty. as possible should be understood in the sense they have
FINALS CONSTITUTION I ACJUCO 277

in common use. What it says according to the text of the As previously discussed, this constitutional mandate emanates
provision to be construed compels acceptance and negates from the inherent power of the President to enter into
the power of the courts to alter it, based on the postulate that agreements with other states, including the prerogative to
the framers and the people mean what they say. Thus, conclude binding executive agreements that do not require
these are the cases where the need for construction is further Senate concurrence. The existence of this presidential
reduced to a minimum.190(Emphases supplied) power203 is so well-entrenched that Section 5(2)(a), Article VIII
of the Constitution, even provides for a check on its exercise.
As applied, verba legis aids in construing the ordinary meaning As expressed below, executive agreements are among those
of terms. In this case, the phrase being construed is "shall not official governmental acts that can be the subject of this Court's
be allowed in the Philippines" and not the preceding one power of judicial review:
referring to "the expiration in 1991 of the Agreement between
the Republic of the Philippines and the United States of (2) Review, revise, reverse, modify, or affirm on
America concerning Military Bases, foreign military bases, appeal or certiorari, as the law or the Rules of Court
troops, or facilities." It is explicit in the wording of the provision may provide, final judgments and orders of lower
itself that any interpretation goes beyond the text itself and into courts in:
the discussion of the framers, the context of the Constitutional
Commission's time of drafting, and the history of the 1947
(a) All cases in which
MBA. Without reference to these factors, a reader would not the constitutionality or
understand those terms. However, for the phrase "shall not be
validity of any treaty, international or ex
allowed in the Philippines," there is no need for such reference. ecutive agreement, law, presidential
The law is clear. No less than the Senate understood this when decree, proclamation, order, instruction,
it ratified the VFA. ordinance, or regulation is in question.
(Emphases supplied)
4. The President may generally enter into executive
agreements subject to limitations defined by the In Commissioner of Customs v. Eastern Sea
Constitution and may be in furtherance of a treaty already Trading, executive agreements are defined as "international
concurred in by the Senate. agreements embodying adjustments of detail carrying out well-
established national policies and traditions and those involving
We discuss in this section why the President can enter into arrangements of a more or less temporary nature."204 In Bayan
executive agreements. Muna v. Romulo, this Court further clarified that executive
agreements can cover a wide array of subjects that have
It would be helpful to put into context the contested language various scopes and purposes.205 They are no longer limited to
found in Article XVIII, Section 25. Its more exacting the traditional subjects that are usually covered by executive
requirement was introduced because of the previous agreements as identified in Eastern Sea Trading. The Court
experience of the country when its representatives felt thoroughly discussed this matter in the following manner:
compelled to consent to the old MBA.191 They felt constrained
to agree to the MBA in fulfilment of one of the major conditions The categorization of subject matters that may be covered
for the country to gain independence from the U.S. 192 As a by international agreementsmentioned in Eastern Sea
result of that experience, a second layer of consent for Trading is not cast in stone. x x x.
agreements that allow military bases, troops and facilities in
the country is now articulated in Article XVIII of our present As may be noted, almost half a century has elapsed
Constitution.
since the Court rendered its decision in Eastern Sea
Trading. Since then, the conduct of foreign affairs has
This second layer of consent, however, cannot be interpreted become more complex and the domain of international law
in such a way that we completely ignore the intent of our wider, as to include such subjects as human rights, the
constitutional framers when they provided for that additional environment, and the sea. In fact, in the US alone, the
layer, nor the vigorous statements of this Court that affirm the executive agreements executed by its President from 1980 to
continued existence of that class of international agreements 2000 covered subjects such as defense, trade, scientific
called "executive agreements." cooperation, aviation, atomic energy, environmental
cooperation, peace corps, arms limitation, and nuclear
The power of the President to enter into binding executive safety, among others. Surely, the enumeration in Eastern
agreements without Senate concurrence is already well- Sea Trading cannot circumscribe the option of each
established in this jurisdiction.193 That power has been alluded state on the matter of which the international agreement
to in our present and past Constitutions, 194 in various format would be convenient to serve its best interest. As
Francis Sayre said in his work referred to earlier:
statutes,195 in Supreme Court decisions,196 and during the
deliberations of the Constitutional Commission.197 They cover
a wide array of subjects with varying scopes and . . . It would be useless to undertake to discuss here the
purposes,198 including those that involve the presence of large variety of executive agreements as such concluded
foreign military forces in the country.199 from time to time. Hundreds of executive agreements, other
than those entered into under the trade-agreement act, have
been negotiated with foreign governments. . . . They cover
As the sole organ of our foreign relations200 and the
constitutionally assigned chief architect of our foreign such subjects as the inspection of vessels, navigation dues,
policy,201the President is vested with the exclusive power to income tax on shipping profits, the admission of civil air craft,
conduct and manage the country's interface with other states custom matters and commercial relations generally,
and governments. Being the principal representative of the international claims, postal matters, the registration of
Philippines, the Chief Executive speaks and listens for the trademarks and copyrights, etc .... (Emphases Supplied)
nation; initiates, maintains, and develops diplomatic relations
with other states and governments; negotiates and enters into One of the distinguishing features of executive agreements is
international agreements; promotes trade, investments, that their validity and effectivity are not affected by a lack of
tourism and other economic relations; and settles international Senate concurrence.206 This distinctive feature was
disputes with other states.202 recognized as early as in Eastern Sea Trading (1961), viz:
FINALS CONSTITUTION I ACJUCO 278

Treaties are formal documents which require ratification MR. CONCEPCION: The Commissioner is free to require
with the approval of two-thirds of the Senate. Executive ratification for validity insofar as the Philippines is concerned.
agreements become binding through executive action
without the need of a vote by the Senate or by Congress.
MS. AQUINO: It is my humble submission that we
should provide, unless the Committee explains to us
xxxx otherwise, an explicit proviso which would except executive
agreements from the requirement of concurrence of two-
thirds of the Members of the Senate. Unless I am
[T]he right of the Executive to enter into binding
agreements without the necessity of subsequent enlightened by the Committee I propose that tentatively, the
Congressional approval has been confirmed by long sentence should read. "No treaty or international agreement
usage. From the earliest days of our history we have entered EXCEPT EXECUTIVE AGREEMENTS shall be valid and
into executive agreements covering such subjects as effective."
commercial and consular relations, most-favored-nation rights,
patent rights, trademark and copyright protection, postal and FR. BERNAS: I wonder if a quotation from the Supreme
navigation arrangements and the settlement of Court decision [in Eastern Sea Trading] might help clarify
claims. The validity of these has never been seriously this:
questioned by our courts. (Emphases Supplied)
The right of the executive to enter into binding
That notion was carried over to the present Constitution. In agreements without the necessity of subsequent
fact, the framers specifically deliberated on whether the Congressional approval has been confirmed by long
general term "international agreement" included executive usage. From the earliest days of our history, we have entered
agreements, and whether it was necessary to include an into executive agreements covering such subjects as
express proviso that would exclude executive agreements commercial and consular relations, most favored nation rights,
from the requirement of Senate concurrence. After noted patent rights, trademark and copyright protection, postal and
constitutionalist Fr. Joaquin Bernas quoted the Court's ruling navigation arrangements and the settlement of claims. The
in Eastern Sea Trading, the Constitutional Commission validity of this has never been seriously questioned by our
members ultimately decided that the term "international Courts.
agreements" as contemplated in Section 21, Article VII, does
not include executive agreements, and that a proviso is no
Agreements with respect to the registration of trademarks have
longer needed. Their discussion is reproduced below: 207 been concluded by the executive of various countries under
the Act of Congress of March 3, 1881 (21 Stat. 502) . .
MS. AQUINO: Madam President, first I would like a clarification . International agreements involving political issues or
from the Committee. We have retained the words "international changes of national policy and those involving international
agreement" which I think is the correct judgment on the matter agreements of a permanent character usually take the form
because an international agreement is different from a treaty. of treaties. But international agreements
A treaty is a contract between parties which is in the nature of embodying adjustments of detail, carrying out well
international agreement and also a municipal law in the sense established national policies and traditions and those
that the people are bound. So there is a conceptual difference. involving arrangements of a more or less temporary
However, I would like to be clarified if the international nature usually take the form of executive agreements.
agreements include executive agreements.
MR. ROMULO: Is the Commissioner, therefore, excluding the
MR. CONCEPCION: That depends upon the parties. All executive agreements?
parties to these international negotiations stipulate the
conditions which are necessary for the agreement or whatever
FR. BERNAS: What we are referring to, therefore, when we
it may be to become valid or effective as regards the parties. say international agreements which need concurrence by
at least two-thirds are those which are permanent in nature.
MS. AQUINO: Would that depend on the parties or would that
depend on the nature of the executive agreement? According MS. AQUINO: And it may include commercial agreements
to common usage, there are two types of executive which are executive agreements essentially but which are
agreement: one is purely proceeding from an executive
proceeding from the authorization of Congress. If that is our
act which affects external relations independent of the understanding, then I am willing to withdraw that amendment.
legislative and the other is an executive act in pursuance
of legislative authorization. The first kind might take the
form of just conventions or exchanges of notes or FR. BERNAS: If it is with prior authorization of Congress,
protocol while the other, which would be pursuant to the then it does not need subsequent concurrence by
legislative authorization, may be in the nature of Congress.
commercial agreements.
MS. AQUINO: In that case, I am withdrawing my amendment.
MR. CONCEPCION: Executive agreements are generally
made to implement a treaty already enforced or to MR. TINGSON: Madam President.
determine the details for the implementation of the treaty.
We are speaking of executive agreements, not international
agreements. THE PRESIDENT: Is Commissioner Aquino satisfied?

MS. AQUINO: I am in full agreement with that, except that it MS. AQUINO: Yes. There is already an agreement among
does not cover the first kind of executive agreement which is us on the definition of "executive agreements" and that
just protocol or an exchange of notes and this would be in the would make unnecessary any explicit proviso on the
nature of reinforcement of claims of a citizen against a country, matter.
for example.
xxx
FINALS CONSTITUTION I ACJUCO 279

MR. GUINGONA: I am not clear as to the meaning of The special nature of an executive agreement is not just a
"executive agreements" because I heard that these executive domestic variation in international agreements. International
agreements must rely on treaties. In other words, there must practice has accepted the use of various forms and
first be treaties. designations of international agreements, ranging from the
traditional notion of a treaty - which connotes a formal, solemn
MR. CONCEPCION: No, I was speaking about the common instrument - to engagements concluded in modem, simplified
use, as executive agreements being the implementation of forms that no longer necessitate ratification. 212 An international
treaties, details of which do not affect the sovereignty of the agreement may take different forms: treaty, act, protocol,
State. agreement, concordat, compromis d'arbitrage, convention,
covenant, declaration, exchange of notes, statute, pact,
charter, agreed minute, memorandum of agreement, modus
MR. GUINGONA: But what about the matter of permanence, vivendi, or some other form.213 Consequently, under
Madam President? Would 99 years be considered permanent? international law, the distinction between a treaty and an
What would be the measure of permanency? I do not conceive international agreement or even an executive agreement is
of a treaty that is going to be forever, so there must be some irrelevant for purposes of determining international rights and
kind of a time limit. obligations.

MR. CONCEPCION: I suppose the Commissioner's question However, this principle does not mean that the domestic law
is whether this type of agreement should be included in a distinguishing treaties, international
provision of the Constitution requiring the concurrence of agreements, and executive agreements is relegated to a mere
Congress. variation in form, or that the constitutional requirement of
Senate concurrence is demoted to an optional constitutional
MR. GUINGONA: It depends on the concept of the executive directive. There remain two very important features that
agreement of which I am not clear. If the executive distinguish treaties from executive agreements and translate
agreement partakes of the nature of a treaty, then it should them into terms of art in the domestic setting.
also be included.
First, executive agreements must remain traceable to an
MR. CONCEPCION: Whether it partakes or not of the nature express or implied authorization under the Constitution,
of a treaty, it is within the power of the Constitutional statutes, or treaties. The absence of these precedents puts the
Commission to require that. validity and effectivity of executive agreements under serious
question for the main function of the Executive is to enforce the
Constitution and the laws enacted by the Legislature, not to
MR. GUINGONA: Yes. That is why I am trying to clarify defeat or interfere in the performance of these rules.214 In turn,
whether the words "international agreements" would executive agreements cannot create new international
include executive agreements. obligations that are not expressly allowed or reasonably
implied in the law they purport to implement.
MR. CONCEPCION: No, not necessarily; generally no.
Second, treaties are, by their very nature, considered superior
xxx to executive agreements. Treaties are products of the acts of
the Executive and the Senate215 unlike executive agreements,
which are solely executive actions. 216Because of legislative
MR. ROMULO: I wish to be recognized first. I have only one participation through the Senate, a treaty is regarded as being
question. Do we take it, therefore, that as far as the on the same level as a statute.217 If there is an irreconcilable
Committee is concerned, the term "international conflict, a later law or treaty takes precedence over one that is
agreements" does not include the term "executive prior.218 An executive agreement is treated differently.
agreements" as read by the Commissioner in that text? Executive agreements that are inconsistent with either a law or
a treaty are considered ineffective.219 Both types of
FR. BERNAS: Yes. (Emphases Supplied) international agreement are nevertheless subject to the
supremacy of the Constitution.220
The inapplicability to executive agreements of the
requirements under Section 21 was again recognized in Bayan This rule does not imply, though, that the President is
v. Zamora and in Bayan Muna v. Romulo. These cases, both given carte blanche to exercise this discretion. Although the
decided under the aegis of the present Constitution, Chief Executive wields the exclusive authority to conduct our
quoted Eastern Sea Trading in reiterating that executive foreign relations, this power must still be exercised within the
agreements are valid and binding even without the context and the parameters set by the Constitution, as well as
concurrence of the Senate. by existing domestic and international laws. There are
constitutional provisions that restrict or limit the President's
prerogative in concluding international agreements, such as
Executive agreements may dispense with the requirement of
those that involve the following:
Senate concurrence because of the legal mandate with which
they are concluded. As culled from the afore-quoted
deliberations of the Constitutional Commission, past Supreme a. The policy of freedom from nuclear weapons within
Court Decisions, and works of noted scholars, 208 executive Philippine territory221
agreements merely involve arrangements on the
implementation of existing policies, rules, laws, or agreements. b. The fixing of tariff rates, import and export quotas,
They are concluded (1) to adjust the details of a treaty; 209 (2) tonnage and wharfage dues, and other duties or
pursuant to or upon confirmation by an act of the imposts, which must be pursuant to the authority
Legislature;210 or (3) in the exercise of the President's granted by Congress222
independent powers under the Constitution. 211 The raison
d'etre of executive agreements hinges on prior constitutional
or legislative authorizations. c. The grant of any tax exemption, which must be
pursuant to a law concurred in by a majority of all the
Members of Congress223
FINALS CONSTITUTION I ACJUCO 280

d. The contracting or guaranteeing, on behalf of the 3. However, an agreement - the subject of which is
Philippines, of foreign loans that must be previously the entry of foreign military troops, bases, or facilities
concurred in by the Monetary Board224 - is particularly restricted. The requirements are that
it be in the form of a treaty concurred in by the
e. The authorization of the presence of foreign Senate; that when Congress so requires, it be ratified
military bases, troops, or facilities in the country must by a majority of the votes cast by the people in a
be in the form of a treaty duly concurred in by the national referendum held for that purpose; and that it
Senate.225 be recognized as a treaty by the other contracting
State.

f. For agreements that do not fall under paragraph 5,


the concurrence of the Senate is required, should the 4. Thus, executive agreements can continue to exist
form of the government chosen be a treaty. as a species of international agreements.

That is why our Court has ruled the way it has in several cases.
5. The President had the choice to enter into EDCA by way
of an executive agreement or a treaty.
In Bayan Muna v. Romulo, we ruled that the President acted
No court can tell the President to desist from choosing an within the scope of her constitutional authority and discretion
when she chose to enter into the RP-U.S. Non-Surrender
executive agreement over a treaty to embody an international
agreement, unless the case falls squarely within Article VIII, Agreement in the form of an executive agreement, instead of a
Section 25. treaty, and in ratifying the agreement without Senate
concurrence. The Court en banc discussed this intrinsic
presidential prerogative as follows:
As can be gleaned from the debates among the members of
the Constitutional Commission, they were aware that legally
binding international agreements were being entered into by Petitioner parlays the notion that the Agreement is of dubious
countries in forms other than a treaty. At the same time, it is validity, partaking as it does of the nature of a treaty; hence, it
clear that they were also keen to preserve the concept of must be duly concurred in by the Senate. x x x x. Pressing its
"executive agreements" and the right of the President to enter point, petitioner submits that the subject of the Agreement does
into such agreements. not fall under any of the subject-categories that xx x may be
covered by an executive agreement, such as
commercial/consular relations, most-favored nation rights,
What we can glean from the discussions of the Constitutional patent rights, trademark and copyright protection, postal and
Commissioners is that they understood the following realities: navigation arrangements and settlement of claims.

1. Treaties, international agreements, and executive The categorization of subject matters that may be covered by
agreements are all constitutional manifestations of international agreements mentioned in Eastern Sea Trading is
the conduct of foreign affairs with their distinct legal not cast in stone. There are no hard and fast rules on the
characteristics. propriety of entering, on a given subject, into a treaty or an
executive agreement as an instrument of international
a. Treaties are formal contracts between relations. The primary consideration in the choice of the
the Philippines and other States-parties, form of agreement is the parties' intent and desire to craft
which are in the nature of international an international agreement in the form they so wish to
agreements, and also of municipal laws in further their respective interests. Verily, the matter of form
the sense of their binding nature.226 takes a back seat when it comes to effectiveness and binding
effect of the enforcement of a treaty or an executive
agreement, as the parties in either international agreement
b. International agreements are similar each labor under the pacta sunt servanda principle.
instruments, the provisions of which may
require the ratification of a designated
number of parties thereto. These xxxx
agreements involving political issues or
changes in national policy, as well as those But over and above the foregoing considerations is the fact
involving international agreements of a that - save for the situation and matters contemplated in Sec.
permanent character, usually take the form 25, Art. XVIII of the Constitution - when a treaty is required, the
of treaties. They may also include Constitution does not classify any subject, like that
commercial agreements, which are involving political issues, to be in the form of, and ratified
executive agreements essentially, but as, a treaty. What the Constitution merely prescribes is that
which proceed from previous authorization treaties need the concurrence of the Senate by a vote defined
by Congress, thus dispensing with the therein to complete the ratification process.
requirement of concurrence by the
Senate.227
xxxx

c. Executive agreements are generally


intended to implement a treaty already x x x. As the President wields vast powers and influence, her
conduct in the external affairs of the nation is, as Bayan would
enforced or to determine the details of the
implementation thereof that do not affect put it, "executive altogether." The right of the President to
enter into or ratify binding executive agreements has been
the sovereignty of the State.228
confirmed by long practice.

2. Treaties and international agreements that cannot


be mere executive agreements must, by In thus agreeing to conclude the Agreement thru E/N BF0-
028-03, then President Gloria Macapagal-Arroyo, represented
constitutional decree, be concurred in by at least two-
thirds of the Senate. by the Secretary of Foreign Affairs, acted within the scope of
the authority and discretion vested in her by the
FINALS CONSTITUTION I ACJUCO 281

Constitution. At the end of the day, the President - by Culling from the foregoing discussions, we reiterate the
ratifying, thru her deputies, the non-surrender agreement following pronouncements to guide us in resolving the present
- did nothing more than discharge a constitutional duty controversy:
and exercise a prerogative that pertains to her
office. (Emphases supplied) 1. Section 25, Article XVIII of the Constitution,
contains stringent requirements that must be fulfilled
Indeed, in the field of external affairs, the President must be by the international agreement allowing the presence
given a larger measure of authority and wider discretion, of foreign military bases, troops, or facilities in the
subject only to the least amount of checks and restrictions Philippines: (a) the agreement must be in the form of
under the Constitution.229 The rationale behind this power and a treaty, and (b) it must be duly concurred in by the
discretion was recognized by the Court in Vinuya v. Executive Senate.
Secretary, cited earlier.230
2. If the agreement is not covered by the above
Section 9 of Executive Order No. 459, or the Guidelines in the situation, then the President may choose the form of
Negotiation of International Agreements and its Ratification, the agreement (i.e., either an executive agreement
thus, correctly reflected the inherent powers of the President or a treaty), provided that the agreement dealing with
when it stated that the DFA "shall determine whether an foreign military bases, troops, or facilities is not the
agreement is an executive agreement or a treaty." principal agreement that first allows their entry or
presence in the Philippines.
Accordingly, in the exercise of its power of judicial review, the
Court does not look into whether an international agreement 3. The executive agreement must not go beyond the
should be in the form of a treaty or an executive agreement, parameters, limitations, and standards set by the law
save in cases in which the Constitution or a statute requires and/or treaty that the former purports to implement;
otherwise. Rather, in view of the vast constitutional powers and and must not unduly expand the international
prerogatives granted to the President in the field of foreign obligation expressly mentioned or necessarily
affairs, the task of the Court is to determine whether the implied in the law or treaty.
international agreement is consistent with the applicable
limitations. 4. The executive agreement must be consistent with
the Constitution, as well as with existing laws and
6. Executive agreements may cover the matter of foreign treaties.
military forces if it merely involves detail adjustments.
In light of the President's choice to enter into EDCA in the form
The practice of resorting to executive agreements in adjusting of an executive agreement, respondents carry the burden of
the details of a law or a treaty that already deals with the proving that it is a mere implementation of existing laws and
presence of foreign military forces is not at all unusual in this treaties concurred in by the Senate. EDCA must thus be
jurisdiction. In fact, the Court has already implicitly carefully dissected to ascertain if it remains within the legal
acknowledged this practice in Lim v. Executive Secretary.231 In parameters of a valid executive agreement.
that case, the Court was asked to scrutinize the
constitutionality of the Terms of Reference of the Balikatan 02- 7. EDCA is consistent with the content, purpose, and
1 joint military exercises, which sought to implement the VFA. framework of the MDT and the VFA
Concluded in the form of an executive agreement, the Terms
of Reference detailed the coverage of the term "activities"
mentioned in the treaty and settled the matters pertaining to The starting point of our analysis is the rule that "an executive
the construction of temporary structures for the U.S. troops agreement xx x may not be used to amend a treaty."234 In Lim
during the activities; the duration and location of the exercises; v. Executive Secretary and in Nicolas v. Romulo, the Court
the number of participants; and the extent of and limitations on approached the question of the validity of executive
the activities of the U.S. forces. The Court upheld the Terms of agreements by comparing them with the general framework
Reference as being consistent with the VFA. It no longer took and the specific provisions of the treaties they seek to
issue with the fact that the Balikatan Terms of Reference was implement.
not in the form of a treaty concurred in by the Senate, even if it
dealt with the regulation of the activities of foreign military In Lim, the Terms of Reference of the joint military exercises
forces on Philippine territory. was scrutinized by studying "the framework of the treaty
antecedents to which the Philippines bound itself,"235 i.e., the
In Nicolas v. Romulo,232 the Court again impliedly affirmed the MDT and the VFA. The Court proceeded to examine the extent
use of an executive agreement in an attempt to adjust the of the term "activities" as contemplated in Articles 1 236 and
details of a provision of the VFA. The Philippines and the U.S. II237 of the VFA. It later on found that the term "activities" was
entered into the Romulo-Kenney Agreement, which undertook deliberately left undefined and ambiguous in order to permit "a
to clarify the detention of a U.S. Armed Forces member, whose wide scope of undertakings subject only to the approval of the
case was pending appeal after his conviction by a trial court for Philippine government"238 and thereby allow the parties "a
the crime of rape. In testing the validity of the latter agreement, certain leeway in negotiation."239 The Court eventually ruled
the Court precisely alluded to one of the inherent limitations of that the Terms of Reference fell within the sanctioned or
an executive agreement: it cannot go beyond the terms of the allowable activities, especially in the context of the VFA and
treaty it purports to implement. It was eventually ruled that the the MDT.
Romulo-Kenney Agreement was "not in accord" with the VFA,
since the former was squarely inconsistent with a provision in The Court applied the same approach to Nicolas v. Romulo. It
the treaty requiring that the detention be "by Philippine studied the provisions of the VFA on custody and detention to
authorities." Consequently, the Court ordered the Secretary of ascertain the validity of the Romulo-Kenney Agreement.240 It
Foreign Affairs to comply with the VFA and "forthwith negotiate eventually found that the two international agreements were
with the United States representatives for the appropriate not in accord, since the Romulo-Kenney Agreement had
agreement on detention facilities under Philippine authorities stipulated that U.S. military personnel shall be detained at the
as provided in Art. V, Sec. 10 of the VFA. "233 U.S. Embassy Compound and guarded by U.S. military
FINALS CONSTITUTION I ACJUCO 282

personnel, instead of by Philippine authorities. According to the Moreover, both the VFA and EDCA are silent on what these
Court, the parties "recognized the difference between custody activities actually are. Both the VFA and EDCA deal with the
during the trial and detention after conviction." 241 Pursuant to presence of U.S. forces within the Philippines, but make no
Article V(6) of the VFA, the custody of a U.S. military personnel mention of being platforms for activity beyond Philippine
resides with U.S. military authorities during trial. Once there is territory. While it may be that, as applied, military operations
a finding of guilt, Article V(l0) requires that the confinement or under either the VFA or EDCA would be carried out in the
detention be "by Philippine authorities." future the scope of judicial review does not cover potential
breaches of discretion but only actual occurrences or blatantly
Justice Marvic M.V.F. Leonen's Dissenting Opinion posits that illegal provisions. Hence, we cannot invalidate EDCA on the
EDCA "substantially modifies or amends the VFA" 242and basis of the potentially abusive use of its provisions.
follows with an enumeration of the differences between EDCA
and the VFA. While these arguments will be rebutted more fully The fourth difference is that EDCA supposedly introduces a
further on, an initial answer can already be given to each of the new concept not contemplated in the VFA or the MDT: Agreed
concerns raised by his dissent. Locations, Contractors, Pre-positioning, and Operational
Control.248
The first difference emphasized is that EDCA does not only
regulate visits as the VFA does, but allows temporary As previously mentioned, these points shall be addressed fully
stationing on a rotational basis of U.S. military personnel and and individually in the latter analysis of EDCA's provisions.
their contractors in physical locations with permanent facilities However, it must already be clarified that the terms and details
and pre-positioned military materiel. used by an implementing agreement need not be found in the
mother treaty. They must be sourced from the authority derived
This argument does not take into account that these from the treaty, but are not necessarily expressed word-for-
word in the mother treaty. This concern shall be further
permanent facilities, while built by U.S. forces, are to be owned
by the Philippines once constructed.243 Even the VFA allowed elucidated in this Decision.
construction for the benefit of U.S. forces during their
temporary visits. The fifth difference highlighted by the Dissenting Opinion is that
the VFA does not have provisions that may be construed as a
The second difference stated by the dissent is that EDCA restriction on or modification of obligations found in existing
statues, including the jurisdiction of courts, local autonomy,
allows the prepositioning of military materiel, which can include
various types of warships, fighter planes, bombers, and and taxation. Implied in this argument is that EDCA contains
vessels, as well as land and amphibious vehicles and their such restrictions or modifications.249
corresponding ammunition.244
This last argument cannot be accepted in view of the clear
However, the VFA clearly allows the same kind of equipment, provisions of EDCA. Both the VFA and EDCA ensure
vehicles, vessels, and aircraft to be brought into the country. Philippine jurisdiction in all instances contemplated by both
Articles VII and VIII of the VFA contemplates that U.S. agreements, with the exception of those outlined by the VFA in
equipment, materials, supplies, and other property are Articles III-VI. In the VFA, taxes are clearly waived whereas in
imported into or acquired in the Philippines by or on behalf of EDCA, taxes are assumed by the government as will be
the U.S. Armed Forces; as are vehicles, vessels, and aircraft discussed later on. This fact does not, therefore, produce a
diminution of jurisdiction on the part of the Philippines, but
operated by or for U.S. forces in connection with activities
under the VFA. These provisions likewise provide for the rather a recognition of sovereignty and the rights that attend it,
some of which may be waived as in the cases under Articles
waiver of the specific duties, taxes, charges, and fees that
correspond to these equipment. III-VI of the VFA.

The third difference adverted to by the Justice Leonen's Taking off from these concerns, the provisions of EDCA must
be compared with those of the MDT and the VFA, which are
dissent is that the VFA contemplates the entry of troops for
training exercises, whereas EDCA allows the use of territory the two treaties from which EDCA allegedly draws its validity.
for launching military and paramilitary operations conducted in
other states.245 The dissent of Justice Teresita J. Leonardo-De "Authorized presence" under the VFA versus "authorized
Castro also notes that VFA was intended for non-combat activities" under EDCA: (1) U.S. personnel and (2) U.S.
activides only, whereas the entry and activities of U.S. forces contractors
into Agreed Locations were borne of military necessity or had
a martial character, and were therefore not contemplated by The OSG argues250 that EDCA merely details existing policies
the VFA.246 under the MDT and the VFA. It explains that EDCA articulates
the principle of defensive preparation embodied in Article II of
This Court's jurisprudence however established in no uncertain the MDT; and seeks to enhance the defensive, strategic, and
terms that combat-related activities, as opposed to actual technological capabilities of both parties pursuant to the
combat, were allowed under the MDT and VFA, viz: objective of the treaty to strengthen those capabilities to
prevent or resist a possible armed attack. Respondent also
Both the history and intent of the Mutual Defense Treaty and points out that EDCA simply implements Article I of the VFA,
the VFA support the conclusion that combat-related activities which already allows the entry of U.S. troops and personnel
as opposed to combat itself such as the one subject of the into the country. Respondent stresses this Court's recognition
in Lim v. Executive Secretary that U.S. troops and personnel
instant petition, are indeed authorized.247
are authorized to conduct activities that promote the goal of
maintaining and developing their defense capability.
Hence, even if EDCA was borne of military necessity, it cannot
be said to have strayed from the intent of the VFA since
EDCA's combat-related components are allowed under the Petitioners contest251 the assertion that the provisions of EDCA
treaty. merely implement the MDT. According to them, the treaty does
not specifically authorize the entry of U.S. troops in the country
in order to maintain and develop the individual and collective
FINALS CONSTITUTION I ACJUCO 283

capacities of both the Philippines and the U.S. to resist an Philippines in connection with activities approved by the
armed attack. They emphasize that the treaty was concluded Philippine Government. Within this definition:
at a time when there was as yet no specific constitutional
prohibition on the presence of foreign military forces in the
1. The term "military personnel" refers
country. to military members of the United States
Army, Navy, Marine Corps, Air Force,
Petitioners also challenge the argument that EDCA simply and Coast Guard.
implements the VFA. They assert that the agreement covers
only short-term or temporary visits of U.S. troops "from time to 2. The term "civilian personnel" refers to
time" for the specific purpose of combined military exercises individuals who are neither nationals of
with their Filipino counterparts. They stress that, in contrast, nor ordinarily resident in the
U.S. troops are allowed under EDCA to perform Philippines and who are employed by
activities beyond combined military exercises, such as those the United States armed forces or who
enumerated in Articles 111(1) and IV(4) thereof. Furthermore, are accompanying the United States
there is some degree of permanence in the presence of U.S. armed forces, such as employees of
troops in the country, since the effectivity of EDCA is the American Red Cross and the United
continuous until terminated. They proceed to argue that while
Services Organization.258
troops have a "rotational" presence, this scheme in fact fosters
their permanent presence.
Article II of EDCA must then be read with Article III of the VFA,
which provides for the entry accommodations to be accorded
a. Admission of U.S. military and civilian personnel into
to U.S. military and civilian personnel:
Philippine territory is already allowed under the VFA

1. The Government of the Philippines shall


We shall first deal with the recognition under EDCA of the facilitate the admission of United States
presence in the country of three distinct classes of individuals
personnel and their departure from the Philippines
who will be conducting different types of activities within the in connection with activities covered by this
Agreed Locations: (1) U.S. military personnel; (2) U.S. civilian
agreement.
personnel; and (3) U.S. contractors. The agreement refers to
them as follows:
2. United States military personnel shall be
exempt from passport and visa regulations upon
"United States personnel" means United States
enteringand departing the Philippines.
military and civilian personnel temporarily in the territory
of the Philippines in connection with activities approved by
the Philippines, as those terms are defined in the VFA.252 3. The following documents only, which shall be
required in respect of United States military
personnel who enter the Philippines; xx xx.
"United States forces" means the entity comprising United
States personnel and all property, equipment, and
materiel of the United States Armed Forces present in the 4. United States civilian personnel shall be exempt
territory of the Philippines.253 from visa requirements but shall present, upon
demand, valid passports upon entry and
departure of the Philippines. (Emphases Supplied)
"United States contractors" means companies and firms,
and their employees, under contract or subcontract to or
on behalf of the United States Department of Defense. United By virtue of Articles I and III of the VFA, the Philippines already
States contractors are not included as part of the definition allows U.S. military and civilian personnel to be "temporarily in
of United States personnel in this Agreement, including the Philippines," so long as their presence is "in connection
within the context of the VFA.254 with activities approved by the Philippine Government." The
Philippines, through Article III, even guarantees that it shall
facilitate the admission of U.S. personnel into the country and
United States forces may contract for any materiel, grant exemptions from passport and visa regulations. The VFA
supplies, equipment, and services (including construction) does not even limit their temporary presence to specific
to be furnished or undertaken in the territory of the Philippines locations.
without restriction as to choice of contractor, supplier, or
person who provides such materiel, supplies, equipment,
or services. Such contracts shall be solicited, awarded, and Based on the above provisions, the admission and presence
administered in accordance with the laws and regulations of of U.S. military and civilian personnel in Philippine
the United States.255 (Emphases Supplied) territory are already allowed under the VFA, the treaty
supposedly being implemented by EDCA. What EDCA has
effectively done, in fact, is merely provide the mechanism to
A thorough evaluation of how EDCA is phrased clarities that identify the locations in which U.S. personnel may perform
the agreement does not deal with the entry into the
allowed activities pursuant to the VFA. As the implementing
country of U.S. personnel and contractors per se. While agreement, it regulates and limits the presence of U.S.
Articles I(l)(b)256 and II(4)257 speak of "the right to access and
personnel in the country.
use" the Agreed Locations, their wordings indicate the
presumption that these groups have already been allowed
entry into Philippine territory, for which, unlike the VFA, EDCA b. EDCA does not provide the legal basis for admission of U.S.
has no specific provision. Instead, Article II of the latter simply contractors into Philippine territory; their entry must be sourced
alludes to the VFA in describing U.S. personnel, a term defined from extraneous Philippine statutes and regulations for the
under Article I of the treaty as follows: admission of alien employees or business persons.

As used in this Agreement, "United States personnel" means Of the three aforementioned classes of individuals who will be
United States military and civilian personnel temporarily in the conducting certain activities within the Agreed Locations, we
note that only U.S. contractors are not explicitly mentioned in
FINALS CONSTITUTION I ACJUCO 284

the VFA. This does not mean, though, that the recognition of xxx
their presence under EDCA is ipso facto an amendment of the
treaty, and that there must be Senate concurrence before they Reaffirming their obligations under the Mutual Defense
are allowed to enter the country.
Treaty of August 30, 1951;

Nowhere in EDCA are U.S. contractors guaranteed immediate


Noting that from time to time elements of the United States
admission into the Philippines. Articles III and IV, in fact, merely armed forces may visit the Republic of the Philippines;
grant them the right of access to, and the authority to conduct
certain activities within the Agreed Locations. Since Article II(3)
of EDCA specifically leaves out U.S. contractors from the Considering that cooperation between the United States and
coverage of the VFA, they shall not be granted the same entry the Republic of the Philippines promotes their common
accommodations and privileges as those enjoyed by U.S. security interests;
military and civilian personnel under the VFA.
xxx
Consequently, it is neither mandatory nor obligatory on the part
of the Philippines to admit U.S. contractors into the Article I - Definitions
country.259 We emphasize that the admission of aliens into
Philippine territory is "a matter of pure permission and simple
tolerance which creates no obligation on the part of the As used in this Agreement, "United States personnel" means
government to permit them to stay."260 Unlike U.S. personnel United States military and civilian personnel temporarily in the
who are accorded entry accommodations, U.S. contractors are Philippines in connection with activities approved by the
subject to Philippine immigration laws. 261The latter must Philippine Government. Within this definition: xx x
comply with our visa and passport regulations262 and prove
that they are not subject to exclusion under any provision of Article II - Respect for Law
Philippine immigration laws.263 The President may also deny
them entry pursuant to his absolute and unqualified power to
prohibit or prevent the admission of aliens whose presence in It is the duty of United States personnel to respect the laws
the country would be inimical to public interest.264 of the Republic of the Philippines and to abstain from any
activity inconsistent with the spirit of this agreement, and,
in particular, from any political activity in the Philippines. The
In the same vein, the President may exercise the plenary Government of the United States shall take all measures within
power to expel or deport U.S. contractors 265 as may be its authority to ensure that this is done.
necessitated by national security, public safety, public health,
public morals, and national interest.266 They may also be
deported if they are found to be illegal or undesirable aliens Article VII - Importation and Exportation
pursuant to the Philippine Immigration Act 267 and the Data
Privacy Act.268 In contrast, Article 111(5) of the VFA requires a 1. United States Government equipment, materials,
request for removal from the Philippine government before a supplies, and other property imported into or acquired in
member of the U.S. personnel may be "dispos[ed] xx x outside the Philippines by or on behalf of the United States armed
of the Philippines." forces in connection with activities to which this
agreement applies, shall be free of all Philippine duties, taxes
c. Authorized activities of U.S. military and civilian personnel and other similar charges. Title to such property shall remain
within Philippine territory are in furtherance of the MDT and the with the United States, which may remove such property from
VFA the Philippines at any time, free from export duties, taxes, and
other similar charges. x x x.

We begin our analysis by quoting the relevant sections of the


MDT and the VFA that pertain to the activities in which U.S. Article VIII - Movement of Vessels and Aircraft
military and civilian personnel may engage:
1. Aircraft operated by or for the United States armed
MUTUAL DEFENSE TREATY forces may enter the Philippines upon approval of the
Government of the Philippines in accordance with procedures
stipulated in implementing arrangements.
Article II
2. Vessels operated by or for the United States armed
In order more effectively to achieve the objective of this Treaty, forces may enter the Philippines upon approval of the
the Parties separately and jointly byself-help and mutual Government of the Philippines. The movement of vessels
aid will maintain and develop their individual and shall be in accordance with international custom and
collective capacity to resist armed attack. practice governing such vessels, and such agreed
implementing arrangements as necessary. x x x
Article III (Emphases Supplied)

The Parties, through their Foreign Ministers or their Manifest in these provisions is the abundance of references to
deputies, will consult together from time to time regarding the creation of further "implementing arrangements" including
the implementation of this Treaty and whenever in the the identification of "activities [to be] approved by the Philippine
opinion of either of them the territorial integrity, political Government." To determine the parameters of these
independence or security of either of the Parties is threatened implementing arrangements and activities, we referred to the
by external armed attack in the Pacific. content, purpose, and framework of the MDT and the VFA.

VISITING FORCES AGREEMENT By its very language, the MDT contemplates a situation in
which both countries shall engage in joint activities, so that
they can maintain and develop their defense capabilities. The
Preamble
FINALS CONSTITUTION I ACJUCO 285

wording itself evidently invites a reasonable construction that other than military. As conceived, the joint exercises may
the joint activities shall involve joint military trainings, include training on new techniques of patrol and surveillance
maneuvers, and exercises. Both the interpretation 269 and the to protect the nation's marine resources, sea search-and-
subsequent practice270 of the parties show that the MDT rescue operations to assist vessels in distress, disaster relief
independently allows joint military exercises in the country. Lim operations, civic action projects such as the building of school
v. Executive Secretary271 and Nicolas v. Romulo272 recognized houses, medical and humanitarian missions, and the like.
that Balikatan exercises, which are activities that seek to
enhance and develop the strategic and technological Under these auspices, the VFA gives legitimacy to the current
capabilities of the parties to resist an armed attack, "fall Balikatan exercises. It is only logical to assume that
squarely under the provisions of the RP-US "Balikatan 02-1," a "mutual anti-terrorism advising,
MDT."273 In Lim, the Court especially noted that the Philippines assisting and training exercise," falls under the umbrella
and the U.S. continued to conduct joint military exercises even of sanctioned or allowable activities in the context of the
after the expiration of the MBA and even before the conclusion
agreement. Both the history and intent of the Mutual Defense
of the VFA.274 These activities presumably related to the Status Treaty and the VFA support the conclusion that combat-related
of Forces Agreement, in which the parties agreed on the status
activities - as opposed to combat itself- such as the one subject
to be accorded to U.S. military and civilian personnel while of the instant petition, are indeed authorized. (Emphases
conducting activities in the Philippines in relation to the MDT. 275 Supplied)

Further, it can be logically inferred from Article V of the MDT


The joint report of the Senate committees on foreign relations
that these joint activities may be conducted on Philippine or on and on national defense and security further explains the wide
U.S. soil. The article expressly provides that the term armed range and variety of activities contemplated in the VFA, and
attack includes "an armed attack on the metropolitan how these activities shall be identified: 277
territory of either of the Parties, or on the island territories
under its jurisdiction in the Pacific or on its armed forces,
public vessels or aircraft in the Pacific." Surely, in These joint exercises envisioned in the VFA are not limited
maintaining and developing our defense capabilities, an to combat-related activities; they have a wide range and
assessment or training will need to be performed, separately variety. They include exercises that will reinforce the AFP's
and jointly by self-help and mutual aid, in the territories of the ability to acquire new techniques of patrol and
contracting parties. It is reasonable to conclude that the surveillance to protect the country's maritime resources; sea-
assessment of defense capabilities would entail understanding search and rescue operations to assist ships in distress;
the terrain, wind flow patterns, and other environmental factors and disaster-relief operations to aid the civilian victims of
unique to the Philippines. natural calamities, such as earthquakes, typhoons and tidal
waves.
It would also be reasonable to conclude that a simulation of
how to respond to attacks in vulnerable areas would be part of xxxx
the training of the parties to maintain and develop their capacity
to resist an actual armed attack and to test and validate the Joint activities under the VFA will include combat maneuvers;
defense plan of the Philippines. It is likewise reasonable to training in aircraft maintenance and equipment repair; civic-
imagine that part of the training would involve an analysis of action projects; and consultations and meetings of the
the effect of the weapons that may be used and how to be Philippine-U.S. Mutual Defense Board. It is at the level of the
prepared for the eventuality. This Court recognizes that all of Mutual Defense Board-which is headed jointly by the Chief of
this may require training in the area where an armed attack Staff of the AFP and the Commander in Chief of the U.S.
might be directed at the Philippine territory. Pacific Command-that the VFA exercises are planned. Final
approval of any activity involving U.S. forces is,
The provisions of the MDT must then be read in conjunction however, invariably given by the Philippine Government.
with those of the VFA.
xxxx
Article I of the VFA indicates that the presence of U.S. military
and civilian personnel in the Philippines is "in connection with Siazon clarified that it is not the VFA by itself that
activities approved by the Philippine Government." While the determines what activities will be conductedbetween the
treaty does not expressly enumerate or detail the nature of armed forces of the U.S. and the Philippines. The VFA
activities of U.S. troops in the country, its Preamble makes regulates and provides the legal framework for the
explicit references to the reaffirmation of the obligations of both presence, conduct and legal status of U.S. personnel while
countries under the MDT. These obligations include the they are in the country for visits, joint exercises and other
strengthening of international and regional security in the related activities. (Emphases Supplied)
Pacific area and the promotion of common security interests.
What can be gleaned from the provisions of the VFA, the
The Court has already settled in Lim v. Executive joint report of the Senate committees on foreign relations
Secretary that the phrase "activities approved by the Philippine and on national defense and security, and the ruling of this
Government" under Article I of the VFA was intended to be Court in Lim is that the "activities" referred to in the treaty
ambiguous in order to afford the parties flexibility to adjust the are meant to be specified and
details of the purpose of the visit of U.S. personnel.276 In ruling identified infurther agreements. EDCA is one such
that the Terms of Reference for the Balikatan Exercises in agreement.
2002 fell within the context of the treaty, this Court explained:
EDCA seeks to be an instrument that enumerates the
After studied reflection, it appeared farfetched that Philippine-approved activities of U.S. personnel referred to in
the ambiguity surrounding the meaning of the word the VFA. EDCA allows U.S. military and civilian personnel to
"activities" arose from accident. In our view, it perform "activities approved by the Philippines, as those terms
was deliberately made that way to give both parties a are defined in the VFA"278 and clarifies that these activities
certain leeway in negotiation. In this manner, visiting US include those conducted within the Agreed Locations:
forces may sojourn in Philippine territory for purposes
FINALS CONSTITUTION I ACJUCO 286

1. Security cooperation exercises; joint and combined training The Exercise is a mutual counter-terrorism advising,
activities; humanitarian assistance and disaster relief activities; assisting and training Exercise relative to Philippine efforts
and such other activities as may be agreed upon by the against the ASG, and will be conducted on the Island of
Parties279 Basilan. Further advising, assisting and training exercises shall
be conducted in Malagutay and the Zamboanga area. Related
2. Training; transit; support and related activities; refueling of activities in Cebu will be for support of the Exercise.
aircraft; bunkering of vessels; temporary maintenance of
vehicles, vessels, and aircraft; temporary accommodation of xx xx.
personnel; communications; prepositioning of equipment,
supplies, and materiel; deployment of forces and materiel; and US exercise participants shall not engage in combat,
such other activities as the Parties may agree280 without prejudice to their right of self-defense.

3. Exercise of operational control over the Agreed Locations These terms of Reference are for purposes of this Exercise
for construction activities and other types of activity, including
only and do not create additional legal obligations between the
alterations and improvements thereof281 US Government and the Republic of the Philippines.

4. Exercise of all rights and authorities within the Agreed II. EXERCISE LEVEL
Locations that are necessary for their operational control or
defense, including the adoption of apfropriate measures to
protect U.S. forces and contractors282 1. TRAINING

5. Use of water, electricity, and other public utilities 283 a. The Exercise shall involve the conduct
of mutual military assisting, advising
and trainingof RP and US Forces with the
6. Operation of their own telecommunication systems,
primary objective of enhancing the
including the utilization of such means and services as are operational capabilities of both forces to
required to ensure the full ability to operate telecommunication
combat terrorism.
systems, as well as the use of the necessary radio spectrum
allocated for this purpose284
b. At no time shall US Forces operate
independently within RP territory.
According to Article I of EDCA, one of the purposes of these
activities is to maintain and develop, jointly and by mutual aid,
the individual and collective capacities of both countries to c. Flight plans of all aircraft involved in the
resist an armed attack. It further states that the activities are in exercise will comply with the local air traffic
furtherance of the MDT and within the context of the VFA. regulations.

We note that these planned activities are very similar to those 2. ADMINISTRATION & LOGISTICS
under the Terms of Reference285 mentioned in Lim. Both
EDCA and the Terms of Reference authorize the U.S. to xxxx
perform the following: (a) participate in training exercises; (b)
retain command over their forces; (c) establish temporary
structures in the country; (d) share in the use of their respective a. RP and US participating forces may share, in accordance
resources, equipment and other assets; and (e) exercise their with their respective laws and regulations, in the use of their
right to self-defense. We quote the relevant portion of the resources, equipment and other assets. They will use their
Terms and Conditions as follows:286 respective logistics channels. x x x. (Emphases Supplied)

I. POLICY LEVEL After a thorough examination of the content, purpose, and


framework of the MDT and the VFA, we find that EDCA has
remained within the parameters set in these two treaties. Just
xxxx like the Terms of Reference mentioned in Lim, mere
adjustments in detail to implement the MDT and the VFA can
No permanent US basing and support facilities shall be be in the form of executive agreements.
established. Temporary structures such as those for troop
billeting, classroom instruction and messing may be set Petitioners assert287 that the duration of the activities
up for use by RP and US Forces during the Exercise. mentioned in EDCA is no longer consistent with the temporary
nature of the visits as contemplated in the VFA. They point out
The Exercise shall be implemented jointly by RP and US that Article XII(4) of EDCA has an initial term of 10 years, a
Exercise Co-Directors under the authority of the Chief of Staff, term automatically renewed unless the Philippines or the U.S.
AFP. In no instance will US Forces operate independently terminates the agreement. According to petitioners, such
during field training exercises (FTX). AFP and US Unit length of time already has a badge of permanency.
Commanders will retain command over their respective
forces under the overall authority of the Exercise Co- In connection with this, Justice Teresita J. Leonardo-De Castro
Directors. RP and US participants shall comply with likewise argues in her Concurring and Dissenting Opinion that
operational instructions of the AFP during the FTX. the VFA contemplated mere temporary visits from U.S. forces,
whereas EDCA allows an unlimited period for U.S. forces to
The exercise shall be conducted and completed within a period stay in the Philippines.288
of not more than six months, with the projected participation of
660 US personnel and 3,800 RP Forces. The Chief of Staff, However, the provisions of EDCA directly contradict this
AFP shall direct the Exercise Co-Directors to wind up and argument by limiting itself to 10 years of effectivity. Although
terminate the Exercise and other activities within the six month this term is automatically renewed, the process for terminating
Exercise period.
FINALS CONSTITUTION I ACJUCO 287

the agreement is unilateral and the right to do so automatically practice of other states in not specifying a non-extendible
accrues at the end of the 10 year period. Clearly, this method maximum term. This practice, however, does not automatically
does not create a permanent obligation. grant a badge of permanency to its terms. Article XII(4) of
EDCA provides very clearly, in fact, that its effectivity is for an
Drawing on the reasoning in Lim, we also believe that it could initial term of 10 years, which is far shorter than the terms of
not have been by chance that the VFA does not include a effectivity between the U.S. and other states. It is simply
maximum time limit with respect to the presence of U.S. illogical to conclude that the initial, extendible term of 10 years
personnel in the country. We construe this lack of specificity as somehow gives EDCA provisions a permanent character.
a deliberate effort on the part of the Philippine and the U.S.
governments to leave out this aspect and reserve it for the The reasoning behind this interpretation is rooted in the
"adjustment in detail" stage of the implementation of the treaty. constitutional role of the President who, as Commander-in-
We interpret the subsequent, unconditional concurrence of the Chief of our armed forces, is the principal strategist of the
Senate in the entire text of the VFA as an implicit grant to the nation and, as such, duty-bound to defend our national
President of a margin of appreciation in determining the sovereignty and territorial integrity;291 who, as chief architect of
duration of the "temporary" presence of U.S. personnel in the our foreign relations, is the head policymaker tasked to assess,
country. ensure, and protect our national security and interests; 292 who
holds the most comprehensive and most confidential
Justice Brion's dissent argues that the presence of U.S. forces information about foreign countries 293 that may affect how we
conduct our external affairs; and who has unrestricted access
under EDCA is "more permanent" in nature. 289However, this
argument has not taken root by virtue of a simple glance at its to highly classified military intelligence data294 that may
provisions on the effectivity period. EDCA does not grant threaten the life of the nation. Thus, if after a geopolitical
permanent bases, but rather temporary rotational access to prognosis of situations affecting the country, a belief is
facilities for efficiency. As Professor Aileen S.P. Baviera notes: engendered that a much longer period of military training is
needed, the President must be given ample discretion to adopt
necessary measures including the flexibility to set an extended
The new EDCA would grant American troops, ships and planes timetable.
rotational access to facilities of the Armed Forces of the
Philippines – but not permanent bases which are prohibited
under the Philippine Constitution - with the result of reducing Due to the sensitivity and often strict confidentiality of these
response time should an external threat from a common concerns, we acknowledge that the President may not always
adversary crystallize.290 be able to candidly and openly discuss the complete situation
being faced by the nation. The Chief Executive's hands must
not be unduly tied, especially if the situation calls for crafting
EDCA is far from being permanent in nature compared to the programs and setting timelines for approved activities. These
practice of states as shown in other defense cooperation activities may be necessary for maintaining and developing our
agreements. For example, Article XIV(l) of the U.S.-Romania capacity to resist an armed attack, ensuring our national
defense agreement provides the following: sovereignty and territorial integrity, and securing our national
interests. If the Senate decides that the President is in the best
This Agreement is concluded for an indefinite period and position to define in operational terms the meaning
shall enter into force in accordance with the internal laws of of temporary in relation to the visits, considered individually or
each Party x x x. (emphasis supplied) in their totality, the Court must respect that policy decision. If
the Senate feels that there is no need to set a time limit to these
visits, neither should we.
Likewise, Article 36(2) of the US-Poland Status of Forces
Agreement reads:
Evidently, the fact that the VFA does not provide specificity in
regard to the extent of the "temporary" nature of the visits of
This Agreement has been concluded for an indefinite period U.S. personnel does not suggest that the duration to which the
of time. It may be terminated by written notification by either President may agree is unlimited. Instead, the boundaries of
Party and in that event it terminates 2 years after the receipt of the meaning of the term temporary in Article I of the treaty must
the notification. (emphasis supplied) be measured depending on the purpose of each visit or
activity.295 That purpose must be analyzed on a case-by-case
Section VIII of US.-Denmark Mutual Support basis depending on the factual circumstances surrounding the
Agreement similarly provides: conclusion of the implementing agreement. While the validity
of the President's actions will be judged under less stringent
standards, the power of this Court to determine whether there
8.1 This Agreement, which consists of a Preamble, SECTIONs was grave abuse of discretion remains unimpaired.
I-VIII, and Annexes A and B, shall become effective on the date
of the last signature affixed below and shall remain in force
until terminated by the Parties, provided that it may be d. Authorized activities performed by US. contractors within
terminated by either Party upon 180 days written notice of its Philippine territory - who were legitimately permitted to enter
intention to do so to the other Party. (emphasis supplied) the country independent of EDCA - are subject to relevant
Philippine statutes and regulations and must be consistent with
the MDT and the VFA
On the other hand, Article XXI(3) of the US.-Australia Force
Posture Agreement provides a longer initial term:
Petitioners also raise296 concerns about the U.S. government's
purported practice of hiring private security contractors in other
3. This Agreement shall have an initial term of 25 years and countries. They claim that these contractors - one of which has
thereafter shall continue in force, but may be terminated by already been operating in Mindanao since 2004 - have been
either Party at any time upon one year's written notice to the implicated in incidents or scandals in other parts of the globe
other Party through diplomatic channels. (emphasis supplied) involving rendition, torture and other human rights violations.
They also assert that these contractors employ paramilitary
The phrasing in EDCA is similar to that in the U.S.-Australia forces in other countries where they are operating.
treaty but with a term less than half of that is provided in the
latter agreement. This means that EDCA merely follows the
FINALS CONSTITUTION I ACJUCO 288

Under Articles III and IV of EDCA, U.S. contractors are consistent with Philippine laws and regulations and pursuant to
authorized to perform only the following activities: the MDT and the VFA.

1. Training; transit; support and related activities; While we recognize the concerns of petitioners, they do not
refueling of aircraft; bunkering of vessels; temporary give the Court enough justification to strike down EDCA. In Lim
maintenance of vehicles, vessels, and aircraft; v. Executive Secretary, we have already explained that we
temporary accommodation of personnel; cannot take judicial notice of claims aired in news reports, "not
communications; prepositioning of equipment, because of any issue as to their truth, accuracy, or impartiality,
supplies, and materiel; deployment of forces and but for the simple reason that facts must be established in
materiel; and such other activities as the Parties may accordance with the rules of evidence."308 What is more, we
agree297 cannot move one step ahead and speculate that the alleged
illegal activities of these contractors in other countries would
2. Prepositioning and storage of defense equipment, take place in the Philippines with certainty. As can be seen
supplies, and materiel, including delivery, from the above discussion, making sure that U.S. contractors
management, inspection, use, maintenance, and comply with Philippine laws is a function of law enforcement.
removal of such equipment, supplies and materiel 298 EDCA does not stand in the way of law enforcement.

3. Carrying out of matters in accordance with, and to Nevertheless, we emphasize that U.S. contractors are
the extent permissible under, U.S. laws, regulations, explicitly excluded from the coverage of the VFA. As visiting
and policies299 aliens, their entry, presence, and activities are subject to all
laws and treaties applicable within the Philippine territory. They
may be refused entry or expelled from the country if they
EDCA requires that all activities within Philippine territory be in engage in illegal or undesirable activities. There is nothing that
accordance with Philippine law. This means that certain prevents them from being detained in the country or being
privileges denied to aliens are likewise denied to foreign subject to the jurisdiction of our courts. Our penal laws, 309 labor
military contractors. Relevantly, providing security300and laws,310 and immigrations laws311 apply to them and therefore
carrying, owning, and possessing firearms301 are illegal for limit their activities here. Until and unless there is another law
foreign civilians. or treaty that specifically deals with their entry and activities,
their presence in the country is subject to unqualified Philippine
The laws in place already address issues regarding the jurisdiction.
regulation of contractors. In the 2015 Foreign Investment
Negative list,302 the Executive Department has already EDCA does not allow the presence of U.S.-owned or -
identified corporations that have equity restrictions in controlled military facilities and bases in the Philippines
Philippine jurisdiction. Of note is No. 5 on the list - private
security agencies that cannot have any foreign equity by virtue
of Section 4 of Republic Act No. 5487; 303 and No. 15, which Petitioners Saguisag et al. claim that EDCA permits the
regulates contracts for the construction of defense-related establishment of U.S. military bases through the
structures based on Commonwealth Act No. 541. "euphemistically" termed "Agreed Locations. "312 Alluding to
the definition of this term in Article II(4) of EDCA, they point out
that these locations are actually military bases, as the definition
Hence, any other entity brought into the Philippines by virtue refers to facilities and areas to which U.S. military forces have
of EDCA must subscribe to corporate and civil requirements access for a variety of purposes. Petitioners claim that there
imposed by the law, depending on the entity's corporate are several badges of exclusivity in the use of the Agreed
structure and the nature of its business. Locations by U.S. forces. First, Article V(2) of EDCA alludes to
a "return" of these areas once they are no longer needed by
That Philippine laws extraneous to EDCA shall govern the U.S. forces, indicating that there would be some transfer of
regulation of the activities of U.S. contractors has been clear use. Second, Article IV(4) ofEDCA talks about American
even to some of the present members of the Senate. forces' unimpeded access to the Agreed Locations for all
matters relating to the prepositioning and storage of U.S.
military equipment, supplies, and materiel. Third, Article VII of
For instance, in 2012, a U.S. Navy contractor, the Glenn EDCA authorizes U.S. forces to use public utilities and to
Marine, was accused of spilling fuel in the waters off Manila operate their own telecommunications system.
Bay.304 The Senate Committee on Foreign Relations and the
Senate Committee on Environment and Natural Resources
chairperson claimed environmental and procedural violations a. Preliminary point on badges of exclusivity
by the contractor.305 The U.S. Navy investigated the contractor
and promised stricter guidelines to be imposed upon its As a preliminary observation, petitioners have cherry-picked
contractors.306 The statement attributed to Commander Ron provisions of EDCA by presenting so-called "badges of
Steiner of the public affairs office of the U.S. Navy's 7th Fleet - exclusivity," despite the presence of contrary provisions within
that U.S. Navy contractors are bound by Philippine laws - is of the text of the agreement itself.
particular relevance. The statement acknowledges not just the
presence of the contractors, but also the U.S. position that
these contractors are bound by the local laws of their host First, they clarify the word "return" in Article V(2) of EDCA.
state. This stance was echoed by other U.S. Navy However, the use of the word "return" is within the context of a
representatives.307 lengthy provision. The provision as a whole reads as follows:

This incident simply shows that the Senate was well aware of The United States shall return to the Philippines any Agreed
the presence of U.S. contractors for the purpose of fulfilling the Locations, or any portion thereof, including non-relocatable
terms of the VFA. That they are bound by Philippine law is clear structures and assemblies constructed, modified, or improved
to all, even to the U.S. by the United States, once no longer required by United States
forces for activities under this Agreement. The Parties or the
Designated Authorities shall consult regarding the terms of
As applied to EDCA, even when U.S. contractors are granted
access to the Agreed Locations, all their activities must be
FINALS CONSTITUTION I ACJUCO 289

return of any Agreed Locations, including possible control over the Agreed Locations specifically for construction
compensation for improvements or construction. activities. They do not allow the overarching power to operate,
maintain, utilize, occupy, garrison, and control a base with full
discretion. EDCA in fact limits the rights of the U.S. in respect
The context of use is "required by United States forces for
activities under this Agreement." Therefore, the return of an of every activity, including construction, by giving the MDB and
Agreed Location would be within the parameters of an activity the SEB the power to determine the details of all activities such
that the Mutual Defense Board (MDB) and the Security as, but not limited to, operation, maintenance, utility,
Engagement Board (SEB) would authorize. Thus, possession occupancy, garrisoning, and control. 322
by the U.S. prior to its return of the Agreed Location would be
based on the authority given to it by a joint body co-chaired by The "species of ownership" on the other hand, is distinguished
the "AFP Chief of Staff and Commander, U.S. PACOM with by the nature of the property. For immovable property
representatives from the Philippines' Department of National constructed or developed by the U.S., EDCA expresses that
Defense and Department of Foreign Affairs sitting as ownership will automatically be vested to the Philippines. 323 On
members."313 The terms shall be negotiated by both the the other hand, for movable properties brought into the
Philippines and the U.S., or through their Designated Philippines by the U.S., EDCA provides that ownership is
Authorities. This provision, seen as a whole, contradicts retained by the latter. In contrast, the MBA dictates that the
petitioners' interpretation of the return as a "badge of U.S. retains ownership over immovable and movable
exclusivity." In fact, it shows the cooperation and partnership properties.
aspect of EDCA in full bloom.
To our mind, both EDCA and the MBA simply incorporate what
Second, the term "unimpeded access" must likewise be viewed is already the law of the land in the Philippines. The Civil
from a contextual perspective. Article IV(4) states that U.S. Code's provisions on ownership, as applied, grant the owner
forces and U.S. contractors shall have "unimpeded access to of a movable property full rights over that property, even if
Agreed Locations for all matters relating to the prepositioning located in another person's property. 324
and storage of defense equipment, supplies, and materiel,
including delivery, management, inspection, use, The parallelism, however, ends when the situation involves
maintenance, and removal of such equipment, supplies and facilities that can be considered immovable. Under the MBA,
materiel." the U.S. retains ownership if it paid for the facility.325 Under
EDCA, an immovable is owned by the Philippines, even if built
At the beginning of Article IV, EDCA states that the Philippines completely on the back of U.S. funding.326 This is consistent
gives the U.S. the authority to bring in these equipment, with the constitutional prohibition on foreign land ownership. 327
supplies, and materiel through the MDB and SEB security
mechanism. These items are owned by the U.S., 314 are Despite the apparent similarity, the ownership of property is but
exclusively for the use of the U.S.315 and, after going through a part of a larger whole that must be considered before the
the joint consent mechanisms of the MDB and the SEB, are constitutional restriction is violated. Thus, petitioners' points on
within the control of the U.S.316 More importantly, before these
operational control will be given more attention in the
items are considered prepositioned, they must have gone discussion below. The arguments on policy are, however,
through the process of prior authorization by the MDB and the
outside the scope of judicial review and will not be discussed
SEB and given proper notification to the AFP. 317

Moreover, a direct comparison of the MBA and EDCA will


Therefore, this "unimpeded access" to the Agreed Locations is result in several important distinctions that would allay
a necessary adjunct to the ownership, use, and control of the
suspicion that EDCA is but a disguised version of the MBA.
U.S. over its own equipment, supplies, and materiel and must
have first been allowed by the joint mechanisms in play
between the two states since the time of the MDT and the VFA. b. There are substantial matters that the US. cannot do under
It is not the use of the Agreed Locations that is exclusive per EDCA, but which it was authorized to do under the 1947 MBA
se; it is mere access to items in order to exercise the rights of
ownership granted by virtue of the Philippine Civil Code.318 The Philippine experience with U.S. military bases under the
1947 MBA is simply not possible under EDCA for a number of
As for the view that EDCA authorizes U.S. forces to use public important reasons.
utilities and to operate their own telecommunications system,
it will be met and answered in part D, infra. First, in the 1947 MBA, the U.S. retained all rights of
jurisdiction in and over Philippine territory occupied by
Petitioners also point out319 that EDCA is strongly reminiscent American bases. In contrast, the U.S. under EDCA does not
of and in fact bears a one-to-one correspondence with the enjoy any such right over any part of the Philippines in which
provisions of the 1947 MBA. They assert that both agreements its forces or equipment may be found. Below is a comparative
(a) allow similar activities within the area; (b) provide for the table between the old treaty and EDCA:
same "species of ownership" over facilities; and (c) grant
operational control over the entire area. Finally, they
argue320 that EDCA is in fact an implementation of the new 1947 MBA/ 1946 Treaty of EDCA
defense policy of the U.S. According to them, this policy was General Relations
not what was originally intended either by the MDT or by the
VFA. 1947 MBA, Art. I(1): EDCA, preamble:

On these points, the Court is not persuaded. The Government of the Affirming that the Parties
Republic of share an understanding for
the Philippines (hereinafter the United States not to
The similar activities cited by petitioners 321 simply show that referred to as the establish a permanent
under the MBA, the U.S. had the right to construct, operate, Philippines) grants to the military presence or base
maintain, utilize, occupy, garrison, and control the bases. The Government of the United
so-called parallel provisions of EDCA allow only operational States of America
FINALS CONSTITUTION I ACJUCO 290

Philippines may be compelled to negotiate with the U.S. the


(hereinafter referred to as in the territory of the
moment the latter requested an expansion of the existing
the United States) the right Philippines;
bases or to acquire additional bases. In EDCA, U.S. access is
to retain the use of the
purely at the invitation of the Philippines.
bases in the xxxx
Philippines listed in Annex
A attached hereto. 1947 MBA/ 1946 Treaty of EDCA
Recognizing that all United
States access to and use of General Relations
1947 MBA, Art. XVII(2): facilities and areas will be at
1947 MBA, Art.I(3): EDCA, preamble:
the invitation of the
All buildings and Philippines and with full
structures which respect for the Philippine The Philippines agree to Recognizing that all United
are erected by the United Constitution and enter into States access to and use of
States in the bases shall be Philippine laws; negotiations with the facilities and areas will be at
the property of the United United States at the the invitation of the
States and may be latter's request, to permit Philippines and with full
xxxx
removed by it before the the United States to respect for the Philippine
expiration of this Agreement expand such bases, to Constitution and
or the earlier relinquishment EDCA, Art. II(4): exchange such bases for Philippine laws;
of the base on which the other bases, to acquire
structures are situated. "Agreed additional bases, or
xxxx
There shall be no obligation Locations" means facilities relinquish rights to bases, as
on the part of the Philippines and areas that are provided any of such exigencies may
or of the United States to be required by military EDCA. Art. II(4):
by the Government of the
rebuild or repair any Philippines through the AFP necessity.
destruction or damage and that United States "Agreed
inflicted from any cause forces, United States 1946 Treaty of Gen. Locations" means facilities
whatsoever on any of the contractors, and others as Relations, Art. I: and areas that are provided
said buildings or structures mutually agreed, shall have by the Government of the
owned or used by the United the right to access and use Philippines through the AFP
States in the bases. x x x x. The United States of and that United States
pursuant to this Agreement.
America agrees to forces, United States
Such Agreed Locations may
withdraw and surrender, contractors, and others as
1946 Treaty of Gen. be listed in an annex to be
and does hereby withdraw mutually agreed, shall have
Relations, Art. I: appended to this Agreement,
and surrender, all rights of the right to access and use
and may be further described
possession, supervision, pursuant to this Agreement.
in implementing
The United States of jurisdiction, control or Such Agreed Locations may
arrangements.
America agrees to withdraw sovereignty existing and be listed in an annex to be
and surrender, and does exercised by the United
appended to this Agreement,
hereby withdraw and EDCA, Art. V: States of America in and and may be further described
surrender, all rights of over the territory and the
in implementing
possession, supervision, 1. people of the Philippine arrangements.
jurisdiction, control or The Philippines shall retain Islands, except the use of
sovereignty existing and ownership of and title to such bases, necessary
exercised by the United Agreed Locations. appurtenances to such
States of America in and bases, and the rights
over the territory and the incident thereto, as the
people of the Philippine xxxx United States of America,
Islands, except the use of by agreement with the
such bases, necessary 4. All buildings, non- Republic of the
appurtenances to such relocatable structures, and Philippines may deem
bases, and the rights assemblies affixed to the necessary to retain for the
incident thereto, as the land in the Agreed mutual protection of the
United States of America, by Locations, including ones Republic of the Philippines
agreement with the altered or improved by and of the United States of
Republic of the Philippines United States forces, remain America. x x x.
may deem necessary to the property of the
retain for the mutual Philippines.Permanent
protection of the Republic of buildings constructed by Third, in EDCA, the Philippines is- guaranteed access over the
the Philippines and of the United States forces become entire area of the Agreed Locations. On the other hand, given
United States of America. x the property of the that the U.S. had complete control over its military bases under
x x. Philippines, once the 1947 MBA, the treaty did not provide for any express
constructed, but shall be recognition of the right of access of Philippine authorities.
used by United States forces Without that provision and in light of the retention of U.S.
until no longer required by sovereignty over the old military bases, the U.S. could
United States forces. effectively prevent Philippine authorities from entering those
bases.

1947 MBA EDCA


Second, in the bases agreement, the U.S. and the Philippines No equivalent provision. EDCA, Art. III(5):
were visibly not on equal footing when it came to deciding
whether to expand or to increase the number of bases, as the
FINALS CONSTITUTION I ACJUCO 291

The Philippine Designated the vicinity of, the


Authority and its bases which
authorized are necessary to
representative shall have provide access
access to the entire area of to them, or
the Agreed Locations. appropriate for
Such access shall be their control.
provided promptly consistent
with operational safety and
security requirements in Fifth, the U.S. under the bases agreement was given the
accordance with agreed authority to use Philippine territory for additional staging areas,
procedures developed by bombing and gunnery ranges. No such right is given under
the Parties. EDCA, as seen below:

Fourth, in the bases agreement, the U.S. retained the right, 1947 MBA EDCA
power, and authority over the establishment, use, operation,
defense, and control of military bases, including the limits of 1947 MBA, Art. EDCA, Art. III(1):
territorial waters and air space adjacent to or in the vicinity of VI:
those bases. The only standard used in determining the extent With consideration of the views of
of its control was military necessity. On the other hand, there The United the Parties,
is no such grant of power or authority under EDCA. It merely States shall, the Philippines hereby authorizes and
allows the U.S. to exercise operational control over the subject to agrees that United States forces, United
construction of Philippine-owned structures and facilities: previous States contractors, and vehicles,
agreement with vessels, and aircraft operated by or for
the Philippines, United States forces may conduct the
1947 MBA EDCA have the right to following activities with respect to
use land and Agreed Locations: training; transit;
1947 MBA, EDCA, Art. III(4): coastal sea support and related activities; refueling
Art.I(2):
areas of of aircraft; bunkering of vessels;
The Philippines hereby grants to the appropriate size temporary maintenance of vehicles,
The Philippines United States, through bilateral and location for vessels, and aircraft; temporary
agrees to permit security mechanisms, such as the periodic accommodation of personnel;
the United MDB and SEB, operational maneuvers, for communications; prepositioning of
States, upon control of Agreed additional equipment, supplies, and materiel;
notice to the Locations for construction staging areas, deploying forces and materiel; and such
Philippines, to use activities and authority to undertake bombing and other activities as the Parties may
such of those such activities on, and make gunnery agree.
bases listed in alterations and improvements to, ranges, and for
Annex B as the Agreed Locations. United States such
United States forces shall consult on issues intermediate
determines to be regarding such construction, airfields as may
required by alterations, and be required for
military improvements based on the Parties' safe and efficient
necessity. shared intent that the technical air operations.
requirements and construction Operations in
standards of any such projects such areas shall
1947 MBA, Art.
III(1): undertaken by or on behalf of United be carried on
States forces should be consistent with with due regard
the requirements and standards of and safeguards
It is mutually both Parties. for the public
agreed that safety.
the United
Statesshall have
the rights, power 1947 MBA,
and authority Art.I(2):
within the
bases which The Philippines
are necessary for agrees to permit
the the United
establishment, States, upon
use, operation notice to the
and defense Philippines,
thereof or to use such of
appropriate for those
the control bases listed in
thereof and all the Annex B as the
rights, power and United States
authority within determines to
the limits of be required by
territorial waters military
and air space necessity.
adjacent to, or in
FINALS CONSTITUTION I ACJUCO 292

Sixth, under the MBA, the U.S. was given the right, power, and
It is mutually agreed that When requested, the
authority to control and prohibit the movement and operation
the United States may Designated Authority of the
of all types of vehicles within the vicinity of the bases. The U.S.
employ and use for United Philippines shall assist
does not have any right, power, or authority to do so under
States military forces any in facilitating transit or
EDCA.
and all public utilities, temporary access by
other services and United States forces to
facilities, airfields, ports, public land and facilities
1947 MBA EDCA
harbors, roads, highways, (including roads, ports, and
1947 MBA, Art. 111(2)(c) No equivalent provision. railroads, bridges, viaducts, airfields), including those
canals, lakes, rivers and owned or controlled by local
streams in the governments, and to other
Such rights, power and Philippines under land and facilities (including
authority shall include, inter conditions no less roads, ports, and airfields).
alia, the right, power and favorable than those that
authority: x x x x to may be applicablefrom time
control (including the to time to the military
right to prohibit) in so far as forces of the Philippines.
may be required for the
efficient operation and safety
of the bases, and within the Ninth, under EDCA, the U.S. no longer has the right, power,
limits of military and authority to construct, install, maintain, and
necessity, anchorages, employ any type of facility, weapon, substance, device, vessel
moorings, landings, or vehicle, or system unlike in the old treaty. EDCA merely
takeoffs, movements and grants the U.S., through bilateral security mechanisms, the
operation of ships and authority to undertake construction, alteration, or
water-borne craft, aircraft improvements on the Philippine-owned Agreed Locations.
and other vehicles on
water, in the air or on land
comprising 1947 MBA EDCA

1947 MBA, Art. III(2)(e): EDCA, Art. III(4):


Seventh, under EDCA, the U.S. is merely given temporary
access to land and facilities (including roads, ports, and
Such rights, power and The Philippines hereby
airfields). On the other hand, the old treaty gave the U.S. the
authority shall include, inter grants to the United States,
right to improve and deepen the harbors, channels, entrances,
alia, the right, power and through bilateral security
and anchorages; and to construct or maintain necessary roads
authority: x x x x mechanisms, such as the
and bridges that would afford it access to its military bases.
to construct, install, MDB and SEB, operational
maintain, and employ on control of Agreed Locations
1947 MBA EDCA any base any for construction activities
type of facilities, weapons, and authority to undertake
1947 MBA, Art. III(2)(b): EDCA, Art. III(2): substance, device, vessel such activities on, and
or vehicle on or under the make alterations and
ground, in the air or on or improvements to, Agreed
Such rights, power and When requested, the under the water that may be Locations. United States
authority shall include, inter Designated Authority of the requisite or appropriate, forces shall consult on
alia, the right, power and Philippines shall assist including meteorological issues regarding such
authority: x x x x to in facilitating transit or systems, aerial and water construction, alterations,
improve and deepen the temporary access by navigation lights, radio and and improvements based on
harbors, channels, United States forces to radar apparatus and the Parties' shared intent
entrances and public land and facilities electronic devices, of any that the technical
anchorages, and to (including roads, ports, and desired power, type of requirements and
construct or maintain airfields), including those emission and frequency. construction standards of
necessary roadsand owned or controlled by local any such projects
bridges affording access to governments, and to other undertaken by or on behalf
the bases. land and facilities (including of United States forces
roads, ports, and airfields). should be consistent with the
requirements and standards
of both Parties.
Eighth, in the 1947 MBA, the U.S. was granted the automatic
right to use any and all public utilities, services and facilities,
airfields, ports, harbors, roads, highways, railroads, bridges, Tenth, EDCA does not allow the U.S. to acquire, by
viaducts, canals, lakes, rivers, and streams in the Philippines condemnation or expropriation proceedings, real property
in the same manner that Philippine military forces enjoyed that belonging to any private person. The old military bases
right. No such arrangement appears in EDCA. In fact, it merely agreement gave this right to the U.S. as seen below:
extends to U.S. forces temporary access to public land and
facilities when requested:
1947 MBA EDCA

1947 MBA EDCA 1947 MBA, Art. XXII(l): No equivalent provision.

1947 MBA, Art. VII: EDCA, Art. III(2):


FINALS CONSTITUTION I ACJUCO 293

Whenever it is necessary authorities. Such persons, the definition of United


to acquire by other than members of the States personnel in this
United States armed forces Agreement, including
condemnation or in uniform, shall present their within the context of the
expropriation proceedings travel documents to the VFA.
real property belonging to appropriate Philippine
authorities for visas, it being
any private
persons, associations or understood that no
objection will be made to
corporations located in
bases named in Annex A their travel to the
and Annex B in order to Philippines as non-
carry out the purposes of this immigrants.
Agreement, the Philippines
will institute and prosecute
Twelfth, EDCA does not allow the U.S. to exercise jurisdiction
such condemnation or
over any offense committed by any person within the Agreed
expropriation proceedings in
Locations, unlike in the former military bases:
accordance with the laws of
the Philippines. The United
States agrees to reimburse
1947 MBA EDCA
the Philippines for all the
reasonable expenses, 1947 MBA, Art. XIII(l)(a): No equivalent provision.
damages and costs therebv
incurred, including the value
of the property as The Philippines consents
determined by the Court. In that the United
addition, subject to the
mutual agreement of the two States shall have the right
Governments, the United to exercise
States will reimburse the jurisdiction over the
Philippines for the following offenses: (a) Any
reasonable costs of offense committed by any
transportation and removal person within any
of any occupants displaced base except where the
or ejected by reason of the offender and offended
condemnation or parties are both Philippine
expropriation. citizens (not members of the
armed forces of the United
States on active duty) or the
Eleventh, EDCA does not allow the U.S. to unilaterally bring offense is against the
into the country non-Philippine nationals who are under its security of the Philippines.
employ, together with their families, in connection with the
construction, maintenance, or operation of the bases. EDCA
strictly adheres to the limits under the VFA. Thirteenth, EDCA does not allow the U.S. to operate military
post exchange (PX) facilities, which is free of customs duties
and taxes, unlike what the expired MBA expressly allowed.
1947 MBA EDCA Parenthetically, the PX store has become the cultural icon of
U.S. military presence in the country.
1947 MBA, Art. XI(l): EDCA, Art. II:

It is mutually agreed that 1. "United States 1947 MBA EDCA


the United States shall personnel" means United
have the right to bring into States military and civilian 1947 MBA, Art. XVIII(l): No equivalent provision.
the Philippines members of personneltemporarily in the
the United States military territory of the Philippines in It is mutually agreed that
forces and the United connection with activities the United States
States nationals employed approved by the
by or under a contract with Philippines, as those terms
the United States together are defined in the VFA. shall have the right to
with their families, and establish on bases, free of
technical personnel of all licenses; fees; sales,
x xx x excise or other taxes, or
other nationalities (not
being persons excluded by imposts; Government
the laws of the Philippines) 3. "United States agencies, including
in connection with the contractors" means concessions, such
construction, maintenance, companies and firms, and as sales commissaries
or operation of the bases. their employees, under and post
The United States shall contract or subcontract to or exchanges; messes and
make suitable arrangements on behalf of the United social clubs, for the
so that such persons may be States Department of exclusive use of the
readily identified and their Defense. United States United States military
status established when contractors are not forces and authorized
necessary by the Philippine includedas part of civilian personnel and
FINALS CONSTITUTION I ACJUCO 294

750 hectares Radio Transmitter in Capas, Tarlac


their families. The
merchandise or services
sold or dispensed by such 900 hectares Radio Bigot Annex at Bamban,
agencies shall be free of all Tarlac329
taxes, duties and
inspection by the The Bases Conversion and Development Act of 1992
Philippine described its coverage in its Declaration of Policies:
authorities. Administrative
measures shall be taken by
the appropriate authorities of Sec. 2. Declaration of Policies. - It is hereby declared the policy
the United States to prevent of the Government to accelerate the sound and balanced
the resale of goods which conversion into alternative productive uses of the Clark and
are sold under the provisions Subic military reservations and their extensions (John Hay
of this Article to persons not Station, Wallace Air Station, O'Donnell Transmitter Station,
entitled to buy goods at such San Miguel Naval Communications Station and Capas Relay
agencies and, generally, to Station), to raise funds by the sale of portions of Metro Manila
prevent abuse of the military camps, and to apply said funds as provided herein for
privileges granted under this the development and conversion to productive civilian use of
Article. There shall be the lands covered under the 194 7 Military Bases Agreement
cooperation between such between the Philippines and the United States of America, as
authorities and the amended.330
Philippines to this end.
The result of the debates and subsequent voting is Section 25,
Article XVIII of the Constitution, which specifically restricts,
In sum, EDCA is a far cry from a basing agreement as was among others, foreign military facilities or bases. At the time of
understood by the people at the time that the 1987 Constitution its crafting of the Constitution, the 1986 Constitutional
was adopted. Commission had a clear idea of what exactly it was restricting.
While the term "facilities and bases" was left undefined, its
Nevertheless, a comprehensive review of what the point of reference was clearly those areas covered by the 1947
Constitution means by "foreign military bases" and "facilities" MBA as amended.
is required before EDCA can be deemed to have passed
judicial scrutiny. Notably, nearly 30 years have passed since then, and the ever-
evolving world of military technology and geopolitics has
c. The meaning of military facilities and bases surpassed the understanding of the Philippine people in 1986.
The last direct military action of the U.S. in the region was the
use of Subic base as the staging ground for Desert Shield and
An appreciation of what a military base is, as understood by Desert Storm during the Gulf War.331In 1991, the Philippine
the Filipino people in 1987, would be vital in determining Senate rejected the successor treaty of the 1947 MBA that
whether EDCA breached the constitutional restriction. would have allowed the continuation of U.S. bases in the
Philippines.
Prior to the drafting of the 1987 Constitution, the last definition
of "military base" was provided under Presidential Decree No. Henceforth, any proposed entry of U.S. forces into the
(PD) 1227.328 Unlawful entry into a military base is punishable Philippines had to evolve likewise, taking into consideration the
under the decree as supported by Article 281 of the Revised subsisting agreements between both parties, the rejection of
Penal Code, which itself prohibits the act of trespass. the 1991 proposal, and a concrete understanding of what was
constitutionally restricted. This trend birthed the VFA which, as
Section 2 of the law defines the term in this manner: "'[M]ilitary discussed, has already been upheld by this Court.
base' as used in this decree means any military, air, naval, or
coast guard reservation, base, fort, camp, arsenal, yard, The latest agreement is EDCA, which proposes a novel
station, or installation in the Philippines." concept termed "Agreed Locations."

Commissioner Tadeo, in presenting his objections to U.S. By definition, Agreed Locations are
presence in the Philippines before the 1986 Constitutional
Commission, listed the areas that he considered as military
bases: facilities and areas that are provided by the Government of the
Philippines through the AFP and that United States forces,
United States contractors, and others as mutually agreed, shall
1,000 hectares Camp O'Donnel have the right to access and use pursuant to this Agreement.
Such Agreed Locations may be listed in an annex to be
20,000 hectares Crow Valley Weapon's Range appended to this Agreement, and may be further described in
implementing arrangements.332
55,000 hectares Clark Air Base
Preliminarily, respondent already claims that the proviso that
the Philippines shall retain ownership of and title to the Agreed
150 hectares Wallace Air Station
Locations means that EDCA is "consistent with Article II of the
VFA which recognizes Philippine sovereignty and jurisdiction
400 hectares John Hay Air Station over locations within Philippine territory.333

15,000 hectares Subic Naval Base By this interpretation, respondent acknowledges that the
contention of petitioners springs from an understanding that
1,000 hectares San Miguel Naval Communication the Agreed Locations merely circumvent the constitutional
FINALS CONSTITUTION I ACJUCO 295

restrictions. Framed differently, the bone of contention is just mulling the prospects of these varying contentions are now
whether the Agreed Locations are, from a legal perspective, expected, like armchair generals, to decide not only on the
foreign military facilities or bases. This legal framework triggers geopolitical aspects and contingent implications of the military
Section 25, Article XVIII, and makes Senate concurrence bases but also on their political, social, economic and cultural
a sine qua non. impact on our national life. We are asked to answer a plethora
of questions, such as: 1) whether the bases are magnets of
Article III of EDCA provides for Agreed Locations, in which the nuclear attack or are deterrents to such attack; 2) whether an
U.S. is authorized by the Philippines to "conduct the following alliance or mutual defense treaty is a derogation of our national
activities: "training; transit; support and related activities; sovereignty; 3) whether criticism of us by Russia, Vietnam and
refueling of aircraft; bunkering of vessels; temporary North Korea is outweighed by the support for us of the ASEAN
maintenance of vehicles, vessels and aircraft; temporary countries, the United States, South Korea, Taiwan, Australia
accommodation of personnel; communications; prepositioning and New Zealand; and 4) whether the social, moral and legal
problems spawned by the military bases and their operations
of equipment, supplies and materiel; deploying forces and
materiel; and such other activities as the Parties may agree." can be compensated by the economic benefits outlined in
papers which have been furnished recently to all of us.335

This creation of EDCA must then be tested against a proper


xxxx
interpretation of the Section 25 restriction.

d. Reasons for the constitutional requirements and legal Of course, one side of persuasion has submitted categorical,
standards for constitutionally compatible military bases and unequivocal and forceful assertions of their positions. They are
entitled to the luxury of the absolutes. We are urged now to
facilities
adopt the proposed declaration as a "golden," "unique"
and "last" opportunity for Filipinos to assert their
Section 25 does not define what is meant by a "foreign military sovereign rights. Unfortunately, I have never been enchanted
facility or base." While it specifically alludes to U.S. military by superlatives, much less for the applause of the moment or
facilities and bases that existed during the framing of the the ovation of the hour. Nor do I look forward to any glorious
Constitution, the provision was clearly meant to apply to those summer after a winter of political discontent. Hence, if I may
bases existing at the time and to any future facility or base. The join Commissioner Laurel, I also invoke a caveat not only
basis for the restriction must first be deduced from the spirit of against the tyranny of labels but also the tyranny of slogans. 336
the law, in order to set a standard for the application of its text,
given the particular historical events preceding the agreement.
xxxx

Once more, we must look to the 1986 Constitutional


SPEECH OF COMMISSIONER SUAREZ337
Commissioners to glean, from their collective wisdom, the
intent of Section 25. Their speeches are rich with history and
wisdom and present a clear picture of what they considered in MR. SUAREZ: Thank you, Madam President.
the crafting the provision.
I am quite satisfied that the crucial issues involved in the
SPEECH OF COMMISSIONER REGALADO334 resolution of the problem of the removal of foreign bases from
the Philippines have been adequately treated by previous
xxxx speakers. Let me, therefore, just recapitulate the arguments
adduced in favor of a foreign bases-free Philippines:

We have been regaled here by those who favor the adoption


1. That every nation should be free to shape its own
of the anti-bases provisions with what purports to be an
objective presentation of the historical background of the destiny without outside interference;
military bases in the Philippines. Care appears, however, to
have been taken to underscore the inequity in their inception 2. That no lasting peace and no true
as well as their implementation, as to seriously reflect on the sovereignty would ever be achieved so long as
supposed objectivity of the report. Pronouncements of military there are foreign military forces in our country;
and civilian officials shortly after World War II are quoted in
support of the proposition on neutrality; regrettably, the 3. That the presence of foreign military bases
implication is that the same remains valid today, as if the world
deprives us of the very substance of national
and international activity stood still for the last 40 years. sovereigntyand this is a constant source of national
embarrassment and an insult to our national dignity
We have been given inspired lectures on the effect of the and selfrespect as a nation;
presence of the military bases on our sovereignty -
whether in its legal or political sense is not clear - and the 4. That these foreign military bases unnecessarily
theory that any country with foreign bases in its territory expose our country to devastating nuclear
cannot claim to be fully sovereign or completely attacks;
independent. I was not aware that the concepts of sovereignty
and independence have now assumed the totality principle,
such that a willing assumption of some delimitations in the 5. That these foreign military bases create social
exercise of some aspects thereof would put that State in a problems and are designed to perpetuate the
lower bracket of nationhood. strangle-hold of United States interests in our
national economy and development;
xxxx
6. That the extraterritorial rights enjoyed by these
foreign bases operate to deprive our country of
We have been receiving a continuous influx of materials on the jurisdiction over civil and criminal
pros and cons on the advisability of having military bases within offenses committed within our own national territory
our shores. Most of us who, only about three months ago, were and against Filipinos;
FINALS CONSTITUTION I ACJUCO 296

7. That the bases agreements are colonial bases in Spain, Egypt and Israel. In doing so, these countries,
impositions and dictations upon our helpless in effect, contributed to the launching of a preventive defense
country; and posture against possible trouble in the Middle East and in the
Indian Ocean for their own protection. 345
8. That on the legal viewpoint and in the ultimate
analysis, all the bases agreements are null and SPEECH OF COMMISSIONER TINGSON346
void ab initio, especially because they did not count
the sovereign consent and will of the Filipino
xxxx
people.338

In the case of the Philippines and the other Southeast Asian


xxxx nations, the presence of American troops in the country is a
projection of America's security interest. Enrile said that
In the real sense, Madam President, if we in the Commission nonetheless, they also serve, although in an incidental and
could accommodate the provisions I have cited, what is our secondary way, the security interest of the Republic of the
objection to include in our Constitution a matter as priceless as Philippines and the region. Yes, of course, Mr. Enrile also
the nationalist values we cherish? A matter of the gravest echoes the sentiments of most of us in this Commission,
concern for the safety and survival of this nation indeed namely: It is ideal for us as an independent and sovereign
deserves a place in our Constitution. nation to ultimately abrogate the RP-US military treaty
and, at the right time, build our own air and naval might.347
xxxx
xxxx
x x x Why should we bargain away our dignity and our self-
respect as a nation and the future of generations to come with Allow me to say in summation that I am for the retention
thirty pieces of silver?339 of American military bases in the Philippines provided that
such an extension from one period to another shall be
concluded upon concurrence of the parties, and such
SPEECH OF COMMISSIONER BENNAGEN340
extension shall be based on justice, the historical amity of
the people of the Philippines and the United States and
xxxx their common defense interest.348

The underlying principle of military bases and nuclear SPEECH OF COMMISSIONER ALONTO349
weapons wherever they are found and whoever owns them is
that those are for killing people or for terrorizing
humanity. This objective by itself at any point in history is xxxx
morally repugnant. This alone is reason enough for us to
constitutionalize the ban on foreign military bases and on Madam President, sometime ago after this Commission
nuclear weapons.341 started with this task of framing a constitution, I read a
statement of President Aquino to the effect that she is for the
removal of the U.S. military bases in this country but that the
SPEECH OF COMMISSIONER BACANI342
removal of the U.S. military bases should not be done just to
give way to other foreign bases. Today, there are two world
xxxx superpowers, both vying to control any and all countries which
have importance to their strategy for world domination. The
x x x Hence, the remedy to prostitution does not seem to Philippines is one such country.
be primarily to remove the bases because even if the bases
are removed, the girls mired in poverty will look for their Madam President, I submit that I am one of those ready to
clientele elsewhere. The remedy to the problem of prostitution completely remove any vestiges of the days of
lies primarily elsewhere - in an alert and concerned citizenry, a enslavement, but not prepared to erase them if to do so would
healthy economy and a sound education in values.343 merely leave a vacuum to be occupied by a far worse type. 350

SPEECH OF COMMISSIONER JAMIR344 SPEECH OF COMMISSIONER GASCON351

xxxx xxxx

One of the reasons advanced against the maintenance of Let us consider the situation of peace in our world today.
foreign military bases here is that they impair portions of Consider our brethren in the Middle East, in Indo-China,
our sovereignty. While I agree that our country's sovereignty Central America, in South Africa - there has been escalation of
should not be impaired, I also hold the view that there are times war in some of these areas because of foreign intervention
when it is necessary to do so according to the imperatives of which views these conflicts through the narrow prism of the
national interest. There are precedents to this effect. Thus, East-West conflict. The United States bases have been
during World War II, England leased its bases in the West used as springboards for intervention in some of these
Indies and in Bermuda for 99 years to the United States for its conflicts. We should not allow ourselves to be party to the
use as naval and air bases. It was done in consideration of 50 warlike mentality of these foreign interventionists. We
overaged destroyers which the United States gave to England must always be on the side of peace – this means that we
for its use in the Battle of the Atlantic. should not always rely on military solution. 352

A few years ago, England gave the Island of Diego Garcia to xxxx
the United States for the latter's use as a naval base in the
Indian Ocean. About the same time, the United States obtained
FINALS CONSTITUTION I ACJUCO 297

x x x The United States bases, therefore, are springboards Mr. Presiding Officer, I feel that banning foreign military bases
for intervention in our own internal affairs and in the is one of the solutions and is the response of the Filipino people
affairs of other nations in this region. against this condition and other conditions that have already
been clearly and emphatically discussed in past deliberations.
xxxx The deletion, therefore, of Section 3 in the Constitution we are
drafting will have the following implications:

Thus, I firmly believe that a self-respecting nation should


First, the failure of the Constitutional Commission to decisively
safeguard its fundamental freedoms which should logically be
declared in black and white in our fundamental law of the land respond to the continuing violation of our territorial
- the Constitution. Let us express our desire for national integrity via the military bases agreement which permits
sovereignty so we may be able to achieve national self- the retention of U.S. facilities within the Philippine soil
determination. Let us express our desire for neutrality so that over which our authorities have no exclusive jurisdiction
we may be able to follow active nonaligned independent contrary to the accepted definition of the exercise of
foreign policies. Let us express our desire for peace and a sovereignty.
nuclear-free zone so we may be able to pursue a healthy and
tranquil existence, to have peace that is autonomous and not Second, consent by this forum, this Constitutional
imposed. 353 Commission, to an exception in the application of a
provision in the Bill of Rights that we have just drafted
xxxx regarding equal application of the laws of the land to all
inhabitants, permanent or otherwise, within its territorial
boundaries.
SPEECH OF COMMISSIONER TADEO354
Third, the continued exercise by the United States of
Para sa magbubukid, ano ha ang kahulugan ng U.S. extraterritoriality despite the condemnations of such practice
military bases? Para sa magbubukid, ang kahulugan nito by the world community of nations in the light of overwhelming
ay pagkaalipin. Para sa magbubukid, ang pananatili ng U.S. international approval of eradicating all vestiges of
military bases ay tinik sa dibdib ng sambayanang colonialism.358
Pilipinong patuloy na nakabaon. Para sa sambayanang
magbubukid, ang ibig sabihin ng U.S. military
bases ay batong pabigat na patuloy na pinapasan ng xxxx
sambayanang Pilipino. Para sa sambayanang magbubukid,
ang pananatili ng U.S. military bases ay isang nagdudumilat Sixth, the deification of a new concept called pragmatic
na katotohanan ng patuloy na paggahasa ng sovereignty, in the hope that such can be wielded to force the
imperyalistang Estados Unidos sa ating Inang Bayan - United States government to concede to better terms and
economically, politically and culturally. Para sa conditions concerning the military bases agreement, including
sambayanang magbubukid ang U.S. military the transfer of complete control to the Philippine
bases ay kasingkahulugan ng nuclear weapon - ang government of the U.S. facilities, while in the meantime we
kahulugan ay magneto ng isang nuclear war. Para sa have to suffer all existing indignities and disrespect towards
sambayanang magbubukid, ang kahulugan ng U.S. military our rights as a sovereign nation.
bases ay isang salot.355
xxxx
SPEECH OF COMMISSIONER QUESADA356
Eighth, the utter failure of this forum to view the issue of
xxxx foreign military bases as essentially a question of
sovereignty which does not require in-depth studies or
analyses and which this forum has, as a constituent assembly
The drift in the voting on issues related to freeing ourselves
from the instruments of domination and subservience has drafting a constitution, the expertise and capacity to decide on
clearly been defined these past weeks. except that it lacks the political will that brought it to existence
and now engages in an elaborate scheme of buck-passing.

xxxx
xxxx

So for the record, Mr. Presiding Officer, I would like to declare


my support for the committee's position to enshrine in the Without any doubt we can establish a new social order in our
country, if we reclaim, restore, uphold and defend our national
Constitution a fundamental principle forbidding foreign military
bases, troops or facilities in any part of the Philippine territory sovereignty. National sovereignty is what the military
bases issue is all about. It is only the sovereign people
as a clear and concrete manifestation of our inherent right
to national self-determination, independence and exercising their national sovereignty who can design an
sovereignty. independent course and take full control of their national
destiny.359

Mr. Presiding Officer, I would like to relate now these attributes


of genuine nationhood to the social cost of allowing foreign SPEECH OF COMMISSIONER P ADILLA360
countries to maintain military bases in our country. Previous
speakers have dwelt on this subject, either to highlight its xxxx
importance in relation to the other issues or to gloss over its
significance and !llake this a part of future negotiations. 357 Mr. Presiding Officer, in advocating the majority committee
report, specifically Sections 3 and 4 on neutrality, nuclear and
xxxx bases-free country, some views stress sovereignty of the
Republic and even invoke survival of the Filipino nation
and people.361
FINALS CONSTITUTION I ACJUCO 298

REBUTTAL OF COMMISSIONER NOLLEDO362 In this case, EDCA explicitly provides that ownership of the
Agreed Locations remains with the Philippine
xxxx govemment.368 What U.S. personnel have a right to, pending
mutual agreement, is access to and use of these locations.369

The anachronistic and ephemeral arguments against the


The right of the owner of the property to allow access and use
provisions of the committee report to dismantle the American
bases after 1991 only show the urgent need to free our is consistent with the Civil Code, since the owner may dispose
of the property in whatever way deemed fit, subject to the limits
country from the entangling alliance with any power bloc.363
of the law. So long as the right of ownership itself is not
transferred, then whatever rights are transmitted by agreement
xxxx does not completely divest the owner of the rights over the
property, but may only limit them in accordance with law.
xx x Mr. Presiding Officer, it is not necessary for us to possess
expertise to know that the so-called RP-US Bases Agreement Hence, even control over the property is something that an
will expire in 1991, that it infringes on our sovereignty and owner may transmit freely. This act does not translate into the
jurisdiction as well as national dignity and honor, that it goes full transfer of ownership, but only of certain rights. In Roman
against the UN policy of disarmament and that it Catholic Apostolic Administrator of Davao, Inc. v. Land
constitutes unjust intervention in our internal Registration Commission, we stated that the constitutional
affairs.364 (Emphases Supplied) proscription on property ownership is not violated despite the
foreign national's control over the property. 370
The Constitutional Commission eventually agreed to allow
foreign military bases, troops, or facilities, subject to the EDCA, in respect of its provisions on Agreed Locations, is
provisions of Section 25. It is thus important to read its essentially a contract of use and access. Under its pertinent
discussions carefully. From these discussions, we can deduce provisions, it is the Designated Authority of the Philippines that
three legal standards that were articulated by the shall, when requested, assist in facilitating transit or access to
Constitutional Commission Members. These are public land and facilities.371 The activities carried out within
characteristics of any agreement that the country, and by these locations are subject to agreement as authorized by the
extension this Court, must ensure are observed. We can Philippine govemment.372 Granting the U.S. operational control
thereby determine whether a military base or facility in the over these locations is likewise subject to EDCA' s security
Philippines, which houses or is accessed by foreign military mechanisms, which are bilateral procedures involving
troops, is foreign or remains a Philippine military base or Philippine consent and cooperation. 373 Finally, the Philippine
facility. The legal standards we find applicable are: Designated Authority or a duly designated representative is
independence from foreign control, sovereignty and applicable given access to the Agreed Locations.374
law, and national security and territorial integrity.
To our mind, these provisions do not raise the spectre of U.S.
i. First standard: independence from foreign control control, which was so feared by the Constitutional
Commission. In fact, they seem to have been the product of
Very clearly, much of the opposition to the U.S. bases at the deliberate negotiation from the point of view of the Philippine
time of the Constitution's drafting was aimed at asserting government, which balanced constitutional restrictions on
Philippine independence from the U.S., as well as control over foreign military bases and facilities against the security needs
our country's territory and military. of the country. In the 1947 MBA, the U.S. forces had "the right,
power and authority x x x to construct (including dredging and
filling), operate, maintain, utilize, occupy, garrison and control
Under the Civil Code, there are several aspects of control the bases."375 No similarly explicit provision is present in
exercised over property. EDCA.

Property is classified as private or public. 365 It is public if Nevertheless, the threshold for allowing the presence of
"intended for public use, such as roads, canals, rivers, torrents, foreign military facilities and bases has been raised by the
ports and bridges constructed by the State, banks, shores, present Constitution. Section 25 is explicit that foreign military
roadsteads, and others of similar character[,]" or "[t]hose which bases, troops, or facilities shall not be allowed in the
belong to the State, without being for public use, and are Philippines, except under a treaty duly concurred in by the
intended for some public service or for the development of the Senate. Merely stating that the Philippines would retain
national wealth. "366 ownership would do violence to the constitutional requirement
if the Agreed Locations were simply to become a less obvious
Quite clearly, the Agreed Locations are contained within a manifestation of the U.S. bases that were rejected in 1991.
property for public use, be it within a government military camp
or property that belongs to the Philippines.1avvphi1 When debates took place over the military provisions of the
Constitution, the committee rejected a specific provision
Once ownership is established, then the rights of ownership proposed by Commissioner Sarmiento. The discussion
flow freely. Article 428 of the Civil Code provides that "[t]he illuminates and provides context to the 1986 Constitutional
owner has the right to enjoy and dispose of a thing, without Commission's vision of control and independence from the
other limitations than those established by law." Moreover, the U.S., to wit:
owner "has also a right of action against the holder and
possessor of the thing in order to recover it." MR. SARMIENTO: Madam President, my proposed
amendment reads as follows: "THE STATE SHALL
Philippine civil law therefore accords very strong rights to the ESTABLISH AND MAINTAIN AN INDEPENDENT AND SELF-
owner of property, even against those who hold the property. RELIANT ARMED FORCES OF THE PHILIPPINES." Allow
Possession, after all, merely raises a disputable presumption me to briefly explain, Madam President. The Armed Forces of
of ownership, which can be contested through normal judicial the Philippines is a vital component of Philippine society
processes.367 depending upon its training, orientation and support. It will
either be the people's protector or a staunch supporter of a
FINALS CONSTITUTION I ACJUCO 299

usurper or tyrant, local and foreign interest. The Armed foreign officers in our schools, we in the Command and
Forces of the Philippines' past and recent experience General Staff College in Fort Bonifacio and in our National
shows it has never been independent and self- Defense College, also in Fort Bonifacio. 377 (Emphases
reliant. Facts, data and statistics will show that it has been supplied)
substantially dependent upon a foreign power. In March 1968,
Congressman Barbero, himself a member of the Armed Forces This logic was accepted in Tañada v. Angara, in which the
of the Philippines, revealed top secret documents showing Court ruled that independence does not mean the absence of
what he described as U.S. dictation over the affairs of the foreign participation:
Armed Forces of the Philippines. He showed that under
existing arrangements, the United States unilaterally
determines not only the types and quantity of arms and Furthermore, the constitutional policy of a "self-reliant and
equipments that our armed forces would have, but also independent national economy" does not necessarily rule
the time when these items are to be made available to us. out the entry of foreign investments, goods and services.
It is clear, as he pointed out, that the composition, It contemplates neither "economic seclusion" nor "mendicancy
capability and schedule of development of the Armed in the international community." As explained by Constitutional
Forces of the Philippines is under the effective control of Commissioner Bernardo Villegas, sponsor of this constitutional
the U.S. government.376 (Emphases supplied) policy:

Commissioner Sarmiento proposed a motherhood statement Economic self reliance is a primary objective of a developing
in the 1987 Constitution that would assert "independent" and country that is keenly aware of overdependence on external
"self-reliant" armed forces. This proposal was rejected by assistance for even its most basic needs. It does not mean
the committee, however. As Commissioner De Castro autarky or economic seclusion; rather, it means avoiding
asserted, the involvement of the Philippine military with mendicancy in the international community. Independence
the U.S. did not, by itself, rob the Philippines of its real refers to the freedom from undue foreign control of the
independence. He made reference to the context of the times: national economy, especially in such strategic industries as in
that the limited resources of the Philippines and the current the development of natural resources and public
insurgency at that time necessitated a strong military utilities.378 (Emphases supplied)
relationship with the U.S. He said that the U.S. would not in
any way control the Philippine military despite this relationship The heart of the constitutional restriction on foreign military
and the fact that the former would furnish military hardware or facilities and bases is therefore the assertion of independence
extend military assistance and training to our military. Rather, from the U.S. and other foreign powers, as independence is
he claimed that the proposal was in compliance with the exhibited by the degree of foreign control exerted over these
treaties between the two states. areas.1âwphi1 The essence of that independence is self-
governance and self-control.379 Independence itself is "[t]he
MR. DE CASTRO: If the Commissioner will take note of my state or condition of being free from dependence, subjection,
speech on U.S. military bases on 12 September 1986, I spoke or control. "380
on the selfreliance policy of the armed forces. However, due to
very limited resources, the only thing we could do is Petitioners assert that EDCA provides the U.S. extensive
manufacture small arms ammunition. We cannot blame the control and authority over Philippine facilities and locations,
armed forces. We have to blame the whole Republic of the such that the agreement effectively violates Section 25 of the
Philippines for failure to provide the necessary funds to make 1987 Constitution.381
the Philippine Armed Forces self-reliant. Indeed that is a
beautiful dream. And I would like it that way. But as of this time,
fighting an insurgency case, a rebellion in our country - Under Article VI(3) of EDCA, U.S. forces are authorized to act
insurgency - and with very limited funds and very limited as necessary for "operational control and defense." The term
number of men, it will be quite impossible for the Philippines to "operational control" has led petitioners to regard U.S. control
appropriate the necessary funds therefor. However, if we say over the Agreed Locations as unqualified and, therefore,
that the U.S. government is furnishing us the military total.382 Petitioners contend that the word "their" refers to the
hardware, it is not control of our armed forces or of our subject "Agreed Locations."
government. It is in compliance with the Mutual Defense
Treaty. It is under the military assistance program that it This argument misreads the text, which is quoted below:
becomes the responsibility of the United States to furnish us
the necessary hardware in connection with the military bases
agreement. Please be informed that there are three (3) treaties United States forces are authorized to exercise all rights and
connected with the military bases agreement; namely: the RP- authorities within Agreed Locations that are necessary for their
US Military Bases Agreement, the Mutual Defense Treaty and operational control or defense, including taking appropriate
the Military Assistance Program. measure to protect United States forces and United States
contractors. The United States should coordinate such
measures with appropriate authorities of the Philippines.
My dear Commissioner, when we enter into a treaty and
we are furnished the military hardware pursuant to that
treaty, it is not in control of our armed forces nor control A basic textual construction would show that the word "their,"
of our government. True indeed, we have military officers as understood above, is a possessive pronoun for the subject
trained in the U.S. armed forces school. This is part of our "they," a third-person personal pronoun in plural form. Thus,
Military Assistance Program, but it does not mean that the "their" cannot be used for a non-personal subject such as
minds of our military officers are for the U.S. government, no. I "Agreed Locations." The simple grammatical conclusion is that
am one of those who took four courses in the United States "their" refers to the previous third-person plural noun, which is
schools, but I assure you, my mind is for the Filipino people. "United States forces." This conclusion is in line with the
Also, while we are sending military officers to train or to study definition of operational control.
in U.S. military schools, we are also sending our officers to
study in other military schools such as in Australia, England a. U.S. operational control as the exercise of authority over
and in Paris. So, it does not mean that when we send military U.S. personnel, and not over the Agreed Locations
officers to United States schools or to other military schools,
we will be under the control of that country. We also have
FINALS CONSTITUTION I ACJUCO 300

Operational control, as cited by both petitioner and properly designated commander over assigned and attached
respondents, is a military term referring to forces in the accomplishment of the mission x x
x."393 Operational control, on the other hand, refers to "[t]hose
functions of command over assigned forces involving the
[t]he authority to perform those functions of command over
subordinate forces involving organizing and employing composition of subordinate forces, the assignment of tasks,
commands and forces, assigning tasks, designating objective, the designation of objectives, the overall control of assigned
and giving authoritative direction necessary to accomplish the resources, and the full authoritative direction necessary to
mission.383 accomplish the mission."394

At times, though, operational control can mean something Two things demonstrate the errors in petitioners' line of
slightly different. In JUSMAG Philippines v. National Labor argument.
Relations Commission, the Memorandum of Agreement
between the AFP and JUSMAG Philippines defined the term Firstly, the phrase "consistent with operational safety and
as follows:384 security requirements in accordance with agreed procedures
developed by the Parties" does not add any qualification
The term "Operational Control" includes, but is not limited to, beyond that which is already imposed by existing treaties. To
all personnel administrative actions, such as: hiring recall, EDCA is based upon prior treaties, namely the VFA and
recommendations; firing recommendations; position the MDT.395 Treaties are in themselves contracts from which
classification; discipline; nomination and approval of incentive rights and obligations may be claimed or waived. 396 In this
awards; and payroll computation. particular case, the Philippines has already agreed to abide by
the security mechanisms that have long been in place between
the U.S. and the Philippines based on the implementation of
Clearly, traditional standards define "operational control" as their treaty relations.397
personnel control. Philippine law, for instance, deems
operational control as one exercised by police officers and
Secondly, the full document cited by petitioners contradicts the
civilian authorities over their subordinates and is distinct from
the administrative control that they also exercise over police equation of "operational control" with "effective command and
control," since it defines the terms quite differently, viz:398
subordinates.385 Similarly, a municipal mayor exercises
operational control over the police within the municipal
government,386 just as city mayor possesses the same power Command and control encompasses the exercise of authority,
over the police within the city government.387 responsibility, and direction by a commander over assigned
and attached forces to accomplish the mission. Command at
Thus, the legal concept of operational control involves all levels is the art of motivating and directing people and
organizations into action to accomplish missions. Control is
authority over personnel in a commander-subordinate
relationship and does not include control over the Agreed inherent in command. To control is to manage and direct forces
Locations in this particular case. Though not necessarily stated and functions consistent with a commander's command
in EDCA provisions, this interpretation is readily implied by the authority. Control of forces and functions helps commanders
reference to the taking of "appropriate measures to protect and staffs compute requirements, allocate means, and
United States forces and United States contractors." integrate efforts. Mission command is the preferred method of
exercising C2. A complete discussion of tenets, organization,
and processes for effective C2 is provided in Section B,
It is but logical, even necessary, for the U.S. to have "Command and Control of Joint Forces," of Chapter V "Joint
operational control over its own forces, in much the same way Command and Control."
that the Philippines exercises operational control over its own
units.
Operational control is defined thus:399

For actual operations, EDCA is clear that any activity must be


planned and pre-approved by the MDB-SEB.388 This provision OPCON is able to be delegated from a lesser authority than
evinces the partnership aspect of EDCA, such that both COCOM. It is the authority to perform those functions of
stakeholders have a say on how its provisions should be put command over subordinate forces involving organizing and
into effect. employing commands and forces, assigning tasks, designating
objectives, and giving authoritative direction over all aspects of
military operations and joint training necessary to accomplish
b. Operational control vis-à-vis effective command and control the mission. It should be delegated to and exercised by the
commanders of subordinate organizations; normally, this
Petitioners assert that beyond the concept of operational authority is exercised through subordinate JFCs, Service,
control over personnel, qualifying access to the Agreed and/or functional component commanders. OPCON provides
Locations by the Philippine Designated Authority with the authority to organize and employ commands and forces as the
phrase "consistent with operational safety and security commander considers necessary to accomplish assigned
requirements in accordance with agreed procedures missions. It does not include authoritative direction for logistics
developed by the Parties" leads to the conclusion that the U.S. or matters of administration, discipline, internal organization, or
exercises effective control over the Agreed Locations. 389 They unit training. These elements of COCOM must be specifically
claim that if the Philippines exercises possession of and control delegated by the CCDR. OPCON does include the authority to
over a given area, its representative should not have to be delineate functional responsibilities and operational areas of
authorized by a special provision.390 subordinate JFCs.

For these reasons, petitioners argue that the "operational Operational control is therefore the delegable aspect of
control" in EDCA is the "effective command and control" in the combatant command, while command and control is the
1947 MBA.391 In their Memorandum, they distinguish effective overall power and responsibility exercised by the commander
command and control from operational control in U.S. with reference to a mission. Operational control is a narrower
parlance.392 Citing the Doctrine for the Armed Forces of the power and must be given, while command and control is
United States, Joint Publication 1, "command and control (C2)" plenary and vested in a commander. Operational control does
is defined as "the exercise of authority and direction by a not include the planning, programming, budgeting, and
FINALS CONSTITUTION I ACJUCO 301

execution process input; the assignment of subordinate Sovereignty is the possession of sovereign power, 406 while
commanders; the building of relationships with Department of jurisdiction is the conferment by law of power and authority to
Defense agencies; or the directive authority for logistics, apply the law.407 Article I of the 1987 Constitution states:
whereas these factors are included in the concept of command
and control.400 The national territory comprises the Philippine archipelago,
with all the islands and waters embraced therein, and all other
This distinction, found in the same document cited by territories over which the Philippines has sovereignty or
petitioners, destroys the very foundation of the arguments they jurisdiction, consisting of its terrestrial, fluvial, and aerial
have built: that EDCA is the same as the MBA. domains, including its territorial sea, the seabed, the subsoil,
the insular shelves, and other submarine areas. The waters
c. Limited operational control over the Agreed Locations only around, between, and connecting the islands of the
for construction activitites archipelago, regardless of their breadth and dimensions, form
part of the internal waters of the Philippines. (Emphasis
supplied)
As petitioners assert, EDCA indeed contains a specific
provision that gives to the U.S. operational control within the
Agreed Locations during construction activities. 401 This From the text of EDCA itself, Agreed Locations are territories
exercise of operational control is premised upon the approval of the Philippines that the U.S. forces are allowed to access
by the MDB and the SEB of the construction activity through and use.408 By withholding ownership of these areas and
consultation and mutual agreement on the requirements and retaining unrestricted access to them, the government asserts
standards of the construction, alteration, or improvement. 402 sovereignty over its territory. That sovereignty exists so long
as the Filipino people exist.409

Despite this grant of operational control to the U.S., it must be


emphasized that the grant is only for construction activities. Significantly, the Philippines retains primary responsibility for
The narrow and limited instance wherein the U.S. is given security with respect to the Agreed Locations. 410Hence,
Philippine law remains in force therein, and it cannot be said
operational control within an Agreed Location cannot be
equated with foreign military control, which is so abhorred by that jurisdiction has been transferred to the U.S. Even the
previously discussed necessary measures for operational
the Constitution.
control and defense over U.S. forces must be coordinated with
Philippine authorities.411
The clear import of the provision is that in the absence of
construction activities, operational control over the Agreed
Jurisprudence bears out the fact that even under the former
Location is vested in the Philippine authorities. This meaning
is implicit in the specific grant of operational control only during legal regime of the MBA, Philippine laws continue to be in force
within the bases.412 The difference between then and now is
construction activities. The principle of constitutional
construction, "expressio unius est exclusio alterius," means that EDCA retains the primary jurisdiction of the Philippines
the failure to mention the thing becomes the ground for over the security of the Agreed Locations, an important
inferring that it was deliberately excluded. 403Following this provision that gives it actual control over those locations.
construction, since EDCA mentions the existence of U.S. Previously, it was the provost marshal of the U.S. who kept the
operational control over the Agreed Locations for construction peace and enforced Philippine law in the bases. In this
activities, then it is quite logical to conclude that it is not instance, Philippine forces act as peace officers, in stark
contrast to the 1947 MBA provisions on jurisdiction. 413
exercised over other activities.

Limited control does not violate the Constitution. The fear of iii. Third standard: must respect national security and territorial
the commissioners was total control, to the point that the integrity
foreign military forces might dictate the terms of their acts
within the Philippines.404 More important, limited control does The last standard this Court must set is that the EDCA
not mean an abdication or derogation of Philippine sovereignty provisions on the Agreed Locations must not impair or threaten
and legal jurisdiction over the Agreed Locations. It is more akin the national security and territorial integrity of the Philippines.
to the extension of diplomatic courtesies and rights to
diplomatic agents,405 which is a waiver of control on a limited This Court acknowledged in Bayan v. Zamora that the
scale and subject to the terms of the treaty.
evolution of technology has essentially rendered the prior
notion of permanent military bases obsolete.
This point leads us to the second standard envisioned by the
framers of the Constitution: that the Philippines must retain Moreover, military bases established within the territory of
sovereignty and jurisdiction over its territory.
another state is no longer viable because of the alternatives
offered by new means and weapons of warfare such as nuclear
ii. Second standard: Philippine sovereignty and applicable law weapons, guided missiles as well as huge sea vessels that can
stay afloat in the sea even for months and years without
returning to their home country. These military warships are
EDCA states in its Preamble the "understanding for the United
States not to establish a permanent military presence or base actually used as substitutes for a land-home base not only of
military aircraft but also of military personnel and facilities.
in the territory of the Philippines." Further on, it likewise states
the recognition that "all United States access to and use of Besides, vessels are mobile as compared to a land-based
facilities and areas will be at the invitation of the Philippines military headquarters.414
and with full respect for the Philippine Constitution and
Philippine laws." The VFA serves as the basis for the entry of U.S. troops in a
limited scope. It does not allow, for instance, the re-
The sensitivity of EDCA provisions to the laws of the establishment of the Subic military base or the Clark Air Field
Philippines must be seen in light of Philippine sovereignty and as U.S. military reservations. In this context, therefore, this
jurisdiction over the Agreed Locations. Court has interpreted the restrictions on foreign bases, troops,
or facilities as three independent restrictions. In accord with
FINALS CONSTITUTION I ACJUCO 302

this interpretation, each restriction must have its own Hence, any armed attack by forces of a third state against an
qualification. Agreed Location can only be legitimate under international
humanitarian law if it is against a bona fide U.S. military base,
facility, or installation that directly contributes to the military
Petitioners quote from the website http://en.wikipedia.org to
define what a military base is.415 While the source is not effort of the U.S. Moreover, the third state's forces must take
authoritative, petitioners make the point that the Agreed all measures to ensure that they have complied with the
Locations, by granting access and use to U.S. forces and principle of distinction (between combatants and non-
contractors, are U.S. bases under a different name. 416 More combatants).
important, they claim that the Agreed Locations invite
instances of attack on the Philippines from enemies of the There is, then, ample legal protection for the Philippines under
U.S.417 international law that would ensure its territorial integrity and
national security in the event an Agreed Location is subjected
We believe that the raised fear of an attack on the Philippines to attack. As EDCA stands, it does not create the situation so
is not in the realm of law, but of politics and policy. At the very feared by petitioners - one in which the Philippines, while not
least, we can say that under international law, EDCA does not participating in an armed conflict, would be legitimately
provide a legal basis for a justified attack on the Philippines. targeted by an enemy of the U.S.431

In the first place, international law disallows any attack on the In the second place, this is a policy question about the wisdom
Agreed Locations simply because of the presence of U.S. of allowing the presence of U.S. personnel within our territory
personnel. Article 2(4) of the United Nations Charter states that and is therefore outside the scope of judicial review.
"All Members shall refrain in their international relations from
the threat or use of force against the territorial integrity or Evidently, the concept of giving foreign troops access to
political independence of any state, or in any other manner "agreed" locations, areas, or facilities within the military base
inconsistent with the Purposes of the United Nations." 418 Any of another sovereign state is nothing new on the international
unlawful attack on the Philippines breaches the treaty, and plane. In fact, this arrangement has been used as the
triggers Article 51 of the same charter, which guarantees the framework for several defense cooperation agreements, such
inherent right of individual or collective self-defence. as in the following:

Moreover, even if the lawfulness of the attack were not in 1. 2006 U.S.-Bulgaria Defense Cooperation
question, international humanitarian law standards prevent Agreement432
participants in an armed conflict from targeting non-
participants. International humanitarian law, which is the 2. 2009 U.S.-Colombia Defense Cooperation
branch of international law applicable to armed conflict,
Agreement433
expressly limits allowable military conduct exhibited by forces
of a participant in an armed conflict.419 Under this legal regime,
participants to an armed conflict are held to specific standards 3. 2009 U.S.-Poland Status of Forces Agreement434
of conduct that require them to distinguish between
combatants and non-combatants,420 as embodied by the 4. 2014 U.S.-Australia Force Posture Agreement435
Geneva Conventions and their Additional Protocols. 421
5. 2014 U.S.-Afghanistan Security and Defense
Corollary to this point, Professor John Woodcliffe, professor of Cooperation Agreement436
international law at the University of Leicester, noted that there
is no legal consensus for what constitutes a base, as opposed
to other terms such as "facilities" or "installation." 422 In strategic In all of these arrangements, the host state grants U.S. forces
literature, "base" is defined as an installation "over which the access to their military bases.437 That access is without rental
user State has a right to exclusive control in an extraterritorial or similar costs to the U.S.438 Further, U.S. forces are allowed
sense."423 Since this definition would exclude most foreign to undertake construction activities in, and make alterations
military installations, a more important distinction must be and improvements to, the agreed locations, facilities, or
made. areas.439 As in EDCA, the host states retain ownership and
jurisdiction over the said bases.440
For Woodcliffe, a type of installation excluded from the
definition of "base" is one that does not fulfill a combat role. He In fact, some of the host states in these agreements give
cites an example of the use of the territory of a state for training specific military-related rights to the U.S. For example, under
purposes, such as to obtain experience in local geography and Article IV(l) of the US.-Bulgaria Defense Cooperation
climactic conditions or to carry out joint exercises.424 Another Agreement, "the United States forces x x x are authorized
example given is an advanced communications technology access to and may use agreed facilities and areas x x x for
installation for purposes of information gathering and staging and deploying of forces and materiel, with the purpose
communication.425 Unsurprisingly, he deems these non- of conducting x x x contingency operations and other missions,
combat uses as borderline situations that would be excluded including those undertaken in the framework of the North
from the functional understanding of military bases and Atlantic Treaty." In some of these agreements, host countries
installations.426 allow U.S. forces to construct facilities for the latter’s exclusive
use.441
By virtue of this ambiguity, the laws of war dictate that the
status of a building or person is presumed to be protected, Troop billeting, including construction of temporary structures,
unless proven otherwise.427 Moreover, the principle of is nothing new. In Lim v. Executive Secretary, the Court
distinction requires combatants in an armed conflict to already upheld the Terms of Reference of Balikatan 02-
distinguish between lawful targets 428 and protected 1, which authorized U.S. forces to set up "[t]emporary
targets.429 In an actual armed conflict between the U.S. and a structures such as those for troop billeting, classroom
third state, the Agreed Locations cannot be considered U.S. instruction and messing x x x during the Exercise." Similar
territory, since ownership of territory even in times of armed provisions are also in the Mutual Logistics Support Agreement
conflict does not change.430 of 2002 and 2007, which are essentially executive agreements
FINALS CONSTITUTION I ACJUCO 303

that implement the VFA, the MDT, and the 1953 Military response, the Philippines will be prepared alongside the U.S.
Assistance Agreement. These executive agreements similarly to defend its islands and insure its territorial integrity pursuant
tackle the "reciprocal provision of logistic support, supplies, to a relationship built on the MDT and VFA.
and services,"442 which include "[b ]illeting, x x x operations
support (and construction and use of temporary structures 8. Others issues and concerns raised
incident to operations support), training services, x x x storage
services, x x x during an approved activity." 443 These logistic
supplies, support, and services include temporary use of A point was raised during the oral arguments that the language
"nonlethal items of military equipment which are not of the MDT only refers to mutual help and defense in the Pacific
designated as significant military equipment on the U.S. area.453 We believe that any discussion of the activities to be
Munitions List, during an approved activity."444 The first Mutual undertaken under EDCA vis-a-vis the defense of areas beyond
Logistics Support Agreement has lapsed, while the second one the Pacific is premature. We note that a proper petition on that
has been extended until 2017 without any formal objection issue must be filed before we rule thereon. We also note that
before this Court from the Senate or any of its members. none of the petitions or memoranda has attempted to discuss
this issue, except only to theorize that the U.S. will not come to
our aid in the event of an attack outside of the Pacific. This is
The provisions in EDCA dealing with Agreed Locations are a matter of policy and is beyond the scope of this judicial
analogous to those in the aforementioned executive
review.
agreements. Instead of authorizing the building of temporary
structures as previous agreements have done, EDCA
authorizes the U.S. to build permanent structures or alter or In reference to the issue on telecommunications, suffice it to
improve existing ones for, and to be owned by, the say that the initial impression of the facility adverted to does
Philippines.445 EDCA is clear that the Philippines retains appear to be one of those that require a public franchise by
ownership of altered or improved facilities and newly way of congressional action under Section 11, Article XII of the
constructed permanent or non-relocatable structures.446 Under Constitution. As respondents submit, however, the system
EDCA, U.S. forces will also be allowed to use facilities and referred to in the agreement does not provide
areas for "training; x x x; support and related activities; x x x; telecommunications services to the public for
temporary accommodation of personnel; communications" and compensation.454 It is clear from Article VIl(2) of EDCA that the
agreed activities.447 telecommunication system is solely for the use of the U.S. and
not the public in general, and that this system will not interfere
with that which local operators use. Consequently, a public
Concerns on national security problems that arise from foreign franchise is no longer necessary.
military equipment being present in the Philippines must
likewise be contextualized. Most significantly, the VFA
already authorizes the presence of U.S. military Additionally, the charge that EDCA allows nuclear weapons
equipment in the country. Article VII of the VFA already within Philippine territory is entirely speculative. It is noteworthy
authorizes the U.S. to import into or acquire in the Philippines that the agreement in fact specifies that the prepositioned
"equipment, materials, supplies, and other property" that will materiel shall not include nuclear weapons. 455Petitioners argue
be used "in connection with activities" contemplated therein. that only prepositioned nuclear weapons are prohibited by
The same section also recognizes that "[t]itle to such property EDCA; and that, therefore, the U.S. would insidiously bring
shall remain" with the US and that they have the discretion to nuclear weapons to Philippine territory. 456 The general
"remove such property from the Philippines at any time." prohibition on nuclear weapons, whether prepositioned or not,
is already expressed in the 1987 Constitution. 457 It would be
unnecessary or superfluous to include all prohibitions already
There is nothing novel, either, in the EDCA provision on the in the Constitution or in the law through a document like EDCA.
prepositioning and storing of "defense equipment, supplies,
and materiel,"448 since these are sanctioned in the VFA. In fact,
the two countries have already entered into various Finally, petitioners allege that EDCA creates a tax exemption,
implementing agreements in the past that are comparable to which under the law must originate from Congress. This
the present one. The Balikatan 02-1 Terms of Reference allegation ignores jurisprudence on the government's
mentioned in Lim v. Executive Secretary specifically assumption of tax liability. EDCA simply states that the taxes
recognizes that Philippine and U.S. forces "may share x x x in on the use of water, electricity, and public utilities are for the
the use of their resources, equipment and other assets." Both account of the Philippine Government. 458 This provision
the 2002 and 2007 Mutual Logistics Support Agreements creates a situation in which a contracting party assumes the
speak of the provision of support and services, including the tax liability of the other.459 In National Power Corporation v.
"construction and use of temporary structures incident to Province of Quezon, we distinguished between enforceable
operations support" and "storage services" during approved and unenforceable stipulations on the assumption of tax
activities.449 These logistic supplies, support, and services liability. Afterwards, we concluded that an enforceable
include the "temporary use of x x x nonlethal items of military assumption of tax liability requires the party assuming the
equipment which are not designated as significant military liability to have actual interest in the property taxed. 460 This rule
equipment on the U.S. Munitions List, during an approved applies to EDCA, since the Philippine Government stands to
activity."450Those activities include "combined exercises and benefit not only from the structures to be built thereon or
training, operations and other deployments" and "cooperative improved, but also from the joint training with U.S. forces,
efforts, such as humanitarian assistance, disaster relief and disaster preparation, and the preferential use of Philippine
rescue operations, and maritime anti-pollution operations" suppliers.461 Hence, the provision on the assumption of tax
within or outside Philippine territory. 451 Under EDCA, the liability does not constitute a tax exemption as petitioners have
equipment, supplies, and materiel that will be prepositioned at posited.
Agreed Locations include "humanitarian assistance and
disaster relief equipment, supplies, and materiel. " 452 Nuclear Additional issues were raised by petitioners, all relating
weapons are specifically excluded from the materiel that will principally to provisions already sufficiently addressed above.
be prepositioned. This Court takes this occasion to emphasize that the
agreement has been construed herein as to absolutely
Therefore, there is no basis to invalidate EDCA on fears that it disauthorize the violation of the Constitution or any applicable
increases the threat to our national security. If anything, EDCA statute. On the contrary, the applicability of Philippine law is
increases the likelihood that, in an event requiring a defensive explicit in EDCA.
FINALS CONSTITUTION I ACJUCO 304

EPILOGUE

The fear that EDCA is a reincarnation of the U.S. bases so


zealously protested by noted personalities in Philippine history
arises not so much from xenophobia, but from a genuine desire
for self-determination, nationalism, and above all a
commitment to ensure the independence of the Philippine
Republic from any foreign domination.

Mere fears, however, cannot curtail the exercise by the


President of the Philippines of his Constitutional prerogatives
in respect of foreign affairs. They cannot cripple him when he
deems that additional security measures are made necessary
by the times. As it stands, the Philippines through the
Department of Foreign Affairs has filed several diplomatic
protests against the actions of the People's Republic of China
in the West Philippine Sea;462 initiated arbitration against that
country under the United Nations Convention on the Law of the
Sea;463 is in the process of negotiations with the Moro Islamic
Liberation Front for peace in Southern Philippines, 464 which is
the subject of a current case before this Court; and faces
increasing incidents of kidnappings of Filipinos and foreigners
allegedly by the Abu Sayyaf or the New People's Army.465 The
Philippine military is conducting reforms that seek to ensure
the security and safety of the nation in the years to come. 466 In
the future, the Philippines must navigate a world in which
armed forces fight with increasing sophistication in both
strategy and technology, while employing asymmetric warfare
and remote weapons.

Additionally, our country is fighting a most terrifying enemy: the


backlash of Mother Nature. The Philippines is one of the
countries most directly affected and damaged by climate
change. It is no coincidence that the record-setting tropical
cyclone Yolanda (internationally named Haiyan), one of the
most devastating forces of nature the world has ever seen hit
the Philippines on 8 November 2013 and killed at least 6,000
people.467 This necessitated a massive rehabilitation
project.468 In the aftermath, the U.S. military was among the
first to extend help and support to the Philippines.

That calamity brought out the best in the Filipinos as thousands


upon thousands volunteered their help, their wealth, and their
prayers to those affected. It also brought to the fore the value
of having friends in the international community.

In order to keep the peace in its archipelago in this region of


the world, and to sustain itself at the same time against the
destructive forces of nature, the Philippines will need friends.
Who they are, and what form the friendships will take, are for
the President to decide. The only restriction is what the
Constitution itself expressly prohibits. It appears that this
overarching concern for balancing constitutional requirements
against the dictates of necessity was what led to EDCA.

As it is, EDCA is not constitutionally infirm. As an executive


agreement, it remains consistent with existing laws and
treaties that it purports to implement.

WHEREFORE, we hereby DISMISS the petitions.

SO ORDERED.
FINALS CONSTITUTION I ACJUCO 305

SUZETTE NICOLAS y SOMBILON, G.R. No. 175888 with the crime of rape committed against a Filipina, petitioner
Petitioner, herein, sometime on November 1, 2005, as follows:

Responden
ts. The undersigned accused LCpl.
Daniel Smith, Ssgt. Chad Brian Carpentier,
X - - - - - - - - - - - - - - - - - - - - - - - - - - - - - - - - - - -X Dominic Duplantis, Keith Silkwood and
Timoteo L. Soriano, Jr. of the crime of Rape
JOVITO R. SALONGA, WIGBERTO G.R. No. 176051 under Article 266-A of the Revised Penal
E. TAADA, JOSE DE LA RAMA, Code, as amended by Republic Act 8353,
EMILIO C. CAPULONG, H. HARRY upon a complaint under oath filed by
L. ROQUE, JR., FLORIN HILBAY, Suzette S. Nicolas, which is attached
and BENJAMIN POZON, hereto and made an integral part hereof as
Petitioners, Annex A, committed as follows:

- versus - That on or
about the First (1st) day
of November 2005,
DANIEL inside the Subic Bay
SMITH, Freeport Zone,
SECRETA Olongapo City and
RY RAUL within the jurisdiction of
GONZALE this Honorable Court,
Z, the above-named
PRESIDEN accuseds (sic), being
TIAL then members of the
LEGAL United States Marine
COUNSEL Corps, except Timoteo
SERGIO L. Soriano, Jr.,
APOSTOL, conspiring,
SECRETA confederating together
RY and mutually helping
RONALDO one another, with lewd
PUNO, design and by means of
SECRETA force, threat and
RY intimidation, with abuse
ALBERTO of superior strength and
ROMULO, taking advantage of the
The intoxication of the
Special victim, did then and
16th Divisi there willfully,
on of the unlawfully and
COURT OF feloniously sexually
APPEALS, abuse and have sexual
and all intercourse with or
persons carnal knowledge of
acting in one Suzette S. Nicolas,
their a 22-year old unmarried
capacity, woman inside a Starex
Respondents. Van with Plate No.
WKF-162, owned by
X - - - - - - - - - - - - - - - - - - - - - - - - - - - - - - - - - - -X Starways Travel
and Tours, with Office
BAGONG ALYANSANG MAKABAYAN G.R. No. 176222, address at 8900 P.
Respondents. Promulgated: Victor St.,
February 11, 2009 Guadalupe, Makati City
, and driven by accused
Timoteo L. Soriano, Jr.,
X --------------------------------------------------------------------------- X against the will and
DECISION consent of the said
Suzette S. Nicolas, to
AZCUNA, J.: her damage and
prejudice.
These are petitions for certiorari, etc. as special civil
actions and/or for review of the Decision of the Court of CONTRARY TO
Appeals in Lance Corporal Daniel J. Smith v. Hon. Benjamin LAW.[1]
T. Pozon, et al., in CA-G.R. SP No. 97212, dated January 2,
2007.
Pursuant to the Visiting Forces Agreement (VFA)
The facts are not disputed. between the Republic of the Philippines and the United States,
entered into on February 10, 1998, the United States, at its
Respondent Lance Corporal (L/CPL) Daniel Smith is request, was granted custody of defendant Smith pending the
a member of the United States Armed Forces. He was charged proceedings.
FINALS CONSTITUTION I ACJUCO 306

During the trial, which was transferred from the accordance with the Visiting Forces
Regional Trial Court (RTC) of Zambales to the RTC of Makati Agreement signed between our two
for security reasons, the United States Government faithfully nations, Lance Corporal Daniel J. Smith,
complied with its undertaking to bring defendant Smith to the United States Marine Corps, be returned to
trial court every time his presence was required. U.S. military custody at the U.S. Embassy
in Manila.
On December 4, 2006, the RTC of Makati, following
the end of the trial, rendered its Decision, finding defendant (Sgd.) KRISTIE A.
Smith guilty, thus: KENNEY (Sgd.) ALBERTO G. ROMULO
Representative of the United
WHEREFORE, premises States Representative of the Republic
considered, for failure of the prosecution to of America of the Philippines
adduce sufficient evidence against
accused S/SGT. CHAD BRIAN DATE: 12-19-06 DATE: December 19,
CARPENTER, L/CPL. KEITH SILKWOOD 2006__
AND L/CPL. DOMINIC DUPLANTIS, all of
the US Marine Corps assigned at the USS
Essex, are hereby ACQUITTED to the
crime charged. and the Romulo-Kenney Agreement of December 22,

The prosecution having 2006 which states:


presented sufficient evidence against
accused L/CPL. DANIEL J. SMITH, also of
The Department of Foreign Affairs of the
the US Marine Corps at the USS Essex,
Republic of the Philippines and the
this Court hereby finds him GUILTY
Embassy of the United States of
BEYOND REASONABLE DOUBT of the
America agree that, in accordance with the
crime of RAPE defined under Article 266-A,
Visiting Forces Agreement signed between
paragraph 1 (a) of the Revised Penal Code,
the two nations, upon transfer of Lance
as amended by R.A. 8353, and, in
Corporal Daniel J. Smith, United States
accordance with Article 266-B, first
Marine Corps, from the Makati City Jail, he
paragraph thereof, hereby sentences him
will be detained at the first floor, Rowe
to suffer the penalty of reclusion
(JUSMAG) Building, U.S. Embassy
perpetua together with the accessory
Compound in a room of approximately 10 x
penalties provided for under Article 41 of
12 square feet. He will be guarded round
the same Code.
the clock by U.S. military personnel. The
Philippine police and jail authorities, under
Pursuant to Article V, paragraph
the direct supervision of the Philippine
No. 10, of the Visiting Forces Agreement
Department of Interior and Local
entered into by the Philippines and
Government (DILG) will have access to the
the United States, accused L/CPL. DANIEL
place of detention to ensure the United
J. SMITH shall serve his sentence in the
States is in compliance with the terms of
facilities that shall, thereafter, be agreed
the VFA.
upon by appropriate Philippine and United
States authorities. Pending agreement on
such facilities, accused L/CPL. DANIEL J.
The matter was brought before the Court of Appeals
SMITH is hereby temporarily committed to
which decided on January 2, 2007, as follows:
the Makati City Jail.
WHEREFORE, all the foregoing
Accused L/CPL. DANIEL J.
considered, we resolved to DISMISS the
SMITH is further sentenced to indemnify
petition for having become moot.[3]
complainant SUZETTE S. NICOLAS in the
amount of P50,000.00 as compensatory
Hence, the present actions.
damages plus P50,000.00 as moral
damages.
The petitions were heard on oral arguments
on September 19, 2008, after which the parties submitted their
SO ORDERED.[2]
memoranda.

Petitioners contend that the Philippines should have


As a result, the Makati court ordered Smith detained
custody of defendant L/CPL Smith because, first of all, the VFA
at the Makati jail until further orders.
is void and unconstitutional.
On December 29, 2006, however, defendant Smith
This issue had been raised before, and this Court
was taken out of the Makati jail by a contingent of Philippine
resolved in favor of the constitutionality of the VFA. This was
law enforcement agents, purportedly acting under orders of the
in Bayan v. Zamora,[4] brought by Bayan, one of petitioners in
Department of the Interior and Local Government, and brought
the present cases.
to a facility for detention under the control of the United States
government, provided for under new agreements between the
Against the barriers of res judicata vis--vis Bayan,
Philippines and the United States, referred to as the Romulo-
and stare decisis vis--vis all the parties, the reversal of the
Kenney Agreement of December 19, 2006 which states:
previous ruling is sought on the ground that the issue is of
primordial importance, involving the sovereignty of the
The Government of the Republic of Republic, as well as a specific mandate of the Constitution.
the Philippines and the Government of
the United States of America agree that, in
FINALS CONSTITUTION I ACJUCO 307

The provision of the Constitution is Art. XVIII, Sec. 25 The fact that the VFA was not submitted for advice
which states: and consent of the United States Senate does not detract from
its status as a binding international agreement or treaty
Sec. 25. After the expiration in recognized by the said State. For this is a matter of
1991 of the Agreement between the internal United States law. Notice can be taken of the
Philippines and the United States of internationally known practice by the United States of
America concerning Military Bases, foreign submitting to its Senate for advice and consent agreements
military bases, troops, or facilities shall not that are policymaking in nature, whereas those that carry out
be allowed in the Philippines except under or further implement these policymaking agreements are
a treaty duly concurred in by the Senate merely submitted to Congress, under the provisions of the so-
and, when the Congress so requires, called CaseZablocki Act, within sixty days from ratification. [6]
ratified by a majority of the votes cast by the
people in a national referendum held for The second reason has to do with the relation
that purpose, and recognized as a treaty by between the VFA and the RP-US Mutual Defense Treaty
the other contracting State. of August 30, 1951. This earlier agreement was signed and
duly ratified with the concurrence of both the Philippine Senate
and the United States Senate.
The reason for this provision lies in history and the
Philippine experience in regard to the United States military The RP-US Mutual Defense Treaty states:[7]
bases in the country.
MUTUAL DEFENSE TREATY BETWEEN
It will be recalled that under the Philippine Bill of THE REPUBLIC OF
1902, which laid the basis for the Philippine Commonwealth THE PHILIPPINES AND THE UNITED
and, eventually, for the recognition of independence, the STATES OF AMERICA. Signed
United States agreed to cede to the Philippines all the territory at Washington, August 30, 1951.
it acquired from Spain under the Treaty of Paris, plus a few
islands later added to its realm, except certain naval ports The Parties of this Treaty
and/or military bases and facilities, which the United States
retained for itself. Reaffirming their faith in the purposes and
principles of the Charter of the United
This is noteworthy, because what this means is Nations and their desire to live in peace
that Clark and Subic and the other places in with all peoples and all governments, and
the Philippines covered by the RP-US Military Bases desiring to strengthen the fabric of peace in
Agreement of 1947 were not Philippine territory, as they were the Pacific area.
excluded from the cession and retained by the US.
Recalling with mutual pride the historic
Accordingly, the Philippines had no jurisdiction over relationship which brought their two
these bases except to the extent allowed by the United peoples together in a common bond of
States. Furthermore, the RP-US Military Bases Agreement sympathy and mutual ideals to fight side-
was never advised for ratification by the United States Senate, by-side against imperialist aggression
a disparity in treatment, because the Philippines regarded it as during the last war.
a treaty and had it concurred in by our Senate.
Desiring to declare publicly and
Subsequently, the United States agreed to turn over formally their sense of unity and their
these bases to the Philippines; and with the expiration of the common determination to defend
RP-US Military Bases Agreement in 1991, the territory covered themselves against external armed
by these bases were finally ceded to the Philippines. attack, so that no potential aggressor
could be under the illusion that either of
To prevent a recurrence of this experience, the them stands alone in the Pacific area.
provision in question was adopted in the 1987 Constitution.
Desiring further to strengthen their
The provision is thus designed to ensure that any present efforts for collective defense for
agreement allowing the presence of foreign military bases, the preservation of peace and
troops or facilities in Philippine territory shall be equally binding security pending the development of a
on the Philippines and the foreign sovereign State more comprehensive system of regional
involved. The idea is to prevent a recurrence of the situation in security in the Pacific area.
which the terms and conditions governing the presence of
foreign armed forces in our territory were binding upon us but Agreeing that nothing in this present
not upon the foreign State. instrument shall be considered or
interpreted as in any way or sense altering
Applying the provision to the situation involved in or diminishing any existing agreements or
these cases, the question is whether or not the presence of US understandings between the Republic of
Armed Forces in Philippine territory pursuant to the VFA is the Philippines and the United States of
allowed under a treaty duly concurred in by the Senate xxx and America.
recognized as a treaty by the other contracting State.
Have agreed as follows:
This Court finds that it is, for two reasons.
ARTICLE I. The parties undertake, as set
First, as held in Bayan v. Zamora,[5] the VFA was forth in the Charter of the United Nations,
duly concurred in by the Philippine Senate and has been to settle any international disputes in which
recognized as a treaty by the United States as attested and they may be involved by peaceful means in
certified by the duly authorized representative of the United such a manner that international peace and
States government. security and justice are not endangered
and to refrain in their international relation
FINALS CONSTITUTION I ACJUCO 308

from the threat or use of force in any DONE in duplicate at Washington this
manner inconsistent with the purposes of thirtieth day of August, 1951.
the United Nations.
For the Republic of the Philippines:
ARTICLE II. In order more effectively to (Sgd.) CARLOS P. ROMULO
achieve the objective of this Treaty, (Sgd.) JOAQUIN M. ELIZALDE
the Parties separately and jointly by (Sgd.) VICENTE J. FRANCISCO
self-help and mutual aid will maintain (Sgd.) DIOSDADO MACAPAGAL
and develop their individual and
collective capacity to resist armed For the United States of America:
attack.
(Sgd.) DEAN ACHESON
ARTICLE III. The Parties, through their (Sgd.) JOHN FOSTER DULLES
Foreign Ministers or their deputies, will (Sgd.) TOM CONNALLY
consult together from time to time (Sgd.) ALEXANDER WILEY[8]
regarding the implementation of this Treaty
and whenever in the opinion of either of
them the territorial integrity, political Clearly, therefore, joint RP-US military exercises for
independence or security of either of the the purpose of developing the capability to resist an armed
Parties is threatened by external armed attack fall squarely under the provisions of the RP-US Mutual
attack in the Pacific. Defense Treaty. The VFA, which is the instrument agreed
upon to provide for the joint RP-US military exercises, is simply
ARTICLE IV. Each Party recognizes that an implementing agreement to the main RP-US Military
an armed attack in the Pacific area on Defense Treaty. The Preamble of the VFA states:
either of the parties would be dangerous to
its own peace and safety and declares that The Government of the United States of
it would act to meet the common dangers America and the Government of the
in accordance with its constitutional Republic of the Philippines,
processes.
Reaffirming their faith in the purposes and
Any such armed attack and all measures principles of the Charter of the United
taken as a result thereof shall be Nations and their desire to strengthen
immediately reported to the Security international and regional security in the
Council of the United Nations. Such Pacific area;
measures shall be terminated when the
Security Council has taken the measures Reaffirming their obligations under the
necessary to restore and maintain Mutual Defense Treaty of August 30,
international peace and security. 1951;

ARTICLE V. For the purpose of Article IV, Noting that from time to time elements
an armed attack on either of the Parties is of the United States armed forces may
deemed to include an armed attack on the visit the Republic of the Philippines;
metropolitan territory of either of the
Parties, or on the island territories under its Considering that cooperation between
jurisdiction in the Pacific Ocean, its armed the United States and the Republic of
forces, public vessels or aircraft in the the Philippines promotes their common
Pacific. security interests;

ARTICLE VI. This Treaty does not affect Recognizing the desirability of defining the
and shall not be interpreted as affecting in treatment of United States personnel
any way the rights and obligations of the visiting the Republic of the Philippines;
Parties under the Charter of the United
Nations or the responsibility of the United Have agreed as follows:[9]
Nations for the maintenance of
international peace and security. Accordingly, as an implementing agreement of the
RP-US Mutual Defense Treaty, it was not necessary to submit
ARTICLE VII. This Treaty shall be ratified the VFA to the US Senate for advice and consent, but merely
by the Republic of the Philippines and the to the US Congress under the CaseZablocki Act within 60 days
United Nations of America in accordance of its ratification. It is for this reason that the US has certified
with their respective constitutional that it recognizes the VFA as a binding international
processes and will come into force when agreement, i.e., a treaty, and this substantially complies with
instruments of ratification thereof have the requirements of Art. XVIII, Sec. 25 of our Constitution. [10]
been exchanged by them at Manila.
The provision of Art. XVIII, Sec. 25 of the
ARTICLE VIII. This Treaty shall remain in Constitution, is complied with by virtue of the fact that the
force indefinitely. Either Party may presence of the US Armed Forces through the VFA is a
terminate it one year after notice has been presence allowed under the RP-US Mutual Defense
given to the other party. Treaty. Since the RP-US Mutual Defense Treaty itself has
been ratified and concurred in by both the Philippine Senate
IN WITHNESS WHEREOF the and the US Senate, there is no violation of the Constitutional
undersigned Plenipotentiaries have signed provision resulting from such presence.
this Treaty.
FINALS CONSTITUTION I ACJUCO 309

The VFA being a valid and binding agreement, the or violated, but rather one in which, as is normally encountered
parties are required as a matter of international law to abide by around the world, the laws (including rules of procedure) of one
its terms and provisions. State do not extend or apply except to the extent agreed
upon to subjects of another State due to the recognition of
The VFA provides that in cases of offenses extraterritorial immunity given to such bodies as visiting foreign
committed by the members of the US Armed Forces in armed forces.
the Philippines, the following rules apply:
Nothing in the Constitution prohibits such
agreements recognizing immunity from jurisdiction or some
Article V aspects of jurisdiction (such as custody), in relation to long-
Criminal Jurisdiction recognized subjects of such immunity like Heads of State,
diplomats and members of the armed forces contingents of a
xxx foreign State allowed to enter another States territory. On the
6. The custody of any United contrary, the Constitution states that the Philippines adopts the
States personnel over whom the generally accepted principles of international law as part of the
Philippines is to exercise jurisdiction shall law of the land. (Art. II, Sec. 2).
immediately reside with United States
military authorities, if they so request, from Applying, however, the provisions of VFA, the Court
the commission of the offense until finds that there is a different treatment when it comes to
completion of all judicial detention as against custody. The moment the accused has to
proceedings. United States military be detained, e.g., after conviction, the rule that governs is the
authorities shall, upon formal notification by following provision of the VFA:
the Philippine authorities and without delay,
make such personnel available to those Article V
authorities in time for any investigative or Criminal Jurisdiction
judicial proceedings relating to the offense
with which the person has been charged. In xxx
extraordinary cases, the Philippine Sec. 10. The confinement or
Government shall present its position to the detention by Philippine authorities
United States Government regarding of United States personnel shall be carried
custody, which the United States out in facilities agreed on by
Government shall take into full account. In appropriate Philippines and United
the event Philippine judicial proceedings Statesauthorities. United States personnel
are not completed within one year, serving sentences in the Philippines shall
the United States shall be relieved of any have the right to visits and material
obligations under this paragraph. The one assistance.
year period will not include the time
necessary to appeal. Also, the one year
period will not include any time during It is clear that the parties to the VFA recognized the
which scheduled trial procedures are difference between custody during the trial and detention after
delayed because United States authorities, conviction, because they provided for a specific arrangement
after timely notification by Philippine to cover detention. And this specific arrangement clearly states
authorities to arrange for the presence of not only that the detention shall be carried out in facilities
the accused, fail to do so. agreed on by authorities of both parties, but also that the
detention shall be by Philippine authorities. Therefore, the
Petitioners contend that these undertakings violate Romulo-Kenney Agreements of December 19 and 22, 2006,
another provision of the Constitution, namely, that providing for which are agreements on the detention of the accused in
the exclusive power of this Court to adopt rules of procedure the United States Embassy, are not in accord with the VFA
for all courts in the Philippines (Art. VIII, Sec. 5[5]). They argue itself because such detention is not by Philippine authorities.
that to allow the transfer of custody of an accused to a foreign
power is to provide for a different rule of procedure for that Respondents should therefore comply with the VFA
accused, which also violates the equal protection clause of the and negotiate with representatives of the United
Constitution (Art. III, Sec. 1.). States towards an agreement on detention facilities under
Philippine authorities as mandated by Art. V, Sec. 10 of the
Again, this Court finds no violation of the VFA.
Constitution.
Next, the Court addresses the recent decision of the
The equal protection clause is not violated, because United States Supreme Court in Medellin v. Texas ( 552 US
there is a substantial basis for a different treatment of a ___ No. 06-984, March 25, 2008), which held that treaties
member of a foreign military armed forces allowed to enter our entered into by the United States are not automatically part of
territory and all other accused.[11] their domestic law unless these treaties are self-executing or
there is an implementing legislation to make them enforceable.
The rule in international law is that a foreign armed
forces allowed to enter ones territory is immune from local On February 3, 2009, the Court issued a Resolution,
jurisdiction, except to the extent agreed upon. The Status of thus:
Forces Agreements involving foreign military units around the
world vary in terms and conditions, according to the situation G.R. No. 175888 (Suzette
of the parties involved, and reflect their bargaining power. But Nicolas y Sombilon v. Alberto
the principle remains, i.e., the receiving State can exercise Romulo, et al.); G.R. No. 176051
jurisdiction over the forces of the sending State only to the (Jovito R. Salonga, et al. v. Daniel
extent agreed upon by the parties.[12] Smith, et al.); and G.R. No.
176222 (Bagong Alyansang
As a result, the situation involved is not one in which Makabayan [BAYAN], et al. v.
the power of this Court to adopt rules of procedure is curtailed
FINALS CONSTITUTION I ACJUCO 310

President Gloria Macapagal- The framers of the Constitution were aware that the
Arroyo, et al.). application of international law in domestic courts varies from
country to country.
The parties, including the Solicitor General,
are required to submit within three (3) days As Ward N. Ferdinandusse states in his Treatise,
a Comment/Manifestation on the following DIRECT APPLICATION OF INTERNATIONAL CRIMINAL
points: LAW IN NATIONAL COURTS, some countries require
legislation whereas others do not.
1. What is the implication on the
RP-US Visiting Forces It was not the intention of the framers of the 1987
Agreement of the recent US Constitution, in adopting Article XVIII, Sec. 25, to require the
Supreme Court decision in Jose other contracting State to convert their system to achieve
Ernesto Medellin v. Texas, dated alignment and parity with ours. It was simply required that the
March 25, 2008, to the effect that treaty be recognized as a treaty by the other contracting
treaty stipulations that are not State. With that, it becomes for both parties a binding
self-executory can only be international obligation and the enforcement of that obligation
enforced pursuant to legislation is left to the normal recourse and processes under international
to carry them into effect; and that, law.
while treaties may comprise
international commitments, they Furthermore, as held by the US Supreme Court
are not domestic law unless in Weinberger v. Rossi,[13] an executive agreement is a treaty
Congress has enacted within the meaning of that word in international law and
implementing statutes or the constitutes enforceable domestic law vis--vis the United
treaty itself conveys an intention States. Thus, the US Supreme Court in Weinberger enforced
that it be self-executory and is the provisions of the executive agreement granting preferential
ratified on these terms? employment to Filipinos in the US Bases here.

2. Whether the VFA is


enforceable in the US as Accordingly, there are three types of treaties in the
domestic law, either because it is American system:
self-executory or because there
exists legislation to implement it. 1. Art. II, Sec. 2 treaties These are advised
and consented to by the US Senate in
3. Whether the RP-US Mutual accordance with Art. II, Sec. 2 of
Defense Treaty of August 30, the US Constitution.
1951 was concurred in by the US
Senate and, if so, is there proof of 2. ExecutiveCongressional Agreements:
the US Senate advice and These are joint agreements of the
consent resolution? Peralta, J., President and Congress and need not be
no part. submitted to the Senate.

After deliberation, the Court holds, on these points, 3. Sole Executive Agreements. These are
as follows: agreements entered into by the
President. They are to be submitted to
First, the VFA is a self-executing Agreement, as that Congress within sixty (60) days of
term is defined in Medellin itself, because the parties intend its ratification under the provisions of the
provisions to be enforceable, precisely because the Case-Zablocki Act, after which they are
Agreement is intended to carry out obligations and recognized by the Congress and may be
undertakings under the RP-US Mutual Defense Treaty. As a implemented.
matter of fact, the VFA has been implemented and executed,
with the US faithfully complying with its obligation to produce As regards the implementation of the RP-US Mutual
L/CPL Smith before the court during the trial. Defense Treaty, military aid or assistance has been given
under it and this can only be done through implementing
Secondly, the VFA is covered by implementing legislation, legislation. The VFA itself is another form of implementation of
namely, the Case-Zablocki Act, USC Sec. 112(b), inasmuch as its provisions.
it is the very purpose and intent of the US Congress that
executive agreements registered under this Act within 60 days WHEREFORE, the petitions are PARTLY
from their ratification be immediately implemented. The parties GRANTED, and the Court of Appeals Decision in CA-G.R. SP
to these present cases do not question the fact that the VFA No. 97212 dated January 2, 2007 is MODIFIED. The Visiting
has been registered under the Case-Zablocki Act. Forces Agreement (VFA) between the Republic of the
Philippines and the United States, entered into on February 10,
In sum, therefore, the VFA differs from the Vienna Convention 1998, is UPHELD as constitutional, but the Romulo-Kenney
on Consular Relations and the Avena decision of the Agreements of December 19 and 22, 2006
International Court of Justice (ICJ), subject matter of are DECLARED not in accordance with the VFA, and
the Medellin decision. The Convention and the ICJ decision respondent Secretary of Foreign Affairs is hereby ordered to
are not self-executing and are not registrable under the Case- forthwith negotiate with the United States representatives for
Zablocki Act, and thus lack legislative implementing authority. the appropriate agreement on detention facilities under
Philippine authorities as provided in Art. V, Sec. 10 of the VFA,
Finally, the RP-US Mutual Defense Treaty was pending which the status quo shall be maintained until further
advised and consented to by the US Senate on March 20, orders by this Court.
1952, as reflected in the US Congressional Record,
82ndCongress, Second Session, Vol. 98 Part 2, pp. 2594-2595. The Court of Appeals is hereby directed to resolve
without delay the related matters pending therein, namely, the
FINALS CONSTITUTION I ACJUCO 311

petition for contempt and the appeal of L/CPL Daniel Smith


from the judgment of conviction.

No costs.

SO ORDERED.
FINALS CONSTITUTION I ACJUCO 312

GR 162230 for official apology and other forms of reparations against Japan before
Romulo vs. Vinuya the International Court of Justice (ICJ) and other international tribunals.
DECISION
Petitioners arguments
DEL CASTILLO, J.:
Petitioners argue that the general waiver of claims made by
The Treaty of Peace with Japan, insofar as it the Philippine government in the Treaty of Peace with Japan is
barred future claims such as those asserted by void. They claim that the comfort women system established
plaintiffs in these actions, exchanged full by Japan, and the brutal rape and enslavement of petitioners
compensation of plaintiffs for a future peace. constituted a crime against humanity,[3] sexual slavery,[4] and
History has vindicated the wisdom of that torture.[5] They allege that the prohibition against these international
bargain. And while full compensation for crimes is jus cogens norms from which no derogation is possible; as
plaintiffs' hardships, in the purely economic such, in waiving the claims of Filipina comfort women and failing to
sense, has been denied these former prisoners espouse their complaints against Japan, the Philippine government is
and countless other survivors of the war, the in breach of its legal obligation not to afford impunity for crimes against
immeasurable bounty of life for themselves and humanity. Finally, petitioners assert that the Philippine governments
their posterity in a free society and in a more acceptance of the apologies made by Japan as well as funds from the
peaceful world services the debt.[1] Asian Womens Fund (AWF) were contrary to international law.

Respondents Arguments
There is a broad range of vitally important areas that must be regularly
decided by the Executive Department without either challenge or Respondents maintain that all claims of the Philippines and its
interference by the Judiciary. One such area involves the delicate nationals relative to the war were dealt with in the San Francisco
arena of foreign relations. It would be strange indeed if the courts and Peace Treaty of 1951 and the bilateral Reparations Agreement of
the executive spoke with different voices in the realm of foreign policy. 1956.[6]
Precisely because of the nature of the questions presented, and the Article 14 of the Treaty of Peace[7] provides:
lapse of more than 60 years since the conduct complained of, we
make no attempt to lay down general guidelines covering other Article 14. Claims and Property
situations not involved here, and confine the opinion only to the very
questions necessary to reach a decision on this matter. a) It is recognized that Japan should pay
reparations to the Allied Powers
Factual Antecedents for the damage and suffering
caused by it during the war.
This is an original Petition for Certiorari under Rule 65 of the Nevertheless it is also
Rules of Court with an application for the issuance of a writ of recognized that the resources
preliminary mandatory injunction against the Office of the Executive of Japan are not presently
Secretary, the Secretary of the Department of Foreign Affairs (DFA), sufficient, if it is to maintain a
the Secretary of the Department of Justice (DOJ), and the Office of the viable economy, to make
Solicitor General (OSG). complete reparation for all such
damage and suffering and at
Petitioners are all members of the MALAYA LOLAS, the present time meet its other
a non-stock, non-profit organization registered with the obligations.
Securities and Exchange Commission, established for the
purpose of providing aid to the victims of rape by Japanese b) Except as otherwise
military forces in the Philippines during the Second World War. provided in the present Treaty,
the Allied Powers waive all
Petitioners narrate that during the Second World War, the reparations claims of the Allied
Japanese army attacked villages and systematically raped the women Powers, other claims of the
as part of the destruction of the village. Their communities were Allied Powers and their
bombed, houses were looted and burned, and civilians were publicly nationals arising out of any
tortured, mutilated, and slaughtered. Japanese soldiers forcibly seized actions taken by Japan and its
the women and held them in houses or cells, where they were nationals in the course of the
repeatedly raped, beaten, and abused by Japanese soldiers. As a prosecution of the war, and
result of the actions of their Japanese tormentors, the petitioners have claims of the Allied Powers for
spent their lives in misery, having endured physical injuries, pain and direct military costs of
disability, and mental and emotional suffering.[2] occupation.
Petitioners claim that since 1998, they have approached
the Executive Department through the DOJ, DFA, and OSG,
requesting assistance in filing a claim against the Japanese officials In addition, respondents argue that the apologies made
and military officers who ordered the establishment of the comfort by Japan[8] have been satisfactory, and that Japan had addressed the
women stations in the Philippines. However, officials of the Executive individual claims of the women through the atonement money paid by
Department declined to assist the petitioners, and took the position the Asian Womens Fund.
that the individual claims of the comfort women for compensation had Historical Background
already been fully satisfied by Japans compliance with the Peace
Treaty between the Philippines and Japan. The comfort women system was the tragic legacy of the Rape of
Nanking. In December 1937, Japanese military forces captured the
Issues city of Nanking in China and began a barbaric campaign of terror
known as the Rape of Nanking, which included the rapes and murders
Hence, this petition where petitioners pray for this court to of an estimated 20,000 to 80,000 Chinese women, including young
(a) declare that respondents committed grave abuse of discretion girls, pregnant mothers, and elderly women.[9]
amounting to lack or excess of discretion in refusing to espouse their
claims for the crimes against humanity and war crimes committed In reaction to international outcry over the incident, the
against them; and (b) compel the respondents to espouse their claims Japanese government sought ways to end international
condemnation[10] by establishing the comfort women system. Under
FINALS CONSTITUTION I ACJUCO 313

this system, the military could simultaneously appease soldiers' sexual investigating crimes committed by Japan against Korean women and
appetites and contain soldiers' activities within a regulated seeking reparations for former comfort women.[29] The UNHRC
environment.[11] Comfort stations would also prevent the spread of placed the issue on its agenda and appointed Radhika
venereal disease among soldiers and discourage soldiers from raping Coomaraswamy as the issue's special investigator. In 1996,
inhabitants of occupied territories.[12] Coomaraswamy issued a Report reaffirming Japan's responsibility in
forcing Korean women to act as sex slaves for the imperial army, and
Daily life as a comfort woman was unmitigated misery.[13] The made the following recommendations:
military forced victims into barracks-style stations divided into tiny
cubicles where they were forced to live, sleep, and have sex with as A. At the national level
many 30 soldiers per day.[14] The 30 minutes allotted for sexual 137. The Government of Japan should:
relations with each soldier were 30-minute increments of
unimaginable horror for the women.[15] Disease was (a) Acknowledge that the system of comfort
rampant.[16] Military doctors regularly examined the women, but these stations set up by the Japanese
checks were carried out to prevent the spread of venereal diseases; Imperial Army during the Second
little notice was taken of the frequent cigarette burns, bruises, bayonet World War was a violation of its
stabs and even broken bones inflicted on the women by soldiers. obligations under international law
and accept legal responsibility for that
Fewer than 30% of the women survived the war.[17] Their agony violation;
continued in having to suffer with the residual physical, psychological,
and emotional scars from their former lives. Some returned home and (b) Pay compensation to individual victims of
were ostracized by their families. Some committed suicide. Others, Japanese military sexual slavery
out of shame, never returned home.[18] according to principles outlined by the
Special Rapporteur of the Sub-
Efforts to Secure Reparation Commission on Prevention of
Discrimination and Protection of
The most prominent attempts to compel the Japanese Minorities on the right to restitution,
government to accept legal responsibility and pay compensatory compensation and rehabilitation for
damages for the comfort women system were through a series of victims of grave violations of human
lawsuits, discussion at the United Nations (UN), resolutions by various rights and fundamental freedoms. A
nations, and the Womens International Criminal Tribunal. The special administrative tribunal for this
Japanese government, in turn, responded through a series of public purpose should be set up with a
apologies and the creation of the AWF.[19] limited time-frame since many of the
victims are of a very advanced age;
Lawsuits
(c) Make a full disclosure of documents and
In December 1991, Kim Hak-Sun and two other survivors materials in its possession with
filed the first lawsuit in Japan by former comfort women against the regard to comfort stations and other
Japanese government. The Tokyo District Court however dismissed related activities of the Japanese
their case.[20] Other suits followed,[21] but the Japanese government Imperial Army during the Second
has, thus far, successfully caused the dismissal of every case.[22] World War;

Undoubtedly frustrated by the failure of litigation before (d) Make a public apology in writing to individual
Japanese courts, victims of the comfort women system brought their women who have come forward and
claims before the United States (US). On September 18, 2000, 15 can be substantiated as women
comfort women filed a class action lawsuit in the US District Court for victims of Japanese military sexual
the District of Columbia[23] "seeking money damages for [allegedly] slavery;
having been subjected to sexual slavery and torture before and during
World War II," in violation of "both positive and customary international (e) Raise awareness of these issues by
law." The case was filed pursuant to the Alien Tort Claims Act amending educational curricula to
(ATCA),[24] which allowed the plaintiffs to sue the Japanese reflect historical realities;
government in a US federal district court.[25] On October 4, 2001, the
district court dismissed the lawsuit due to lack of jurisdiction over (f) Identify and punish, as far as possible,
Japan, stating that [t]here is no question that this court is not the perpetrators involved in the
appropriate forum in which plaintiffs may seek to reopen x x x recruitment and institutionalization of
discussions nearly half a century later x x x [E]ven if Japan did not comfort stations during the Second
enjoy sovereign immunity, plaintiffs' claims are non-justiciable and World War.
must be dismissed.

The District of Columbia Court of Appeals affirmed the Gay J. McDougal, the Special Rapporteur for the UN Sub-
lower court's dismissal of the case.[26] On appeal, the US Supreme Commission on Prevention of Discrimination and Protection of
Court granted the womens petition for writ of certiorari, vacated the Minorities, also presented a report to the Sub-Committee on June 22,
judgment of the District of Columbia Court of Appeals, and remanded 1998 entitled Contemporary Forms of Slavery: Systematic Rape,
the case.[27] On remand, the Court of Appeals affirmed its prior Sexual Slavery and Slavery-like Practices During Armed Conflict. The
decision, noting that much as we may feel for the plight of the report included an appendix entitled An Analysis of the Legal Liability
appellants, the courts of the US simply are not authorized to hear their of the Government of Japan for 'Comfort Women Stations'
case.[28] The women again brought their case to the US Supreme established during the Second World War,[30] which contained the
Court which denied their petition for writ of certiorari on February 21, following findings:
2006.
68. The present report concludes that the
Efforts at the United Nations Japanese Government remains liable for grave
violations of human rights and humanitarian law,
In 1992, the Korean Council for the Women Drafted for violations that amount in their totality to crimes
Military Sexual Slavery by Japan (KCWS), submitted a petition to the against humanity. The Japanese Governments
UN Human Rights Commission (UNHRC), asking for assistance in arguments to the contrary, including arguments
FINALS CONSTITUTION I ACJUCO 314

that seek to attack the underlying humanitarian Resolution was formally passed on July 30, 2007,[33] and made four
law prohibition of enslavement and rape, remain distinct demands:
as unpersuasive today as they were when they
were first raised before the Nuremberg war [I]t is the sense of the House of Representatives
crimes tribunal more than 50 years ago. In that the Government of Japan (1) should
addition, the Japanese Governments argument formally acknowledge, apologize, and accept
that Japan has already settled all claims from historical responsibility in a clear and
the Second World War through peace treaties unequivocal manner for its Imperial Armed
and reparations agreements following the war Forces' coercion of young women into sexual
remains equally unpersuasive. This is due, in slavery, known to the world as comfort women,
large part, to the failure until very recently of the during its colonial and wartime occupation of
Japanese Government to admit the extent of the Asia and the Pacific Islands from the 1930s
Japanese militarys direct involvement in the through the duration of World War II; (2) would
establishment and maintenance of these rape help to resolve recurring questions about the
centres. The Japanese Governments silence sincerity and status of prior statements if the
on this point during the period in which peace Prime Minister of Japan were to make such an
and reparations agreements apology as a public statement in his official
between Japan and other Asian Governments capacity; (3) should clearly and publicly refute
were being negotiated following the end of the any claims that the sexual enslavement and
war must, as a matter of law and justice, trafficking of the comfort women for the
preclude Japan from relying today on these Japanese Imperial Army never occurred; and
peace treaties to extinguish liability in these (4) should educate current and future
cases. generations about this horrible crime while
following the recommendations of the
69. The failure to settle these claims more than international community with respect to the
half a century after the cessation of hostilities is comfort women.[34]
a testament to the degree to which the lives of
women continue to be undervalued. Sadly, this In December 2007, the European Parliament, the
failure to address crimes of a sexual nature governing body of the European Union, drafted a resolution similar to
committed on a massive scale during the House Resolution 121.[35] Entitled, Justice for Comfort Women, the
Second World War has added to the level of resolution demanded: (1) a formal acknowledgment of responsibility
impunity with which similar crimes are by the Japanese government; (2) a removal of the legal obstacles
committed today. The Government of Japan preventing compensation; and (3) unabridged education of the past.
has taken some steps to apologize and atone for The resolution also stressed the urgency with which Japan should act
the rape and enslavement of over 200,000 on these issues, stating: the right of individuals to claim reparations
women and girls who were brutalized in comfort against the government should be expressly recognized in national
stations during the Second World law, and cases for reparations for the survivors of sexual slavery, as a
War. However, anything less than full and crime under international law, should be prioritized, taking into account
unqualified acceptance by the Government of the age of the survivors.
Japan of legal liability and the consequences
that flow from such liability is wholly The Canadian and Dutch parliaments have each followed
inadequate. It must now fall to the Government suit in drafting resolutions against Japan. Canada's resolution
of Japan to take the necessary final steps to demands the Japanese government to issue a formal apology, to
provide adequate redress. admit that its Imperial Military coerced or forced hundreds of
thousands of women into sexual slavery, and to restore references in
Japanese textbooks to its war crimes.[36] The Dutch parliament's
The UN, since then, has not taken any official action directing Japan to resolution calls for the Japanese government to uphold the 1993
provide the reparations sought. declaration of remorse made by Chief Cabinet Secretary Yohei Kono.

Women's International War Crimes Tribunal The Foreign Affairs Committee of the United Kingdoms
Parliament also produced a report in November, 2008 entitled, "Global
The Women's International War Crimes Tribunal (WIWCT) was a Security: Japan and Korea" which concluded that Japan should
people's tribunal established by a number of Asian women and acknowledge the pain caused by the issue of comfort women in order
human rights organizations, supported by an international coalition of to ensure cooperation between Japan and Korea
non-governmental organizations.[31] First proposed in 1998, the
WIWCT convened in Tokyo in 2000 in order to adjudicate Japan's Statements of Remorse made by representatives of the
military sexual violence, in particular the enslavement of comfort Japanese government
women, to bring those responsible for it to justice, and to end the
ongoing cycle of impunity for wartime sexual violence against women.
Various officials of the Government of Japan have issued
After examining the evidence for more than a year, the the following public statements concerning the comfort system:
tribunal issued its verdict on December 4, 2001, finding the former
Emperor Hirohito and the State of Japan guilty of crimes against a) Statement by the Chief Cabinet Secretary Yohei Kono in 1993:
humanity for the rape and sexual slavery of women.[32] It bears
stressing, however, that although the tribunal included prosecutors, The Government of Japan has been conducting
witnesses, and judges, its judgment was not legally binding since the a study on the issue of wartime "comfort
tribunal itself was organized by private citizens. women" since December 1991. I wish to
announce the findings as a result of that study.
Action by Individual Governments
As a result of the study which indicates that
On January 31, 2007, US Representative Michael Honda comfort stations were operated in extensive
of California, along with six co-sponsor representatives, introduced areas for long periods, it is apparent that there
House Resolution 121 which called for Japanese action in light of the existed a great number of comfort women.
ongoing struggle for closure by former comfort women. The Comfort stations were operated in response to
FINALS CONSTITUTION I ACJUCO 315

the request of the military authorities of the day. the women who endured immeasurable and
The then Japanese military was, directly or painful experiences and suffered incurable
indirectly, involved in the establishment and physical and psychological wounds as comfort
management of the comfort stations and the women.
transfer of comfort women. The recruitment of
the comfort women was conducted mainly by I believe that our country, painfully aware of its
private recruiters who acted in response to the moral responsibilities, with feelings of apology
request of the military. The Government study and remorse, should face up squarely to its past
has revealed that in many cases they were history and accurately convey it to future
recruited against their own will, through coaxing generations.
coercion, etc., and that, at times,
administrative/military personnel directly took
part in the recruitments. They lived in misery at d) The Diet (Japanese Parliament) passed resolutions in
comfort stations under a coercive atmosphere. 1995 and 2005

As to the origin of those comfort women who Solemnly reflecting upon the many instances of
were transferred to the war areas, excluding colonial rule and acts of aggression that
those from Japan, those from occurred in modern world history, and
the Korean Peninsula accounted for a large recognizing that Japan carried out such acts in
part. The Korean Peninsula was under the past and inflicted suffering on the people of
Japanese rule in those days, and their other countries, especially in Asia, the Members
recruitment, transfer, control, etc., were of this House hereby express deep remorse.
conducted generally against their will, through (Resolution of the House of Representatives
coaxing, coercion, etc. adopted on June 9, 1995)

Undeniably, this was an act, with the


involvement of the military authorities of the day, e) Various Public Statements by Japanese Prime Minister Shinzo Abe
that severely injured the honor and dignity of I have talked about this matter in the Diet
many women. The Government of Japan would sessions last year, and recently as well, and to
like to take this opportunity once again to extend the press. I have been consistent. I will stand by
its sincere apologies and remorse to all those, the Kono Statement. This is our consistent
irrespective of place of origin, who suffered position. Further, we have been apologizing
immeasurable pain and incurable physical and sincerely to those who suffered immeasurable
psychological wounds as comfort women. pain and incurable psychological wounds as
comfort women. Former Prime Ministers,
It is incumbent upon us, the Government of including Prime Ministers Koizumi and
Japan, to continue to consider seriously, while Hashimoto, have issued letters to the comfort
listening to the views of learned circles, how best women. I would like to be clear that I carry the
we can express this sentiment. same feeling. This has not changed even
slightly. (Excerpt from Remarks by Prime
We shall face squarely the historical facts as Minister Abe at an Interview by NHK, March 11,
described above instead of evading them, and 2007).
take them to heart as lessons of history. We
hereby reiterated our firm determination never to I am apologizing here and now. I am apologizing
repeat the same mistake by forever engraving as the Prime Minister and it is as stated in the
such issues in our memories through the study statement by the Chief Cabinet Secretary Kono.
and teaching of history. (Excerpt from Remarks by Prime Minister Abe
at the Budget Committee, the House of
As actions have been brought to court Councilors, the Diet of Japan, March 26, 2007).
in Japan and interests have been shown in this
issue outside Japan, the Government of Japan I am deeply sympathetic to the former comfort
shall continue to pay full attention to this matter, women who suffered hardships, and I have
including private researched related thereto. expressed my apologies for the extremely
agonizing circumstances into which they were
placed. (Excerpt from Telephone Conference by
b) Prime Minister Tomiichi Murayamas Statement in Prime Minister Abe to President George W.
1994 Bush, April 3, 2007).

On the issue of wartime comfort women, which I have to express sympathy from the bottom of
seriously stained the honor and dignity of many my heart to those people who were taken as
women, I would like to take this opportunity once wartime comfort women. As a human being, I
again to express my profound and sincere would like to express my sympathies, and also
remorse and apologies as prime minister of Japan I need to apologize to
them. My administration has been saying all
along that we continue to stand by the Kono
c) Letters from the Prime Minister of Japan to Individual Comfort Statement. We feel responsible for having
Women forced these women to go through that hardship
The issue of comfort women, with the and pain as comfort women under the
involvement of the Japanese military authorities circumstances at the time. (Excerpt from an
at that time, was a grave affront to the honor and interview article "A Conversation with Shinzo
dignity of a large number of women. Abe" by the Washington Post, April 22, 2007).

As Prime Minister of Japan, I thus extend anew x x x both personally and as Prime Minister of
my most sincere apologies and remorse to all Japan, my heart goes out in sympathy to all
FINALS CONSTITUTION I ACJUCO 316

those who suffered extreme hardships as unquestioning adherence to a political decision


comfort women; and I expressed my apologies already made; or the potentiality of
for the fact that they were forced to endure such embarrassment from multifarious
extreme and harsh conditions. Human rights are pronouncements by various departments on
violated in many parts of the world during the question.
20th Century; therefore we must work to make
the 21st Century a wonderful century in which no
human rights are violated. And the Government In Taada v. Cuenco,[40] we held that political questions refer "to those
of Japan and I wish to make significant questions which, under the Constitution, are to be decided by the
contributions to that end. (Excerpt from Prime people in their sovereign capacity, or in regard to which full
Minister Abe's remarks at the Joint Press discretionary authority has been delegated to the legislative or
Availability after the summit meeting at Camp executive branch of the government. It is concerned with issues
David between Prime Minister Abe and dependent upon the wisdom, not legality of a particular measure."
President Bush, April 27, 2007).
Certain types of cases often have been found to present political
questions.[41] One such category involves questions of foreign
The Asian Women's Fund relations. It is well-established that "[t]he conduct of the foreign
relations of our government is committed by the Constitution to the
Established by the Japanese government in 1995, the executive and legislative--'the political'--departments of the
AWF represented the government's concrete attempt to address its government, and the propriety of what may be done in the exercise of
moral responsibility by offering monetary compensation to victims of this political power is not subject to judicial inquiry or decision."[42] The
the comfort women system.[37] The purpose of the AWF was to show US Supreme Court has further cautioned that decisions relating to
atonement of the Japanese people through expressions of apology foreign policy
and remorse to the former wartime comfort women, to restore their
honor, and to demonstrate Japans strong respect for women.[38] are delicate, complex, and involve large
elements of prophecy. They are and should be
The AWF announced three programs for former comfort women who undertaken only by those directly responsible to
applied for assistance: (1) an atonement fund paying 2 million the people whose welfare they advance or
(approximately $20,000) to each woman; (2) medical and welfare imperil. They are decisions of a kind for which
support programs, paying 2.5-3 million ($25,000-$30,000) for each the Judiciary has neither aptitude, facilities nor
woman; and (3) a letter of apology from the Japanese Prime Minister responsibility.[43]
to each woman. Funding for the program came from the Japanese
government and private donations from the Japanese people. As of
March 2006, the AWF provided 700 million (approximately $7 million) To be sure, not all cases implicating foreign relations present political
for these programs in South Korea, Taiwan, and the Philippines; 380 questions, and courts certainly possess the authority to construe or
million (approximately $3.8 million) in Indonesia; and 242 million invalidate treaties and executive agreements.[44]However, the
(approximately $2.4 million) in the Netherlands. question whether the Philippine government should espouse claims
of its nationals against a foreign government is a foreign relations
On January 15, 1997, the AWF and the Philippine government signed matter, the authority for which is demonstrably committed by our
a Memorandum of Understanding for medical and welfare support Constitution not to the courts but to the political branches. In this case,
programs for former comfort women. Over the next five years, these the Executive Department has already decided that it is to the best
were implemented by the Department of Social Welfare and interest of the country to waive all claims of its nationals for reparations
Development. against Japan in the Treaty of Peace of 1951. The wisdom of such
decision is not for the courts to question. Neither could petitioners
Our Ruling herein assail the said determination by the Executive
Department via the instant petition for certiorari.
Stripped down to its essentials, the issue in this case is
whether the Executive Department committed grave abuse of In the seminal case of US v. Curtiss-Wright Export Corp.,[45] the US
discretion in not espousing petitioners claims for official apology and Supreme Court held that [t]he President is the sole organ of the nation
other forms of reparations against Japan. in its external relations, and its sole representative with foreign
relations.
The petition lacks merit.
It is quite apparent that if, in the maintenance of
From a Domestic Law Perspective, the Executive Department our international relations, embarrassment --
has the exclusive prerogative to determine whether to espouse perhaps serious embarrassment -- is to be
petitioners claims against Japan. avoided and success for our aims achieved,
congressional legislation which is to be made
effective through negotiation and inquiry within
Baker v. Carr[39] remains the starting point for analysis the international field must often accord to the
under the political question doctrine. There the US Supreme Court President a degree of discretion and freedom
explained that: from statutory restriction which would not be
admissible where domestic affairs alone
x x x Prominent on the surface of any case held involved. Moreover, he, not Congress, has the
to involve a political question is found a textually better opportunity of knowing the conditions
demonstrable constitutional commitment of the which prevail in foreign countries, and especially
issue to a coordinate political department or a is this true in time of war. He has his confidential
lack of judicially discoverable and manageable sources of information. He has his agents in the
standards for resolving it, or the impossibility of form of diplomatic, consular and other officials. x
deciding without an initial policy determination of xx
a kind clearly for non-judicial discretion; or the
impossibility of a court's undertaking
independent resolution without expressing lack This ruling has been incorporated in our jurisprudence through Bayan
of the respect due coordinate branches of v.
government; or an unusual need for
FINALS CONSTITUTION I ACJUCO 317

Executive Secretary[46] and Pimentel v. Executive Secretary;[47] its property confiscated, or extinguished, during the
overreaching principle was, perhaps, best articulated in (now Chief) war, by any of the United States, could only be
Justice Punos dissent in Secretary of Justice v. Lantion:[48] provided for by the treaty of peace; and if there
had been no provision, respecting these
x x x The conduct of foreign relations is full of subjects, in the treaty, they could not be agitated
complexities and consequences, sometimes after the treaty, by the British government, much
with life and death significance to the nation less by her subjects in courts of justice.
especially in times of war. It can only be (Emphasis supplied).
entrusted to that department of government
which can act on the basis of the best available
information and can decide with decisiveness. x This practice of settling claims by means of a peace treaty is certainly
x x It is also the President who possesses the nothing new. For instance, in Dames & Moore v. Regan,[51] the US
most comprehensive and the most confidential Supreme Court held:
information about foreign countries for our
diplomatic and consular officials regularly brief Not infrequently in affairs between nations,
him on meaningful events all over the world. He outstanding claims by nationals of one country
has also unlimited access to ultra-sensitive against the government of another country are
military intelligence data.In fine, the presidential sources of friction between the two
role in foreign affairs is dominant and the sovereigns. United States v. Pink, 315 U.S. 203,
President is traditionally accorded a wider 225, 62 S.Ct. 552, 563, 86 L.Ed. 796 (1942). To
degree of discretion in the conduct of foreign resolve these difficulties, nations have often
affairs. The regularity, nay, validity of his actions entered into agreements settling the claims of
are adjudged under less stringent standards, their respective nationals. As one treatise writer
lest their judicial repudiation lead to breach of an puts it, international agreements settling claims
international obligation, rupture of state relations, by nationals of one state against the government
forfeiture of confidence, national of another are established international practice
embarrassment and a plethora of other reflecting traditional international theory. L.
problems with equally undesirable Henkin, Foreign Affairs and the Constitution 262
consequences. (1972). Consistent with that principle, the United
States has repeatedly exercised its sovereign
authority to settle the claims of its nationals
The Executive Department has determined that taking up petitioners against foreign countries. x x x Under such
cause would be inimical to our countrys foreign policy interests, and agreements, the President has agreed to
could disrupt our relations with Japan, thereby creating serious renounce or extinguish claims of United States
implications for stability in this region. For us to overturn the Executive nationals against foreign governments in return
Departments determination would mean an assessment of the foreign for lump-sum payments or the establishment of
policy judgments by a coordinate political branch to which authority to arbitration procedures. To be sure, many of
make that judgment has been constitutionally committed. these settlements were encouraged by
the United States claimants themselves, since a
In any event, it cannot reasonably be maintained that the Philippine claimant's only hope of obtaining any payment
government was without authority to negotiate the Treaty of Peace at all might lie in having his Government
with Japan. And it is equally true that, since time immemorial, when negotiate a diplomatic settlement on his behalf.
negotiating peace accords and settling international claims: But it is also undisputed that the United States
has sometimes disposed of the claims of its
x x x [g]overnments have dealt with x x x private citizens without their consent, or even without
claims as their own, treating them as national consultation with them, usually without exclusive
assets, and as counters, `chips', in international regard for their interests, as distinguished from
bargaining. Settlement agreements have those of the nation as a whole. Henkin, supra, at
lumped, or linked, claims deriving from private 262-263. Accord, Restatement (Second) of
debts with others that were intergovernmental in Foreign Relations Law of the United States 213
origin, and concessions in regard to one (1965) (President may waive or settle a claim
category of claims might be set off against against a foreign state x x x [even] without the
concessions in the other, or against larger consent of the [injured] national). It is clear that
political considerations unrelated to debts.[49] the practice of settling claims continues today.

Indeed, except as an agreement might otherwise provide, Respondents explain that the Allied Powers concluded the Peace
international settlements generally wipe out the underlying private Treaty with Japan not necessarily for the complete atonement of the
claims, thereby terminating any recourse under domestic law. In Ware suffering caused by Japanese aggression during the war, not for the
v. Hylton,[50] a case brought by a British subject to recover a debt payment of adequate reparations, but for security purposes. The
confiscated by the Commonwealth of Virginia during the war, Justice treaty sought to prevent the spread of communism in Japan, which
Chase wrote: occupied a strategic position in the Far East. Thus, the Peace Treaty
compromised individual claims in the collective interest of the free
I apprehend that the treaty of peace abolishes world.
the subject of the war, and that after peace is
concluded, neither the matter in dispute, nor the This was also the finding in a similar case involving American victims
conduct of either party, during the war, can ever of Japanese slave labor during the war.[52] In a consolidated case in
be revived, or brought into contest again. All the Northern District of California,[53] the court dismissed the lawsuits
violences, injuries, or damages sustained by the filed, relying on the 1951 peace treaty with Japan,[54] because of the
government, or people of either, during the war, following policy considerations:
are buried in oblivion; and all those things are
implied by the very treaty of peace; and therefore The official record of treaty negotiations
not necessary to be expressed. Hence it follows, establishes that a fundamental goal of the
that the restitution of, or compensation for, British agreement was to settle the reparations issue
FINALS CONSTITUTION I ACJUCO 318

once and for all. As the statement of the flourish. In short, [it] would
chief United States negotiator, John Foster be contrary to the basic
Dulles, makes clear, it was well understood purposes and policy of x x
that leaving open the possibility of future x the United States x x x.
claims would be an unacceptable
impediment to a lasting peace: We thus hold that, from a municipal law perspective, that certiorari will
not lie. As a general principle and particularly here, where such an
Reparation is usually the extraordinary length of time has lapsed between the treatys
most controversial aspect conclusion and our consideration the Executive must be given ample
of peacemaking. The discretion to assess the foreign policy considerations of espousing a
present peace is no claim against Japan, from the standpoint of both the interests of the
exception. petitioners and those of the Republic, and decide on that basis if
apologies are sufficient, and whether further steps are appropriate or
On the one hand, there necessary.
are claims both vast and
just. Japan's aggression The Philippines is not under any international obligation to
caused tremendous cost, espouse petitioners claims.
losses and suffering.

On the other hand, to meet In the international sphere, traditionally, the only means available for
these claims, there stands individuals to bring a claim within the international legal system has
a Japan presently been when the individual is able to persuade a government to bring a
reduced to four home claim on the individuals behalf.[55] Even then, it is not the individuals
islands which are unable rights that are being asserted, but rather, the states own
to produce the food its rights. Nowhere is this position more clearly reflected than in the
people need to live, or the dictum of the Permanent Court of International Justice (PCIJ) in the
raw materials they need to 1924 Mavrommatis Palestine Concessions Case:
work. x x x
By taking up the case of one of its subjects and
The policy of the United States that Japanese by resorting to diplomatic action or international
liability for reparations should be sharply limited judicial proceedings on his behalf, a State is in
was informed by the experience of six years of reality asserting its own right to ensure, in the
United States-led occupation of Japan. During person of its subjects, respect for the rules of
the occupation the Supreme Commander of the international law. The question, therefore,
Allied Powers (SCAP) for the region, General whether the present dispute originates in an
Douglas MacArthur, confiscated Japanese injury to a private interest, which in point of fact is
assets in conjunction with the task of managing the case in many international disputes, is
the economic affairs of the vanquished nation irrelevant from this standpoint. Once a State has
and with a view to reparations payments. It taken up a case on behalf of one of its subjects
soon became clear that Japan's financial before an international tribunal, in the eyes of the
condition would render any aggressive latter the State is sole claimant.[56]
reparations plan an exercise in futility. Since the exercise of diplomatic protection is the right of the
Meanwhile, the importance of a stable, State, reliance on the right is within the absolute discretion of states,
democratic Japan as a bulwark to and the decision whether to exercise the discretion may invariably be
communism in the region increased. At the influenced by political considerations other than the legal merits of the
end of 1948, MacArthur expressed the view that particular claim.[57] As clearly stated by the ICJ in
[t]he use of reparations as a weapon to retard Barcelona Traction:
the reconstruction of a viable economy in Japan
should be combated with all possible means The Court would here observe that, within the
and recommended that the reparations issue be limits prescribed by international law, a State
settled finally and without delay. may exercise diplomatic protection by
whatever means and to whatever extent it
That this policy was embodied in the treaty is thinks fit, for it is its own right that the State
clear not only from the negotiations history but is asserting. Should the natural or legal
also from the Senate Foreign Relations person on whose behalf it is acting consider
Committee report recommending approval of that their rights are not adequately
the treaty by the Senate. The committee noted, protected, they have no remedy in
for example: international law. All they can do is resort to
national law, if means are available, with a view
Obviously insistence upon to furthering their cause or obtaining redress.
the payment of The municipal legislator may lay upon the State
reparations in any an obligation to protect its citizens abroad, and
proportion commensurate may also confer upon the national a right to
with the claims of the demand the performance of that obligation, and
injured countries and their clothe the right with corresponding sanctions.
nationals would However, all these questions remain within the
wreck Japan's economy, province of municipal law and do not affect the
dissipate any credit that it position internationally.[58] (Emphasis supplied)
may possess at present,
destroy the initiative of its
people, and create misery The State, therefore, is the sole judge to decide whether its
and chaos in which the protection will be granted, to what extent it is granted, and when will it
seeds of discontent and cease. It retains, in this respect, a discretionary power the exercise of
communism would
FINALS CONSTITUTION I ACJUCO 319

which may be determined by considerations of a political or other person, including protection from slavery and
nature, unrelated to the particular case. racial discrimination. Some of the corresponding
rights of protection have entered into the body of
The International Law Commissions (ILCs) Draft Articles on general international law others are conferred by
Diplomatic Protection fully support this traditional view. They (i) state international instruments of a universal or quasi-
that "the right of diplomatic protection belongs to or vests in the universal character.
State,[59] (ii) affirm its discretionary nature by clarifying that diplomatic
protection is a "sovereign prerogative" of the State;[60] and (iii) stress
that the state "has the right to exercise diplomatic protection The Latin phrase, erga omnes, has since become one of the rallying
on behalf of a national. It is under no duty or obligation to do so."[61] cries of those sharing a belief in the emergence of a value-based
international public order. However, as is so often the case, the reality
It has been argued, as petitioners argue now, that the State is neither so clear nor so bright. Whatever the relevance of
has a duty to protect its nationals and act on his/her behalf when rights obligations erga omnes as a legal concept, its full potential remains to
are injured.[62] However, at present, there is no sufficient evidence to be realized in practice.[69]
establish a general international obligation for States to exercise The term is closely connected with the international law concept of jus
diplomatic protection of their own nationals abroad.[63] Though, cogens. In international law, the term jus cogens (literally, compelling
perhaps desirable, neither state practice nor opinio juris has evolved law) refers to norms that command peremptory authority, superseding
in such a direction. If it is a duty internationally, it is only a moral and conflicting treaties and custom. Jus cogens norms are considered
not a legal duty, and there is no means of enforcing its fulfillment.[64] peremptory in the sense that they are mandatory, do not admit
derogation, and can be modified only by general international norms
We fully agree that rape, sexual slavery, torture, and sexual violence of equivalent authority.[70]
are morally reprehensible as well as legally prohibited under
contemporary international law.[65] However, petitioners take quite a Early strains of the jus cogens doctrine have existed since the
theoretical leap in claiming that these proscriptions automatically imply 1700s,[71] but peremptory norms began to attract greater scholarly
that that the Philippines is under a non-derogable obligation to attention with the publication of Alfred von Verdross's influential 1937
prosecute international crimes, particularly since petitioners do not article, Forbidden Treaties in International Law.[72] The recognition
demand the imputation of individual criminal liability, but seek to of jus cogens gained even more force in the 1950s and 1960s with the
recover monetary reparations from the state of Japan. Absent the ILCs preparation of the Vienna Convention on the Law of Treaties
consent of states, an applicable treaty regime, or a directive by the (VCLT).[73] Though there was a consensus that certain international
Security Council, there is no non-derogable duty to institute norms had attained the status of jus cogens,[74] the ILC was unable to
proceedings against Japan. Indeed, precisely because of states reach a consensus on the proper criteria for identifying peremptory
reluctance to directly prosecute claims against another state, norms.
recent developments support the modern trend to empower After an extended debate over these and other theories of jus cogens,
individuals to directly participate in suits against perpetrators of the ILC concluded ruefully in 1963 that there is not as yet any generally
international crimes.[66]Nonetheless, notwithstanding an array of accepted criterion by which to identify a general rule of international
General Assembly resolutions calling for the prosecution of crimes law as having the character of jus cogens.[75] In a commentary
against humanity and the strong policy arguments warranting such a accompanying the draft convention, the ILC indicated that the prudent
rule, the practice of states does not yet support the present existence course seems to be to x x x leave the full content of this rule to be
of an obligation to prosecute international crimes.[67] Of course a worked out in State practice and in the jurisprudence of international
customary duty of prosecution is ideal, but we cannot find enough tribunals.[76] Thus, while the existence of jus cogens in international
evidence to reasonably assert its existence. To the extent that any law is undisputed, no consensus exists on its substance,[77] beyond a
state practice in this area is widespread, it is in the practice of granting tiny core of principles and rules.[78]
amnesties, immunity, selective prosecution, or de facto impunity to
those who commit crimes against humanity.[68] Of course, we greatly sympathize with the cause of
petitioners, and we cannot begin to comprehend the unimaginable
Even the invocation of jus cogens norms and erga omnes obligations horror they underwent at the hands of the Japanese soldiers. We are
will not alter this analysis. Even if we sidestep the question of also deeply concerned that, in apparent contravention of fundamental
whether jus cogens norms existed in 1951, petitioners have not principles of law, the petitioners appear to be without a remedy to
deigned to show that the crimes committed by the Japanese army challenge those that have offended them before appropriate
violated jus cogens prohibitions at the time the Treaty of Peace was fora. Needless to say, our government should take the lead in
signed, or that the duty to prosecute perpetrators of international protecting its citizens against violation of their fundamental human
crimes is an erga omnes obligation or has attained the status of jus rights. Regrettably, it is not within our power to order the Executive
cogens. Department to take up the petitioners cause. Ours is only the power
to urge and exhort the Executive Department to take up petitioners
The term erga omnes (Latin: in relation to everyone) in international cause.
law has been used as a legal term describing obligations owed by
States towards the community of states as a whole. The concept was WHEREFORE, the Petition is hereby DISMISSED.
recognized by the ICJ in Barcelona Traction: SO ORDERED.

x x x an essential distinction should be drawn


between the obligations of a State towards the
international community as a whole, and those
arising vis--vis another State in the field of
diplomatic protection. By their very nature, the
former are the concern of all States. In view of
the importance of the rights involved, all States
can be held to have a legal interest in their
protection; they are obligations erga omnes.

Such obligations derive, for example, in


contemporary international law, from the
outlawing of acts of aggression, and of
genocide, as also from the principles and rules
concerning the basic rights of the human
FINALS CONSTITUTION I ACJUCO 320

G.R. No. L-14279 October 31, 1961 Customs vs. Serree Investment Co., L-12007 [May 16, 1960];
Commissioner of Customs vs. Serree Investment Co., L-14274
THE COMMISSIONER OF CUSTOMS and THE [November 29, 1960]), for the reason that the broad powers of
the Central Bank, under its charter, to maintain our monetary
COLLECTOR OF CUSTOMS, petitioners,
vs. stability and to preserve the international value of our currency,
EASTERN SEA TRADING, respondent. under section 2 of Republic Act No. 265, in relation to section
14 of said Act — authorizing the bank to issue such rules and
regulations as it may consider necessary for the effective
Office of the Solicitor General for petitioners. discharge of the responsibilities and the exercise of the powers
Valentin Gutierrez for respondent. assigned to the Monetary Board and to the Central Bank —
connote the authority to regulate no-dollar imports, owing to
CONCEPCION, J.: the influence and effect that the same may and do have upon
the stability of our peso and its international value.
Petition for review of a judgment of the Court of Tax Appeals
reversing a decision of the Commissioner of Customs. The Court of Tax Appeals entertained doubts on the legality of
the executive agreement sought to be implemented by
Executive Order No. 328, owing to the fact that our Senate had
Respondent Eastern Sea Trading was the consignee of not concurred in the making of said executive agreement. The
several shipments of onion and garlic which arrived at the Port concurrence of said House of Congress is required by our
of Manila from August 25 to September 7, 1954. Some fundamental law in the making of "treaties" (Constitution of the
shipments came from Japan and others from Hong Kong. In as Philippines, Article VII, Section 10[7]), which are, however,
much as none of the shipments had the certificate required by distinct and different from "executive agreements," which may
Central Bank Circulars Nos. 44 and 45 for the release thereof, be validly entered into without such concurrence.
the goods thus imported were seized and subjected to
forfeiture proceedings for alleged violations of section 1363(f)
of the Revised Administrative Code, in relation to the Treaties are formal documents which require
aforementioned circulars of the Central Bank. In due course, ratification with the approval of two thirds of the
the Collector of Customs of Manila rendered a decision on Senate. Executive agreements become binding
September 4, 1956, declaring said goods forfeited to the through executive action without the need of a vote
Government and — the goods having been, in the meantime, by the Senate or by Congress.
released to the consignees on surety bonds, filed by the same,
as principal, and the Alto Surety & Insurance Co., Inc., as xxx xxx xxx
surety, in compliance with orders of the Court of First Instance
of Manila, in Civil Cases Nos. 23942 and 23852 thereof —
. . . the right of the Executive to enter into binding
directing that the amounts of said bonds be paid, by said
principal and surety, jointly and severally, to the Bureau of agreements without the necessity of subsequent
Customs, within thirty (30) days from notice. Congressional approval has been confirmed by long
usage. From the earliest days of our history we have
entered into executive agreements covering such
On appeal taken by the consignee, said decision was affirmed subjects as commercial and consular relations, most-
by the Commissioner of Customs on December 27, 1956. favored-nation rights, patent rights, trademark and
Subsequently, the consignee sought a review of the decision copyright protection, postal and navigation
of said two (2) officers by the Court of Tax Appeals, which arrangements and the settlement of claims. The
reversed the decision of the Commissioner of Customs and validity of these has never been seriously questioned
ordered that the aforementioned bonds be cancelled and by our courts.
withdrawn. Hence, the present petition of the Commissioner of
Customs for review of the decision of the Court of Tax Appeals.
xxx xxx xxx

The latter is based upon the following premises, namely: that


the Central Bank has no authority to regulate transactions not Agreements with respect to the registration of trade-
involving foreign exchange; that the shipments in question are marks have been concluded by the Executive with
in the nature of "no-dollar" imports; that, as such, the various countries under the Act of Congress of March
3, 1881 (21 Stat. 502). Postal conventions regulating
aforementioned shipments do not involve foreign exchange;
that, insofar as a Central Bank license and a certificate the reciprocal treatment of mail matters, money
orders, parcel post, etc., have been concluded by the
authorizing the importation or release of the goods under
consideration are required by Central Bank Circulars Nos. 44 Postmaster General with various countries under
and 45, the latter are null and void; and that the seizure and authorization by Congress beginning with the Act of
forfeiture of the goods imported from Japan cannot be justified February 20, 1792 (1 Stat. 232, 239). Ten executive
under Executive Order No. 328,1 not only because the same agreements were concluded by the President
seeks to implement an executive agreement2 — extending the pursuant to the McKinley Tariff Act of 1890 (26 Stat.
effectivity of our3 Trades and Financial Agreements4 with 567, 612), and nine such agreements were entered
into under the Dingley Tariff Act 1897 (30 Stat. 151,
Japan — which (executive agreement), it believed, is of
dubious validity, but, also, because there is no governmental 203, 214). A very much larger number of
agreements, along the lines of the one with Rumania
agency authorized to issue the import license required by the
aforementioned executive order. previously referred to, providing for most-favored-
nation treatment in customs and related matters have
been entered into since the passage of the Tariff Act
The authority of the Central Bank to regulate no-dollar imports of 1922, not by direction of the Act but in harmony
and the validity of the aforementioned Circulars Nos. 44, and with it.
45 have already been passed upon and repeatedly upheld by
this Court (Pascual vs. Commissioner of Customs, L-10979
[June 30, 1959]; Acting Commissioner of Customs vs. xxx xxx xxx
Leuterio, L-9142 [October 17, 1959] Commissioner of Customs
vs. Pascual, L-9836 [November 18, 1959]; Commissioner of
FINALS CONSTITUTION I ACJUCO 321

International agreements involving political issues or The validity of the executive agreement in question is thus
changes of national policy and those involving patent. In fact, the so-called Parity Rights provided for in the
international arrangements of a permanent character Ordinance Appended to our Constitution were, prior thereto,
usually take the form of treaties. But international the subject of an executive agreement, made without the
agreements embodying adjustments of concurrence of two-thirds (2/3) of the Senate of the United
detail carrying out well-established national policies States.
and traditions and those involving arrangements of a
more or less temporary nature usually take the form Lastly, the lower court held that it would be unreasonable to
of executive agreements. require from respondent-appellee an import license when the
Import Control Commission was no longer in existence and,
xxx xxx xxx hence, there was, said court believed, no agency authorized to
issue the aforementioned license. This conclusion is
Furthermore, the United States Supreme Court has untenable, for the authority to issue the aforementioned
expressly recognized the validity and constitutionality licenses was not vested exclusively upon the Import Control
of executive agreements entered into without Senate Commission or Administration. Executive Order No. 328
approval. (39 Columbia Law Review, pp. 753-754) provided for export or import licenses "from the Central Bank
of the Philippines or the Import Control Administration" or
(See, also, U.S. vs. Curtis-Wright Export
Corporation, 299 U.S. 304, 81 L. ed. 255; U.S. vs. Commission. Indeed, the latter was created only to perform the
task of implementing certain objectives of the Monetary Board
Belmont, 301 U.S. 324, 81 L. ed. 1134; U.S. vs. Pink,
315 U.S. 203, 86 L. ed. 796; Ozanic vs. U.S., 188 F. and the Central Bank, which otherwise had to be undertaken
2d. 288; Yale Law Journal, Vol. 15, pp. 1905-1906; by these two (2) agencies. Upon the abolition of said
California Law Review, Vol. 25, pp. 670-675; Hyde Commission, the duty to provide means and ways for the
on International Law [Revised Edition], Vol. 2, pp. accomplishment of said objectives had merely to be
1405, 1416-1418; Willoughby on the U.S. discharged directly by the Monetary Board and the Central
Constitutional Law, Vol. I [2d ed.], pp. 537-540; Bank, even if the aforementioned Executive Order had been
Moore, International Law Digest, Vol. V, pp. 210-218; silent thereon.
Hackworth, International Law Digest, Vol. V, pp. 390-
407). (Emphasis supplied.) WHEREFORE, the decision appealed from is hereby reversed
and another one shall be entered affirming that of the
In this connection, Francis B. Sayre, former U.S. High Commissioner of Customs, with cost against respondents
Commissioner to the Philippines, said in his work on "The defendant-appellee, Eastern Sea Trading. It is so ordered.
Constitutionality of Trade Agreement Acts":

Agreements concluded by the President which fall


short of treaties are commonly referred to as
executive agreements and are no less common in
our scheme of government than are the more formal
instruments — treaties and conventions. They
sometimes take the form of exchanges of notes and
at other times that of more formal documents
denominated "agreements" time or "protocols". The
point where ordinary correspondence between this
and other governments ends and agreements —
whether denominated executive agreements or
exchanges of notes or otherwise — begin, may
sometimes be difficult of ready ascertainment. It
would be useless to undertake to discuss here the
large variety of executive agreements as such,
concluded from time to time. Hundreds of executive
agreements, other than those entered into under the
trade-agreements act, have been negotiated with
foreign governments. . . . It would seem to be
sufficient, in order to show that the trade agreements
under the act of 1934 are not anomalous in
character, that they are not treaties, and that they
have abundant precedent in our history, to refer to
certain classes of agreements heretofore entered
into by the Executive without the approval of the
Senate. They cover such subjects as the inspection
of vessels, navigation dues, income tax on shipping
profits, the admission of civil aircraft, customs
matters, and commercial relations generally,
international claims, postal matters, the registration
of trademarks and copyrights, etcetera. Some of
them were concluded not by specific congressional
authorization but in conformity with policies declared
in acts of Congress with respect to the general
subject matter, such as tariff acts; while still others,
particularly those with respect of the settlement of
claims against foreign governments, were concluded
independently of any legislation." (39 Columbia Law
Review, pp. 651, 755.)
FINALS CONSTITUTION I ACJUCO 322

G.R. No. L-23846 September 9, 1977 The court, citing the said obiter dictum in the Qua Chee Gan
case, held that mere ion of forged dollar checks is not a ground
GO TEK petitioner-appelle, for deportation under the Immigration Law; that under section
37(3) of the law before an alien may be deported for having
vs.
DEPORTATION BOARD, respondent-appellant. been convicted and sentenced to imprisonment for a term of
one year or more for a crime involving moral turpitude a
conviction is and that since Go Tek had not been convicted of
Teodoro C. Ronquillo for appelle. the offense punished in article 168, the deportation was
premature.
Solicitor-General Arturo A. Alafriz, Assistant Solicitor general
Felicisimo R. Rosete and Solicitor Octavio R. Ramirez for The Board appealed to this Court on the ground that the
appellant. decision is contrary to law. The Solicitor General contends that
the trial court erred in assuming that the President may deport
AQUINO, J.: undesirable aliens only to grounds enumerated by law; in
holding that mere possession of forged dollar checks is not a
ground for deportation and that a criminal conviction is
This is a deportation case. On March 3, 1964 the chief necessary, and in not finding that the Board has jurisdiction
prosecutor of the Deportation Board filed a complaint against over Go Tek's case.
Go Tek a Chinaman residing at Ilagan, Isabela and 1208-B,
Misericordia Street, Sta. Cruz Manila.
The parties stipulated that the Deportation Board is an of the
President of the Philippines charged with the investigation of
It was alleged in the complaint that in December, 1963 certain undesirable aliens and to report and recommend proper action
agents of the National Bureau of Investigation (NBI) searched on the basis of its findings therein."
an office located at 1439 O'Donnel Street, Sta. Cruz, Manila
believed to be the headquarters of a guerilla unit of the
"Emergency Intelligence Section, Army of the United States" The issue is whether the Deportation Board can entertain a
and that among those arrested thereat was Go Tek an alleged deportation proceeding based on a ground which is not
sector commander and intelligence and record officer of that specified in section 37 of the Immigration Law and although the
guerilla unit. aliens has not yet been convicted of the offense imputed to
him.
It was further alleged that fake dollar checks were found in Go
Tek's possession and that, therefore, he had violated article We hold that the Board has jurisdiction to investigate Go Tek
168 of the Revised Penal Code and rendered himself an for illegal possession of fake dollar checks (as well as his
undesirable alien. alleged "guerilla" activities) in spite of the fact that he has not
yet been convicted of illegal possession thereof under article
168 of the Revised Penal Code and notwithstanding that act is
The prosecutor prayed that after trial the Board should not the grounds for the deportation of undesirable aliens as
recommend to the President of the Philippines the . immediate enumerated in section 37 of the Immigration Law. The charge
deportation of Go Tek as an undesirable alien, "his presence against Go Tek before- the Board was not premature.
in this country having been, and will always be and a menace
to the peace , welfare, and security of the community". Case
No. R1116). The aforementioned obiter dictum the Qua Chee Gan case
invoked by Go Tek and relied upon by the trial court, is not of
this case. In the Qua Chee Gan case the aliens were with
Go Tek filed a motion to dismiss on the ground that the economic sabotage which is a ground for deportation under
complaint was premature bemuse them was a pending case Republic Act No. 503.
against him in the city fiscal's office of Manila for violation of
article 168 (IS 64-7267). He contended that the board had no
jurisdiction to try the case in view of the obiter dictum in Qua The ratio decidendi of the Qua Chee Gan case is that the
Chee Gan vs. Deportation Board, 118 Phil. 868, 875, that the t provision of Executive Order No. 398, of 1951, the Deportation
may deport aliens only on the grounds specified in the law. Board to issue a warrant of arrest upon the filing of formal
charges against an alien, is "illegal" or unconstitutional
because it is contrary to the provinsion in section 1(3), Article
The Board, composed of Manuel A. Concordia, Arturo A. III of the 1935 Constitution that warrants shall issue upon to be
Alafriz and Manuel V. Reyes, in its resolution of April 21, 1964 de by the judge after e under oath the t and the witness he may
denied Go Teks motion. The Board reasoned out that a produce. (Note that under section 3, Article IV of the 1973
conviction is not a prerequisite before the State my exercise its Constitution cause may be determined "by the judge, or such
rights to deport an undesirable alien and that the Board is only other responsible officer as may be authorized by law". See
a fact finding body whose function is to make a report and Santos vs. Commissioner of Immigration L-25694, November
recommendation to the President in whom is lodged the 29, 1976, 74 SCRA 96, per Fernando, J.)
exclusive power to deport an alien or a deportation proceeding.
A thorough comprehension of the President's power to deport
In view of the denial of his motion to quash, Go Tek on June aliens may show the baselessness of the instant prohibition
10, 1964 filed in the Court of First Instance of Manila a action of Go Tek The President's power to deport aliens and
prohibition action against the Board. On July 8, 1964 the court the investigation of aliens subject to deportation are provided
issued a writ of preliminary injunction restraining the board for in the following provisions of the Revised Administrative
from hearing Go Tek's case. Code:

After hearing, the trial court (Judge Federico C. Alikpala SEC. 69. Deportation of subject of foreign
presiding) in its decision of 31, 1964 granted the writ prohibition power. — A subject of a foreign power
and the Board to desist from taking cognizance of the Go Tek. residing in the Philippine Islands shall not
be deported expelled, or excluded from
said Islands or repatriated to his own
FINALS CONSTITUTION I ACJUCO 323

country by the Governor-General except country is so undesirable as to affect or injure the security
upon prior investigator, conducted by said welfare or interest of the state. The adjudication of facts upon
Executive or his authorized agent, of the which deportation is predicated also devolves on the Chief
ground upon which such action is Executive whose decision is final and executory." (Tan Tong
contemplated. In such case the person vs. Deportation Board 96 Phil. 934, 936; Tan Sin vs.
concerned shall he informed of the charge Deportation Board, 104 PhiL 868, 872).
or charges against him and he shall be
allowed not less than three days for the It has been held that the Chief Executive is the sole and
preparation of his defense. He shall also exclusive judge of the existence of facts which warrant the
have the right to be heard by himself or deportation of aliens as disclosed in an investigation
counsel, to produce witnesses in his own conducted in accordance with 69. No other tribunal is at liberty
behalf, and to cross-examine the opposing to reexamine or to controvert the sufficiency of the evidence on
witnesses.
which he acted. (Martin vs. Mott 12 Wheat., 19, 31, cited in In
re McCulloch Dick, 38 Phil. 41, 62).
On the other hand, section 37 of the Immigration Law Provides
that certain aliens may be arrested upon the warrant of the In the Dick case it was noted "that every alien forfeits his right
Commissioner of Immigration or of any other officer designated
to asylum in the country in which he resides, in the absence of
by him for the purpose and deported upon the Commissioner's treaty provisions to the contrary when his conduct or his mode
warrant - "after a determination by the Board of Commissioners
of life renders his prance there inimical to the public interests".
of the existence of the ground for deportation as charged "The reasons may be summed up and co in a single word: the
against the alien." Thirteen classes of aliens who may be public interest of the State ." (38 Phil. 41, 47, 100).
deported by the Commissioner are specified in section 37 (See
Po Siok Pin vs. Vivo, L-24792, February 14, 1975, 62 SCRA
363, 368). "It is fundamental that an executive order for deportation is not
dependent on a prior judicial conviction in a case" (Ang Bong
vs. Commissioner of Immigration, 100 Phil. 801, 803). Thus, it
So, under existing law; the deportation of an undesirable alien was held that the fact that an alien has been acquitted in a of
may be effected (1) by order of the President, after due the charge does not prevent the deportation of such alien
investigation, pursuant to section 69 of the Revised based on the same charge. Such acquittal does not
Administrative Code and (2) by the Commissioner of constitute res judicata in the deportation proceedings.
Immigration upon recommendation of the Board of Conviction of a crime is not n to warrant deportation. (3 C.J.S.
Commissioners under section 37 of the immigration Law (Qua 743, note 40, citing Lewis vs. Frick, 233 U.S. 291, 58 L. Ed.
Chee Gan vs- Deportation Board, supra).
967 and U.S. ex.. Mastoras vs. McCandless 61 F. 2nd 366;
Tama Miyake vs. U.S. 257 F. 732).
The State has the inherent power to deport undesirable aliens
(Chuoco Tiaco vs. Forbes, 228 U.S. 549, 57 L. Ed. 960, 40 And in the Tan Tong case, supra, it was ruled that the
Phil. 1122, 1125). That power may be exercise by the Chief
Deportation Board could take cognizance of the charge of
Executive "when he deems such action necessary for the illegal importation against an alien as a ground for deportation,
peace and domestic tranquility of the nation Justice Johnson's
even if he of the Deportation Board is merely recommendatory.
is that there the Chief Executive rinds that there are aliens The Chief Executive has to approve the board's
whose continued in the country is injurious to the public interest recommendatory Abuses or rents committed by the prosecutor
he may, even in the absence of express law, deport them or by the Board should first be brought to his attention.
(Forbes vs. Chuoco Tiaco and Crossfield 16 Phil. 534, 568,
569; In re McCulloch Dick, 38 Phil. 41).
WHEREFORE, the lower court's decision is reversed and set
aside. The writ of preliminary injunction is dissolved. The case
The right of a country to expel or deport is to the Deportation Board for further proceedings. Costs
aliens because their continued presence is against the petitioner-appellee.
rental to public welfare is absolute and
unqualified (Tiu Chun Hai and Go Tam vs.
Commissioner of Immigration and the SO ORDERED.
Director of NBI, 104 Phil. 949, 956).

The Deportation Board is composed of the Undersecretary of


Justice as chairman , the solicitor General, and a
representative of the Secretary of National Defense (Executive
Order No. 455 dated June 25, 1951, 47 O.G. 28M).

SECTION 69 and Executive Order No. 398, the Deportation


Board, do not specify the grounds for deportation Paragraph
l(a) of Executive Order No. 398 merely provides that "the
Deportation Board, motu proprio or upon complaint of any
person is authorized to conduct investigations in the manner
prescribed in section 69 of the Revised Administrative Code to
determine whether a subject of a foreign power in the
Philippines is an undesirable alien or not, and thereafter to
recommend to the President of the Philippines the deportation
of such alien."

As observed by Justice Labrador, there is no legal nor


constitutes provision defining the power to deport aliens
because the intention of the law is to grant the Chief Executive
"full discretion to determine whether an alien's residence in the
FINALS CONSTITUTION I ACJUCO 324

G.R. No. 82585 November 14, 1988 affirmed the resolution of the Secretary of Justice on May 2,
1988. The motion for reconsideration was denied by the
MAXIMO V. SOLIVEN, ANTONIO V. ROCES, FREDERICK Executive Secretary on May 16, 1988. With these
developments, petitioners' contention that they have been
K. AGCAOLI, and GODOFREDO L.
MANZANAS, petitioners, denied the administrative remedies available under the law has
vs. lost factual support.
THE HON. RAMON P. MAKASIAR, Presiding Judge of the
Regional Trial Court of Manila, Branch 35, It may also be added that with respect to petitioner Beltran, the
UNDERSECRETARY SILVESTRE BELLO III, of the allegation of denial of due process of law in the preliminary
Department of Justice, LUIS C. VICTOR, THE CITY FISCAL investigation is negated by the fact that instead of submitting
OF MANILA and PRESIDENT CORAZON C. his counter- affidavits, he filed a "Motion to Declare
AQUINO, respondents. Proceedings Closed," in effect waiving his right to refute the
complaint by filing counter-affidavits. Due process of law does
G.R. No. 82827 November 14, 1988 not require that the respondent in a criminal case actually file
his counter-affidavits before the preliminary investigation is
deemed completed. All that is required is that the respondent
LUIS D. BELTRAN, petitioner, be given the opportunity to submit counter-affidavits if he is so
vs. minded.
THE HON. RAMON P. MAKASIAR, Presiding Judge of
Branch 35 of the Regional Trial Court, at Manila, THE HON.
LUIS VICTOR, CITY FISCAL OF MANILA, PEOPLE OF THE The second issue, raised by petitioner Beltran, calls for an
interpretation of the constitutional provision on the issuance of
PHILIPPINES, SUPERINTENDENT OF THE WESTERN
POLICE DISTRICT, and THE MEMBERS OF THE PROCESS warrants of arrest. The pertinent provision reads:
SERVING UNIT AT THE REGIONAL TRIAL COURT OF
MANILA, respondents. Art. III, Sec. 2. The right of the people to be
secure in their persons, houses, papers
G.R. No. 83979 November 14, 1988. and effects against unreasonable searches
and seizures of whatever nature and for
any purpose shall be inviolable, and no
LUIS D. BELTRAN, petitioner, search warrant or warrant of arrest shall
vs. issue except upon probable cause to be
EXECUTIVE SECRETARY CATALINO MACARAIG, determined personally by the judge after
SECRETARY OF JUSTICE SEDFREY ORDOÑEZ, examination nder oath or affirmation of the
UNDERSECRETARY OF JUSTICE SILVESTRE BELLO III, complainant and the witnesses he may
THE CITY FISCAL OF MANILA JESUS F. GUERRERO, and produce, and particularly describing the
JUDGE RAMON P. MAKASIAR, Presiding Judge of Branch place to be searched and the persons or
35 of the Regional Trial Court, at Manila, respondents. things to be seized.

Angara, Abello, Concepcion, Regala and Cruz for petitioners The addition of the word "personally" after the word
in G.R. No. 82585. "determined" and the deletion of the grant of authority by the
1973 Constitution to issue warrants to "other responsible
Perfecto V. Fernandez, Jose P. Fernandez and Cristobal P. officers as may be authorized by law," has apparently
Fernandez for petitioner in G.R. Nos. 82827 and 83979. convinced petitioner Beltran that the Constitution now requires
the judge to personally examine the complainant and his
witnesses in his determination of probable cause for the
RESOLUTION issuance of warrants of arrest. This is not an accurate
interpretation.

What the Constitution underscores is the exclusive and


PER CURIAM: personal responsibility of the issuing judge to satisfy himself of
the existence of probable cause. In satisfying himself of the
existence of probable cause for the issuance of a warrant of
In these consolidated cases, three principal issues were arrest, the judge is not required to personally examine the
raised: (1) whether or not petitioners were denied due process complainant and his witnesses. Following established doctrine
when informations for libel were filed against them although the and procedure, he shall: (1) personally evaluate the report and
finding of the existence of a prima faciecase was still under the supporting documents submitted by the fiscal regarding the
review by the Secretary of Justice and, subsequently, by the existence of probable cause and, on the basis thereof, issue a
President; (2) whether or not the constitutional rights of Beltran warrant of arrest; or (2) if on the basis thereof he finds no
were violated when respondent RTC judge issued a warrant probable cause, he may disregard the fiscal's report and
for his arrest without personally examining the complainant require the submission of supporting affidavits of witnesses to
and the witnesses, if any, to determine probable cause; and (3) aid him in arriving at a conclusion as to the existence of
whether or not the President of the Philippines, under the probable cause.
Constitution, may initiate criminal proceedings against the
petitioners through the filing of a complaint-affidavit.
Sound policy dictates this procedure, otherwise judges would
be unduly laden with the preliminary examination and
Subsequent events have rendered the first issue moot and investigation of criminal complaints instead of concentrating on
academic. On March 30, 1988, the Secretary of Justice denied hearing and deciding cases filed before their courts.
petitioners' motion for reconsideration and upheld the
resolution of the Undersecretary of Justice sustaining the City
Fiscal's finding of a prima facie case against petitioners. A On June 30, 1987, the Supreme Court unanimously adopted
second motion for reconsideration filed by petitioner Beltran Circular No. 12, setting down guidelines for the issuance of
was denied by the Secretary of Justice on April 7, 1988. On warrants of arrest. The procedure therein provided is reiterated
appeal, the President, through the Executive Secretary, and clarified in this resolution.
FINALS CONSTITUTION I ACJUCO 325

It has not been shown that respondent judge has deviated from
the prescribed procedure. Thus, with regard to the issuance of
the warrants of arrest, a finding of grave abuse of discretion GUTIERREZ, JR., J., concurring:
amounting to lack or excess of jurisdiction cannot be sustained.

I concur with the majority opinion insofar as it involves the three


Anent the third issue, petitioner Beltran argues that "the
principal issues mentioned in its opening statement. However,
reasons which necessitate presidential immunity from suit as to the more important issue on whether or not the
impose a correlative disability to file suit." He contends that if
prosecution of the libel case would produce a "chilling effect"
criminal proceedings ensue by virtue of the President's filing of on press freedom, I beg to reserve my vote. I believe this is the
her complaint-affidavit, she may subsequently have to be a more important issue in these petitions and it should be
witness for the prosecution, bringing her under the trial court's resolved now rather that later.
jurisdiction. This, continues Beltran, would in an indirect way
defeat her privilege of immunity from suit, as by testifying on
the witness stand, she would be exposing herself to possible Consistent with our decision in Salonga v. Cruz Pano (134
contempt of court or perjury. SCRA 438 [1985]), the Court should not hesitate to quash a
criminal prosecution in the interest of more enlightened and
substantial justice where it is not alone the criminal liability of
The rationale for the grant to the President of the privilege of an accused in a seemingly minor libel case which is involved
immunity from suit is to assure the exercise of Presidential but broader considerations of governmental power versus a
duties and functions free from any hindrance or distraction, preferred freedom.
considering that being the Chief Executive of the Government
is a job that, aside from requiring all of the office holder's time,
also demands undivided attention. We have in these four petitions the unusual situation where the
highest official of the Republic and one who enjoys
unprecedented public support asks for the prosecution of a
But this privilege of immunity from suit, pertains to the newspaper columnist, the publisher and chairman of the
President by virtue of the office and may be invoked only by
editorial board, the managing editor and the business manager
the holder of the office; not by any other person in the in a not too indubitable a case for alleged libel.
President's behalf. Thus, an accused in a criminal case in
which the President is complainant cannot raise the
presidential privilege as a defense to prevent the case from I am fully in accord with an all out prosecution if the effect will
proceeding against such accused. be limited to punishing a newspaperman who, instead of
observing accuracy and fairness, engages in unwarranted
personal attacks, irresponsible twisting of facts, of malicious
Moreover, there is nothing in our laws that would prevent the distortions of half-truths which tend to cause dishonor,
President from waiving the privilege. Thus, if so minded the
discredit, or contempt of the complainant. However, this case
President may shed the protection afforded by the privilege is not a simple prosecution for libel. We have as complainant a
and submit to the court's jurisdiction. The choice of whether to powerful and popular President who heads the investigation
exercise the privilege or to waive it is solely the President's and prosecution service and appoints members of appellate
prerogative. It is a decision that cannot be assumed and courts but who feels so terribly maligned that she has taken the
imposed by any other person. unorthodox step of going to court inspite of the invocations of
freedom of the press which would inevitably follow.
As regards the contention of petitioner Beltran that he could
not be held liable for libel because of the privileged character I believe that this Court should have acted on this issue now
or the publication, the Court reiterates that it is not a trier of instead of leaving the matter to fiscals and defense lawyers to
facts and that such a defense is best left to the trial court to argue before a trial judge.
appreciate after receiving the evidence of the parties.

There is always bound to be harassment inherent in any


As to petitioner Beltran's claim that to allow the libel case to criminal prosecution. Where the harassment goes beyond the
proceed would produce a "chilling effect" on press freedom, usual difficulties encountered by any accused and results in an
the Court finds no basis at this stage to rule on the point. unwillingness of media to freely criticize government or to
question government handling of sensitive issues and public
The petitions fail to establish that public respondents, through affairs, this Court and not a lower tribunal should draw the
their separate acts, gravely abused their discretion as to demarcation line.
amount to lack of jurisdiction. Hence, the writs of certiorari and
prohibition prayed for cannot issue. As early as March 8, 1918, the decision in United States v.
Bustos (37 Phil. 731) stated that "(c)omplete liberty to
WHEREFORE, finding no grave abuse of discretion amounting comment on the conduct of public men is a scalpel in the case
to excess or lack of jurisdiction on the part of the public of free speech. The sharp incision of its probe relieves the
respondents, the Court Resolved to DISMISS the petitions in abscesses of officialdom. Men in public life may suffer under a
G. R. Nos. 82585, 82827 and 83979. The Order to maintain hostile and unjust accusation; the wound can be assuaged with
the status quo contained in the Resolution of the Court en the balm of a clear conscience." The Court pointed out that
banc dated April 7, 1988 and reiterated in the Resolution dated while defamation is not authorized, criticism is to be expected
April 26, 1988 is LIFTED. and should be borne for the common good.

Fernan, C.J., Narvasa, Melencio-Herrera, Cruz, Paras, In People v. Perfecto (43 Phil. 887 [1922]), the Court stated:
Feliciano, Gancayco, Padilla, Bidin, Sarmiento, Cortes, Griño-
Aquino Medialdea and Regalado, JJ., concur. xxx xxx xxx

... No longer is there a Minister of the


Crown own or a person in authority of such
Separate Opinions exalted position that the citizen must speak
FINALS CONSTITUTION I ACJUCO 326

of him only with bated breath. "In the eye of government; that the path of safety lies in
our Constitution and laws, every man is a the opportunity to discuss freely supposed
sovereign, a ruler and a freeman, and has grievances and proposed remedies; and
equal rights with every other man." (at p. that the fitting remedy for evil counsel is
900) good ones. Believing in the power of
reason as applied through public
In fact, the Court observed that high official position, instead of discussion, they eschewed silence coerced
affording immunity from slanderous and libelous charges, by law—the argument of force in its worst
would actually invite attacks by those who desire to create form. ...
sensation. It would seem that what would ordinarily be slander
if directed at the typical person should be examined from Thus we consider this case against the
various perspectives if directed at a high government official. background of a profound national
Again, the Supreme Court should draw this fine line instead of commitment to the principle that debate on
leaving it to lower tribunals. public issues should be uninhibited, robust,
and wide open, and that it may well include
This Court has stressed as authoritative doctrine in Elizalde v. vehement, caustic, and sometimes
unpleasantly sharp attacks on government
Gutierrez (76 SCRA 448 [1977]) that a prosecution for libel
lacks justification if the offending words find sanctuary within and public officials. ... (at pp. 700-701)
the shelter of the free press guaranty. In other words, a
prosecution for libel should not be allowed to continue, where Shunting aside the individual liability of Mr. Luis Beltran, is
after discounting the possibility that the words may not be really there a prima facie showing that Messrs. Maximo Soliven,
that libelous, there is likely to be a chilling effect, a patently Antonio V. Roces, Frederick K. Agcaoili, and Godofredo L.
inhibiting factor on the willingness of newspapermen, Manzanas knowingly participated in a wilful purveying of
especially editors and publishers to courageously perform their falsehood? Considering the free speech aspects of these
critical role in society. If, instead of merely reading more petitions, should not a differentiated approach to their
carefully what a columnist writes in his daily column, the editors particular liabilities be taken instead of lumping up everybody
tell their people to lay off certain issues or certain officials, the with the offending columnist? I realize that the law includes
effect on a free press would be highly injurious. publishers and editors but perhaps the "chilling effect" issue
applies with singular effectivity to publishers and editors vis-a-
Because many questions regarding press freedom are left vis newspaper columnists. There is no question that, ordinarily,
unanswered by our resolution, I must call attention to our libel is not protected by the free speech clause but we have to
understand that some provocative words, which if taken
decisions which caution that "no inroads on press freedom
should be allowed in the guise of punitive action visited on what literally may appear to shame or disparage a public figure, may
really be intended to provoke debate on public issues when
otherwise should be characterized as libel." (Lopez v. Court of
Appeals, 34 SCRA 117 [1970]; See also the citations in uttered or written by a media personality. Will not a criminal
Elizalde v. Gutierrez, supra). prosecution in the type of case now before us dampen the vigor
and limit the variety of public debate? There are many other
questions arising from this unusual case which have not been
The United States Supreme Court is even more emphatic, to considered.
wit:
I, of course, concur with the Court's opinion because it has
In deciding the question now, we are decided to limit the issues to narrowly drawn ones. I see no
compelled by neither precedent nor policy reason to disagree with the way the Court has resolved them.
to give any more weight to the epithet "libel" The first issue on prematurity is moot. The second issue
than we have to other "mere labels" of state discusses a procedure now embodied in the recently amended
law. N. A. A. C. P. v. Button, 371 US 415, Rules of Court on how a Judge should proceed before he
429, 9L ed 2d 405, 415, 83 S Ct 328. Like issues a warrant of arrest. Anent the third issue, considerations
insurrection, contempt, advocacy of of public policy dictate that an incumbent President should not
unlawful acts, breach of the peace, be sued. At the same time, the President cannot stand by
obscenity, solicitation of legal business, helplessly bereft of legal remedies if somebody vilifies or
and the other various other formulae for the maligns him or her.
repression of expression that have been
challenged in this Court, libel can claim no
The Court has decided to defer the "chilling effect" issue for a
talismanic immunity from constitutional
limitations. It must be measured by later day. To this, I take exception. I know that most of our
standards that satisfy the First fiscals and judges are courageous individuals who would not
Amendment. allow any considerations of possible consequences to their
careers to stand in the way of public duty. But why should we
subject them to this problem? And why should we allow the
xxx xxx xxx possibility of the trial court treating and deciding the case as
one for ordinary libel without bothering to fully explore the more
Those who won our independence important areas of concern, the extremely difficult issues
believed ... that public discussion is a involving government power and freedom of expression.
political duty; and that this should be a
fundamental principle of the American However, since we have decided to defer the "chilling effect"
government. They recognized the risk to issue for a later day, I limit myself to reiterating the dissenting
which all human institutions are subject. words of Mr. Justice Jackson in the American case
But they knew that order cannot be secured of Beaurnhais v. Illinois (343 U. S. 250) when he said:
merely through fear of punishment for its
infraction; that it is hazardous to discourage
thought, hope and imagination; that fear If one can claim to announce the judgment
of legal history on any subject, it is that
breeds repression; that repression breeds
hate; that hate menaces stable criminal libel laws are consistent with the
FINALS CONSTITUTION I ACJUCO 327

concept of ordered liberty only when


applied with safeguards evolved to prevent
their invasion of freedom of expression.

In the trial of the libel case against the petitioners, the


safeguards in the name of freedom of expression should be
faithfully applied.

Potrebbero piacerti anche